Download as pdf or txt
Download as pdf or txt
You are on page 1of 599

Contents

Past Year Papers Solutions


CLAT CLAT
1. CLAT – 2008 01 1. CLAT – 2008 01

2. CLAT – 2009 30 2. CLAT – 2009 10

3. CLAT – 2010 50 3. CLAT – 2010 21

4. CLAT – 2011 70 4. CLAT – 2011 30

5. CLAT – 2012 107 5. CLAT – 2012 40

6. CLAT – 2013 138 6. CLAT – 2013 53

7. CLAT – 2014 164 7. CLAT – 2014 62

8. CLAT – 2015 196 8. CLAT – 2015 73

9. CLAT – 2016 220 9. CLAT – 2016 84

AILET AILET
10. AILET – 2008 242 10. AILET – 2008 92

11. AILET – 2009 263 11. AILET – 2009 98

12. AILET – 2010 282 12. AILET – 2010 105

13. AILET – 2011 305 13. AILET – 2011 112

14. AILET – 2012 328 14. AILET – 2012 119

15. AILET – 2013 348 15. AILET – 2013 128

16. AILET – 2014 369 16. AILET – 2014 135

17. AILET – 2015 395 17. AILET – 2015 144

18. AILET – 2016 417 18. AILET – 2016 154


CLAT Question Paper 2008
SECTION - I: ENGLISH

PART - A

Instructions:Read the given passage carefully and attempt the questions that follow and shade
the appropriate answer in the space provided for it on the OMR answer sheet.

Example: If the appropriate answer is (a), shade the appropriate oval on the OMR sheet.

Marks: Each question carries 1 (one) mark. (Total 10 marks)

MY LOVE OF NATURE, goes right back to my childhood, to the times when I stayed on my grandparents’
farm in Suffolk. My father was in the armed forces, so we were always moving and didn’t have a home base
for any length of time, but I loved going there. I think it was my grandmother who encouraged me more than
anyone: she taught me the names of wild flowers and got me interested in looking at the countryside, so
it seemed obvious to go on to do Zoology at University.

I didn’t get my first camera until after I’d graduated, when I was due to go diving in Norway and needed a
method of recording the sea creatures I would find there. My father didn’t know anything about photography,
but he bought me an Exacta, which was really quite a good camera for the time, and I went off to take my
first pictures of sea anemones and starfish. I became keen very quickly, and learned how to develop and
print; obviously I didn’t have much money in those days, so I did more black and white photography than
colour, but it was all still using the camera very much as a tool to record what I found both by diving and on
the shore. I had no ambition at all to be a photographer then, or even for some years afterwards.

Unlike many of the wildlife photographers of the time, I trained as a scientist and therefore my way of
expressing myself is very different. I’ve tried from the beginning to produce pictures which are always
biologically correct. There are people who will alter things deliberately: you don’t pick up sea creatures
from the middle of the shore and take them down to attractive pools at the bottom of the shore without
knowing you’re doing it. In so doing you’re actually falsifying the sort of seaweeds they live on and so on,
which may seem unimportant, but it is actually changing the natural surroundings to make them prettier.
Unfortunately, many of the people who select pictures are looking for attractive images and, at the end of
the day, whether it’s truthful or not doesn’t really matter to them.

It’s important to think about the animal first, and there are many occasions when I’ve not taken a picture
because it would have been too disturbing. Nothing is so important that you have to get that shot; of
course, there are cases when it would be very sad if you didn’t, but it’s not the end of the world. There can
be a lot of ignorance in people’s behavior towards wild animals and it’s a problem, that more and more
people are going to wild places: while some animals may get used to cars, they won’t get used to people
suddenly rushing up to them. The sheer pressure of people, coupled with the fact that there are increasingly
fewer places where no-one else has photographed, means that over the years, life has become much more
difficult for the professional wildlife photographer.

Nevertheless, wildlife photographs play a very important part in educating people about what is out there
and what needs conserving. Although photography can be an enjoyable pastime, as it is to many people,

Previous Years
CLAT & AILET Papers Page 1
it is also something that plays a very important part in educating young and old alike. Of the qualities it
takes to make a good wildlife photographer, patience is perhaps the most obvious - you just have to be
prepared to sit it out. I’m actually more patient now because I write more than ever before, and as long as
I’ve got a bit of paper and a pencil, I don’t feel I’m wasting my time. And because I photograph such a wide
range of things, even if the main target doesn’t appear I can probably find something else to concentrate on
instead.

1. The writer decided to go to university and study Zoology because


(a) she wanted to improve her life in the countryside.
(b) she was persuaded to do so by her grandmother.
(c) she was keen on the natural world.
(d) she wanted to stop moving around all the time.

2. Why did she get her first camera?


(a) she needed to be able to look back at what she had seen.
(b) she wanted to find out if she enjoyed photography.
(c) her father thought it was a good idea for her to have one.
(d) she wanted to learn how to use one and develop her own prints.

3. She did more black and white photography than colour because
(a) she did not like colour photograph.
(b) she did not have a good camera.
(c) she wanted quality photograph.
(d) she didn’t have much money is those days.

4. How is she different from some of the other wildlife photographers she meets?
(a) She tries to make her photographs as attractive as possible.
(b) She takes photographs which record accurate natural conditions.
(c) She likes to photograph plants as well as wildlife.
(d) She knows the best places to find wildlife.

5. What does ‘them’ refer to in the line “In so doing ... make ‘them’ prettier”?
(a) sea creatures
(b) attractive pools
(c) seaweeds
(d) natural surroundings

6. What the writer means by ‘ignorance in people’s behaviour’ is


(a) altering things deliberately.
(b) people suddenly rushing up to animals.
(c) people taking photographs of wild animals.
(d) people not thinking about the animals in the first place.

7. The writer now finds it more difficult to photograph wild animals because
(a) there are fewer of them.
(b) they have become more nervous of people.
(c) it is harder to find suitable places.
(d) they have become frightened of cars.

Previous Years
Page 2 CLAT & AILET Papers
8. Wildlife photography is important because it can make people realise that
(a) photography is an enjoyable hobby.
(b) we learn little about wildlife at school.
(c) it is worthwhile visiting the countryside.
(d) wildlife photographs educate people about wild animals.

9. Why is she more patient now?


(a) She does other things while waiting.
(b) She has got used to waiting.
(c) She can concentrate better than she used to.
(d) She knows the result will be worth it.

10. Which of the following describes the writer?


(a) proud (b) sensitive (c) aggressive (d) disappointed

PART – B

Instructions: Three of the four words given below are spelt wrongly. Select the word that is spelt
correctly and shade the appropriate answer in the space provided for it on the OMR answer
sheet.

Marks: Each question carries 1 (One) mark. (Total 5 marks)

11. (a) acquintence (b) acquaintence (c) acquaintance (d) acquintance

12. (a) neglegense (b) negligence (c) negligance (d) negligence

13. (a) grevance (b) greivance (c) grievance (d) grievence

14. (a) heirarchical (b) hierarchical (c) hiererchical (d) heirerchical

15. (a) garanter (b) garantor (c) guaranter (d) guarantor

PART – C

Instructions: Select the best option from the four alternatives given and shade the appropriate
answer in the space provided for it on the OMR answer sheet.

Marks: Each question carries 1 (one) mark. (Total 10 marks)

16. They live on a busy road. ___ a lot of noise from the traffic.
(a) It must be (b) It must have (c) There must have (d) There must be

17. The more electricity you use, _____.


(a) your bill will be higher (b) will be higher your bill
(c) the higher your bill will be (d) higher your bill will be

Previous Years
CLAT & AILET Papers Page 3
18. Ben likes walking. _______.
(a) Every morning he walks to work (b) He walks to work every morning
(b) He walks every morning to work (d) He every morning walks to work

19. It’s two years ___ Sophy.


(a) that I don’t see (b) that I haven’t seen
(c) since I didn’t see (d) since I last saw

20. What was the problems? Why ___ leave early?


(a) have you to (b) did you have to
(c) must you (d) you had to

21. Nobody believed Arun at first, but he ___ to be right.


(a) worked out (b) came out (c) found out (d) turned out

22. We can’t ___ making a decision. We have to decide now.


(a) put away (b) put over (c) put off (d) put out

23. The accident was my fault, so I had to pay for the damage ____ the other car.
(a) of (b) for (c) to (d) on

24. I really object ___ people smoking in my house.


(a) to (b) about (c) for (d) on

25. A contract may be ___ if the court finds there has been misinterpretation of the facts.
(a) restrained (b) rescinded (c) compelled (d) conferred

PART – D

Instructions: The five paragraphs given below have all had their constituent sentences jumbled.
Read each jumbled passage carefully and then pick the option in which the best sequence is
shown and shade the appropriate answer in the space provided for it on the OMR answer sheet.

Marks: Each question carries 1 (one) mark. (Total 5 marks)

26. UNIT I
i) The Supertag scanner could revolutionise the way people shop, virtually eradicating supermarket
queues;
ii) The face of retailing will change even more rapidly when the fibre optic networks being built by
cable TV companies begin to be more widely used;
iii) The scanner would have a double benefit for supermarkets - removing the bottleneck which
causes frustration to most customers and reducing the number of checkout staff;
iv) An electronic scanner which can read the entire contents of a supermarket trolley at a glance
has just been developed.

The best sequence is:


(a) ii, i, iii, iv (b) iv, i, iii, ii (c) iv, iii, ii, i (d) iii, i, iv, ii

Previous Years
Page 4 CLAT & AILET Papers
27. UNIT II
i) Of course, modern postal services now are much more sophisticated and faster, relying as they
do on motor vehicles and planes for delivery.
ii) Indeed, the ancient Egyptians had a system for sending letters from about 2000 BC, as did the
Zhou dynasty in China a thousand years later.
iii) Letters, were, and are, sent by some form of postal service, the history of which goes back a
long way.
iv) For centuries, the only form of written correspondence was the letter.

The best sequence is:


(a) ii, i, iii, iv (b) iv, i, iii, ii (c) iv, iii, ii, i (d) iii, i, iv, ii

28. UNIT III


i) Converting money into several currencies in the course of one trip can also be quite expensive,
given that banks and bureaux de change charge commission on the transaction.
ii) Trying to work out the value of the various notes and coins can be quite a strain, particularly if
you are visiting more than one country.
iii) Travel can be very exciting, but it can also be rather complicated.
iv) One of these complications is, undoubtedly, foreign currency.

The best sequence is:


(a) ii, i, iii, iv (b) iv, i, iii, ii (c) iv, iii, ii, i (d) iii, iv, ii, i

29. UNIT IV
i) She was right about three-curiosity, freckles, and doubt-but wrong about love.
ii) “Four of the things I’d be better without: Love, curiosity, freckles, and doubt”.
iii) Love is indispensable in life.
iv) So wrote Dorothy Parker, the American writer.

The best sequence is:


(a) ii, iv, i, iii (b) ii, i, iii, iv (c) ii, i, iv, iii (d) iii, iv, i, ii

30. UNIT V
i) This clearly indicates that the brains of men and women are organised differently in the way they
process speech.
ii) Difference in the way men and women process language is of special interest to brain researchers.
iii) However, women are more likely than men to suffer aphasia when the front part of the brain is
damaged.
iv) It has been known that aphasia - a kind of speech disorder - is more common in men than in
women when the left side of the brain is damaged in an accident or after a stroke.

The best sequence is:


(a) ii, i, iv, iii (b) iv, i, iii, ii (c) iv, iii, ii, i (d) ii, iv, iii, i

Previous Years
CLAT & AILET Papers Page 5
PART - E

Instructions: Given below are five list of words followed by some choices. In each case, choose
the alternative that you can combine with every word in that particular list to form a familiar
word phrase and shade the appropriate answer in the space provided for it on the OMR answer
sheet.
Marks: Each question carries 1 (one) mark. (Total 5 marks)
31. Down, aside, about, forth
(a) set (b) fly (c) burn (d) take
32. Over, about, after, at
(a) cross (b) lay (c) here (d) go

33. Forward, across, around, upon


(a) straight (b) come (c) fast (d) mark
34. In, down, for, out
(a) pray (b) try (c) grow (d) stand
35. Away, through, up, down
(a) stray (b) come (c) break (d) speak

PART - F

Instructions: Given below are a few foreign language phrases that are commonly used. Choose
the correct meaning for each of the phrases and shade the appropriate answer in the space
provided for it on the OMR answer sheet.
Marks: Each question carries 1 (one) mark. (Total 5 marks)

36. Prima facie


(a)the most important (b) that which comes first
(c) at first view (d) the face that is young

37. Sine die


(a) without setting a fixed day
(b) by voice vote
(b) applying mathematical concepts to solve a difficult problem
(d) signing legal documents before death

38. Bona fide


(a) identification card (b) without doubt
(c) in good faith (d) indispensable condition

39. Status quo


(a) legally valid (b) present condition (c) social position (d) side remarks

40. De jure
(a) here and there (b) as per law (c) small details (d) in the same place

Previous Years
Page 6 CLAT & AILET Papers
SECTION - II: GENERAL KNOWLEDGE

Instructions: From the four answers, shade the most appropriate answer in the space provided
for it on the OMR answer sheet.

Marks: Each question carries 1/2 (half) mark. (Total 50 marks)

41. The Supreme Court of India upheld the decision to implement the quota for other backward classes
(OBCs) in higher educational institutions. The court, however, excluded the “creamy layer” from
being a beneficiary. The reason is:
(a) Creamy layer is not an OBC; it is a forward caste
(b) Creamy layer is politically powerful
(c) It can complete with others on equal footing
(d) The inclusion of creamy layer would be unjust

42. Hegde fund is a fund


(a) used for absorbing inflation
(b) used for cushioning health risks
(c) applied to minimize the risk of financial market transactions
(d) applied for absorbing the risk in commodity trading

43. What does strong Rupee as against the dollar mean to India?
(a) There is a Balance of Payments surplus
(b) Indian economy is globally respected
(b) It is a sign of economic buoyancy
(d) Income from exports is falling

44. Name the latest State which declared independence in 2008


(a) Serbia (b) Kosovo (c) Kurdistan (d) Tibet

45. Name the Finance Minister who presented the highest number of Budgets in the Parliament so far:
(a) P. Chidambaram (b) Morarji Desai
(c) Manmohan Singh (d) T.T. Krishnamachari

46. Who is the Chairman of 13th Finance Commission constituted in 2007?


(a) Vijay Kelkar (b) C. Rangrajan (c) Ashok Lahiri (d) K.C.Pant

47. IIndo-U.S. nuclear deal was opposed in Parliament mainly because


(a) all Indian nuclear reactors would fall under American supervision
(b) Nuclear energy sector will be dominated by American corporations
(c) Nuclear relations between India and the U.S.A will be governed by the Hyde Act
(d) The USA will dictate Indian policies

48. The Indian industrialist who bought Tippu Sultan’s sword in an auction in London was
(a) Vijay Mallya (b) Anil Ambani
(c) Amar Singh (d) Lakshmi Mittal

Previous Years
CLAT & AILET Papers Page 7
49. The contentious Baglihar dam is built on the river
(a) Indus (b) Jheelum (c) Chenab (d) Satlez

50. Which country has its richest man as the head of the Government?
(a) The USA (b) Italy (c) Saudi Arabia (d) Russia

51. Who is the person known as the Father of Modern Indian Retail Trade?
(a) Mukhesh Ambani (b) Kishore Bijani (c) Aditya Birla (d) Anil Ambani

52. The largest software service company in Asia is


(a) WIPRO (b) INFOSYS
(c) Tata Consultancy Services (d) Satyam Computers

53. Taikonaut means


(a) a character in comic strips (b) a character in Russian opera
(c) astronaut in China (d) a delicious Japanese dish

54. The CEO of Microsoft Corporation is


(a) Bill Gates (b) Warren Buffett (c) Steve Ballmer (d) John Wallace

55. The country which stands for Gross National Happiness in contradistinction to Gross National
Product
(a) Sweden (b) Switzerland (c) Bhutan (d) Finland

56. The highest paid head of the government in the world at present is in
(a) The USA (b) Russian Federation (c) Singapore (d) Japan

57. The current impasse in Doha Round of Negotiations is centered around:


(a) access to cheaper drugs
(b) access to markets of developed countries
(c) agricultural subsidies provided by developed countries
(d) removal of non-tariff barriers

58. The phenomenon called “Equinox” is due to the


(a) rotation of the Earth on its own axis
(b) revolution of the Earth on its inclined axis
(c) oblate-spheroid shape of the Earth
(d) gravitational pull of the planet

59. The Director-General of the World Trade Organization is


(a) Renalto Ruggiero (b) Pascal Lamy (c) Arthur Dunkell (d) Oliver Long

60. Capital account convertibility signifies


(a) guaranteeing the right to investment to foreigners
(b) ensuring the right of buyers to make international payments
(c) ensuring free international movement of capital
(d) ensuring the right of an individual to invest in foreign capital markets

Previous Years
Page 8 CLAT & AILET Papers
61. The purpose of Kyoto Protocol is
(a) to promote tourism
(b) to contribute sustainable development
(c) to promote renewable sources of energy
(d) to put a limit on greenhouse gas emissions by states

62. What do carbon credits signify?


(a) Credits given in the course of carbon products sales
(b) Entitlements to emit certain quantity of green house gases
(c) Permissible amount of Carbon dioxide in the atmosphere
(d) The extent of carbon required to ensure sustainable development

63. The practice of selling goods in a foreign country at a price below their domestic selling price is
called
(a) discrimination (b) dumping (c) double pricing (d) predatory pricing

64. Which of the following is considered as bulwark of personal freedom?


(a) Mandamus (b) Habeus Corpus (c) Certiorari (d) Quo Warranto

65. Vande Mataram is composed by


(a) Rabindranath Tagore (b) Sharatchandra Chatterji
(c) Bankimchandra Chatterjee (d) Surendranath Banerji

66. How many minutes for each degree of longitude does the local time of any place vary from the
Greenwich time ?
(a) Four minutes (b) Two minutes (c) Eight minutes (d) Ten minutes

67. Article 1 of Indian Constitution defines India as


(a) Federal State (b) Unitary State (c) Union of State (d) Quasi-Federal State

68. Which is the highest body that approves Five Year Plans in India ?
(a) Parliament (b) Planning Commission
(b) National Development Council (d) Council of Ministers

69. The economist who for the first time scientifically determined national income in India:
(a) Dr. D.R. Gadgil (b) Dr. V.K.R.V. Rao
(c) Dr. Manmohan Singh (d) Dr. Y.V. Alagh

70. Which of the following is the largest peninsula in the world ?


(a) Indian Peninsula (b) Arabian Peninsula (c) Malay Peninsula (d) Chinese Peninsula

71. The person responsible for economic model for Indian Planning:
(a) Jawaharlal Nehru (b) P.C. Mahalanobis (c) Tarlok Singh (d) V.T. Krishnamachari

Previous Years
CLAT & AILET Papers Page 9
72. Social Forestry aims at
(a) Ensuring fuel and forest produces to weaker sections
(b) Medicinal and fruit plantation
(c) Large scale afforestation
(d) Scientific forestry

73. The Great Barrier Reef refers to


(a) Coral formation (b) Tidal waves (c) Hill range (d) Man-made walls

74. A nautical mile is equal to


(a) 1825 metres (b) 2000 metres (c) 1575 metres (d) 2050 metres

75. Which of the following is concerned with land forms ?


(a) Geology (b) Geomorphology (c) Ecology (d) Geography

76. The country known as the Land of Midnight Sun


(a) Sweden (b) Norway (c) Finland (d) Denmark

77. The monk who spread Buddhism in Tibet and Far-East:


(a) Ananda (b) Nagarjuna (c) Padmasambava (d) Mahendra

78. TRIPs, forming part of the World Trade Organization is intended


(a) to provide for stronger patent protection
(b) to promote transnational corporate interests
(c) to harmonize IPR regime internationally
(d) to replace World Intellectual Property Organization

79. Carbon dating method is used to determine the age of


(a) Rocks (b) Fossils (c) Trees (d) Ancient monuments

80. The Managing Director of Delhi Metropolitan Railway Corporation


(a) Chairman of Indian Railway Board (b) Lt. Governor of Delhi
(c) C. Sreedharan (d) Sheela Dikshit

81. 18 carat gold signifies


(a) 18 parts of gold and 82 parts of other metal
(b) 18 parts of gold and 6 parts of other metal
(c) 82 parts of gold and 18 parts of other metal
(d) None of the above

82. Bali Road map adopted in December 2007 provides for


(a) fixation of greenhose gas emission limits
(b) launching of an adaptation fund
(c) amendment to UN Framework Convention on Climate Change
(d) special and differential treatment for developing countries

Previous Years
Page 10 CLAT & AILET Papers
83. Enriched uranium, used in a nuclear reactor, is
(a) uranium freed of all impurities (b) uranium treated with radiation
(c) uranium mixed with isotopes (d) uranium alloy with aluminum

84. The scientist responsible for developing atomic energy in India


(a) C.V. Raman (b) H.J. Bhabha (c) H.K. Sethna (d) Vikram Surabhai

85. Salwa Judum practised in certain places in India refers to


(a) witchcraft
(b) arming civilians to fight militants
(c) training civilians in the use of fire arms
(d) training civilians to be home guards

86. Indian who won Raman Magasaysay award in 2007:


(a) Arun Roy (b) P. Sainath (c) Medha Patkar (d) Ruth Manorama

87. The person who won Jawaharlal Nehru award in 2007:


(a) Lula de Silva (b) Hugo Chavez (c) Aung Saan Suu Ki (d) Fidel Castro

88. Free Trade Area means:


(a) the area where anything can be bought and sold
(b) countries between whom trade barriers have been substantially reduced
(c) countries which have common external tariff
(d) countries which have common currency

89. Affirmative action in Indian context signifies:


(a) providing security to weaker sections
(b) welfare measures to alleviate the sufferings of poor people
(c) providing positive opportunities to deprived sections
(d) giving incentives to start industries

90. Special Economic zones are


(a) the places where industries can operate without any control
(b) the places wherein any person can start any industry
(c) the places where industries get certain tax advantages
(d) the places wherein the national labour laws do not apply

91. TThe space shuttle which successfully carried Sunita Williams to space:
(a) Challenger (b) Atlantis (c) Discovery (d) Columbus

92. The leader who led the country in atoning for the past wrongs:
(a) John Howard (b) Desmond Tutu (c) Kevin Rudd (d) Jimmy Carter

93. Gandhiji expounded his economic ideas in


(a) Hind Swaraj (b) My Experiments with Truth
(c) Unto the Last (d) Economics of Permanence

Previous Years
CLAT & AILET Papers Page 11
94. Bio-fuels have become controversial because,
(a) they increase environmental pollution (b) they slow down industrialization
(c) they reduce food cultivation (d) they lead to degeneration of soil

95. Evergreening of patents means


(a) granting patents in perpetuity
(b) granting patents for 100 years
(c) granting protection to incremental inventions having no substantial significance
(d) patenting of green technology

96. By signing which pact with Gandhiji did Ambedkar give up his demand for separate electorates:
(a) Poona Pact (b) Aligarh Pact (c) Deccan Pact (d) Delhi Pact

97. India earns maximum foreign exchange from the export of


(a) Garments (b) Jute
(c) Gems and Jewelleries (d) Light engineering goods

98. Sunita Williams, renowned astronaut of Indian origin, spent a record of _____ days in space
(a) 195 (b) 185 (c) 200 (d) 160

99. The second biggest greenhouse gas emitter (after the USA) in the world is:
(a) Russia (b) Germany (c) China (d) Japan

100. The author of the management principle - In a hierarchy, every employee tends to rise to his level of
incompetence
(a) Prof. Ducker (b) Prof. J. Peter (c) Prof. C.H. Prahlad (d) Prof. Schmitthoff

101. The World Trade Organization was earlier known as


(a) UNCTAD (b) GATT (c) UNIDO (d) UNCITRAL

102. The “wailing wall” is associated with


(a) Christians (b) Bahais (c) Jews (d) Shias

103. An Education Minister who got Bharata Ratna in India


(a) G.B. Pant (b) M.C. Chagla (c) Abul Kalam Azad (d) Humayun Kabir

104. Why is Ozone Layer important?


(a) It absorbs greenhouse gases
(b) It protects Earth from ultraviolet radiation
(c) It maintains Earth’s temperature
(d) It is a buffer against extra-terrestrial hazards

105. The World’s largest river is


(a) Brahmaputra (b) Amazon (c) Nile (d) Mississippi

106. Tsunami is caused by


(a) plate tectonics (b) underwater ridges
(c) underwater volcanic activity (d) pressure from Earth’s core

Previous Years
Page 12 CLAT & AILET Papers
107. The Chipko movement is associated with
(a) preventing the felling of trees (b) afforestation
(c) transparency in public life (d) sustainable development

108. The first Great Indian Empire was


(a) Magadhan Empire (b) Kuru Empire (c) Pandava Empire (d) Gandhara Empire

109. The first President of Indian National Congress


(a) A.O. Hume (b) W.C. Banerji (c) Dadabhai Nauroji (d) Phirozeshah Mehta

110. The King who gave permission to establish East India Company in India
(a) Jahangir (b) Aurangzeb (c) Shahjahan (d) Shershah

111. The person who conceptualized the idea of Pakistan


(a) M.A. Jinnah (b) Hakim Azmal Khan (c) Mohammad Iqbal (d) Liaqusat Ali Khan

112. Khilafat movement was organized


(a) for getting Muslim Homeland
(b) as a protest against British suppression of Turks
(c) to preserve Turkish Empire with Khilafat as temporal head
(d) as a protest against communal politics

113. The pattern of Centre-State relations in India can be traced back to


(a) The U.S. Constitution (b) The Government of India Act, 1935
(c) Motilal Nehru Committee Report (d) Ambedkar’s vision

114. Indian who played a very important role in World Communist Movement:
(a) Jyoti Basu (b) M.N. Roy (c) A.K. Gopalan (d) Prakash Karat

115. Who was the first recipient of Jnanapith award?


(a) Amrita Pritam (b) Dinkar (c) D.V. Gundappa (d) G. Shankara Kurup

116. Name the winner of 2007 Nobel Prize for Literature


(a) Dorris Lessing (b) V.S. Naipaul (c) Doras Lessing (d) Salman Raushdie

117. Plea bargaining is


(a) permissible in India (b) illegal in India
(c) mandatory in India (d) allowed subject to the permission of the court

118. The person appointed by two parties to settle a dispute is known as:
(a) Judge (b) Arbitrator (c) Solicitor (d) Conciliator

119. Right to travel is a fundamental right under


(a) Article 19 of the Constitution (b) Article 21 of the Constitution
(c) Article 14 of the Constitution (d) None of the above

Previous Years
CLAT & AILET Papers Page 13
120. Genetically modified seeds have become controversial mainly because of
(a) adverse impact on human health (b) adverse impact on flora around
(c) adverse impact on ozone layer (d) emission of greenhouse gases

121. Legal aid for an accused is


(a) fundamental right (b) legal right
(c) Directive Principles of State Policy (d) Discretion of State

122. The members of Constituent Assembly who framed the Constitution were:
(a) directly elected by the people (b) indirectly elected
(c) nominated (d) appointed by political parties

123. Ambedkar acted in Constituent Assembly as:


(a) President of the Assembly
(b) Chairman of the Drafting Committee
(c) the leading spokesman of weaker sections
(d) a strong defender of fundamental rights

124. In India, international treaties are ratified by


(a) Parliament (b) President (c) Prime Minister (d) The Union Cabinet

125. It is a constitutional requirement that the Parliament shall meet at least


(a) twice in a year (b) thrice in a year
(c) once in a year (d) none of the above

126. Governor of a State can be removed by


(a) impeachment by State Legislature
(b) the President
(c) by the State Cabinet
(d) the Union Government at the request of the Chief Minister

127. Sovereignty under the Constitution belongs to


(a) the Parliament (b) the People
(c) the Supreme Court (d) the President along with Parliament

128. The Supreme Court upheld Mandal Commission Report in


(a) Bommai v. Union of India (b) Indra Sawhney v. Union of India
(c) Unnikrishan v. Union of India (d) Maneka Gandhi v. Union of India

129. Under our Constitution, right to property is


(a) fundamental right (b) basic structure of the Constitution
(c) Constitutional right (d) a mere legal right

130. The Chairman of Sixth Pay Commission


(a) Justice B.N. Srikrishna (b) Justice Ratnavel Pandian
(c) Justice Jagannatha Shetty (d) Justice A.K. Majumdar

Previous Years
Page 14 CLAT & AILET Papers
131. Right to education emanates from:
(a) right to culture and education under Articles 29 and 30
(b) right to equality under Article 14
(c) freedom of speech & expression under Article 19
(d) right to life and personal liberty under Article 21

132. International Court of Justice is:


(a) an independent international institution (b) a principal organ of the U.N.O.
(c) a subsidiary organ of the U.N.O. (d) an European Institution

133. The Liberhan Commission which received repeated extensions has been inquiring into:
(a) Godhra riots (b) Mumbai riot
(c) demolition of Babri Masjid (d) killing of Sikhs in Delhi

134. The Monopolies and Restrictive Trade Practices Act was repealed by:
(a) Competition Act
(b) Consumer Protection Act
(c) Foreign Trade (Development and Regulation) Act
(d) Liberalization Policy of the Government

135. Only Judge against whom a motion of impeachment was introduced into Parliament
(a) Justice Subba Rao (b) Justice Ramaswami
(c) Justice Mahajan (d) Justice Viraswami

136. The Mallimath Committee Report deals with:


(a) judicial delays in India (b) criminal justice administration
(c) stock market reforms (d) review of Constitutional system

137. The first Woman Chief Justice of High Court in India


(a) Leila Mukherji (b) Leila Seth
(c) Fatima Bibi (d) Ruma Pal

138. Lok Adalats have been created under:


(a) Legal Services Authority Act (b) Arbitration and Conciliation Act
(c) Administration of Justice Act (d) None of the above

139. Recent Nepal elections are globally significant because


(a) Communists came to power through ballot box for the first time in the world
(b) Monarchy was defeated by democratic forces
(c) A militant movement joined the mainstream
(d) Secularism triumphed over theocracy

140. The Third World leader who has been defying the U.S.A:
(a) Fidel Castro (b) Hugo Chavez
(c) Robert Mughabe (d) Hu Jintao

Previous Years
CLAT & AILET Papers Page 15
SECTION - III : MATHEMATICAL ABILITY

Instructions: From the four answers given, shade the appropriate answer in the space provided
for it on the OMR answer sheet.

Marks: Each question carries 2 (two) marks. (Total 20 marks)

141. Raju earns twice as much in March as in each of the other months of the year. What part of his
annual earnings he earns in that month?
(a) 1/5 (b) 5/7 (c) 2/13 (d) 1/10

142. Sanjay sold his watch for Rs. 1140 and thereby loses 5%. In order to gain 5% he has to sell the
watch for
(a) Rs. 1254 (b) Rs. 1260 (c) Rs. 1197 (d) Rs. 1311

143. A mixture of 40 litres of milk and water contains 10% of water. How much water is to be added to the
mixture so that the water may be 20% in the new mixture.
(a) 5 Litres (b) 4 Litres (c) 6.5 Litres (d) 7.5 Litres

144. A train 100 meters long running at 54 km/hr takes 20 seconds to pass a bridge. The length of the
bridge is
(a) 50 mt (b) 150 mt (c) 200 mt (d) 620 mt

145. Sameer is as much younger to Mohan as he is older to Arun. If the sum of the ages of Mohan and
Arun is 48, the age of Sameer is
(a) 20 years (b) 24 years (c) 30 years (d) cannot be determined

146. A tank can be filled up by two pipes A and B in 2 hours and 3 hours respectively. A third pipe C can
empty the full tank in 6 hours. If all the taps can be turned on at the same time, the tank will be full
in
(a) 1 hour (b) 40 minutes (c) 1 1/2 hours (d) 3 hours

147. Of the three numbers, the first is one third of the second and twice the third. The average of these
numbers is 27. The largest of these numbers is
(a) 18 (b) 36 (c) 54 (d) 108

148. The length of a square is increased by 15% and breadth decreased by 15%. The area of the
rectangle so formed is
(a) neither increases nor decreases (b) decreases by 2.25%
(c) increases by 2.25% (d) decreases by 22.5%

149. The ratio of milk and water in 60 Litres of adulterated milk is 2 : 1. If the ratio of milk and water is to
be 1 : 2, then the amount of water to be added further is
(a) 20 Litres (b) 30 Litres (c) 40 Litres (d) 60 Litres
150. A piece of cloth costs Rs. 70. If the piece is 4 metre longer and each metre costs Rs. 2 less, the
cost remains unchanged. The length of the piece is
(a) 8 m (b) 9 m (c) 10 m (d) 12 m

Previous Years
Page 16 CLAT & AILET Papers
SECTION - IV: LOGICAL REASONING

Instructions: Read carefully the questions and shade the appropriate answer in the space provided
for it on the OMR answer sheet.

Marks: Each question carries 2 (two) marks. (Total 50 marks)

151. A college received fifty applications for a certain course. In the qualifying examination, one-tenth of
them secured marks in 90-95% range. Within remaining segment, three-fifths of them secured
marks in 75-90% range. The rest secured below 75%. To get admission, the following restrictions
hold good:
i) No student who has scored below 75% can seek admission to Physics course.
ii) No student is allowed to opt Physics without opting Mathematics.
iii) No student is allowed to opt Physics and Astrophysics simultaneously.
iv) To opt Mathematics or Astrophysics, a student should have scored at least 70% in the qualifying
examination.

Which one of the following alternatives is possible?


(a) Ninety percent of the applicants are admitted to Physics course.
(b) Thirty-five percent of the applicants who are otherwise inelligible to join Physics course are
admitted to Mathematics and Astrophysics course.
(c) Students of Physics course outnumber those of Mathematics.
(d) Whoever is eligible to study Mathematics is also eligible to study Physics.

152. A tourist can tour utmost four places out of A, B, C, D, E, F and G. Out of four, not more than two can
come under holiday tour and at least two must come under business trip. The break up is as
follows:
A, B C and D - Business tour; E, F and G - Holiday tour
The following restrictions hold good.
(A) If A is included, then both C and G are excluded.
(B) If neither E nor F is included, then B or G or both of them can be included.
(C) If G is included, then D cannot be included.

Which one of the following combinations is possible?


(a) A, C, E and F (b) B, G and E
(c) A, D and G (d) A, B and D

153. Under the same fact situation as above, suppose that the following restrictions hold good:
(A) A can be included provided C is included.
(B) E is included provided B or G is included but not both.
(C) C can be included provided at least D or F is excluded.

Which one of the following is a certainty?


(a) A, B, C, & E (b) A, C, D & F
(c) B, C, D & E (d) A, B, C & F

Previous Years
CLAT & AILET Papers Page 17
154. Four members have to be nominated to a Committee and there are six candidates: A, B, C, D, E
and F. The following restrictions hold good:
(A) If A is nominated, then D does not find any place.
(B) If B is nominated, then either E or F has to be nominated, but not both.
(C) If C is nominated, then both D and E have to be nominated.

Which one of the following is an acceptable combination?


(a) A, B, C and E (b) A, B, C and D (c) B, C, D and E (d) B, C, D and F

155. Political turmoil in a country is mainly caused by widespread violence and flawed economic policies
of successive governments. If at all this has to be crushed, it can be achieved only by a dictatorial
government which rules with iron hand. Therefore, the need of the hour is to elect a government
which imposes fresh set of stringent legislations.

The alternatives suggested (not necessarily all), if true, considerably weaken the argument. However,
one of them is most forceful. Identify the same.
(a) It is not the imposition of new legislations which is required, but effective adherence to the
existing legislations.
(b) That government is the best government which governs least.
(c) It is possible to overcome any evil by educating people.
(d) Only dialogue in a free society can eradicate political turmoil.

156. Under the same fact situation as above, the alternatives suggested (not necessarily all), if true,
significantly strengthen the argument. However, one of them is most forceful. Identify the same.
(a) Espionage activities by enemy nations, which contribute to political turmoil, can be prevented
only if the government is very strong.
(b) The philosophy behind any economic policy, ‘push from bottom, press from top’ is to be followed
to mitigate violence, and it is not observed.
(c) Political turmoil is due to corrupt establishment.
(d) Man is, by nature, a beast.

157. Exploitation of poor by rich can be stemmed only if the state exercises complete control over
agriculture and industrial production. But state control is beset by two evils; corruption and delay.
The net result is that if man tries to escape from one evil, then he is trapped by another. Suffering,
hence, is inescapable.

The argument presented above seems to imply the following conclusions. Identify the one which is
least dubious. Apply common sense.
(a) If agriculture and industrial production can be abolished, we can free ourselves from all forms of
evil.
(b) To avoid evil austere life shall be encouraged.
(c) The gap between poor and rich can be bridged by heavily taxing the rich and passing on the
largesse to the poor.
(d) Man is, by nature, dishonest.

Previous Years
Page 18 CLAT & AILET Papers
158. That the human soul is immaterial is an undisputed fact. Significantly, what is not matter is not
spatial and consequently, it is not vulnerable to motion. Evidently, no motion no dissolution. What
escapes from dissolution is also free from corruptibility.

Therefore, the human soul is immortal. In this argument, one premise is missing. Complete the
argument by choosing from the following:
(a) Nothing is free from dissolution (b) What is incorruptible is immortal
(c) There is no motion (d) Matter does not exist

159. Under the same fact situation as above, which one of the following, if true, affects seriously the
argument presented above?
(a) Matter is not bound by space
(b) Matter is indestructible
(c) Whatever exists is not necessarily affected by motion.
(d) What is not matter also is vulnerable to motion.

160. Protagonists of human rights vehemently oppose capital punishment. Their opposition stems mainly
from three reasons. Firstly, man cannot terminate what he cannot generate. Secondly, the function
of punishment is to reform the culprit. Thirdly, a culprit should be given an opportunity to repent.
Admittedly, death penalty fails on all three counts. However, the defenders argue that a person is
punished because he has to pay for his deeds. Reformation or repentance, according to them, is
peripheral. Hence, death penalty is admissible.

Which one of the following is the focus of this debate?


(a) Man’s rights and privileges (b) Nature and purpose of punishment
(c) Prevention of crime (d) Mercy and revenge

161. Since Venus rotates slowly, Fred Whipple thought that like Mercury, Venus keeps one face always
towards the Sun. If so, he said that the dark side would be very cold. However, he knew with the help
of earlier study carried out by Petit and Nicholson that it was not the case. So, he concluded that
the planet must rotate fairly often to keep the darker side warmer.

Which of the following is the original premise?


(a) Slow rotation of Venus (b) Temperature of Venus.
(c) Frequent rotation of Venus (d) Equality of the rate of rotation and revolution.

162. Before formulating the laws of motion, Galileo distinguished between mathematical study and empirical
study. He, first, theoretically derived the relation between distances and times for uniformly
accelerating motion by letting the ball roll a quarter, then half, then two-thirds and so on of the length
of the groove and then measured the times on each occasion, which he repeated hundred times. He
calculated, based on this study, that the distance travelled equalled the square of the time on all
occasions.

Which one of the following characterizes Galileo’s method?


(a) Speculation (b) Theoretical analysis
(c) Generalization (d) Statistical analysis

Previous Years
CLAT & AILET Papers Page 19
163. Read carefully a brief summary of one of the investigations of Sherlock Holmes: “While investigating
the murders of Stangerson and Enoch Drebber he got into conversation with fellow detectives which
runs as follows: “The last link...... My case is complete.... Could you lay your hands upon those
pills”. After he got those pills, Holmes cut one of them, dissolved it in water and placed it in front of
the terrier. Contrary to his expectations, the animal survived. Though disappointed a bit, he thought
for a while and then cut the other pill, dissolved it, added milk and placed before the animal. The
moment it licked, the animal died. Those were the pills present at the scenes of crime.
Which one of the following aptly describes the method which this passage indicates?
(a) Imagination (b) Experiment (c) Observation (d) Thought experiment

164. There has been much speculation concerning the origin of lunar craters. One hypothesis is that
they are the results of the impact of heavy meteors on the surface of the moon while still soft. The
most probable explanation is that they were produced by the gases liberated from the rocky matter.
While solidification was taking place these gases and water vapors steadily escaped through viscous
surface, raising giant bubbles. The reader can easily visualize the process that took place by
watching frying of pancakes and noticing the formation of bubbles and craters on their surface.
Which one of the following actually helps us to determine the origin of lunar craters?
(a) Analogy (b) Study of foreign body
(c) Course and effect relation (d) Speculation

165. “Perhaps the earliest work of Archimedes that we have is that on ‘Plane Equilibrium’. In this, some
fundamental principles of mechanics are set forth as rigorous geometric propositions. The work
opens with famous postulate ‘Equal weights at equal distances are in equilibrium; equal weights at
unequal distances are not in equilibrium, but incline toward the weight at the greater distance”.
According to this passage, which factor or factors determine equilibrium?
(a) weight (b) distance
(c) weight & distance (d) equality of weights & distances

166. According to the above passage, which one of the following values can be assigned to the
statement inequal weights at equal distances are in disequilibrium?
(a) true (b) false (c) highly probable (d) highly improbable

167. According to the above passage, which one of the following values can be assigned to the
statement ‘inequal weights at inequal distances are in disequilibrium’?
(a) true (b) false (c) uncertain (d) unverifiable

168. ‘Gregor Mendel in examining tea-plants found two sharply marked races, the tall and the short. He
experimentally fertilized flowers of tall plants with pollen of short. The off spring were tall plants. He
next let the flowers of this first generation be fertilized with their own pollen. In the following generation,
shortness reappeared. Tallness and shortness were distributed not at random but in a definite,
constant, and simple ratio: three dominant talls to one recessive short’. Which one of the following
aptly describes the distribution of dominant and recessive characteristics?
(a) systematic (b) equal interval (c) unpredictable (d) irregular interval

Previous Years
Page 20 CLAT & AILET Papers
169. It is said that in his strongly worded reaction to quantum Physics, Einstein remarks ‘God does not
play dice’ to which Bohr, another great physicist, reacted saying ‘Do not tell God what to do’. Bohr,
earlier had argued that we can never know what the properties of an isolated quantum system,
though we can know the properties of macrocosmic objects.

Which one of the following is the focus of their debate?


(a) the behaviour of God
(b) probabilistic interpretation of the behaviour of quantum object
(c) limits of human knowledge
(d) irrelevance of microcosmic object

170. An efficient and diesel-independent public transport system is essential to the economic development
of nation. Suppose that the government adopts a policy to that effect, then there is another favorable
result. The pollution of environment is reduced to a greater extent. But, then it has two-pronged
backlash. The sale and consequently the production of two and four wheelers reduce to the minimum
which in turn render a large number of people jobless. Cash flow to the treasury also is adversely
affected. Such a step, therefore, is self-defeating unless the government evolves a counter-strategy
to nullify the adverse effects.

Which one of the following accurately projects the opinion of an imaginary speaker or author, as the
case may be, of this passage?
(a) Abandon the idea of efficient and diesel-independent public transport system.
(b) Ensure sustained cash flow and create better job opportunities by inventing an alternate or more
than one alternate.
(c) Public transport system shall be given up.
(d) Maintain production and sale at the same level by offering incentives.

171. A moot question to be considered is whether democratic form of government is a boon or bane, no
matter what Lincoln might or might not have said. Rather his most (in?) famous adage, ‘by the
people, for the people and of the people’ misses the most pertinent question; which attitude works
behind when a person declares that he is a (or the right?) candidate to serve the people, and does
not hesitate to contest and fight tooth and nail the election, an euphemism for battle with or without
bullets. Admittedly, the covert attitude is different from overt attitude. Hardly any one contests the
election unwillingly. A contestant is not persuaded by any one, but driven by his own passions and
dubious motives. Contrast this picture with Socrates’ version; no honest man willingly takes up the
job of ruler. If at all he accepts, he does so for fear of being ruled by one made up of inferior mettle.
It is beyond even the wildest imagination, to expect an honest person to contest the election.

Assuming that every statement is true, identify from among the given alternatives the one which
strictly follows from the passage.
(a) No assessment or appraisal of democracy is possible.
(b) Lincoln and Socrates are talking differently.
(c) Actually, Socrates scores over Lincoln on this issue.
(d) Rulers can be honest.

Previous Years
CLAT & AILET Papers Page 21
172. According to the above passage, which one of the following correctly differentiates Lincoln’s and
Socrates’ analyses?
(a) the nature of democracy (b) merits and demerits of politicians
(c) qualities of election (d) difference in mind set of respective men

173. Many environmentalists either adopt double standard or do not know what they are talking about. A
protagonist of environment, for obvious reasons, ought not to bat for any type of progress because
progress without meddling with nature is a myth. But none can live without scientific and technological
advance which has singularly made progress possible. Furthermore, environment includes not just
forest wealth and hills, but animal wealth also. An honest environmentalist is obliged to address the
following questions. First, should man in the interest of hygiene, kill any living being be it an insect
purported to be harmful or stray dogs? After all, this world does not belong to man alone.

Which one of the following runs counter to the spirit of the passage?
(a) In the interest of health and cleanliness, our surroundings must be from disease-spreading
bacteria
(b) Non-violence as a moral principle extends to all living creatures
(c) Vegetarian food is ideal to all men
(d) Man should protect his environment because he has to live

174. Does our society need reservation in jobs? Before we defend reservation, we must consider some
issues. Why do we need reservation? Obviously, reservation is required to lift the downtrodden and
thereby achieve equality. How do you achieve this? Every individual, without exception, has a right
to receive quality education. It is more so in the case of downtrodden people. Only a good-natured
meritorious teacher can impart quality education. Suppose that a person who is neither good natured
nor meritorious becomes a teacher thanks to reservation system. Then generations of students
suffer.

Suppose that there is some merit in this argument. Then which of the following aptly describes the
fall-out of this argument?
(a) Reservation is individual - centric, but not group - centric
(b) Reservation, in at least one field, is self-defeating
(c) The argument is biased
(d) Education is not required to uplift the downtrodden

175. Under the same fact situation as above, which one of the following helps you to circumvent the
situation?
(a) Replace education with money and make poor rich.
(b) To achieve equality encourage inter-caste marriage.
(c) Only downtrodden people should form the government.
(d) Identify good natured and meritorious people within downtrodden group to make them teachers.

Previous Years
Page 22 CLAT & AILET Papers
SECTION - V
LEGAL REASONING

PART - A

Instructions: Each question contains some basic principles and fact situation in which these basic
principles have to be applied. A list of probable decisions and reasons are given. You have to
choose a decision with reasons specified by shading the appropriate answer in the space provided
for it on the OMR answer sheet.

Marks: Each question carries 3 (three) marks. (Total 30 marks)

176. PRINCIPLES:
(1) On the death of husband, the widow shall inherit the property of her deceased husband along
with children equally.
(2) A widow can not claim the property of the deceased if on the date when the question of succession
opens, she has remarried.
(3) A female acquiring property in any way has the absolute title to the property.
Apply the above three principles and decide the case of the following fact situation:

FACTS:
When Sudhir died, he had 1/3rd share of the family property, which the three brothers Rudhir, Sudhir
and Yasu inherited from their father, B.
Sudhir died on September 23rd 2006 without having any issue. The widow of Sudhir, Ms. Win
remarried on January 1, 2007.
Rudhir and Yasu refused ‘Win’ the share from Sudhir’s portion when Win claimed the entire property
belonging to Sudhir on January 30, 2007.
Select your decision from the possible decisions given in list I and the appropriate reason from the
indicated reasons given in list II given below:

List I - Decisions
(a) Widow cannot inherit the property of Sudhir
(b) Widow can inherit the property of Sudhir

List II - Reasons
i) Widow does not belong to the family
ii) Widow was remarried
iii) Her claim was on the date of Sudhir’s death
iv) Her claim was submitted after she was remarried
Your decision and reason

Shade the right decision with reason from the following


(a) (a) (i) (b) (a) (ii) (c) (b) (iii) (d) (b) (iv)

Previous Years
CLAT & AILET Papers Page 23
177. PRINCIPLES:
1) If a person commits an act by which death is caused to another person and the act is done with
the intention of causing death, that person is liable for murder.
2) A person has a right of self defence to the extent of causing death to another provided he
apprehends death by the act of the latter.
FACTS:
Shuva went to a hardware shop owned by Anup. Bargaining on some item led to altercation between
the two and Shuva picked up a sharp object and hit at Anup. When Anup started bleeding, his wife
Mridula intervened and she was also hit by Shuva and she became unconscious. Finding himself
totally cornered, Anup delivered a severe blow to Shuva with a sharp object. Shuva died instantly.
Possible decisions
a) Anup murdered Shuva.
b) Anup killed Shuva with the intention of killing to save himself and his wife.
c) Anup killed Shuva without any intention to do so just to save himself and his wife.
Probable reasons for the decision
i) If a person kills another instantly on the spot, the intention to kill is obvious.
ii) Anup used force apprehending death of himself and his wife.
iii) Anup used disproportionate force.
iv) There was nothing to show that Shuva wanted to kill Anup or his wife.
Your decision with the reason
(a) (a) (i) (b) (a) (iii) (c) (c) (ii) (d) (b) (i)

178. PRINCIPLES:
1) Consumable goods which are not fit for consumption are not marketable.
2) A consumer shall not suffer on account of unmarketable goods.
3) A seller is liable for knowingly selling unmarketable goods.
4) A manufacturer shall be liable for the quality of his products.
FACTS:
Ram bought a Coca Cola bottle from Shama’s shop. Back at home, the server opened the bottle
and poured the drink into the glasses of Ram and his friend Tom. As Tom started drinking, he felt
irritation in his throat. Immediately, Ram and Tom took the sample to test and found nitric acid in the
content. Ram filed a suit against Shama, Coca Cola company and the bottler, Kishen and Co.
Suggested Decisions
(a) Ram cannot get compensation
(b) Tom can get compensation
(c) Both Ram and Tom can get compensation
Suggested Reasons
i) Shama did not know the contents of sealed bottle.
ii) Ram did not actually suffer though he bought the bottle.
iii) Tom did not buy the bottle.
iv) Coca Cola company is responsible since it supplied the concentrate.
v) Kishen & Co, is responsible since it added water, sugar, etc., and sealed the bottle.
vi) Shama is responsible for selling the defective product.
Your decision with the reason
(a) (a) (ii) (b) (b) (vi) (c) (c) (v) (d) (c) (iv)
Previous Years
Page 24 CLAT & AILET Papers
179. PRINCIPLES
1. If A is asked to do something by B, B is responsible for the act, not A.
2. If A, while acting for B commits a wrong, A is responsible for the wrong, not B.
3. If A is authorised to do something for B, but in the name of A without disclosing B’s presence,
both A and B may be held liable.

FACTS:
Somu contracted with Amar whereunder Amar would buy a pumpset to be used in Somu’s farm.
Such a pumpset was in short supply in the market. Gulab, a dealer, had such a pumpset and he
refused to sell it to Amar. Amar threatened Gulab of serious consequences if he fails to part with the
pumpset. Gulab filed a complaint against Amar.
PROPOSED DECISION
(a) Amar alone is liable for the wrong though he acted for Somu.
(b) Amar is not liable for the wrong, though he is bound by the contract with Somu.
(c) Somu is bound by the contract and liable for the wrong.
(d) Both Somu and Amar are liable for the wrong.
SUGGESTED REASONS

(i) Amar committed the wrong while acting for the benefit of Somu.
(ii) Amar cannot do while acting for Somu something which he cannot do while acting for himself.
(iii) Both Amar and Somu are liable since they are bound by the contract.
(iv) Somu has to be responsible for the act of Amar committed for Somu’s benefit.

YOUR DECISION WITH THE REASON


(a) (a) (i) (b) (a) (ii) (c) (c) (iii) (d) (d) (iv)

180. PRINCIPLES:
1. The owner of a land has absolute interest on the property including the contents over and under
the property.
2. Water flowing below your land is not yours though you can use it.
3. Any construction on your land belongs to you.
4. All mineral resources below the land belongs to the State.

FACTS:
There is a subterranean water flow under Suresh’s land surface. Suresh constructed a huge reservoir
and drew all subterranean water to the reservoir. As a result, the wells of all adjacent property
owners have gone dry. They demanded that either Suresh must demolish the reservoir or share the
reservoir water with them.

Proposed Decision
(a) Suresh need not demolish the reservoir.
(b) Suresh has to demolish the reservoir.
(c) Suresh has to share the water with his neighbours.
(d) The Government can take over the reservoir.

Previous Years
CLAT & AILET Papers Page 25
Possible reasons
i) Water cannot be captured by one person for his personal use.
ii) The Government must ensure equitable distribution of water.
iii) Whatever is under Suresh’s land may be used by him.
iv) Suresh has to respect the rights of others regarding water.

Your decision with the reason


(a) (a) (iii) (b) (b) (i) (c) (c) (iv) (d) (d) (ii)

181. PRINCIPLES
1. An employer shall be liable for the wrongs committed by his employees in the course of
employment.
2. Third parties must exercise reasonable care to find out whether a person is actually acting in the
course of employment.

FACTS:
Nandan was appointed by Syndicate Bank to collect small savings from its customers spread over
in different places on daily basis. Nagamma, a housemaid, was one of such customers making use
of Nandan’s service. Syndicate Bank after a couple of years terminated Nandan’s service. Nagamma,
unaware of this fact, was handing over her savings to Nandan who misappropriated them. Nagamma
realised this nearly after three months, when she went to the Bank to withdraw money. She filed a
complaint against the Bank.

POSSIBLE DECISION

(a) Syndicate Bank shall be liable to compensate Nagamma.


(b) Syndicate Bank shall not be liable to compensate Nagamma.
(c) Nagamma has to blame herself for her negligence.

POSSIBLE REASONS

(i) Nandan was not acting in the course of employment after the termination of his service.
(ii) A person cannot blame others for his own negligence.
(iii) Nagamma was entitled to be informed by the Bank about Nandan.
(iv) The Bank is entitled to expect its customers to know actual position.

YOUR DECISION WITH THE REASON

(a) (b) (i) (b) (c) (ii) (c) (a) (iii) (d) (b) (iv)

182. PRINCIPLES
1. A master shall be liable for the fraudulent acts of his servants committed in the course of
employment.
2. Whether an act is committed in the course of employment has to be judged in the context of
case.
3. Both master and third parties must exercise reasonable care in this regard.

Previous Years
Page 26 CLAT & AILET Papers
FACTS:
Rama Bhai was an uneducated widow and she opened a S.B. account with Syndicate Bank with
the help of her nephew by name Keshav who was at that time working as a clerk in the Bank.
Keshav used to deposit the money of Rama Bhai from time to time and get the entries done in the
passbook. After a year or so, Keshav was dismissed from the service by the Bank. Being unaware
of this fact, Rama Bhai continued to hand over her savings to him and Keshav misappropriated
them. Rama Bhai realised this only when Keshav disappeared from the scene one day and she
sought compensation from the Bank.

POSSIBLE DECISIONS

(a) Syndicate Bank shall be liable to compensate Rama Bhai.


(b) Syndicate Bank shall not be liable to compensate Rama Bhai.
(c) Rama Bhai cannot blame others for her negligence.
POSSIBLE REASONS

(i) Keshav was not an employee of the Bank when the fraud was committed.
(ii) The Bank was not aware of the special arrangement between Rama Bhai and Keshav.
(iii) It is the Bank’s duty to take care of vulnerable customers.
(iv) Rama Bhai should have checked about Keshav in her own interest.
Your decision with the reason
(a) (a) (iii) (b) (c) (iv) (c) (b) (ii) (d) (b) (i)

183. PRINCIPLES
1. A person is liable for negligence, if he fails to take care of his neighbour’s interest.
2. A neighbour is anyone whose interests should have been foreseeable by a reasonable man
while carrying on his activities.

FACTS:
A cricket match was going on in a closed door stadium. A cricket fan who could not get into the
stadium was watching the game by climbing up a nearby tree and sitting there. The cricket ball in
the course of the game went out of the stadium and hit this person and injured him. He filed a suit
against the organizers.
POSSIBLE DECISIONS
(a) The organizers are liable to compensate the injured person.
(b) The organizers are not liable to compensate the injured person.
(c) The injured person should have avoided the place where he might be hit by the cricket ball.

POSSIBLE REASONS
(i) The organizers are responsible for the people inside the stadium.
(ii) The organizers could not have foreseen somebody watching the game by climbing up a tree.
(iii) A person crazy about something must pay the price for that.
(iv) The organizers shall be liable to everybody likely to watch the game.

YOUR DECISION WITH THE REASON


(a) (a) (iv) (b) (a) (iii) (c) (b) (iii) (d) (c) (i)

Previous Years
CLAT & AILET Papers Page 27
184. PRINCIPLES
1. When a person unlawfully interferes in the chattel of another person by which the latter is
deprived of its use, the former commits the tort of conversion.
2. Nobody shall enrich himself at other’s expense.
FACTS:
A patient suffering from stomach ailment approached a teaching hospital. He was diagnosed as
suffering from appendicitis and his appendix was removed. He became alright. The hospital however
found some unique cells in the appendix and using the cell lines thereof, it developed drugs of
enormous commercial value. When the erstwhile patient came to know about it, he claimed a share
in the profit made by the hospital.

POSSIBLE DECISIONS
(a) The hospital need not share its profits with the patient.
(b) The hospital may share its profits on ex gratia basis.
(c) The hospital shall share its profits with the patient.
POSSIBLE REASONS
(i) The patient, far from being deprived of the use of his appendix, actually benefitted by its removal.
(ii) The hospital instead of throwing away the appendix conducted further research on it on its own
and the development of drug was the result of its own effort.
(iii) The hospital could not have achieved its success without that appendix belonging to the patient.
(iv) Everybody must care for and share with others.
YOUR DECISION WITH THE REASON
(a) (a) (i) (b) (a) (ii) (c) (c) (iii) (d) (c) (iv)

185. PRINCIPLES:
1. Copying including attempt to copy in examinations is a serious offence.
2. One shall not take any unauthorised materials into the examination hall.

FACTS:
Rohini, an examinee in PUC., was thoroughly checked while entering into the examination hall. She
did not have anything other than authorised materials such as pen, instrument box, etc., with her.
As she was writing her paper, an invigilator found close to her feet a bunch of chits. The invigilator on
scrutiny found that the chits contained answers to the paper being written by Rohini. Rohini’s
answers tallied with the answers in the chits. A charge of copying was levelled against Rohini.
Probable Decisions
a) Rohini shall be punished for copying.
b) Rohini cannot be punished for copying.
Probable Reasons
i) Something lying near the feet does not mean that the person is in possession of that thing.
ii) The fact that she was checked thoroughly while getting into the hall must be conclusive.
iii) Similarities between her answers and the answers in the chit indicate that she used those chits.
iv) After using those chits, she must have failed to dispose of them properly.

Your decision with the reason


(a) (a) (iii) (b) (a) (iv) (c) (b) (iii) (d) (b) (i)

Previous Years
Page 28 CLAT & AILET Papers
PART - B

Instructions: From the four answers given, shade the appropriate answer in the space provided
for it on the OMR answer sheet.

Marks: Each question carries 2 (two) marks. (10 marks)

186. All contracts are agreements. All agreements are accepted offers.
Which of the following derivation is correct?
(a) All accepted offers are contracts (b) All agreements are contracts
(c) All contracts are accepted offers (d) None of the above.

187. No minor can enter into a contract of work. Working in a shop can be done only by a contract.
Which of the following derivation is correct?
(a) A minor cannot work in a shop
(b) A shop cannot contract with a minor
(c) There cannot be a contract to which minor is a party
(d) None of the above.

188. All motor vehicles are required to have third party insurance. Any vehicle not using mechanical
device is not a motor vehicle.
Which of the following is correct derivation from the above?
(a) All Third Party Insurances relate to motor vehicles
(b) Vehicles not using mechanical device need not have Third Party Insurance
(c) All vehicles must have Third Party Insurance
(d) None of the above.

189. A contract contravening public policy is void. There cannot be a general definition of public policy.
Which of the following is correct derivation from the above?
(a) There cannot be a general definition of contract
(b) Since public policy is uncertain, contract is also uncertain
(c) The impact of public policy on contract is to be judged in individual cases
(d) None of the above

190. International Law is the law between sovereign states. A sovereign is the supreme authority not
bound by legal constraints.
Which of the following is correct derivation from the above?
(a) International law is not law binding on the sovereign states
(b) International Law is only a positive morality
(c) International law is in the nature of pact between sovereign states
(d) None of the above

Previous Years
CLAT & AILET Papers Page 29
CLAT Question Paper 2009

SECTION – I
English Including Comprehension

PART – A

Instructions: (Question 1-10), Read the given passage carefully and answer the questions that
follow. Shade the appropriate answer in the space provided for it on the OMR Answer Sheet.

Marks: Each question carries 1 (one) mark. (Total 10 marks)

There is a fairly universal sentiment that the use of nuclear weapons is clearly contrary to morality and that
its production probably so, does not go far enough. These activities are not only opposed to morality but
also to law if the legal objection can be added to the moral, the argument against the use and the manufacture
of these weapons will considerably be reinforced. Now the time is ripe to evaluate the responsibility of
scientists who knowingly use their expertise for the construction of such weapons, which has deleterious
effect on mankind.

To this must be added the fact that more than 50 percent of the skilled scientific manpower in the world is
now engaged in the armaments industry. How appropriate it is that all this valuable skill should be devoted
to the manufacture of weapons of death in a world of poverty is a question that must touch the scientific
conscience.

A meeting of biologists on the Long-Term Worldwide Biological consequences of nuclear war added frightening
dimension to those forecasts. Its report suggested that the long biological effects resulting from climatic
changes may at least be as serious as the immediate ones. Sub-freezing temperatures, low light levels,
and high doses of ionizing and ultraviolet radiation extending for many months after a large-scale nuclear
war could destroy the biological support system of civilization, at least in the Northern Hemisphere.
Productivity in natural and agricultural ecosystems could be severely restricted for a year or more. Post
war survivors would face starvation as well as freezing conditions in the dark and be exposed to near lethal
doses of radiation. If, as now seems possible, the Southern Hemisphere were affected also, global disruption
of the biosphere could ensure. In any event, there would be severe consequences, even in the areas not
affected directly, because of the inter-dependence of the world economy. In either case the extinction of a
large fraction of the earth’s animals, plants and microorganism seems possible. The population size of
Homo sapiens conceivably could be reduced to prehistoric levels or below, and extinction of the human
species itself cannot be excluded.

1. Choose the word, which is most opposite in meaning of the word, printed in bold as used
in the passage Deleterious.
(a) Beneficial (b) Harmful (c) Irreparable (d) Non-cognizable

2. The author’s most important objective of writing the above passage seems to ……….
(a) Highlight the use of nuclear weapons as an effective population control measures.
(b) Illustrate the devastating effects of use of nuclear weapons on mankind.
(c) Duly highlight the supremacy of the nations which possess nuclear weapons.
(d) Summarise the long biological effects of use of nuclear weapons.

Previous Years
Page 30 CLAT & AILET Papers
3. The scientist engaged in manufacturing destructive weapons are………… .
(a) Very few in number
(b) Irresponsible and imcompetent
(c) More than half of the total number
(d) Engaged in the armaments industry against their desire

4. According to the passage, the argument on use and manufacture of nuclear weapons
(a) Does not stand the test of legality
(b) Possesses legal strength although it does not have moral standing
(c) Is acceptable only on moral grounds
(d) Becomes stronger if legal and moral considerations are combined

5. The author of the passage seems to be of the view that


(a) Utilization of scientific skills in manufacture of weapons is appropriate.
(b) Manufacture of weapons of death would help eradication of poverty.
(c) Spending money on manufacture of weapons may be justifiable subject to the availability of
funds.
(d) Utilization of valuable knowledge for manufacture of lethal weapons is inhuman.

6. Which of the following is one of the consequences of nuclear war?


(a) Fertility of land will last for a year or so.
(b) Post-war survivors being very few will have abundant food.
(c) Lights would be cooler and more comfortable.
(d) Southern Hemisphere would remain quite safe in the Post-war period.

7. Which of the following best explains the word devoted, as used in the passage?
(a) Dedicated for a good cause
(b) Utilised for betterment
(c) Abused for destruction
(d) Underutilised

8. The biological consequences of nuclear war as given in the passage include all the
following, except
(a) Fall in temperature below zero degree Celsius.
(b) Ultraviolet radiation
(c) High doses of ionizing
(d) Abundant food for smaller population.

9. It appears from the passage that the use of nuclear weapons is considered against morality
by
(a) Only such of those nations who cannot afford to manufacture and sell weapons
(b) Almost all the nations of the world
(c) Only the superpowers who can afford to manufacture and sell weapons
(d) Most of the scientists devote their valuable skills to manufacture nuclear weapsons.

Previous Years
CLAT & AILET Papers Page 31
10. Which of the following statements I, II, III, and IV is definitely true in the context of the
passage?
(I) There is every likelihood of survival of the human species as a consequence of nuclear war.
(II) Nuclear war risks and harmful effects are highly exaggerated.
(III) The post war survivors would be exposed to the benefits of non-lethal radiation.
(IV) Living organisms in the areas which are not directly affected by nuclear was would also suffer.
(a) I (b) III (c) II (d) IV

PART – B
Instructions: (Questions 11-15), Three of the four words given in these questions are spelt wrong.
Select the word that is spelt correct and shade the appropriate answer in the space provided for
it on the OMR Answer Sheet.

Marks: Each question carries 1 (One) mark. (Total 5 marks)

11. (a) renaissance (b) renaisance (c) renaissence (d) renaisence

12. (a) malaese (b) melaize (c) melaise (d) malaise

13. (a) irelevant (b) itrelevent (c) irrelevant (d) irrellevant

14. (a) survilance (b) surveillance (c) surveiliance (d) surveilliance

15. (a) gaiety (b) gaietty (c) gaeity (d) gaitty

PART – C

Instructions: (Question 16-20), Given below are a few foreign language phrases that are commonly
used. Choose the correct meaning for each of the phrases and shade the appropriate answer in
the space provided for it on the OMR Answer Sheet.

Marks: Each question carries 1 (One) (Total 5 marks)

16. Mala fide


(a) in good faith (b) in bad faith (c) without any faith (d) with full faith

17. Pro rata


(a) at the rate of (b) at quoted rate (c) in proportion (d) beyond all proportion

18. Vice versa


(a) in verse (b) versatile verse (c) in consonance with (d) the other way round

19. Ab initio
(a) from the very beginning (b) high initiative
(c) thing done later (d) without initiative

20. Alibi
(a) every where (b) else where
(c) no where (d) without any excuse
Previous Years
Page 32 CLAT & AILET Papers
PART – D

Instructions: (Question 21-25), Some idioms given below are commonly used. Choose the correct
meaning for each of the idioms and shade the appropriate answer in the space provided for it on
the OMR Answer Sheet.

Marks: Each question carries 1 (One) (Total 5 marks)

21. To give the game away


(a) To lose the game (b) To give a walk-over in a game
(c) To reveal the secret (d) To play the game badly

22. To cool one’s heels


(a) To close the chapter (b) To walk on the heels
(c) To kick someone with the heels (d) To wait and rest for some time

23. To bury the hatchet


(a) To fight with the hatchet (b) To forget the enmity
(c) To bury the treasure under ground (d) To pick up enmity

24. Gift of the gab


(a) Gift for hard work (b) Gift undeserved
(c) Gift of being a good conversationalist (d) Gift from unknown person

25. To smell a rat


(a) To suspect a trick (b) To detect a foul smell
(c) To behave like a rat (d) To trust blindly

PART — E

Instructions: (Questions 26-35), Given below are sentences with a blank in each sentence. Choose
the right answer to fill in the blank by shading your answer in the space provided for it on the
OMR Answer Sheet.

Marks : Each question carries 1 (One) mark. (Total 10 marks)

26. Some people believe that in emotional maturity men are inferior ____________ women.
(a) Than (b) To (c) From (d) Against

27. My father was annoyed ____________ me.


(a) Towards (b) Against (c) With (d) Upon

28. Some orthodox persons are averse ____________ drinking liquor.


(a) Against (b) For (c) Towards (d) To

29. The Cinema Hall was on fire and the Cinema owner had to send ____________ the Fire Brigade.
(a) For (b) Through (c) Off (d) In

30. He was not listening ____________ I was saying.


(a) That (b) Which (c) To what (d) What

Previous Years
CLAT & AILET Papers Page 33
31. Drinking country liquor at marriage is a custom ____________ certain tribes.
(a) In (b) Among (c) Between (d) With

32. The struggle for justice brings ____________ the best of moral qualities of men.
(a) Forward (b) About (c) In (d) Out

33. If he ____________ a horse he would fly.


(a) Was (b) Were (c) Is (d) Goes

34. Mohan has a bad habit of ____________ at on odd hour.


(a) Turning up (b) Turning in (c) Turning over (d) Turning Off

35. He must refrain ____________ immoral conducts.


(a) Off (b) Through (c) From (d) Against

PART — F

Instructions: (Questions 36-40), The constituent sentences of a passage have been jumbled up.
Read jumbled sentences carefully and then choose the option which shows the best sequence
of sentences of the passage and shade the appropriate answer in the space provided for it on the
OMR Answer Sheet.

Marks: Each question carries 1 (One) mark. (Total 5 marks)

36. (i) The Collector said that the Dams should receive
(ii) To ensure uninterrupted
(iii) Water up to a particular level
(iv) Supply of water for irrigation
The best sequence is:
(a) ii, i, iv, iii (b) i, iii, ii, iv (c) iv, i, iii, ii (d) ii, iv, i, iii

37. (i) He loved to distribute them among small kids.


(ii) He wore a long, loose shirt with many pockets.
(iii) And in doing so his eyes brightened.
(iv) The pockets of his shirt bulged with toffees and chocolates.
The best sequence is:
(a) ii, i, iii, iv (b) i, iv, ii, iii (c) iv, i, iii, ii (d) ii, iv, i, iii

38. (i) As we all know, a legislation


(ii) Needs the assent of the President
(iii) Passed by the Houses of Parliament
(iv) To become law.
The best sequence is:
(a) i, iii, ii, iv (b) i, iv, ii, iii (c) iv, i, iii, ii (d) ii, iv, i, iii

Previous Years
Page 34 CLAT & AILET Papers
39. (i) The farmers grow food for the whole country.
(ii) And therefore it is our duty to improve their lot.
(iii) Yet these fellows are exploited by the rich.
(iv) Hence they are the most useful members of the society.

The best sequence is:


(a) ii, i, iv, iii (b) i, iv, ii, iii (c) i, iv, iii, ii (d) ii, iv, i, iii

40. (i) The ripples looked enchanting in the light of the Sun.
(ii) We went to the pond.
(iii) We flung stones to create ripples.
(iv) We stood knee-deep in the muddy water of the pond.

The best sequence is:


(a) ii, i, iv, iii (b) ii, iv, iii, i (c) iv, i, iii, ii (d) iv, ii, i, iii

SECTION – II

General Knowledge/ Current Affairs

Instructions: (Questions 41-90), Out of the four answers, shade the correct answer in the space
provided for it on the OMR Answer Sheet.

Marks: Each question carries 1 (one) mark (Total 50 marks)

41. Capital market means


(a) Mutual Funds (b) Money Market (c) Securities Market (d) Banking Business

42. From which river would the National River Project be started?
(a) Yamuna (b) Gomti (c) Ganga (d) Krishna

43. “The Audacity of Hope” is a book written by


(a) Bill Clinton (b) Barack Obama (c) Gorge Bush (d) Bill Gates

44. ‘WPI’ is used as an acronym for


(a) World Price Index (b) World Price Indicators
(c) Wholesale Price Index (d) Wholesale Price Indicators

45. If the tax rate increases with the higher level of income, it shall be called
(a) Progressive Tax (b) Proportional Tax (c) Lump sum Tax (d) Regressive Tax

46. Who is the Director of “Chak De India”?


(a) Shimit Amiro (b) Yash Chopra (c) Shahrukh Khan (d) Ram Gopal Verma

47. What is the full form of the scanning technique CAT?


(a) Complete Anatomical Trepanning (b) Computerized Automatic Therapy
(c) Computerized Axial Tomography (d) Complete Axial Transmission

Previous Years
CLAT & AILET Papers Page 35
48. Who got the world Food Prize?
(a) Kofi Annan (b) Man Mohan Singh (c) Hillary Clinton (d) Bhumibol Adulyadej

49. ISO 9000 is a


(a) Quality Standard Mark (b) Space Project
(c) Trade Technique (d) Loan Security

50. What is ‘AGMARK’?


(a) Name of Brand
(b) A Marketing Research Organisation
(c) Eggs supplied by Government-run cooperative
(d) Agriculture marketing for agro products

51. The Headquarters of Indian Space Research Organisation is at


(a) Trivandrum (b) New Delhi (c) Bangalore (d) Ahmedabad
52. “Saras” is the name of
(a) An Aircraft (b) A Tank (c) A Missile (d) A Submarine

53. First woman Prime-Minister in the World was from


(a) Sri Lanka (b) Bhutan (c) India (d) Nepal

54. Who was felicitated with ‘Nishan-e-Pakistan’?


(a) Shatrughan Sinha (b) Maulana Azad (c) Dilip Kumar (d) Khan Abdul Gaffar Khan

55. Which State provided separate reservation for Muslims and Christians in the State Backward Classes
List in 2007?
(a) Andhra Prades (b) Tamil Nadu (c) Bihar (d) Kerala

56. Which of the following dances is not a classical dance?


(a) Kathakali (b) Garba (c) Odissi (d) Manipuri

57. Sulabh International is an organisation which provides


(a) Health Services in Rural Areas (b) Good Sanitation at Cheap Rates
(c) Low Cost Accommodation (d) Low Cost Credit

58. Who among the following was honoured with ‘Officer of the Legion of Honour’ award by French
Government in July 2008?
(a) Dev Anand (b) Yash Chopra (c) B.R. Chopra (d) Mrinal Sen

59. The largest gland in the human body is


(a) Liver (b) Pancreas (c) Thyroid (d) Endocrine

60. The Railway Budget for 2007-2008 has declared the year 2007 as the year of
(a) Cleanliness (b) Passenger Comfort
(c) Staff Welfare (d) Computerization of Reservations

61. In the world of Hindi Cinema, who was affectionately called as ‘Dada Moni’?
(a) Balraj Sahni (b) Prithviraj Kapoor (c) Ashok Kumar (d) Utpal Dutt

Previous Years
Page 36 CLAT & AILET Papers
62. Out standing Parliamentarian Award (2006) was presented in 2007 to
(a) P. Chidambram (b) Sarad Pawar (c) Mani Shankar Aiyar (d) Sushma Swaraj

63. World’s longest sea bridge has taken shape in to 2007 in


(a) China (b) Japan (c) Singapore (d) U.S.A.

64. Who among the following Indians became Citigroup’s Investment Banking head?
(a) L. N. Mittal (b) A.K. Subramaniyam
(c) Vineet Seth (d) Vikram Pandit

65. The Green Revolution in India has been identified with


(a) Dr. Man Mohan Singh (b) Dr. Montek Singh Ahluwalia
(c) Mr. Rajendra Singh ‘waterman’ (d) Dr. M.S. Swaminathan

66. ndian Judge in the UN Law of the Sea Tribunal is


(a) Dr. P.S. Rao (b) Dr. P.C. Rao
(c) Mr. Justice Jagannath Rao (d) Mr. Justice Rajendra Babu

67. The Ozone Layer thins down as a result of a chain chemical reaction that separates from the layer
(a) Oxygen (b) Chlorine (c) Nitrogen (d) Hydrogen

68. Joint SAARC University of eight SAARC Nations has been established in
(a) Colombo (b) Dhaka (c) Kathmandu (d) New Delhi

69. Which country recently produced the world’s first cloned rabbit using a biological process that
takes cells from a fetus?
(a) U.K. (b) China (c) U.S.A. (d) Germany

70. Which one of the following Vitamins is responsible for blood clotting?
(a) Vitamin A (b) Vitamin E (c) Vitamin C (d) Vitamin K

71. What is the meaning of ‘Gilt Edged Market’?


(a) Market in Government securities (b) Market of smuggled goods
(c) Market of auctioned goods (d) Market of Gold products

72. Who is the Central Chief Information Commissioner of India?


(a) Prof. Ansari (b) Mrs. Padma Subramanian
(c) Mr. Wajahat Habibullah (d) Dr. O.P. Kejariwal

73. The youngest recipient of Padma Shri so far is


(a) Sachin Tendulkar (b) Shobana Chandrakumar
(c) Sania Mirza (d) Billy Arjan Singh

74. Who is the Director of the film “Elizabeth : The Golden Age Cast”?
(a) Rama Nand Sagar (b) Ram Gopal Verma
(c) Karan Johar (d) Shekhar Kapur

Previous Years
CLAT & AILET Papers Page 37
75. The territorial waters of India extend up to?
(a) 12 Nautical Miles (b) 6 Km (c) 10 Nautical Miles (d) 15 Nautical Miles

76. ‘Samjhouta Express’ runs between the Railway Stations of


(a) New Delhi - Wagah (b) New Delhi - Lahore
(c) Amritsar - Lahore (d) New Delhi – Islamabad

77. Blue revolution refers to


(a) Forest Development (b) Fishing
(c) Poultry Farming (d) Horticulture

78. Dr. A.P.J. Abdul Kalam has been appointed as Chancellor of


(a) IIM Mumbai (b) IIT Kanpur
(c) IIM Ahmedabad (d) IIST Thiruvananthapuram

79. In which State “Kanya Vidyadhan Yojna’ is operational?


(a) Andhra Pradesh (b) Uttar Pradesh (c) Rajasthan (d) Haryana

80. Who emerged the fastest woman of the world at Beijing Olympics?
(a) Sheron Sumpson (b) Kerron Stewart (c) Ann Fraster (d) Elina Basiena

81. Savannath grasslands are found in


(a) North America (b) Africa (c) Australia (d) East Asia

82. Which State has launched the “Aarogya Sri” a health Insurance Scheme for families below poverty
line?
(a) Andhra Pradesh (b) Uttar Pradesh (c) Maharasthra (d) Kerala

83. The first nuclear reactor of India is named


(a) Rohini (b) Vaishali (c) Apsara (d) Kamini

84. In May 2007 Air Sahara acquired by Jet Airways is being operated as a separate airline under the
name of
(a) Jet Lite (b) Jet Sahara (c) Air Jet Line (d) Jet Sahara Lite

85. Suez Canal connects


(a) Mediterranean Sea and Red Sea (b) Mediterranean Sea and Black Sea
(c) Baltic Sea and Red Sea (d) Baltic Sea and Black Sea

86. Government has launched E – Passport Scheme and first E – Passport was issued to
(a) Mr. Arjun Singh (b) Mrs. Sonia Gandhi
(c) Dr. Man Mohan Singh (d) Mrs. Pratibha Patil

87. The Hindu outfit ‘Hindraf’ has been banned in


(a) Pakistan (b) Thailand (c) Malaysia (d) Bangladesh

Previous Years
Page 38 CLAT & AILET Papers
88. Which organization is headed by Indian Environmentalists R.K. Pachauri, a Nobel Laureate?
(a) International Environment Panel (b) International Panel on Climate Change
(c) International Pollution Control Panel (d) International Panel on Global Warming

89. Kandhamal, the worst affected town by sectarian violence in September-October 2008 is situated in
(a) Orissa (b) Gujarat (c) Andhra Pradesh (d) Karnataka

90. World “No Tobacco Day” is observed on


(a) January 10 (b) June 1 (c) May 31 (d) March 5

SECTION – III

Elementary Mathematics (Numerical Ability)

Instructions: (Questions 91-110), From the four answers given, shade the appropriate answer in
the space provided for it on the OMR Answer Sheet.

Marks: Each question carries 1 (one) mark (Total 20 marks)

91. The average monthly income of a person in a certain family of 5 members is Rs. 1000. What will be
monthly average income of one person in the same family if the income of one person increased by
Rs. 12,000/- per year?
(a) Rs. 1200/- (b) Rs. 1600/- (c) Rs. 2000/- (d) Rs. 3400/-

92. A dishonest shopkeeper uses a weight of 800 gm for a kg and professes to sell his good at cost
price. His profit is
(a) 20% (b) 21% (c) 24% (d) 25%

93. By selling 11 oranges for a rupee, a man loses 10%. How many oranges for a rupee should he sell
to gain 10%?
(a) 9 (b) 10 (c) 8 (d) 5

94. A person takes 3 hours to walk a certain distance and riding back. He could walk both ways in 5
hours. How long could it take to ride both ways.
(a) 1.5 hr (b) 1 hr (c) 0.5 hr (d) 2 hrs

95. Change 1/8 into percentage


(a) 12.5% (b) 15% (c) 8% (d) 25%

96. 12.5% of 80 is equal to


(a) 8 (b) 20 (c) 10 (d) 40

97. Which number should fill the blank space to complete the series: 1, 2, 4, 5, 7, 8, 10, 11…
(a) 12 (b) 13 (c) 14 (d) 15

98. The smallest of the fractions given below


(a) 9/10 (b) 11/12 (c) 23/28 (d) 32/33

Previous Years
CLAT & AILET Papers Page 39
99. Three friends shared the cost of a television. If Amit, Bharat and Dinesh each paid Rs. 3000 and Rs.
1800 respectively, then Dinesh paid what percent of the total cost?
(a) 10% (b) 20% (c) 30% (d) 40%

100. The average age of 29 boys of a class is equal to 14 years. When the age of the class teacher is
included the average becomes 15 years. Find the age of the class teacher.
(a) 44 years (b) 40 years (c) 52 years (d) 66 years

101. It takes 8 people working at equal rates to finish a work in 96 days. How long will 6 workers take for
the same work?
(a) 92 days (b) 128 days (c) 111 days (d) 84 days

102. Ram’s income is 20% less than Shyam’s. How much is Shyam’s income more than Ram’s in
percentage terms?
(a) 20% (b) 30% (c) 25% (d) 15%

103. The monthly salary of A, B and C are in the ratio 2 : 3 : 5. If C’s monthly salary is Rs. 1,200 more
than that of A, find B’s annual salary.
(a) Rs. 2000 (b) Rs. 1000 (c) Rs. 1500 (d) Rs. 1200

104. In a town there are 94500 people. 2/9 of them are foreigners, 6400 are immigrants and the rest are
natives. How many are natives?
(a) 67100 (b) 27400 (c) 77600 (d) 88100

105. Total salary of three persons A, B and C is Rs. 1,44,000. They spend 80%, 85% and 75% respectively.
If their savings are in the ratio 8 : 9 : 20, find C’s salary.
(a) 48000 (b) 64000 (c) 40000 (d) 32000

106. The population of a town is 155625. For every 1000 males, there are 1075 females. If 40% of the
males and 24% of the females are literate, find the literacy percentage in the town.
(a) 33.7% (b) 32.7% (c) 31.7% (d) 30.7%

107. 10 sheep and 5 pigs were brought for Rs. 6,000. If the average price of a sheep is Rs. 450, find the
average price of a pig.
(a) Rs. 380 (b) Rs. 410 (c) Rs. 340 (d) Rs. 300

108. Ram weighs 25 kg more than Shyam. Their combined weight is 325 kg. How much does Shyam
weigh?
(a) 150 kg (b) 200 kg (c) 125 kg (d) 160 kg

109. Find out the wrong number in the series: 3,8,15,24,34,48,63


(a) 24 (b) 34 (c) 15 (d) 63

110. What is the location value of 7 in the figure 9872590?


(a) 72590 (b) 7 (c) 70000 (d) 7000

Previous Years
Page 40 CLAT & AILET Papers
SECTION – IV

Legal Aptitude

Instructions: (Questions 111-155), From the four options given, shade the appropriate correct option
in the space provided for it on the OMR Answer Sheet.

Marks: Each question carries 1 (one) mark (Total 45 marks)

111. Which is the oldest Code of Law in India?


(a) Naradasmriti (b) Manusmriti (c) Vedasmriti (d) Prasarsmriti

112. Private international law is also called…….


(a) Civil Law (b) local laws (c) Conflict of laws (d) Common law

113. A nominal sum given as a token for striking a sale is called


(a) Earnest money (b) Advance (c) Interest (d) Solatium

114. Joint heirs to a property are called


(a) Co-heirs (b) Coparceners (c) Successors (d) Joint owners

115. The right of a party to initiate an action and be heard before a Court of law is called
(a) Right in rem (b) Right in personam (c) Fundamental right (d) Locus standi

116. Indian Parliament is based on the principle of Bicameralism


(a) Universal (b) Adult Franchise (c) Dyarchy (d) Federalismralism

117. The Supreme Court held that evidence can be recorded by video-conferencing in the case…
(a) State of Maharashtra v. Prafull B.Desai (b) Paramjit Kaur v. State of Punjab
(c) Pappu Yadav v. State of Bihar (d) Bachan Singh v. State of Punjab

118. When the master is held liable for the wrongful of his servant, the liability is called
(a) Strict liability (b) Vicarious liability (c) Tortous liability (d) Absolute liability

119. The act of unlawfully entering into another’s property constitutes


(a)Trespass (b) Restraint (c) Appropriation (d) Encroachment

120. Which Parliamentary Committee in India system of democracy is chaired by a member of Opposition
Party?
(a) Estimates Committee (b) Joint Parliamentary Committee
(c) Public Accounts Committee (d) Finance Committee

121. Supreme Court held that Preamble as a basic feature of Constitution cannot be amended in the
case of
(a) Golknath v. State of Punjab (b) Maneka Gandhi v. Union of India
(c) S. R. Bommai v. Union of India (d) Kesavananda Bharti V. State of Kerala

Previous Years
CLAT & AILET Papers Page 41
122. In the year 2002 the Competition Act was enacted replacing
(a) Trade Marks Act (b) Copy Right Act
(c) Contract Act (d) MRTP Act

123. A right to recover time barred debt is


(a) Universal right (b) Perfect right
(c) Imperfect right (d) Fundamental right

124. The law relating to prisoners of war has been codified by


(a) Geneva Convention (b) Vienna Convention
(c) Paris Convention (d) None of the above

125. Public holiday are declared under


(a) Criminal Procedure Code (b) Civil Procedure Code
(c) Constitution of India (d) Negotiable Instruments Act

126. When a person is prosecuted for committing a criminal offence, the burden of proof is on
(a) Accused (b) Prosecution (c) Police (d) Complainant

127. Offence which can be compromised between the parties is known as


(a) Conjugal right (b) Cognizable offence
(c) Compoundable offence (d) Complainant

128. Husband and wife have a right to each others company. The right is called
(a) Conjugal right (b) Human right
(c) Civil right (d) Fundamental right

129. A person ‘dying intestate’ means he


(a) Died without legal heirs (b) Died without making a will
(c) Died without any property (d) Died without a son

130. If a witness makes a statement in Court, Knowing it to be false, he commits the offence of
(a) Forgery (b) Falsehood (c) Perjury (d) Breach of trust

131. A child born after father’s death is


(a) Posthumous (b) Heir (c) Intestate (d) Bastard

132. A formal instrument by which one person empowers another to represent him is known as
(a) Affidavit (b) Power of attorney (c) Will (d) Declaration

133. The temporary release of a prisoner is called


(a) Parole (b) Amnesty (c) Discharge (d) Pardon

134. The offence of inciting disaffection, hatred or contempt against Government is


(a) Perjury (b) Forgery (c) Sedition (d) Revolt

135. India became the member of United Nations in the Year


(a) 1956 (b) 1945 (c) 1946 (d) 1950

Previous Years
Page 42 CLAT & AILET Papers
136. A party to the suit is called
(a) Accused (b) Plaintiff (c) Litigant (d) Complainant

137. Who heads the four member committee appointed to study the Centre-State relations especially
the changes took place since Sarkaria Commission
(a) Justice M.M.Punchi (b) Justice Nanavati
(c) Justice Barucha (d) Justice Kuldip Singh

138. No one can be convicted twice for the same offence. This doctrine is called
(a) Burden of proof (b) Double conviction
(c) Double jeopardy (d) Corpus delicti

139. A participant in commission of crime is popularly known as


(a) Respondent (b) Under-trial (c) Defendant (d) Accomplice

140. Which of the following is not payable to Central Government?


(a) Land revenue (b) Customs duty (c) Income tax (d) Wealth tax

141. Where is the National Judicial Academy located?


(a) Kolkata (b) Bhopal (c) Delhi (d) Mumbai

142. Who have constitutional right to audience in all India Courts?


(a) President (b) Chief Justice of India
(c) Attorney General (d) Solicitor General

143. Which of the following is not included in the Preamble to the Constitution?
(a) Morality (b) Justice (c) Sovereign (d) Socialist

144. ‘Court of Record’ is a Court which?


(a) Maintains records (b) Preserves all its records
(c) Can Punish for its contempt (d) Is competent to issue writs

145. A judge pf the Supreme Court can be removed from office only on grounds of
(a) Gross inefficiency (b) Delivering wrong judgments
(c) Senility (d) Proven misbehaviour or incapacity

146. Fiduciary relationship means a relationship based on


(a) Trust (b) Money (c) Senility (d) Blood relation

147. The Chairman of judicial review has been borrowed from the Constitution of
(a) Justice Kripal (b) Justice S.N.Phukan
(c) Justice Saharia (d) Justice Liberhan

148. The concept of judicial review has been borrowed from the Constitution of
(a) U.S.S.R (b) U.K (c) U.S.A (d) Switzerland

149. Every duty enforceable by law is called


(a) Accountability (b) Obligation (c) Burden (d) Incidence

Previous Years
CLAT & AILET Papers Page 43
150. The killing of a new born child by its parents is
(a) Malfeasance (b) Infanticide (c) Abortion (d) Foeticide

151. Offence of breaking a divine idol is


(a) Salus populi (b) Infanticide (c) Sacrilege (d) Blasphemy

152. A person who goes under-ground or evades the jurisdiction of the Court is known as
(a) Offender (b) Under-ground (c) Absentee (d) Absconder

153. What is a caveat?


(a) A warning (b) An injunction (c) Writ (d) Certiorari

154. Muslim religious foundations are known as


(a) Din (b) Wakfs (c) Ulema (d) Quzat

155. Beyond what distance from the coast, is the sea known as “High Sea”?
(a) 20 miles (b) 300 miles (c) 200 kms. (d) 12 miles

SECTION – V

Logical Reasoning

Marks: Each question carries 1 (one) mark (Total 45 marks)

Instructions: (questions 156-165), Each question below consists of one Assertion (A) and one Reason
(R). Examine them and shade the correct answers using the Code below on the OMR Answer
Sheet.

Code:
(a) Both A and R are individually true and R is the correct explanation of A.
(b) Both A and R are individually true but R is not the correct explanation of A.
(c) A is true but R is false.
(d) A is false but R is true.

156. A: Area along the Equator records the highest temperature throughout the year.
R: On the equator, days and nights are equal for the largest part of the year.

157. A: Commercial fisheries have not developed in tropics.


R: The demand for marine food from low income population is low in the tropics.

158 A: Lightning, thunder and heavy rain accompany volcanic activity.


R: Volcanoes throw water vapour and charged particles into the atmosphere.

159. A: Soils in some parts of Punjab, Haryana and Rajasthan are saline.
R: Evaporation exceeds precipitation.

Previous Years
Page 44 CLAT & AILET Papers
160. A: The monsoons arrive suddenly in India in June.
R: The monsoonal low pressure trough is well-developed over India.

161. A: India built dams and reservoirs to manage water resources.


R: India had enough experience in canals.

162. A: The life expectancy in European countries is very high.


R: European countries have low mortality rate.

163. A: The nomadic herders slaughter their animals for meat.


R: Animals form the chief source of food and livelihood for nomadic herders.

164. A: Exploitation of equatorial rain forest of Amazon basin is not easy.


R: This region is very rich in several types of deadly animals and insects.

165. A: The Sea remains free from ice from British Columbia to Bering Sea.
R: Air moving off the comparatively warm waters of North Pacific Drift gives the coastal areas of
British Columbia a warmer climate.

Instructions: (questions 166-175), In each question below, one statement and two assumptions I
and II are given. Examine the statements and shade the correct assumption which is implicit in
the statement on the OMR Answer Sheet using the following Code.

Code:
(a) If only assumption I is implicit
(b) If only assumption II is implicit.
(c) If either I or II is implicit.
(d) If neither I nor II is implicit.

166. Statement:
The patient’s condition would improve after operation.

Assumptions:
I. The patient can be operated upon in this condition.
II. The patient cannot be operated upon in this condition.

167. Statement:
Detergents should be used to clean clothes.
Assumptions:
I. Detergent forms more lather.
II. Detergent helps to dislodge grease and dirt.

168. Statement:.
“As there is a great demand, every person seeking ticket of the programme will be given only five
tickets”.
Assumptions:
I. The organizers are not keen on selling the tickets.
II. No one is interested in getting more than five tickets.

Previous Years
CLAT & AILET Papers Page 45
169. Statement:
Double your money in five months-An advertisement.
Assumptions:
I. The assurance is not genuine.
II. People want their money to grow.

170. Statement:
Films have become indispensable for the entertainment of people.
Assumptions:
I. Films are the only media of entertainment.
II. People enjoy films.

171. Statement:.
“To keep myself up-to-date, I always listen to 9.00 p.m. news on radio”. – A candidate tells the
interview board.
Assumptions:
I. The candidate does not read newspaper.
II. Recent news are broadcast only on radio.

172. Statement:
Never before such a lucid book was available on the topic.
Assumptions:
I. Some other books were available on this topic.
II. You can write lucid books on very few topics.

173. Statement:
In case of any difficulty about this case, you may contact our company’s lawyer.
Assumptions:
I. Each company has a lawyer of its own.
II. The company’s lawyer is thoroughly briefed about this case.

174. Statement: “Present day education is in shambles and the country is going to the dogs”.
Assumptions:
I. A good education system is essential for the well being of a nation.
II. A good education alone is sufficient for the well being of a nation.

175. Statement:
Children are influenced more by their teachers nowadays.
Assumptions:
I. The children consider teachers as their models.
II. A large amount of children’s time is spent in school.

Instructions: (questions 176-180), Each question below contains a Statement on relationship and
a question regarding relationship based on the Statement. Shade the correct option on relationship
on the OMR Answer Sheet.

176. Pointing to a photograph, a lady tells Ram, “I am the only daughter of this lady and her son is your
maternal uncle”. How is the speaker related to Ram’s father?
(a) Sister-in-law (b) Wife (c) Either (a) or (b) (d) Neither (a) nor (b)

Previous Years
Page 46 CLAT & AILET Papers
177. Introducing a man, a woman said, “He is the only son of my mother. How is the woman related to the
man?
(a) Mother (b) Aunt (c) Sister (d) Niece

178. Shyam said, “ This girl is the wife of the grandson of my mother”. Who is Shyam to the girl?
(a) Father (b) Grandfather (c) Husband (d) Father-in-law

179. Pointing to a man on the stage, Sunita said, “He is the brother of the daughter of the wife of my
husband”. How is the man on the stage related to Sunita?
(a) Son (b) Husband (c) Cousin (d) Nephew

180. Introducing a man to her husband, a woman said, “His brother’s father is the only son of my
grandfather”. How is the woman related to this man?
(a) Mother (b) Aunt (c) Sister (d) Daughter

Instruction: (question 181-190), In each question below two words are paired which have certain
relationships. After the double colon (::), another word is given and shade the correct option on
the OMR Answer Sheet which pairs with this word taking into account the pair already given.

181. Legislation : Enactment :: Executive :?


(a) Minister (b) Officer (c) Implementation (d) Leader

182. UP : Uttranchal :: Bihar :?


(a) Jharkhand (b) Chhatisgarh (c) Madhya Pradesh (d) Manipur

183. Gold : Silver :: Cotton :?


(a) Yarn (b) Silk (c) Fibre (d) Synthetic

184. Botany : Flora ::Zoology :?


(a) Fauna (b) Biology (c) Fossils (d) Pathology

185. Cold wave : Winter :: Loo:?


(a) Humidity (b) Frostbite (c) Summer (d) Storm

186. King : Royal :: Saint :?


(a) Religious (b) Red (c) Priesthood (d) Blue

187. Sculptor : Statue :: Poet:?


(a) Painter (b) Singer (c) Poem (d) Writer

188. Laugh : Happy :: Cry :?


(a) Sad (b) Bickering (c) Frown (d) Complain

189. Black : Absence :: White :?


(a) Red (b) Presence (c) Rainbow (d) Crystal

190. Governor : President :: Chief-Minister :?


(a) Commissioner (b) Attorney General (c) Justice (d) Prime-Minister

Previous Years
CLAT & AILET Papers Page 47
Instruction : (question 191-195), Each question below contains a Statement and two Courses of
Action I and II. Assuming the statement to be true, decide which of the two suggested Courses of
Action logically follows and shade on the OMR Answer Sheet, using the Code given below.

Code:
(a) If only I follows.
(b) If only II follows.
(c) If either I or II follows.
(d) If neither I nor II follows.

191. Statement:
One of the problems facing the food processing industry is the irregular supply of raw material. The
producers of raw materials are not getting a reasonable price.

Courses of Action :
I. The government should regulate the supply of raw material to other industries also.
II. The government should announce an attractive package to ensure regular supply of raw material
for food processing industry.

192. Statement:
The Officer In-charge of a Company had a hunch that some money was missing from the safe.

Courses of Action :
I. He should get it recounted with the help of the staff and check it with the balance sheet.
II. He should inform the police.

193. Statement:
If the retired Professors of the same Institutes are also invited to deliberate on restructuring of the
organisation, their contribution may be beneficial to the Institute.

Courses of Action:
I. Management may seek opinion of the employees before calling retired Professors.
II. Management should involve experienced people for the systematic restructuring of the
organisation.

194. Statement:
The sale of a particular product has gone down considerably causing great concern to the company.

Courses of Action:
I. The company should make a proper study of rival products in the market.
II. The price of the product should be reduced and quality improved.

195. Statement:
Mr. X, an active member of the Union, often insults his superiors in the office with his rude behaviour.

Courses of Action:
I. He should be transferred to some other department.
II. The matter should be referred to the Union.

Previous Years
Page 48 CLAT & AILET Papers
Instructions: (questions 196-200), Each question below contains a Statement and two Arguments
I and II. Assume the statement to be true, shade the Argument which is strong on the OMR
Answer Sheet using the Code below.

Code:
(a) If only argument I is strong.
(b) If only argument II is strong.
(c) If either argument I or II is strong.
(d) If neither argument I nor II is strong.

196. Statement:
Should a total ban be put on trapping wild animals?

Arguments:
I. Yes. Trappers are making a lot of money.
II. No. Bans on hunting and trapping are not effective.

197. Statement: Should school education be made free in India?

Arguments:
I. Yes. This is the only way to improve the level of literacy.
II. No. It would add to the already heavy burden on the exchequer.

198. Statement: Should government jobs in rural areas have more incentives?

Arguments:
I. Yes. Incentives are essential for attracting government servants there.
II. No. Rural areas are already cheaper, healthier and less complex than big. So, why offer extra
incentives!

199. Statement: Should luxury hotels be banned in India?

Arguments:
I. Yes. They are places from where international criminals operate.
II. No. Affluent foreign tourists will have no place to stay.

200. Statement: Should the political parties be banned?

Arguments:
I. Yes. It is necessary to teach a lesson to the politicians.
II. No. It will lead to an end of democracy.

Previous Years
CLAT & AILET Papers Page 49
CLAT Question Paper 2010

This Booklet contains 200 objective questions. Each question carries one mark.
Every question contains four choices of answers. Select the most appropriate answer and shade
the corresponding oval in the O.M.R. sheet with HB pencil only. Answer all the questions.

ENGLISH

The constituent phrases of a sentence are jumbled up in question no. 1-3. Select the most
appropriate sequence to make the sentence meaningful.

1. (i) built on the site of a church destroyed


(ii) in the hilly area of the city is the famous Shandon Steeple
(iii) the bell tower of St. Anne’s Church
(iv) when the city was besieged by the Duke of Marlborough

(a) ii, iii, i, iv (b) ii, i, iii, iv (c) iv, iii, i, ii (d) iii, ii, i, iv

2. (i) no law giving effect to the policy of the state towards securing all or any of the principles laid
in part IV
(ii) notwithstanding anything contained in Article 13
(iii) and no law containing a declaration that it is for giving effect to such policy shall be called in
question in any court on the ground that it does not give effect to such policy
(iv) shall be deemed to be void on the ground that it is inconsistent with or takes away or
abridges any of the rights conferred by Article 14 or 19

(a) ii, i, iii, iv (b) iv, i, ii, iii (c) ii, i, iv, iii (d) i, ii, iii, iv

3. (i) neither House shall proceed further with the Bill,


(ii) if he does so, the houses shall meet accordingly
(iii) but the President may at any time after the date of his notification summon the Houses to
meet in a joint sitting for the purpose specified in the notification and,
(iv) where the President has under clause (1) notified his intention of summoning the Houses to
meet in a joint sitting,

(a) iv, i , iii, ii (b) iv, i , ii, iii (c) iv, ii, iii, i (d) i, ii, iii, iv

Substitute the underlined phrases with any of the given choices to express the opposite meaning
In the sentences in questions number 4-6

4. She always praises everything I say


(a) picks holes in (b) dislikes (c) rebukes (d) picks holes to

5. He often says how wonderful his school is?


(a) says he is unworthy (b) appreciates (c) runs up (d) runs down

Previous Years
Page 50 CLAT & AILET Papers
6. She said I was the best boss they’d ever had. It was obvious she was praising me sincerely.
(a) not appreciating me (b) befooling me
(c) buttering me up (d) disliking me

Identify the part of speech of the underlined words in the given sentences from Questions number
7 to 9
7. I must perfect the operation to make the perfect robot.
(a) verb (b) adverb (c) adjective (d) noun

8. A kindly person is one who behaves kindly.


(a) noun (b) preposition (c) adverb (d) verb

9. He is not normally a very fast runner, but he runs fast in major events.
(a) adverb (b) adjective (c) verb (d) noun

Read the following passage carefully and answer question numbers 10 to 19


Antigone was one of the daughters of Oedipus, that tragic figure of male power who had been cursed by
Gods for mistakenly killing his father and subsequently marrying his mother and assuming the throne of
Thebes. After the death of Oedipus civil war broke out and a battle was waged in front of the seventh gate
of Thebes- his two sons led opposing factions and at the height of the battle fought and killed each other.
Oedipus’ brother, Creon, uncle of Antigone, was now undisputed master of the city. Creon resolved to make
an example of the brother who had fought against him, Polynices, by refusing the right of honourable burial.
The penalty of death was promulgated against any who should defy this order.
Antigone was distraught. Polynices had been left unburied, unwept, a feast of flesh for keen eyed carrion
birds. Antigone asks her sister Ismene, for it was a challenge to her royal blood. “Now it is time to show
weather or not you are worthy of your royal blood. Is he not my brother and yours? Whether you like it or
not? I shall never desert him-never!” But Ismene responds, “How could you dare- when Creon has expressly
forbidden it? Antigone, we are women, it is not for us to fight against men”. With a touch of bitterness,
Antigone releases her sister from the obligation to help her, but argues she cannot shrug off the burden. “If
I die for it what happiness! Live, if you will live, and defy the holiest of laws of heaven.”
10. What is the main theme of the story of Antigone?
(a) One must be truthful and honest
(b) There is a conflict between the laws of men and heavenly laws
(c) One must be true to one’s kins
(d) War is an evil

11. Why did Antigone decide to defy the orders of Creon?


(a) She loved her brother
(b) She was to give an honorable burial to her brother
(c) She felt she was bound by her heavenly obligation
(d) To teach Creon a lesson

12. What, in your opinion, would have been the logical end of the story?
(a) Antigone might have agreed with her sister and refrained from giving a burial to Polynices
(b) Antigone might have been allowed by Creon to give a decent burial to her brother
(c) Antigone might have defied the order of Creon but forgiven by him
(d) Antigone might have been executed for defying the order of the king

Previous Years
CLAT & AILET Papers Page 51
13. What was the status of women in the contemporary society? They
(a) were liberated
(b) could have taken their own decisions
(c) considered themselves inferior and subordinate to men
(d) claimed equality with men

14. Why did a civil war break out in Thebes? The war broke out because
(a) of the curse of the Gods
(b) the brothers of Antigone were greedy
(c) there was a fight among sons of Oedipus for the inheritance of the kingdom
(d) there was a conflict between a son of Oedipus and Creon

15. A carrion bird is a bird


(a) of prey (b) which eats human flesh
(c) which eats dead bodies (d) which eats only grain

16. Why did Creon deny decent burial to Polynices? He did so because
(a) he did not love Polynices (b) Polynices fought against Creon
(c) Polynices was disobedient to Creon (d) Polynices did not show bravery

17. Why did Ismene not support Antigone? Ismene


(a) was weak and did not have the courage to defy orders of the powerful king
(b) did not consider it right to defy the king
(c) did not think it fit to defy her uncle especially after the death of her father
(d) did not believe that Polynices deserved better treatment

18. Why did the Gods curse Oedipus? Because Oedipus


(a) killed his father and married his mother
(b) killed his father
(c) married his mother
(d) committed an unknown sin

19. Does the story approve the principle of vicarious liability? If so how?
(a) No, it does not
(b) Yes, it does, because of the acts of Oedipus his children suffered
(c) Yes, it does, because his father was killed by Oedipus
(d) Yes, it does, because he married his mother

Select the meaning of the underlined idioms and phrases in sentences in questions 20 to 25

20. I have hit upon a good plan to get rid of him.


(a) found (b) chanced upon
(c) decided to beat him (d) borrowed

21. He is sticking out for better terms.


(a) threatens to take action (b) insists on using the force
(c) decides to give concessions (d) persists in demanding

22. He broke off in the middle of the story.


(a) failed (b) began crying (c) stopped suddenly (d) felt uneasy

Previous Years
Page 52 CLAT & AILET Papers
23. He refused to be led by the nose.
(a) to follow like an animal (b) to be treated as a fool
(c) to follow submissively (d) to be treated violently

24. The new cotton mill is mortgaged up to the eye.


(a) apparently (b) completely (c) deceptively (d) actually

25. When they embraced a new religion, it is safe to say they did it for loaves and fishes.
(a) selflessly (b) honest reasons (c) material benefits (d) because of fear

Choose the correct spelling out of four choices in questions no. 26 to 30


26. (a) Misogynists (b) Mysogynists (c) Mysoginists (d) Mysagynists

27. (a) Aracnophobia (b) Aranchophobia (c) Arochnophobia (d) Arachnophobia

28. (a) Cmamon (b) Cinnamon (c) Cinnaman (d) Cinaman

29. (a) Alcohol (b) Aichohol (c) Aichohal (d) Aichohel

30. (a) Bioclymatalogy (b) Bioclimatalogy (c) Bioclimatology (d) Bioclimatelogy

Select the correct meanings of the given words in question number 31 to 35


31. Lexicon
(a) number (b) legal document (c) dictionary (d) captain’s dog

32. Hex
(a) crude person (b) herb (c) parrot (d) evil spell

33. Seminary
(a) chapel (b) college (c) convocation hail (d) hostel

34. Litergy
(a) prayer (b) priest (c) ritual (d) church

35. Laity
(a) church members not baptized (b) church members baptized
(c) priests (d) church members who are not ordained priests

Fill in the blanks in questions 36 to 40:


36. Slavery was not done away ______ until the last century.
(a) with (b) for (c) to (d) off

37. Does he not take ______his father?


(a) before (b) for (c) after (d) like

38. We will have to take ______ more staff if we’re to take on more work.
(a) up (b) onto (c) into (d) on

Previous Years
CLAT & AILET Papers Page 53
39. Mother takes everything in her ______.
(a) steps (b) face (c) stride (d) work

40. Sale have really taken ______now.


(a) up (b) on (c) of (d) off

GENERAL KNOWLEDGE

41. Deodhar trophy is given for the game of


(a) Cricket (b) Football (c) Hockey (d) Golf

42. Periyar Wild-Life sanctuary is situated in the state of


(a) Tamilnadu (b) Karnataka (c) Kerala (d) Andhra-Pradesh

43. Nepanagar in Madhya Pradesh is known for


(a) steel mill (b) sugar mills (c) potteries (d) news print factory

44. Who destroyed the Somnath temple in Gujrat?


(a) Mohammed Ghouri (b) Mahmud Ghaznavi (c) Changhez Khan (d) Taimur Lang

45. In Cape Trafalgar, the famous battle of Trafalgar was fought in 1805. Where is Cape Trafalgar situated?
(a) Italy (b) Greece (c) Spain (d) Portugal

46. Lumbini is the place where the Buddha


(a) attained nirvana (b) attained enlightenment
(c) was born (d) was married

47. Galvanometer is an instrument to measure


(a) relative density of liquids (b) electric currents
(c) pressure of gases (d) distances

48. Who wrote ‘Mudra Rakshasa’?


(a) Vishakhadatta (b) Bana Bhatta (c) Kaliclasa (d) Jaya Dev

49. The venue of Asian Games in 1970 was


(a) Beijing (b) Jakarta (c) Bangkok (d) New Dethi

50. 1929 is known for


(a) visit of Simon Commission to India
(b) congress resolution for complete independence of India
(c) coming of Indian National Congress under the leadership of Mahatma Gandhi
(d) burma was made a part of India

51. I.N.S. Airavat is India’s


(a) amphibious ship (b) submarine (c) destroyer (d) frigate

52. Astra-Missile is
(a) surface to surface missile (b) surface to air missile
(c) air to surface missile (d) air to air missile

Previous Years
Page 54 CLAT & AILET Papers
53. Nehru Institute of Mountaineering is situated at
(a) Nainital (b) Darjeeling (c) Shimla (d) Uttarkashi

54. The chief guest on the 60th Republic day of India was the President of
(a) Uzbekistan (b) Tajikistan (c) Kazakastan (d) South-Africa

55. Which of the following was not the base of L.T.T.E. before being captured by Srilankan Forces?
(a) Elephant Pass (b) Kilinochchi (c) Mullaithivu (d) Murid ke

56. Who was given Col. C. K. Nayudu Life Time Achievement Award for 2007-2008 for exemplary
contribution to cricket
(a) Kapil Dev (b) Gundappa Vishwanath
(c) Sunil Gavaskar (d) Vijay Hazare

57. Who is the foreign minister of European Union?


(a) Baroness Ashlon (b) Massimo D’ Alema
(c) David Miliband (d) Carl Bildt

58. Who is the author of ‘Godan’?


(a) Bhishma Sahani (b) Premehand
(c) Manohar Shyam Joshi (d) Sharad Chandra Chattopadhyay

59. Which political party does Raj Babbar belong to?


(a) Samajwadi (b) Bharatiya Janata Party
(c) Congress (d) Bahujan Samaj Party

60. In which year Bastille fell on 14th of July?


(a) 1879 (b) 1789 (c) 1787 (d) none of them

61. Alberto Fujimori is the former president of


(a) Japan (b) Peru (c) South Korea (d) Vietnam

62. Which of the following is the national river of India?


(a) Brahmaputra (b) Narmada (c) Ganga (d) Kaveri

63. The highest number of telephone users are in


(a) China (b) U.S.A. (c) Canada (d) India

64. The president of Maldives is


(a) Maumoom Abdul Gayoom (b) Mohammad Ashraf
(c) Mahmood Alam (d) Mohammed Nasheed

65. Arabinda Rajkhowa is a member of


(a) B.J.P. (b) Assam Gana Parishad
(c) Maoist Centre (d) Ulfa

66. The breakthrough in the ideas of Darwin on the evolution of species came after his visit to a cluster
of islands and where he saw that each island supported its own form of finch. Name the islands?
(a) Ice land (b) Greenland (c) Galapagos (d) Christian Islands

Previous Years
CLAT & AILET Papers Page 55
67. Mahatma Gandhi never became a Nobel Laureate, but he was nominated five times. In which of the
following years he was not nominated?
(a) 1937 (b) 1939 (c) 1948 (d) 1940

68. Which of the following countries has not, till the end of 2009, decoded the entire genome of a human
being?
(a) India (b) Russia (c) China (d) Canada

69. When was the University of Bombay established?


(a) 1861 (b) 1857 (c) 1909 (d) 1890

70. Harare is the capital of


(a) Zambia (b) Zaire Republic (c) Zimbabwe (d) Yemen

71. Before the Indian team left for Conference on Climate Change at Copenhagen, Jairam Ramesh
announced that India would work for voluntary reduction of
(a) 40 to 45 percent
(b) 20 to 25 percent
(c) 30 to 35 percent
(d) 10 to 15 percent in energy intensity in 2020 compared to 2005

72. Which is the largest island in the world (if Australia is not considered an island)?
(a) Iceland (b) Borneo (c) Sumatra (d) Greenland

73. Indo-Pak summit between Parvez Musharraf and Atal Bihari Bajpayee was held in 2001 at
(a) Delhi (b) Shimla (c) Agra (d) Mumbai

74. In May 2009, in the final of Sultan Azlan Shah Championship Indian men’s Hockey team defeated
(a) Pakistan (b) Argentina (c) New Zealand (d) Malaysia

75. Which of the following is not a union territory?


(a) Tripura (b) Daman and Diu (c) Lakshadweep (d) Puducherry

76. Who was the founder editor of”Kesari”?


(a) Lala Lajpat Rai (b) Bal Gangadhar Tilak
(c) Gopal Krishna Gokhle (d) Dadabhai Naoroji

77. With whose permission did the English set up their first factory at Surat?
(a) Akbar (b) Shahjahan (c) Jahangir (d) Aurangzeb

78. The group of nations known as G-8 started as 0-7. Which among the following was not one of them?
(a) Canada (b) Italy (c) Russia (d) Japan

79. Emperor Akbar the Great, died in the year


(a) 1505 (b) 1605 (c) 1606 (d) 1590

80. What was the name of Emperor Shahjahan before he became Emperor?
(a) Kusrau (b) Khurram (c) Parvez (d) Shaheryar

Previous Years
Page 56 CLAT & AILET Papers
81. Which of the following trees has medicinal value?
(a) pine (b) teak (c) oak (d) neem

82. Identify the main principle on which the Parliamentary system operates?
(a) Responsibility of executive to legislature
(b) Supremacy of democracy
(c) Rule of law
(d) Supremacy of the constitution

83. Great tennis player Bjorn Borg belongs to which country?


(a) Italy (b) Latvia (c) U.S.A. (d) Sweden

84. Which of the following is the national song of India?


(a) Vande Mataram... (b) Jana gana mana adhinayak...
(c) Ye mera chaman, ye mera chaman... (d) Sarejahari se achchha...

85. Which country was known as sick man of Europe?


(a) Greece (b) Latvia (c) Turkey (d) Austria

86. in Malaysia, the word ‘bhumiputra’ refers to


(a) Malays (b) Chinese (c) Indians (d) Buddhists

87. What was the real name of Munshi Premchand?


(a) Nabab Rai (b) Dhanpat Rai
(c) Ram Chandra Srivastava (d) Han Shankar

88. Who is the author of ‘Old Man and the Sea’?


(a) John Ruskin (b) Raja Rao (c) Gunter Grass (d) Ernest Hemingway

89. Prophet Mohammed was born in


(a) 570 A.D. (b) 720 A.D. (c) 620 A.D. (d) 510 A.D.

90. When was the First World Wan declared?


(a) 1914 (b) 1915 (c) 1918 (d)1913

LEGAL APTITUDE

91. ‘No-fault liability’ means


(a) liability for damage caused through negligence
(b) liability for damage caused through fault
(c) absolute liability even without any negligence or fault
(d) freedom from liability

92. An ‘encumbrance’ in legal parlance is a


(a) liability on property (b) grant of property
(c) gift of property (d) restriction on property

93. A husband and wife have a right to each other’s company. This right is called
(a) matrimonial right (b) consortium right (c) marital right (d) conjugal right

Previous Years
CLAT & AILET Papers Page 57
94. Release of prisoner before completion of his sentence is called
(a) release (b) parole (c) acquittal (d) lease

95. Result of successful prosecution is


(a) acquittal (b) discharge (c) conviction (d) charge sheeting

96. The manager of waqf is known as


(a) Sajjadanashin (b) Khadim (c) Mutawalli (d) Mujawar

97. “Ipso facto”means


(a) in place of (b) by reason of that fact
(c) by the same source (d) by the way

98. ‘Requisition’ means


(a) permanent transfer of the title of the property
(b) supervision of property
(c) taking control of property temporarily
(d) taking possession permanently

99. ‘Corroborative evidence’ means


(a) main evidence in a case
(b) evidence which supports other evidence
(c) evidence that proves the guilt of an accused person
(d) evidence of a person who supports the accused

100. Ex-parte decision means a decision given


(a) after hearing both the parties (b) without proper procedure
(c) after observing proper procedure (d) without hearing the opponent

101. Which of the following constitutions is a unitary constitution


(a) U.S. (b) British (c) Indian (d) Australian

102. Which of the following is not a fundamental right in India?


(a) right to form association (b) freedom of religion
(c) right to property (d) right to move throughout the territory of India

103. Which of the following marriages is approved by Islamic law? Between a Muslim
(a) male and a Christian female (b) female and a Hindu male
(c) female and a Christian male (d) female and a Jew male

104. Which of the following constitutions when framed did not provide for judicial review?
(a) Indian (b) Pakistani (c) U.S. (d) Australian

105. Ratio decidendi means


(a) a judicial decision
(b) part of the judgment which possesses authority
(c) any observation made by the court which goes beyond the requirement of the case
(d) an observation made by a judge

Previous Years
Page 58 CLAT & AILET Papers
106. ‘Dyarchy’ under the government of India Act 1919 meant
(a) division of powers between the central and provincial government
(b) separation of judiciary from executive
(c) division of executive departments under elected ministers and the members of the governor’s
executive council
(d) separation between legislature and executive

107. Fringe benefit tax is a tax


(a) paid by an employer in respect of the fringe benefits provided or deemed to have been provided
by an employer to his employee
(b) paid by an employer for the benefits which he enjoys
(c) paid by a person for the benefits which he gets from his employer
(d) paid by a member of scheduled castes and scheduled tribes for benefits they receive from the
government

108. Which of the following is not true about a criminal proceeding?


(a) the court may ask to pay a fine.
(b) the court may order the transfer of the ownership of the property.
(c) there is prosecution.
(d) the court may discharge an accused.

109. In Ram v/s Shayam, Ram cannot be a


(a) plaintiff (b) appellants (c) defendant (d) prosecutor

110. Cr.P.C. stands for


(a) Criminal proceedings code (b) Criminal proceedings court
(c) Crime prevention code (d) Criminal Procedure code

111. Medical Science used for investigating crimes is known as


(a) Criminal Medicine (b) Epistemological Science
(c) Forensic science (d) Ontological Science

112. Apuisne judge of a High Court is


(a) a judge other than a Chief Justice (b) the Chief Justice
(c) a temporary judge (d) a retired judge

113. Intra vires means


(a) within the powers (b) outside the powers
(c) within the scope of fundamental rights (d) regular

114. X, the servant of Y, takes a hundred rupee note from Y’s pocket and hides it under the carpet in the
house of Y. X tells Z another servant of Y, about the currency note and both agree to share the
money when the currency note is taken by X from the hiding place. Before X could recover the note,
it was found by Y. Decide if an offence was committed and if so who committed the offence?
(a) No offence was committed. (b) Only X committed the offence.
(c) Both X and Z committed the offence. (d) Only Z committed the offence.

Previous Years
CLAT & AILET Papers Page 59
115. Moots, in law schools, are
(a) exercises of law teaching
(b) legal problems in the form of imaginary cases, argued by two opposing students before a bench
pretending to be a real court
(c) imaginary class room where a student acts as a teacher
(d) a debate on a legal problem

116. Scheduled Tribe status is


(a) restricted to Hindus (b) religiously neutral
(c) restricted to Hindus and Christians (d) restricted to Hindus and Muslims

117. Which of the following has not been a woman judge of the Supreme Court of India, till 2009?
(a) Justice Gyan Sudha Mishra (b) Justice Sujata Manohar
(c) Justice Ruma Pal (d) Justice Fathima Beevi
118. What is the meaning of chattel?
(a) any property (b) immovable property
(c) movable property (d) cattle

119. In a civil suit, the person who files suit and the person against whom the suit is filed are called
(a) accused, prosecutor (b) accuser, defendant
(c) appellant, respondent (d) plaintiff, defendant

120. In a criminal case, an accused person, who in consideration of his non- prosecution offers to give
evidence against other accused, is called
(a) accomplice (b) hostile witness
(c) approver (d) hostile accomplice

121. The President of India is elected by an electoral college consisting of


(a) all the members of both the Houses of Parliament and all the members of all the Legislative
Assemblies
(b) all the elective members of both the Houses of Parliament and all the members of all the
Legislative Assemblies
(c) all the members of both the Houses of Parliament and all the elected members of all the Legislative
Assemblies
(d) all the elected members of both the Houses of Parliament and all the elected members of all the
Legislative Assemblies

122. Which of the following is not a fundamental right?


(a) freedom of speech (b) right to life (c) right to equality (d) right to work

123. International Labour Organization has its headquarters at


(a) The Hague (b) Geneva (c) New York (d) London

124. The Child Marriage Restraint Act 2006 is applicable to


(a) only Hindus
(b) all Indians except Muslims as the minimum age of marriage among Muslim girls is puberty
(beginning of menstruation) in Muslim personal law
(c) all irrespective of religion
(d) all except Muslim, Christians and Jews

Previous Years
Page 60 CLAT & AILET Papers
125. X, a shopkeeper, leaves a sealed 5 kilogram bag of a branded wheat flour at the door of Y with a note
“you will like this quality wheat flour and pay Rupees 100 for this bag” without being asked to do so.
Y on coming back, collects the bag from his door, opens the seal of the bag, and uses a quarter of
kilogram for making chapattis (unleavened bread). But next day returns the bag. Is he bound to pay
for the bag? He is
(a) not bound to pay as he did not ask the shopkeeper to deliver the bag
(b) bound to pay as he has opened the bag
(c) bound to pay only for the quantity used
(d) neither bound to pay nor return the bag

126. Within the jurisdiction of which High Court does Lakshdweep fall
(a) Bombay High Court (b) Kerala High Court
(c) Madras High Court (d) Delhi High Court

127. Which of the following is not the function of the International Court of Justice? It
(a) gives advisory opinion at the request of general Assembly
(b) gives advisory opinion at the request of Security Council
(c) interprets treaties when considering legal disputes brought before it by nations
(d) decides international crimes

128. Bank nationalization case relates to the nationalization of


(a) some banks by the government of India after economic liberalisation in 1991.
(b) some banks under a law during the Prime Ministership of Mrs. Indira Gandhi.
(c) all the private Indian Banks during the Prime Ministership of Narasimha Rao.
(d) all the private Indian Banks during the Prime Ministership of Mrs. Indira Gandhi

129. Which of the following is not included within the meaning of intellectual property
(a) Patents (b) Copyrights
(c) Trade mark (d) Property of an intellectual

130. The main aim of the competition Act 2002 is to protect the interests of
(a) the multinational corporation (b) the Indian companies
(c) the consumers (d) the market

131. Which of the following judges had never been the chairman of the Law Commission of India?
(a) Justice R.C. Lahoti (b) Justice A.R. Lakshamanan
(c) Justice Jeevan Reddy (d) Justice Jagannadha Rao

132. Who among the following was the first chief Information Commissioner of India?
(a) Wajahat Habibullah (b) Irrfan Habib
(c) Tahir Mahmood (d) Najma Heptullah

133. R.T.I. stands for


(a) Revenue transactions in India (b) Research and Technology Institute
(c) Rural and Transparency infrastructure (d) Right to information

134. Fiduciary relationship is relationship based on


(a) contract (b) trust (c) blood relationship (d) money

Previous Years
CLAT & AILET Papers Page 61
135. Human rights day is observed on
(a) 14th February (b) 26th November (c) 2nd October (d) 10th December

LOGICAL REASONING

In each of the questions 136 to 140 two statements are given. There may or may not be cause and
effect relationship between the two statements. Mark your answer using this code.

(a) Statement I is the cause and statement II is the effect


(b) Statement Il is the cause and statement I is the effect
(c) Both the statements are independent causes
(d) Both the statements are independent effects

136. Statement I: School education has been made free for children of poor families.
Statement II: Literacy rate among the poor is steadily growing.

137. Statement I: Hallmarking of gold jewellery has been made compulsory.


Statement II: Many persons do not prefer to buy hallmarked jewellery.

138. Statement I: Many vegetarians are suffering from stomach ailments.


Statement II: Many dead fish were found near the lake shore.

139. Statement I: Ahmed is a healthy boy.


Statement II: His mother is very particular about the food he eats.

140. Statement I: Rate of crime is very low in this city.


Statement II: The police is efficient in this city.

In each of the questions 141 to 145 a statement is followed by two assumptions. These
assumptions may or may not be implicit in the statement. Select your response in accordance
with the following code.

(a) Only assumption I is implicit


(b) Only assumption II is implicit
(c) Both assumptions I and II are implicit
(d) Neither of the assumptions I nor II is implicit

141. Statement: If Ram has finished reading the instructions, let him begin activities accordingly.
Assumption I: Ram has understood the instructions.
Assumption II: Ram would be able to act accordingly.

142. Statement: Children below the age of seven should not be prosecuted for crimes.
Assumption I: Generally, children below seven cannot distinguish between right and wrong.
Assumption II: Children below the age of seven are generally mentally unsound.

Previous Years
Page 62 CLAT & AILET Papers
143. Statement: The employer has a right to reject the application of any candidate for employment
without assigning any reason, while short listing candidates for interview.
Assumption I: The employer is impartial and believes in transparency in employment practices.
Assumption II: The employer wants to call only those candidates for interview, who in his opinion
are eligible.

144. Statement: The government has decided to reduce custom duty on computers.
Assumption I: The government wants to make computers accessible to larger number of people.
Assumption II: Prices in domestic market may go up in near future.

145. Statement: You can win over new friends by your warm smile.
Assumption I: It is necessary to win over new friends.
Assumption II: It is always better to smile warmly to new persons.

146. Six students A, B, C, D, E and F are sitting. A and B are from Mumbai, rest are from Delhi. D and F
are tall but others are short. A, C, and D are girls, others are boys. Which is the tall girl from Delhi?
(a) F (b) D (c) E (d) C

147. P is the brother of Q. R is the sister of Q. S is the Sister of R. How is Q related to S?


(a) brother (b) sister (c) brother or sister (d) son

In question numbers 148 to 150 two sets of words have certain relation. Select a word to replace
the question mark so as to make a similar relational pair with the other word in the third set.

148. Cat-Kitten; Goat-kid; Sheep-?


(a) colt (b) filly (c) lamb (d) wool

149. Cataract-eye; jaundice-liver; pyorrhea-?


(a) breath (b) tongue (c) ears (d) teeth

150. Blue-moon; blue-black; black-?


(a) sheep (b) goat (c) sky (d) star

A principle/principles and a fact situation are given in question no.151-155 Decide only on the
basis of the principle(s).

151. Principles
(i) Neighbour principle-A person is liable if he harms his neighbour. A neighbour is one whose
action affects another
(ii) One is liable only for contractual relations
Facts
X manufactures a food item and sells his food item to Y, a whole seller. Y appoints Z, a retailer to
retail these items. Z sells the food item to a consumer who after eating them falls ill. X is liable to the
consumer because
(a) of contractual relations (b) of the Food Adulteration Act
(c) the consumer is the neighbor of X (d) of the consumer protection law

Previous Years
CLAT & AILET Papers Page 63
152. Principle
(i) Freedom consists in making choices out of two or more alternatives
(ii) Everyone has freedom to speak
Facts
X says his freedom to speech includes freedom not to speak. X’s assertion is
(a) wrong
(b) right
(c) wrong because the freedom to speak cannot mean freedom not to speak
(d) right because X may opt to speak or not to speak

153. Principles
(i) A master is liable for the wrongful acts of his servant
(ii) A person can be called a servant only if there is a relation of employment and he acts under
the order and on behalf of his master
Facts
X bank launched a saving scheme for poor sections of the society and the customer can deposit
Rs. 10 per day. Y an unemployed youth collected money from several customers, and on behalf of
them deposited the money at the Bank evely day. The bank gave to Y a small commission. After
sometime, Y disappeared without depositing the money given by the customers. The customers
bring a suit alleging that the Bank is liable. Decide
(a) the Bank is liable because it paid commission to Y
(b) the Bank is liable because Y was their servant
(c) the Bank is not liable because Y was not their servant
(d) No one is liable

154. Principles
X propounds the principle that everyone in this world always speaks lies

Fact
X wants to know weather this principle is logically true or false
(a) logically the principle may be true
(b) everyone in the whole of this world does not always speak lies
(c) logically X is also speaking lies.
(d) everyone is basically an honest person

155. Principle
Whosoever enters into or upon the property in the possession of another, with intent to commit an
offence or to intimidate or annoy any person in possession of the property, and remains there with
intent thereby to intimidate or annoy another person or with intent to commit an offence is guilty of
criminal trespass
Facts
The accused entered at night into a house to carry on intimate relations with an unmarried major
girl on her invitation and information that her family members are absent. However, he was caught
by her uncle before he could get away. Is the accused guilty of criminal trespass? He is
(a) guilty of criminal trespass as he annoyed the uncle
(b) guilty because he entered the house to commit a crime against the girl
(c) guilty because no one should enter into the house of another at night
(d) not guilty of criminal trespass

Previous Years
Page 64 CLAT & AILET Papers
156. In a code, every letter of the alphabet is replaced by some other letter. The code for the name
Ram Kumar is
(a) Ten Ronet (b) Len Finol (c) Pen Sinet (d) Elephant

In questions 157 and 158 four of the given five options are alike in a certain way and form a
group. Which is the one that does not belong to that group?

157. (I) 217 (II)143 (III) 214 (IV)157 (V)131

(a) I (b) V (c) I (d) III

158. (I) Gourd (II) Radish (III) Spinach (IV) Cucumber (V) Beetroot

(a) radish (b) beetroot (c) Potatoes (d) Spinach

Point out the entry which does not form a class with the other entries in question number 159 to
161.

159. (a) house (b) mortgage (c) hypothecation (d) immovable property

160. (a) Law (b) Court (c) Morality (d) Judge

161. (a) Freedom of speech (b) Right to equality


(c) Freedom of religion (d) Right to make contract

In question 162 to 166 a question and two arguments are given. Arguments in relation to the
question are either weak or strong. Use this key to give your responses.

(a) Argument I is strong


(b) Argument Il is strong
(c) Both I and II are strong
(d) Both I and II are weak

162. Question: Should there be complete ban on manufacture of fire crackers in India?
Argument I: No, this will render thousands of workers jobless.
Argument II: Yes, the fire cracker manufacturers use child labour.

163. Question: Should private operators be allowed to operate passenger train service in India?
Argument I: No, private operators do not agree to operate on non profitable sectors.
Argument II: Yes, it will improve the quality of Indian Railway Service.

164. Question: Should the system of reservation of posts for scheduled castes be introduced in
private sector?
Argument I: Yes, this would give more opportunity of development to these groups.
Argument II: No, this would effect merit.

Previous Years
CLAT & AILET Papers Page 65
165. Question: Would the problem of old parents be solved if children are made legally responsible to
take care of their parents in old age?
Argument I: Yes, such problems can be solved only through law.
Argument II: Yes, this will bring relief to old parents.

166. Question: Should right to primary education be made a fundamental right?


Argument I: We should first complete other developmental projects, education of children can wait.
Argument II: Yes, without primary education for all there cannot be inclusive development.

167. Pramesh is heavier than Jairam but lighter than Gulab. Anand is heavier than Gulab. Mohan is
lighter than Jairam. Who among them is the heaviest?
(a) Jairam (b) Anand (c) Gulab (d) Pramesh

168. Ravi is the brother of Amit’s son. How is Amit related to Ravi?
(a) Cousin (b) Father (c) Son (d) Grandfather

169. If CABLE is coded as ZCDAY, then STABLE will be coded as


(a) TPADAY (b) TPCDCY (c) TPCDAY (d) TPCYAY

170. If CHARTER is coded as UMOEPYE then PARTNER will be coded as


(a) AONPCYE (b) AEEPCYE (c) AOEACYE (d) AOEPCYE

In each of the questions 171 to 180, two words are paired which have a certain relation. Select a
correct option to substitute question mark so as to make a similar relational pair with the word
given after double colon (: :)

171. Constituent Assembly : Constitution : : Parliament :?


(a) Statute (b) Legislative bills (c) Speaker (d) Prime Minister

172. Right : duty : : Power :?


(a) Wrong (b) Weak (c) Powerless (d) Liability

173. Elephant : Calf : : Tiger :?


(a) Pup (b) Tigress (c) Cub (d) Baby Tiger

174. Patient : Doctor : : Litigant :?


(a) Advisor (b) Help (c) Legal aid (d) Lawyer

175. Prosecutor : Accused : : plaintiff :?


(a) Appellant (b) Defendant (c) Plaint (d) Suit

176. Lok Sabha : Meera Kumar : : Rajya Sabha :?


(a) Hameed Ansari (b) Najma Heptullah (c) Sushma Swaraj (d) Arun Jaitely

177. President of India : 35 : : Member of Parliament (Rajya Sabha):?


(a)18 (b) 21 (c) 25 (d) 30

Previous Years
Page 66 CLAT & AILET Papers
178. India : Parliamentary System : : U.S.A. :?
(a) Democratic System (b) Presidential System
(c) Federal System (d) Republican system

179. Executive : President : : Judiciary :?


(a) Supreme Court (b) Chief Justice
(c) Constitution (d) Government of India

180. World War II : United Nations : : World War I :?


(a) Treaty of Versailles (b) International Commission of Jurists
(c) League of Nations (d) International court of Justice

ELEMENTARY MATHEMATICS (NUMERICAL ABILITY)

181. The square root of 289 is


(a)13 (b)17 (c) 27 (d) 23

182. Find the factors of 330.


(a) 2 × 4 × 5 × 11 (b) 2 × 3 × 7 × 13 (c) 2 × 3 × 5 ×13 (d) 2 × 3 × 5 ×11

183. Find the factors of 1122.


(a) 3 × 9 × 17 × 2 (b) 3 × 11 × 17 × 2 (c) 9 × 9 × 17 × 2 (d) 3 × 11 × 17 × 3

184. Which of the following is not a prime number?


(a) 23 (b) 29 (c) 43 (d) 21

185. If the numbers from 1 to 24, which are divisible by 2 are arranged in descending order, which number
will be at the 8th place from the bottom?
(a) 10 (b) 12 (c) 16 (d) 18

186. Average age of ten persons learning yoga is 32 years. When the age of their instructor is added, the
average age becomes 34 years. The age of their instructor is
(a) 54 (b) 52 (c) 46 (d) 56

187. Find 12% of 5000.


(a) 620 (b) 600 (c) 680 (d) 720

188. Square root of 400 is


(a) 40 (b) 25 (c) 20 (d) 100

189. What is the place (location) value of 5 in 3254710?


(a) 10000 (b) 5 (c) 54710 (d) 50000

190. 6 × 3 (3 – 1) is equal to
(a) 53 (b) 36 (c) 20 (d) 19

Previous Years
CLAT & AILET Papers Page 67
191. Y travels 15 kilometers due South, then 5 kilometers due West, then 18 kilometers due North, then
3 kilometers due South, then 5 kilometers due East. How far is he from the starting point?
(a) 6 kilometers (b) 3 kilometers (c) 0 kilometers (d) 9 kilometers

For questions 192 and 193, select appropriate numbers to fill in the blanks in the given series.

192. 3, 9, ......, 6561


(a) 18 (b) 27 (c) 81 (d) 61

193. 100, 50, 33.33, ......, 20


(a) 25 (b) 30 (c) 22 (d) 21

194. Which of the following fractions has the highest value?


3/5, 4/3, 2/5, 1/2
(a) 3/5 (b) 4/3 (c) 2/5 (d) 1/2

195. Four gardeners with four grass mowers mow 400 square meters of ground in four hours. How long
would it take for eight gardeners with eight grass mowers to mow 800 square meters of ground?
(a) 8 hours (b) 6 hours (c) 12 hours (d) 4 hours

For questions 196 to 198, select one of the given numbers to replace the question mark in the
series.

196. 2, 3, 4, 6, 7, 8, 9, 11, 12, 13, 14, 15, 17, 18, 19, 20, 21, ?
(a) 22 (b) 23 (c) 24 (d) 25

197. 117, 104, 91, 78, ?


(a) 39 (b) 60 (c) 65 (d) 36

198. 88, 96, 104, ?


(a) 100 (b) 110 (c) 120 (d) 112

199. Ram borrows Rs.520 from Govind at a simple interest of 13% per annum. What amount of money
(in Rs.) should Ram pay to Govind after six months to be absolved of the debt?
(a) 552.80 (b) 553.80 (c) 453.80 (d) 353.80

200. Which of the following is a prime number?


(a) 19 (b) 20 (c) 21 (d) 22

Previous Years
Page 68 CLAT & AILET Papers
CLAT Question Paper 2011
ENGLISH (COMPREHENSION)

The questions in this section are based on a single passage. The questions are to be answered
on the basis of what is stated or implied in the passage.

Please note that for some of the questions, more than one of the choices could conceivably
answer the question. However, you are to choose the best answer; that is, the response that
most accurately and completely answers the question.

Passage for Questions 1 to 10:


In 1954, a Bombay economist named A.D. Shroff began a Forum of Free Enterprise, whose ideas on
economic development were somewhat at odds with those then influentially articulated by the Planning
Commission of the Government of India. Shroff complained against the ‘indifference, if not discouragement’
with which the state treated entrepreneurs.

At the same time as Shroff, but independently of him, a journalist named Philip Spratt was writing a series
of essays in favour of free enterprise. Spratt was a Cambridge communist who was sent by the party in
1920s to foment revolution in the subcontinent. Detected in the act, he spent many years in an Indian jail.
The books he read in the prison, and his marriage to an Indian woman afterwards, inspired a steady move
rightwards. By the 1950s, he was editing a pro- American weekly from Bangalore, called MysIndia. There
he inveighed against the economic policies of the government of India. These, he said, treated the entrepreneur
‘as a criminal who has dared to use his brains independently of the state to create wealth and give
employment’. The state’s chief planner, P.C. Mahalanobis, had surrounded himself with Western leftists
and Soviet academicians, who reinforced his belief in ‘rigid control by the government over all activities’.
The result, said Spratt, would be ‘the smothering of free enterprise, a famine of consumer goods, and the
tying down of millions of workers to soul-deadening techniques.’

The voices of men like Spratt and Shroff were drowned in the chorus of popular support for a model of heavy
industrialization funded and directed by the governments. The 1950s were certainly not propitious times for
free marketers in India. But from time to time their ideas were revived. After the rupee was devalued in 1966,
there were some moves towards freeing the trade regime, and hopes that the licensing system would also
be liberalized. However, after Indira Gandhi split the Congress Party in 1969, her government took its ‘left
turn’, nationalizing a fresh range of industries and returning to economic autarky.

1. Which of the following statements can most reasonably be inferred from the information available in
the passage:
(a) P.C. Mahalanobis believed in empowering private entrepreneurs and promoting free market.
(b) Phillip Spratt preferred plans that would create economic conditions favourable for a forward
march by the private enterprise.
(c) Restrictions on free markets enriched large Indian companies.
(d) Philip Spratt opposed the devaluation of rupee in 1966.

Previous Years
Page 70 CLAT & AILET Papers
2. Which of the following statements is least likely to be inferred from the passage:
(a) Acceptance of A.D. Shroff’s plans in the official circles smothered free enterprise in India.
(b) The views of the Forum of Free Enterprise ran against the conception of development then
prevalent among the policy makers.
(c) A.D. Shroff believed that state should actively support the private sector.
(d) Philip Spratt had been educated in Cambridge.

3. Select the statement that best captures the central purpose of this passage:
(a) Highlight that even though there were advocates for free-market and private enterprise in the
early years of independent India, they were crowded out by others who supported a dominant
role for state over private enterprise.
(b) Explain the politics behind Indira Gandhi’s decision to nationalise the banks.
(c) Demonstrate with the help of statistics how the preference of policy makers for Soviet-style
economic policies prevented India’s economic growth.
(d) Establish that devaluation of rupee in 1966 was vindicated by subsequent experience.

4. Philip Spratt came to India because he:


(a) Fell in love with an Indian women
(b) Wanted to protest against the economic policies of the Indian government.
(c) Was offered the editorship of Mysindia.
(d) Had been instructed to work towards the goal of inciting a revolution in India.

5. The author avers that A.D Shroff’s ideas were somewhat at odds with the views of Planning
Commission because:
(a) A.D. Shroff was in favour of rigid governmental control over all economic activities.
(b) Shroff had opposed government’s decision to devalue Indian rupee.
(c) The hostility of the government to private entrepreneurs was complained against by A.D. Shroff.
(d) Shroff had been critical of the influence of Soviet academicians over India’s economic policy.

6. The ideological shift of Philip Spratt to the right was caused by:
(a) The demise of the Soviet Union.
(b) The start of the weekly called MysIndia.
(c) The books that he encountered in the prison.
(d) The dissolution of his first marriage to his college friend.

7. Select the statement that could be most plausibly inferred from this passage:
(a) Philip Spratt and A.D. Shroff were members of the Forum for Free Enterprise.
(b) The first two Five Year Plans emphasised on the importance of private enterprise as the spearhead
of economic growth.
(c) P.C. Mahalanobis had mooted the expulsion of foreign firms like Coca Cola and IBM from India.
(d) The hopes that the licensing regime would be liberalized after the devaluation of Indian rupee
were belied in the aftermath of the split in the Congress Party.

8. The author alludes to nationalization of industries in 1969 in order to:


(a) Show the contradictions between AD Shroff’s economic views and the official economic policies
of the Government of India.
(b) Exemplify the shift of the Indira Gandhi led government to the ‘left’
(c) Demonstrate the ideological changes in the worldview of Philip Spratt.
(d) Highlight the negative political repercussions of the decision to devalue the Indian currency.

Previous Years
CLAT & AILET Papers Page 71
9. “Neither Philip Spratt nor A.D. Shroff __________ able to convince Mahalanobis.” Select the most
appropriate phrase out of the four options for filling the blank space in the aforesaid sentence.
(a) Were (b) Are (c) Was (d) Is.

10. The word ‘inveighed’ in this passage means:


(a) Praised (b) Recited (c) Proclaimed (d)Remonstrated

Passage for Questions 11 to 20

In Manu Joseph’s debut novel Serious Men, the protagonist, Ayyan Mani, is a sly, scheming Dalit-Buddhist
who almost gets away with passing off his partially deaf son, Adi, as a prodigy, a genius who can recite the
first 1,000 prime numbers. The garb of satire—where almost every character cuts a sorry figure—gives the
author the licence to offer one of the most bleak and pessimistic portrayals of urban Dalits. Despite his
savage portrayal of Dalit (and female) characters—or perhaps because of it?—Serious Men has won
critical appreciation from a crosssection of readers and critics.

At a time when a formidable body of Dalit literature— writing by Dalits about Dalit lives—has created a
distinct space for itself, how and why is it that a novel such as Serious Men, with its gleefully skewed
portrayal of an angry Dalit man, manages to win such accolades? In American literature—and particularly
in the case of African-American authors and characters—these issues of representation have been debated
for decades. But in India, the sustained refusal to address issues related to caste in everyday life—and the
continued and unquestioned predominance of a Brahminical stranglehold over cultural production—have
led us to a place where non-Dalit portrayal of Dalits in literature, cinema and art remains the norm.

The journey of modern Dalit literature has been a difficult one. But even though it has not necessarily
enjoyed the support of numbers, we must engage with what Dalits are writing—not simply for reasons of
authenticity, or as a concession to identity politics, but simply because of the aesthetic value of this body
of writing, and for the insights it offers into the human condition. In a society that is still largely unwilling to
recognise Dalits as equal, rights-bearing human beings, in a society that is inherently indifferent to the
everyday violence against Dalits, in a society unwilling to share social and cultural resources equitably
with Dalits unless mandated by law (as seen in the anti-reservation discourse), Dalit literature has the
potential to humanise non- Dalits and sensitise them to a world into which they have no insight. But before
we can understand what Dalit literature is seeking to accomplish, we need first to come to terms with the
stranglehold of non-Dalit representations of Dalits.

Rohinton Mistry’s A Fine Balance, published 15 years ago, chronicles the travails of two Dalit characters—
uncle Ishvar and nephew Omprakash—who migrate to Bombay and yet cannot escape brutality. While the
present of the novel is set at the time of the Emergency, Ishvar’s father Dukhi belongs to the era of the anti-
colonial nationalist movement. During one of Dukhi’s visits to the town, he chances upon a meeting of the
Indian National Congress, where speakers spread the “Mahatma’s message regarding the freedom struggle,
the struggle for justice,” and wiping out “the disease of untouchability, ravaging us for centuries, denying
dignity to our fellow human beings.”

Neither in the 1940s, where the novel’s past is set, nor in the Emergency period of the 1970s— when the
minds and bodies Ishvar and Omprakash, are savaged by the state—do we find any mention of a figure like
BR Ambedkar or of Dalit movements. In his ‘nationalist’ understanding of modern Indian history, Mistry
seems to have not veered too far from the road charted by predecessors like Mulk Raj Anand and Premchand.
Sixty years after Premchand, Mistry’s literary imagination seems stuck in the empathy-realism mode,

Previous Years
Page 72 CLAT & AILET Papers
trapping Dalits in abjection. Mistry happily continues the broad stereotype of the Dalit as a passive sufferer,
without consciousness of caste politics.

11. Which of the following is the closest description of the central argument of this passage:
(a) Manu Joseph’s novel presents a scathing portrayal of Dalits.
(b) Contemporary American literature is very cautious on politically correct representation of
minorities.
(c) The last two decades have witnessed the rise of a very vibrant Dalit literature.
(d) Portrayal of Dalits by non-Dalits merely as passive victims has been the dominant norm in
Indian literature, cinema and art.

12. According to this passage, Premchand and Mulk Raj Anand:


(a) Presented a stereotyped version of Dalit characters in their writings.
(b) Excelled in writing satires on social inequality.
(c) Were politically opposed to the views of B.R. Ambedkar.
(d) Were closely involved with the leadership of the nationalist movement.

13. The writer refers to the ‘anti-reservation discourse’ in order to argue that:
(a) Dalit literature has had a very difficult journey since its origins.
(b) Manu Joseph is viscerally opposed to Dalits.
(c) Persons belonging to the upper castes are inherently indifferent to routine violence against
Dalits.
(d) Indian society is not yet ready to equitably share, on its own, social, cultural and political space
with Dalits.

14. Which of the following statements is least likely to be inferred from this passage:
(a) The author of Serious Men has used the literary device of satire to present an unflattering picture
of women characters.
(b) Issues of representation of minorities have been debated extensively in American literature.
(c) The writer of this passage believes that engagement with Dalits is necessary only because such
engagement affirms the importance of identity politics.
(d) The writer believes that Rohinton Mistry presented a stereotypical representation of Dalits character
in his book.

15. According to the information available in the passage, the writer attributes the prevalence of
representation of Dalits by non-Dalits in literature, art and media to:
(a) The nationalist understanding of Indian history.
(b) Marginalisation of B.R Ambedkar from nationalist movement.
(c) The anti-reservation discourse
(d) Brahminical control over cultural production.

16. Which of the following is not among the reasons suggested by the writer for engaging with Dalit
writing:
(a) Dalit literature has the potential to sensitize non-Dalits about the experiences of the former.
(b) Dalit writing is more authentic than representation of Dalits by non-Dalits.
(c) Dalit literature does not have the support of numbers.
(d) The aesthetic value of Dalit writing.

Previous Years
CLAT & AILET Papers Page 73
17. Which of the following statement cannot be inferred from the passage:
(a) Upper-castes have dominated the instruments of cultural production in Indian society.
(b) Indian society is unwilling to recognise Dalits as equal, rights-bearing human beings.
(c) Dalit writers have carved out a space for writings on Dalit experience and world view.
(d) The judiciary in India, in its opposition to reservation, has betrayed its unwillingness to acknowledge
Dalits as equal bearer of rights.

18. The writer of this passage is critical of Rohinton Mistry’s A Fine Balance for the reason that:
(a) It is an example of a book on Dalit characters by a Non-Dalit.
(b) The book suggests that Dalits are nothing more than passive sufferers without any agency.
(c) The book ignores the everyday violence that Dalits have to confront with.
(d) It bares the passive literary style of the author, Rohinton Mistry.

19. Which of the following words would be the best substitute for the word ‘sly’ in this passage:
(a) Bright (b) wise (c) devious (d) dim

20. “It is not as if Dalit movements ______ not active during the periods that form A Fine Balance’s
backdrop.” Select the most appropriate choice to fill in the blank in the above sentence:
(a) is (b) was (c) were (d) are

Passage for Questions 21 to 30

In recent weeks, the writers William Dalrymple and Patrick French, among others, have come before a
fusillade of criticism in India, much of it questioning not their facts, not their interpretations, but their
foreignness.

“Who gets to write about India?” The Wall Street Journal asked on Wednesday in its own report on this
Indian literary feuding. It is a complicated question, not least because to decide who gets to write about
India, you would need to decide who gets to decide who gets to write about India. Rather than conjecturing
some Committee for the Deciding of the Deciding of Who Gets to Write About India, it might be easier to
let writers write what they please and readers read what they wish.

The accusations pouring forth from a section of the Indian commentariat are varied. Some criticism is of a
genuine literary nature, fair game, customary, expected. But lately a good amount of the reproaching has
been about identity.

In the case of Mr. Dalrymple, a Briton who lives in New Delhi, it is — in the critics’ view — that his writing
is an act of re-colonization. In the case of Mr. French, it is that he belongs to a group of foreign writers who
use business-class lounges and see some merit in capitalism and therefore do not know the real India,
which only the commentariat member in question does.

What is most interesting about these appraisals is that their essential nature makes reading the book
superfluous, as one of my Indian reviewers openly admitted. (His review was not about the book but about
his refusal to read the book.) The book is not necessary in these cases, for the argument is about who can
write about India, not what has been written.

For critics of this persuasion, India surely seems a lonely land. A country with a millennial history of
Hindus, Christians, Jews, Muslims and Buddhists living peaceably together; a country of hundreds of
dialects in which so many Indians are linguistic foreigners to each other, and happily, tolerantly so; a

Previous Years
Page 74 CLAT & AILET Papers
country that welcomes foreign seekers (of yoga poses, of spiritual wisdom, of ancestral roots) with open
arms; a country where, outside the elite world of South Delhi and South Bombay, I have not heard an Indian
ask whether outsiders have a right to write, think or exist on their soil.

But it is not just this deep-in-the-bones pluralism that challenges the who-gets-to-write-about- India contingent.
It is also that at the very heart of India’s multifarious changes today is this glimmering idea: that Indians
must be rewarded for what they do, not who they are. Identities you never chose — caste, gender, birth
order — are becoming less important determinants of fate. Your deeds — how hard you work, what risks
you take — are becoming more important.

It is this idea, which I have found pulsating throughout the Indian layers, that leaves a certain portion of the
intelligentsia out of sync with the surrounding country. As Mr. French has observed, there is a tendency in
some of these writers to value social mobility only for themselves. When the new economy lifts up the
huddled masses, then it becomes tawdry capitalism and rapacious imperialism and soulless globalization.

Fortunately for those without Indian passports, the nativists’ vision of India is under demographic siege.
The young and the relentless are India’s future. They could not think more differently from these literatis.

They savour the freedom they are gaining to seek their own level in the society and to find their voice; and
they tend to be delighted at the thought that some foreigners do the same in India and love their country as
much as they do.

21. Which of the following statements is least likely to be inferred from the passage:
(a) Younger generations of Indians are more tolerant of foreign scribes who write about their country.
(b) The writer believes that a section of Indian intelligentsia is very hostile to upward economic
mobility.
(c) Mr. William Dalrymple has been accused of recolonising India through his writings.
(d) Most of the criticism that has been recently directed at Patrick French has emphasized mainly
on the writer’s underwhelming literary style.

22. Which of the following would be the best substitute for the word, ‘fusillade’ in the passage?
(a) Barrage (b) Breach (c) Temper (d) Row

23. The writer uses the phrase, ‘who-gets-to-write-about-India contingent’ in this passage to refer to:
(a) Foreign writers who have written books on India.
(b) Critics who have attacked foreign writers writing on India for their mere foreignness.
(c) Elite residents of South Delhi and South Bombay.
(d) Cultural pluralists.

24. The writer believes that the most peculiar aspect of the criticisms that Patrick French and William
Dalrymple have received is that:
(a) Most such condemnation has emerged from elite Indians.
(b) Such critics are hostile to upward immobility.
(c) These censures are not centered on the books of such writers or their literary styles but are
targeted at their identity instead.
(d) These critics ignore the plural ethos of India.

Previous Years
CLAT & AILET Papers Page 75
25. Which of the following statements can be inferred from the passage:
(a) Ascriptive identities like caste, tribe, etc. are becoming more and more important with the
passage of time.
(b) Patrick French believes that the new market friendly economic policies followed for the last
decades have resulted in the rise of tawdry capitalism and rapacious imperialism.
(c) The writer is of the opinion that a section of the intelligentsia is divorced from the views of their
compatriots.
(d) While India has historically been very hospitable to a variety of religions, it has not been equally
open to linguistic foreigners.

26. According to the information available in the passage, the writer is of the opinion that:
(a) Writers like Patrick French do not know the real India.
(b) Most of the condemnation heaped on Dalrymple, French and himself has been on expected
lines.
(c) India’s reputation of pluralism is cosmetic at best, one that hides deep rooted hatred towards
foreigners.
(d) The new generation of Indians have internalized the idea that people should be rewarded for what
they do and not who they are.

27. The writer refers to the history of Hindus, Christians, Jews, Muslims and Buddhists living peaceably
together in India for millions of years in order to:
(a) Show India’s openness to foreigners who have visited Indian in the quest for yoga.
(b) Argue that India is a country of hundreds of dialects.
(c) Demonstrate the religiosity pervading in an average Indian.
(d) India’s deep-in the bones pluralism.

28. The writer argues that the nature of criticism he, Dalrymple and French have received for their books
renders reading their books superfluous because:
(a) Such criticism has been limited to a very small minority of Indians.
(b) These writers are popular among Indian youth, even among those who have not read their books.
(c) The literary styles of these writers are not the sole focus of such criticism.
(d) Such criticism is less about what has been written in their books than about who can write on
India.

29. According to the passage, the question ‘who gets to write about India’ is complicated because:
(a) India has been historically open to and tolerant of foreign writers and artists.
(b) This issue can be satisfactorily resolved only if we can decide who gets to decide who gets to
write about India.
(c) Ascriptive identities are becoming more and more important in a globalised world.
(d) This would result in a shift of attention from what has been written to who has written.

30. “But with many outsiders’ India-related books recently hitting bookstores there, the sensitivity _____
flared into a bout of vigorous literary nativism, with equally vigorous counterpunches.” Select the
most appropriate choice to fill in the blank in the above sentence:
(a) Has (b) Have (c) Was (d) did

Previous Years
Page 76 CLAT & AILET Papers
Passage for Questions 31 to 40

If religion and community are associated with global violence in the minds of many people, then so are
global poverty and inequality. There has, in fact, been an increasing tendency in recent years to justify
policies of poverty removal on the ground that this is the surest way to prevent political strife and turmoil.
Basing public policy – international as well as domestic – on such an understanding has some evident
attractions. Given the public anxiety about wars and disorders in the rich countries in the world, the indirect
justification of poverty removal – not for its own sake but for the sake of peace and quiet in the world –
provides an argument that appeals to selfinterest for helping the needy. It presents an argument for allocating
more resources on poverty removal because of its presumed political, rather than moral, relevance.

While the temptation to go in that direction is easy to understand, it is a perilous route to take even for a
worthy cause. Part of the difficulty lies in the possibility that if wrong, economic reductionism would not
only impair our understanding of the world, but would also tend to undermine the declared rationale of the
public commitment to remove poverty. This is a particularly serious concern, since poverty and massive
inequality are terrible enough in themselves, and deserve priority even if there were no connection whatsoever
with violence. Just as virtue is its own reward, poverty is at least its own penalty. This is not to deny that
poverty and inequality can – and do – have far reaching consequences with conflict and strife, but these
connections have to be examined and investigated with appropriate care and empirical scrutiny, rather than
being casually invoked with unreasoned rapidity in support of a ‘good cause.”

Destitution can, of course, produce provocation for defying established laws and rules. But it need not give
people the initiative, courage, and actual ability to do anything very violent. Destitution can be accompanied
not only by economic debility, but also by political helplessness. A starving wretch can be too frail and too
dejected to fight and battle, and even to protest and holler. It is thus not surprising that often enough
intense and widespread suffering and misery have been accompanied by unusual peace and silence.

Indeed, many famines have occurred without there being much political rebellion or civil strife or intergroup
warfare. For example, the famine years in the 1840s in Ireland were among the most peaceful, and there
was little attempt by the hungry masses to intervene even as ship after ship sailed down the river Shannon
with rich food. Looking elsewhere, my own childhood memories in Calcutta during the Bengal famine of
1943 include the sight of starving people dying in front of sweetshops with various layers of luscious food
displayed behind the glass windows, without a single glass being broken, or law or order being disrupted.

31. Select the statement that can be most plausibly inferred from the aforesaid passage:
(a) A society plagued by recurrent famines can never witness political revolution.
(b) Religious discrimination inevitably leads to violence and strife.
(c) Destitution of the masses leads to peace and social stability.
(d) Famines and starvation do not necessarily result in political rebellion.

32. The author believes that it may not be advisable to emphasise on the connection between poverty
and violence as:
(a) Emphasis on such connection appeals only to self-interest of persons.
(b) Linking poverty and violence undermines the moral character of anti-poverty measures.
(c) The absence of any essential connection between poverty and violence may then weaken the
very rationale of anti-poverty policies.
(d) There is no necessary link between poverty and inequality.

Previous Years
CLAT & AILET Papers Page 77
33. Which of the following best captures the central argument of this passage:
(a) Religion is inextricably linked with violence
(b) Famines may not necessarily result in civil unrest.
(c) Global poverty and inequality are one of the fundamental causes of global violence and strife.
(d) Basing anti-poverty programmes on the need for avoidance of violence and strife is dotted with
many pitfalls.

34. In the given passage, the word ‘perilous’ means:


(a) Scared (b) Costly (c) Futile (d) Dangerous

35. The author refers to his own experience as a child during the Bengal famine of 1943 in order to:
(a) Illustrate how religiosity may instill passive acceptance of even the worst forms of starvation
among people.
(b) Repudiate the argument that religious discrimination usually tends to inspire violent protests.
(c) Substantiate his assertion that it is not unusual to have the most intense suffering and misery
coexist with complete peace.
(d) Demonstrate that people confronted with acute starvation are rendered too helpless to protest
ever at all.

36. The word ‘destitution’ in this passage can be best substituted by:
(a) Dejection (b) Indigence (c) Default (d) Dereliction

37. Which of the following statement is least likely to be inferred from the passage:
(a) History is replete with instance of famines that have occurred without there being much violent
protest.
(b) Many writers and critics are increasingly advocating for stronger policies on poverty removal on
the ground that this would help prevent political turmoil.
(c) The author believes that the links between poverty and violence must never be emphasized at
all.
(d) Economic debility in turn inhibits political freedom.

38. The author asserts that basing anti-poverty measures on the avowed connections between poverty
and violence has certain apparent benefits because:
(a) Poverty is similar to religious exploitation in terms of the potential violent consequences.
(b) It leads to allocation of more resources on anti-poverty policies.
(c) The widespread concern about war and violence provides a rationale for povertyremoval that
appeals to the ‘self-interest’ of persons
(d) Otherwise, there would not have been the tendency to justify anti-poverty policies on the ground
that they prevent political turmoil.

39. ‘Economic reductionism’ in this passage means;


(a) Neglecting the economic connection between poverty and violence
(b) Excessive accent on poverty and inequality
(c) Emphasizing on the linkage between violence, poverty and economic equality.
(d) The view that every conflict is caused by underlying economic tensions.

40. “A sense of encroachment, degradation and humiliation can be even easier __ mobilize for rebellion
and revolt.” Select the most appropriate word out of the four options for filling the blank space in the
aforesaid sentence
(a) for (b) as (c) into (d) to

Previous Years
Page 78 CLAT & AILET Papers
GENERAL KNOWLEDGE/CURRENT AFFAIRS

41. Why was Arundhati Roy investigated for sedition?


(a) For committing contempt of court
(b) For saying that Kashmir is not an integral part of India
(c) For sympathising with the Maoists
(d) For condemning nuclear tests conducted by India

42. Damon Galgut’s ‘In a Strange Room’ was recently in news for:
(a) Man Booker Prize shortlist (b) Winning the Pulitzer Prize
(c) Winning the Orange Prize for fiction (d) None of the above

43. Who was recently in the news when the Supreme Court of India rejected her plea for Euthanasia,
but paved the way for legalization of passive euthanasia?
(a) Aruna Shanbaug (b) Aruna Roy
(c) Mary Roy (d) Medha Patkar

44. Nagoya Protocol, signed by India on 30th October, 2010 is:


(a) an international treaty of bilateral investment between India and Japan
(b) an international treaty to ensure that local produce are exploited only under license and for the
common good of the mankind
(c) an international treaty to ensure that the benefits of natural resources and their commercial
derivatives are shared with local communities
(d) None of the above.

45. Julian Assange, the founder of WikiLeaks, was arrested in which of the following nations?
(a) U.K (b) Sweden (c) U.S.A (d) Denmark

46. Which of the following are the five countries that have decided to bid for 2017 World Athletics
Championships?
(a) Qatar, USA, China, Sri Lanka and Brazil
(b) Germany, Britain, Hungary, Qatar and Spain
(c) Germany, Qatar, India, Spain and China
(d) Germany, Britain, China, Qatar and Spain

47. The recent Tunisian revolution is known as:


(a) Orange Revolution (b) Jasmine Revolution
(c) Purple Revolution (d) Crescent Revolution

48. ‘The Naïve and the Sentimental Novelist’ is a 2010 publication of Harvard University Press of which
of the following authors?
(a) Orhan Pamuk (b) J.M Coetzie
(c) Partha Chatterjee (d) Ben Okri

49. Who replaced Lalit Modi as the IPL Chairman and Commissioner from this year’s edition of the IPL?
(a) Chirayu Amin (b) Rajiv Shukla
(c) Ratnakar Shetty (d) Shashank Manohar

Previous Years
CLAT & AILET Papers Page 79
50. Which one of the following films was officially selected to compete in the Un Certain Regard (A
Certain Glance) category at the 2010 Cannes Film Festival?
(a) Udaan (b) My Name is Khan
(c) Wednesday (d) Dhobi Ghat

51. Rani Kumudini Devi, whose birth centenary is being celebrated in 2011, was the:
(a) First woman barrister of India (b) First woman Mayor of Hyderabad
(c) First woman photographer in India (d) First woman doctor of India

52. The Supreme Court in 2010 upheld an order of the Bombay High Court to lift a four-yearold ban
imposed by the Maharashtra government on publication and circulation of a controversial book,
authored by American scholar James Laine. Identify the book from the following.
(a) Chatrpati Shivaji (b) Shivaji – The Warrior King
(c) Shivaji - The Hindu King in Muslim India (d) None of the above.

53. In which case did the Nagpur Bench of the Bombay High Court on July 14, 2010 commute the death
sentence of six accused to rigorous life imprisonment?
(a) Khairlanji case (b) Bhopal Gas Leak case
(c) Bhagalpur case (d) Nithari Case

54. The Shunglu panel was constituted for which of the following issues?
(a) Investigate the 2G Spectrum Scam
(b) Suggest Civil Service Reforms
(c) Probe the Commonwealth Games Scam
(d) Suggest reforms on centre-state relationship

55. Who was appointed as the Chairman of the National Innovation Council in August 2010?
(a) Shashi Tharoor (b) Chetan Bhagat
(c) Arindam Chaudhury (d) Sam Pitroda

56. Name the Kenya-born political lobbyist who runs a firm called Vaishnavi Corporate Communications,
and has recently been in news?
(a) Barkha Dutt (b) Vir Sanghhvi (c) Vina Ramani (d) Niira Radia

57. Irom Sharmila has been fasting for the last 10 years to protest against which of the following issues?
(a) Rape cases against Indian Army in Manipur
(b) Emergency in Manipur
(c) Prevention of Terrorism Act 2005
(d) Application of the Armed Forces (Special Powers) Act 1958 in Manipur

58. Thein Sein is the newly-appointed President of which of the following nations?
(a) Indonesia (b) Malaysia (c) Myanmar (d) Thailand

59. Baglihar dam has been a matter of dispute between which nations?
(a) Bangladesh and Myanmar (b) Myanmar and China
(c) India and China (d) India and Pakistan

Previous Years
Page 80 CLAT & AILET Papers
60. Who is the author of the book “TINDERBOX - The Past and Future of Pakistan”?
(a) Husain Haqqani (b) Yasmeen Niaz Mohiuddin
(c) Ishrat Husain (d) M.J. Akbar

61. On 25th January 2011, BJP leaders Sushma Swaraj and Arun Jaitley were prevented from entering
Srinagar and unfurl the national flag. What was that Rath Yatra called?
(a) Swabhimaan Yatra (b) Ekta Yatra
(c) Mukti Yatra (d) Swaraj Yatra

62. Who is chairing the Joint Parliamentary Committee (JPC) on the 2G Spectrum allocation issue?
(a) Murli Manohar Joshi (b) A Raja
(c) M. Thambi Durai (d) P.C. Chacko

63. Saina Nehwal recently defeated Ji Hyun Sung of South Korea to win which of the following titles?
(a) Swiss Open Grand Prix Gold Badminton
(b) Commonwealth Games
(c) Singapore Open
(d) Hong Kong Open

64. ‘Moner Manush’, the film to win the ‘Golden Peacock’ at the 41st International Film Festival of India
was based on the life of which legendary 19th century folk singer and spiritual leader?
(a) Kabir (b) Surjya Sen (c) Kabir Suman (d) Lalan Fakir

65. Justice P.C. Phukan Commission of Inquiry was constituted to enquire into which of the following
incidents?
(a) Clashes between Nagas and Lepchas in Nagaland on 14th August 2008
(b) Clashes between Bodos and Muslims in Northern Assam’s Udalguri district on August 14th 2008
(c) Clashes between Assamese and Bengalis in Guwahati on August 14th 2008
(d) None of the above

66. The first woman Secretary General of SAARC is from which country?
(a) Maldives (b) Bhutan (c) Sri Lanka (d) India

67. Under whose premiership was the Women’s Reservations Bill (to secure quotas for women in
Parliament and state legislative assemblies) first introduced in Parliament?
(a) Rajiv Gandhi (b) HD Deve Gowda
(c) Atal Bihari Vajpayee (d) Manmohan Singh

68. Which Irish player scored the fastest Century in the history of World Cup Cricket?
(a) Niall O’ Brien (b) Kevin O’ Brien (c) A. Cusack (d) Ed Joyce

69. Which of the following report brought out the 2G spectrum scam?
(a) CBI report (b) WikiLeaks (c) CAG report (d) None of the above

70. In February 2011, Gopa Sabharwal was appointed as the first Vice Chancellor of which University of
international stature?
(a) Jawaharlal Nehru University (b) Azim Premji University
(c) Visva-Bharati University (d) Nalanda International University

Previous Years
CLAT & AILET Papers Page 81
71. With which Hindutva association are Sadhvi Pragya Singh Thakur and Swami Aseemanand allegedly
associated?
(a) Vishwa Hindu Parishad (b) Shri Ram Sena
(c) Abhinav Bharat (d) Arya Samaj

72. With which of the following do you associate the name P.J Thomas?
(a) Central Vigilance Commission (b) Lokpal
(c) Anti corruption Ombudsman (d) Banking Ombudsman

73. The 17th Commonwealth Law Conference was held in which city?
(a) Delhi (b) Bangalore (c) Kolkata (d) Hyderabad

74. Gustavo Santaolall who composed the music to the song “Stranger Lives” in the movie “Dhobi Ghat”,
is from which of the following nations?
(a) Chile (b) Peru (c) Argentina (d) Mexico

75. Which one of the following was not awarded a portion of the contested land by the judgment of the
Allahabad High Court in 2010 pertaining to the Ayodhya dispute?
(a) Nirmohi Akhada (b) Sunni Central Board of Waqfs
(c) Rashtriya Swayamsevak Sangh (d) Bhagwan Sri Ram Lala Virajman

76. Sania Mirza claimed silver in the tennis mixed doubles category in the Asian Games in Guanzhou
in November 2010. Who was her Partner?
(a) Mahesh Bhupathi (b) Somdev Devvarman
(c) Leander Paes (d) Vishnu Vardhan

77. China objected to the Dalai Lama’s recent visit to Tawang in Arunachal Pradesh. What was he
visiting in Tawang?
(a) A Buddhist monastery
(b) A memorial to Tibetans massacred by the Chinese army
(c) The residence of Panchen Lama
(d) None of the above

78. Which one of the following tribes lives in the Niyamgiri Hills, which is at the heart of the controversy
surrounding Vedanta Resources’ mining operations?
(a) Manna Dhora (b) Dongria Kondh (c) Pardhan (d) Mal Pahariya

79. Which prominent Barrister-politician, who was closely linked with the emergency proclamation of
1975, breathed his last in Kolkata on 6th November 2010?
(a) Jyoti Basu. (b) Siddhartha Shankar Ray.
(c) Hiren Mukherjee. (d) Indrajit Gupta

80. Who is the author of the book ‘Great Soul: Mahatma Gandhi and his Struggle with India’, criticised
for its content?
(a) Nirad C. Chaudhury (b) Joseph Lelyveld
(c) Khushwant Singh (d) Hermann Kallenbach

Previous Years
Page 82 CLAT & AILET Papers
81. Which Gharana of Classical singing did Late Pandit Bhimsen Joshi belong to?
(a) Dhrupad (b) Maihar (c) Kirana (d) Etawah
82. 14th March 2011 was the 80th Anniversary of the first Indian sound Film (talkie). Which Movie was
it?
(a) Jahan Ara (b) Alam Ara (c) Noorjehan (d) None of the above

83. Which internationally renowned musician collaborated with Rahul Sharma to release a music album
titled ‘Namaste India’?
(a) Richard Clayderman (b) Kenny G
(c) Yanni (d) Ricky Martin

84. “War on Terrorism or American Strategy for Global Dominance” is authored by which of the following
authors?
(a) Noam Chomsky (b) Demetrios Caraley
(c) Lea Brilmayer (d) Manzoor Alam

85. Mohammad Asif, Mohammad Amir and Salman Butt (Cricket players of Pakistan) have been banned
for being found guilty of spot fixing. To which of the following institutions have they appealed?
(a) Pakistan Cricket Board
(b) Anti-Corruption Tribunal of the Asian Cricket Council
(c) Court of Arbitration for Sport
(d) Anti-Corruption Tribunal of the International Cricket Council

86. Indian driver Karun Chandok was recently in the news for which of the following?
(a) Being selected as a reserve driver by Team Force India for the 2011 Formula One season.
(b) Being selected as a reserve driver by Team Lotus for the 2011 Formula One season.
(c) Being selected as a reserve driver by Team Ferrari for the 2011 Formula One season.
(d) None of the above.

87. The United Nations Framework Convention on Climate Change meeting of 2010 December was held
in which of the following places?
(a) Colorado. (b) Canberra. (c) Cancun. (d) None of the above.

88. Which of the following pairings is incorrect?


(a) Muammar Gaddafi – Syria. (b) Fidel Castro – Cuba.
(c) Pol Pot – Cambodia. (d) Hosni Mubarak – Egypt.

89. The Right of Children to Full and Compulsory Education Act 2009 requires private schools to ensure
that ___ percent of their students come from weaker sections and disadvantaged groups?
(a) 2 (b) 15 (c) 25 (d) 40

90. Srikrishna Committee, which recently submitted its report, was constituted for which of the following
issues?
(a) Mumbai bomb blast case (b) Malegaon blast case
(c) Telengana issue (d) 2G Spectrum corruption issue

Previous Years
CLAT & AILET Papers Page 83
MATHEMATICS
91. Akbar will turn 50 when his son Jehangir turns 18. What will be Akbar’s age when it will be exactly
5 times that of Jehangir?
(a) 36 (b) 40 (c) 44 (d) 48

92. Arun can climb a Coconut tree by 1.5 feet by each lift; however he slips 0.5 feet every time he
makes the next lift. How many individual lifts will he have to make to reach the top of the Coconut
tree of 18.5 feet?
(a) 20 (b)19 (c)18 (d)17

93. Jogen’s taxable income for 2010-11 is Rs.5,00,000. The tax rates are (i) nil for first 1,50,000, (ii) 10%
for 150,001—3,00,000, and (iii) 20% for the remaining. His Tax liability is:
(a) Rs.45,000 (b) Rs.50,000 (c) Rs.55,000 (d) Rs.60,000

94. The ratio of two numbers is 4 : 5. But, if each number is increased by 20, the ratio becomes 6 : 7.
The sum of such numbers is:
(a) 90 (b) 95 (c) 100 (d) 60

95. During the academic session 2009-10, in Banaras Hindu University, Varanasi, the number of students
studying Arts, Law and Commerce was in the ratio of 5 : 6 : 7. If during the academic session 2010-
11 the number of students studying Arts, Law and Commerce increased by 20%, 30% and 40%
respectively, what will be new ratio?
(a) 26 : 42 : 63 (b) 36 : 44 : 73 (c) 26 : 39 : 49 (d) 30 : 39 : 49

96. A customised jewellery was sold at Rs.1000 with 90% discount on the ‘making charges’. If the
payment made for making charges was Rs.100, what is the approximate rate of discount on the
product?
(a) 35 (b) 45 (c) 47 (d) 90

97. A man walks from his house to the Railway Station to catch a train, which is running as per
schedule. If he walks at 6 km/hr, he misses the train by 9 minutes. However, if he walks at 7 km/hr,
he reaches the station 6 minutes before the departure of train. The distance between his home and
the Railway Station is:
(a) 20 km (b) 15 km (c)10.5 km (d) 12.5 km

98. Difference between two numbers is 9 and difference between their squares is 981. Lowest of the two
numbers is:
(a) 40 (b) 50 (c) 55 (d) 59

99. Ms. Jhulan Goswami scores 102 runs in the 18th innings of her career and thus increases her
average by 5. After the 18th inning, her average is:
(a) 17 (b) 21 (c) 26 (d) 28

Previous Years
Page 84 CLAT & AILET Papers
100. In a staff room of 25 teachers, 13 drink black coffee, 7 drink milk coffee, 9 drink both tea and either
type of coffee, and everyone drinks either of the beverages. How many teachers drink only tea?
(a) Insufficient information (b) 5 (c) 6 (d) 9

101. A box contains 90 discs which are numbered from 1 to 90. If one disc is drawn at random from the
box, the probability that it bears a perfect square number is:

1 1 1 1
(a) (b) (c) (d)
10 11 90 9

102. Two coins are tossed simultaneously. The probability of getting at the most one head is:

1 1 3
(a) (b) (c) (d) 1
4 2 4

103. A flag pole 18 m high casts a shadow 9.6 m long. What is the distance of the top of the pole from the
far end of the shadow?
(a) 20 metres (b) 20.04 metres (c) 20.4 metres (d) 24 metres

104. The 10th term of the series: 5, 8, 11, 14, ... is:
(a) 32 (b) 35 (c) 38 (d) 185

105. A bag contains 19 red balls, 37 blue balls and 27 green balls. If a ball is picked up from this bag at
random, what is the probability of picking a blue ball?

19 37 34
(a) (b) (c) (d) None of the above
83 87 81

106. A cylindrical tennis ball container can contain maximum three balls stacked on one another. The top
and bottom balls also touch the lid and the base of the container respectively. If the volume of a
tennis ball is 240 cm3, then what is the volume of the container?
(a) 1080 cm3 (b) 840 cm3 (c)1440 cm3 (d) 720 cm3

107. Ranjeetha walks around the circular park in 15 minutes. If she walks at the rate of 5 km/hr, how
much distance would she have to travel, at the minimum, to reach the centre of the park from any
point on its perimeter?
(a) 100 metre (b) 200 metre (c) 250 metre (d) 300 metre

3 2x − 6 9
 9   49  7
108. If   x   =   , then the value of x is:
 7   81  9
(a) 12 (b) 9 (c) 8 (d) 6

Previous Years
CLAT & AILET Papers Page 85
109. Francis has 18 eggs, out of which 12 eggs were sold at 10% loss than the cost price. At what mark
up price should he sell the remaining eggs to cover his losses?
(a) 5% (b) 10% (c) 15% (d) 20%

110. If the length and height of a brick increases by 10% each respectively, and the breadth reduces by
20%, what is the percentage change in the volume of the brick?
(a) 2.8 (b) 3.0 (c) 3.2 (d) 3.6

LOGICAL REASONING

Note: The following six questions comprise of one or more statements. Please answer the questions
on the basis of the given statement(s). Please make the factual assumptions required by the
question even if you believe the statement is false.

111. ‘Where there is smoke, there is fire.’ Which of the following statements, if true, would show that the
above statement is false?
(a) There is sometimes smoke where there is no fire.
(b) There is sometimes fire where there is no smoke.
(c) There is no fire where there is no smoke.
(d) None of the above.

112. ‘Where there is poverty, there are always thieves.’ Which of the following statements, if true, would
show that the above statement is false?
(a) America is a rich country and there are thieves in America.
(b) Bhutan is a poor country and there are no thieves in Bhutan.
(c) Bangladesh is a poor country and there are many thieves in Bangladesh.
(d) Nepal is a rich country and there are no thieves in Nepal.

113. Statement 1: Sugar is bad for people with diabetes. Statement 2: Leela does not eat sugar. Assuming
that Statements 1 and 2 are true, which of the following statements follows?
(a) Leela has diabetes.
(b) Sugar is bad for Leela.
(c) People with diabetes do not eat sugar.
(d) None of the above.

114. Statement 1: People who read fashion magazines do not like to read fiction. Statement 2: Tenzin
does not read fashion magazines. Assuming that Statements 1 and 2 are true, which of the following
conclusions might be said to follow?
(a) Tenzin likes to read fiction.
(b) Tenzin may or may not like to read fiction.
(c) Tenzin does not like to read fiction.
(d) Tenzin does not like fashion magazines.

115. Suleiman: All Communists are atheists. Sheeba: That is not true. Which of the following, if true,
would make Sheeba’s reply the most convincing?
(a) My uncle is an atheist but he is not a Communist.
(b) My uncle is a Communist but he is not an atheist.
(c) My uncle is a Communist and an atheist.
(d) My uncle is neither a Communist nor an atheist.
Previous Years
Page 86 CLAT & AILET Papers
116. Statement 1: All pingos are byronic. Statement 2: Shalisto is byronic. Statement 3: Therefore
______________.
Fill in the blank.
(a) Shalisto is a pingo. (b) Shalisto is not a pingo.
(c) Shalisto is not byronic. (d) None of the above

For the following eight questions, study the sequence of letters, numbers or words carefully to
work out the pattern on which it is based, and therefore what the next item in the sequence must
be. For example, the sequence ‘A, C, E, G, __’, has odd-numbered letters of the alphabet; therefore,
the next item must be ‘I’. The meanings of the words are irrelevant.

117. A, B, D, G, K, P, ___ What is the last alphabet in this sequence?


(a) Z (b) Y (c) V (d) X

118. B, C, E, G, K, M, Q, S, __ What is the next alphabet in this sequence?


(a) T (b) U (c) V (d) W

119. Z, X, T, N, __
What is the next alphabet in this sequence?
(a) E (b) F (c) G (d) H

120. ‘Apple, Application, ______, Approval, Apricot, April’ Which of the following best fits in the blank?
(a) Arrogant (b) April (c) Appropriate (d) Apiary.

121. ‘Sunday, Monday, Wednesday, Saturday, Wednesday, Monday, Sunday, ___________’ Which of
the following best fits in the blank?
(a) Sunday (b) Tuesday (c) Saturday (d) Thursday

122. ‘387924, __________, 3724, 423, 32, 2’. Which is the missing number?
(a) 42978 (b) 42183 (c) 42983 (d) 42783

123. ‘Gym, hymn, lynx, pygmy, rhythm’ Which one of the following words does not belong to the above
set?
(a) Myrrh (b) Mythic (c) Flyby (d) Syzygy

124. Aadvark, Eerie, Iiwi, Oolong, _________


Which one of the following words follow the pattern of this series?
(a) Uvula (b) Uulium (c) Uranium (d) Uranus

For the following nine questions, read the given argument or statement carefully, making any
factual assumptions necessary. Then choose the best answer out of the four choices to the question
asked. Note that not all the facts given will be relevant for determining the answer.

125. ‘China has a higher literacy rate than India. This is due to the greater efficiency of the Communist
system. Efficiency is sorely lacking in India’s democratic system. Therefore, democracy is the
biggest obstacle to India’s achieving 100% literacy.’ Which of the following, if true, would directly
undermine the above argument?
(a) Inefficiency is equally a problem in democracies and Communist countries.
(b) Communist systems do not respect human rights.
Previous Years
CLAT & AILET Papers Page 87
(c) Freedom is more important than literacy.
(d) China is slowly making the transition to democracy.

126. ‘In 399 BC, a jury in Athens condemned Socrates to death for impiety and corrupting the morals of
the youth. Socrates’ friends offered to help him escape, but Socrates refused. Socrates argued that
the fact that he had lived in Athens for so many years meant that he had committed himself to
obeying its laws. It would therefore be wrong for him to break those very laws he was implicitly
committed to obeying.’ Which one of the following claims constitutes the most plausible challenge
to Socrates’ argument?
(a) Long residence only commits someone to obeying just laws and Socrates was convicted
under an unjust law.
(b) Long residence by itself does not imply a commitment to obeying laws since one never made
any explicit commitment.
(c) Obedience to the law is not always required.
(d) There is no point in escaping from prison since one will anyway be captured again.

127. ‘Soft drinks have been shown by scientists to be bad for the teeth. Therefore, the government would
be justified in banning all soft drinks from the Indian market.’ Assuming that the factual claim in the
above argument is true, what else needs to be assumed for the conclusion to follow?
(a) The government is justified in banning anything that is bad for dental health.
(b) Soft drinks are also bad for gastric health.
(c) Dental hygiene is a matter of great concern.
(d) No further assumptions are necessary.

128. ‘In order to be eligible for election to the Lok Sabha, a person must be at least 25 years of age.
Moreover, one must not be bankrupt. Therefore, Jatinder Singh, over 50 years of age and without any
criminal convictions, cannot be the Speaker of the Lok Sabha since he has just filed for bankruptcy.’
Which of the following must be assumed for the conclusion to follow logically?
(a) Anyone over 50 years of ago is eligible to be Speaker of the Lok Sabha as long as he or she has
no criminal convictions.
(b) People without criminal convictions cannot be elected to the Lok Sabha even if they are not
bankrupt.
(c) Only those eligible for election to the Lok Sabha are eligible to be the Speaker of the Lok Sabha.
(d) There is no minimum age requirement for the Speaker of the Lok Sabha.

129 ‘As a century draws to a close, people start behaving much like people coming to the end of a long
life. People approaching death often start reflecting on the events of their lives. Similarly, people
alive in 1999 ______’ Which of the following most logically completes the paragraph above?
(a) ... started reflecting on the events of the twentieth century.
(b) ... started to reflect on the events of their lives.
(c) ... started to fear death.
(d) ... started to wonder what the year 2000 would bring.

130. ‘The Roman poet and philosopher Lucretius proposed the following thought experiment. If the universe
has a boundary, we can throw a spear at this boundary. If the spear flies through, then it is not a
boundary. If the spear bounces back, there must be something beyond this boundary that is itself in
space, which means it is not a boundary at all. Either way, it turns out that the universe has no
boundary.’ How best can the form of Lucretius’ argument for the infinity of space be described?

Previous Years
Page 88 CLAT & AILET Papers
(a) Lucretius shows that positing finite space leads to a contradiction.
(b) Lucretius shows that finite space is inconsistent with physics.
(c) Lucretius shows that finite space is inconsistent with mathematics.
(d) Lucretius shows that finite space is inconceivable.

131. ‘Utilitarians believe that the right action is that which produces the most happiness.’ Which of the
following claims is incompatible with the utilitarian view?
(a) The right thing to do is to make the consequences of our actions as good as possible.
(b) The right thing to do is to do our duty, whatever the consequences.
(c) The right thing to do is to act on a rule which, if followed widely, produces the most happiness.
(d) The right thing to do is to act from motivations which produce the most happiness.

132. ‘Senthil goes to Ambala for the first time in his life. On the way from the railway station to his hotel,
he sees twelve people, all of them male. He concludes that there are no women in Ambala. As a
matter of fact, there are many thousands of women in Ambala.’ Which of the following best describes
Senthil’s error?
(a) Senthil was misled by irrelevant details.
(b) Senthil generalised on the basis of insufficient evidence.
(c) Senthil was biased against women.
(d) Senthil was bad at counting.

133. ‘It took many centuries before the countries of Europe could resolve their internal problems of
violence and corruption to become the stable nation-states they are. Therefore, it will take many
centuries for India to achieve internal stability.’ Assuming that the factual claims in the argument
above are true, what must be assumed in order for the conclusion to follow?
(a) Countries everywhere must follow the same paths towards stability.
(b) India is gradually progressing towards internal stability.
(c) Indian can learn how to achieve stability by studying Europe’s example.
(d) India has no hope of achieving stability in the near future.

In the following three questions, assume that a ‘fact’ expresses something that can be proved by
clear and objective data. An opinion expresses a judgment, view, attitude, or conclusion that is
not backed by data.

134. Which one of the following statements is best described as an assertion of opinion rather than an
assertion of fact?
(a) Brazil, China and India are now among the largest emitters of greenhouse gases.
(b) Scientists agree that human activity is an important cause of climate change.
(c) The Indian government’s policy on climate change is misguided.
(d) The Indian government’s policy on climate change has changed significantly in the last five
years.

135. ‘(A) The number of people migrating into Bengaluru has increased significantly in recent years. (B)
This is because Bengaluru provides more economic opportunities than the towns and villages from
which these migrants come. (C) This sudden influx of migrants has made the city less pleasant to
live in. (D) The success of the government’s rural employment guarantee act might have the effect of
stemming some rural-urban migration.’ Which one of the above statements is best described as an
assertion of opinion rather than an assertion of fact?
(a) A (b) B (c) C (d) D
Previous Years
CLAT & AILET Papers Page 89
136. Which one of the following statements, if true, is best described as an assertion of opinion rather
than an assertion of fact?
(a) Mumbai is larger than Pune. (b) Mumbai is smaller than Pune.
(c) Mumbai is more cultured than Pune. (d) Mumbai is more crowded than Pune.

For the following three questions, there are two or more statements along with few conclusions
deduced from the statements. You are required to answer on the basis of the statements and the
conclusions. Make the factual assumptions required by the question even if you believe the
statement is actually false.

137. Statement 1: Some aeroplanes are balloons.


Statement 2: Some balloons are rockets.

Conclusions:
(A) Some aeroplanes are rockets.
(B) Some rockets are tables.
(C) All the rockets are balloons.
(D) All the balloons are aeroplanes. Assuming that statements 1 and 2 are true, which conclusions
follow:

(a) Only (B) and (D) (b) Only (A) and (C) (c) Only (D) (d) None of the above.

138. Statement 1: All whales are fish.


Statement 2: Some fish are not amphibians.
Statement 3: All whales are amphibians.
Statement 4: Some amphibians are not fish.

Conclusions:
(A) Some fish are amphibians.
(B) Some amphibians are fish.
(C) Only whales are both fish and amphibians.
(D) All amphibians are fish. Assuming only that Statements 1, 2, 3 and 4 are true, which of the
above conclusions may be deduced?

(a) Only (A) and (B). (b) Only (C). (c) Only (D). (d) None of the above.

139. Statement 1: All libraries are laboratories.


Statement 2: No laboratories are hostels.

Conclusions:
(A) All laboratories are libraries.
(B) Some hostels are libraries.
(C) Some libraries are hostels.
(D) No library is a hostel. Assuming that statements 1 and 2 are true, which conclusions follow?

(a) Only (A) and (B) (b) Only (B) and (C) (c) Only (C) and (D) (d) Only D

Previous Years
Page 90 CLAT & AILET Papers
For the following sixteen questions, read the given passage carefully and answer the questions
that follow.

Passage for questions 140-142


‘Instead of being concerned with what actually happens in practice ... [economics] is increasingly
preoccupied with developing pseudo-mathematical formulas. These provide models of behaviour which
never quite fit what actually happens, in a way which resembles the physical sciences gone wrong: instead
of equations describing reality, economics produces equations describing ideal conditions and theoretical
clarity of a type which never occurs in practice’.

140. Which of the following best summarises the argument of this paragraph?
(a) Economics ought to be more like the physical sciences.
(b) Theoretical clarity is undesirable in economics.
(c) The physical sciences are wrong to emphasise mathematic formulae.
(d) The mathematical equations used by economists do not accurately describe the real world.

141. Which of the following claims is not implied in the paragraph above?
(a) Economists should stop using mathematical models.
(b) Equations describing ideal conditions should not be mistaken for equations describing reality.
(c) Theoretical clarity should not come at the expense of accuracy.
(d) Models of human behaviour should be true to the complexity of human nature.

142. Which of the following, if true, would most weaken the argument of the passage above?
(a) The physical sciences are themselves reducing their reliance on mathematical formulae.
(b) The real world in fact closely approximates ideal theoretical conditions.
(c) We do not at present have the mathematical expertise to model the full complexity of the world
economy.
(d) Academic economists need to be more sensitive to human nature.

Passage for questions 143-144


‘Religions, like camel caravans, seem to avoid mountain passes. Buddhism spread quickly south from
Buddha’s birth-place in southern Nepal across the flat Gangetic plain to Sri Lanka. But it took a millennium
to reach China ... The religious belt stretched eventually to Mongolia and Japan, but in Afghanistan Buddhism
filled only a narrow belt that left pagans among the valleys to the east and west in Kailash and Ghor’.

143. Which of the following best summarises the subject of this paragraph?
(a) The Afghan people were hostile to Buddhism.
(b) Geography has considerable impact on the spread of religions.
(c) Buddhism does not flourish in mountainous regions.
(d) Religion has a considerable impact on geography.

144. Which of the following, if true, would most weaken the conclusion to the above argument?
(a) Christianity took several centuries to cross the Atlantic Ocean to America.
(b) The Hindukush mountains made no difference to the speed with which Islam spread.
(c) Buddhism is strongest in mountainous regions.
(d) Jainism is less popular in cold climates.

Previous Years
CLAT & AILET Papers Page 91
Passage for questions 145-146 ‘Friendship was indeed a value for the villagers, more for men than for
women. Two good friends were said to be ‘like brothers’ (literally, ‘like elder brother-younger brother’,
annatammandirahage). I heard this expression several times and I could not help recalling the statement of
an elderly English colleague who had told me that he and his brother were very close and had written to
each other every week. He had added, ‘We are very good friends.’ That is, friendship connoted intimacy in
England while in Rampura (as in rural India everywhere), brotherhood conveyed intimacy’.

145. Which of the following best summarises the conclusion of the argument of this paragraph?
(a) Friendship has greater value for men than for women.
(b) People in England have different attitudes to brotherhood and friendship than people in rural
India.
(c) Brotherhood has greater value in rural India than in England.
(d) Friendship has greater value in England than in India.

146. Which of the following, if true, would directly contradict the conclusion of the above argument?
(a) People are less likely to have large families in England.
(b) People in England are no longer close to their family members.
(c) People in England do not think that friendship connotes intimacy.
(d) People in rural India think that sisters cannot be intimate.

Passage for questions 147-148


‘A language is most easily learnt when it is in tune with the social context. To teach an Indian child in
English at the primary stage ... strengthens distinctions of class and status and warps the mind. Failure to
resort to regional languages in literacy campaigns also hampers their success’.

147. Which of the following best summarises the argument of the passage above?
(a) Learning English warps the mind.
(b) Language policy should be devised with an eye to social context.
(c) Literacy campaigns in India have failed.
(d) English should be taught at the secondary school level.

148. Which of the following claims, if true, would weaken the argument in the passage above?
(a) Literacy campaigns are more successful when conducted in English.
(b) Learning English at an early age has been found to weaken class distinctions.
(c) Children who learn English at the primary-school stage have been found to show greater intellectual
ability than those who learn English only at the secondary level.
(d) All of the above.

Passage for questions 149-151


‘The tribes should develop their own culture and make their contribution to the cultural richness of the
country. It is unnecessary to cause them to change their customs, habits or diversions so far as to make
themselves indistinguishable from other classes. To do so would be to rob rural and pastoral life of its
colour and stimulating diversity’.

149. Which of the following conclusions is not implied by the passage above?
(a) It is good for India’s tribal people to develop their culture.
(b) Tribal customs should not be allowed to change in any respect.
(c) Forcing tribal people to change their customs reduces the diversity of rural life.
(d) Tribal culture is part of the cultural richness of India.
Previous Years
Page 92 CLAT & AILET Papers
150. Which of the following claims runs directly counter to the spirit of the passage above?
(a) Tribal people should be able to decide what elements of the modern world to adopt.
(b) The government should make modern science and medicine available to tribal people.
(c) Tribal people should not be subjected to any coercion to conform to non-tribal cultural norms as
long as they do not violate the law.
(d) The tribals should assimilate as far as possible into non-tribal culture as a condition of full
citizenship.

151. Which of the following is not an assumption required by the above argument?
(a) Colour and diversity are desirable things.
(b) Tribal people are capable of contributing to India’s cultural diversity.
(c) Changing tribal customs is a necessary condition of making modern medicine available to tribal
people.
(d) Rural life is presently full of colour and diversity.

Passage for questions 152-153


‘India is the only country in the world where, in the States which are governed by the Communist party,
human rights are fully respected — and that is only because the Bill of Rights is firmly entrenched in our
national Constitution. We can proudly say that our Constitution gave us a flying start and equipped us
adequately to meet the challenges of the future.’

152. Which of the following conclusions is not implied by the passage above?
(a) Communist states often do not respect human rights.
(b) Communist states never respect human rights.
(c) The entrenchment of the Bill of Rights is what ensures that even Communist-governed states
respect human rights.
(d) The Indian Constitution prepared India for the challenges of independence.

153. Which of the following, if true, would weaken the above argument?
(a) Communist governments are motivated to respect human rights out of Communist principles,
not Constitutional ones.
(b) The Constitution of India is itself sympathetic to Communist principles.
(c) Human rights need to be enforceable to have any meaning.
(d) Countries without a strong culture of human rights are prone to oppressing minorities.

Passage for questions 154-155


‘Poverty is ... more restrictive and limiting than anything else. If poverty and low standards continue then
democracy, for all its fine institutions and ideals, ceases to be a liberating force. It must therefore aim
continuously at the eradication of poverty and its companion unemployment. In other words, political
democracy is not enough. It must develop into economic democracy also’.

154. Which of the following is not implied by the above passage?


(a) Democracy has ceased to be a liberating force.
(b) Democracies should aim to eliminate poverty.
(c) Poverty and unemployment go hand in hand.
(d) Political democracy should develop into economic democracy.

Previous Years
CLAT & AILET Papers Page 93
155. Which of the following views, if true, would weaken the argument of the above passage?
(a) Political democracy is inseparable from economic democracy.
(b) Poverty does not in fact restrict freedom.
(c) Democracy flourishes most in poor societies.
(d) Economic democracy is a necessary condition for the elimination of unemployment.

LEGAL APTITUDE
Instructions to Candidates:

This section consists of ten problems (with 45 questions) in total. Each problem consists of a set of rules
and facts. Apply the specified rules to the set of facts and answer the questions.

In answering the following questions, you should not rely on any rule(s) except the rule(s) that are supplied
for every problem. Further, you should not assume any fact other than those stated in the problem. The aim
is to test your ability to properly apply a rule to a given set of facts, even when the result is absurd or
unacceptable for any other reason. It is not the aim to test any knowledge of law you may already
possess.

Problem 1 (For questions 156 to 160)

Rules
A. The fundamental right to freedom of association includes the right to form an association as well as
not join an association.
B. The fundamental right to freedom of association also includes the freedom to decide with whom to
associate.
C. The fundamental right to freedom of association does not extend to the right to realise the objectives
of forming the association.
D. Fundamental rights are applicable only to laws made by or administrative actions of the State and
do not apply to actions of private persons.
E. Any law in contravention of fundamental rights is unconstitutional and therefore cannot bind any
person.

Facts Gajodhar Pharmaceuticals, a private company, offered an employment contract of two years to
Syed Monirul Alam. One of the clauses in the employment contract provided that Syed Monirul
Alam must join Gajodhar Mazdoor Sangh (GMS), one of the trade unions active in Gajodhar
Pharmaceuticals.

156. Decide which of the following propositions can be most reasonably inferred through the application
of the stated legal rules to the facts of this case:
(a) The employment contract offered to Monirul Alam to join GMS is legal as it does not restrict his
freedom not to join any association.
(b) The condition requiring Monirul Alam to join GMS cannot bind him as it impinges on his freedom
not to join any association.
(c) Syed Monirul Alam cannot claim a fundamental right to freedom of association against Gajodhar
Pharmaceuticals and therefore, the contract would bind him even though his freedom of association
is restricted.
(d) The employment contract infringes Syed Monirul Alam’s freedom to decide with whom to associate
and therefore is legally not enforceable.

Previous Years
Page 94 CLAT & AILET Papers
157. If Parliament enacts a law which requires every employee to join the largest trade union in their
workplace mandating Syed Monirul Alam to join GMS, then:
(a) Such a law would merely govern private action to which fundamental rights do not apply.
(b) Such a law would not curtail any individual’s right to freedom of association.
(c) Neither the employment contract, nor the law of the parliament would be enforceable as they
would curtail the freedom of association.
(d) The law of parliament would violate an individual’s freedom not to join any association and
therefore be unconstitutional.

158. If Parliament enacts a law that requires a trade union to open its membership to all the employees,
then
(a) Such a law would not infringe any fundamental right to freedom of association.
(b) The law of the parliament would curtail an individual’s right not to join any association.
(c) Such a law would curtail the union members’ right to decide with whom they would like to
associate.
(d) Such a law would render the employment contract offered by Gajodhar Pharmaceuticals to
Syed Monirul Alam unenforceable.

159. If Gajodhar Pharmaceuticals enter into an agreement with GMS wherein the former agrees to hire
only the existing members of GMS as employees, then:
(a) The agreement would be illegal as it would curtail the union members’ right to decide with whom
they would like to associate.
(b) Such an agreement would infringe the union’s right to decide with whom to associate and
therefore is legally not enforceable.
(c) The agreement would not be enforceable as it would infringe upon the employer’s right not to join
an association.
(d) The constitutionality of this agreement cannot be contested on grounds of contravention of
fundamental rights as such rights are not applicable to private persons.

160. If Parliament enacts a legislation prohibiting strikes by trade unions of employees engaged in
pharmaceutical industry, then:
(a) The legislation would not violate the right to freedom of association.
(b) The legislation would curtail the right of trade unions to strike, and therefore violate freedom of
association.
(c) Since strike is only one of the objectives with which a trade union is formed, right to strike is not
protected by the right to freedom of association.
(d) None of the above

Problem 2 (For questions 161 to 163)

Rule: Whoever finds an unattended object can keep it unless the true owner claims that object. This does
not affect the property owner’s right to the ownership of the property on which the object is found.
The right to ownership of a property does not include the right to ownership of unattended objects on
that property.

Facts: Elizabeth is the CEO of a global management services company in Chennai and is on her way to
Ranchi to deliver the convocation address at India’s leading business school on the outskirts of
Ranchi. Flying business class on Dolphin Airlines, she is entitled to use the lounge owned by the
airline in Chennai Airport while waiting for her flight. She finds a diamond ear-ring on the floor of the
Previous Years
CLAT & AILET Papers Page 95
lounge and gives it to the staff of Dolphin Airlines expressly stating that in the event of nobody
claiming the ear-ring within six months, she would claim it back. The airline sells the ear-ring after
eight months and Elizabeth files a case to recover the value of the ear-ring from the airline when she
is informed about its sale.
161. As a judge you would order that:
(a) Elizabeth is not entitled to compensation because the ear-ring was found on the property of the
airline and therefore, the airline is entitled to sell it.
(b) The airline must compensate Elizabeth because owning the lounge does not give the airline the
right over all things that might be found on it.
(c) The airline must compensate Elizabeth because while accepting the ear-ring from Elizabeth
they had agreed to return it if nobody claimed it within six months.
(d) Elizabeth is not entitled to compensation because she did not claim the ear-ring after the expiry
of six months and the airline waited for a couple more months before selling it.

162. Assume now that Elizabeth was only an economy class passenger and was not entitled to use the
airline’s lounge. However, she manages to gain entry and finds the ear-ring in the lounge. The rest of
the above facts remain the same. Will her illegal entry into the Lounge affect Elizabeth’s right to
keep the ear-ring (or be compensated for its value)?
(a) Yes, the airline claims that Elizabeth’s entry into the lounge was illegal and therefore she has no
right over anything she found there
(b) No, because Elizabeth’s class of travel has no bearing on the outcome in this case
(c) Cannot be determined as we need to know how Elizabeth was able to access the airline’s
lounge.
(d) None of the above.

163. To the original fact scenario, the following fact is added: In the lounge there are numerous signboards
which proclaim ‘Any unattended item will be confiscated by Dolphin Airlines’. In this case, you
would:
(a) Order the airline to pay compensation to Elizabeth because the board in the lounge cannot grant
property rights over unattended objects to the airline.
(b) Deny Elizabeth compensation because the signboard makes it evident that the airline, as owner
of the lounge, is exercising all rights over all unattended items in the lounge and the ear-ring is
one such item.
(c) Deny Elizabeth compensation because she knew any unattended item belonged to the airline.
(d) Order the airline to pay compensation to Elizabeth because the property rights of the airline are
relevant only if the item is unattended. The moment Elizabeth found the earring, it belonged to
her.

Problem 3 (For questions 164 to 168)

Rules
A: The State shall not discriminate, either directly or indirectly, on the grounds of sex, race, religion,
caste, creed, sexual orientation, marital status, disability, pregnancy, place of birth, gender orientation
or any other status.
B: Direct discrimination occurs when for a reason related to one or more prohibited grounds a person
or group of persons is treated less favourably than another person or another group of persons in a
comparable situation.

Previous Years
Page 96 CLAT & AILET Papers
C: Indirect discrimination occurs when a provision, criterion or practice which is neutral on the face of
it would have the effect of putting persons having a status or a characteristic associated with one or
more prohibited grounds at a particular disadvantage compared with other persons.
D: Discrimination shall be justified when such discrimination is absolutely necessary in order to promote
the well-being of disadvantaged groups, such as women, dalits, religious minorities, sexual minorities
or disabled persons.

Facts: On 2nd October 2010, the Governor of the state of Bihar ordered the release of all women prisoners
who were serving sentence of less than one year imprisonment to mark the occasion of Mahatma
Gandhi’s birthday.

164. Which of the following is correct with respect to the Governor’s order?
(a) It discriminates directly on the ground of sex.
(b) It discriminates indirectly on the ground of sex.
(c) It does not discriminate on the ground of sex.
(d) It discriminates directly as well as indirectly on the ground of sex.

165. Is the governor’s order justified under Rule D?


(a) Yes, because it is for the well-being of women prisoners.
(b) No, because it is not absolutely necessary for the well-being of women prisoners.
(c) No, because it does not promote the well-being of women prisoners or the society.
(d) None of the above

166. Assume that the Governor also made a second order requiring the release of all persons under the
age of 25 and over the age of 65 who were serving a sentence of less than one year’s imprisonment.
Under the Rules, this order is:
(a) Directly discriminatory.
(b) Indirectly discriminatory.
(c) Not discriminatory.
(d) Discriminatory, but justifiable.

167. Assume further that the government made a third order, releasing all graduate prisoners who are
serving a sentence of less than one year’s imprisonment. Which of the following statistics would
have to be true for this order to be indirectly discriminatory?
(a) Only 13% of the prison population in Bihar have a graduation degree.
(b) Of the graduate prisoners, 89% belong to upper castes.
(c) Only 25% women in Bihar get a graduation degree.
(d) All of the above.

Rule E:
‘A discriminatory act shall be justified if its effect is to promote the well-being of disadvantaged groups,
such as women, dalits, religious minorities, sexual minorities or disabled persons.’

168. Would the first Order of release of all women prisoners be justified under Rule E?
(a) Yes, because it promotes the well-being of women.
(b) No, because it does not promote the well-being of women prisoners
(c) No, because it does not promote the well-being of all disadvantaged groups equally.
(d) None of the above.

Previous Years
CLAT & AILET Papers Page 97
Problem 4 (For questions 169 to 173)
Rules: A. A minor is a person who is below the age of eighteen. However, where a guardian administers the
minor’s property the age of majority is twenty-one. B. A minor is not permitted by law to enter into a
contract. Hence, where a minor enters into a contract with a major person, the contract is not enforceable.
This effectively means that neither the minor nor the other party can make any claim on the basis of the
contract. C. In a contract with a minor, if the other party hands over any money or confers any other benefit
on the minor, the same shall not be recoverable from the minor unless the other party was deceived by the
minor to hand over money or any other benefit. The other party will have to show that the minor misrepresented
her age, he was ignorant about the age of the minor and that he handed over the benefit on the basis of
such representation.

Facts Ajay convinces Bandita, a girl aged 18 that she should sell her land to him. Bandita’s mother Chaaru
is her guardian. Nonetheless Bandita, without the permission of Chaaru, sells the land to Ajay for a total
sum of rupees fifty lakh, paid in full and final settlement of the price. Chaaru challenges this transaction
claiming that Bandita is a minor and hence the possession of the land shall not be given to Ajay. Thus Ajay
is in a difficult situation and has no idea how to recover his money from Bandita.

169. Chaaru is justified in challenging the sale transaction because:


(a) Bandita is of unsound mind and is not in a position to make rational decisions.
(b) Though Bandita is eighteen year old, she will be treated as a minor, as Chaaru is her guardian.
(c) Though Bandita is eighteen year old, she cannot sell the land without the permission of her
mother.
(d) Though Bandita is eighteen year old she should not be treated like a person who has attained
the age of majority.

170. Ajay can be allowed to recover the money only if he can show that:
(a) He was deceived by Bandita who misrepresented her age.
(b) He honestly believed that Bandita was empowered under the law to sell the land.
(c) He was an honest person who had paid the full price of the land to Bandita.
(d) Both (a) and (b).

171. In order to defend the sale, Bandita will need to show that
(a) Bandita has attained the age of majority.
(b) Bandita is mature enough to make rational decisions regarding her own affairs.
(c) The sale transaction was beneficial to her interest and will enhance her financial status.
(d) None of the above.

172. Which of the following is correct?


(a) Ajay should be allowed to recover the money because even though there is no contract, Bandita
and Chaaru should not be allowed to unjustly benefit from Ajay’s money.
(b) Ajay should be allowed the possession of the land because Chaaru can always decide to
approve the transaction between Ajay and Bandita.
(c) Ajay should not be allowed to recover because he induced Bandita, a minor, to sell the land.
(d) None of the above.

Previous Years
Page 98 CLAT & AILET Papers
173. Which of the following is correct?
(a) If Ajay is allowed to recover the money, that will defeat the law framed for protecting the minors
against fraudulent persons.
(b) If Ajay is not allowed to recover that will cause him injustice as he has not paid off the entire sale
price.
(c) If Ajay is allowed to recover, Chaaru will benefit from both the money and the land.
(d) None of the above.

Problem 5 (For questions 174 to 177)


Rules:
A. The act of using threats to force another person to enter into a contract is called coercion.
B. The act of using influence on another and taking undue advantage of that person is called undue
influence.
C. In order to prove coercion, the existence of the use of threat, in any form and manner, is necessary.
If coercion is proved, the person who has been so threatened can refuse to abide by the contract.
D. In order to prove undue-influence, there has to be a pre-existing relationship between the parties to
a contract. The relationship has to be of such a nature that one is in a position to influence the other.
If it is proven that there has been undue influence, the party who has been so influenced need not
enforce the contract or perform his obligations under the contract.

Facts
Aadil and Baalu are best friends. Aadil is the son of multi millionaire business person, Chulbul who owns
Maakhan Pharmaceuticals. Baalu is the son of a bank employee, Dhanraj. One day, Aadil is abducted
from his office by Baalu. Chulbul receives a phone call from Dhanraj telling him that if he does not make
Baalu the CEO of Maakhan Pharmaceuticals, Aadil will be killed. Chulbul reluctantly agrees to make the
Baalu the CEO. Subsequently Chulbul and Baalu sign an employment contract. However as soon as Aadil
is released and safely returns home, Chulbul tells Baalu that he shall not enforce the employment contract.
Baalu and Dhanraj are not sure as to what is to be done next.

174. As per the rules and the given facts, who coerces whom:
(a) Aadil coerces Baalu.
(b) Baalu coerces Chulbul.
(c) Dhanraj coerces Chulbul
(d) None of the above.

175. In the above fact situation:


(a) There is undue influence exercised by Dhanraj on Baalu.
(b) There is undue influence exercised by Aadil on Chulbul.
(c) There is no undue influence.
(d) None of the above.

176. Chulbul is:


(a) Justified in refusing to enforce the employment contract as Chulbul was coerced by Dhanraj.
(b) Justified in refusing to enforce the employment contract as Baalu was complicit in the coercive
act
(c) Not justified in refusing to enforce the employment contract as Baalu was an innocent person
and has not coerced Chulbul.
(d) Both (a) and (b).

Previous Years
CLAT & AILET Papers Page 99
177. Baalu will succeed in getting the employment contract enforced if he can show that
(a) He is the best friend of Aadil
(b) It was his father, and not he, who used coercion against Chulbul.
(c) Chulbul has promised his father to employ him.
(d) None of the above.

Problem 6 (For questions 178 to 181)


Rule A:
When a State undertakes any measure, the effects of the measure must be the same for all those who are
affected by it.
Facts
100 mountaineers embarked on an extremely risky climbing expedition in Leh. Weather conditions worsened
five days into the expedition and the mountaineers are trapped under heavy snow. The government received
information of this tragedy only two weeks after the unfortunate incident and has only 24 hours in which to
send rescue helicopters. Weather stations across the world confirm that this particular region of Leh will
experience blizzards of unprecedented intensity for almost two weeks after this 24 hour window rendering
any helicopter activity in the region impossible and certain death for anyone left behind. The government
has only five rescue helicopters with a maximum capacity of 50 people (excluding pilots and requisite
soldiers) and these helicopters can fly only once in 24 hours to such altitudes.

As the Air Force gets ready to send the helicopters, an emergency hearing is convened in the Supreme
Court to challenge this measure as this would leave 50 people to die.

178. If you were the judge required to apply Rule A, you would decide that:
(a) As many lives must be saved as possible.
(b) If everyone cannot be rescued, then everyone must be left behind.
(c) A measure cannot be upheld at the cost of 50 lives.
(d) It must be left to those who are trapped to decide if they want half amongst them to be saved and
leave the rest to die.
Rule B:
When a State undertakes any measure, everyone affected must have an equal chance to benefit from it.
179. As the government prepares to send in rescue helicopters, which option would be acceptable only
under Rule B and not Rule A:
(a) A lottery to choose the 50 survivors excluding those diagnosed with terminal illnesses from
participating in the lottery.
(b) A lottery to decide the 50 survivors with single parents of children below five years of age
automatically qualifying to be rescued.
(c) The 50 youngest people should be rescued.
(d) None of the above.

180. Choosing 50 survivors exclusively by a lottery would be:


(a) Permissible under Rules A and B.
(b) Impermissible under Rule A and B.
(c) Permissible only under Rule B.
(d) Permissible only under Rule A.

Previous Years
Page 100 CLAT & AILET Papers
181. If the government decides that it will either save everyone or save none, it would be:
(a) Permissible under Rules A and B.
(b) Impermissible under Rules A and B.
(c) Permissible only under Rule A.
(d) Permissible only under Rule B.

Problem 7 (For questions 182 to 186)

Rules:
A. A person is an employee of another if the mode and the manner in which he or she carries out his
work is subject to control and supervision of the latter.
B. An employer is required to provide compensation to his or her employees for any injury caused by
an accident arising in the course of employment. The words ‘in the course of the employment’ mean
in the course of the work which the employee is contracted to do and which is incidental to it.

Facts:
Messrs. ZafarAbidi and Co. (Company) manufactures bidis with the help of persons known as ‘pattadars’.
The pattadars are supplied tobacco and leaves by the Company and are required to roll them into bidis and
bring the bidis back to the Company. The pattadars are free to roll the bidis either in the factory or anywhere
else they prefer. They are not bound to attend the factory for any fixed hours of work or for any fixed number
of days. Neither are they required to roll up any fixed number of bidis. The Company verifies whether the
bidis adhere to the specified instructions or not and pays the pattadars on the basis of the number of bidis
that are found to be of right quality. Aashish Mathew is one of the pattadars of the Company. He was hit by
a car just outside the precinct of the factory while he was heading to have lunch in a nearby food-stall.
Aashish Mathew has applied for compensation from the Company.

182. Which of the following statements can most plausibly be inferred from the application of the rules to
the given facts:

(a) Aashish Mathew is an employee of the Company because the latter exercises control over the
manner in which Aashish Mathew carries out his work.
(b) Aashish Mathew is not an employee but an independent contractor as he does not have a fixed
salary.
(c) Aashish Mathew is an employee because the Company exercises control over the final quality
of the bidis.
(d) Verification of the quality of bidis amounts to control over the product and not control over the
mode and method of work and therefore, Aashish Mathew is not an employee of the Company.

183. In case the pattadars were compulsorily required to work in the factory for a minimum number of
hours every day, then it would be correct to state that:

(a) The injury was not caused by an accident in the course of employment
(b) Aashish Mathew would not be an employee as the Company would have still not exercised
control over the manner of work
(c) The injury suffered by Aashish Mathew could not be held to be one caused by an accident.
(d) Stipulations on place and hours of work relate to manner and mode of work and therefore,
Aashish Mathew would be held to be an employee of the Company.

Previous Years
CLAT & AILET Papers Page 101
184. According to the facts and the rules specified, which of the following propositions is correct:
(a) The Company is not liable to pay compensation as the injury to Aashish Mathew was not
caused by an accident arising in the course of employment.

(b) The Company is liable to pay the compensation.


(c) Since the injury did not arise in the course of employment, the Company would not be liable to
pay the compensation even though Aashish Mathew is an employee of the company.
(d) The Company is liable to pay the compensation as Aashish Mathew is a contracted pattadar
with the company.

185. Select the statement that could be said to be most direct inference from specified facts:

(a) The injury to Aashish Mathew did not arise in the course of employment as he was not rolling
bidis at the time when he was hit by the car.
(b) Since the Ashish Mathew is a contracted pattadar with the Company, it shall be presumed that
the injury was caused by an accident in the course of employment.
(c) Since there was no relationship of employment between Aashish Mathew and the Company, the
injury suffered by Aashish Mathew could not be held to be one arising in ‘the course of employment’
notwithstanding the fact that the concerned injury was caused while he was involved in an
activity incidental to his duties.
(d) As the concerned injury was caused to Aashish Mathew while he was involved in an activity
incidental to his duties, the injury did arise in the course of employment.

186. If the pattadars were compulsorily required to work in the factory for a minimum number of hours
every day, then the Company would have been liable to pay compensation to Aashish Mathew if the
latter:

(a) Had been assaulted and grievously hurt by his neighbour inside the factory precincts over a
property dispute.
(b) Had slipped and fractured his arm while trying to commute on a city bus from his home to the
factory.
(c) Had been injured while commuting on a bus provided by the Company and which he was required
by his contract to use every day.
(d) Had been caught in the middle of a cross-fire between police and a gang of robbers while
travelling to work on a city bus.

Problem 8 (For questions 187 to 191)


Rules:
A. Whoever intending to take any moveable property out of the possession of any person without that
person’s consent, moves that property out of his or her possession, is said to commit theft.
B. A person who, without lawful excuse, damages any property belonging to another intending to
damage any such property shall be guilty of causing criminal damage.
C. Damage means any impairment of the value of a property.

Facts:
Veena, an old lady of 78 years, used to live with her granddaughter Indira. Veena was ill and therefore bed-
ridden for several months. In those months, she could not tolerate any noise and it became quite difficult to
clean her room. After she died, Indira hired a cleaner, Lucky, to clean the room and throw away any rubbish
that may be there.

Previous Years
Page 102 CLAT & AILET Papers
There was a pile of old newspapers which Veena had stacked in a corner of her room. Lucky asked Indira if
he should clear away the pile of old newspapers, to which she said yes. Lucky took the pile to a municipality
rubbish dump. While Lucky was sorting and throwing away the newspapers, he was very surprised to find a
beautiful painting in between two sheets of paper. He thought that Indira probably wouldn’t want this old
painting back, especially because it was torn in several places and the colour was fading. He took the
painting home, mounted it on a wooden frame and hung it on the wall of his bedroom. Unknown to him, the
painting was an old masterpiece, and worth twenty thousand rupees. Before mounting the painting, Lucky
pasted it on a plain sheet of paper so that it does not tear any more. By doing so, he made its professional
restoration very difficult and thereby reduced its value by half.

Lucky’s neighbour Kamala discovered that the painting belonged to Indira. With the motive of returning the
painting to Indira, Kamala climbed through an open window into Lucky’s room when he was away one
afternoon and removed the painting from his house.

187. Has Lucky committed theft?


(a) Yes, Lucky has committed theft of the newspapers and the painting.
(b) No, Lucky has not committed theft because he had Veena’s consent.
(c) Yes, Lucky has committed theft of the painting, but not of the newspapers.
(d) No, Lucky has not committed theft because he has not moved the painting out of Veena’s
possession.

188. Is Lucky guilty of criminal damage?


(a) No, Lucky is not guilty of criminal damage as he did not intentionally impair the value of the
painting.
(b) Yes, Lucky is guilty of criminal damage as he intentionally stuck the paper on to the painting.
(c) No, Lucky is not guilty of criminal damage as he does not have the painting in his possession
anymore.
(d) No, Lucky is not guilty of criminal damage as he has not destroyed the painting.

189. If Lucky had discovered the painting before leaving Indira’s house rather than at the rubbish dump,
would he have been guilty of theft in this case?
(a) Yes, he would be guilty of theft of the newspapers and the paintings.
(b) No, he would not be guilty of theft.
(c) Yes, he would be guilty of theft of the painting.
(d) None of the above.

190. Is Kamala guilty of theft?


(a) No, Kamala is not guilty of theft since the person she took the painting from (Lucky) was not its
lawful owner.
(b) No, Kamala is not guilty of theft since she took the painting only with the motive of returning it to
Indira.
(c) Yes, Kamala is guilty of theft as she took the painting out of Lucky’s possession without his
consent.
(d) None of the above

Previous Years
CLAT & AILET Papers Page 103
191. Which of the following propositions could be inferred from the facts and the rules specified,
(a) Kamala is guilty of criminal damage as the person she took the painting from (Lucky) was not its
lawful owner.
(b) Kamala is guilty of criminal damage as she took the painting without Lucky’s consent.
(c) Kamala is not guilty of criminal damage as the painting has not been completely destroyed.
(d) None of the above.

Problem 9 [For questions 192 to 195]


Rules:
A. When land is sold, all ‘fixtures’ on the land are also deemed to have been sold.
B. If a moveable thing is attached to the land or any building on the land, then it becomes a ‘fixture’.

Facts:
Khaleeda wants to sell a plot of land she owns in Baghmara, Meghalaya and the sale value decided for the
plot includes the fully-furnished palatial six-bedroom house that she has built on it five years ago. She sells
it to Gurpreet for sixty lakh rupees. After completing the sale, she removes the expensive Iranian carpet
which used to cover the entire wooden floor of one of the bedrooms. The room had very little light and
Khaleeda used this light-coloured radiant carpet to negate some of the darkness in the room. Gurpreet,
after moving in, realises this and files a case to recover the carpet from Khaleeda.

192. As a judge you would decide in favour of:


(a) Gurpreet because when the price was agreed upon, Khaleeda did not inform her about removing
the carpet.
(b) Gurpreet because the carpet was integral to the floor of the bedroom and therefore attached to
the building that was sold.
(c) Khaleeda because a fully-furnished house does not entail the buyer to everything in the house.
(d) Khaleeda because by virtue of being a carpet it was never permanently fixed to the floor of the
building. Assume that in the above fact scenario, Khaleeda no longer wants the carpet. She
removes the elaborately carved door to the house after the sale has been concluded and claims
that Gurpreet has no claim to the door. The door in question was part of Khaleeda’s ancestral
home in Nagercoil, Tamil Nadu for more than 150 years before she had it fitted as the entrance
to her Baghmara house.
193. As a judge you would decide in favour of:
(a) Khaleeda because while the rest of the building belongs to Khaleeda exclusively, the door is
ancestral property and therefore the decision to sell it cannot be Khaleeda’s alone.
(b) Gurpreet because the door is an integral part of the building as it is attached to it.
(c) Khaleeda because the door can be removed from the building and is therefore not attached to it.
(d) Gurpreet because the contract is explicitly for the whole house and since the door is part of
house, it cannot be removed subsequent to the sale.
194. Amongst the following options, the most relevant consideration while deciding a case on the basis
of the above two principles would be:
(a) Whether the moveable thing was included in the sale agreement.
(b) Whether the moveable thing was merely placed on the land or building.
(c) Whether the moveable thing had become an inseparable part of the land or building.
(d) Whether the moveable thing could be removed.

Previous Years
Page 104 CLAT & AILET Papers
195. Rule C: If a moveable thing is placed on land with the intention that it should become an integral part
of the land or any structure on the land it becomes a fixture. Applying Rules A and C, to the fact
situations in questions 192 and 193, as a judge you would decide in favour of:
(a) Khaleeda in both situations.
(b) Gurpreet only in 192.
(c) Khaleeda only in 193.
(d) Gurpeet in both situations.

Problem 10 [For questions 196 to 200]

Rule A:
An owner of land has the right to use the land in any manner he or she desires. The owner of land also owns
the space above and the depths below it.

Rule B:
Rights above the land extend only to the point they are essential to any use or enjoyment of land.

Rule C:
An owner cannot claim infringement of her property right if the space above his or her land is put to
reasonable use by someone else at a height at which the owner would have no reasonable use of it and it
does not affect the reasonable enjoyment of his or her land.

Ramesh’s case: Ramesh owns an acre of land on the outskirts of Sullurpeta, Andhra Pradesh. The
Government of India launches its satellites into space frequently from Sriharikota, near Sullurpeta. The
Government of India does not deny that once the satellite launch has traveled the distance of almost 7000
kilometres it passes over Ramesh’s property. Ramesh files a case claiming that the Government of India
has violated his property rights by routing its satellite over his property, albeit 7000 kilometres directly
above it.

196. Applying only Rule A to Ramesh’s case, as a judge you would decide:

(a) In favour of the Government of India because the transgression was at a height at which Ramesh
could not possibly have any use for.
(b) That ownership of land does not mean that the owner’s right extends infinitely into space above
the land.
(c) In favour of Ramesh because he has the right to infinite space above the land he owns.
(d) In favour of the Government of India because it would lead to the absurd result that Ramesh and
most other property owners would have a claim against airline companies and other countries of
the world whose satellites orbit the earth.

Shazia’s case: Shazia owns a single storeyed house in Ahmedabad which has been in her family for more
than 75 years. The foundation of the house cannot support another floor and Shazia has no intention of
demolishing her family home to construct a bigger building. Javed and Sandeep are business partners and
own three storey houses on either side of Shazia’s house. Javed and Sandeep are also Ahmedabad’s main
distributors for a major soft drinks company. They have erected a huge hoarding advertising their products,
with the ends supported on their roofs but the hoarding also passes over Shazia’s house at 70 feet and
casts a permanent shadow on her terrace. Shazia decides to hoist a huge Indian flag, going up to 75 feet,
on her roof. She files a case, asking the court to order Javed and Sandeep to remove the hoarding for all
these reasons.
Previous Years
CLAT & AILET Papers Page 105
197. Applying only Rule B to Shazia’s case, you would decide in favour of:

(a) Javed and Sandeep because Shazia can easily hoist a flag below 70 feet.
(b) Shazia because she has the right to put her land to any use and the court cannot go into her
intentions for hoisting a flag at 75 feet.
(c) Shazia because she has the absolute right to the space above her land.
(d) Javed and Sandeep because hoisting a flag 75 feet above one’s roof is not essential to the use
and enjoyment of the land.

198. Applying only Rules A and B to Shazia’s case, you would decide:

(a) In favour of Shazia only under Rule A.


(b) In favour of Shazia under Rule A as well as B.
(c) Against Shazia under Rule B.
(d) Against Shazia under Rule A as well as B.

199. Applying only Rule B and C to Ramesh’s case, you would decide:
(a) In favour of Ramesh only under Rule B.
(b) In favour of Ramesh under Rule B as well as C.
(c) Against Ramesh under Rule C.
(d) Against Ramesh under Rule B as well as C.

200. Applying Rule C to Shazia’s case, you would decide:


(a) In her favour because hoisting a 75 feet high flag is reasonable.
(b) Against her because hoisting a 75 feet high flag is not reasonable.
(c) Against her because the hoarding is a reasonable use of the space above her land.
(d) In her favour because the permanent shadow cast by the hoarding affects the reasonable
enjoyment of her land.

Previous Years
Page 106 CLAT & AILET Papers
CLAT Question Paper 2012
ENGLISH
Instructions (1 to 10): Read the given passage carefully and attempt the questions that follow.

The work which Gandhiji had taken up was not only regarding the achievement of political freedom but also
the establishment of a new social order based on truth and non-violence, unity and peace, equality and
universal brotherhood and maximum freedom for all. This unfinished part of his experiment was perhaps
even more difficult to achieve than the achievement of political freedom. In the political struggle, the fight
was against a foreign power and all one could do was either join it or wish it success and give it his/her
moral support. In establishing a social order on this pattern, there was a strong possibility of a conflict
arising between diverse groups and classes of our own people. Experience shows that man values his
possessions even more than his life because in the former he sees the means for perpetuation and survival
of his descendants even after his body is reduced to ashes. A new order cannot be established without
radically changing the mind and attitude of men towards property and, at some stage or the other, the
‘haves’ have to yield place to the ‘have-nots’. We have seen, in our time, attempts to achieve a kind of
egalitarian society and the picture of it after it was achieved. But this was done, by and large, through the
use of physical force.

In the ultimate analysis it is difficult, if not impossible, to say that the instinct to possess has been rooted
out or that it will not reappear in an even worse form under a different guise. It may even be that, like a gas
kept confined within containers under great pressure, or water held back by a big dam, once the barrier
breaks, the reaction will one day sweep back with a violence equal in extent and intensity to what was
used to establish and maintain the outward egalitarian form. This enforced egalitarianism contains, in its
bosom, the seed of its own destruction.

The root cause of class conflict is possessiveness or the acquisitive instinct. So long as the ideal that is
to be achieved is one of securing the maximum material satisfaction, possessiveness is neither suppressed
nor eliminated but grows on what it feeds. Nor does it cease to be possessiveness, whether it is confined
to only a few or is shared by many.

If egalitarianism is to endure, it has to be based not on the possession of the maximum material goods by
a few or by all but on voluntary, enlightened renunciation of those goods which cannot be shared by others
or can be enjoyed only at the expense of others. This calls for substitution of material values by purely
spiritual ones. The paradise of material satisfaction, which is sometimes equated with progress these
days, neither spells peace nor progress. Mahatma Gandhi has shown us how the acquisitive instinct
inherent in man can be transmuted by the adoption of the ideal of trusteeship by those who ‘have’ for the
benefit of all those who ‘have not’ so that, instead of leading to exploitation and conflict, it would become a
means and incentive for the amelioration and progress of society respectively.

1. According to the passage, egalitarianism will not survive if


(A) It is based on voluntary renunciation
(B) It is achieved by resorting to physical force
(C) Underprivileged people are not involved in its establishment.
(D) People’s outlook towards it is not radically changed.

Previous Years
CLAT & AILET Papers Page 107
2. According to the passage, why does man value his possessions more than his life?
(A) He has inherent desire to share his possession with others.
(B) He is endowed with the possessive instinct.
(C) Only his possession helps him earn love and respect from his descendants.
(D) Through his possessions he can preserve his name even after his death.

3. According to the passage, which was the unfinished part of Gandhi’s experiment?
(A) Educating people to avoid class conflict.
(B) Achieving total political freedom for the country
(C) Establishment of an egalitarian society
(D) Radically changing the mind and attitude of men towards truth and non-violence.

4. Which of the following statements is ‘not true’ in the context of the passage?
(A) True egalitarianism can be achieved by giving up one’s possessions under compulsion.
(B) Man values his life more than his possessions.
(C) Possessive instinct is a natural desire of human beings
(D) In the political struggle, the fight was against alien rule.

5. According to the passage, true egalitarianism will last only if


(A) It is thrust upon people.
(B) It is based on truth and non-violence.
(C) People inculcate spiritual values instead of material values.
(D) ‘Haves’ and ‘have-nots’ live together peacefully

6. According to the passage, people ultimately overturn a social order ———-


(A) which is based on coercion and oppression.
(B) which does not satisfy their basic needs
(C) which is based upon conciliation and rapprochement.
(D) which is not congenital to the spiritual values of the people

7. According to the passage, the root cause of class conflict is


(A) The paradise of material satisfaction.
(B) Dominant inherent acquisitive instinct in man.
(C) Exploitation of the ‘have-nots’ by the ‘haves’.
(D) A Social order where the unprivileged are not a part of the establishment.

8. Which of the following statements is ‘not true’ in the context of the passage?
(A) A new order can be established by radically changing the outlook of people towards it.
(B) Adoption of the ideal of trusteeship can minimize possessive instinct.
(C) Enforced egalitarianism can be the cause of its own destruction
(D) Ideal of new order is to secure maximum material satisfaction

9. Which of the following conclusions can be deduced from the passage?


(A) A social order based on truth and non-violence alone can help the achievement of political
freedom.
(B) After establishing the social order of Gandhiji’s pattern, the possibility of a conflict between
different classes of society will hardly exist.
(C) It is difficult to change the mind and attitude of men towards property.
(D) In an egalitarian society, material satisfaction can be enjoyed only at the expense of others.
Previous Years
Page 108 CLAT & AILET Papers
10. According to the passage, what does “adoption of the ideal of trusteeship” mean?
(A) Equating peace and progress with material satisfaction.
(B) Adoption of the ideal by the ‘haves’ for the benefit of ‘have-nots’.
(C) Voluntary enlightened remuneration of the possessive instinct by the privileged class.
(D) Substitution of spiritual values by material ones by those who live in the paradise of material
satisfaction.

Instructions (11 to 15): Choose the correct synonym out of the four choices given.

11. Lethargy
(A) Serenity (B) listlessness (C) impassivity (D) laxity

12. Emaciated
(A) tall (B) languid (C) very thin (D) wise

13. Latent
(A) concealed (B) apparent (C) lethargic (D) prompt

14. Sporadic
(A) epidemic (B) whirling (C) occasional (D) stagnant

15. Compendium
(A) summary (B) index (C) reference (D) collection

Instructions (16 to 25): Choose the correct option out of the four choices given.

16. Give an example pertinent ______________ the case.


(A) with (B) on (C) for (D) to

17. My voice reverberated _____________ the walls of the castle.


(A) with (B) from (C) in (D) on

18. The reward was not commensurate _________ the work done by us.
(A) for (B) on (C) with (D) order

19. Our tragic experience in the recent past provides an index _______ the state of lawlessness in this
region.
(A) of (B) in (C) at (D) by

20. Your conduct smacks ___________recklessness.


(A) of (B) with (C) from (D) in

21. A good judge never gropes ____________the conclusion.


(A) to (B) at (C) on (D) for

22. Nobody in our group is a genius _________winning friends and in convincing people.
(A) for (B) in (C) of (D) at

23. If you are averse _________recommending my name, you should not hesitate to admit it.
(A) about (B) for (C) to (D) against

Previous Years
CLAT & AILET Papers Page 109
24. Religious leaders should not delve ________ politics.
(A) in (B) with (C) at (D) into

25. What you say has hardly any bearing ________ the lives of tribals.
(A) about (B) for (C) on (D) with

Instruction (26 to 30): Select the correct meaning of the italicized idioms and phrases out of the four
choices given.

26. He burnt his fingers by interfering in his neighbor’s affair.


(A) got himself into trouble (B) burnt himself
(C) got himself insulted (D) got rebuked

27. Mr. Gupta, who is one of the trustees of a big charity, is suspected of feathering his own nest.
(A) being lazy in doing his work (B) being too generous
(C) neglecting his job (D) making money unfairly

28. Mrs. Hashmi has been in the blues for the last several weeks.
(A) abroad (B) unwell
(C) depressed (D) penniless

29. For the first week, the apprentice felt like a fish out of water.
(A) frustrated (B) homeless
(C) disappointed (D) uncomfortable

30. His friends failed to see why he should ride the high horse just because he had won an election.
(A) become abnormal (B) appear arrogant
(C) indulge in dreams (D) hate others

Instructions (31 to 35): Given below are the jumbled sentences of a paragraph. The first and the last
sentence of the jumbled paragraph are given in correct order. Arrange the middle sentences in the correct
sequence.

31. i. On one hand we are proud of being Indians,


ii. on the other hand we behave as if we were still at the dawn of our civilization
iii. murders of our own brothers and sisters is not the way to please Ram or Rahim
iv. the citizens of the land where Buddha and Gandhi taught
v. the principles of love and non-violence,
vi. nor does it fetch us any prosperity.
(A) ii, iii, iv, v (B) iii, iv, v, ii (C) iv, v, iii, ii (D) iv, v, ii, iii

32. i. On the basis of experiments with rats


ii. health experts here say that
iii. exercise more and consume vitamins,
iv. they will live up to 100 years or more
v. if humans eat less,
vi. and be vigorous in their eighties and nineties.
(A) ii, iii, v, iv (B) ii, v, iii, iv (C) ii, v, iv, iii (D) v, ii, iii, iv
Previous Years
Page 110 CLAT & AILET Papers
33. i. The release of atomic energy is the greatest achievement which science has yet attained
ii. but the first invention to which their discoveries were applied was a bomb
iii. the atom was split by physicists whose minds were set on the search for knowledge
iv. it was more deadly than any other weapon invented so far
v. it is with dread that scientists regard the first use to which their greatest discovery was put
vi. however, they are gratified by the numerous applications of atomic energy for peaceful and
constructive population.

(A) ii, iii, iv, v (B) v, iii, ii, iv (C) iii, ii, iv, v (D) iv, v, iii, ii

34. i. The problem of food is intimately connected with population


ii. wages will seldom rise in proportion to the rising prices
iii. the market is governed by demand and supply
iv. without enough food, such people lack health, strength of efficiency
v. if too many people demand goods to go round, prices will rise and poor classes will starve
vi. they fall an easy prey to all sorts of diseases.

(A) iii, v, ii, iv (B) ii, iii, iv, v (C) iv, ii, v, iii D) v, iii, iv, ii

35. i. India’s message has always been one of love and peace.
ii. our Buddha was the light of Asia
iii. it has been a source of light and wisdom to the rest of the world
iv. Ashoka, moved by the horrors of Kalinga War, adopted the message of non-violence
v. the greatest apostle of non-violence in recent years was Mahatma Gandhi
vi. he shook the foundation of the British rule in India through non-violence.

(A) ii, v, iii, iv (B) iv, ii, iii, v (C) v, iv, iii, ii (D) iii, ii, iv, v

Instructions (36 to 40): Given below are a few commonly used foreign language phrases. Select the
correct answer from the four options given below each phrase.

36. Mala fide


(A) generous (B) bad intention (C) trustworthy (D) genuine

37. Tabula rasa


(A) clean slate (B) agitated (C) deprived (D) creative

38. Carte blanche


(A) slavery (B) complete discretion
(C) anarchy (D) dependent

39. De jure
(A) illegal (B) heir (C) concerning law (D) forbidden

40. Raison d’etre


(A) logical conclusion (B) reason for existence
(C) free choice (D) dubious argument

Previous Years
CLAT & AILET Papers Page 111
GENERAL KNOWLEDGE/CURRENT AFFAIRS
41. Who said that, ‘Man is a social animal’?
(A) Socrates (B) Aristotle (C) Kahn (D)Plato

42. World Computer Literacy day is celebrated on


(A) November 14 (B) November 3 (C) December 2 (D) July 5

43. Whose teaching inspired the French Revolution?


(A) Rousseau (B) Locke (C) Hegel (D) Wagner

44. The II Africa-India Summit was held in May 2011 in


(A) New Delhi (B) Lagos (C) Nairobi (D) Addis Ababa

45. The famous Akshardham temple is situated in the city of


(A) Jamnagar (B) Gandhinagar (C) Jammu (D) Madurai

46. Who out of the following was the recipient of Dhyan Chand Award in 2011?
(A) Satish Pillai (B) Hukam Singh (C) Shabbir Ali (D) Mukh Bain Singh

47. Name the annual fair of Rajasthan that is famous for its camel trading event.
(A) Maru Mela (B) Pushkar Mela (C) Suraj Kund Mela (D) Sonepur Mela

48. The 38th G-8 summit will be held in 2012 in


(A) USA (B) UK (C) Germany (D) Canada

49. Who was awarded the Nobel Prize for literature in 2011?
(A) Paul Lauterbur (B) Bill Clinton (C) Tomas Transtromer (D) Shirin Ebadi

50. Who was awarded the UNESCO King Sejong Literacy Prize in 2011?
(A) Nelson Mandela (B) National Literacy Service, Burundi
(C) Medha Patkar (D) National Literacy Mission, India

51. December 10 is observed as


(A) World Health Day (B) U. N. Day (C) Red Cross Day (D) Human Rights Day

52. Which is the largest gland in human body?


(A) Pancreas (B) Liver (C) Thyroid (D) Pituitary

53. The book titled ‘The Google Story’ has been authored by
(A) David A. Vice (B) Shobha Dey (C) Fredrick Forsyth (D) Vikram Seth

54. Which strait separates Europe from Africa


(A) Mallaica (B) Gibralter (C) Berring (D) Palk

55. Taiwan was earlier known as


(A) Fuchow (B) Marshall Island (C) Formosa (D) Macau

Previous Years
Page 112 CLAT & AILET Papers
56. Identify the Indian Tennis player who has turned Hollywood filmmaker?
(A) Vijay Amritraj (B) Mahesh Bhupathi (C) Leander Paes (D) Ashok Amritraj

57. Where will the next Olympic Games be held in 2012?


(A) Tokyo (B) Berlin (C) London (D) Toronto

58. Which of the following teams has won the Santosh Trophy Football Championship in 2011?
(A) Punjab (B)West Bengal (C) Goa (D) Railways

59. Excess of money supply as compared to supply of goods results in


(A) Depression (B) Deflation (C) Trade deficit (D) Inflation

60. The largest living flightless bird is


(A) Emu (B) Kiwi (C) Ostrich (D) Penguin

61. Which of the following oceans has the shape of the English letter ‘S’?
(A) Atlantic (B) Pacific (C) Indian (D) Arctic

62. Which is the longest shipping canal in the world?


(A) Panama Canal (B) Suez Canal
(C) White Sea-Baltic Canal (D)Kiel Canal

63. Le Corbusier, the architect of Chandigarh was a national of


(A) Britain (B) Portugal (C) France (D) Netherlands

64. India became a member of UNO in


(A) 1945 (B) 1947 (C)1950 (D) 1952

65. To which country does India export the largest quantity of iron ore?
(A) USA (B) Japan (C) Egypt (D) Germany

66. The longest highway in India runs from


(A) Kolkata to Jammu (B) Shillong to Amritsar
(C) Ambala to Nagercoil (D) Varanasi to Kanyakumari

67. The longest irrigation canal in India is called


(A) Upper Bari Doab Canal (B) Indira Gandhi Canal
(C) Sirhind Canal (D) Yamuna Canal

68. Leukemia is a disease related to


(A) Kidney (B) Throat (C) Blood (D) Eyes

69. In which city was Osama Bin Laden killed in May 2011?
(A) Islamabad (B) Abbottabad (C) Faisalabad (D) Peshawar

70. The XI Five Year Plan envisaged the highest growth in the sector of
(A) Industry (B) Agriculture (C) Services (D) Manufacturing

Previous Years
CLAT & AILET Papers Page 113
71. Light year is a unit of
(A) Distance (B) Time (C) Sound (D) Light intensity

72. The IV summit of BRICS was held in New Delhi on


(A) 11th January 2012 (B)1st April 2012 (C) 29th March 2012 (D) 28th February 2012

73. An indigenous nuclear submarine still under construction has been named as
(A) Chakra (B) Sudarshan (C) Arihant (D) Ghaatak

74. Government of India has launched a publicity campaign for census 2011 in association with which of
the following UN organization?
(A) United Nations Development Programme (UNDP)
(B) World Health Organization (WHO)
(C) United Nations Children’s Fund (UNICEF)
(D) United Nations Population Fund (UNPF)

75. Mahatma Gandhi National Rural Employment Guarantee Act (MGNREGA) completed how many
years of operation in 2011?
(A) 3 years (B) 4 years (C) 5 years (D) 6 years

76. The first ever formula one race in India was held in
(A) Greater Noida (B) New Delhi (C) Faridabad (D) Pune

77. Name the actor who has been honoured with the Dadasaheb Phalke Award in 2012.
(A) Naseerudin Shah (B) Kamal Hasan (C) Soumitra Chatterjee (D) Amol Paleker

78. In which city was the Arab Summit held in the last week of March 2012?
(A) Bagdad (B) Cairo (C) Beirut (D) Riyadh

79. The two Supreme Court Judges who delivered the famous 2-G judgment in February 2012 were
(A) Justice G.S. Singhvi and Justice Gyan Sudha Mishra
(B) Justice G.S. Singhvi and Justice A.K. Ganguly
(C) Justice S.H. Kapadia and Justice A.K. Ganguly
(D) Justice Chandramauli Kumar Prasad and Justice H.L. Gokhale

80. Who presides over the joint sitting of both houses of Parliament?
(A) Speaker of Lok Sabha (B) President
(C) Chairman of Rajya Sabha (D) Prime Minister

81. Christian Lagarde heads the


(A) World Bank (B) UNICEF
(C) International Monetary Fund (D) WHO

82. The seat of International Criminal Court is at


(A) The Hague (B) Geneva (C) Washington (D) Tokyo

83. First Indian to ski to North Pole is


(A) Arun Nayyar (B) Ajeet Bajaj
(C) Sq. Ldr. Sanjay Thapar (D) Neal Paramjeet
Previous Years
Page 114 CLAT & AILET Papers
84. First woman Director General of Police in India was
(A) Kanchan Choudhary (B) Kavitha Choudhary
(C) Kiran Bedi (D) Aswathy Tonge

85. Which countries co-hosted the One-day cricket World Cup in 2011?
(A) India, Bangladesh and Sri Lanka
(B) India, Bangladesh and Pakistan
(C) India, Sri Lanka and Pakistan
(D) India, Sri Lanka, Bangladesh and Pakistan

86. Priyanka Chopra has been named National Ambassador of


(A) WHO (B) UNICEF
(C) UNESCO (D) International Red Cross Society

87. Who is leading in the Republican primaries to contest the American Presidential election scheduled
in November 2012?
(A) Sara Palin (B) Newt Gingrich (C) Rick Santorum (D) Mitt Romney

88. Supreme Court recently declared ‘Salva Judum’ unconstitutional. What is ‘Salva Judam’?
(A) A terrorist outfit
(B) An armed civilian group formed to combat Maoists
(C) A money-chain business
(D) Custom of killing a girl for inter-caste marriage in the name of honour

89. As per the Indian Union Budget of 2012-13, the income-tax exemption limit for persons below 65
years of age is
(A) Rs. 175000 (B) Rs. 200000 (C) Rs. 250000 (D) Rs.190000

90. The U. N. Climate Change Conference 2011 was held in


(A) New Delhi (B) Doha (C) Durban (D) Geneva

ELEMENTARY MATHEMATICS (NUMERIC ABILITY)

91. P sells a table to Q at a profit of 10% and Q sells it to R at a profit of 12%. If R pays Rs.246.40 for
it, then how much had P paid for it?
(A) 200.00 (B) 300.00 (C) 248.00 (D) 346.00

92. The least value of x, for which the expression x2 + x + 17 will not give a prime number, is
(A) 7 (B) 11 (C) 13 (D) 17

93. A train 300 meters long is running at a speed of 25 meters per second, it will cross a bridge 200
meters long in
(A) 5 seconds (B) 10 seconds (C) 20 seconds (D) 25 seconds

94. If 0.06% of a number is 84, then 30% of that number is


(A) 25.2 (B) 420 (C) 42000 (D) 2520

Previous Years
CLAT & AILET Papers Page 115
95. A sum was divided among P, Q & R. R got double than P who got double than Q. If the difference
between the shares of Q and R is Rs.3675.00, then the sum in rupees is
(A) 4900 (B) 8575 (C) 11025 (D) 7350

96. If the ratio of the areas of two squares is 25 : 36, then the ratio of their perimeters is
(A) 5 : 6 (B) 25 : 36 (C) 6 : 5 (D) 36 : 25

97. The denominator of a fraction is greater than its numerator by 11. If 8 is added to both its numerator
3
and denominator, then it becomes . The fraction is
4
25 35 26 25
(A) (B) (C) (D)
26 26 35 36

1 1
98. The value of x 2 + y 2 , where x = 2 + 3 and y = 2 – 3, is

(A) 12 (B) 16 (C) 14 (D) 10

99. If the volume of a sphere is divided by its surface area, we obtain 27 cm. The radius of the sphere is
(A) 9 cm (B) 81 cm (C) 27 cm (D) 24 cm

100. One-third of one fourth of a number is 12. Then the number is


(A) 96 (B) 144 (C) 108 (D) 36

101. In the number series 4,10,23,50,104,216,439 the wrong number is


(A) 10 (B) 23 (C) 104 (D) 50

102. The price of 2 trousers and 4 shirts is Rs.1,600. With the same amount one can buy 1 trouser and
6 shirts. If one wants to buy 12 shirts, he has to pay
(A) Rs.2400 (B) Rs.4800 (C) Rs.1200 (D) Rs.3700

2 −4
3 2 –2
103. If x =     , the value of x is
2 3
12 12 −12 −12
2 3 6 5
(A)   (B)   (C)   (D)  
3 2 5 6

104. A dealer buys an article for Rs. 380.00. What price should he mark so that after allowing a discount
of 5% he still makes a profit of 25% on the article?
(A) Rs.500 (B) Rs.475 (C) Rs.95 (D) Rs.465

105. In a factory, the production of scooters rose to 48400 from 40000 in 2 years. The rate of growth per
annum is
(A) 20% (B) 10% (C) 30% (D) 8%

Previous Years
Page 116 CLAT & AILET Papers
1 1
106. If x + = 3, then x 2 + 2 will be
x x
(A) 9 (B) 10 (C) 27 (D) 7

107. When 16x4 + 12x3 – 10x2 + 8x + 20 is divided by 4x – 3, the quotient and the remainder are,
respectively
61 7 51
(A) 4x + 6x + 2x and (B) 4x + 6x +
3 2 3 2
and
2 2 2
2 61 7 61
(C) 6x + 2x + (D) 4x + 6x + 2x +
2 3 2
and and
7 2 2 2

108. The sum of two numbers is 2490. If 6.5% of one number is equal to 8.5% of the other, the numbers
are
(A) 1411 and 1079 (B) 1412 and 1080 (C) 1141 and 1709 (D)1214 and 1800

109. 120 men had food provision for 200 days. After 5 days, 30 men died of an epidemic. The food will last
for further
(A) 280 days (B) 260 days (C) 290 days (D) 252 days

110. Out of the total income, X spends 20% on house rent and 70% of the remaining amount on household
expenditure. If X saves Rs.1800, the total income is
(A) Rs.8000 (B) Rs.9500 (C) Rs.7500 (D) Rs.8500

LOGICAL REASONING

Instructions (111 to 116): Given below are some statements followed by two arguments. Read carefully
and decide which of the arguments strongly support the statement.

111. Statement: Should the pay scale and conditions of service of government employees be made
applicable to private sector employees?
Arguments:
(i) No, this will develop inertia, inefficiency and would adversely affect spirit of competition.
(ii) Yes, this will enhance dedication to work and institutional loyalty.

(A) Argument (i) is strong. (B) Argument (ii) is strong.


(C) Both (i) and (ii) are strong. (D) Neither (i) nor (ii) is strong.

112. Statement: Should a strong institution of ombudsman be created in India?


Arguments:
(i) Yes, this will bring transparency and accountability in the administration.
(ii) No, this will develop lack of initiative and flexibility in the administration.

(A) Argument (i) is strong. (B) Argument (ii) is strong.


(C) Both (i) and (ii) are strong. (D) Neither (i) nor (ii) is strong.

Previous Years
CLAT & AILET Papers Page 117
113. Statement: Should internal assessment in colleges and universities be abolished?
Arguments:
(i) Yes, this will eliminate the possibility of favoritism.
(ii) No, teaching faculty will lose control over the students and this would adversely affect their
academic growth.

(A) Argument (i) is strong. (B) Argument (ii) is strong.


(C) Both (i) and (ii) are strong. (D)Neither (i) nor (ii) is strong.

114. Statement: Should military training be made compulsory for all college and university students?
Arguments:
(i) Yes, this will develop in them a sense of punctuality and discipline.
(ii) No, military training should be given only to those students who are physically fit.

(A) Argument (i) is strong. (B) Argument (ii) is strong.


(C) Both (i) and (ii) are strong. (D) Neither (i) nor (ii) is strong.

115. Statement: Should students’ union in colleges and universities be abolished?


Arguments:
(i) Yes, it detracts students from academic and career development.
(ii) No, all great leaders have been students’ union leaders.

(A) Argument (i) is strong. (B)Argument (ii) is strong.


(C) Both (i) and (ii) are strong. (D) Neither (i) nor (ii) is strong.

116. Statement: Should the age of marriage be raised to 25 years for boys and 21 for girls?
Arguments:
(i) No, it is difficult to change a social practice in Indian conditions.
(ii) Yes, by that age people develop a sense of responsibility and also complete their education.

(A) Argument (i) is strong. (B) Argument (ii) is strong.


(C) Both (i) and (ii) are strong. (D)Neither (i) nor (ii) is strong.

Instructions (117 to 120): Each question contains six statements followed by four sets of combination of
three. Choose the set in which the statements are logically related.

117. Statements:
(i) X and Y are siblings.
(ii) X and Y do not quarrel.
(iii) Siblings are known to quarrel often.
(iv) X and Y quarrel often.
(v) All those who quarrel are siblings.
(vi) X and Y cannot be siblings.

(A) ii, iv,v (B) i, iv, vi (C) i, iii, iv (D) i, ii, v

Previous Years
Page 118 CLAT & AILET Papers
118. Statements:
(i) All mangoes are fruits.
(ii) All mangoes are green.
(iii) All mangoes are oval shaped.
(iv) All fruits are sweet.
(v) All mangoes are sweet.
(vi) All fruits are expensive.

(A) i, ii, iii (B) i, iv, v (C) ii, iii, iv (D) iv, v, vi

119. Statements:
(i) All frogs are amphibians.
(ii) All amphibians are not frogs.
(iii) All amphibians are cold blooded.
(iv) All frogs lay eggs.
(v) All amphibians lay eggs.
(vi) Frogs are cold blooded.

(A) i, iii, vi (B) i, iv, v (C) i, ii, v (D)ii, v, iv

120. Statements:
(i) Some men are of short- height.
(ii) Short-heighted men are intelligent.
(iii) Sudhir is a man.
(iv) Sudhir is of short-height.
(v) Sudhir is intelligent.
(vi) All men are intelligent.

(A) i, ii, vi (B) iii, iv, ii (C) ii, iv, v (D) ii, iv, vi

Instructions (121 to 123): Of the four alternatives given in each of the following questions, find the one
which is different from the rest.

121. (A) Fast-Slow (B) Bright-Dark (C) Day-Night (D) Valley-Depth

122. (A ) Body-Hand (B) Foot-Ankle (C) Eye-Ear (D) Wrist-Finger

123. (A) Snake-Frog (B) Goat-Hen (C) Dog-Cat (D) Tiger-Deer

Previous Years
CLAT & AILET Papers Page 119
Instructions (124-126): Each question below has two statements followed by four conclusions I, II, III and
IV. You have to accept the given statements to be true, even if they appear to be at variance from commonly
known facts. Read all the conclusions and then decide which of the given conclusions logically follows
from the two statements:

124. Statement One: All girls are students.


Statement Two: All doctors are students.
Conclusions:
I. All girls are students.
II. Some students are girls.
III. Some students are doctors.
IV. All doctors are girls.

(A) Only I follows. (B) Only I and II follows.


(C) Only II and IV follow. (D) Only I and II and III follows.

125. Statement One: All researchers are sociologists


Statement Two: Some researchers are professors.
Conclusions:
I. All researchers are professors.
II. Some researchers are professors.
III. Some professors are sociologists.
IV. Some sociologists are researchers.

(A) Only III and II follow. (B) Only II and IV follow.


(C) Only III follows. (D) None follows.

126. Statement One: Some democracies are dictatorship.


Statement Two: No dictatorship is a monarchy.
Conclusions:
I. No democracy is a monarchy.
II. No dictatorship is a democracy.
III. Some democracies are monarchy.
IV. Some dictatorships are democracies.

(A) None follows (B) Only IV follows.


(C) II and III follow. (D) I and IV follow.

Instructions (127 to 129): The following questions comprise of one or more statements. Answer the
questions on the basis of the given statement(s). Accept the factual assumptions required by the question,
even if you believe that the statement is false.

127. Statements:
I. Cheese is bad for people with high-cholesterol.
II. Sumeet does not eat cheese.
Assuming that (i) and (ii) are true, which of the following statement follows?
(A) Sumeet has high-cholesterol.
(B) Cheese is bad for Sumeet.
(C) People with high-Cholesterol do not eat cheese.
(D) None of the above.

Previous Years
Page 120 CLAT & AILET Papers
128. Statement:
I. Democrats are secularists.
Which of the following statements, if true, would show that the above statement is false?
(A) My father is a secularist but he is not a democrat.
(B) My father is a democrat but he is not secularist.
(C) My father is a democrat and he is secularist.
(D) My father is neither a democrat nor a secularist.

129. Statement:
“Where there is a cloud, there is a rain.” Which of the following statements, if true, would show that
the above statement is false?
(A) Sometimes there is cloud, but there is no rain.
(B) Sometimes there is rain, but there is no cloud.
(C) There is no rain where there is no cloud.
(D) None of the above.

Instructions (130 to 132): Read the following information carefully and then answer the questions.
Four friends W, X, Y and Z are students of Class 10th. W and X are good in Hindi but poor in English. W and
Y are good in Science but poor in Mathematics. Y and Z are good in English but poor in Social Studies. Z
and X are good in Mathematics as well as in Science.

130. Who amongst the following friends is not good in Mathematics but good in Hindi?
(A) W (B) Y (C) X (D) Z

131. Which of the following pairs of friends are good, both in English and Science?
(A) W and Y (B) W and Z (C) Y and Z (D) Z and X

132. Which of the following statements is definitely true?


(A) Y and Z are good in English as well as in Hindi
(B) All four friends are good in Science
(C) W is good in Social Studies, Hindi and Science
(D) Y is not good in Mathematics, Hindi and Social Studies

Instructions (133 to 135): Select the statement which logically follows the two given statements.

133. Statements:
I No athletes are vegetarians.
II All players are athletes.
III Therefore ———————————-
(A) no players are vegetarians (B) all players are vegetarian
(C) some players are vegetarian (D) all vegetarians are players

134. Statements :
I All persons who have done any creative work can be responsible critics
II Z has not done any creative work
III Therefore —————————————
(A) Z can be a responsible critic (B) Z cannot be a responsible critic
(C) Z can become a responsible critic (D) Z cannot become a responsible critic

Previous Years
CLAT & AILET Papers Page 121
135. Statement:
I One who has squared a circle is not a mathematician
II Therefore ———————————-
(A) No one who has squared a circle is a mathematician
(B) All non-mathematicians have squared a circle
(C) Some mathematicians have squared a circle
(D) All mathematicians square a circle

136. Statement: The Supreme Court of India is encouraging Public Interest Litigation
Reasons:
I To increase the reach of justice to the disadvantaged sections of society
II To quicken the pace of Justice
Identify the correct reason for the aforementioned statement.

(A) Both I and II are correct reasons of the statement


(B) I is the correct reason of the statement
(C) Both I and II are not correct reasons of the statement
(D) II is the correct reason of the statement

137. Yoga has become a very popular exercise, but it may not be for everyone. If you are interested in
high energy and fast workouts, yoga may not be the best choice. Therefore, evaluate your fitness
requirement before joining yoga classes.

This paragraph best supports the statement that:


(A) Yoga is more popular than high energy exercise
(B) Yoga is changing the concept of fitness in various ways
(C) Before opting for Yoga, assess your fitness requirements
(D) Yoga is a holistic fitness regime

138. Statistics allows us to understand the reality. It indicates developmental directions. Statistics is
good for exposing reality, but it can also be manipulated to perpetuate untruth and misunderstanding.
Data has power to mislead people.

This paragraph best supports the statement that:


(A) Words are more truthful than numbers
(B) Study of statistics is more important than any other discipline
(C) Numbers never lie
(D) Numbers can be used to mislead people.

139. Technology has developed out of stone tools which were used in ancient times. At first, development
of new technology was slow, but after neo-liberal economic policy was adopted there has been a
tremendous growth in technology sector.

This paragraph best supports the statement that:


(A) Stone tools were not really technology
(B) Stone tools were in use in Ancient India
(C) Today new technologies are developing at a fast pace
(D) New technology has nothing in common with the stone tools

Previous Years
Page 122 CLAT & AILET Papers
Instructions (140 to 144): Given below is a pair of events I and II. You have to decide their nature of
relationship. Assume that the given information is correct and final.
140. I Prices of toys in the market have gone down
II Government has reduced import duty on toys.
(A) I is the main cause and II is the main effect
(B) I is effect but II is not the main cause
(C) II is the main cause and I is the main effect
(D) II is an effect but I is not the main cause.

141. I Inflation rate in India has come down


II Reserve Bank of India has increased interest rate.
(A) I is the main cause and II is the main effect
(B) I is effect but II is not the main cause
(C) II is the main cause and I is the main effect
(D) II is an effect but I is not the main cause

142. I More and more students are opting for legal education
II Bar Council of India has introduced Bar Examination
(A) I is the main cause and II is the main effect
(B) I is effect but II is not the main cause
(C) II is the main cause and I is the main effect
(D) II is an effect but I is not the main cause

143. I Sea level is steadily rising


II Global Warming is a serious problem which the world is facing
(A) I is the main cause and II is the main effect
(B) I is effect but II is not the main cause
(C) II is the main cause and I is the main effect
(D) II is an effect but I is not the main cause

144. I Financial Institutions are largely unregulated


II Today, world is passing through a serious phase of economic crisis
(A) I is the main cause and II is the main effect
(B) I is effect but II is not the main cause
(C) II is the main cause and I is the main effect
(D) II is an effect but I is not the main cause

145. An argument is given below, on the basis of that argument; find out the parallel argument from the
given list of subsequent arguments
Argument: Himalayan Sparrows are disappearing. This bird is an Indian bird; therefore, Indian birds
are disappearing.
Subsequent Arguments:
(A) Industrialists pay most of the taxes. ‘Z’ is a wealthy man therefore ‘Z’ must pay most taxes
(B) A pineapple is a fruit, a mango is a fruit therefore pineapple is a mango
(C) Snow tigers are an endangered species; all endangered species must be protected; therefore
snow-tiger must be protected
(D) John is his father’s favorite son, and John knows this must be true because his father told him
this; and no father would lie to his favorite son
Previous Years
CLAT & AILET Papers Page 123
Instructions (146 to 149): Each question below is followed by arguments. Choose the most appropriate
choice from the options given

146. Question: Should Judicial Activism be discouraged?


Argument I: No, it would lead to executive dictatorship
Argument II: Yes, Judiciary should stay in the constitutional limits

(A) Argument I is strong (B) Argument II is strong


(C) Both the arguments are strong (D) Both the arguments are weak

147. Question: Should the Judiciary be independent of Executive and Legislature?


Argument I: Yes, this is necessary to ensure impartiality in the administration of Justice
Argument II: No, it will develop inertia in Executive and Legislature

(A) Argument I is strong (B) Argument II is strong


(C) Both the arguments are strong (D) Both the arguments are weak

148. Question: Should E-Governance be introduced at every level of public administration?


Argument I: Yes, it will reduce corruption
Argument II: No, it will lead to unemployment

(A) Argument I is strong (B) Argument II is strong


(C) Both the arguments are strong (D) Both the arguments are weak

149. Question: Should there be a world Government?


Argument I: Yes, it will eliminate inter-state conflicts
Argument II: No, Rich and Powerful countries will dominate it

(A) Argument I is strong (B) Argument II is strong


(C) Both the arguments are strong (D) Both the arguments are weak

150. “Some philosophers believe that a concept which cannot be verified can still be valid because of its
inner logic which ennobles it.”
In the light of the above statement, decide the status of the statement given below.
Statement: “Every person has certain inherent and inalienable rights which must be protected by
Rule of Law.”

(A) True (B) False


(C) Difficult to determine (D) Both True and False

Previous Years
Page 124 CLAT & AILET Papers
LEGAL APTITUDE

151. PRINCIPLE:
Only Parliament or State Legislatures have the authority to enact laws on their own. No law made by
the State can take away a person’s fundamental right.

FACTS:
Parliament enacted a law, which according to a group of lawyers is violating the fundamental rights
of traders. A group of lawyers files a writ petition challenging the Constitutional validity of the statute
seeking relief to quash the statute and further direct Parliament to enact a new law.
(A) No writ would lie against Parliament, as the court has no authority to direct Parliament to enact
or re-enact a law
(B) The court can quash existing law if it violates fundamental rights and can direct Parliament to
make a new law
(C) The court can quash the existing law if it violates fundamental rights but cannot direct Parliament
to make a new law.
(D) None of these

152. PRINCIPLE:
When one person signifies to another his willingness to do or abstain from doing anything, with a
view to obtaining the assent of that person to such an act or abstinence, he is said to have made a
proposal.

FACT:
“Ramanuj telegraphed to Shyam Sunder, writing: “Will you sell me your Rolls Royce CAR? Telegram
the lowest cash price.” Shyam Sunder also replied by telegram: “Lowest price for CAR is Rs. 20
lakh.” Ramanuj immediately sent his consent through telegram stating: “I agree to buy the CAR for
Rs. 20 lakh asked by you.” Shyam Sunder refused to sell the car.

(A) He cannot refuse to sell the CAR because the contract has already been made.
(B) He can refuse to sell the CAR because it was only invitation to offer and not the real offer
(C) It was not a valid offer because willingness to enter into a contract was absent
(D) None of these

153. PRINCIPLE:
Every person, who is of the age of majority, is competent to contract according to the law to which
he is subject.

FACTS:
A minor mortgaged his house in favour of Thakur Das, a money lender, to secure a loan of Rs.
20000. A part of this, i.e. Rs. 10500 was actually advanced to him. While considering the proposed
advance, the attorney who was acting for the money lender, received information that the plaintiff
was still a minor. Subsequently the minor commenced an action stating that he was underage when
he executed the mortgage and the same should, therefore, be cancelled. He prayed for setting
aside the mortgage. The mortgagee money lender prayed for the refund of Rs. 10500 from the minor.
(A) As a minor’s contract is void, any money advanced to a minor can be recovered.
(B) A minor’s contract is void ab initio, any money advanced to a minor cannot be recovered.
(C) A minor’s contract is voidable; any money advanced to a minor can be recovered
(D) Advanced money can be recovered because minor has given wrong information about his age.

Previous Years
CLAT & AILET Papers Page 125
154. PRINCIPLE:
A person is said to be of sound mind for the purpose of making a contract if, at the time when he
makes it, he is capable of understanding it and of forming a rational judgment as to its effect upon
his interests.

FACTS:
Mr. X who is usually of sound state of mind, but occasionally of unsound state of mind, enters into
a contract with Mr. Y when he was of unsound state of mind. Mr. Y having come to know about this
fact afterwards, wants to file a suit against Mr. X

(A) Mr. X cannot enter into contract because he is of unsound state of mind when he entered into
contract.
(B) Mr. X can enter into contract but the burden is on the other party to prove that he was of unsound
state of mind at the time of contract.
(C) Mr. X can enter into contract but the burden is on Mr. X to prove that he was of sound state of
mind at the time of contract
(D) None of these

155. PRINCIPLE:
(1). The state shall not deny to any person equality before the law and equal protection of the laws
within the territory of India.
(2). The State shall not discriminate against any citizen on grounds only of religion, race, caste, sex
and place of birth or any of them.

FACTS:
The Government of Rajasthan, passed an order providing for reservations for the Scheduled Castes/
Scheduled Tribes and Socially and Educationally Backward Classes (including Muslims), and
Women, in all institutions of higher education, including private educational institutions, both aided
as well as unaided, in the following manner: Scheduled Caste- 15%; Scheduled Tribe- 7.5%, Socially
and Educationally Backward Classes (including Muslims) - 27%
I. The reservation policy of the government is violative of the principle of equality envisaged in the
Constitution
II. The reservation policy is unconstitutional because it is based on ‘caste’ which is a prohibited
marker
III. Reservation does not violate equality clause as it entails “like should be treated like and unlike
should be treated differently.”
IV. Reservation does not violate equality clause as the Constitution itself enables the State to make
special provision for the advancement of socially and educationally backward classes of citizens
or for the Scheduled Castes and Scheduled Tribes.

(A) I is correct
(B) I and II are both correct answers
(C) III is correct answer
(D) III and IV both are correct answers

Previous Years
Page 126 CLAT & AILET Papers
156. PRINCIPLE:
Nothing is an offence merely by reason of its being done with the knowledge that it is likely to cause
harm, if it be done without any criminal intention to cause harm, and in good faith for the purpose of
preventing or avoiding other harm to a person or property.

FACTS:
Mr. Sharman, the Captain of a steam vessel, suddenly and without any fault or negligence on his
part, finds himself in such a position that, before he can stop his vessel, he must inevitably run down
a boat B, with twenty or thirty passengers on board, unless he changes the course of his vessel,
and that, by changing his course, he must incur the risk of running down a boat C with only two
passengers on board and which he may possibly clear.

(A) Sharman has committed no offence because this was done out of necessity.
(B) Sharman can be held responsible for the act of criminal negligence.
(C) Sharman can be held responsible for culpable homicide.
(D) This is a clear case of accident so Sharman cannot be held responsible

157. PRINCIPLE:
Willful rash driving is an offense.

FACTS:
Mr. Tiwari was driving his car after drinking alcohol. Police books him for willful negligent driving. Is
the act of the police lawful?

(A) No, because Mr. Tiwari was not driving rashly; he was drunk while driving.
(B) No, this is not a negligent act.
(C) Yes, because Mr. Tiwari was driving rashly.
(D) Yes, because the police has the power to arrest a person driving rashly.

158. PRINCIPLE:
Whoever, intending to take dishonestly any movable property out of the possession of any person
without that person’s consent, moves that property with an intention to take it, is said to commit
theft.

FACTS:
Y cuts down a tree on Z’s ground, with the intention of dishonestly taking it out of Z’s possession
without Z’s consent. Y could not take away the tree.

(A) Y can be prosecuted for theft.


(B) Y cannot be prosecuted for theft.
(C) Y can be prosecuted for attempt to theft.
(D) Y has neither committed theft nor attempted to commit theft.

Previous Years
CLAT & AILET Papers Page 127
159. PRINCIPLE:
Injuria Sine Damnum i.e. Injury (violation of legal right) without damage

FACTS:
X, who was the returning officer at a polling booth in Amethi, wrongly refused to register a duly
tendered vote of Y in the recent UP elections, even though Y was an eligible voter. The candidate in
whose favour Y wanted to vote, was declared elected. Give the appropriate answer-

(A) Y can sue X on the ground that he was denied the right to cast vote, which is a fundamental
right.
(B) Y can sue X on the ground that he was denied the right to cast vote, which is a legal right
(C) Y cannot sue X because there is no injury or damage caused to Y
(D) Y cannot sue X because the candidate in whose favor he wanted to vote was declared elected.

160. PRINCIPLE:
Nothing is an offence which is done by a person who, at the time of doing it, by reason of unsound
state of mind, is incapable of knowing the nature of the act, or something that he is doing is either
wrong or contrary to law.

FACT:
X takes his son Y who is three years old, for bathing to the well. He throws his son inside the well
so that the son can have a good bath. After 10 minutes he also jumps into the well to take bath and
get his son out of the well. Both were rescued by the villagers but his son was found dead.
(A) X has committed culpable homicide amounting to murder.
(B) X has committed murder.
(C) X has done no offence as he can plead the defense of unsound state of mind.
(D) X’s family should be held responsible for allowing him to take the child to the well.

161. PRINCIPLE:
Ignorance of Fact is excused but ignorance of law is no excuse

FACT:
X was a passenger from Zurich to Manila in a Swiss Plane. When the plane landed at the Airport of
Bombay on 28 Nov. 1962 it was found on searching that X carried 34 kg of Gold Bars on his person
and that he had not declared it in the ‘Manifest for Transit’. On 26th Nov. 1962 the Government of
India had issued a notification modifying its earlier exemption, making it mandatory now that the
gold must be declared in the “Manifest” of the aircraft.

(A) X cannot be prosecuted because he had actually no knowledge about the new notification
issued two days ago
(B) X cannot be prosecuted because ignorance of fact is excusable
(C) X can be prosecuted because ignorance of law is not excusable
(D) X’s liability would depend on the discretion of the court

Previous Years
Page 128 CLAT & AILET Papers
162. PRINCIPLE:
Proposal (communication) + Acceptance (communication) + Consideration = Contract. The
communication of a proposal is complete when it comes to the knowledge of the person to whom it
is made.

FACTS:
X’s nephew absconded from home. He sent his servant in search of the boy. After the servant had
left, X by handbills offered to pay Rs. 501 to anybody finding his nephew. The servant came to know
of this offer only after he had already traced the missing child. He, therefore, brought an action to
recover the reward.

(A) His action would fail because he was not aware of the offer
(B) His action would not fail because it was a general offer
(C) The fact that he was not aware of the offer does not make any difference and hence it was a valid
contract. It is a mere formality
(D) None of the above

163. PRINCIPLE:
Agreements, the meaning of which is not certain, or not capable of being made certain, are void.

FACTS:
A horse was bought for a certain price coupled with a promise to give Rs.500 more if the horse is
proved lucky.
(A) This is a valid agreement.
(B) This agreement is void for uncertainty because it is very difficult to determine what luck, bad or
good, the horse has brought to the buyer.
(C) The agreement is partially valid and partially void.
(D) None of the above.

164. PRINCIPLE:
Mere silence as to the facts likely to affect the willingness of a person to enter into a contract is not
a fraud, unless the circumstances of the case are such that, on close examination it is found to be
the duty of the person keeping silent to speak, or unless his silence is, in itself, equivalent to
speech.

FACTS:
X sells by auction to Y, a horse which X knows to be of unsound state of mind. X says nothing to Y
about the horse’s unsound state of mind. Give the correct answer-

(A) X can be held liable for fraud.


(B) X can be held liable for misrepresentation
(C) X cannot be held liable, because he did not say anything positive about the mental state of the
horse.
(D) X cannot be held liable because it is the buyer who must be aware of the things.

Previous Years
CLAT & AILET Papers Page 129
165. PRINCIPLE:
Any direct physical interference with goods in somebody’s possession without lawful justification is
called trespass of goods.

FACTS:
Z purchased a car from a person who had no title to it and sent it to a garage for repair. X believing
wrongly that the car was his, removed it from the garage.

(A) X can be held responsible for trespass of goods


(B) X cannot be held responsible for trespass of goods as he was under a wrong belief.
(C) X has not committed any wrong.
(D) None of the above.

Instructions (166 to 180): Each of the next nine questions consists of two statements, one labeled as
‘Assertion’ (A) and other as ‘Reason’ (R). You are to examine these two statements carefully and select
the correct answers.

166. Assertion (A): A void contract is not necessarily illegal

Reason (R): Every illegal contract is void.


(A) Both A and R are individually true and R is the correct explanation of A.
(B) Both A and R are individually true but R is not the correct explanation of A
(C) A is true but R is false
(D) A is false but R is true

167. Assertion (A): The Indian Constitution was adopted on 26th November, 1949.

Reason (R): Law Day is celebrated in India on 26th November every year.
(A) Both A and R are individually true and R is the correct explanation of A.
(B) Both A and R are individually true but R is not the correct explanation of A
(C) A is true but R is false
(D) A is false but R is true

168. Assertion (A): The state shall not make any law, which takes away or abridges the rights conferred
by Part III (Fundamental Rights) and any law made in contravention of this clause shall, to the
extent of the contravention, be void.

Reason (R): The fundamental rights are the rights reserved by the people and for this reason they
are eternal and sacrosanct.
(A) Both A and R are individually true and R is the correct explanation of A.
(B) Both A and R are individually true but R is not the correct explanation of A
(C) A is true but R is false
(D) A is false but R is true

Previous Years
Page 130 CLAT & AILET Papers
169. Assertion (A): Directive Principles of State Policy contained in Part IV shall not be enforceable by
any court, but the principles therein laid down are nevertheless fundamental in the governance of the
country and it shall be the duty of the State to apply these principles in making laws.

Reason (R): Directive Principles of State Policy and Fundamental Rights are both complementary
to each other but in case of any controversy fundamental rights will prevail.
(A) Both A and R are individually true and R is the correct explanation of A.
(B) Both A and R are individually true but R is not the correct explanation of A
(C) A is true but R is false
(D) A is false but R is true

170. Assertion (A): All minorities, whether based on religion or language, shall have the right to establish
and administer educational institutions of their choice.

Reason (R): Institutions established by the minorities are not entitled to governmental aid and
government is not under an obligation to give aid.
(A) Both A and R are individually true and R is the correct explanation of A.
(B) Both A and R are individually true but R is not the correct explanation of A
(C) A is true but R is false
(D) A is false but R is true

171. Assertion (A): The right to move the Supreme Court under Article 32 of the Constitution by appropriate
proceedings for the enforcement of the fundamental rights is guaranteed as a fundamental right.

Reason (R): Supreme Court of India has been appointed as the guardian of the Constitution.
(A) Both A and R are individually true and R is the correct explanation of A.
(B) Both A and R are individually true but R is not the correct explanation of A
(C) A is true but R is false
(D) A is false but R is true

172. Assertion (A): If the budget presented to the Rajya Sabha in not passed in the stipulated period, the
budget proposals are not affected.

Reason (R): The Lok Sabha is more powerful, in financial matters, than the Rajya Sabha.
(A) Both A and R are individually true and R is the correct explanation of A.
(B) Both A and R are individually true but R is not the correct explanation of A
(C) A is true but R is false
(D) A is false but R is true

Previous Years
CLAT & AILET Papers Page 131
173. Assertion (A):
In the Event of violation of any legal right (tort) the aggrieved party is entitled to recover unliquidated
damages.

Reason (R):
The object of awarding damages to the aggrieved party is to put him in the same position in which
he would have been if the wrong would not have been committed. Damages are therefore, assessed
on that basis.

(A) Both A and R are individually true and R is the correct explanation of A.
(B) Both A and R are individually true but R is not the correct explanation of A
(C) A is true but R is false
(D) A is false but R is true

174. Assertion (A): During inflation, there is increase in money supply and rise in price level.

Reason (R): The rise in prices is due to shortage in supply of essential consumer goods.
(A) Both A and R are individually true and R is the correct explanation of A.
(B) Both A and R are individually true but R is not the correct explanation of A
(C) A is true but R is false
(D) A is false but R is true

175. Assertion (A): X, because of unsound state of mind and not knowing the nature of the act, attacks
Y, who in self defense and in order to ward off the attack, hits him thereby injuring him. Y has not
committed an offence.

Reason (R): Y had a right of private defense against X under Section 98 of the Indian Penal Code.

(A) Both A and R are individually true and R is the correct explanation of A.
(B) Both A and R are individually true but R is not the correct explanation of A.
(C) A is true but R is false.
(D) A is false but R is true.

176. Assertion (A): X and Y independently entertained the idea to kill Z. Accordingly, each of them
separately inflicted wounds on Z who died as a consequence. X and Y are liable for murder under
341 IPC.

Reason (R): When a criminal act is done by several persons in furtherance of common intention of
all, each of such persons is liable as if the whole act was done by him alone.

(A) Both A and R are individually true and R is the correct explanation of A.
(B) Both A and R are individually true but R is not the correct explanation of A.
(C) A is true but R is false.
(D) A is false but R is true.

Previous Years
Page 132 CLAT & AILET Papers
177. Assertion (A): A person claims compensation for his non-gratuitous act.

Reason (R): A person who enjoys benefit from lawful, non-gratuitous act of another must compensate
him even though there is no contract.

(A) Both A and R are individually true and R is the correct explanation of A.
(B) Both A and R are individually true but R is not the correct explanation of A
(C) A is true but R is false
(D) A is false but R is true

178. Assertion (A): Freedom of Speech is the most important civil liberty of people in a democratic polity.

Reason (R): State can regulate free speech in the interest of public order.
(A) Both A and R are individually true and R is the correct explanation of A.
(B) Both A and R are individually true but R is not the correct explanation of A
(C) A is true but R is false
(D) A is false but R is true

179. Assertion (A): Austin’s concept of law is known as imperative theory

Reason (R): Austin emphasized on the commanding character of law.

(A) Both A and R are individually true and R is the correct explanation of A.
(B) Both A and R are individually true but R is not the correct explanation of A
(C) A is true but R is false
(D) A is false but R is true

180. Assertion (A): The essence of joint liability under section 149 of the IPC is that the criminal act
must have been done with a view to fulfill the common object of an unlawful assembly.

Reason (R): Any sudden and provocative act done by a member of an unlawful assembly would
render the other members of that assembly liable.

(A) Both A and R are individually true and R is the correct explanation of A.
(B) Both A and R are individually true but R is not the correct explanation of A.
(C) A is true but R is false.
(D) A is false but R is true.

181. The following are enshrined in the Preamble of the Constitution of India
I. Equality of status and of opportunity
II. Liberty of thought, expression, belief, faith and worship
III. Justice-social, economic and political
IV. Fraternity assuring the dignity of the individual
V. Unity and Integrity of the nation

Which of the following is the correct order in which they appear in the preamble?
(A) V-I-II-IV-III (B) III-II-I-IV-V (C)III-I-II-V-IV (D) I-II-IV-III-V

Previous Years
CLAT & AILET Papers Page 133
182. Which one of the following statements is correct?
Right to free and compulsory education for all children of the age of 6 to 14 years is:
(A) a fundamental right enforceable in law
(B) a fundamental duty
(C) a directive principle of state policy
(D) a fundamental right which, however, can be enforced only if the state makes an enabling legislation

183. Affirmative action connotes:


I. Measures taken by the state to help the socially disadvantaged groups
II. Positive discrimination
III. Strict quotas for the socially and educationally backward class in school/college admissions
and jobs.

Which of the above mentioned is true?


(A) I and II only (B) II only (C) I,II and III (D) II and III only

184. Identify the correct statement:


(A) Federalism implies a system of government which embodies a division of powers between a
central and a number of regional authorities
(B) Federalism implies a system of government which embodies a division of powers between
Legislature, Executive and Judiciary
(C) Federalism implies a system of Government which embodies Parliamentary supremacy
(D) None of these

185. Consider the following statements:


I. In a recent Supreme Court verdict pronounced by Justice Markandeya Katju and Justice Gyan
Sudha Mishra, the court upheld the constitutionality of the Haj subsidy
II. Muslims are not the only beneficiaries of the secular state’s generosity. Hindus have also received
substantial financial support from the Government

With reference to the statements mentioned above, which of the following is correct?
(A) I only (B) II only (C) Both I and II (D) Neither I nor II

186. X, a married woman, agreed to live in adultery with B and also agreed to serve him as his housekeeper.
In return, B agreed to pay X Rs. 500 per month for living in adultery and Rs. 500 per month for
housekeeping. The agreement is
(A) Valid
(B) Void
(C) Void as to the first object but valid with respect to the second object
(D) Unlawful as being opposed to public policy

Previous Years
Page 134 CLAT & AILET Papers
187. Ramu applied for the post of Director in an organization. The governing body of the organization
passed a resolution appointing him to the post. After the meeting, one of the members of the
governing body informed him privately of the resolution. Subsequently, the resolution was rescinded.
Ramu claims damages. Which one of the following is the correct legal proposition in the case?
(A) Ramu cannot claim damages as he had not resigned from his existing post in anticipation of
getting the appointment letter
(B) Ramu cannot claim damages as there was no formal communication
(C) Ramu can claim damages as governing body cannot rescind the resolution once passed
(D) Ramu can claim damages as there was private communication

188. The Railway authorities allowed a train to be over crowded. In consequence, a legitimate passenger
Mr. X got his pocket picked. Choose the appropriate answer:

(A) Mr. X can sue the Railway authorities for the loss suffered
(B) Mr. X cannot sue because he had given his consent to travel in a over-crowded train
(C) Mr. X cannot sue railway authorities because there was no infringement of his legal right and
mere fact that the loss was caused does not give rise to a cause of action
(D) None of these

189. Z is carried off by a tiger. X fires at the tiger, knowing that the shot might kill Z, but with no intention
to kill Z, and in good faith trying to save Z. X’s shot, however, gives Z a mortal wound. Choose the
correct option –
(A) X has committed an offence of a grievous nature.
(B) X has no moral duty to save Z therefore he can be held liable.
(C) X has not committed any offence, as the act was in good faith and for the benefit of Z.
(D) None of the above

190. Ms. Usha wants to file a suit against Bhagyalaxmi Theatre praying for a permanent injunction (stay
order) restraining the theatre from running the film named “Jai Santoshi Maa”. Her contention is that
the film hurt her religious feelings and sentiments as Goddess Saraswati, Laxmi and Parvati were
depicted as jealous and were ridiculed.

(A) She cannot file a suit because injury to religious feelings is not a legally recognized right.
(B) She cannot file a suit because the Theatre has a fundamental right to speech and expression.
(C) She can file a suit as injury to religious feelings has been legally recognized as a right (injuria
sine damnum)
(D) It is a case of complete judicial discretion.

Previous Years
CLAT & AILET Papers Page 135
191. Match schedule one and two and choose the appropriate answer-

Schedule I Schedule II
i. Concurrent list 1. Constitution of Japan
ii. Rule of Law 2. Constitution of Ireland
iii. Directive Principle of State Policy 3. British Constitution
iv. Procedure established by law 4. Constitution of Australia

i ii iii iv
(A) 1 2 3 4
(B) 2 4 3 1
(C) 1 3 4 2
(D) 4 3 2 1
192. P, Q and R made a joint promise to give S a sum of Rs.3000. S recovered the whole amount from P.
Q was declared insolvent and cannot give anything. Which statement out of the following is correct?
(A) P cannot get anything from R. (B) P can recover Rs.1000 from R.
(C) P can recover Rs.1500 from R (D)P can recover Rs.2000 from R.

193. X went to Y’s house and forgot his bag which contained 1 kg sweets. Y’s children consumed the
sweets. Decide the liability of Y.
(A) Y is bound to pay the price of sweets to X
(B) Y is not bound to pay anything
(C) Y is bound to pay half the price of sweets.
(D) Y would not have to pay anything because X loves Y’s children.

194. Which one of the following is not correct?


(A) Freedom of speech and expression includes freedom of press.
(B) Right to life and personal liberty includes right to carry on any trade and profession
(C) Right to equality includes the principles of natural justice
(D) Freedom of conscience includes the wearing and carrying of kirpans by the Sikhs

195. Y makes an attempt to steal some jewels by breaking open a box and finds, after opening the box,
that there is no jewel in it. Choose the appropriate answer.
(A) Y is not guilty of attempt to theft because the box was empty.
(B) Y is guilty of attempt to commit theft.
(C) Y is guilty of trespass.
(D) Y is not guilty of any offence.

196. A lady wanted to get a railway ticket but finding a crowd near the ticket window at the station, asked
Raju, who was near the window, to get a ticket for her and handed him money for the same. Raju
took the money and instead of getting the ticket, ran away with it. What offence has been committed
by Raju?
(A) No offence (B) Criminal breach of trust
(C) Criminal misappropriation (D) Theft

197. The Right to Equality is guaranteed by-


(A) Article 14 to 18 (B) Article 14
(C) Article 14 and 15 (D) Article 14, 15 and 16

Previous Years
Page 136 CLAT & AILET Papers
198. Mr. Samay was severely hurt while working in his factory and fell unconscious. He was rushed to a
hospital by his fellow workers. In the hospital (at emergency/casualty ward) the doctors opined that
he should be operated immediately. While conducting preliminary examinations, he was found to be
HIV positive. The doctors are in a dilemma regarding what should they do first-
(A) Doctors should operate first
(B) Doctors should inform his family members
(C) Doctors should inform his employers
(D) Doctors should not inform anyone because it would violate patient’s right of privacy.

199. Match the schedule I and II and choose the appropriate answer-

Schedule I Schedule II
i Republic 1. Head of the state is elected by the people
ii Secular 2. State does not recognize any religion as religion of the state
iii Democracy 3. The government which gets authority from the will of the people

i ii iii
(A) 1 2 3
(B) 1 3 2
(C) 2 3 1
(D) 3 2 1

200. In which of the following cases can a Constitutional amendment be passed just by a simple majority
in Parliament?
(A) Election matters
(B) Change in the name and boundaries of states
(C) Powers of the President
(D) None of the above

Previous Years
CLAT & AILET Papers Page 137
CLAT Question Paper 2013
Time : 2 hours Maximum Marks : 200

ENGLISH COMPREHENSION 1. What does the theory "Divine Right of King"


stipulate?
Directions for (Q. 1 – 10) : Read the given passage (a) The kings are God
carefully and attempt the questions that follow and (b) That the right of governing is conferred
shade the appropriate answer in the space provided upon the kings by God
for it on the OMR answer sheet. (c) They have the right to be worshipped like
Gods by their subjects
It is an old saying that knowledge is power. Education (d) That the right of kings are divine and
is an instrument which imparts knowledge and therefore sacred
therefore, indirectly controls power. Therefore, ever
since the dawn of our civilisation, persons in power 2. Who controlled education during the
have always tried to supervise or control education. Renaissance?
It has been handmaid of the ruling class. During the (a) The common people
Christian era, the ecclesiastics controlled the (b) The prince
institution of education and diffused among the people (c) The church and the priests
the gospel of the Bible and religious teachings. These (d) None of these
gospels and teachings were no other than a
philosophy for the maintenance of the existing 3. What did the ruling class in the Christian era
society. It taught the poor man to be meek and to think of the poor man?
earn his bread with the sweat of his brow, while the (a) That he is the beloved of God
priests and the landlords lived in luxury and fought (b) That he deserves all sympathy of the rich
duels for the slightest offence. During the (c) That he should be strong and lord over
Renaissance, education passed more from the others
clutches of the priest into the hands of the prince. In (d) That he is meant for serving the rich
other words, it became more secular. Under the
control of the monarch, education began to devise 4. Who controlled the institution of education
and preach the infallibility of its masters, the monarch during the Christian Era?
or king. It also invented and supported fantastic (a) The secular leaders of society
theories like "The Divine right Theory" and that the (b) The church and the priests
king can do no wrong, etc. With the advent of the (c) The monarchs
industrial revolution, education took a different turn (d) The common people
and had to please the new masters. It now no longer
remained the privilege of the baron class, but was 5. What does the word "infallibility" mean?
thrown open to the new rich merchant class of the (a) That every man is open to error
society. The philosophy which was in vogue during (b) Sensitivity
this period was that of "Laissez Faire" restricting (c) The virtue of not making mistake
the function of the state to a mere keeping of laws (d) That some divine power is responsible for
and order while on the other hand, in practice the determining the fate of the man
law of the jungle prevailed in the form of free
competition and the survival of the fittest.

Previous Years
Page 108 CLAT & AILET Papers
6. What do you mean by the "sweat of his brow"? 14. (a) hemorhage (b) haemorhhage
(a) Very hard work (c) haemorrhage (d) hemoorhage
(b) The tiny droplets of sweat on the forehead
(c) The wrinkles visible on the forehead 15. (a) dylexsia (b) dyslexia
(d) The sign of innocence (c) dislexia (d) dislecsia

7. What does the policy of "Laissez Faire" stand Directions for (Q. 16 – 25): Select the best option
for? from the four alternatives given below and shade
(a) Individual freedom in the economic field the appropriate answer in the space provided for it in
(b) State control over law and order in society the OMR answer sheet.
(c) Joint control of the means of production
by the state and private enterprise 16. Unless he _________ this office, I will not
(d) Decontrol over law and order by the ruling say anything.
class (a) Left (b) Did not leave
(c) Leaves (d) Had left
8. Which of the following describes the writer?
(a) Concerned (b) Unconcerned 17. ___________, I would help all the poor people.
(c) Aggressive (d) Frustrated (a) If I am rich (b) If I was rich
(c) If I were rich (d) In case I am rich
9. Choose the correct synonym out of the four
choices given: 18. I ________ the news an hour ago.
(a) Have heard (b) Heard
Gospels (c) Was hearing (d) Have been hearing
(a) Chit chat
(b) A teaching or doctrine of a religious 19. He spoke __________ about his prospects.
teacher (a) Confidentially (b) Consciously
(c) Rumour (c) Confidently (d) Conscientiously
(d) Guidance
20. The boy is not interested in playing, _____ ?
10. Choose the correct synonym out of the four (a) Doesn't he? (b) Isn't he?
choices given: (c) Didn't he? (d) Is he?
Vogue
(a) Uncertain 21. He told us that we should never live beyond
(b) Out-dated ___________ means.
(c) The prevailing fashion or style (a) His (b) Their
(d) Journey (c) Our (d) Her

Directions for (Q. 11 – 15): Select the word that is 22. May I request _______ you again to consider
spelt correctly my case favourably.
(a) To
11. (a) paraphernalia (b) paraphrenalia (b) Onto
(c) parapherenalia (d) paraphrennalia (c) Of
(d) Noproposition required
12. (a) enterprenuer (b) entrepreneur
(c) entrepenur (d) enteruepeur 23. Known as devout and serious person, she
also has ______ sense of humour.
13. (a) onomaetopoeia (b) onomoatopoeia (a) Better (b) Quick
(c) onomatopoeia (d) onomatopoeia (c) Good (d) Beautiful

Previous Years
CLAT & AILET Papers Page 109
24. Galileo said, "The Earth ______ around the D. In the Indian context, though venture
sun". capital has been a relatively late entrant,
(a) Revolved (b) Is revolving it has already made a reasonable impact.
(c) Revolves (d) Is revolved
(a) A B C D
25. We ____________ our work before the guests (b) A D B C
arrived at our house. (c) A C B D
(a) Shall finish (b) Have finished (d) A D C B
(c) Had finished (d) Shall have finished
28. A. Progress in diagnosis, in preventive
Direction for (Q. 26 – 30): The sentences given in medicine and in treatment, both medicinal
each question, when properly sequenced, form a and surgical, has been rapid and breathe
coherent paragraph. Each sentenced is labelled with taking.
a letter. Choose the most logical order of sentences B. Much in medicine which is not taken for
from among the given choices to construct a coherent granted was undreamt of even as recently
paragraph. Shade the appropriate answer in the as 20 years ago.
space provided for it in the OMR sheet. C. Presently small pox has been eradicated,
poliomyelitis practically banished,
26. A. People who start up their own business tuberculosis has become curable and
typically come from two extreme coronary artery disease surgically
backgrounds: One is the business family relievable.
background and the other is a steady D. The dramatic surge in the field of molecular
professional family background. biology and research by immunologists
B. Typically, people from different and geneticists has succeeded in
backgrounds face different kinds of basic controlling parasitic diseases like malaria
problems. and river blindness that affect millions of
C. The people from both the backgrounds find people round the world.
it very difficult to establish and manage
an enterprise. (a) B D C A
D. Starting up and managing a small (b) B A C D
business is no joke. (c) B C A D
(d) B D A C
(a) D B C A
(b) B A C D 29. A. Instead, many deaths and injuries result
(c) D A C B from falling objects and the collapse of
(d) C D A B buildings, bridges and other structures.
B. Earthquakes almost never kill people
27. A. Venture capital is recommended as the directly.
ideal source of financing for a successfully C. Fire resulting from broken gas or power
small business. lines is another major danger during a
B. Several companies including start ups quake.
have been funded by dedicated venture D. Spills of hazardous chemicals are also a
funds during this decade. concern during an earthquake.
C. Despite this, an average Indian
entrepreneur understands and (a) C A B D
appreciation of venture capital concept has (b) D A C B
been woefully inadequate. (c) D C A B
(d) B A C D

Previous Years
Page 110 CLAT & AILET Papers
30. A. The Winchester or hard disk drives can Directions for (Q. 36 – 40) : Choose the explanation
store much more data than what can be that catches the spirit of the idiom given in each
stored on a floppy diskette. question.
B. Hard disks come sealed and they cannot
be removed or changed like floppy 36. To blaze a trail:
diskettes. (a) To set on fire
C. Often floppy disk system is used in (b) To blow the trumpet
conjunction with the Winchester disk (c) To initiate work in a movement
system. (d) To be hopeful
D. This makes for an ideal system for
secondary storage. 37. A snake in the grass:
(a) A secret or hidden enemy
(a) C A B D (b) An unreliable person
(b) C B D A (c) Unforeseen happening
(c) B A C D (d) A dangerous enemy
(d) A B C D
38. Have too many irons in the fire:
Directions for (Q. 31 – 35): Given below are a few (a) Engaged in too many enterprises at the
foreign language phrases which are commonly used. same time
Choose the correct meaning for each of the phrases (b) Facing too many problems at the same
and shade the appropriate answer in the space time
provided for it in the OMR sheet. (c) Said or done too many things at the same
time
31. El Dorado (d) To incite the feeling amongst the people
(a) An imaginary place
(b) High altitude 39. A fair weather friend:
(c) A literary man (a) A friend who is fair to us at all the times
(d) A country full of gold and precious stones (b) A friend who deserts us in difficulties
(c) A friend whom we love the most
32. quantum ramifactus (d) A friend who loves us the most
(a) The amount of damage suffered
(b) The amount of damage caused 40. A Panacea:
(c) The amount of damage paid (a) An injection that serves as a life line
(d) The amount of damage received (b) A lecture full of precepts
(c) A strong drug that induces sleep
33. Corpus delicti (d) A single cure for all diseases or troubles
(a) Fake evidence of an offence
(b) Hearsay evidence of an offence ELEMENTARY MATHEMATICS
(c) Lack of evidence of an offence NUMERICAL ABILITY
(d) An evidence which constitute an offence
41. 2 / 3 is a rational number whereas 2/ 3
34. Vis-a-vis
is
(a) Direct (b) Opposite
(a) Also a rational number
(c) Face to face (d) Agree
(b) An irrational number
(c) Not a number
35. Carte blanche
(d) A natural periodic number
(a) Complete authority(b) Issue the warrant
(c) No authority (d) Lack of authority

Previous Years
CLAT & AILET Papers Page 111
42. Greatest number which divides 926 and 2313, The mean daily expenditure on food is
leaving 2 and 3 remainders, respectively, is (a) ` 111 (b) ` 161 (c) ` 211 (d) ` 261
(a) 52 (b) 54 (c) 152 (d) 154
51. A box contains 24 marbles, some are green
43. A single discount equivalent to a discount and others are blue. If a marble is drawn at
series 15% and 5% is random from the jar, the probability that it is
(a) 32% (b) 20% (c) 10% (d) 8.5% green is 2/3. The number of blue balls in the
jar is
44. By selling a cycle for ` 2345, a student loses (a) 13 (b) 12 (c) 16 (d) 8
19%. His cost price is nearly
(a) ` 4000 (b) ` 5000 52. The population of a city is 250 thousand. It is
(c) ` 3000 (d) ` 3500 increasing at the rate of 2% every year. The
growth in the population after 2 years is
45. Diagonals of a rhombus are 1 meter and 1.5 (a) 2500 (b) 10000
meter in lengths. The area of the rhombus is (c) 252000 (d) 10100
(a) 0.75 m2 (b) 1.5 m
(c) 1.5 m 2 (d) 0.375 m2 53. If a point (x, y) in a OXY plane is equidistant
from (–1, 1) and (4, 3) then
46. An angle in a semi circle is (a) 10x + 4y = 23 (b) 6x + 4y = 23
(c) –x + y = 7 (d) 4x + 3y = 0
π π
(a) π (b) (c) (d) 2π
4 2 54. Sum of first 15 multiples of 8 is
(a) 960 (b) 660
47. In a school for midday meal food is sufficient (c) 1200 (d) 1060
for 250 students for 33 days, if each student is
given 125 gm meals. 80 more students joined 55. A rod of 2 cm diameter and 30 cm length is
the school. If same amount of meal is given to converted into a wire of 3 meter length of
each student, then the food will last for uniform thickness. The diameter of the wire
(a) 20 days (b) 40 days is
(c) 30 days (d) 25 days
2 2
(a) cm (b) cm
48. In a school of 500 students, 102 students 10 10
can read Hindi and Tamil both, 200 students
can read only Hindi. The students who can 1 1
(c) cm (d) cm
read Tamil are 10 10
(a) 98 (b) 402 (c) 302 (d) 300
56. Two straight poles of unequal length stand
49. The value of k for which kx+ 3y– k + 3 = 0 upright on a ground. The length of the shorter
and 12x + ky = k, have infinite solutions, is pole is 10 meter. A pole joins the top of the
(a) 0 (b) – 6 (c) 6 (d) 1 two vertical poles. The distance between the
two tops is 5 meter. The distance between
50. Table shows the daily expenditure on food of
the poles along the ground is 4 meter. The
25 households in a locality:
area thus formed by the three poles with the
< Households ground is
100 –150 4 (a) 52 meter2 (b) 46 meter2
150 – 200 5 (c) 20 meter 2 (d) 50 meter2
200 – 250 12
250 – 300 2
300 – 350 2

Previous Years
Page 112 CLAT & AILET Papers
57. Pipe A can fill a tank in 10 hours and pipe B 65. Dr. Norman Borlaugh is famous as father of
can fill the same tank in 12 hour. Both the the Green Revolution in 1960s. His initial goal
pipes are opened to fill the tank and after 3 was to create varieties of wheat adapted to
hours pipe A is closed. Pipe B will fill the the climate of
remaining part of the tank in (a) Mexico (b) India
(a) 5 hours (b) 4 hours (c) USA (d) China
(c) 5 hours 24 minutes(d) 3 hours
66. A feature ‘Bluetooth' now common in mobile
58. A ground 100 × 80 m2 has two cross roads in phones, gets its name from a
its middle. The road parallel to the length is 5 (a) Chinese 10th Century King
meter wide and the other road is 4 meter wide, (b) UK Software Company
both roads are perpendicular to each other. (c) Greek Goddess
The cost of laying the bricks at the rate of ` (d) Danish 10th Century King
10 per m2, on the roads, will be
(a) ` 7000 (b) ` 8000 67. Which pair of states does not touch each other
(c) ` 9000 (d) ` 10000 (a) Meghalaya, Manipur
(b) Chhattisgarh, U.P.
59. If selling price of 10 articles is equal to cost (c) Rajasthan, Punjab
price of 11 articles, then gain is (d) J.K., H.P.
(a) 8% (b) 9% (c) 8.5% (d) 10%
68. Baglihar dam, is constructed on river
60. Angles of a quadrilateral are in the ratio (a) Ravi (b) Chenab
3 : 4 : 5 : 8. The smallest angle is (c) Indus (d) Sutlej
(a) 54° (b) 72° (c) 36° (d) 18°
69. Navjivan Trust was instituted with the
GENERAL KNOWLEDGE AND CURRENT AFFAIRS objectives of propagating peaceful means of
attaining third Swaraj, by
61. The Headquarter of European Union is situated (a) Mohan Das Karam Chand Gandhi
in (b) Lala Lajpat Rai
(a) England (b) Germany (c) Bal Gangadhar Tilak
(c) France (d) Belgium (d) Dr. Ambedkar

62. India in 2008 successfully put CHANDRAYAAN- 70. World Cup Football 2014 and Olympics 2016
1 into its initial orbit by will be held in
(a) PSLV- C12 (b) PSLV- C11 (a) USA (b) Brazil
(c) PSLV-14 (d) GSLV-D3 (c) Russia (d) South Africa

63. Vishwanathan Anand retained the World 71. In 2012-13, India's target is to restrict the
Chess Championship in 2012 by defeating fiscal deficit to x % of the GDP, where x is
Boris Gelfand. Mr. Gelfand belongs to (a) 10 (b) 8.3
(a) Israel (b) Russia (c) 15 (d) 5.1
(c) Poland (d) USA
72. POSCO steel project to come up but being
64. Kapilvastu Relics (fragments of Lord Buddha's strongly protested by the people is located in
bone), for the second time in 114 years, (a) Chhattisgarh (b) Jharkhand
recently travelled from India to (c) Odisha (d) Andhra Pradesh
(a) China (b) Sri Lanka
(c) Myanmar (d) Japan

Previous Years
CLAT & AILET Papers Page 113
73. Bharat Ratna and Padma Awards in our 82. On which date Maha Kumbh Mela started in
country were instituted in the year Prayag this year?
(a) 1952 (b) 1954 (a) 14-1-2013 (b) 1-1-2013
(c) 1962 (d) 1964 (c) 26-1-2013 (d) 4-1-2013

74. Who was crowned the Miss World 2012 on 83. On which date International Women's Day is
August 18, 2012? celebrated?
(a) Ms. Jessica Kahawaty (a) 18th March (b) 8th March
(b) Ms. Wenxia Yu th
(c) 28 March (d) 18th Feb
(c) Ms. Vanya Mishra
(d) Ms. Sophie Moulds 84. Duration of which Five Year Plan was 2007-
2012
75. Vishwaroopam is a 2013 Tamil spy thriller film (a) X (b) XII (c) IX (d) XI
written, directed and co-produced by ____
__________ who also enacts the lead role. 85. As per census 2011, which State has the
(a) Prakash Raj (b) Rajni Kant lowest sex ratio (877 : 1000) ?
(c) Kamal Haasan (d) Chiranjeevi (a) Punjab (b) Haryana
(c) Uttar Pradesh (d) Bihar
76. Vijay Kumar, who clinched a silver medal in
London Olympics in 2012 is associated with 86. Hugo Chavez who died on March 5, 2013 after
(a) Boxing (b) Shooting losing his battle with cancer, was the
(c) Weight Lifting (d) Wrestling President of which country?
(a) Argentina (b) Cuba
77. Sushil Kumar who won a silver medal in (c) Brazil (d) Venezuela
London Olympics in 2012 is associated with
(a) Shooting (b) Boxing 87. Carlos Slim, who tops the list of world's
(c) Wrestling (d) Weight lifting wealthiest people, for the fourth year in a row,
belongs to which country?
78. How many medals did India win in London (a) USA (b) England
Olympics 2012? (c) Mexico (d) Germany
(a) 3 (b) 4 (c) 5 (d) 6
88. In the name P. Chidambaram, the present
79. The present Pope chosen in March 2013 hails Union Finance Minister, what does ‘P' stands
from which country? for?
(a) Brazil (b) Mexico (a) Palghat (b) Pallakudi
(c) Argentina (d) Panama (c) Palaniappan (d) Perumal

80. The first person to set foot on the moon on 89. The earlier name of which city was New
July 20, 1969 and who died on Aug. 25, 2012 Amsterdam?
was (a) Chicago (b) California
(a) Nevil Armstrong (b) Neil Armstrong (c) Washington D.C. (d) New York City
(c) Gagan Narang (d) Michael Phelps
90. The grant of Diwani of Bengal, Bihar and
81. In which place, on Feb 21, 2013, two powerful Orissa is associated with
explosive devices planted on bicycles had (a) Nawab Shuja-ud-Daula
exploded in Andhra Pradesh? (b) Nawab Asif-ud-Daula
(a) Dilrubnagar (b) Dilsukhnagar (c) Shah Alam I
(c) Dilkushnagar (d) Dilshaknagar (d) Shah Alam II

Previous Years
Page 114 CLAT & AILET Papers
91. Where did Kuchipudi, an eminent dance form, 100. The Constitution (One Hundred Seventeenth
originate? Amendment) Bill, 2012 makes provisions
(a) Kerala (b) Andhra Pradesh regarding
(c) Uttar Pradesh (d) Tamil Nadu (a) Reservation in matters of promotions for
Scheduled Castes and Scheduled Tribes
92. Light Year is the unit of (b) Reservation in matters of appointments
(a) Time (b) Distance for Scheduled Castes and Scheduled
(c) Light (d) None of these Tribes
(c) Reservation in matters of appointments
93. The leaning tower of Pisa does not fall and promotions for Scheduled Castes and
because Scheduled Tribes
(a) It is tapered at the top (d) None of the above
(b) It covers a large base area
(c) Its C.G. is inside the tower 101. The number of High Courts in India is
(d) The vertical line passing through the C.G. (a) 18 (b) 24 (c) 21 (d) 28
of the tower falls within its base
102. The last British Emperor of India was
94. "Paradise Regained" was written by (a) King George I (b) King George III
(a) John Milton (b) Michel Angelo (c) King George V (d) King George VI
(c) John Keats (d) Lord Byron
103. Palaeolithic period is also known as
95. Which is the richest temple in India? (a) Mesolithic Age (b) Late Stone Age
(a) Balaji Temple of Tirupathi (c) Old Stone Age (d) Neolithic Age
(b) Padmanabha Swamy Temple of
Thiruvananthapuram 104. Capital of India was transferred from Calcutta
(c) Shirdi Sai Baba Temple to Delhi in the year
(d) Jagannatha Temple of Puri (a) 1901 (b) 1911
(c) 1921 (d) 1922
96. Who founded the Red Cross?
(a) Henry Dunant (b) Alexander 105. The chairman of Fundamental Rights Sub-
(c) James Cook (d) Bismark Committee of the Constituent Assembly was
(a) Rajendra Prasad (b) B.R. Ambedkar
97. World Literacy Day is celebrated on (c) Jawaharlal Nehru (d) J.B. Kripalani
(a) 5th September (b) 6th September
th
(c) 8 September (d) None of these 106. The Environment Protection Act was passed
by the Parliament of India in the year
98. South African Paralympics icon Oscar (a) 1976 (b) 1986
Pistorius has been accused of killing (c) 1996 (d) 2006
(a) Julia Kamp (b) Reeva Steenkamp
(c) Pistorius Kamp (d) Shakeera Kamp 107. International Year of Biodiversity is/was/will
be
99. In the month of March, 2013 the Supreme (a) 2010 (b) 2011
Court of India issued a notice that the (c) 2012 (d) 2014
ambassador of the following country shall not
leave India without the permission of the 108. The first Shaka king in India was
Supreme Court (a) Rudradaman (b) Menadar
(a) Germany (b) Maldives (c) Maues (d) Damanrudra
(c) Italy (d) Nepal

Previous Years
CLAT & AILET Papers Page 115
109. Potential Energy is described by the Directions for (Q. 114 – 115): Answer the questions
expression: on the basis of the information given below:
(a) PE = mgh (b) PE = ngh
(c) PE = oph (d) PE = pph The head of a newly formed Government desires to
appoint five of the six elected ministers P, Q, R, S, T
110. Where was 16th NAM Summit held? and U to portfolios of Foreign, Industry and
(a) Tehran (b) Mehran Commerce, Agriculture, Rural Development and
(c) Turban (d) Bagdad Human Resources. U does not want any portfolio if
S gets one of the five. R wants either Foreign or
LOGICAL REASONING Human Resources or no portfolio. Q says that if S
gets Industry and Commerce or Rural Development
Directions for (Q. 111– 113) : Answer the following then she must get the other one. T insists on a
questions based on the statements given below: portfolio if P gets one.
i. There are 3 poles on each side of the road.
ii. These six poles are labelled A, B, C, D, E 114. Which of the following is a valid assignment
and F. (a) P- Foreign, Q- Industry and Commerce,
iii. The poles are of different colours namely R- Agriculture, S- Rural Development,
Golden, Silver, Metallic, Black, Bronze and T- Human Resources
White. (b) R- Foreign, S- Industry and Commerce,
iv. The poles are of different heights. P- Agriculture, Q- Rural Development,
v. E, the tallest pole, is exactly opposite to the T- Human Resources
Golden colours pole. (c) P- Foreign, Q- Industry and Commerce,
vi. The shortest pole is exactly opposite to the T- Agriculture, S- Rural Development,
Metallic coloured pole. U- Human Resources
vii. F, the Black coloured pole, is located between (d) Q- Foreign, U- Industry and Commerce,
A and D. T- Agriculture, R- Rural Development,
viii. C, the Bronze coloured pole, is exactly P- Human Resources
opposite to A.
ix. B, the Metallic coloured pole, is exactly 115. If P gets Foreign and R gets Human
opposite to F. Resources, then which is NOT a valid
x. A, the White coloured pole, is taller than C assignment of Agriculture and Rural
but shorter than D and B. Development
(a) S- Agriculture, Q- Rural Development
111. What is the colour of the pole diagonally (b) U- Agriculture, Q- Rural Development
opposite to the Bronze coloured pole? (c) Q- Agriculture, T- Rural Development
(a) White (b) Silver (d) Q- Agriculture, S- Rural Development
(c) Metallic (d) Golden
Directions for (Q. 116 – 120) : In each of the
112. Which is the second tallest pole? following questions, a related pair of words is followed
(a) A (b) D by four pairs of words or phrases. Select the pair
(c) B (d) Cannot be determined that best expresses a relationship similar to the one
expressed in the question pair.
113. Which is the colour of the tallest pole?
(a) Golden (b) Silver 116. Action : Reaction
(c) Bronze (d) None of these (a) Introvert : Extrovert
(b) Assail : Defend
(c) Diseased : Treatment
(d) Death : Rebirth

Previous Years
Page 116 CLAT & AILET Papers
117. Sorrow : Misery 124. Deepak said to Nitin, "That boy playing with
(a) Love : Obsession the football is the younger of the two brothers
(b) Amity : Harmony of the daughter of my father's wife". How is
(c) Happiness : Joy the boy playing football related to Deepak?
(d) Enemy : Hatred (a) Son (b) Brother
(c) Cousin (d) Brother-in-law
118. Drama : Audience
(a) Brawl : Vagabonds 125. Pointing to a woman in the photograph, Rajesh
(b) Game : Spectators said, "The only daughter of her grandfather is
(c) Art : Critic my wife". How is Rajesh related to that woman?
(d) Movie : Actors (a) Uncle (b) Father
(c) Maternal Uncle (d) Brother
119. Nuts : Bolts
(a) Nitty : Gritty (b) Bare : Feet Directions for (Q. 126 – 130): Read the information
(c) Naked : Clothes (d) Hard : Soft given below to answer the questions.
i. Kareena's dieting schedule consists of having
120. Book : Author only one fruit on a given day of the week.
(a) Rain : Flood ii. Dietician has prescribed banana, papaya,
(b) Light : Switch pomegranate, apple and grape from Sunday
(c) Symphony : Composer to Friday, one day being a fasting day.
(d) Song : Music Kareena cannot eat any fruit on Saturday.
iii. Pomegranate day is neither on the first day
Directions for(Q. 121 – 125) : Each question nor on the last day but earlier than the papaya
contains a statement on relationship and a question day.
regarding relationship based on the statement. iv. Apple day is on the immediate next day of
Select the correct option. papaya day.
v. Banana day is on the immediate previous day
121. Moni is daughter of Sheela. Sheela is wife of of the fasting day.
my wife's brother. How Moni is related to my vi. Apple day and grape day must have a gap of
wife? two days between them.
(a) Cousin (b) Niece vii. Grape day is the day immediately following
(c) Sister (d) Sister-in-law the fasting day.

122. Annu is daughter of my mother's brother 126. Which of the following is the fasting day?
Abahi. Pari is grand daughter of my mother. (a) Monday (b) Tuesday
Pari should call Annu as (c) Wednesday (d) Thursday
(a) Maternal Aunt (b) Sister
(c) Cousin (d) Niece 127. Banana day and apple day have a gap of how
many days between them?
123. Markandey is Rajiv's mother's father. (a) One (b) Two (c) Three (d) Four
Markandey is three brother One of them has
grandson Abahi. Rajan is son of Abahi. Rajan 128. Which day is grape day?
is related to Rajiv as (a) Monday (b) Tuesday
(a) Brother (b) Nephew (c) Thursday (d) Sunday
(c) Cousin (d) Uncle
129. Which day is pomegranate day?
(a) Sunday (b) Monday
(c) Tuesday (d) Wednesday

Previous Years
CLAT & AILET Papers Page 117
130. Which of the following is the correct 134. Statement : School dropout rate is very high
statement? in the rural areas as children support their
(a) Apple day is after papaya day. parents in income earning activities.
(b) Banana day is on Wednesday. I. Public awareness programme on primary
(c) Fasting day is on Tuesday. education should be expanded
(d) Papaya day is earlier than banana day. immediately to educate parents.
II. Compensation is not a remedy.
Directions for (Q. 131 – 135): Each question
contains one statement and two courses of action I 135. Statement : Smoking is one of those human
and II. Assuming the statements to be true, decide weaknesses which tend to test the will power
of the smoker of the edge.
which of the two courses of action most logically
I. It is very difficult for the smoker to give up
follows.
smoking even if they want to do so.
II. Human beings have other weaknesses as
Code:
well
(a) If only I follows.
(b) If only II follows. Directions for (Q. 136 – 140): Complete the series
(c) If either I or II follows. by choosing the correct option.
(d) If neither I nor II follows.
136. 0, 1, 1, 2, 3, 5, 8, 13, 21, ?
131. Statement: Indian children are very talented (a) 34 (b) 35
but are instead weak in science and (c) 33 (d) 36
mathematics.
I. Teaching and textbooks are not available 137. A -10, E -15, I -20, M -25,…….
in mother language. (a) Q-5 (b) Q-30
II. Education based on experiments in both (c) P-30 (d) R-30
the subjects is lacking.
138. 17, 36, 74, 150, ? , 606
132. Statement: Despite of child labour laws, (a) 250 (b) 303
children can be seen working in hotels, shops, (c) 300 (d) 302
houses, very frequently.
I. The government should not make such 139. 2, 1, 4, 3, 6, 5, 8, ?
laws which cannot be enforced. (a) 9 (b) 10
II. A proper education system for the primary (c) 7 (d) 8
level particularly for lower caste
140. 1, 4, 27, 256, ?
community may eradicate this problem.
(a) 625 (b) 3125
(c) 3025 (d) 1225
133. Statement : Kyoto protocol on environment
is signed by almost every country of the world.
Directions for (Q. 141 – 145): In each question
I. As a result air, water and soil pollution below are given two statements numbered I and II.
have come down.
II. Increasing production of automobiles, You have to take the two given statements as true
refrigerators and fertilisers do not affect even if they seem to be at variance with commonly
our environment. known facts. Read all the conclusions and then
decide which of the given conclusions logically follow
from the given statements, disregarding commonly
known facts.

Previous Years
Page 118 CLAT & AILET Papers
141. I. All vegetables have gravy. ii. H will sit beside A
II. All lunch has vegetable. iii. C will sit beside E
iv. F will sit beside H
(a) All lunch has gravy. v. E will sit beside G
(b) All gravy has lunch. vi. D will sit beside F
(c) Both (a) and (b). vii. G will sit beside B
(d) None of the above viii. B will sit beside D

142. I. Karan Johar is a good director. 146. Which of the following is wrong?
II. Directors are intelligent. (a) A will be to the immediate right of C
(b) D will be to the immediate left of B
(a) All intelligent are directors (c) E will be to the immediate right of A
(b) Karan Johar is intelligent. (d) F will be to the immediate left of D
(c) Both (a) and (b)
(d) None of the above 147. Which of the following is correct?
(a) B will be to the immediate left of D
143. I. Some blues are green. (b) H will be to the immediate right of A
II. Pink is green (c) C will be to the immediate right of F
(d) B will be to the immediate left of H
(a) Some blue is pink.
(b) Some green is pink. 148. A and F will become neighbours if:
(c) If either (a) or (b) follows. (a) B agrees to change her sitting position
(d) Some pinks are blues. (b) C agrees to change her sitting position
(c) G agrees to change her sitting position
144. I. All boys are tall. (d) H agrees to change her sitting position
II. All Punjabi are tall.
149. During sitting:
(a) All boys are Punjabi. (a) A will be directly facing C
(b) Some boys are Punjabi. (b) B will be directly facing C
(c) Both of the above. (c) A will be directly facing B
(d) None of the above. (d) B will be directly facing D

145. I. All girls go to the college. 150. H will be sitting between:


II. Rina does not go to the college. (a) C and B (b) A and F
(c) D and G (d) E and G
(a) Rina is not a girl.
(b) Going to college is not essential to be a LEGAL APTITUDE
girl.
(c) Rina is a girl. This section consists of fifty (50) questions. Each
(d) None of the above. question consists of legal propositions/ principles
(hereinafter referred to as ‘principle') and facts. These
Directions for (Q. 146–150): Read the information principles have to be applied to the given facts to
given below to answer the questions. arrive at the most reasonable conclusion. Such
principles may or may not be true in the real sense,
A, B, C, D, E, F, G and H want to have a dinner on a yet you have to conclusively assume them to be
round table and they have worked out the following true for the purposes of this section. In other words,
seating arrangements. in answering the following questions, you must not
i. A will sit beside C rely on any principles except the principles that are

Previous Years
CLAT & AILET Papers Page 119
given herein below for every question. Further you Facts: William has an old car of which he
must not assume any facts other than those stated makes seldom use. He voluntarily enters into
in the question. The objective of this section is to an agreement with Smith to sell this car for
test your interest towards study of law, research rupees ten thousand. Thereafter one Anson
aptitude and problem solving ability even if the "most approaches William and offers to buy that car
reasonable conclusion" arrived at may be for rupees one lac as the car was one which
unacceptable for any other reason. It is not the Anson has been searching for long. Now
objective of this section to test your knowledge of William wants to cancel his agreement with
law. Smith and refuses to deliver the car to him
saying that consideration (price) for the car
151. Principle: When an offer is accepted by a promised by Smith is negligible and, therefore,
person to whom it is made, it becomes a agreement with him can not be said to be
promise. But this promise will become legally valid one.
binding only when the acceptance of the offer
is unconditional. (a) William can cancel his agreement with
Smith as the consideration involved in that
Facts : Ram makes an offer to sell his house is really inadequate.
to Shyam for ` 50 lacs. Shyam accepts this (b) William can not cancel his agreement
offer but wants to pay the price of the house with Smith as the sale of car for rupees
in five quarterly instalments. Ram does not ten thousand was voluntary and this price
agree to it. Thereafter Shyam agrees to pay has some value in the eyes of law.
the price of the house in the way as originally (c) William can cancel his agreement with
desired by Ram. But Ram does not reply to Smith as he was ignorant about the value
it. Can Shyam compel Ram to sell his house / price of the car for which it could be sold.
to him? (d) William can cancel his agreement with
Smith as he is entitled to get full market
(a) Shyam can compel Ram to sell his house value/price of his car.
because Shyam ultimately agrees to pay
the price as originally desired by Ram 153. Principle: In order to be illegible to appear in
(b) Shyam can compel Ram to sell his house the semester examination, a student is
because Shyam in the first instance required to attend, under all circumstances,
substantially complied with the desire of at least 70% of the total classes held in that
Ram semester as per University rules.
(c) Shyam can compel Ram to sell his house
because Ram's offer does not exclude the Facts: Anand, an economically poor but a
payment of price in instalments very brilliant student of LL.B. final semester,
(d) Shyam can not compel Ram to sell his while going to his University bycycle received
house because Shyam imposes a new some leg injuries in road accident.
condition about payment of price of the Consequently Anand could not attend his
house while accepting the offer which is classes for one week as he was advised rest
not ultimately accepted by Ram. by his doctor for that period. Due to this
absence from the University, Anand failed to
152. Principle: Generally an agreement without have 70% attendance essential to appear in
consideration is not valid. Therefore, in order the examination and, therefore, he was
to make a valid agreement, some debarred from appearing in the examination
consideration which may have some value in by the University authorities. Anand
the eyes of law, is essentially required. challenges this decision in the court of law

Previous Years
Page 120 CLAT & AILET Papers
(a) Anand will succeed in the court of law as Facts: A large number of persons had been
the accident was beyond his control carrying on the business of dyeing and printing
(b) Anand will definitely get favour of the court in Rajkot area for the last 25 years providing
on humanitarian ground as he comes from employment to about 30000 families. From
a economically poor family and may not these business places untreated dirty water
afford to take readmission was being discharged on the roads thereby
(c) Anand will not succeed as he could very causing damage to the public health. A notice,
easily fulfill eligibility criteria for appearing therefore, was given to close this business
in the examination by being reasonably till necessary measures to protect public
regular in the class throughout the health as provided under the environmental
semester. statutes were taken by those business men.
(d) Anand will succeed as requirement of 70%
attendance may be declared arbitrary and, (a) Notice can not be justified as it will cause
therefore, unreasonable by the court of loss of employment to 30000 families.
law. (b) Notice can not be justified as it amounts
to violation of the fundamental right of the
154. Principle: A seller of goods cannot transfer persons who have been carrying on the
better rights than he himself possesses in business for the last 25 years.
the goods sold to the buyer. (c) The notice can not be justified on the
ground of damage to public health as the
Facts: Komal leaves his watch by mistake persons in that area have been voluntarily
on a seat in the park. Sonal finds that watch residing for long and have become used
and immediately sells the same for good price to that environment.
to Monal who without inquiring whether Sonal (d) The notice can be justified as the right to
is its owner or not. Komal later on claims business is not absolute and reasonable
that watch from Monal. Decide whether Komal restriction can be imposed by law in the
can succeed? interest of the public.

(a) Komal cannot succeed as Monal has paid 156. Principle: A Contract can not be enforced
good price of the watch. by or against a person who is not a party to
(b) Komal cannot succeed as Monal is it. However, where some benefit is conferred
unaware of the fact that Sonal is not its on third party by the contract itself, there third
owner. party can be allowed to enforce that contract
(c) Komal can not succeed as it was his to get such benefit.
carelessness and nothing else which
enabled Sonal to sell the watch to Monal. Facts: Dinesh is liable to pay ` 50000/- to
(d) Komal can not succeed as Sonal is merely Suresh. In order to discharge this liability
finder of the watch and, therefore, cannot Dinesh enters into a contract with Ramesh
transfer ownership rights thereon to by which Dinesh sells his car to Ramesh for
Monal. ` 1 lac. Ramesh takes the delivery of the car
and promises/ assures to pay its price at the
155. Principle: All citizens shall have the earliest. Dinesh separately informs Suresh
Fundamental Right to carry on any about this contract for his satisfaction.
occupation, trade or business. But reasonable Ramesh fails to pay the car's price. Suresh
restrictions on the exercise of such rights can wants to join Dinesh in filing suit against
be imposed by law in the interest of the Ramesh for the recovery of price of the car.
general public. Whether Suresh is entitled to do so?

Previous Years
CLAT & AILET Papers Page 121
(a) Suresh is entitled to do so because the (c) Hani can succeed as he properly posted
contract was made for his benefit. the letter of acceptance and the delay was
(b) Suresh is entitled to do so because beyond his control
Dinesh is liable to him and discharge of (d) Hani can succeed as contract became
this liability depends upon the payment complete in the eyes of law on the date of
of the price of the car by Ramesh. posting the letter of acceptance
(c) Suresh is not entitled to do so because
liability of Dinesh does not depand upon 158. Principle: He, who goes to the court of law
any assurance of Ramesh to seek justice, must come with clean hands.
(d) Suresh is not entitled to do because he
is not a party to the contract between Facts: P enters into a contract with S under
Dinesh and Ramesh. which S has to construct a house for P and
has to complete the same within one year
157. Principle: If a contract is made by post from the date of the contract. This contract
between two persons living in two different includes two very important terms. According
cities, then the contract is said to be complete to first term if there is price hike of the
as soon as the letter of acceptance is properly materials to be used in the construction, then
posted, and the place of completion of the the escalation charges at a particular rate shall
contract is that city where acceptance is be payable by P to S. According to second
posted. It is worth mentioning here that in term if the construction of the house is not
every contract there is always an offer from completed within the period prescribed for it,
one party and the acceptance of the offer from then S will have to pay penalty at a particular
the other party. rate to P. Before the completion of the
construction work the workers of S go on
Facts: Sani, a resident of Patna, gives an strike and strike continues up to three months
offer by post to sell his house for ` 25 lacs to even after the expiry of one year. After that
Hani, a resident of Allahabad. This offer letter period workers return and the construction
is posted on 1st January 2013 from Patna and work again starts. During the last three
reaches Allahabad on 7th January 2013. Hani months period of strike there was a
accepts this offer and posts the letter of considerable rise in the price of the building
acceptance on 8 th January 2013 from material. S claimed escalation cost from P.
Allahabad which reaches Patna on 16th P did not agree to it. S filed a suit in the court
January 2013. But Sani presuming that Hani of law either to order the payment of the price
is not interested in accepting his offer, sells of the building material on the basis of
his house to Gani at same price on 15th of escalated price or to allow him to stop the
January, 2013. Hani files a suit against Sani work without incurring any penal liability
for the breach of contract in the competent towards P.
court of Allahabad. Whether Hani will
succeed? (a) S will succeed as strike by his workers
was unexpected and beyond his control.
(a) Hani can not succeed as Sani can not be (b) S can succeed as there is an escalation
compelled by law to wait for the answer clause in the contract
from Hani for an indefinite period of time. (c) S can not succeed as he has failed to
(b) Hani can not succeed as he could use complete the construction work in time
some other effective and speedy mode and strike can not be treated as a valid
for communicating his acceptance in excuse for delay in work.
minimum possible time (d) S can succeed if he pays penalty to P for
delay.

Previous Years
Page 122 CLAT & AILET Papers
159. Principle: If the object of an agreement is or Due to some sudden mechanical/ technical
becomes unlawful or immoral or opposed to problem, these plants ceased to work properly
public policy in the eyes of law, then the courts and, therefore, caused environmental
will not enforce such agreements. Law pollution, which ultimately caused substantial
generally prohibits Child labour. harm to the environment and to the people
living around the factory. Victims of such
Facts: P enters into an agreement with T by pollution file a suit for suitable remedy.
which P has to let his house to T for two years
and T has to pay ` 20000.00 per month to P (a) Victims can not succeed as necessary
as rent. T starts a child care centre in that precautions to prevent any harm were
house. But after some time in order to earn taken by Hari.
some money for the maintenance of the (b) Victims can not succeed as the
centre, T starts sending the children of the mechanical/ technical problem was
centre on the rotation basis to work for four sudden and, therefore, beyond the control
hour a day in some nearby chemical and of Hari.
hazardous factories. When P comes to know (c) Victims can succeed as it is the duty of
about this new development, he asks T either Hari to see that no harm is caused to the
to stop the children from working in factories environment/ people due to his activity
or to leave his house immediately. T neither under any circumstances.
agrees to leave the house nor to stop the (d) Victims could succeed if treatment/
children from working in the factories. P files disposal plant were not installed in the
a suit in the court of law for appropriate relief/ factory.
action
161. Principle: If a person transfers movable or
(a) P can not succeed as the agreement was immovable property with its full ownership and
for the two years and it can not be without any consideration to some other
terminated before the expiry of that period person, then it is called a gift.
(b) P can not succeed as the object at the Facts: S, who has no child of his own, makes
time of making of the agreement was not a gift of his house worth ` 25 lacs to his
clear nephew R. After completing all the legal
(c) P will succeed as the object of the formalities required for a valid gift, S says to
agreement has become unlawful. R that in case of need R will provide that
(d) P will not succeed if T agrees to share house to S for use without any questions. R
the wages of the children with P does not react to it. After one year of this gift,
S really needs that house and request R to
160. Principle: whosoever by his act or omission make the house available to him, but R
causes environmental pollution shall be held refuses to do so.
liable for any loss caused by such pollution.
It shall be no defence in such cases that all (a) R cannot refuse as he got the house
due diligence or reasonable care was taken without paying any consideration for that.
while carrying out the act or omission in (b) R cannot refuse as S is without children.
question. (c) R can refuse as he has become full owner
of the house.
Facts: Hari is carrying on a chemical and (d) R can refuse as he himself may be in need
fertilizer industry near a bank of a river. In order of that house.
to prevent and control any kind of harm to the
environment, suitable waste treatment and
disposal plants were installed in the factory.

Previous Years
CLAT & AILET Papers Page 123
162. Principle: An agreement to do an act (a) The author is likely to succeed in the case
impossible in itself cannot be enforced by a as the agreement is not in accordance
court of law. with the law
(b) The author cannot succeed in the case
Facts: Ramesh agrees with his girl friend as he has given his consent to the
Shilpa to pluck stars from the sky through his agreement
extraordinary will power, and bring them down (c) The author is not likely to succeed in the
on earth for her within a week. After the expiry case because he has already accepted
of one week, Shilpa filed a suit for damages the amount of ` 10,00,000/- as full an
against Ramesh for the breach of contract as final payment
Ramesh failed to perform his promise. (d) The author can succeed in the case as
the consideration is not adequate
(a) Shilpa can succeed in getting damages
as Ramesh has deceived her. 164. Principle: A person is said to be of sound
(b) Ramesh cannot be held liable as he mind for the purpose of making a contract if,
honestly believes that his love for Shilpa at the time when he makes it, he is capable
is true and, therefore, he will succeed in of understanding it and of forming a rational
his endeavour. judgment as to its effect upon his interest.
(c) The court cannot entertain such suits as
the act promised under the agreement is Facts: X who is usually of sound mind, but
impossible in itself. occasionally of unsound mind enters into a
(d) Ramesh can be held liable for making an contract with Y when he (X) is of unsound
absurd promise. mind. Y came to know about this fact
afterwards and now wants to file a suit against
163. Principle: An agreement may be oral or X.
written. However, if a law specifically requires
that an agreement must be in writing then (a) X cannot enter into contract because he
the agreement must be in writing. A law is of unsound mind when he entered into
specifically requires that the agreements contract.
relating to transfer of the copyright in novel (b) X can enter into contract but the burden
between an author of a novel and the producer is on the other party to prove that he was
of a motion picture must be in writing. of unsound mind at the time of contract.
(c) X can enter into contract but the burden
Facts: The author of a novel, Love at Lost is on X to prove that he was of sound mind
Sight, had several rounds of discussion with at the time of contract.
a producer of motion picture regarding making (d) None of these.
of a motion picture based on Love at Lost
Sight. During the last discussion, they 165. Principle: Whosoever commits any act
decided to make a motion picture on Love at forbidden by the Indian Penal code with a view
Lost Sight. The producer made a motion to obtain the consent of any person to enter
picture on Love at Lost Sight after making a into an agreement, he cannot get the
payment of ` 10, 00,000/- (Ten Lac Only) in agreement enforced by law but the person
cash to the author who happily accepted this whose consent has been so obtained may
amount as full and final payment. Later on, get the agreement enforced by law. The Indian
on the advice of his lawyer, the author brought Penal code defines various offences and
a case in a court of law against the producer prescribes punishments therefor.
on the ground that there is no written
agreement between the producer and him.

Previous Years
Page 124 CLAT & AILET Papers
Facts: A obtains the consent of B to enter Facts: X, the uncle of Y, made a promise to
into an agreement by an act amounting to pay him an amount of ` 10,000/- as reward if
criminal intimidation under the Indian Penal Y quits smoking and drinking within one year.
Code. A brings a case against B for X also deposited the above mentioned amount
performance of agreement. in a bank and informed Y that the said amount
will be paid to him if he quits smoking and
(a) A will succeed in the case drinking within one year. Within a period of
(b) A may succeed in the case six months of making the promise X died.
(c) B will succeed in the case After the expiry of one year of making the
(d) B will not succeed in the case promise by X, Y made a request to the legal
heirs of X demanding the promised money.
166. Principle: A contract between the father and The legal heirs of X declined the request of Y.
his son is a contract of utmost good faith. In
such a type of contract law presumes that at
(a) The promise of X to Y is enforceable by
the time of entering into the contract the father
law because Y has quitted smoking and
was in a position to dominate the will of his
drinking
son. Where one of the parties was in position
(b) The promise of X to Y is not enforceable
to dominate the will of the other party, the
contract is enforceable only at the option of by law because Y has benefitted by
the party whose will was so dominated. quitting smoking and drinking
(c) The promise of X to Y is enforceable by
Facts: Ram had advanced a sum of ` 10,000/ law because the amount of ` 10, 000/-
- to his minor son Shyam. When Shyam has been deposited in a bank
became major, his father Ram misused his (d) The promise of X to Y is enforceable by
parental position and entered into an law because X has died within a period of
agreement with Shyam and obtained a bond six months of making the promise
from him for a sum of ` 30,000 in respect of
the advance. Whether this agreement is 168. Principle: 'Work' means literary work, artistic
enforceable? work, dramatic work, musical work,
cinematographic film and sound recording but
(a) The agreement is enforceable against does not include acting in a cinematographic
Shyam only for ` 10,000/-, the actual film.
amount of money advanced to him Only the works, as enumerated above, can
(b) The agreement is enforceable against be protected under copyright law.
Shyam for ` 30,000/- because he has
signed the bond Facts: A very famous actor acted in a
(c) The agreement is enforceable against cinematographic film. The actor was also the
Shyam because he was major at the time producer and director of cinematographic film.
of agreement.
(d) The agreement is not enforceable against (a) The acting of the actor can be protected
Shyam because Ram has misused his under copyright law
position as father to obtain an unfair (b) The acting of the actor can be protected
advantage.
under copyright law only as an artistic
work
167. Principle: When at the desire of one person,
(c) The acting of the actor cannot be protected
any other person has done or abstained from
under copyright law
doing something, such act or abstinence or
promise is called a consideration for the (d) The acting of the actor cannot be protected
promise. Only a promise coupled with under copyright law as a cinematographic
consideration is enforceable by law. film

Previous Years
CLAT & AILET Papers Page 125
169. Principle: In case where the government is (c) The agreement between A and B is both
a party, the government shall be the first owner in the nature of assignment and licence
of the copyright in the work unless there is (d) The agreement between A and B is neither
an agreement to the contrary. in the nature of assignment nor in the
nature of licence
Facts: The Government of the State of X
entered into an agreement with a retired 171. Principle: Qui facit per alium facit per se,
Professor of Botany. The Professor agreed to i.e. he who does things through others does
write a text book on Botany. The Government it himself.
agreed to pay a sum of ` Ten Lacs to the
Professor for this work. Facts: Nisha, the owner of a car, asked her
friend Saurabh to take her car and drive the
(a) The Government of the State of X shall be same to her office. As the car was near her
first owner of copyright in the text book office, it hit a pedestrian Srikant on account
(b) The Professor shall be first owner of of Saurabh's negligent driving and injured him
copyright in the text book seriously. Now Srikant files a suit for damages
(c) Both the Government of the State of X against Nisha.
and the Professor shall be the joint owners
of copyright in the text book (a) Nisha is not liable as it was the negligence
(d) The Professor shall be first owner of of Saurabh.
copyright in the text book only if he (b) Saurabh is solely liable as Nisha was not
refuses to accept the amount of ` Ten driving the car.
Lacs from the Government (c) Nisha is liable as Saurabh was driving
under her authority and for her purpose.
170. Principle: Licence is an agreement whereby (d) Saurabh will be exempted from liability
the owner of the copyright agrees to grant an under the principle of inevitable accident.
interest in the copyright to the licencee.
Assignment is an agreement whereby the 172. Principle: Words describing quality of things
owner of the copyright transfers all the cannot be registered as trade mark. However,
property rights to the assignee. Property right such words may be registered as trade mark
is a bundle of rights consisting of right to if they acquire a secondary meaning. Words
possess, right to use, right to alienate, and acquire secondary meaning when people
the right to exclude other start associating the descriptive words with a
person specific.
Facts: A, an owner of copyright in a
cinematographic film enters into an Facts: A hatchery located in Raipur is owned
agreement with B, a film distributer. B agrees by ‘X'. X has been using the slogan "new laid
to distribute the film only in Mumbai. A also eggs sold here" since 1970 to describe the
enters into many such agreements with other quality of eggs sold in his hatchery. Over a
distributers for distribution of his film in other period of time because of continuous use of
cities. this slogan, people started associating this
slogan with X. X filed an application for
(a) The agreement between A and B is more registration of the words "new laid eggs" as
in the nature of assignment than in the trade mark in the year 1970.
nature of licence
(b) The agreement between A and B is more (a) The words will be registered as trade mark
in the nature of licence than in the nature (b) The words will not be registered as trade
of assignment mark

Previous Years
Page 126 CLAT & AILET Papers
(c) The words may be registered as trade (c) Surgeon will be responsible and Seema
mark need not to prove surgeon's negligence
(d) The words may not be registered as trade because presence of abdominal pack in
mark her abdomen is sufficient proof therefor.
(d) None of the above
173. Principle: Whoever takes away anything from
the land of any person without that person's 175. Principle: Whoever takes away with him any
consent is said to commit theft. A thing so minor person less than sixteen years of age
long as it is attached to the earth is not the if a male, or less than eighteen years of age if
subject of theft; but it becomes capable of a female out of the keeping of parents of such
being the subject of theft as soon as it is minor person without the consent of such
severed from the earth. parent, is said to kidnap such minor person.

Facts: Y cuts down a tree standing on the Facts: A female born on January 01, 1995
land of X with the intention of dishonestly got admitted to an undergraduate program of
taking the tree out of X's possession without a reputed University on July 01, 2012. She
the consent of X. But Y is yet to take away became friendly with one of the boys, born
the tree out of X's possession. on June 01, 1994, of her class. The boy
and the girl decided to marry. The parents of
(a) Y has committed theft as soon as he the boy agreed but the family of the girl did
came to the land of X not agree. On December 15, 2012 the girl
(b) Y has committed theft as soon as the made a call from her Blackberry to the boy.
tree has been completely cut down by The girl told the boy to come in his car at a
him particular place and time. The boy reached
(c) Y has committed theft as soon as he has the stipulated place before the stipulated time.
started cutting down the tree He waited there for about half an hour. The
(d) Y has not committed theft because he is girl reached the stipulated place. She opened
yet to take away the tree out of X the door of the car and sat beside the boy
possession who was on the driving seat. Without
exchanging any pleasantries, the boy drove
174. Principle: res ipsa loquitur i.e. the thing the car to an unknown place. The father of
speaks for itself. the girl lodged an FIR in the nearest police
station on January 20, 2013.
Facts: Seema got herself operated for the
removal of her uterus in the defendant's (a) The boy has committed the offence of
hospital, as there was diagnosed to be a cyst kidnapping
in one of her ovaries. Due the negligence of (b) The boy has not committed the offence of
the surgeon, who performed the operation, kidnapping
abdominal pack was left in her abdomen. The (c) The boy has not committed the offence of
same was removed by a second surgery. kidnapping for there is a delay in filing the
FIR
(a) Surgeon cannot be held responsible (d) The boy has not committed the offence of
because it is merely a human error. kidnapping because the girl was his
(b) Surgeon can be held responsible but classmate
Seema will have to prove in the court of
law that the surgeon was grossly
negligent.

Previous Years
CLAT & AILET Papers Page 127
176. Principle: Whoever voluntarily has carnal 179. Principle: A pact, other than a pact to
intercourse against the order of nature with commit suicide, to suffer any harm is not an
any man, woman or animal shall be punished. offence, provided the age of the person who
has given his consent to suffer harm is above
Facts: Two adult men were found engaged in eighteen year
carnal intercourse by the police. The police
arrested the men and produced them before Facts: A enters into a pact with B, a boy of
the Court. less than 18 years of age, to fence with each
other for amusement. They agreed to suffer
(a) Court will punish the police officer who any harm which, in the course of such fencing,
arrested the men may be caused without foul play.
(b) Court will not punish the men for they were
adults (a) A, while playing fairly, hurts B, A commits
(c) Court will punish the men no offence
(d) Court may be requested to declare the (b) A, while playing only unfairly, hurts B, A
law unconstitutional. commits an offence
(c) A, while playing fairly, hurts B, A commits
177. Principle: Wherever the causing of a certain an offence
effect, or an attempt to cause that effect, by (d) A, while playing unfairly, hurts B, A
an act or by an omission, is an offence, it commits no offence
is to be understood that the causing of that
effect partly by an act and partly by an 180. Principle: When an act, which would
omission is the same offence. otherwise be an offence, is not that offence
by reason of the youth, the want of maturity
Facts: A intentionally omitted to give food to of understanding, the unsoundness of mind
his father. He also used to beat his father. or the intoxication of the person doing that
Consequently A's father died. act, every person has the same right of private
defence against that act which he would have
(a) A did not commit any offence if the act were that offence. Nothing is an
(b) A committed only the offence of omitting offence which is done in the exercise of the
to give food to his father right of private defence.
(c) A committed only the offence of beating
of his father Facts: A, under the influence of madness,
(d) A committed the offence of killing of his tempts to kill B. B in order to save his life
father causes grievous hurt to A.

178. Principle: Nothing is an offence which is done (a) A has committed an offence
by a child under seven years of age. (b) A has not committed an offence
(c) B has committed an offence
Facts: A, a child born on January 01, 2005 (d) B has not committed any offence
killed another child ‘B' on December 30, 2011.
181. Principle: Mere silence as to facts likely to
(a) A has committed no offence. affect the decision of a person to enter into a
(b) A has committed the offence as it is contract does not amount to fraud, unless
heinous crime his silence is in itself equivalent to speech.
(c) Killing of one child by another child is not
an offence Facts: A sells to B a horse which A knows to
(d) A has not committed the offence for on be of unsound mind. B says to A that if A
the date of killing of B, A was a minor

Previous Years
Page 128 CLAT & AILET Papers
does not say anything about the state of mind 183. Principle: Where two or more persons have
of horse, then B shall presume that the horse made a complaint for the grant of compulsory
is of sound mind. A says nothing to B about licence to the Copyright Board, the licence
the mental condition of horse. shall be granted to that complainant only who,
in the opinion of the Copyright Board, would
(a) A has committed fraud best serve the interests of the general public.
(b) A has committed misrepresentation
(c) There cannot be a fraud because A says Facts: Four persons made a complaint for
nothing about the mental condition of the the grant of compulsory licence to the
horse Copyright Board.
(d) There can not be a fraud because B did
not ask A whether the horse is of sound (a) Licence shall be granted to only one
mind complainant
(b) Licence may be granted to two
182. Principle: Whoever by words, either spoken complainants
or written brings or attempts to bring into (c) Licence may be granted to three
hatred or contempt, or excites or attempts to complainants
excite disaffection towards the Government (d) Licence must be granted to all the four
established by law in India shall be punished. complainants
However, comments expressing
disapprobation of the administrative or other 184. Principle: Any police officer, not below the
action of the Government without exciting or rank of a sub-inspector, may, if he is satisfied
attempting to excite hatred, contempt or that an offence in respect of the infringement
disaffection, do not constitute an offence. of copyright in any work has been, is being,
or is likely to be, committed, seize without
Facts: A renowned professor of economics warrant, all copies of the work, and all plates
wrote a critical comment on the economic used for the purpose of making infringing
policies of the Government of India in a copies of the work, wherever found, and all
National Daily. This piece of writing generated copies and plates so seized shall, as soon
academic debate not only in the print media as practicable, be produced before a
but also on television and internet. A student Magistrate.
of law asked the fellow Indians on a social
networking website to assemble at a particular Facts: A Superintendent of Police (SP)
place for peaceful and silent demonstration conducted a raid on a shop and found pirated
against the said economic policies on a copies of books. The SP formed an opinion
stipulated date and time. The crowed that infringement of copyright is taking place.
assembled at that venue and started shouting He arrested the shop owner without warrant
anti-government slogans. Police arrested the in the light of above mentioned propositions.
professor.
(a) The arrest of the shop owner was within
(a) The professor has committed the offence the power of the SP
(b) The professor has not committed any (b) The arrest of the shop owner was not
offence within the power of the SP
(c) The student of law has committed the (c) The shop owner can never be arrested
offence (d) SP was not competent to know whether
(d) The crowed has committed an offence infringement of copyright has taken place

Previous Years
CLAT & AILET Papers Page 129
185. Principle: Whoever attempts to commit an Facts: ‘A' and ‘B' were working in a factory
offence punishable by the Indian Penal Code as unskilled labourers. A was carrying a
and in such attempt does any act towards basket of stones on his head. B was sitting
the commission of the offence, shall be on the ground. When A crossed B, all of a
punished. Stealing is an offence punishable sudden a stone fell down from the basket and
by the Indian Penal Code. hit B on his head. B died immediately.

Facts: A makes an attempt to steal some (a) The owner of the factory will be liable
jewels by breaking open a box, and after so (b) A and the owner of the factory shall be
opening the box, finds that there is no jewel jointly liable
in it. (c) The owner of the factory will not be liable
(d) None of the above
(a) A has committed no offence
(b) A has committed the offence of stealing 188. Principle: Damages are the money
(c) A has attempted to commit the offence of recompense, as far as money can do, for the
stealing violation of a right.
(d) None of the above
Facts: A, an Indian citizen, having a right to
186. Principle: Whoever by words either spoken vote, was not allowed to cast his vote on the
or intended to be read, or by signs or by visible polling booth, by the returning officer. Name
representations, makes or publishes any of A was mentioned in the voter's list. A has
imputation concerning any person intending also reported at the polling booth in time.
to harm, or knowing or having reason to However, the candidate in whose favour A
believe that such imputation will harm, the would have cast his vote won the election. A
reputation of such person, is said to defame filed a suit claiming damages.
that person.
(a) A will be entitled to damages
Facts: In a community there is a custom of (b) A will not be entitled to damages
stealing shoes of bridegroom during the (c) A will be entitled to only nominal damages
marriage ceremony. The shoes of the (d) A will be entitled to exemplary damages
bridegroom were stolen by Y. 'A' announced
that Z has stolen the shoes. Everyone present 189. Principle: When a party to a contract has
in the marriage party started staring at Z with refused to perform, or disabled himself from
great surprise. Z felt very ashamed. performing, his promise in its entirety, the
other party may put an end to the contract.
(a) A defamed Z
(b) A did not defame Z Facts: A engaged B on April 12 to enter his
(c) A defamed Z for Z felt very ashamed service on June 1, but on May 11, A wrote to
(d) A defamed the whole marriage party B that his services would not be needed. On
May 22, B joined C for employment.
187. Principle: An employer is liable for the
negligence of his employee. But an employer (a) B must wait till June 1
is not liable for the negligence of his employee (b) B must have joined C on May 11
if the victim of such negligence is one of his (c) B is not bound to wait till June 1
other employees. (d) A must pay damages to B

Previous Years
Page 130 CLAT & AILET Papers
190. Principle: When a person voluntarily agrees 192. Principle: A condition precedent must be
to suffer some harm, he is not allowed to complied with before the happening of the
complain for that. event to which such a condition is attached.
Fulfillment of such a condition after the
Facts: ‘A' was one of the spectators at a happening of the event is no fulfillment of
formula one car race, being held at Gurgaon, condition.
on a track owned by one ‘M' company. During
the race, there was a collision between two Facts: A transfers ` 5000 to B on condition
racing cars, one of which was thrown away that he shall marry with the consent of C, D
amidst spectators, thereby causing an injury and E. As C, D and E had to go abroad for
to ‘A'. ‘A' claims damages for the injuries some business purposes and as the date of
caused to him. marriage was already fixed, therefore, B
marries without the consent of C, D and E,
(a) M company will be liable for damages but obtains their consent after the marriage
because the injury was caused during the when C, D and E return to their country.
race organized by it
(b) M company will not be liable for damages (a) B has fulfilled the condition
because A had come to see the race on (b) B has not fulfilled the condition
his own will. (c) B was free to marry any one without the
(c) M company will not be liable for damages consent of any body
because the collision between the cars (d) B must divorce his wife as he married her
was beyond its control without fulfilling the condition
(d) M company will be liable because it has
earned huge revenue by way of sale of 193. Principle: In an agreement, a condition
tickets for the event subsequent must be complied with, to claim
the benefit of that agreement.
191. Principle: An interest which is created on a
transfer of property and depends upon the
Facts: A agrees to transfer a farm to B,
fulfillment of a condition will fail if the fulfillment
provided that, if B does not go to England
of the condition is impossible or is forbidden
within three years after the date of the
by law or is of such a nature that, if permitted,
agreement, his interest in the farm shall
it would defeat the provisions of any law or is
cease. B does not go to England within the
fraudulent or involves or implies injury to the
term prescribed.
person or property of another or the court
regards it as immoral or opposed to public
policy. (a) B's interest in the farm continues
(b) B's interest in the farm does not continue
Facts: A gives ` Ten Lacs to B on condition (c) B has a fundamental right to go to England
that B shall marry A's daughter C. On the or not to go to England and hence the
date on which A gave ` Ten Lacs to B, C was condition was illegal
dead. (d) The agreement between A and B was void

(a) B's interest in ` Ten Lacs fails because of


impossibility
(b) B's interest in ` Ten Lacs fails because of
immorality
(c) B's interest in ` Ten Lacs fails because of
prohibition by law
(d) B's interest in ` Ten Lacs does not fail

Previous Years
CLAT & AILET Papers Page 131
194. Principle: Existence of all the alleged facts 196. Principle: The fact that any person was born
is relevant whether they occurred at the same during the continuance of a valid marriage
time and place or at different times and places. between his mother and any man, or within
two hundred and eighty days after its
Facts: A, a permanent resident in a foreign dissolution, the mother remaining unmarried,
country who never visited India, is accused of shall be conclusive proof that he is the
waging war against the Government of India legitimate son of that man, unless it can be
by taking part in an armed insurrection in shown that the parties to the marriage had
which property is destroyed, troops are no access to each other at any time when he
attacked and prisons are broken open. could have been begotten.

(a) The existence of all the above mentioned Facts: X and Y married on January 15, 1995.
alleged facts is relevant Y, the wife of X, never left her parental home
(b) Only the alleged fact that A is accused of and never went to her husband's home. A boy
waging war against the Government of was born to Y on July 15, 1995. For the Court:
India is relevant
(c) The fact that A was a permanent resident (a) There shall be a conclusive proof that the
in a foreign country who never visited India boy is the legitimate son of X
is not relevant (b) There shall be no conclusive proof that
(d) Only the alleged fact of taking part by A the boy is the legitimate son of X
in armed resurrection is relevant (c) There shall be a conclusive proof that the
boy is the illegitimate son of X
195. Principle: Whoever desires any Court to give (d) There shall be no evidence at all.
judgment about any legal right or liability which
depends on the existence of those facts which 197. Principle: An unlawful interference with a
he asserts, must prove that those facts exist. person's use or enjoyment of land, or some
right over, or in connection with it, is a
Facts: A asserts that B, C and D have nuisance in law of tort.
committed an offence of criminal conspiracy
and therefore A desires a Court to give Facts: During the scarcity of onions, long
judgment that B, C and D shall be punished queues were made outside the defendant's
for that crime which A says B, C and D have shop who having a license to sell fruits and
committed. vegetables used to sell only 1 Kg. of onion
per ration card. The queues extended on to
(a) A must prove that B, C, and D have the highway and also caused some
committed the crime obstruction to the neighbouring shops.
(b) B, C, and D must prove that they have The neighboring shopkeepers filed a suit for
not committed the crime nuisance against the defendant. Which one
(c) A must prove that B, C, and D were of the following decisions will be correct in
present at the place of crime this suit?
(d) Police must prove that B, C, and D have
committed the crime (a) The defendant is liable for nuisance
(b) The defendant is not liable for nuisance
(c) The defendant is liable under the principle
of strict liability
(d) The plaintiff's suit should be decreed in
favour of the neighbouring shopkeeper

Previous Years
Page 132 CLAT & AILET Papers
198. Principle: Every agreement in restraint of the Whether Sharman has committed an
marriage of any person, other than a minor, offence?
is void. (a) Sharman has committed no offence
because this was done out of necessity
Facts: Qadir Khan died in a road accident. (b) Sharman can be held responsible for the
Two co-widows, Sultana and Marjina enter act of criminal negligence
into an agreement that if any of them will (c) Sharman can be held responsible for
remarry, would forfeit her right to her share in culpable homicide
the deceased husband's property. (d) This is a clear case of accident so
Sharman cannot be held responsible
(a) The agreement is void because it was
restraint of marriage 200. Principle: Only the Parliament or the State
(b) The agreement is not void because no Legislatures have the authority to enact laws
restraint was imposed upon either of two on their own. No law made by State can take
widows for remarriage. away a person's fundamental right.
(c) The restraint was partial so agreement is
valid Facts: Parliament enacted a law, which
(d) None of the above according to a group of lawyers is violating
the fundamental rights of traders. The group
199. Principle: Nothing is an offence merely by of lawyers filed a writ petition against the
reason of its being done with the knowledge Parliament for enacting such law and requests
that it is likely to cause harm, if it be done the court to quash the law and to direct the
without any criminal intention to cause harm, Parliament to make a new law.
and in good faith for the purpose of
preventing or avoiding other harm to a person (a) No writ would lie against the Parliament,
or property. as the Court has no authority to direct
the Parliament to enact or re-enact a law.
Facts: Mr. Sharman, the Italian captain of a (b) The Court can quash the existing law if it
steam vessel, suddenly and without any fault violates fundamental right and can direct
or negligence on his part, finds himself near to make a new law.
the Kochi coast in such a position that before (c) The Court can quash the existing law if it
he can stop his vessel, he must inevitably violates fundamental right but cannot
run down a boat B with twenty or thirty direct the Parliament to make a new law.
passengers on board, unless he changes the (d) None of these
course of his vessel, and that by changing
his course, he must incur risk of running down
a boat C with only two passengers on board,
which he may possibly clear.

Previous Years
CLAT & AILET Papers Page 133
CLAT Question Paper 2014

ENGLISH INCLUDING COMPREHENSION


Directions for Questions 1 to 10: Fill in the blank by choosing the most appropriate option.

1. A vote of ................. proposed at the end of the meeting.


(A) thanks were (B) thank was (C) thanks had been (D) thanks was

2. During the recession many companies will.................lay off workers.


(A) be forced to (B) have the force to (C) forcefully (D) be forced into

3. She has good.................over the famous foreign languages.


(A) expertise (B) command (C) control (D) authority

4. The Chairman pointed out in favour of the manager that the profitability of the industrial plant had
............. since he took over the administration.
(A) arisen (B) declined (C) added (D) increased

5. When the examinations were over,................went to Paris.


(A) me and Rohan (B) I and Rohan (C) Rohan and me (D) Rohan and I

6. Let's go for a walk,......................... ?


(A) can we (B) shall we (C) cant' we (D) shouldn’t we

7. Had Anil been on time, he................................ missed the train.


(A) would not have been (B) had not
(C) will not have (D) would not have

8. The most alarming fact is that infection is spreading.......................the state and reaching villages
and small towns.
(A) over (B) across (C) far (D) from

9. In big cities people are cut............................... from nature.


(A) off (B) down (C) away (D) out

10. The dissidents.............a great problem in every political party.


(A) give (B) cause (C) pose (D) hold

Directions for Questions 11 to 15: The sentences given in each question, when properly sequenced, form
a coherent paragraph. Each sentence is labelled with a letter. Choose the most logical order of sentences
from among the given choices to construct a coherent paragraph.

11. (a) Payment for imports and exports is made through a system called foreign exchange. The value
of the money of one country in relation to the money of other countries is agreed upon.
(b) The rates of exchange vary from time to time.

Previous Years
Page 134 CLAT & AILET Papers
(c) For example, an American dollar or a British pound sterling is worth certain amounts in the
money of other countries.
(d) Sometimes a US dollar is worth 60 rupees in India.

(A) abcd (B) bacd (C) acbd (D) cabd

12. (a) When a dictionary is being edited, a lexicographer collects all the alphabetically arranged citation
slips for a particular word.
(b) The moment a new word is coined, it usually enters the spoken language.
(c) The dictionary takes note of it and makes a note of it on a citation slip.
(d) The word then passes from the realm of hearing to the realm of writing.

(A) abcd (B) acbd (C) bacd (D) bcad

13. (a) The impression that corruption is a universal phenomenon persists and the people do not co-
operate in checking this evil.
(b) So there is hardly anything that the government can do about it now.
(c) It is regrettable that there is a widespread corruption in the country at all levels.
(d) Recently several offenders were brought to book, but they were not given deterrent punishment.

(A) cdab (B) adbc (C) adcb (D) cbad

14. (a) In all social affairs convention prescribes more or less generally accepted rules of behaviour.
(b) Of course, there is nothing absolute about conventions.
(c) They vary from country to country, from age to age.
(d) Convention has a necessary part to play in the life of everyone.

(A) abcd (B) adbc (C) dacb (D) dabc

15. (a) In fact, only recently there have been serious studies to find out how many of us actually have
nightmares.
(b) Now that is changing.
(c) The study of nightmares has been curiously neglected.
(d) While results so far are inconclusive, it seems fair to say that at least half the population has
occasional nightmares.

(A) cadb (B) abdc (C) adcb (D) cbad

Directions for Questions 16 to 20: Given below are a few foreign language phrases which are commonly
used. Choose the correct meaning for each of the phrases.
16. Ex officio
(A) By virtue of previously held position (B) Former official
(C) By virtue of office (D) Outside the office

17. Ultra Vires


(A) Within powers (B) Full powers (C) Near powers (D) Beyond powers

18. Quid pro quo


(A) Something for nothing (B) Something for something
(C) Everything for something (D) Something for everything
Previous Years
CLAT & AILET Papers Page 135
19. Inter vivos
(A) between the living (B) among the living and the dead
(C) between the dead (D) among the dead and the living

20. Corpus juris


(A) body of judges (B) group of jurists
(C) body of law (D) knowledge of law

Directions for Questions 21 to 25: Select the word that is spelt CORRECTLY.

21. Which of the following spellings is correct?


(A) Concensus (B) Consencus (C) Consenssus (D) Consensus

22. Which of the following spellings is correct?


(A) Procede (B) Proceed (C) Proceede (D) Proced

23. Which of the following spellings is correct?


(A) Accommodate (B) Acommodate (C) Accomodate (D) Acomodate

24. Which of the following spellings is correct ‘for a page at the beginning of a book’?
(A) Foreward (B) Forword (C) Forworde (D) Foreword

25. Which of the following spellings is correct?


(A) Arguement (B) Argument (C) Arguemant (D) Arguemint

Directions for Questions 26 to 30: Choose the explanation that best reflects the spirit of the idiom/
proverb/phrase given in each question.

26. To make clean breast of:


(A) To tell the truth about something (B) To gain prominence
(C) To destroy before it blooms (D) To praise oneself
27. A man of straw:
(A) A creditable man (B) A very active man
(C) A man of no or little substance (D) An unreasonable man

28. A wild-goose chase:


(A) A wise search (B) A fruitful search
(C) A worthwhile hunt (D) A futile pursuit

29. Put on the market:


(A) To offer for sale (B) Alongside the market
(C) Already purchased (D) None of the above

30. To meet someone halfway:


(A) To show that you are prepared to strain your relationship with someone
(B) To compromise with someone
(C) Confrontation
(D) Incongruity

Previous Years
Page 136 CLAT & AILET Papers
Directions for Questions 31 to 40: The questions in this section are based on a single passage. The
questions are to be answered on the basis of what is stated or implied in the passage. Kindly note that
more than one of the choices may conceivably answer some of the questions. However, you are to choose
the most appropriate answer; that is, the response that most accurately and completely answers the
question.

The spread of education in society is at the foundation of success in countries that are latecomers to
development. In the quest for development, primary education is absolutely essential because it creates
the base. But higher education is just as important, for it provides the cutting edge. And universities are
the life-blood of higher education. Islands of excellence in professional education, such as Indian Institutes
of Technology (IITs) and Indian Institutes of Management (IIMs), are valuable complements but cannot be
substitutes for universities which provide educational opportunities for people at large.

There can be no doubt that higher education has made a significant contribution to economic development,
social progress and political democracy in independent India. It is a source of dynamism for the economy.
It has created social opportunities for people. It has fostered the vibrant democracy in our polity. It has
provided a beginning for the creation of a knowledge society. But it would be a mistake to focus on its
strengths alone. It has weaknesses that are a cause for serious concern.
There is, in fact, a quiet crisis in higher education in India that runs deep. It is not yet discernible simply
because there are pockets of excellence, an enormous reservoir of talented young people and an intense
competition in the admissions process. And, in some important spheres, we continue to reap the benefits
of what was sown in higher education 50 years ago by the founding fathers of the Republic. The reality is
that we have miles to go. The proportion of our population, in the age group 18-24, that enters the world of
higher education is around 7 per cent, which is only one-half the average for Asia. The opportunities for
higher education, in terms of the number of places in universities, are simply not enough in relation to our
needs. What is more, the quality of higher education in most of our universities requires substantial
improvement.

It is clear that the system of higher education in India faces serious challenges. It needs a systematic
overhaul, so that we can educate much larger numbers without diluting academic standards. This is
imperative because the transformation of economy and society in the 21st century would depend, in significant
part, on the spread and the quality of education among our people, particularly in the sphere of higher
education. It is only an inclusive society that can provide the foundations for a knowledge society.

The challenges that confront higher education in India are clear. It needs a massive expansion of opportunities
for higher education, to 1500 universities nationwide, that would enable India to attain a gross enrolment
ratio of at least 15 per cent by 2015. It is just as important to raise the average quality of higher education
in every sphere. At the same time, it is essential to create institutions that are exemplars of excellence at
par with the best in the world. In the pursuit of these objectives, providing people with access to higher
education in a socially inclusive manner is imperative. The realisation of these objectives, combined with
access, would not only develop the skills and capabilities we need for the economy but would also help
transform India into a knowledge economy and society.

31. The principal focus of the passage is:


(A) Primary education (B) Intermediate education
(C) Higher education (D) Entire education system

Previous Years
CLAT & AILET Papers Page 137
32. The style of the passage can be best described as:
(A) Academic (B) Critical and analytical
(C) Comparative (D) None of the above

33. What kind of society can provide the foundation for a knowledge society?
(A) Elite society (B) Contracted society
(C) Exclusive society (D) Inclusive society

34. According to the passage, which one of the following is INCORRECT?


(A) There are no quality institutes providing excellent professional education in India.
(B) Not many people go for higher education in India.
(C) Education is the basis of success.
(D) All the above options are correct.

35. According to the passage, the current state of affairs of higher education in India is:
(A) Satisfactory
(B) Excellent, and there is no need of any expansion of opportunities for higher education
(C) Not good enough, and there is a need of expansion of opportunities for higher education, besides
creating institutions and universities that are models of excellence
(D) Not explained in the passage

36. According to the passage, which of the following is NOT a challenge that confronts higher education
in India?
(A) Expanding opportunities for higher education
(B) Creating institutions and universities that are exemplars of excellence
(C) Substantial improvement in the quality of higher education in most of our universities
(D) Getting into World University Rankings

37. According to the passage, which of the following is CORRECT?


(A) Primary education is very important
(B) Universities are the life-blood of higher education
(C) Transformation of economy and society in the 21st century would depend, in significant part, on
the spread and the quality of education among our people, particularly in the sphere of higher
education
(D) All the above propositions are correct

38. Should the entire university system in India be modelled on premier institutes, such as the IITs and
the IIMs, providing professional education?
(A) Yes (B) For sure
(C) No (D) The passage is silent on this question

39. What is the antonym of the expression ‘cutting edge’?


(A) Conventional (B) Avant-garde (C) Advanced (D) Contemporary

40. What is the meaning of the word ‘discernible’?


(A) Unobtrusive (B) Noticeable (C) Unremarkable (D) Inconspicuous

Previous Years
Page 138 CLAT & AILET Papers
ELEMENTARY MATHEMATICS (NUMERICAL ABILITY)
41. The next number in the sequence is: 19, 29, 37, 43, ............
(A) 45 (B) 47 (C) 50 (D) 53

42. An unknown man is found murdered. The corpse has one gold plated tooth, right ear is pierced and
a healed fracture of left hand thumb. A man with these characteristics is reported missing. What are
the chances (probability) of the corpse being the missing man? (Given the occurrence of the gold-
plated teeth in the area, 1 in 5000, left hand thumb fractures 1 in 20000 and of right ear pierces 1 in
100).
(A) 1 in 1000 (B) 1 in 1000,000 (C) 1 in 1000,000,000 (D)1 in 10,000,000,000

120
43. Value of "A" in the expression, 5 + 12 × 10 ÷ = A × 10, is:
240
(A) 11 (B) 24.5, (C) 34 (D) 6.5

44. The Least Common Multiple (L.C.M.) of 0.12, 9.60 and 0.60 is:
(A) 9.60 (B) 0.12 (C) 0.6 (D) None of these

45. There are 30 boys and 40 girls in a class. If the average age of boys is 10 yr and average age of girls
is 8 yr, then the average age of the whole class is:
(A) 8 yr (B) 8.86 yr (C) 8.2 yr (D) 9 yr

1 1
46. A person spends part of his income on food, part on house rent and remaining <630 on other
3 4
items. The house rent is:
(A) <504 (B) <1512 (C) <378 (D) None of these

47. A person covers a certain distance by car at a speed of 30 km/h and comes back at a speed of 40
km/h. The average speed during the travel is:
(A) 34.3 km/h (B) 35 km/h (C) 37.5 km/h (D) 32.8 km/h

48. An employer reduces the number of employees in the ratio 8:5 and increases their wages in the
ratio 7:9. Therefore, the overall wages bill is:
(A) Increased in the ratio 45:56 (B) Decreased in the ratio 56:45
(C) Increased in the ratio 13:17 (D) Decreased in the ratio 72:35

49. Father is 3 yr. older than the mother and the mother's age is now twice the daughter's age. If the
daughter is 20 yr. old now, then the father's age when the daughter was born is:
(A) 20 yr. (B) 40 yr. (C) 43 yr. (D)23 yr.

50. If 80% of A = 20% of B and B = 5x% of A, then the value of x is:


(A) 75 (B) 80 (C) 90 (D) 85

51. A mixture of 40 L of alcohol and water contains 10% water. How much water should be added to this
mixture, so that the new mixture contains 20% water?
(A) 9L (B) 5L (C) 7L (D) 6L

Previous Years
CLAT & AILET Papers Page 139
52. ‘A’ can do a piece of work in 20 days and ‘B’ can do the same work in 15 days. How long will they
take to finish the work, if both work together?
1 4 1
(A) 13 days (B) 10 days (C) 8 days (D) 17 days
2 7 2

53. A man can row 5 km/h in still water. If the speed of the current is 1 km/h, it takes 3 h more in
upstream than in the downstream for the same distance. The distance is:
(A) 36 km (B) 24 km (C) 20 km (D) 32 km

54. A starts a business with <5000 and B joins the business 5 months later with an investment of
<6000. After a year, they earn a profit of <34000. Find the shares of A and B in the profit amount
depending on their individual investment.
(A) <20000, <14000 (B) <16000, <16000
(C) <14000, <20000 (D) None of these

55. A farmer has some hens and some goats. If the total number of animal heads is 80 and the total
number of animal feet is 200, what is the total number of goats?
(A) 40 (B) 60
(C) 20 (D) Cannot be determined

56. A square field has its area equal to 324 m2. The perimeter of the field is:
(A) 36 m (B)72 m (C) 18 m (D) 6561 m

57. A closed metal box measure 30 cm x 20 cm x 10 cm. Thickness of the metal is 1 cm. The volume
of the metal required to make the box is:
(A) 1041 cm3 (B) 6000 cm3 (C) 4536 cm3 1968 cm3

58. The difference between the simple interest and the compound interest (compounded annually) on
<2000 for 2 yr at 8% per annum will be:
(A) <10 (B) <20 (C) <13 (D) <25

59. A dealer marked his goods 20% above the cost price and allows a discount of 10%. Then the gain
percent is:
(A) 2% (B) 4% (C) 6% (D) 8%

60. A man went to the Reserve Bank of India with <2000. He asked the cashier to give him <10 and <20
notes only in return. The man got 150 notes in all. How many notes of <10 did he receive?
(A)100 (B) 150 (C) 50 (D) 70

GENERAL KNOWLEDGE AND CURRENT AFFAIRS


61. Which of the following Judges of the Supreme Court of India is famously known as the ‘Green
Judge’?
(A) Justice VR Krishna lyyar (B) Justice PN Bhagwati
(C) Justice Kuldip Singh (D) Justice BN Kirpal

62. Law Day is observed on:


(A) 26th January (B) 26th May (C) 15th August (D)26th November

Previous Years
Page 140 CLAT & AILET Papers
63. In which year, Defence Research & Development Organisation (DRDO) was formed by the
amalgamation of the Technical Development Establishment (TDE) of the Indian Army and the
Directorate of Technical Development & Production (DTDP) with the Defence Science Organisation
(DSO):
(A) 1955 (B)1958 (C) 1959 (D) 1963

64. The Reserve Bank of India was established in the year:


(A) 1858 (B) 1935 (C) 1947 (D) 1950

65. Lord Buddha’s image is sometimes shown with the hand gesture, called ‘Bhumisparsha Mudra’. It
signifies:
(A) Buddha’s calling of the Earth to watch over Mara and to prevent Mara from disturbing his meditation.
(B) Buddha’s calling of the Earth to witness his purity and chastity despite the temptations of Mara.
(C) The gesture of debate or discussion/argument.
(D) Both (A) and (B) are correct.

66. Which of the following statements is INCORRECT about Fundamental Duties under the Constitution
of India? It shall be the duty of every citizen of India —
(A) To uphold and protect the sovereignty, unity and integrity of India.
(B) To strive towards excellence in all spheres of individual and collective activity so that the nation
constantly rises to higher levels of endeavour and achievement.
(C) Who is a parent or guardian to provide opportunities for education to his child or, as the case
may be, ward between the age of six and fourteen years.
(D) To vote in public elections.

67. Which one of the following scripts of ancient India was mostly written from right to left?
(A) Nandnagari (B) Brahmi (C) Kharoshti (D) Sharada

68. During the time of which Mughal Emperor did the East India Company establish its first factory in
India?
(A) Akbar (B)Jahangir (C) Shahjahan (D) Aurangzeb

69. Who, among the following, first translated the Bhagwat Gita into English?
(A) Charles Wilkins (B) Alexander Cunningham
(C) William Jones (D) James Prinsep

70. Match List I with List II and select the correct answer using the codes given below the lists:

List I (Author) List II (Work)


(a) Devaki Nandan Khatri (1) Chandrakanta
(b) Premchand (2) Durgeshnandini
(c) Bankim Chandra Chattopadhyay (3) Nil Daman
(4) Seva Sadan
Codes:
(A) a-4; b-3; c-1 (B) a-1; b-4; c-3
(C) a-2; b-1; c-3 (D) a-1; b-4; c-2

Previous Years
CLAT & AILET Papers Page 141
71. Match List I with List II and select the correct answer using the codes given below the lists:

List I (Person) List II (Position)


(a) Nagendra Singh
(1) Chief Election Commissioner of India
President of the International Court of
(b) SH Kapadia (2)
Justice
(c) NR Madhava Menon (3) Former Chief Justice of India
Legal educator and Founder-Director of
(d) VS Sampath (4)
National Law School of India University
Codes:
(A) a-2; b-3; c-l; d-4 (B) a-4; b-3; c-2; d-1
(C) a-1; b-2; c-4; d-3 (D) a-2; b-3; c-4; d-1

72. Who was not a Chief Justice of India?


(A) Justice M Patanjali Sastri (B) Justice KN Wanchoo
(C) Justice HR Khanna (D) Justice MN Venkatachaliah

73. Who was the first Attorney-General for India?


(A) CK Daphtary (B) MC Setalvad (C) Niren De (D) LN Sinha

74. Under the leadership of Mahatma Gandhi, the Civil Disobedience Movement, launched in 1930,
started from?
(A) Sabarmati (B) Dandi (C) Sevagram (D) Champaran

75. Match List I with List II and select the correct answer using the codes given below the lists:

List I (Events) List II (Results)


(a) Dandi March (1) Communal electorate
(b) Chauri Chaura (2) Illegal manufacture of salt
(c) Simon Commission (3) Country-wise agitation
(d) Morley Minto Reforms (4) Withdrawal of a movement

Codes:
(A) a-1; b-2; c-3; d-4 (B) a-4; b-3; c- 2; d-1
(C) a-2; b-4; c-3; d-1 (D) a-1; b-4; c-3; d-2

76. Which of the following planets has the maximum number of natural satellites?
(A) Earth (B) Mars (C Jupiter (D) Saturn

77. In India, the interest rate on savings accounts in all the nationalised commercial banks is fixed by?
(A) Union Ministry of Finance (B) Union Finance Commission
(C) Indian Banks' Association (D) None of the above

78. Solvents are the substances used to dissolve other substances. Consider the following substances:
(1) Water (2) Ether (3) Toluene (4) Chloroform (5) Ethanol
Which of the above can be used as solvents?
(A) 1 and 2 (B) 2 and 3 (C) 3, 4 and 5 (D) All of these

Previous Years
Page 142 CLAT & AILET Papers
79. Viruses are parasitic, having DNA/RNA; but, they can be crystallised and lack respiration. Therefore,
they are treated as:
(A) Living beings (B) Non-living beings
(C) Both living and non-living beings (D) None of the above

80. Who is the winner of the coveted Dadasaheb Phalke Award for the year 2013?
(A) Javed Akhtar (B)Gulzar (C) AR Rahman (D) Indeevar

81. Who among the following has been recently appointed as brand ambassador for Central Reserve
Police Force (CRPF):
(A) Aamir Khan (B) Mahendra Singh Dhoni
(C) MC Mary Kom (D) Sania Mirza

82. Till date (2014), how many people have been awarded the Bharat Ratna Award?
(A) 40 (B) 41 (C) 42 (D) 43

83. Which country Malala Yousafzai belongs to?


(A Pakistan (B) Afghanistan (C) The USA (D) England

84. Who recently became the first woman chief of the State Bank of India?
(A) Arundhati Bhattacharya (B) Shubhalakshmi Panse
(C) Vijaylakshmi Iyer (D) Chanda Kochhar

85. Who is the Chairman of the 14th Finance Commission?


(A) Dr M Govinda Rao (B) Dr Vijay Kelkar
(C) Dr YV Reddy (D) Dr Raghuram Rajan

86. The Chairperson of the Seventh Pay Commission is:


(A) Justice BN Srikrishna (B) Justice DK Jain
(C) Justice AP Shah (D) Justice AK Mathur

87. Which one of the following is NOT correctly matched?

United Nations Specialized Agency Headquarters


(A) International Civil Aviation Organization Montreal
(B) World Trade Organization Geneva
(C) United Nations Industrial Development Organization Brussels
(D) International Fund for Agricultural Development Rome

88. The First Five Year Plan of India was based on:
(A) John W Miller Model (B) PC Mahalanobis Model
(C) Gadgil Yojana (D) Herrod-Domar Model

89. Which one of the following measures is NOT likely to aid in improving India's balance of payment
position?
(A) Promotion of import substitution policy (B) Devaluation of rupee
(C) Imposition of higher tariff on imports (D) Levying the higher duty on exports

Previous Years
CLAT & AILET Papers Page 143
90. Find out the correct chronological sequence of the following persons’ visit to lndia at one time or
another: (1) Fa-Hien (2) I-Tsing (Yijing) (3) Megasthanese (4) Hiuen-Tsang.
(A) 1, 3, 2, 4 (B) 1, 3, 4, 2 (C) 3, 1, 4, 2 (D) 3, 1, 2, 4

91. Which was the first newspaper to be published in India?


(A) Bombay Samachar (B) The Hindu (C) Bengal Chronicle (D) Bengal Gazette

92. The change in the colour of stars is linked to:


(A) Variation in their surface temperature
(B) Variation in their distance from the earth
(C) Fluctuations in their composition and size
(D) Irregular absorption or scattering in earth’s atmosphere

93. Which one of the following does NOT remain to be a planet now?
(A) Neptune (B) Uranus (C) Pluto (D) Venus

94. Who was appointed as the 23rd Governor of the Reserve Bank of India?
(A) KC Chakrabarty (B) Urijit Patel (C) Raghuram Rajan (D) D Subbarao

95. Which one of following offices is held during the pleasure of the President of India?
(A) Vice-President (B) Governor of a State
(C) Chief Justice of India (D) Comptroller and Auditor General of India

96. Who was the first winner of the prestigious Jnanpith Award?
(A) Tarasankar Bandyopadhyay (B) Kuppali Venkatappagowda Puttappa
(C) G Sankara Kurup (D) Umashankar Joshi

97. Match List I with List II and choose the correct answer:
List I List II
(a) Visakhadatta (1) Surgery
(b) Varahamihira (2) Drama
(c) Sushruta (3) Astronomy
(d) Brahmagupta (4) Mathematics

Codes:
(a) (b) (c) (d)
(A) 1 3 4 2
(B) 2 1 3 4
(C) 2 3 1 4
(D) 3 4 1 2

98. Who was among the following honoured with Arjun Award in Chess for the year 2013?
(A) Kavita Chahal (B) Parimarjan Negi (C) Ronjan Sodhi (D) Abhijeet Gupta

99. Economic growth rate projected by the IMF for India in the fiscal year 2014-15 is:
(A) 5.4 percent (B) 5.5 percent (C) 5.6 percent (D) 5.7 percent

Previous Years
Page 144 CLAT & AILET Papers
100. Who was sworn in as the Prime Minister of Italy on 22nd February 2014?
(A) Oleksandr Turchynov (B) Matteo Renzi
(C) Enrico Letta (D) Giorgio Napolitano

101. Which one of following is the highest peacetime gallantry award of India?
(A) Param Vir Chakra (B) Ashok Chakra (C) Maha Vir Chakra (D) Kirti Chakra

102. Which one of the following gases is lighter than air?


(A) Carbon dioxide (B) Chlorine (C) Oxygen (D) Hydrogen

103. Name the scientist who is known as the father of modern genetics:
(A) Jean-Baptiste Lamarck (B) Hugo de Vries
(C) Gregor Johann Mendel (D) Charles Darwin

104. Which year was designated by the United Nations as International Women’s Year?
(A) 1974 (B) 1975 (C) 1976 (D) 1977

105. Mention the correct abbreviation for ATM:


(A) Automated Teller Machine (B) Any Time Money
(C) All Time Money (D) Auto-limited Teller Machine

106. General Election is being held in India from 7 April to 12 May 2014 to constitute:
(A) 14th Lok Sabha (B) 15th Lok Sabha (C)16th Lok Sabha (D) 17th Lok Sabha

107. Mention the name of the current Chief Justice of India:


(A) Justice Altamas Kabir (B) Justice P Sathasivam
(C)Justice R M Lodha (D) Justice H L Dattu

108. The 9th Ministerial Conference of the WTO, held during 3 December-6 December 2013, was concluded
at:
(A) Geneva, Switzerland (B) Kaula Lumpur, Malaysia
(C) Warsaw, Poland (C) Bali, Indonesia

109. Nobel Prize in Literature for the year 2013 was awarded to:
(A) Mo Yan (B) James E Rothman (C) Lars Peter Hansen (D)Alice Munro

110. Which one of the following satellites was successfully launched by the ISRO’s Polar Satellite
Launch Vehicle (PSLV-C24) on 4th April 2014?
(A) IRNSS-1B (B) GSAT-14 (C) INSAT-3D (D) SARAL

Previous Years
CLAT & AILET Papers Page 145
LOGICAL REASONING
Directions for Questions 111 to 113: Read the following information carefully and answer the questions
given below:

Five friends Satish, Rajesh, Rehman, Rakesh, and Vineet, — each presents one paper to their class on
Physics, Zoology, Botany, English, or Geology — one day a week, Monday through Friday.
(i) Vineet does not present English and does not give his presentation on Tuesday.
(ii) Rajesh makes the Geology presentation, and does not do it on Monday or Friday.
(iii) The Physics presentation is made on Thursday.
(iv) Rehman makes his presentation, which is not on English, on Wednesday.
(v) The Botany presentation is on Friday, and not by Rakesh.
(vi) Satish makes his presentation on Monday.

111. What day is the English presentation made?


(A) Friday (B) Monday (C) Tuesday (D) Wednesday

112. What presentation does Vineet do?


(A) English (B) Geology (C) Physics (D) Botany

113. What day does Rakesh make his presentation on?


(A) Monday (B) Tuesday (C) Wednesday (D) Thursday

Directions for Questions 114 to 118: Each question contains a statement on relationship and a question
regarding relationship based on the statement. Choose the correct option.

114. Pointing to a photograph, a man said, “I have no brother or sister but that man’s father is my father's
son”. Whose photograph was that?
(A) His father’s (B) His own (C) His son’s (D) His nephew’s

115. Ranjan introduces Abhay as the son of the only brother of his father’s wife. How is Abhay related to
Ranjan?
(A) Son (B) Brother (C) Cousin (D) Uncle

116. Pointing to a lady on the stage, Bhumika said, “She is the sister of the son of the wife of my
husband. How is the lady related to Bhumika?
(A) Cousin (B) Sister-in-law (C) Sister (D) Daughter

117. Pinky, who is Victor’s daughter, says to Lucy, “Your mother Rosy is the younger sister of my father,
who is the third child of Joseph”. How is Joseph related to Lucy?
(A) Father-in-law (B) Father (C) Maternal uncle (D) Grandfather

118. Pramod told Vinod, “Yesterday I defeated the only brother of the daughter of my grandmother”.
Whom did Pramod defeat?
(A) Father (B) Son (C) Father-in-law (D) Cousin

Previous Years
Page 146 CLAT & AILET Papers
Directions for Questions 119 to 123: Read the information given below to answer the questions.
(i) In a family of six persons, there are people from three generations. Each person has separate
profession and also each one likes different colours. There are two couples in the family.
(ii) Charan is a CA and his wife neither is a doctor nor likes green colour.
(iii) Engineer likes red colour and his wife is a teacher.
(iv) Vanita is mother-in-law of Namita and she likes orange colour.
(v) Mohan is grandfather of Raman and Raman, who is a principal, likes black colour.
(vi) Sarita is granddaughter of Vanita and she likes blue colour. Sarita’s mother likes white colour.

119. Who is an Engineer?


(A) Sarita (B) Vanita (C) Namita (D) Mohan

120. What is the profession of Namita?


(A) Doctor B) Engineer (C) Teacher (D) Cannot be determined.

121. Which of the following is the correct pair of two couples?


(A) Mohan — Vanita and Charan — Sarita (B) Vanita — Mohan and Charan — Namita
(C) Charan — Namita and Raman — Sarita (D) Cannot be determined

122. How many ladies are there in the family?


(A) Two (B) Three (C) Four (D) None of these

123. Which colour is liked by CA?


(A) White (B) Blue (C) Black (D) None of these

Directions for Questions 124 to 128: Read the information given below carefully and answer the questions.

124. Two buses start from the opposite points of a main road, 150 kms apart. The first bus runs for 25
kms and takes a right turn and then runs for 15 kms. It then turns left and runs for another 25 kms
and takes the direction back to reach the main road. In the meantime, due to a minor breakdown,
the other bus has run only 35 kms along the main road. What would be the distance between the
two buses at this point?
(A) 65 kms (B) 75 kms (C) 80 kms (D) 85 kms

125. ‘G’, ‘H’, ‘I’, ‘J’, ‘K’, ‘L’, ‘M’, ‘N’ are sitting around a round table in the same order for group discussion
at equal distances. Their positions are clock wise. If ‘M’ sits in the north, then what will be the
position of ‘J’?
(A) East (B) South-East (C) South (D) South-West

126. Roshan, Vaibhav, Vinay and Sumit are playing cards. Roshan and Vaibhav are partners. Sumit
faces towards North. If Roshan faces towards West, then who faces towards South?
(A) Vinay (B) Vaibhav (C) Sumit (D) Data is inadequate

127. Five boys are standing in a row facing East. Pavan is to the left of Tavan, Vipin, Chavan. Tavan,
Vipin, Chavan are to the left of Nakul. Chavan is between Tavan and Vipin. If Vipin is fourth from the
left, then how far is Tavan from the right?
(A) First (B) Second (C) Third (D) Fourth

Previous Years
CLAT & AILET Papers Page 147
128. One morning after sunrise, Suraj was standing facing a pole. The shadow of the pole fell exactly to
his right. Which direction was Suraj facing?
(A) West (B) South (C) East (D) Data is inadequate

Directions for Questions 129 to 130: Read the information given below to answer the questions.
Diana is three times older than Jackson; Edward is half the age of Stephen. Jackson is older than Edward.

129. Which one of the following can be inferred?


(A) Jackson is older than Stephen
(B) Diana is older than Stephen
(C) Diana may be younger than Stephen
(D) None of the above

130. Which one of the following information will be sufficient to estimate Diana’s age?
(A) Edward is 10 year old
(B) Both Jackson and Stephen are older than Edward by the same number of years
(C) Both A and B above
(D) None of the above

Directions for Questions 131 to 135: Study the sequence/pattern of letters or numbers carefully to work
out the pattern on which it is based, and answer what the next item in the sequence must be. For example,
the sequence A, C, E, G, ?’ has odd numbered letters of alphabet; therefore, the next item must be T.

131. 0, 3, 8, 15,?
(A) 24 (B) 26 (C) 35 (D) None

132. 8, 16, 28, 44, ?


(A) 60 (B) 64 (C) 62 (D) 66

133. 0, 6, 24, 60, 120, 210, ?


(A) 290 (B) 240 (C) 336 (D) 504

134. POQ, SRT, VUW, ?


(A) XYZ (B) XZY (C) YZY (D) YXZ
135. A1, C3, F6, J10, O15,?
(A) U21 (B) V21 (C) T20 (D) U20

Directions for Questions 136 to 140: Two words, which have a certain relation, are paired. Select a
correct option to substitute the question mark so as to make a similar relational pair with the word given
after double colon (::).

136. Bow : Arrow :: Pistol : ?


(A) Gun (B) Shoot (C) Rifle (D) Bullet

137. Eye : Wink :: Heart: ?


(A) Throb (B) Move (C) Pump (D) Respirate

138. Ocean : Water :: Glacier : ?


(A) Cooling (B) Cave (C) Ice (D) Mountain

Previous Years
Page 148 CLAT & AILET Papers
139. Prima facie : On the first view :: In pari delicto : ?
(A) Both parties equally at fault (B) While litigation is pending
(C) A remedy for all disease (D) Beyond powers

140. Delusion : hallucination :: Chagrin : ?


(A) Illusion (B) Ordered (C) Cogent (D) Annoyance
Directions for Questions 141 to 142: Each question comprises two statements (numbered as I and II).
You have to take the statements as true even if they seem to be at variance with commonly known facts.
Read all the conclusions and then decide which of the given conclusions logically follow from the given
statements, disregarding commonly known facts.

141. (I) All contracts are agreements.


(II) All agreements are accepted offers.
Which of the following derivations is correct?
(A) All accepted offers are contracts (B) All agreements are contracts
(C) All contracts are accepted offers (D) None of the above

142. (I) Some beautiful women are actresses.


(II) All actresses are good dancers.
Which of the following derivations is correct?
(A) Some beautiful women are good dancers (B) All good dancers are actresses
(C) Both (A) and (B) (D) None of the above

Directions for Questions 143 to 145: Two statements are given below followed by two conclusions (I and
II). You have to consider the two statements to be true even if they seem to be at variance with commonly
known facts. You have to decide which of the conclusions, if any, follow from the given statements.

143. Statements:
Some books are magazines.
Some magazines are novels.
Conclusions:
(I) Some books are novels.
(II) Some novels are magazines.

(A) Only (I) follows (B) Only (II) follows


(C) Both (I) and (II) follow (D) Neither (I) nor (II) follows

144. Statements:
All students like excursions.
Some students go for higher education.

Conclusions:
(I) Students who go for higher education also like excursions.
(II) Some students do not go for higher education, but like excursions.

(A) Only (I) follows (B) Only (II) follows


(C) Both (I) and (II) follow (D) Neither (I) nor (II) follows

Previous Years
CLAT & AILET Papers Page 149
145. Statements:
All good hockey players are in the Indian Hockey team.
‘X’ is not a good hockey player.

Conclusions:
(I) ‘X’ is not in the Indian Hockey team.
(II) ‘X’ wants to be in the Indian Hockey team.

(A) Only (I) follows (B) Only (II) follows


(C) Both (I) and (II) follow (D) Neither (I) nor (II) follows

Directions for Questions 146 to 148: In each of the following questions, a related pair of words is followed
by four pairs of words or phrases. Select the pair that best expresses a relationship similar to the one
expressed in the question pair.

146. India : Tricolour


(A) China : Sickle and Hammer (B) UK : Red Cross
(C) USA : Stars and Stripes (D) None of the above

147. Statute : Law


(A) Proviso : Clause (B) Chapter : Exercise
(C) University : School (D) Section : Illustration

148. Buddhists : Pagoda


(A) Parsis : Temple (B) Christians : Cross
(C) Jains : Sun Temple (D) Jews : Synagogue

Directions for Questions 149 to 150: Each question consists of, five statements (a-e) followed by options
consisting of three statements put together in a specific order. Choose the option which indicates a valid
argument; that is, where the third statement is a conclusion drawn from the preceding two statements.

149. a. Law graduates are in great demand.


b. Rajesh and Krishna are in great demand.
c. Rajesh is in great demand.
d. Krishna is in great demand.
e. Rajesh and Krishna are law graduates.

Choose the correct option:


(A) abe (B) ecd (C) aeb (D) eba

150. a. All captains are great players.


b. Some captains are successful sports administrators.
c. Ritwik is a great player.
d. Ritwik is a captain and successful sports administrator.
e. Some successful sports administrators are great players.

Choose the correct option:


(A) acd (B) abe (C) dca (D) edc

Previous Years
Page 150 CLAT & AILET Papers
LEGAL APTITUDE
Directions for Questions 151 to 200: This section consists of fifty (50) questions. Each question
consists of legal propositions/principles (hereinafter referred to as ‘principle’) and .facts. These
principles have to be applied to the given facts to arrive at the most reasonable conclusion. Such
principles may or may not be true in the real sense, yet you have to conclusively assume them to be
true for the purposes of this section. In other words, in answering the following questions, you must
not rely on any principles except the principles that are given herein below for every question.
Further, you must not assume any facts other than those stated in the question. The objective of this
section is to test your interest towards study of law, research aptitude and problem solving ability
even if the `most reasonable conclusion' arrived at may be unacceptable for, any other reason. It is
not the object of this section to test your knowledge of law.

151. Principle: When one person signifies to another his willingness to do or to abstain from doing
anything, with a view to obtaining the assent of that other to such act or abstinence, he is said to
make a proposal. The expression of willingness/desire results in a valid proposal only when it is
made/addressed to some person(s).
Facts: `X' makes the following statement in an uninhabited hall: ‘I wish to sell my mobile phone for
<1,000.’

Which of the following derivations is CORRECT?


(A) `X' made a statement that resulted in a promise
(B) `X' made a statement that resulted in a proposal
(C) `X' made a statement that did not result in any proposal
(D) `X' made a statement that resulted in an agreement

152. Principle: A proposal (offer) should be made with an intention that after its valid acceptance, a
legally binding promise or agreement will be created. The test for the determination of such intention
is not subjective, rather it is objective. The intention of the parties is to be ascertained from the
terms of the agreement and the surrounding circumstances under which such an agreement is
entered into. As a general rule, in the case of arrangements regulating social relations, it follows as
a matter of course that the parties do not intend legal consequences to follow. On the contrary, as
a general rule, in the case of arrangements regulating business affairs, it follows as a matter of
course that the parties intend legal consequences to follow. However, the above rules are just
presumptive in nature, and hence, can be rebutted.
Facts: One morning while having breakfast, `X', the father, says to `Y' (X's son), in a casual manner,
‘I shall buy a motorbike for you if you get through the CLAT.'

Which of the following derivations is CORRECT?


(A) `X' made a statement that resulted in an enforceable promise
(B) `X' made a statement that resulted in a valid proposal
(C) `X' made a statement that resulted in an enforceable agreement
(D) `X' made a statement that did not result in any enforceable agreement

Previous Years
CLAT & AILET Papers Page 151
153. Principle: Acceptance (of offer) must be communicated by the offeree to the offeror so as to give
rise to a binding obligation. The expression `by the offeree to the offeror' includes communication
between their authorised agents.
Facts: `X' made an offer to buy Y's property for a stipulated price. `Y' accepted it and communicated
his acceptance to `Z', a stranger.

Which of the following derivations is CORRECT?


(A) Y’s acceptance resulted in an agreement
(B) Y’s acceptance did not result in any agreement
(C) Y’s acceptance resulted in a contract
(D) Y’s acceptance resulted in a promise

154. Principle: Acceptance should be made while the offer is still subsisting. The offeror is free to retract
his offer at any time before his offer gets accepted by the offeree. Once the offer is withdrawn or is
lapsed, it is not open to be accepted so as to give rise to a contract. Similarly, if a time is prescribed
within which the offer is to be accepted, then, the offer must be accepted within the prescribed time.
And, if no time is prescribed, then, the acceptance must be made within a reasonable time. `What
is a reasonable time', is a question of fact which is to be determined by taking into account all the
relevant facts and surrounding circumstances.
Facts: `X' makes an offer to `Y' to sell his equipment for <1,000.00. No time is specified for the
acceptance. `Y' sends his reply two years after receiving the offer.

Which of the following derivations is CORRECT?


(A) There arises a contract between `X' and `Y' to sell/buy the equipment in question for <1,000.00
(B) There does not arise any contract between `X' and `Y' to sell/buy the equipment in question for
<1,000.00
(C) `X' is bound by his offer, and hence, cannot reject the acceptance made by `Y'
(D) There arises a promise by `Y' to buy the equipment

155. Principle: Minor's agreement is void from the very beginning. It can never be validated. It cannot be
enforced in the court of law.
Facts: `A', a boy of 16 years of age, agrees to buy a camera from `B', who is a girl of 21 years of age.

Which of the following derivations is CORRECT?


(A) There arises a contract between `A' and `B' to sell/buy the camera in question
(B) There arises an enforceable agreement between `A' and `B' to sell/buy the camera in question
(C) There does not arise any contract between `A' and 'B' to sell/buy the camera in question
(D) There arises a voidable contract between `A' and `B', to sell/buy the camera in question

156. Principle: A contract which is duly supported by real and lawful consideration is valid notwithstand-
ing the fact that the consideration is inadequate. The quantum of consideration is for the parties to
decide at the time of making a contract, and not for the courts (to decide) when the contract is
sought to be enforced. An agreement to which the consent of the promisor is freely given is not void
merely because the consideration is inadequate; but the inadequacy of the consideration may be
taken into account by the Court in determining the question whether the consent of the promisor
was freely given.
Facts: `A' agrees to sell his mobile phone worth <20,000/- for <100/- only to 'B'. A's consent is freely
given.

Previous Years
Page 152 CLAT & AILET Papers
Which of the following derivations is CORRECT?
(A) There is a contract between `A' and `B'
(B) There is no contract between `A' and `B' because consideration is not adequate
(C) There is no contract between `A' and `B' because a mobile phone worth <20,000/- cannot be
sold for just <100/-
(D) None of the above.

157. Principle: The consideration or object of an agreement is unlawful if it is forbidden by law. Every
agreement of which the object or consideration is unlawful is void.
Facts: `X', promises to pay `Y' <50,000, if he (`Y') commits a crime. `X' further promises to indemnify
him (`Y') against any liability arising thereof. `Y' agrees to act as per X's promise.

Which of the following derivations is CORRECT?


(A) There is a contract between `X' and `Y'
(B) There is an agreement between `X' and `Y' which can be enforced by the court of law
(C) There is an agreement between `X', and `Y' which cannot be enforced by the court of law
(D) There is a voidable contract between `X' and `Y'

158. Principle: The consideration or object of an agreement is unlawful if the Court regards it as
opposed to public policy. Every agreement of which the object or consideration is unlawful is void.
Facts: `X' promises to obtain for `Y' an employment in the public service; and `Y' promises to pay
<5,00,000/- to `X'.

Which of the following derivations is CORRECT?


(A) There is a contract between `X' and `Y'
(B) There is a voidable contract between `X' and `Y'
(C) There is an agreement between `X' and `Y' which can be enforced by the court of law
(D) There is an agreement between `X', and `Y' which cannot be enforced by the court of law

159. Principle: Two or more persons are said to consent if they agree upon the same thing in the same
sense. Consent is said to be free when it is not caused by coercion, or undue influence, or fraud, or
misrepresentation, or mistake. When consent to an agreement is caused by coercion, undue influ-
ence, fraud or misrepresentation, the agreement is a contract voidable (rescindable or terminable)
at the option of the party whose consent was so caused. However, when consent to an agreement
is caused by mistake as to a matter of fact essential to the agreement, the agreement is void.
Facts: `X' threatens to gun down `Y', if he (`Y') does not sell his property worth <20,00,000/- for
<1,00,000/- only. As a consequence, `Y' agrees to sell it as demanded by A'.

Which of the following derivations is CORRECT?


(A) There is a contract between `X' and `Y'
(B) There is an agreement between `X' and `Y' which can be enforced by the court of law
(C) There is an agreement between `X', and `Y' which cannot be enforced by the court of law
(D) There is a contract between `X' and `Y' which voidable at the option of `Y'

Previous Years
CLAT & AILET Papers Page 153
160. Principle: Agreements in restraint of marriage are void.
Facts: `X' enters into an agreement with `Y' where under he agrees not to marry anybody else other
than a person whose name starts with the letter `A', and promises to pay <1,00,000/- to `Y' if he (`X')
breaks this agreement.

Which of the following derivations is CORRECT?


(A) There is a contract between `X' and `Y'
(B) There is an agreement between `X' and `Y' which can be enforced by the court of law
(C) There is an agreement between `X', and `Y' which cannot be enforced by the court of law
(D) There is a voidable contract between `X' and `Y'

161. Principle: Vicarious liability is the liability of the Master or Principal for the tort committed by his
servant or agent, provided the tort is committed in the course of employment. The Master or Princi-
pal is not liable for private wrongs of the servant/ agent.
Facts: `X' hands over some cash money at his house to `Y', who is his (X's) neighbour and is also,
cashier in a bank, to be deposited in A's account in the bank. Instead of depositing the money, `Y'
misappropriates it.

Which of the following statements depicts correct legal position in this given situation?
(A) The bank would not be liable because `Y' did not do any wrong in the course of his employment
(B) The bank would be vicariously liable because `Y' was the employee of the bank
(C) The bank would not be liable because `Y' did not do any wrong
(D) The bank would be liable because `Y' acted as bank's agent

162. Principle: A person has no legal remedy for an injury caused by an act to which he has consented.
Facts: `R', a cricket enthusiast, purchases a ticket to watch a T20 match organised by the Indian
Premier League (IPL). During the match, a ball struck for six hits `R' on his body and injures him. He
sues IPL for compensation for the medical expenses.

Which of the following derivations is CORRECT?


(A) `R' should be compensated as he purchased the ticket to get entertainment and not to get
injured
(B) `R' would fail in his action, as he voluntarily exposed himself to the risk
(C) IPL would be liable as it did not ensure that the spectators were protected from the risk of such
injuries
(D) None of the above

163. Principle: Ignorance of law excuses no one.


Facts: `X' fails to file his income tax returns for a considerable number of years. The Income Tax
department serves upon him a `show-cause notice' as to why proceedings should not be initiated
against him for the recovery of the income tax due from him with interest and penalty.
Which of the following derivations is CORRECT?

(A) `X' may defend himself by taking the plea that his legal advisor had not advised him to file the
return
(B) `X' would have to pay the due, as ignorance of law and failure to comply with law is no legal
ground of defence
(C) `X' may defend himself successfully by taking the plea that he was unaware of any such law
being in force
(D) None of the above
Previous Years
Page 154 CLAT & AILET Papers
164. Principle: Damage without the violation of a legal right is not actionable in a court of law. If the
interference with the rights of another person is not unlawful or unauthorised, but a necessary
consequence of the exercise of defendant's own lawful rights, no action should lie.
Facts: There was an established school (`ES') in a particular locality. Subsequently, a new school
('NS') was set up in the same locality, which charged lower fees, on account of which people started
patronising the new school. Because of the competition, `ES' had to reduce its fees. `ES' filed a
case against 'NS' saying that 'NS' had caused it (`ES') financial loss and, thus, claimed compensa-
tion.

Which of the following derivations is CORRECT?


(A) Since no legal right of `ES' had been violated, therefore, as such, no compensation could be
granted
(B) Since damage is caused to `ES', therefore, it should be awarded compensation
(C) `ES' should be awarded compensation, as opening of school in competition is not good
(D) No compensation could be granted, as reduction in fees is good for the public

165. Principle: Whenever there is an invasion of a legal right, the person in whom the right is vested, is
entitled to bring an action though he has suffered no actual loss or harm, and may recover damages
(compensation).
Facts: `A' was a qualified voter for the Lok Sabha election. However, a returning officer wrongfully
refused to take A's vote. In spite of such wrongful refusal, the candidate, for whom `A' wanted to vote,
won the election. But, `A' brought an action for damages:

Which of the following derivations is CORRECT?


(A) Since no legal right of `A' had been violated, therefore, as such, no compensation could be
granted
(B) Since legal right of `A' had been violated, therefore, compensation should be granted
(C) No compensation could be granted, as `A' had suffered no loss as his candidate won the
election
(C) Since no fundamental right of `A' had been violated, therefore, as such, no compensation could
be granted

166. Principle: In a civil action for defamation, truth of the defamatory matter is an absolute defence.
However, the burden of proving truth is on the defendant; and he is liable if he does not successfully
discharge this burden.
Facts: `D', who was the editor of a local weekly, published a series of articles mentioning that `P',
who was a government servant, issued false certificates, accepted bribe, adopted corrupt and illegal
means to mint money and was a `mischief monger'. `P' brought a civil action against `D', who could
not prove the facts published by him.
Under the circumstances, which of the following derivations is CORRECT?
(A) `D' would be liable, since he could not prove the facts published by him
(B) `D' would not be liable, as such an action could curtail the right of expression and speech of
press
(C) `D' would not be liable, as media could publish anything
(D) None of the above

Previous Years
CLAT & AILET Papers Page 155
167. Principle: A gift comprising both existing and future property is void as to the latter.
Facts: `X' has a house which is owned by him. He contracted to purchase a plot of land adjacent to
the said house, but the sale (of the plot of land) in his favour is yet to be completed. He makes a gift
of both the properties (house and land) to 'Y'.

Under the afore-mentioned circumstances, which of the following derivations is CORRECT?


(A) Gift of both the properties is valid
(B) Gift of both the properties is void
(C) Gift of house is void, but the gift of the plot of land is valid
(D) Gift of house is valid, but the gift of the plot of land is void

168. Principle: Caveat emptor, i.e. `let the buyer beware' stands for the practical skill and judgment of
the buyer in his choice of goods for purchase. It is the business of the buyer to judge for himself that
what he buys has its use and worth for him. Once bought, and if the buy is not up to his expecta-
tions, then he alone is to blame and no one else.
Facts: For the purpose of making uniform for the employees, `A' bought dark blue coloured cloth
from 'B', but did not disclose to the seller (B') the specific purpose of the said purchase. When
uniforms were prepared and used by the employees, the cloth was found unfit. However, the cloth
was fit ' for a variety of other purposes (such as, making caps, boots and carriage lining, etc).

Applying the afore-stated principle, which of the following derivations is CORRECT as regards rem-
edy available to `A' in the given situation?
(A) `A' (the buyer) would succeed in getting some remedy from `B' (the seller)
(B) `A' (the buyer) would not succeed in getting any remedy from `B' (the seller)
(C) `A' (the buyer) would succeed in getting refund from `B' (the seller)
(D) `A' (the buyer) would succeed in getting a different variety of cloth from 'B' (the seller), but not the
refund

169. Principle: The transferor of goods cannot pass a better title than what he himself possesses.
Facts: `X' sells a stolen bike to `Y' buys it in good faith.

As regards the title to bike, which of the following derivations is CORRECT?


(A) The real owner cannot get back the bike from `Y'
(B) `Y' will get no title, as transferor's (X's) title was defective
(C) `Y' will get good title, as he is a bona fide buyer
(D) `Y' will get good title, as has not committed any wrong (stolen the bike)

170. Principle: Negligence is a breach of duty or a .failure of one party to exercise the standard of care
required by law, resulting in damage to the party to whom the duty was owed. A plaintiff can take civil
action against the respondent, if the respondent's negligence causes the plaintiff injury or loss of
property.
Facts: `D' went to a cafe and ordered and paid for a tin/can of soft drink. The tin was opaque, and,
therefore, the contents could not be seen from outside. She (‘D') consumed some of the contents
and then lifted the tin to pour the remainder of the content into a tumbler. The remains of a snail in
decomposed state dropped out of the tin into the tumbler. `D' later complained of a stomach pain
and her doctor diagnosed her as having gastroenteritis and being in a state of severe shock. She
sued the manufacturer of the drink for negligence.

Previous Years
Page 156 CLAT & AILET Papers
Applying the afore-stated principle, which of the following derivations is CORRECT as regards liabil-
ity of the manufacturer in the given situation?
(A) The manufacturer is liable for negligence, as it owed a duty (to consumers) to take reasonable
care to ensure that its products are safe for consumption
(B) The manufacturer is not liable for negligence, as there is no direct contract between `D' and the
manufacturer. No duty is owed by the manufacturer towards a particular consumer (‘D')
(C) The manufacturer is not liable for negligence because it would otherwise become very difficult for
the manufacturers to do business
(D) The manufacturer could be made liable under criminal law, but not for tort of negligence

171. Principle: Master is liable for the wrongful acts committed by his servant; provided the acts are
committed during the course of employment. However, the master is not liable if the wrongful act
committed by his servant has no connection, whatsoever, with the servant's contract of employ-
ment.
Facts: `D' is a driver employed by `M', who is the owner of a company. During the lunch time, `D'
goes to a closeby tea shop to have a cup of tea. There he (`D') picks up fight with the tea shop owner
('T'), which resulted in some damage to his shop. `T' wants to sue `M' for claiming compensation for
the damage caused by the fight.

Which of the following derivations is CORRECT?


(A) `M' will be liable because `D' is his servant
(B) Both `M' and `D' will be liable
(C) `M' will not be liable because the wrongful act (picking up fight) was not committed in the course
of D's employment
(D) `M' will be liable albeit the wrongful act (picking up fight) was not committed in the course of D's
employment

172. Principle: The Constitution of India guarantees the `right to life', which means `right - to live with
human dignity'. The right to life under the Constitution, however, does not include the right to die.
Facts: `M', who is 90, lives all alone as he has no family or children or grandchildren. He suffers from
physical and mental distress, as there is no one to look after him. He has little means to foot his
medical expenses. Under these circumstances, he approaches the court with a prayer that he
should be granted the right to die with dignity because he does not want to be a burden on the
society. Further, as it is his life, he has a right to put an end to it.

Which of the following derivations is CORRECT?


(A) The prayer can be granted, as suicide is not an offense in India
(B) The prayer can be granted, as the right to life under the Constitution includes the right to die
(C) The prayer can be granted, as a person cannot be forced to enjoy right to life to his detriment,
disadvantage and disliking
(D) The prayer cannot be granted, as the right to life under the Constitution does not include the right
to die

Previous Years
CLAT & AILET Papers Page 157
173. Principle: Trespass to land means direct interference with the possession of land without lawful
justification. Trespass could be committed either by a person himself entering the land of another
person or doing the same through some tangible object(s).
Facts: `A' throws some stones upon his neighbour's (B's) premises.

Which of the following derivations is CORRECT?


(A) `A' has committed trespass
(B) `A' has not committed trespass, as he has not entered B's premises
(C) `A' has committed nuisance
(D) None of the above

174. Principle: Nuisance is an unlawful interference with a person's use or enjoyment of land or some
right over or in connection with it. If the interference is `direct', the wrong is trespass; whereas, if the
interference is `consequential, it amounts to nuisance.
Facts: `A' plants a tree on his land. However, he allows its branches to project over the land of 'B'.

Which of the following derivations is CORRECT?


(A) `A' has committed trespass
(B `A' has committed nuisance
(C) `A' has not committed nuisance
(D) None of the above

175. Principle: Interference with another's goods in such a way as to deny the latter's title to the goods
amounts to conversion, and thus it is a civil wrong. It is an act intentionally done inconsistent with
the owner's right, though the doer may not know of, or intend to challenge, the property or posses-
sion of the true owner.
Facts: `R' went to a cycle-stand to park his bicycle. Seeing the stand fully occupied, he removed a
few bicycles in order to rearrange a portion of the stand and make some space for his bicycle. He
parked his bicycle properly, and put back all the bicycles except the one belonging to 'S'. In fact, `R'
was in a hurry, and therefore, he could not put back S's bicycle. Somebody came on the way and
took away S's bicycle. The watchman of the stand did not take care of it assuming that the bicycle
was not parked inside the stand. `S' filed a suit against `R' for conversion.

Which of the following derivations is CORRECT?


(A) `R' could not be held liable for the negligence of the watchman
(B) `S' would succeed because R's act led to the stealing of his bicycle
(C) `S' would not succeed because `R' did not take away the bicycle himself
(D) `S' would not succeed because R's intention was not bad

176. Principle: Nothing is an offence which is done by a person who is bound by law to do it.
Facts: `A', a police officer, without warrant, apprehends, who has committed murder.

(A) `A' is guilty of the offence of wrongful confinement


(B) `A' is not guilty of the offence of wrongful confinement
(C) `A' may be guilty of the offence of wrongful restraint
(D) `A' cannot apprehend `Z' without a warrant issued by a court of law

Previous Years
Page 158 CLAT & AILET Papers
177. Principle: When a criminal act is done by several persons in furtherance of the common intention
of all, each of such persons is liable for that act in the same manner as if it were done by him alone.
Facts: Roshan along with two of his friends, Tushar and Tarang proceeded to the house of Darshan
in order to avenge an insult made by the brother of Darshan. They opened fire on the members of
Darshan' s family. It was found that the shots of Roshan did not hit anyone, but the shots of Tushar
and Tarang succeeded in killing Darshan.

(A) Roshan was not liable for the offence of murder of Darshan, as Roshan's shots did not hit
Darshan
(B) Only Tushar and Tarang were liable for the offence of murder of Darshan, as their shots hit
Darshan
(C) Roshan along with Tushar and Tarang was liable for the offence of murder of Darshan
(D) Roshan was liable to a lesser extent comparing to his friends for the offence of murder of
Darshan, as Roshan' s shots did not hit Darshan

178. Principle: No communication made in good faith is an offence by reason of any harm to the person
to whom it is made, if it is made for the benefit of that person.
Facts: `A', a surgeon, in good faith, communicates to a patient his opinion that he cannot live. The
patient dies in consequence of the shock.

(A) `A' has committed the offence of causing death of his patient
(B) `A' has not committed the offence of causing death of his patient
(C) `A' has only partially committed the offence of causing death of his patient
(D) None of the above

179. Principle: Whoever, being legally bound to furnish information on any subject to any public servant,
as such, furnishes, as true, information on the subject which he knows or has reason to believe to
be false, has committed a punishable offence of furnishing false information.
Facts: Sawant, a landholder, knowing of the commission of a murder within the limits of his estate,
willfully misinforms the Magistrate of the district that the death has occurred by accident in conse-
quence of the bite of a snake.

(A) Sawant is not guilty of the offence of furnishing false information to the Magistrate
(B) Sawant is guilty of the offence of furnishing false information to the Magistrate
(C) Sawant is not legally bound to furnish true information to the Magistrate
(D) Sawant has the discretion to furnish true information to the Magistrate, as the murder was
committed within the limits of his estate

180. Principle: Whoever unlawfully or negligently does any act which is, and which he knows or has
reason to believe to be, likely to spread the infection of any disease dangerous to life, shall be guilty
of a negligent act likely to spread infection of disease dangerous to life.
Facts: `K', a person, knowing that he is suffering from Cholera, travels by a train without informing
the railway officers of his condition.

(A) `K' has committed an unlawful and negligent act, which is likely to spread the infection of
Cholera disease dangerous to the life of fellow-passengers
(B) Railway officers are guilty of an unlawful and negligent act, as `K' who is suffering from Cholera
disease has travelled by the train

Previous Years
CLAT & AILET Papers Page 159
(C) `K' has not committed an unlawful and negligent act, which is likely to spread the infection of
Cholera disease dangerous to the life of fellow-passengers
(D) Both `K' and Railway officers are guilty of an unlawful and negligent act, which is likely to spread
the infection of Cholera disease dangerous to the life of fellow passengers

181. Principle: Whoever drives any vehicle, or rides, on any public way in a manner so rash or negligent
as to endanger human life, or to be likely to cause hurt or injury to any other person, has committed
an offence, which shall be punished in accordance with the law.
Facts: `X', a truck driver, driving his vehicle rashly and negligently at a high speed climbed the
footpath and hit `Y', a pedestrian, from behind causing his death.

(A) `X' is not guilty of rash and negligent driving


(B) `Y' should have taken sufficient care on the footpath
(C) `X' is guilty of rash and negligent driving
(D) `X' is only in part guilty of rash and negligent driving

182. Principle: Whoever causes death by doing an act with the intention of causing death, or with the
intention of causing such bodily injury as is likely to cause death, or with the knowledge that he is
likely by such act to cause death, commits the offence of culpable homicide.
Facts: `A' knows `Z' to be behind a bush. `B' does not know it. `A', intending to cause, or knowing it
to be likely to cause Z's death, induces `B' to fire at the bush. 'B' fires and kills `Z'.

(A) 'B' has committed the offence of culpable homicide


(B `A' has committed the offence of culpable homicide
(C) Both `A' and `B' have committed the offence of culpable homicide
(D) None of them has committed the offence of culpable homicide

183. Principle: Whoever, intending to take dishonestly any movable property out of the possession of
any person without that person's consent, moves that property in order to such taking, is said to
commit theft.
Facts: `Z', going on a journey, entrusts his plate to the possession of `A', the keeper of a warehouse,
till `Z' shall return. Then, `A' carries the plate to a goldsmith and sells it.

(A) `A' has committed theft


(B) `A' has not committed theft
(C) `A' lawfully sold the plate to the goldsmith
(D) None of the above is true

184. Principle: Whoever makes any false document or part of a document with intent to cause damage
or injury, to the public or to any person, or to support any claim or title, or to cause any person to
part with property, or to enter into any express or implied contract, or with intent to commit fraud or
that fraud may be committed, commits forgery.
Facts: `A' without Z's authority, affixes Z's seal to a document purporting to be a conveyance of an
estate from `Z' to `A', with the intention of selling the estate to `B' and thereby of obtaining from `B'
the purchase-money.

(A) 'B' has committed forgery (B) `Z' has committed forgery
(C) `A' has committed forgery (D) `A' and 'B' have committed forgery

Previous Years
Page 160 CLAT & AILET Papers
185. Principle: Whoever intentionally uses force to any person, without that person's consent, in order
to the committing of any offence, or intending by the use of such force to cause, or knowing it to be
likely that by the use of such force he will cause injury, fear or annoyance to the person to whom the
force is used, is said to use criminal force to that other.
Facts: `Z' is riding in a palanquin. `A', intending to rob `Z', seizes the pole and stops the palanquin.
Here `A' has caused cessation of motion to `Z', and `A' has done this by his own bodily power.

(A) `A' has used criminal force to `Z'


(B) 'A' has no intention to use criminal force to `Z'
(C) `A' has used force with the consent of `Z'
(D) None of the above is correct

186. Principle: One of the essential conditions for a marriage between any two persons to be solem-
nized under the Special Marriage Act, '1954 is that at the time of the marriage the male has com-
pleted the age of twenty-one years and the female the age of eighteen years. If the said condition is
not fulfilled such a marriage is null and void.
Facts: `A', a male aged twenty-two years, proposes to many `B', a female aged sixteen years, at
Delhi in the month of June 2014 under the Special Marriage Act, 1954.

(A) Marriage between `A' and 'B' can be legally solemnized under the Special Marriage Act, 1954
(B) Marriage between `A' and `B' cannot be legally solemnized under the Special Marriage Act, 1954
(C) Marriage between `A' and 'B' can remain valid for A under the Special Marriage Act, 1954
(D) None of the above is correct

187. Principle: Under the Hindu Marriage Act, 1955 either the husband or the wife can move a petition
for a decree of divorce on the ground of desertion. The term `desertion' means desertion of the
petitioner by the other party to the marriage for a continuous period of not less than two years
immediately preceding the presentation of the petition, without reasonable cause and without the
consent or against the wish of such party and includes the willful neglect of the petitioner by the
other party to the marriage, and its grammatical variations and cognate expressions shall be con-
strued accordingly. It is also said that desertion is withdrawal not from a place but from a state of
things.
Facts: Rohan, a technocrat, went to US in January 2011 for pursuing his higher studies for a period
of three years. In fact, Rohan went to US with the consent of his wife Basanti, who stayed at her
parents' home, and with a promise of his return to India upon the completion of his studies. From US
he has quite often been in touch with his wife. Subsequently, Rohan has got a job there in US and
he wishes to take his wife. She refuses to go to US and, in the meanwhile, she files a petition for a
decree of divorce on the ground of desertion by her husband.

(A) Rohan's three year stay in US in the above context can amount to a ground of desertion for
divorce.
(B) Rohan's three year stay in US in the above context cannot amount to a ground of desertion for
divorce.
(C) Rohan's continued stay after three years can amount to a ground of desertion for divorce.
(D) Basanti's refusal can amount to a ground of desertion for divorce.

Previous Years
CLAT & AILET Papers Page 161
188. Principle: Under the Hindu Adoptions and Maintenance Act, 1956, no person shall be capable of
being taken in adoption unless he or she is a Hindu, he or she not already been adopted, he or she
has not been married, unless there is a custom or usage applicable to the parties which permits
persons who are married being taken in adoption, and he or she has not completed the age of fifteen
years, unless there is a custom or usage applicable to the parties which permits persons who have
completed the age of fifteen years being take in adoption.
Facts: Vijay being natural father had given Tarun, a boy aged 10 years, in adoption to Manoj in
March 2010 in accordance with the Hindu Adoptions and Maintenance Act, 1956. In May 2012
Manoj gave Tarun in adoption to Sanjay. Subsequently in December 2013, Sanjay gave Tarun in
adoption to Vijay.

(A) Adoption of Tarun by Sanjay is valid


(B) Adoption of Tarun by Vijay is valid
(C) Adoption of Tarun by Manoj is valid
(D) None of the above adoptions is valid

189. Principle: Under copyright law copyright subsists in original literary works also. A literary work
need not be of literary quality. Even so prosaic a work as an index of railway stations or a railway
guide or a list of stock exchange quotations qualifies as a literary work if sufficient work has been
expended in compiling it to give it a new and original character.
Facts: Michael works hard enough, walking down the streets, taking down the names of people who
live at houses and makes a street directory as a result of that labour.

(A) Michael's exercise in making a street directory is sufficient to justify in making claim to copy-
right in that work which is ultimately produced
(B) Michael's exercise in making a street directory is not enough to justify in making claim to
copyright in that work
(C) A street directory cannot be enough to be considered as a literary work
(D) None of the above statements is correct

190. Principle: Every person shall be liable to punishment under the Indian Penal Code and not other-
wise for every act or omission contrary to the provisions of the Code of which he shall be guilty within
the territory of India. In other words, the exercise of criminal jurisdiction depends upon the locality of
the offence committed, and not upon the nationality or locality of the offender.
Facts: `X', a Pakistani citizen, while staying at Karachi, made false representations to `Y', the
complainant, at Bombay through letters, telephone calls and telegrams and induced the complain-
ant to part with money amounting to over rupees five lakh to the agents of `X' at Bombay, so that rice
could be shipped from Karachi to India as per agreement. But the rice was never supplied to the
complainant.

(A) The offence of cheating under section 420 of the Code was committed by `X' within India, even
though he was not physically present at the time and place of the crime
(B) The offence of cheating as per section 420 of the Code was not committed by `X' within India, as
he was not physically present at the time and place of the crime
(C) Only the agents of `X' had committed the offence of cheating under section 420 of the Code
within India, as they were physically present at the time and place of the crime
(D) `Y' was also liable for the offence of cheating under section 420 of the Code within India, as he
was physically present at the time and place of the crime

Previous Years
Page 162 CLAT & AILET Papers
191. Principle: When two or more persons agree to do, or cause to be done, (1) an illegal act, or (2) an
act which is not illegal by illegal means, through such an agreement such persons are said to have
been engaged in a criminal conspiracy to commit an offence. It is said that no consummation of the
crime need be achieved or even attempted.
Facts: `X', `Y' and `Z' plan to kill `D'. They agree that only one among them, that is `Z', will execute
the plan. In pursuance of it `Z' buys a gun and loads it.

(A) Only `Z' can be charged with criminal conspiracy to kill `D'
(B) All of them, i.e., `X', `Y' and `Z', can be charged with criminal conspiracy to kill `D'
(C) `X' and `Y' cannot be charged with criminal conspiracy to kill `D'
(D) None of them can be charged with criminal conspiracy to kill `D'

192. Principle: `Wrongful gain' is gain by unlawful means of property to which the person gaining is not
legally entitled. `Wrongful loss' is the loss by unlawful means of property to which the person losing
it is legally entitled.
Facts: `X' takes away Y's watch out of Y's possession, without Y's consent and with the intention of
keeping it.

(A) `X' causes `wrongful gain' to `Y'


(B) `Y' causes `wrongful gain' to `X'
(C) `X' causes `wrongful loss' to `Y'
(D) `Y' causes `wrongful loss' to `X'

193. Principle: Nothing is an offence by reason that it causes, or that it is intended to cause, or that it
is known to be likely to cause, any harm, if that harm is so slight that no person of ordinary sense
and temper would complain of such harm.
Facts: 'X' takes a plain sheet of paper from Y's drawer without Y's consent to write a letter to his
friend.

(A) 'X' has committed an offence in the above context


(B) 'X' has committed no offence in the above context
(C) 'Y' can sue 'X' for an offence in the above context
(D) None of the above is correct in the above context

194. Principle: When an act which would otherwise be a certain offence, is not that offence, by reason
of the youth, the want of maturity of understanding, the unsoundness of mind or the intoxication of
the person doing that act, or by reason of any misconception on the part of that person, every
person has the same right of private defence against that act which he would have if the act were
that offence.
Facts: 'X', under the influence of madness, attempts to kill 'Y'.

(A) 'Y' has the right of private defence against 'X'


(B) 'Y' does not have the right of private defence against 'X'
(C) 'Y' has the right of private defence against 'X', only if 'X' is not under the influence of madness
(D) 'X' has the right of private defence against 'Y'

Previous Years
CLAT & AILET Papers Page 163
195. Principle: Where a person fraudulently or erroneously represents that he is authorized to transfer
certain immovable property and professes to transfer such property for consideration, such transfer
shall, at the option of the transferee, operate on any interest which the transferor may acquire in
such property at any time during which the contract of transfer subsists.
Facts: 'A', a Hindu who has separated from his father `B', sells to 'C' three fields, X, Y and Z,
representing that 'A' is authorized to transfer the same. Of these fields Z does not belong to 'A', it
having been retained by 'B' on the partition; but on B's dying 'A' as successor obtains Z, and at that
time 'C' had not cancelled the contract of sale.

(A) 'A' can sell Z to a third party


(B) 'A' is not required to deliver Z to 'C'
(C) 'A' is required to deliver Z to 'C'
(D) None of the above statements is correct

196. Principle: Under the Transfer of Property Act, 1882 a property must be transferred by one living
person to another living person. The Act deals only with transfer of property between living persons.
Facts: `X' wants to transfer his property to the presiding deity in a temple situated within the estate
of 'A'.

(A) Transfer of property by `X' will be valid


(B) Transfer of property by `X' will be invalid
(C) Transfer of property by `X' to the presiding deity will become a valid transfer to `A'
(D) None of the above is correct

197. Principle: Where there is transfer of ownership of one thing for the ownership of some other thing it
is called exchange; while transfer of ownership for consideration of money is called sale, whereas,
without consideration it becomes gift.
Facts: `A' transfers his house worth <50 Lakhs to `B' for a shopping building worth the same
amount, as consideration, from `B'.

(A) The transaction is a gift


(B) The transaction is a sale
(C) The transaction is an exchange
(D) The transaction is a mortgage

198. Principle: One of the principles of natural justice is Nemo judgx in causa sua, which means that no
one should be a judge in his own cause. In other words, no person can judge a case in which he has
an interest.
Facts: `X', a member of the selection board for a government service, was also a candidate for
selection for the same service. `X' did not take part in the deliberations of the board when his name
was considered and approved.

(A) Selection of `X' is against the principle of natural justice.


(B) Selection of `X' is not against the principle of natural justice.
(C) Non-selection of `X' will be against the principles of natural justice.
(D) Non-participation of `X' in the board deliberations will render his selection valid.

Previous Years
Page 164 CLAT & AILET Papers
199. Principle: Strike is a collective stoppage of work by workmen undertaken in order to bring pressure
upon those who depend on the sale or use of the products of work; whereas, lock-out is a weapon
in the hands of the employer, similar to that of strike in the armoury of workmen, used for compelling
persons employed by him to accept his terms or conditions of or affecting employment. While in
closure there is permanent closing down of a place of employment or part thereof, in lay-off an
employer, who is willing to employ, fails or refuses or is unable to provide employment for reasons
beyond his control.
Facts: Workmen of a textile factory went on strike as per law, demanding the payment of bonus.
Employer of the factory refused to pay any extra allowances, including bonus, and besides he
closed down the factory till the strike was stopped.

(A) Act of closing down the factory by the employer amounted to strike
(B) Act of closing down the factory by the employer amounted to lay-off
(C) Act of closing down the factory by the employer amounted to lock-out
(D) Act of closing down the factory by the employer amounted to closure

200. Principle: Trade dispute means any dispute between employers and workmen or between work-
men and workmen, or between employers and employers which is connected with the employment
or non-employment, or the terms of employment or the conditions of labour, of any person. Disputes
connected with the non-employment must be understood to include a dispute connected with a
dismissal, discharge, removal or retrenchment of a workman.
Facts: `X', an employee in a sugar factory, raised a dispute against `Y', the employer, through trade
union regarding certain matters connected with his suspension from the employment.

(A) Matters connected with suspension can amount to a trade dispute


(B) Matters connected with suspension cannot amount to a trade dispute
(C) Only after dismissal, matters connected with suspension can amount to a trade dispute
(D) None of the above is correct.

Previous Years
CLAT & AILET Papers Page 165
CLAT Question Paper 2015

ENGLISH Jeans that flair [A]/flare [B] at the bottom


are in fashion these days.
Direction (Qs. 1–3) : In the question, there are five They heard the bells peeling [A]/pealing [B]
sentences. Each sentence has pairs of words/phrases far and wide.
that are italicized and highlighted. From the italicized The students baited [A]/bated [B] the
and highlighted word(s)/phrase(s), select the most instructor with irrelevant questions.
appropriate word(s)/phrase(s) to form correct (a) BBABB (b) ABBBB
sentences. Then, from the options given, choose the (c) BABBA (d) BBBBA
best one.
Direction (Qs. 4-6) : Identify the incorrect sentence/
1. The further [A]/farther [B] he pushed himself, sentences.
the more disillusioned he grew.
For the crowd it was more of a historical [A]/ 4. A. I want to do an MBA before going into
historic [B] event, for their leader it was just business.
another day. B. Priti's husband has been on active service
The old has a healthy distrust [A]/mistrust for three months.
[B] for all new technology. C. The horse suddenly broke into a buckle.
The film is based on a worthy [A]/true [B] D. I need to file an insurance claim.
story.
She coughed discreetly [A]/discretely [B] (a) B and A (b) C only
to announce her presence. (c) B and C (d) B, C and D
(a) BABAB (b) ABBBA
(c) BAABA (d) BBAAB 5. A. I must run fast to catch up with him
B. The newly released took is enjoying a
2. Regrettably [A]/ Regretfully [B] I have to popular run.
decline your invitation. The critics censored [A]/ C. The doctor is on a hospital round.
censured [B] the new movie because of its D. You can't run over him like that.
social unacceptability.
He was besides [A]/beside [B] himself with (a) A and C (b) D only
range when I told him what I had done. (c) A, C and D (d) A only
Anita had a beautiful broach [A]/brooch [B]
on the lapel of her jacket. 6. A. The letter was posted to the address.
He has the same capacity as an adult to B. Your stand is beyond all reasons.
consent [A]/assent [B] to surgical treatment. C. How do you deal with friend who doesn't
(a) BABBA (b) BBAAB listen to a reason?
(c) ABBBA (d) BABAB D. My wife runs profitable business in this
suburb.
3. The prisoner's interment [A]/internment [B]
came to an end with his early release (a) A only (b) D only
She managed to bite back the ironic [A]/ (c) B and C (d) C and D
caustic [B] retort on the tip of her tongue.

Previous Years
Page 166 CLAT & AILET Papers
Direction (Qs. 7–12) : Fill up the blanks, numbered want to get away to some place warm to relax
[1] [2] [3] [4] [5] and [6] in the passage given below with your family. Will you accept your in-laws,
with the most appropriate word from the options given offer of free use of their Florida beachfront
for each blank. condo? Sure. You like your employer and feel
ready to move forward in your career. Will you
“Between the year 1946 and the year 1995, I did not step in for your boss for three weeks while
file any income tax returns.” With that [1] statement, she attends a professional development
Soubhik embarked on an account of his encounter course? Of course.
with the Income Tax Department. “I originally owed A. Some decisions are obvious under certain
Rs. 20,000 in unpaid taxes. With [2] and [3], the circumstances. You may, for example,
20,000 became 60,000. The Income Tax Department readily accept a relative's offer of free
then went into action, and I learned first-hand just holiday accommodation. Or step in for
how much power the Tax Department wields. your boss when she is away.
Royalties and trust funds can be [4]; automobiles B. Some decisions are no-brainers. You need
may be [5], and auctioned off. Nothing belongs to not think when making them. Examples
the [6] until the case is settled.” are condo offers from in-laws and job offers
from bosses when your bank account is
7. Fill up the blank [1] low or boss is away.
(a) devious (b) blunt C. Easy decisions are called "no-brainers"
(c) tactful (d) pretentious because they do not require any cerebral
activity. Examples such as accepting free
8. Fill up the blank [2] holiday accommodation abound in our
(a) interest (b) taxes lives.
(c) principal (d) returns D. Accepting an offer from in-laws when you
are short on funds and want a holiday is a
9. Fill up the blank [3] no-brainer. Another no-brainer is taking the
(a) sanctions (b) refunds boss's job when she is away.
(c) fees (d) fines
(a) A (b) B (c) C (d) D
10. Fill up the blank [4]
(a) closed (b) detached 14. Physically, inertia is a feeling that you just
(c) attached (d) impounded can't move; mentally, it is a sluggish mind.
Even if you try to be sensitive, if your mind is
11. Fill up the blank [5] sluggish, you just don't feel anything intensely.
(a) smashed (b) seized You may even see a tragedy enacted in front
(c) dismantled (d) frozen of your eyes and not be able to respond
meaningfully. You may see one person
12. Fill up the blank [6] exploiting another, one group persecuting
(a) purchaser (b) victim another, and not be able to get angry. Your
(c) investor (d) offender energy is frozen. You are not deliberately
refusing to act; you just don't have the
Directions (Qs. 13–14) : Four alternative summaries capacity.
are given the text. Choose the option that best A. Inertia makes your body and mind
captures the essence of the text. sluggish. They become insensitive to
tragedies, exploitation, and persecution
13. Some decisions will be fairly obvious-"no- because it freezes your energy and
brainers". Your bank account is low, but you decapacitates it.
have a two-week vacation coming up and you

Previous Years
CLAT & AILET Papers Page 167
B. When you have inertia you don't act Directions (Qs. 20–21) : Answer the question based
although you see one person exploiting on the following information. Indicate which of the
another or one group persecuting another. statements given with that particular question
You don't get angry because you are consistent with the description of unreasonable man
incapable. in the passage below.
C. Inertia is of two types - physical and
mental. Physical inertia restricts bodily Unreasonableness is a tendency to do socially
movements. Mental inertia prevents permissible things at the wrong time. The
response to events enacted in front of your unreasonable man is the sort of person who comes
eyes. to confide in you when you are busy. He serenades
D. Physical inertia stops your body from his beloved when she is ill. He asks a man who has
moving; mental inertia freezes your just lost money by paying a bill for a friend to pay a
energy and stops your mind from bill for him. He invites a friend to go for a ride just
responding meaningfully to events, even after the friend has finished a long car trip. He is
tragedies, in front of you. eager to offer services which are not wanted, but
which cannot be politely refused. If he is present at
(a) A (b) B (c) C (d) D an arbitration, he stirs up dissension between the
two parties, who were really anxious to agree. Such
Directions (Qs. 15–19) : For the word a contextual is the unreasonable man.
sentence is given. Pick the word from the alternatives
given that is most inappropriate in the given context. 20. The unreasonable man tends to
(a) entertain women
15. SPECIOUS: A specious argument is not (b) be a successful arbitrator when dissenting
simply a false one but one that has the ring parties are anxious to agree
of truth. (c) be helpful when solicited
(a) Deceitful (b) Fallacious (d) tell a long story to people who have heard
(c) Credible (d) Deceptive it many times before

16. OBVIATE: The new mass transit system may 21. The unreasonable man tends to
obviate the need for the use of personal cars. (a) bring a higher bidder to a salesman who
(a) Prevent (b) Forestall has just closed a deal
(c) Preclude (d) Bolster (b) disclose confidential information to others
(c) sing the praise of the bride when he goes
17. DISUSE: Some words fall into disuse as to a wedding
technology makes objects obsolete. (d) sleep late and rise early
(a) Prevalent (b) Discarded
(c) Obliterated (d) Unfashionable Directions (Qs. 22–24) : In the following sentence,
a part of the sentence is underlined. Beneath each
18. PARSIMONIOUS: The evidence was sentence, four different ways of paraphrasing the
constructed from every parsimonious scraps underlined part are indicated. Choose the best
of information. alternative among the four options.
(a) Prevalent (b) Penurious
(c) Thrifty (d) Altruistic 22. The management can still hire freely but
cannot scold freely.
19. FACETIOUS: When I suggested that war is (a) cannot scold at will
a method of controlling population, my father (b) cannot give umbrage
remarked that I was being facetious. (c) cannot take decision to scold
(a) Jovian (b) Jovial (d) cannot scold willfully
(c) Jocular (d) Joking

Previous Years
Page 168 CLAT & AILET Papers
23. This government has given subsidies to the Directions (Qs. 28–37): In view of the passage given
Navratnas but there is no telling whether the below. Choose the best option for question.
subsequent one will do.
(a) whether the subsequent government will do When talks come to how India has done for itself in
so 50 years of Independence, the world has nothing
(b) if the government to follow will accept the but praise for our success in remaining a democracy.
policy On other fronts, the applause is less loud. In absolute
(c) if the government to follow will adhere to terms, India has not done too badly, of course, life
the policy expectancy has increased. So has literacy. Industry,
(d) no telling whether the subsequent one will do which was barely a fledging, has grown tremendously.
so And as far as agriculture is concerned, India has
been transformed from a country perpetually on the
24. The Romanians may be restive under Soviet edge of starvation into a success story held up for
direction but they are tied to Moscow by others to emulate. But these are competitive times
ideological and military links. when change is rapid, and to walk slowly when the
(a) they are close to Moscow from ideological rest of the world is running is almost as bad as
and military perspective standing still on walking backwards.
(b) they are preparing for a greater revolution
(c) secretly they rather enjoy the prestige of Compared with large chunks of what was then the
being protected by the mighty Soviets developing world South Korea, Singapore, Malaysia,
(d) there is nothing they can do about it Thailand, Indonesia, China and what was till lately a
separate Hong Kong-India has fared abysmally. It
Directions (Qs. 25–27) : In the question, a related pair began with a far better infrastructure than most of
of words or phrases is followed by a pair of words or these countries had. It suffered hardly or not at all
phrases. Select the pair that best expresses a during the Second World War. It had advantages like
relationship similar to the one expressed in the original an English speaking elite, quality scientific manpower
pair. (including a Nobel laureate and others who could be
ranked among the world's best) and excellent
25. Dulcet : Raucous business acumen. Yet, today, when countries are
(a) Sweet : Song ranked according to their global competitiveness, it
(b) Crazy : Insane is tiny Singapore that figures at the top. Hong Kong
(c) Palliative : Exacerbating is an export powerhouse. So is Taiwan. If a symbol
(d) Theory : Practical were needed of how far we have fallen back, note that
while Korean Cielos are sold in India, no one is South
26. Malapropism : Words Korea is rushing to buy an Indian car. The reasons
(a) Anachronism : Time list themselves. Topmost is economic isolationism.
(b) Ellipsis : Sentence
(c) Jinjanthropism : Apes The government discouraged imports and encouraged
(d) Catechism : Religion self-sufficiency. Whatever the aim was, the result
was the creation of a totally inefficient industry that
27. Peel : Peal failed to keep pace with global trends and, therefore,
(a) Coat: Rind became absolutely uncompetitive. Only when the
(b) Laugh : Bell trade gates were opened a little did this become
(c) Rain : Reign apparent. The years since then have been spent in
(d) Brain : Cranium merely trying to catch up. That the government
actually sheltered its industrialists from foreign
competition is a little strange. For in all other
respects, it operated under the conviction that

Previous Years
CLAT & AILET Papers Page 169
businessmen were little more than crooks how were business policy is the best pro-people policy. By
to be prevented from entering the most important then of course, the world would have moved even
areas of the economy, how were to be hamstrung in farther ahead.
as many ways as possible, how were to be tolerated
in the same way as an inexcisable wart. The high 28. The writer's attitude towards the Government
expropriatory rates taxation, the licensing laws, the is...
reservation of whole swathes of industry for the public (a) critical (b) ironical
sector, and the granting of monopolies to the public (c) sarcastic (d) derisive
sector firms were the principle manifestations of this
attitude. The government forgot that before wealth 29. The writer is surprised at the Government's
could be distributed, it had to be created. attitude towards its industrialists because.......
(a) the government did not need to protect
The government forgot that it itself could not create, its industrialists.
but only squander wealth. Some of the manifestations (b) the issue of competition was non-existent.
of the old attitude have changed. Tax rates have fallen. (c) the government looked upon its
Licensing has been all but abolished. And the gates industrialists as crooks.
of global trade have been opened wide. But most of (d) the attitude was a conundrum.
these changes were first by circumstances partly
by the foreign exchange bankruptcy of 1991 and the 30. The Government was compelled to open the
recognition that the government could no longer economy due to....
muster the funds of support the public sector, leave (a) pressure from international market.
alone expand it. Whether the attitude of the (b) pressure from domestic market.
government itself, or that of more than handful of (c) foreign change bankruptcy and paucity of
ministers, has changed, is open to question. In many funds with the government.
other ways, however, the government has not (d) All of the above.
changed one with. Business still has to negotiate a
welter of negotiations. Transparency is still a longer 31. The writer ends the passage on a note of.....
way off. And there is no exit policy. In defending the (a) cautious optimism (b) pessimism
existing policy, politicians betray an inability to see (c) optimism (d) pragmatism
beyond their noses. A no-exit policy for labour is
equivalent to a no-entry policy for new business. If 32. According to the writer India should have
one industry is not allowed to retrench labour, other performed better than the other Asian nations
industries will think a hundred times before employing because.......
new labour. In other ways too, the government hurts (a) it had adequate infrastructure
industries. (b) it had better infrastructure
(c) it had better politicians who could take
Public sector monopolies like the department of the required decisions.
telecommunications and Videsh Sanchar Nigam Ltd. (d) All of the above.
make it possible for Indian business to operate only
at a cost several times that of their counterparts 33. India was in better condition than the other
abroad. The infrastructure is in a shambles partly Asian nations because....
because it is unable to formulate a sufficiently (a) it did not face the ravages of the Second
remunerative policy for private business, and partly World War
because it does not have the stomach to change (b) it had an English speaking populace and
market rates for services. After a burst of activity in good business sense.
the early nineties, the government is dragging its (c) it had enough wealth through its exports.
feet. At the rate it is going, it will be another fifty (d) Both (a) and (b) above
years before the government realises that a pro-

Previous Years
Page 170 CLAT & AILET Papers
34. The major reason for India's poor performance GENERAL KNOWLEDGE AND
is........ CURRENT AFFAIRS
(a) economic isolationism
(b) economic mismanagement 41. Attukal Pongal festival, which is figured in
(c) inefficient industry Guinness Book of World Records is
(d) All of these celebrated in ..........
(a) Tamil Nadu (b) Kerala
35. One of the factors of the government's (c) Telangana (d) Goa
projectionist policy was......
(a) encouragement of imports 42. In February 2015, which Indian Cricket legend
(b) discouragement of imports has been inducted into the ICC Hall of Fame?
(c) encouragement of exports (a) Rahul Dravid
(d) discouragement of exports (b) Anil Kumble
(c) Sachin Tendulkar
(d) Mohammad Azharuddin
36. The example of the Korean Cielo has been
presented to highlight......
43. Which of the following Acts formally
(a) India's lack of stature in the international introduced the principle of elections for the
market. first time?
(b) India's poor performance in the international (a) The Indian Councils Act, 1909
market. (b) Government of India Act, 1919
(c) India's lack of credibility in the (c) The Government of India Act, 1935
international market (d) India's Independence Act, 1947
(d) India's disrepute in the international
market. 44. IRCTC has recently launched a new service
called 'RuPay prepaidcards' which will enable
37. According to the writer..... passengers to book their tickets, do shopping
(a) India's politicians are myopic in their vision and pay service bills online. This service was
of the country's requirements. launched in collaboration with which bank?
(b) India's politicians are busy lining their (a) Union Bank of India
pockets. (b) State Bank of India
(c) India's politicians are not conversant with (c) ICICI Bank
the needs of the present scenario. (d) Bharatiya Mahila Bank
(d) All of the above.
45. Garuda Shakti III is the military exercise
Direction (Qs. 38–40) : Choose the option closest between India and which country?
in meaning to the Capitalized word. (a) Nepal (b) Russia
(c) Indonesia (d) China
38. GRANDIOSE
46. Match List-I with List-II and select the best
(a) imposing (b) unpretentious
option using the code given below the lists:
(c) boring (d) lanky List-I List-II
(Organization/Centers) (Locations)
39. SPRY A. High Altitude Warfare 1. Chennai
(a) doubtful (b) nimble School
(c) prognosticate (d) leave B. Indian Air force Training 2. Gulmarg
Center
40. FUDGE C. National Defense 3. New Delhi
(a) to sweeten (b) smear College
(c) irritate (d) falsify D. Institute of National 4. Pune
Integration

Previous Years
CLAT & AILET Papers Page 171
A B C D 54. Prime Minister Modi has launched the "Give
(a) 2 1 3 4 It Up" campaign for voluntarily giving up ..........
(b) 1 2 3 4 (a) Use of tobacco products
(c) 1 2 4 3 (b) LPG Subsidy
(d) 2 1 4 3 (c) Use of incandescent bulbs
(d) Use of plastics
47. Name India's Beyond Visual Range (BVR) Air-
to-Air missile which was successfully test fired 55. Which one of the following Railway Zones and
on 19 March 2015 from a Sukhoi-30 fighter the corresponding Headquarter pairs is not
aircraft? correctly matched?
(a) Astra (b) K-100 (a) North Eastern Railway : Gorakhpur
(c) Mitra (d) Tejas (b) South Eastern Railway : Bhubaneswar
(c) Eastern Railway : Kolkata
48. The ISRO has developed a "Flood Hazard (d) South East Central Railway : Bilaspur
Atlas" by mapping flood prone and vulnerable
areas in which state? 56. Which among the following is the world's
(a) Kerala (b) Maharashtra largest e-commerce company?
(c) Assam (d) Tripura (a) Amazon (b) eBay
(c) Alibaba (d) Flipkart
49. Which South East Asian country has recently
banned surrogacy service to end its flourishing 57. Which committee was constituted by RBI to
rent-a-womb industry? review governance of boards of banks in India?
(a) Singapore (b) Laos (a) P J Nayak Committee
(c) Thailand (d) Vietnam (b) H R Khan Committee
(c) Harsh Vardhan Committee
50. Which of the following is the oldest share (d) K Subramanian Committee
market in India?
(a) Bombay (b) Madras 58. The recently announced Paramparagat Krishi
(c) Calcutta (d) Delhi Vikas Yojana aims to boost __?
(a) Organic Farming
51. The name of new Andhra Pradesh Capital is (b) Drip Irrigation
likely to be .......... (c) Horticulture crops
(a) Amaravathi (b) Badrachala (d) Vegetable production
(c) Krishna Nagar (d) Varshavathi
59. The winner of 2015 Malaysian Grand Prix is
52. The Ufa city, where annual BRICS summit- __?
2015 is scheduled to be held is in which (a) Sebastian Vettel
country? (b) Kimi Raikkonen
(a) China (b) Russia (c) Lewis Hamilton
(c) South Africa (d) Brazil (d) Jenson Button

53. The "Ease of Doing Business Index" is 60. Which one of the following is essentially a
prepared and published by ........... solo dance nowadays performed in group as
(a) World Trade Organisation well?
(b) World Bank Group (a) Kuchipudi (b) Kathak
(c) United Nations (c) Manipuri (d) Mohiniattam
(d) European Union

Previous Years
Page 172 CLAT & AILET Papers
61. Who among the following was the author of 68. What is the correct sequence of the following
Rajtarangini, commonly regarded as the first movements in chronological order?
genuine history of India written by an Indian? 1. Civil Disobedience Movement
(a) Banbhatta (b) Ravikirti 2. Khilafat Movement
(c) Pushpananda (d) Kalhana 3. Home Rule Movement
4. Quit India Movement
62. Name the golfer who won the Indian Open
title on 22 February 2015. (a) 1, 2, 3, 4 (b) 4, 3, 2, 1
(a) SSP Chowrasia (b) Anirban Lahiri (c) 3, 2, 1,4 (d) 2, 4, 1, 3
(c) Siddikur Rahman (d) Daniel Chopra
69. Recently, which country became the first
63. Which space agency has successfully member country to the UN Framework
launched the world's first all electric satellites Convention on Climate Change (UNFCCC) to
in March, 2015? submit its action plan on Intended Nationally
(a) Russia Federal Space Agency Determined Contribution (INDC)?
(b) China National Space Administration (a) India (b) Switzerland
(c) SpaceX (c) Australia (d) Singapore
(d) European Space Agency
70. Lysosomes, which are known as suicide
64. Who among the following 18th century Indian bags, are produced by which organelle?
rulers has been called 'Plato of his tribe'? (a) Mitochondria (b) Golgi body
(a) Sawai Jai Singh (c) Ribosome (d) Peroxisome
(b) Badam Singh
(c) SurajMal 71. Which is the single policy rate to
(d) Guru Gobind Singh unambiguously signal the stance of monetary
policy as recently recommended by RBI?
65. Bhalchandra Nemade who has been selected (a) PLR (b) Repo
for the 50th Jnanpith Award for 2014, on 6 (c) Bank (d) CLR
February 2015, is a famous writer in which
language? 72. Which city has become India's first fully WiFi-
(a) Marathi (b) Oriya enabled metro city on 5 February 2015?
(c) Malayalam (d) Urdu (a) Mumbai (b) Kolkata
(c) Chennai (d) Delhi
66. A Snickometer is associated with which
sports? 73. Who among the following is India's first chief
(a) Tennis (b) Cricket of Cyber Security?
(c) Hockey (d) Football (a) B. J. Srinath
(b) Gulshan Rai
67. Which of the following is incorrect option? (c) A. S.Kamble
(a) Within the Arctic and Antarctic Circles (d) Amardeep S. Chawla
there is at least one day in the year during
which the sun does not set and at least 74. Which one of the following is the online
one day on which it never rises. grievances monitoring portals launched by
(b) At the North Pole there is darkness for half union government for Indians living abroad?
the year. (a) Madad (b) Sankalp
(c) At the summer solstice, the sun shines (c) Mythri (d) Rakshan
vertically over the Tropic of Capricorn.
(d) The sun shines vertically over the Equator
twice in the year.

Previous Years
CLAT & AILET Papers Page 173
75. Which of the following is a Direct Tax? 82. Project Varsha, India's new naval base under
(a) Excise duty (b) Sales tax construction is near which of the following
(c) Income tax (d) None of the above cities?
(a) Kochi (b) Karwar
76. Indian Space Research Organisation was (c) Visakhapatnam (d) Chennai
recently conferred ‘Space Pioneer Award’ by
the National Space Society (NSS) of which 83. Young Indian shuttler K. Srikanth on 15 March
country over the historic feat on successfully 2015 won which of the following major
sending an orbit to Martian atmosphere in its badminton championship?
very first attempt? (a) China Grand Prix Gold
(a) France (b) European Union (b) Australian Grand Prix Gold
(c) China (d) USA (c) Swiss Grand Prix Gold
(d) Indonesian Grand Prix Gold
77. Recently in which country did Indian Prime
Minister Narendra Modi inaugurate the first of 84. What is the name given to the dedicated TV
the eight Coastal Surveillance Radar Systems channel for farmers that was announced in
(CSRS) being set up by India? the Union Budget for 2014-15 and Rs. 100
(a) Mauritius (b) Maldives crore was set aside for its establishment?
(c) Sri Lanka (d) Seychelles (a) Kisan (b) Farmers Show
(c) Krishi Channel (d) None of the above
78. Greece and Turkey are working to resolve
their dispute over sovereignty and related 85. The "Friends for Life" - an elephant
rights in the area of which sea? conservation project has been launched by
(a) Black Sea (b) Sea of Marmara World Wide Fund for Nature India and ...........
(c) Aegean Sea (d) Mediterranean Sea (a) Aditya Birla Group (b) Muthoot Group
(c) Manapuram Group (d) Reliance Ltd
79. Name the Indian industrialist on whose 175th
birth anniversary, Union government launched 86. The National Industrial Corridor (NIC) that was
the commemorative stamp on 6th January proposed to be established in the Union
2015? Budget of 2014-15, will have its headquarters
(a) G D Birla at which city?
(b) T V Sundaram lyengar (a) Pune (b) Bhubaneswar
(c) Kasturbhai Lalbhai (c) Bangalore (d) Hyderabad
(d) Jamsetji Nusserwanji Tata
87. President Pranab Mukherjee on 6th January
80. Which one of the following pairs is not 2015 signed the ordinance to amend
correctly matched? Citizenship Act, 1955. Which of the statements
(a) Kunal Bahl & Rohit Bansal- Snapdeal in this regard is/are right?
(b) Sachin Bansal & Binny Bansal- Red Bus I. The ordinance exempts Person of Indian
(c) Deepinder goyal & Pankaj Chaddah - Origin (PIO) from appearing before the
Zomato local police station on every visit
(d) Bhavish Aggarwal & Ankit Bhati - Ola Cab II. It replaced the clause that says foreigners
marrying Indians must continuously stay
81. Which state is to host the 36th National in the country for a period of six months
Games in 2016? before they get an Indian citizenship
(a) Karnataka (b) Goa
(c) Tamil Nadu (d) Telangana (a) I Only (b) II Only
(c) Both I and II (d) Neither I nor II

Previous Years
Page 174 CLAT & AILET Papers
88. Name the renowned Indian ecologist who has 92. What is the approximate percent increase in
been chosen for the prestigious 2015 Tyler the number of units sold by Company E in
Prize for Environmental Achievement on 23 the year 2007 from the previous year?
March 2015? (a) 17 (b) 36 (c) 27 (d) 21
(a) MS Swaminathan
(b) Kasthuri Rangan 93. The number of units sold by Company D in
(c) Jayaram Ramesh the year 2006 is what percent of the number
(d) Madhav Gadgil of units manufactured by it in that year?
(rounded off to two digits after decimal)
89. How much Foreign Direct Investment (FDI) in (a) 52.63 (b) 61.57
country's defence sector was proposed in the (c) 85.15 (d) 73.33
Union Budget 2014-15 presented on 10 July
2014? 94. What is the respective ratio of total number
(a) 51% (b) 49% of units manufactured by Company A and B
(c) 29% (d) 10% together in the year 2009 to those sold by
them in the same year?
90. The protein CA-125 (Cancer Antigen-125) is (a) 2 : 01 (b) 3 : 02
used as biomarker for detection of which type (c) 5 : 02 (d) None of the Above
of cancer?
(a) Ovarian cancer (b) Bone cancer 95. What is the average number of units sold by
(c) Lung cancer (d) Oral cancer Company D over all the years together?
(a) 166 (b) 158 (c) 136 (d) 147
ELEMENTARY MATHEMATICS
(NUMERICAL ABILITY) 96. What is the value of (x) in the following
equation?
Direction (Qs. 91–95) : Answer the questions based x0.4/16 = 32/x2.6
on the information given in the following table. A, B, (a) 8 (b) 9 (c) 6 (d) 7
C, D, E denote companies and 2006, 2007, 2008,
2009 & 2010 denote years. 97. The simplified value of [(0.111)3 + (0.222)3 –
(0.333)3 + (0.333)2 x (0.222)]3 is:
Number of Units Manufactured (M) and Number of (a) 0.999 (b) 0.111
Units Sold (S) (in hundreds) by five different (c) 0 (d) 8.888
companies over the years.
98. When 2½ is added to a number and the sum
A B C D E is multiplied by 4½ and then 3 is added to
M S M S M S M S M S the product and then the sum is divided by
2006 2.8 1.3 3.3 2.2 2.6 1.7 3.0 2.2 1.9 1.4 11/5, the quotient becomes 25. What is that
2007 3.2 2.0 2.4 1.6 2.2 1.5 2.5 1.9 2.0 1.7 number?
2008 1.9 0.9 2.9 1.6 2.1 1.0 2.3 1.5 1.6 1.1 (a) 2½ (b) 3 ½ (c) 4 ½ (d) 5 ½
2009 1.0 0.4 2.4 1.3 2.8 1.4 2.1 1.2 3.2 2.5
2010 2.5 1.5 2.3 1.2 2.6 2.1 1.8 1.1 3.1 2.6 99. If x = (163+ 173 + 183 + 193), then x divided
by 70 leaves a remainder of ....
(a) 0 (b) 1 (c) 69 (d) 35
91. What is the total number of units
manufactured by Company C over all the years 100. A man has 9 friends: 4 boys and 5 girls. In
together? how many ways can he invite them, if there
(a) 1420 (b) 1030 have to be exactly 3 girls in the invitees?
(c) 1230 (d) 1320 (a) 320 (b) 160 (c) 80 (d) 200

Previous Years
CLAT & AILET Papers Page 175
101. A group of 630 children is arranged in rows tournament, considering all the n teams
for a group photograph session. Each row together?
contains three fewer children than the row in (a) k(n – 1) (b) n(k – 2)
front of it. What number of rows is not (c) k(n – 2) (d) n(k – 1)
possible?
(a) 3 (b) 4 (c) 5 (d) 6 108. If n2 = 12345678987654321, what is n?
(a) 12344321 (b) 1235789
102. A die is rolled twice. What is the probability (c) 111111111 (d) 11111111
that the sum of the numbers on the two faces
is 5? 109. Along a road lie an odd number of stones
(a) 3/13 (b) 4/13 (c) 6/13 (d) 1/9 placed at intervals of 10m. These stones have
to be assembled around the middle stone. A
103. Two trains, one from Howrah to Patna and person can carry only one stone at a time. A
other from Patna to Howrah, start man carried out the job starting with the stone
simultaneously. After they meet, the trains in the middle, carrying stones in succession,
reach their destinations after 9 hours and 16 thereby covering a distance of 4.8 km. Then,
hours respectively. The ratio of their speed the number of stones is:
is... (a) 35 (b) 15 (c) 31 (d) 29
(a) 2:03 (b) 4:03 (c) 6:07 (d) 9:06
110. What are the last two digits of 72008?
104. A watch which gains uniformly is 2 minutes (a) 01 (b) 21 (c) 61 (d) 71
slow at noon on Monday and is 4 minute 48
second fast at 2 p.m. on the following Monday. LEGAL APTITUDE
When was it correct?
(a) 2 p.m. on Tuesday Direction (Qs. 111–125) : The question consist of
(b) 2 p.m. on Wednesday two statements, one labelled as PRINCIPLE and
(c) 3 p.m. on Thursday other as FACT. You are to examine the principle and
(d) 1 p.m. on Friday apply it to the given facts carefully and select the
best option.
105. A speaks truth in 75% cases and B in 80% of
the cases. In what percentage of cases are 111. PRINCIPLE : Whoever, intending to take
they likely to contradict each other, narrating dishonestly any movable property out of the
the same incident? possession of any person without that
(a) 5% (b) 15% (c) 35% (d) 45% person's consent moves that property, such
taking is said to commit theft.
106. The sum of all the natural numbers from 200
to 600 (both inclusive) which are neither FACT : RAMU cuts down a tree on RINKU'S
divisible by 8 nor by 12 is: ground, with the intention of dishonestly
(a) 1, 23, 968 (b) 1, 33, 068 taking the tree out of RINKU'S possession
(c) 1, 33, 268 (d) 1, 87, 332 without RINKU'S consent. A could not take
the tree away.
107. In a tournament, there are n teams T 1;
T2,.......Tn, with n > 5. Each team consists of (a) RAMU can be prosecuted for theft
k players, k > 3. The following pairs of teams (b) RAMU cannot be prosecuted for theft
have one player in common T1 and T2, T2 and (c) RAMU can be prosecuted for attempt to
T3, ....., Tn–1 and Tn and Tn and T1. No other theft
pair of teams has any player in common. How (d) RAMU has neither committed theft nor
many players are participating in the attempt to commit theft

Previous Years
Page 176 CLAT & AILET Papers
112. PRINCIPLE : injuria sine damnum i.e. injury the assent of that other to such act or
without damage. abstinence, he is said to make a proposal.

FACT : SONU, who was a returning officer at FACT : "Ramanuj telegrammed to the
a polling booth, wrongly refused to register a Shyamsunder, writing: "will you sell me your
duly tendered vote of MONU, though he was Rolls Royce CAR? Telegram the lowest cash
a qualified voter. The candidate, whom MONU price." Shyamsunder replied, too by telegram:
sought to vote, was declared elected. 'Lowest price for CAR is Rs. 20 lacs.' Ramanuj
immediately sends his consent through
telegram stating: 'I agree to buy the CAR for
(a) MONU can sue SONU on the ground that
Rs. 20 laks asked by you.' Now Shyamsunder
he was denied to cast vote, which is a
refused to sell the CAR.
fundamental right.
(b) MONU can sue SONU on the ground that
(a) He cannot refuse to sell the CAR because
he was denied to cast vote, which is a
the contract has already been made.
legal right.
(b) He can refuse to sell the CAR because it
(c) MONU cannot sue SONU because there was only invitation to offer and not the real
is no injury or damage cause to MONU. offer.
(d) MONU cannot sue SONU because to (c) It was not a valid offer because willingness
whom he sought to vote was declared to enter into a contract was absent.
elected. (d) None of the above.

113. PRINCIPLE : A person is said to be of sound 115. PRINCIPLE : A master is liable for the acts
mind for the purpose of making a contract if, committed by his servant in the course of
at the time when he makes it, he is capable employment.
of understanding it and of forming a rational
judgment as to its effect upon his interests. FACT : Sanjay is a driver working in
Brookebond and Co. One day, the Manager
FACT : Mr. X who is usually of sound mind, asked him to drop a customer at the airport
but occasionally of unsound mind enters into and get back at the earliest. On his way back
a contract with Mr. Y when he is of unsound from the airport, he happened to see his fiance
mind. Y came to know about this fact Ruhina waiting for a bus to go home. He
afterwards and now wants to file a suit against offered to drop her at home, which happened
Mr. X. to be close to his office. She got into the car
and soon thereafter; the car somersaulted due
(a) Mr. X cannot enter into contract because to the negligence of Sanjay. Ruhina was
he is of unsound mind when he entered thrown out of the car and suffered multiple
into contract. injuries. She seeks compensation from
(b) Mr. X can enter into contract but the burden Brookebond and Co.
is on the other party to prove that he was
of unsound mind at the time of contract. (a) Brookebond and Co., shall be liable,
(c) Mr. X can enter into a contract but the because Sanjay was in the course of
burden is on Mr. X to prove that he was of employment at the time of accident
unsound mind at the time of contract. (b) Brookebond and Co., shall not be liable,
(d) None of the above. Sanjay was not in the course of
employment when he took Ruhina inside
114. PRINCIPLE : When one person signifies to the car.
another his willingness to do or to abstain (c) Ruhina got into the car at her own risk,
from doing anything, with a view to obtaining and therefore, she cannot sue anybody.
(d) None of the above.

Previous Years
CLAT & AILET Papers Page 177
116. PRINCIPLE: Nuisance as a tort (civil wrong) FACT: George was a passenger from Zurich
means an unlawful interference with a to Manila in a Swiss Plane. When the plane
person's use or enjoyment of land, or some landed at the airport at Bombay on 28th Nov.
right over, or in connection with it. 1962 it was found on search that George
carried 34 kgs of gold bars in person and that
FACT: During the scarcity of onions, long he had not declared it in the 'Manifest for
queues were made outside the defendant's transit'. On 26th Nov. 1962 Government of
shop who having a license to sell fruits and India issued a notification and modified its
vegetables used to sell only 1 Kg. of onion earlier exemption and now it is necessary
per ration card. The queues extended on to that, the gold must be declared in the
the highway and also caused some "Manifest" of the aircraft.
obstruction to the neighboring shops. The
neighboring shopkeepers brought an action (a) George cannot be prosecuted because
for nuisance against the defendant. he had actually no knowledge about the
new notification issued only two days ago.
(a) The defendant is liable for nuisance (b) George cannot be prosecuted because it
(b) The defendant was not liable for nuisance is mistake of fact which is excusable.
(c) The defendant was liable under the principle (c) George's will be prosecuted because
of strict liability mistake of law is not excusable.
(d) The plaintiffs suit should be decreed in (d) George liability would depend on the
his favour discretion of the court

117. PRINCIPLE: Nothing is an offence which is 119. PRINCIPLE : Everybody is under a legal
done by a person who, at the time of doing it, obligation to take reasonable care to avoid
by reason of unsoundness of mind, is an act or omission which he can foresee
incapable of knowing the nature of the act, or would injure his neighbor. The neighbor for
that he is doing what is either wrong or this purpose, is any person whom he should
contrary to law. have in his mind as likely to be affected by
his act.
FACT: A takes his son B who is three years
old, for a bath to the well. He throws his son FACT : Krishnan, while driving a car at high
inside the well so that he could have a good speed in a crowded road, knocked down a
bath. After 10 minutes he also jumped in the cyclist. The cyclist died on the spot with a lot
well to take a bath and take his son out of the of blood spilling around, Lakshmi, a pregnant
well. Both were rescued by the villagers but woman passing by, suffered from a nervous
his son was found dead. shock, leading to abortion. Lakshmi filed a
suit against Krishnan claiming damages.
(a) A has committed culpable homicide not
amounting to murder (a) Krishna will be liable, because he owed a
(b) A has committed murder duty of reasonable care to everybody on
(c) A has done no offence as he can plead the road including Lakshmi
the defence of unsoundness of mind (b) Krishna will not be liable, because he
(d) A's family should be responsible for this could not have foreseen Lakshmi suffering
incident to let him to take child to the well from nervous shock as a result of his act.
(c) Krishna will be liable to Lakshmi because
118. PRINCIPLE: ignorantia juris non excusat and he failed to drive carefully
ingnorantia facit excusat (d) None of the above

Previous Years
Page 178 CLAT & AILET Papers
120. PRINCIPLE : Preparation is not an offence (c) A cannot be held liable, because he did
except the preparation of some special not say anything positive about the
offences. soundness of horse.
(d) A cannot be held liable because it is the
FACT : Ramesh keeps poisoned halua in his buyer who must be aware of the things.
house, wishing to kill Binoy whom he invited
to a party and to whom he wishes to give it. 123. PRINCIPLE : Any direct physical interference
Unknown to Ramesh, his only son takes the with the goods in somebody's possession
halua and dies. In this case without lawful justification is called trespass
to goods.
(a) Ramesh is liable for the murder.
(b) He is not liable for murder since it is a FACT : A purchased a car from a person who
preparation alone. had no title to it and had sent it to a garage
(c) He is liable for culpable homicide for repair. X, believing, wrongly, that the car
(d) None of the above was his, removed it from the garage.

121. PRINCIPLE : Agreements, the meaning of (a) X can be held responsible for trespass to
which is not certain, or capable of being made goods.
certain, are void. (b) X cannot be held responsible for trespass
to good as he was under a wrong belief.
FACT : A horse was bought for a certain price (c) X has not committed any wrong.
coupled with a promise to give Rs. 500 more (d) None of the above.
if the horse proved lucky.
124. PRINCIPLE : "Nobody shall unlawfully
(a) This is a valid agreement. interfere with a person's use or enjoyment of
(b) This agreement is void for uncertainty land, or some right over, or in connection with
because it is very difficult to determine it. The use or enjoyment, envisaged herein,
what luck, bad or good, the horse had should be normal and reasonable taking into
brought to the buyer. account surrounding situation."
(c) The agreement is partially valid and
partially void. FACT : Jeevan and Pavan were neighbours in
(d) None of the above. a residential locality. Pavan started a typing
class in a part of his house and his typing
122. PRINCIPLE : Mere silence as to facts likely sound disturbed Jeevan who could not put up
to affect the willingness of a person to enter with any kind of continuous noise. He filed a
into a contract is not fraud, unless the suit against Pavan.
circumstances of the case are such that,
regard being had to them, it is the duty of the (a) Pavan is liable, because he should not
person keeping silence to speak, or unless have started typing class in his house
his silence is, in itself, equivalent to speech. (b) Pavan is liable, because as a neighbour,
he should have realised Jeevan's delicate
FACT : A sells, by auction, to B, a horse nature
which A knows to be unsound. A says nothing (c) Pavan is not liable, because typing sound
to B about the horse's unsoundness. did not disturb anyone else other than
Jeevan
(a) A can be held liable for fraud (d) None of the above.
(b) A can be held liable for misrepresentation

Previous Years
CLAT & AILET Papers Page 179
125. PRINCIPLE : Doctrine of Double Jeopardy: 128. Assertion (A) : Laws are means of achieving
No person shall be prosecuted and punished an end namely social control.
for the same offence twice. Reason (R) : The ultimate end of law is to
secure greatest happiness to greatest
FACT : Maqbool brought some gold into India number.
without making any declaration to Custom (a) Both A and R are individually true and R
department on the airport. The custom is correct explanation to A
authorities confiscated the gold under the Sea (b) Both A and R are individually true but R is
Customs Act. Maqbool was later charged for not correct explanation of A
having committed an offence under Foreign (c) A is true but R is false.
Exchange Regulation Act. (d) A is false but R is true.
(a) He cannot be prosecuted because it 129. Assertion (A) : Every person should have the
would amount to double jeopardy. freedom of speech and expression.
(b) He can be prosecuted because Reason (R) : If a person is stopped from
confiscation of good by custom authorities speaking then mankind will lose the truth.
does not amount to prosecution by the (a) Both A and R are individually true and R is
Court. correct explanation to A
(c) Maqbool ought to have known that he can (b) Both A and R are individually true but R is
be stopped by the custom authorities. not correct explanation of A
(d) None of the above. (c) A is true but R is false.
(d) A is false but R is true.
Direction (Qs. 126–135) : The question consist of
two statements, one labelled as Assertion (A) and 130. Assertion (A) : Attempt to commit an offence
other as Reason (R). You are to examine the two though does not result in any harm, should
statements carefully and select the best option. also be punished.
Reason (R) : A person who tries to cause a
126. Assertion (A) : Custom per se is law, prohibited harm and fails, is, in terms of moral
independent of prior recognition by the culpability, not materially different from the
sovereign or the judge. person who tries and succeeds.
Reason (R) : Custom is source of law but by (a) Both A and R are individually true and R
itself is not law. is correct explanation to A
(a) Both A and R are individually true and R (b) Both A and R are individually true but R is
is correct explanation to A not correct explanation of A
(b) Both A and R are individually true but R is (c) A is true but R is false.
not correct explanation of A (d) A is false but R is true.
(c) A is true but R is false.
(d) A is false but R is true. 131. Assertion (A) : In India, every state has a
High Court in its territory.
127. Assertion (A) : Idol is a person who can hold Reason (R) : The Constitution of India
property. provides for a High Court in each state.
Reason (R) : Only human being can be called (a) Both A and R are individually true and R
person not the lifeless things. is correct explanation to A
(a) Both A and R are individually true and R (b) Both A and R are individually true but R is
is correct explanation to A not correct explanation of A
(b) Both A and R are individually true but R is (c) A is true but R is false.
not correct explanation of A (d) A is false but R is true.
(c) A is true but R is false.
(d) A is false but R is true.

Previous Years
Page 180 CLAT & AILET Papers
132. Assertion (A) : The Council of Ministers at (b) Both A and R are individually true but R is
the centre is collectively responsible both to not correct explanation of A
the Lok Sabha and Rajya Sabha. (c) A is true but R is false.
Reason (R) : The members of both Lok Sabha (d) A is false but R is true.
and Rajya Sabha are eligible to be ministers
of the Union Government. Direction (Qs. 136–138) : Read the definition and
(a) Both A and R are individually true and R elements of the attempt, apply them on the given
is correct explanation to A fact situations and answer the question:
(b) Both A and R are individually true but R is
not correct explanation of A Definition of Attempt : Lord Blackburn has said that
(c) A is true but R is false. "there is no doubt that there is difference between a
(d) A is false but R is true. preparation antecedent to an attempt and the actual
attempt, but if the actual transaction has commenced
133. Assertion (A) : The reservation of thirty-three which would have ended in the crime if not interrupted,
percent of seats for women in Parliament and there is clearly an attempt to commit the crime."
State Legislature does not require 1. Fault element: Intention or knowledge
Constitutional Amendment. requisite for committing an offence; and
Reason (R) : Political parties contesting 2. Conduct Element: does any act towards its
elections can allocate thirty-three per cent of commission and has crossed the stage of
seats they contest to women candidates preparation. This act is so closely connected
without any Constitutional Amendment. with, and proximate to the commission that
(a) Both A and R are individually true and R it fails in object because of facts not known
is correct explanation to A to him or because of circumstances beyond
(b) Both A and R are individually true but R is his control.
not correct explanation of A
(c) A is true but R is false. 136. 'RANI' ran to a well stating that she would
(d) A is false but R is true. jump into it, and she started running towards
the well but she was caught before she could
134. Assertion (A) : We, the people of India, having reach it.
solemnly resolved to constitute India into a (a) She is not guilty of attempt to commit
Democratic Republic. suicide because she might have changed
Reason (R) : A republic will ensure we have a her mind before jumping into the well.
head of state that is democratically elected (b) She is guilty of attempt to commit suicide
and accountable to voters. As a result the (c) Right to life includes rights right to die
head of state will be a more effective hence a person should not be held
constitutional safeguard. responsible for attempt to commit suicide.
(a) Both A and R are individually true and R (d) None of the above.
is correct explanation to A
(b) Both A and R are individually true but R is 137. 'SINY' with an intention to pick-pocket puts
not correct explanation of A his hand into MINU's pocket. MINU had a
(c) A is true but R is false. loaded pistol in his pocket. The thief touches
(d) A is false but R is true. the pistol and trigger goes on, whereby MINU
is shot dead.
135. Assertion (A) : Republic Day is celebrated (a) SINY will be liable only for attempting to
on 26th January every year in the country. pick-pocket and not for killing because
Reason (R) : The Constitution of India came she cannot be treated differently from all
into force on 26th January 1950. other pick-pockets who steal under
(a) Both A and R are individually true and R exactly similar circumstances and same
is correct explanation to A

Previous Years
CLAT & AILET Papers Page 181
intention, with no risk of causing death 140. The ................Legislative Assembly on 31st
and with no greater care to avoid it. March 2015 passed a controversial Anti-
(b) SINY will be liable for attempting to murder Terrorism Law. Earlier, the passed bill was
(c) SINY will be liable for culpable homicide rejected two times by the then ................ in
not amounting to murder as his intention 2004 and 2008.
was definitely not to kill. (a) Bihar, Presidents
(d) None of the above (b) Madhya Pradesh, Governors
(c) Gujarat, Presidents
138. 'JAM' denied food to his wife JANE for several (d) Maharashtra, Governors
days by keeping her confined in a room with
an intention to accelerate her death. JANE 141. The Union Government on the
ultimately managed to escape. recommendation of the ............under the
(a) JAM is guilty for attempt to murder his chairmanship of ................. has decided to
wife. decriminalise Section...............of the Indian
(b) JAM is not guilty for attempt to murder Penal Code.
his wife and he was only doing preparation. (a) 20th Law Commission, Justice A.P. Shah
& 309
(c) JAM is not guilty for attempt to murder
(b) 20th Law Commission, Justice M.P. Shah
his wife as she always had option to
& 307
escape.
(c) Supreme Court, Justice H.L. Dattu &
(d) None of the above.
Section 309
(d) Planning Commission, Law Minister,
Direction (Qs. 139–142) : Fill in the blanks: Choose
section 309
the pair of words which complete the sentence to
make logical sense. 142. A bench headed by ................ quashed
allocation of 214 ......... ...as ....................
139. The NDA led Government notified the (a) Justice H L Dattu, coal blocks, illegal and
.................... and the National Judicial arbitrary
Appointments Commission Act, thus ending (b) Justice R. M. Lodha, coal blocks, illegal
the over two-decade-old .................of and arbitrary
appointing judges of Supreme Court and high (c) Justice T. S. Thakur, licenses, illegal and
courts. Under the new law, a six-member arbitrary
panel headed by..................... will select (d) None of the above.
judges of the apex court and state high courts.
(a) 99th Constitutional (Amendment) Act 143. Choose the best option for the following
2015, collegium system, the Chief Justice statement:
of India No one can be compelled to sing the National
(b) 121st Constitutional (Amendment) Act Anthem since:
2015, collegium system, the Union Law 1. It will be violative of the right to freedom of
Minister speech and expression.
(c) 121st Constitutional (Amendment) Act 2. It will be violative of the right to freedom of
2015, collegium system, the Prime conscience and practice & propagation
Minister of religion.
(d) 99th Constitutional (Amendment) Act 3. There is no legal provision obliging anyone
2015, cabinet system, the Prime Minister to sing the National Anthem.

(a) 1 and 2 are correct


(b) 2 and 3 are correct
(c) 1, 2 and 3 are correct
(d) None is correct
Previous Years
Page 182 CLAT & AILET Papers
144. Five years' experience is a must to be able to 4. Hon'ble Mr. Justice T. S. Thakur is the
practice as an advocate in the Supreme Court present Chairman of NALSA.
of India. This rule was prescribed by the .........
(a) Bar Council of India (a) 1 and 2 are correct
(b) Supreme Court of India (b) 2 and 3 are correct
(c) High Court of Delhi (c) 3 and 4 are correct
(d) Ministry of Law and Justice, Government (d) None is correct
of India
148. India and Britain recently signed an
145. Union Government recently approved 33 per "extradition treaty". Extradition means -
cent Reservation for Women in: (a) Exports without double taxation
(a) Horizontally and each category (OBC, SC, (b) Order of Indian courts will apply to Indians
ST, and others) in direct recruitment in all living in the U.K.
(d) India and the U.K. will deport criminals on
non-gazetted Police Posts in all Union
reciprocal basis to each other
Territories including Delhi.
(d) None of the above
(b) Horizontally and each category (OBC, SC,
ST, and others) in direct recruitment in all
149. What is a 'moot’?
gazetted Police Posts in all Union
(a) A basic point of law
Territories including Delhi. (b) A basic fact of case
(c) Horizontally and each category (OBC, SC, (c) Mock court for practice by students in
ST, and others) in direct recruitment in all general
gazetted and non-gazetted Police Posts (d) Another name for magistrate's court
in all Union Territories including Delhi.
(d) Horizontally and each category (OBC, SC, 150. The temporary release of a convicted prisoner
ST, and others) in direct recruitment in all from jail for a fixed period is called -
gazetted and non-gazetted Posts in all (a) Bail (b) Parole
Union Territories including Delhi. (c) Acquittal (d) Discharge

146. As per Indian Protocol, who among the 151. The Railway authorities allowed a train to be
followings ranks highest in the order of over-crowded. In consequence, a legitimate
precedence? passenger, Mr. X got his pocket picked.
(a) Deputy Prime Minister Choose appropriate answer -
(b) Former President (a) Mr. X can sue the railway authorities for
(c) Governor of a State within his State the loss suffered.
(d) Speaker of Lok Sabha (b) Mr. X cannot sue because he had given
his consent to travel in a over-crowded
147. Consider the following statements and choose train.
the best option: (c) Mr. X cannot sue the railway authorities
1. The Chairman of the National Legal because there was no infringement of legal
Services Authority (NALSA) is the Chief right and mere fact that the loss was
Justice of India. caused does not give rise to a cause of
2. Chief Justice Mr. Justice H. L Dattu is the action.
(d) None of the above
present Chairman of NALSA.
3. The Chairman of the National Legal
Services Authority (NALSA) is the senior
most judge (after CJI) of the Supreme
Court of India.

Previous Years
CLAT & AILET Papers Page 183
152. Choose the best option for the following 154. Select the correct statements on Social
statement: Justice Bench constituted on social issue
The distinction between fraud and 1. Constituted by Supreme Court on 3
misrepresentation: December 2014
1. Fraud is more or less intentional wrong, 2. Started operation on 12 December 2014
whereas misrepresentation may be quite 3. The brainchild of Chief Justice of India H
innocent. L Dattu
2. In addition to rendering the contract 4. Two-judge bench to be headed by Justice
voidable, is a cause of action in tort for Madan B Lokur
damages. Simple misrepresentation is not 5. The other member is Justice U U Lalit
a tort but a person who rightfully rescinds
a contract is entitled to compensation for (a) 1, 2 & 5 are correct
any damages which he has sustained (b) 1, 2 & 3 are correct
through the non-fulfilment of the contract. (c) 1, 3 & 4 are correct
3. A person complaining of misrepresentation (d) All are correct
can be met with the defence that he had
"the means of discovering the truth with 155. Select the correct statements about 14th
ordinary diligence". But excepting fraud Finance Commission which submitted its
by silence in other cases of fraud it is no report to President
defence that "the plaintiff had the means 1. It covers the period between 1 April 2015
of discovering the truth by ordinary and 31 March 2020.
diligence". 2. The Commission headed by former RBI
4. None of the above. Governor Y V Reddy.
3. Provides for devolution of tax receipts from
(a) 1 is correct. the Centre to the States.
(b) 1 & 2 are correct. 4. Article 280 of Constitution provides for
(c) 1, 2 & 3 are correct. appointment of Finance Commission.
(d) Only 4 is correct. 5. 1st and 13th Finance Commission was
headed by K C Neogy & Dr Vijay Kelkar
153. In a recent case a Supreme Court bench respectively. Options :
comprising of Justice Dipak Misra and Justice
Prafulla C Pant held that the amount of (a) 1, 3 & 5 are correct
maintenance to be awarded under Section (b) 1, 2 & 3 are correct
125 of CrPC cannot be restricted for the iddat (c) 1, 3 & 4 are correct
period (three months) only as the inherent (d) All are correct
and fundamental principle behind Section 125.
Also, it said that an order under Section 125 156. Who administers oath of office to the Governor
CrPC can be passed if a person, despite of a State?
having sufficient means, neglects or refuses (a) President of India
to maintain the wife. (b) Chief Justice of High Court of the respective
(a) Shamima Farooqui v. Shahid Khan state
(b) Mohd. Ahmad Khan v. Shah Bano Begum (c) Chief Justice of India
(c) Hamida Bano v. Abdul Rasheed (d) Speaker of State Assembly
(d) Abdul Kadir v. Salima
157. Governor of a State can make Laws during
recess of State Legislative Assembly
through..............
(a) Act (b) Bills
(c) Notification (d) Ordinance

Previous Years
Page 184 CLAT & AILET Papers
158. Who called Indian Constitution as Quasi- 164. Who became the gainer by highest amount
Federal? through exchange?
(a) Austin (b) K. C. Wheare (a) W (b) X (c) Y (d) Z
(c) H. M. Servai (d) Jennings
165. The person incurring the highest amount of
159. President of India exercises his powers.......... financial loss through exchange lost an
(a) Either directly or through officers amount of:
subordinate to him (a) Rs. 600 (b) Rs. 1000
(b) Through ministers (c) None (d) Rs. 500
(c) Through Prime Minister
(d) Through Cabinet 166. The amount of price of all the things remaining
with the four persons lie between:
160. Vote on accounts is meant for.......... (a) Rs. 800 - Rs. 900
(a) Vote on the report of CAG (b) Rs.10000 -Rs. 12000
(b) To meet un for seen expenditure (c) Rs. 9000-Rs. 10000
(c) Appropriating funds pending passing of (d) Rs.10000-Rs. 11000
budget
(d) Budget 167. Even after exchanges, an item of highest value
remained in possession of:
LOGICAL REASONING (a) W (b) X (c) Y (d) Z

Direction (Qs. 139–142) : W, X, Y, and Z are four 168. Among the things exchanged, which one
friends, who do not mind exchanging items. X has faced the highest exchange value in
two chessboards each costing Rs. 500, and a record percentage term.
player. Z originally had a cycle and a walkman. Each (a) Cricket Bat (b) Record Player
cricket bat costs Rs. 700. Both W and Z got a (c) Camera (d) Cycle
cricket bat from Y. X gave his record player costing
Rs. 2000 to Y. Z get a camera costing Rs. 1500 Direction (Qs. 169–173) : Read the following
from W. The cycle of Z costs Rs. 1000 and the information and choose the best option.
walkman is for Rs. 700. Y had three cricket bats at
the beginning and W had two cameras the total cost Recently, the answers of a test held nationwide were
of which is Rs. 5000. X gave one of his chessboards leaked to a group of unscrupulous people. The
to Z and took Z’s cycle. Z gave his walkman to W. investigative agency has arrested the mastermind
and nine other people A, B, C, D, E, F, G, H and I in
161. Total cost of materials Z had at the beginning this matter. Interrogating them, the following facts
was : have been obtained regarding their operation. Initially,
(a) Rs. 1500 (b) Rs. 1700 the mastermind obtains the correct answer-key. All
(c) RS. 1000 (d) RS. 2000 the others create their answer-key from one or two
people who already possess the same. These people
162. At the beginning who had the costliest items: are called his/her "sources". If the person has two
(a) W (b) X sources, then he/she compares the answer-keys
(c) Y (d) Z obtained for both sources. If the key to a question
from both sources is identical, it is copied, otherwise
163. Who did not have a cricket bat after the it is left blank. If the person has only one source, he/
exchange of items was over? she copies the source's answer into his/her copy.
(a) W (b) X Finally, each person compulsorily replaces one of
(c) Y (d) Z the answers (not a blank one) with a wrong answer
in his/her answer key.

Previous Years
CLAT & AILET Papers Page 185
The paper contained 200 questions; so the Direction (Qs. 174–178) : In the following question,
investigative agency has ruled out the possibility of a group of numerals is given followed by four groups
two or more of them introducing wrong answers to of symbol/letter combinations lettered (A), (B), (C)
the same question. The investigative agency has a and (D). Numerals are to be coded as per the codes
copy of the correct answer key and has tabulated and conditions.
the following data. This data represents question
numbers. You have to find out which of the combinations (A),
(B), (C) and (D) is correct and indicate your answer
Name Wrong Answer(s) Blank Answer(s) accordingly.
A 46 –––
B 96 46,90,25
Numerals: 3 5 7 4 2 6 8 1 0 9
C 27,56 17,46,90
Letter/symbol code: * B E A @ F K % R M
D 17 –––
E 46,90 –––
F 14,46 92,90 Following conditions apply:
G 25 ––– 1. If the first digit as well as the last digit is odd,
H 46,92 ––– both are to be coded as 'X’.
I 27 17,26,90 2. If the first digit as well as the last digit is even,
both are to be coded as ‘$’.
169. Which one of the following must have two 3. If the last digit is 'O', it is to be coded as #.
sources?
(a) A (b) B (c) C (d) D 174. Find out the combination for : 487692
(a) $KEFM@ (b) AKEFM@
170. How many people (excluding the mastermind) (c) AKFEM@ (d) $KEFM$
needed to make answer keys before C could
make his answer key? 175. Find out the combination for : 713540
(a) 2 (b) 3 (c) 4 (d) 5 (a) X%*BA#
(b) E%*BA#
171. Both G and H were sources to .... Options : (c) E%*BAR
(a) F (b) B (d) None of the above
(c) A (d) None of the above
176. Find out the combination for : 765082
172. Which of the following statement is true? (a) EFB#K@
(a) introduced the wrong answer to question (b) XFBRK@
27 (c) EFBRK@
(b) E introduced the wrong answer to (d) None of the above
question 46
(c) F introduced the wrong answer to 177. Find out the combination for : 364819
question 14 (a) *FAK%X
(d) H introduced the wrong answer to (b) XFAK&M
question 46 (c) *FAK%M
(d) None of the above
173. Which of the following two groups of people
has identical sources? 178. Find out the combination for : 546839
I. A, D and G (a) XAFK*X
II. E and H (b) XAFK*M
(c) BAFK*X
(a) Only (I) (b) Only (II) (d) None of the above
(c) Neither (I) nor (II) (d) Both (I) and (II)

Previous Years
Page 186 CLAT & AILET Papers
Direction (Qs. 179–181) : From the alternatives given C. The protected environment in our country
below. is helping the local manufacturers.
D. The quality of goods suffers if the manu-
Choose the best option that correctly classifies the facturers take undue advantage of this.
four sentences as a :
F: Fact : If it relates to a known matter of direct (a) IJFJ (b) JFJI (c) IJJF (d) IFJJ
observation, or an existing reality or something
known to be true. Direction (Qs. 182–184) : Question consists of five
J: Judgment : If it is an opinion or estimate or statements followed by options consisting of three
anticipation of common sense or intention. statements put together in a specific order. Choose
I: Inference : if it is a logical conclusion or the best option which indicates a valid argument,
deduction about something based on the that is, where the third statement is a conclusion
knowledge of facts. drawn from the preceding two statements.

179. A. Everyday social life is impossible without 182. A. Traffic congestion increases carbon
interpersonal relationships. monoxide in the environment.
B. The root of many misunderstandings has B. Increase in carbon monoxide is hazardous
been cited in poor relations among to wealth.
individuals. C. Traffic congestion is hazardous to health.
C. Assuming the above to be true, social life D. Some traffic congestion does not cause
will be much better if people understand increase carbon monoxide.
the importance of good interpersonal E. Some traffic congestion is not hazardous
relations. to health
D. A study reveals that interpersonal relations
and hence life in general can be improved (a) CBA (b) BDE
with a little effort on the part of individuals. (c) CDE (d) BAC

(a) FJIJ (b) JFIF (c) FIFJ (d) IFFJ 183. A. MBAs are in great demand.
B. Samrat and Akshita are in great demand.
C. Samrat is in great demand.
180. A. The Cabinet minister definitely took the
D. Akshita is in great demand.
wrong step in giving the government
E. Samrat and Akshita are MBAs
contract.
B. Under the circumstances, he had many
(a) ABE (b) ECD
other alternatives. (c) AEB (d) EBA
C. The Prime Minister is embarrassed due
to the minister's decision. 184. A. All software companies employ
D. If he has put the government in jeopardy, knowledge workers.
the minister must resign. B. Infotech employees are knowledge
workers.
(a) JFFI (b) IFJI (c) FFJI (d) IFIJ C. Infotech is a software company.
D. Some software companies employ
181. A. If democracy is to survive, the people must knowledge workers.
develop a sense of consumerism. E. Infotech employs only knowledge workers.
B. Consumerism has helped improve the
quality of goods in certain countries. (a) ABC (b) ACB
(c) CDB (d) ACE

Previous Years
CLAT & AILET Papers Page 187
Direction (Qs. 185–186) : Read the following 187. Can a democratic system operate without an
information carefully to choose best option for the effective opposition?
question: I. The opposition is indispensable.
A. 'L % M' means that M is brother of L. II. A good politician always learns more from
B. 'L ´ M' means that L is mother of M. his opponents than from his fervent
C. 'L ¸ M' means that L is the sister of M. supporters.
D. 'L = M' means that M is father of L.
(a) Answer (1) (b) Answer (2)
185. Which of the following means "I is the nephew (c) Answer (3) (d) Answer (4)
of Q?"
1. Q % J = I 188. Do habits make men's life rigid?
2. Q ¸ M ´ B % I I. It is out of habit that people envy others.
3. C ¸ I = B % Q II. Men become slave of habits.

(a) Only 3 (b) Only 1 (a) Answer (1) (b) Answer (2)
(c) Only 2 (d) None of the above (c) Answer (3) (d) Answer (4)

186. If 'A $ B' means that A is the father of B, 'A * B' 189. Does intelligence predict the child's ability to
means that A is the mother of B, 'A @ E' learn?
means that A is the wife of B, then which of I. Intelligence is unaffected by bad teaching
the following means that M is the grand-mother or dull home environment.
of N? II. Children from poor home backgrounds do
(a) M * R $ T @ N not do well in their school-work.
(b) M * R @ T @ N
(c) M * T $ N @ R (a) Answer (1) (b) Answer (2)
(d) M * T $ N @ R (c) Answer (3) (d) Answer (4)

Direction (Qs. 187–189) : The question contains Direction (Qs. 190–192) : In the following question
two statements numbered I and II. some capital alphabets are written in a row, below
them their coding has been given. In the question, a
You have to decide whether the information provided particular word has been coded in a particular manner
in the statements are sufficient to answer the using codes as given below the capital letters.
question. Read both the statements and give your
answer as: You have to understand the pattern of coding and
Answer (1) If the information in the statement I alone have to answer the question asked subsequently.
are sufficient to answer the questions. A B C D E F G H I J K L M NO P Q R S T U VW X Y Z
Answer (2) If the information in the statement II alone Ua 2 v bw3 t 4 x s y 5 z 6 c d 8 7 e r h 9 I p q
are sufficient to answer the questions.
Answer (3) If the information either in the statement
I alone or in statement II alone are sufficient to answer 190. If DASH is 2a84, then SMASH is?
the questions. (a) 7 5 U 7 t (b) e y a 8 4
Answer (4) If the information even in both statement (c) 8 z q e 3 (d) 8 z q e 4
I and II together are not sufficient to answer the
questions. 191. If FASHION is z64t7w, then POSITION is?
(a) z 6 4 e 4 7 6 c (b) z 6 4 e 4 7 c 6
(c) c 6 7 4 e 4 6 z (d) c 6 7 4 e 4 z 6

Previous Years
Page 188 CLAT & AILET Papers
192. If LONDON is 5c62z5, then EUROPE is? 196. There are six houses in a row. Mr. Aalekh
(a) w h 7c d w (b) w h 7 z 6 v has Mr. Mishra and Mr. Iliyas as neighbours.
(c) b r 7 6 c b (d) w h 7 6 c b Mr. Mritynjay has Mr. Sandeep and Mr. Nayak
as neighbours. Mr. Sandeep's house is not
Direction (Qs. 193–194) : The question contains next to Mr. Mishra or Mr. Iliyas and Mr. Nayak
two statements and two conclusions numbered I does not live next to Mr. Iliyas. Who are Mr.
and II. Mishra's next door neighbours?
(a) Mr. Aalekh and Mr. Mishra
You have to take the two given statements to be (b) Mr. Aalekh and Mr. Iliyas
true even if they seem to be at variance from (c) Mr. Nayak and Mr. Aalekh
commonly known facts and decide which of the given (d) None of the above
conclusion (s) logically follow(s) from the two given
statements. 197. From the word 'LAPAROSCOPY', how many
Answer (1) If only conclusion I follows. independent meaningful English words can
Answer (2) If only conclusion II follows. be made without changing the order of the
Answer (3) If neither I nor II follows. letters and using each letter only once?
Answer (4) If both I and II follows. (a) 1 (b) 2
(c) 3 (d) 4
193. Statements:
I. Some players are singers. 198. If Monday falls on 1st of October, which day
II. All singers are tall. will fall three days after the 20th in that month?
Conclusions: (a) Monday
I. Some players are tall. (b) Tuesday
II. All players are tall. (c) Saturday
(d) Wednesday
(a) Answer (1) (b) Answer (2)
(c) Answer (3) (d) Answer (4) 199. In the word GRAPHOLOGIST, if 1st and 7th
194. Statements: letters, 2nd and 9th letters, 3rd and 11th letters,
I. Some vegetables are fruits. 4th and 8th letters and 5th and 12th letters are
II. No fruit is black. mutually interchanged then which letter will
Conclusions: be 6th letter from the left of 10th letter from
I. Some fruits are vegetables. the left side?
II. No vegetable is black. (a) S (b) T
(c) G (d) None of the above
(a) Answer (1) (b) Answer (2)
(c) Answer (3) (d) Answer (4) 200. The son of M is the father of N and grandfather
(Mother's father) of R. S is the daughter of N
195. Amit first goes in South direction, then he and sister of B. On the basis of this
turns towards left and travel for some distance. information, how is M related to B?
After that he turns right and moves certain (a) Grandfather
distance. At last he turns left and travels (b) Grandmother
again for some distance. Now, in which (c) Grandmother's mother
direction is he moving? (d) None of the above
(a) South (b) West
(c) East (d) None of the above

Previous Years
CLAT & AILET Papers Page 189
CLAT Question Paper 2016

Direction for Questions 1-10: Fill in the blanks by 9. Professor Ahmed __________ teaching us
choosing the most appropriate option: __________ August, 2012.
(1) had been, since (2) has been, since
1. I like reading journals ______________ novels. (3) was, for (4) has been, for

(1) the less than (2) most than 10. There isn't ____________ water in the
(3) the best (4) more than overhead tank.
(1) little (2) some
2. If you promise ____________ angry with me, (3) lot of (4) any
I'll tell you what I broke.
(1) get not (2) not to get Direction for questions 11-15: Complete the
(3) not getting (4) not get sentences with the correct options:

3. They always give the available seats to 11. I can't bear ___________ late.
__________________ comes first. (1) been (2) to
(1) whomever (2) whoever (3) be (4) being
(3) whom (4) whichever
12. Mani, along with his friends,
4. A thief does not _____________ the door. ________________ for basket ball practice
(1) knock (2) knock at every morning.
(3) knock on (4) knock to (1) were going (2) goes
(3) are going (4) go
5. It's unfortunate that he died ___________
cancer. 13. I have never come ______________ any one
(1) by (2) of as rude as him.
(3) with (4) from (1) across (2) around
(3) to (4) at
6. The method and practice of teaching is called
_________. 14. Her parents have arrived ___________ the
(1) philately (2) paediatrics airport.
(3) training (4) pedagogy (1) at (2) on (3) to (4) in

7. There is a lot of work __________ hand. Let's 15. They returned home _____________ a taxi.
cancel ____________ picnic. (1) from (2) in (3) with (4) on
(1) in, the (2) in, a
(3) upon, the (4) on, a Direction for questions 16-20: Choose the correct
spellings in questions given below:
8. A fire broke __________ in the neighbourhood.
(1) away (2) off 16. Choose the correct spellings
(3) from (4) out (1) sacribegous (2) sacrilegious
(3) sacrilegiuos (4) saereligious

Previous Years
Page 190 CLAT & AILET Papers
17. Choose the correct spellings 24. Choose the correct option
(1) barrister (2) barisster (1) The Titanic did not reach their destination.
(3) berister (4) berrister (2) The Titanic did not reach her destination.
(3) The Titanic did not reach his destination.
18. Choose the correct spellings (4) The Titanic did not reach it's destination.
(1) deceive (2) decieve
(3) diecieve (4) deceeve Direction for Questions 25-28: Choose the correct
option for each of the following questions:
19. Choose the correct spellings
(1) colaborate (2) collaborrate 25. "Faux pas" means:
(3) collaborat (4) collaborate (1) false
(2) expected to happen
20. Choose the correct spellings (3) fake identity
(1) integrity (2) integerity (4) social blunder
(3) integerety (4) intigerity
26. "To bury the hatchet" means:
Direction for Questions 21-24: Choose the most (1) to plant grass in the field
appropriate option for each of the following questions: (2) to end a feud with an enemy
(3) to hide some treasure
(4) to cremate carcass of an animal
21. Choose the correct option
(1) Please stop interfering with my romantic 27. "Ab initio" means:
life. (1) from the beginning
(2) Please stop interfering for my romantic (2) in connection with the future
life. (3) in continuation with
(3) Please stop interfering into my romantic (4) in defence of a comment
life.
(4) Please stop interfering in my romantic life. 28. "Amicus curiae" means:
(1) One among the parties to the dispute
22. Choose the correct option (2) One among the expert witnesses in a case
(1) The Film Star got off the car and smiled at (3) One of the judges of the court
the people. (4) A friend of the court
(2) The Film Star come out of the car and
smiled at the people. Direction for Questions 29-30: Complete the
(3) The Film Star got out of the car and smiled proverb, in the following questions:
at the people.
(4) The Film Star climbed off the car and 29. When _____________ is bliss, it is ______
smiled at the people. to be wise.
(1) knowledge, better
23. Choose the correct option (2) bachelorhood, single
(1) Your grammar is very good but you need (3) ignorance, folly
to work on managing your pronunciation. (4) romance, boring
(2) Your grammar is very good, but you need
to work on correcting your pronunciation. 30. ______________ waters run ____________.
(3) Your grammar is very good, but you need (1) Stagnant, leisurely
to work on repairing your pronunciation. (2) Still, deep
(4) Your grammar is very good, but you need (3) Muddy, slow
to work on modifying your pronunciation. (4) Hot, fast

Previous Years
CLAT & AILET Papers Page 191
Direction for Questions 31-40: Read the given The matter was considered and a project
passage carefully and answer the questions given called the Manhattan Project was initiated.
after the passage: The project involved intense nuclear research
1. Often, we passionately pursue matters that the construction of the world's first atomic
in the future appear to be contradictory to our bomb. All this while, Einstein had the
real intention or nature; and triumph is impression that the bomb would be used to
followed by remorse or regret. There are protect the world from the Nazis. But in 1945,
numerous examples of such a trend in the when Hiroshima was bombed to end World
annals of history and contemporary life. War II, Einstein was deeply grieved and he
2. Alfred Nobel was the son of Immanuel Nobel, regretted his endorsement of the need for
an inventor who experimented extensively with nuclear research.
explosives. Alfred too carried out research and 6. He also stated that had he known that the
experiments with a large range of chemicals; Germans would be unsuccessful in making
he found new methods to blast rocks for the the atomic bomb, he would have probably
construction of roads and bridges; he was never recommended making one. In 1947,
engaged in the development of technology and Einstein began working for the cause of
different weapons; his life revolved around disarmament. But, Einstein's name still
rockets and cannons and gun powder. The continues to be linked with the bomb.
ingenuity of the scientist brought him enough Man's fluctuating thoughts, changing
wealth to buy the Bofors armament plant in opinions, varying opportunities keep the mind
Sweden. in a state of flux. Hence, the paradox of life:
3. Paradoxically, Nobel's life was a busy one it's certain that nothing is certain in life.
yet he was lonely; and as he grew older, he
began suffering from guilt of having invented 31. The passage is _____________.
the dynamite that was being used for (1) a descriptive essay.
destructive purposes. He set aside a huge (2) an argumentative essay
part of his wealth to institute Nobel Prizes. (3) an expository essay.
Besides honouring men and women for their (4) a process essay.
extraordinary achievements in physics,
chemistry, medicine and literature, he wished 32 Einstein had the impression that the Germans
to honour people who worked for the would __________.
promotion of peace. (1) be successful in making the world's first
4. It's strange that the very man whose name atomic bomb.
was closely connected with explosives and (2) bomb Hiroshima.
inventions that helped in waging wars willed (3) work for humanity.
a large part of his earnings for the people who (4) be unsuccessful in making the atomic
work for the promotion of peace and the bomb.
benefit of mankind. The Nobel Peace Prize is
intended for a person who has accomplished 33. Alfred established the Nobel Prizes to
the best work for fraternity among nations, ____________.
for abolition or reduction of war and for (1) honour only those people who are
promotion of peace. intelligent.
5. Another example that comes to one's mind (2) ease his guilt and promote work for the
is that of Albert Einstein. In 1939, fearing that betterment of mankind.
the Nazis would win the race to build the (3) use his wealth for hard working people.
world's first atomic bomb, Einstein urged (4) remind people of his achievements.
President Franklin D Roosevelt to launch an
American programme on nuclear research.

Previous Years
Page 192 CLAT & AILET Papers
34. Immanuel's interest in dynamites influenced GENERAL KNOWLEDGE AND
Alfred's inclination for working ___________. CURRENT AFFAIRS
(1) with explosives
(2) with contradiction 1. In January, 2016, at the first stage, the Union
(3) for humanity Ministry of Urban Development unveiled the
(4) for the Nobel Peace Prize list of ________ cities for smart cities mission
out of 98 shortlisted cities:
35. In the fifth paragraph, the word 'endorsement' (1) 20 (2) 25 (3) 30 (4) 15
means
(1) expressing one's approval or support. 2. Which organization has the motto 'Be
(2) expressing one's regret. Prepared'?
(3) expressing one's opposition. (1) National Cadet Corps
(4) making a promise to do something. (2) National Disaster Management Authority
(3) Boys' Scout
36 In paragraph 4, the word 'accomplished' (4) National Service Scheme
means ___________.
(1) won awards. 3. In which of the following States / Union
(2) completed successfully. Territories, the Election Commission has
(3) worked hard. decided to hold election in a single phase on
(4) made an effort to do something. May 16, 2016?
(1) West Bengal, Kerala and Assam.
37. The paradox, 'it's certain that nothing is certain (2) West Bengal, Kerala and Tamil Nadu.
in life', indicates the writer's (3) Assam, Kerala and Pondicherry.
(1) scientific mind (4) Kerala, Tamil Nadu and Pondicherry.
(2) persuasive nature
(3) analytical mind 4. "Beyond the Lines - An Autobiography" is
(4) hatred for scientists authored by:
(1) Mr. Arun Shourie (2) Justice Krishna Iyer
38. One of the paradoxes in Alfred's life was that (3) Mr. Kuldip Nayar (4) Mr. Soli Sarab Ji
he was ___________.
(1) lonely yet rich 5. The Currency of Thailand is:
(2) hard working but a failure (1) Ringgit (2) Baht
(3) intelligent yet lonely (3) Peseta (4) Rupee
(4) occupied yet lonely
6. Which State has become the first State to
39. The Manhattan Project was initiated introduce compulsory gender education at the
_____________. graduate level?
(1) to carry out nuclear research. (1) Punjab (2) Telangana
(2) to protect the Nazis. (3) Andhra Pradesh (4) Kerala
(3) to bomb Hiroshima.
(4) in honour of Einstein. 7. A Japanese maglev, which is the fastest
passenger train in the world, has broken its
40. Working with arms and ammunition helped own record in 2015. The train reached
Alfred to amass _________. ____________ kmph in the test run.
(1) wealth (2) intelligence (1) 453 (2) 603
(3) popularity (4) enemies (3) 503 (4) 403

Previous Years
CLAT & AILET Papers Page 193
8. The Rajya Sabha in April, 2016 passed two 16. Under the Constitution of India the official
Bills, which had already been passed by the language of the Union is:
Lok Sabha earlier, repealing certain outdated/ (1) Hindi in Devanagari Script
old laws. The Bills intended to repeal: (2) Hindi
(1) 315 laws (2) 1053 laws (3) English
(3) 153 laws (4) 513 laws (4) English and Hindi

9. Country's first 'visually-challenged friendly' 17. Power to summon the Houses of the
railway station is: Parliament is vested with:
(1) Varanasi (2) Tirupathi (1) Prime Minister
(3) Mysuru (4) New Delhi (2) Vice-President
(3) Speaker and Vice-President
10. The country which cloned a buffalo calf first (4) President
is: 18. In April 2016, The Duke and Duchess of
(1) India (2) Sweden Cambridge visited India. Their names are:
(3) Denmark (4) U. S. A. (1) Prince Hillery and Victoria.
(2) Prince William and Kate Middleton
11. Who was crowned as the winner of Femina (3) Prince Robert and Kateler
Miss India 2016 and who will represent India (4) Prince Charles and Cathy
at the Miss World pageant?
(1) Sushruthi Krishna 19. Name the country that has six Deputy Prime
(2) Pankhuri Gidwani Ministers.
(3) Sushobita Kapoor (1) Nigeria (2) Nicaragua
(4) Priyadarshini Chatterjee (3) Maldives (4) Nepal

12. Who is the Chairperson of NITI Aayog? 20. In 2015, Prime Minister Mr. Narendra Modi
(1) Sh. Narendra Modi launched a new campaign, "Start up India,
(2) Ms. Sushmita Mahajan Stand up India". The campaign is aimed at:
(3) Ms. Sushma Swaraj (1) Promoting bank financing for start ups and
(4) Sh. Arun Jately offer incentives to boost entrepreneurship
and job creation.
13. Who was the Constitutional Advisor to the (2) Promoting free education to all children
Constituent Assembly in framing the Indian below the age of 14 years and assuring
Constitution? them jobs after obtaining higher education.
(1) Sir B.N. Rau (3) Making India Digital
(2) Dr. B.R. Ambedkar (4) Promoting Swatch Bharat
(3) Pandit Jawaharlal Nehru
(4) Dr. Rajendra Prasad 21. Which State has the largest number of foreign
students in India?
14. The largest diaspora in the world is from: (1) Karnataka (2) Maharashtra
(1) South Africa (2) India (3) Uttar Pradesh (4) Haryana
(3) Mexico (4) China
22. Parliament of India consists of:
15. Who is the President of Sri Lanka? (1) President, House of the People and Council
(1) Jayavardane of States
(2) Ranil Wickramasinghe (2) House of the People and Council of States
(3) Maithripala Sirisena (3) Council of States
(4) K. Sripavan (4) House of the People

Previous Years
Page 194 CLAT & AILET Papers
23. At the Asian Indoor Athletics Championships 30. The direction to hold floor test to prove majority
held at Doha in February, 2016, who among in the Legislative Assembly of Uttarakhand ,
women won the gold medal in Long Jump? to be held on 10th May, 2016 has been given
(1) Irina Ektova of Khazagstan on 6th May, 2016 , by :
(2) Prajusha of India (1) The Supreme Court of India
(3) Mayooka Johny of India (2) The Governor of Uttarakhand
(4) Olga Rypakova of Khazakstan (3) The President of India
(4) The High Court of Uttarakhand
24. Who was conferred the Rajiv Gandhi Khel
Ratna award during August, 2015? 31. Who lit the torch from the sun's rays reflected
(1) Sania Mirza (2) Virat Kohli in a parabolic mirror during the Olympic flame
(3) Dhoni (4) Saina Nehwal lighting ceremony for the Rio 2016 Olympic
Games at the site of ancient Olympia in
25. Who is the author of "Crime & Punishment"? Greece on 22nd April, 2016?
(1) Geoffrey Chaucer (1) Ronaldo
(2) Karl Marx (2) Joanie Laureh aka Chyna
(3) Harold Joseph Laski (3) Katerina Lehou
(4) Fyodor Dostoevsky (4) Katerina Kaif
26. Who is the Union Finance Minister of India? 32. Prime Minister, Mr. Narendra Modi, in March,
2016, launched an ambitious programme
(1) Ms. Sushma Swaraj "Setu Bharatam". The programme is aimed
(2) Mr. D.V. Sadananda Gowda at:
(3) Mr. Arun Jaitly (1) Providing travel concession to senior
(4) Mr. Rajnath Singh citizens throughout India.
27. The Constitution (One Hundredth Amendment) (2) Making all National Highways railway level
Act, 2015 was enacted to give effect to: crossing free by 2019.
(1) the acquiring of territories by India from (3) Linking all major cities and towns by rail,
Bangladesh. air and road by 2019.
(2) transfer of certain territories by India to (4) Providing travel concession to women and
Bangladesh. children throughout India.
(3) National Judicial Appointment
Commission 33. The highest peace time gallantry award Ashok
(4) the transfer of certain territories by India Chakra was awarded posthumously during
to Bangladesh and transfer of certain 2016 to:
territories from Bangladesh to India (1) Mahendra Singh
(2) Rajesh Atra
28. Which day is celebrated as "World Consumer (3) Jagdish Chand
Rights Day"? (4) Mohan Nath Goswami
(1) 5th June (2) 15th July
(3) 15th March (4) 5th September 34. Who is the Chairperson of National Human
Rights Commission?
29. Which country is the first developed nation to (1) Justice B.S. Chauhan
default on debt of IMF? (2) Justice Balakrishnan
(1) Japan (2) UK (3) Justice H.L. Dattu
(3) USA (4) Greece (4) Justice D.K. Jain

Previous Years
CLAT & AILET Papers Page 195
35. Who is the Chairman of National Green 42. Mars is also known as the:
Tribunal? (1) Red Planet (2) Brown Planet
(1) Justice Bhandhari (3) Blue planet (4) Green Planet
(2) Justice Balakrishna Reddy
(3) Justice R.C. Lahoti 43. How many Indian Universities/Institutes figure
(4) Justice Swatanter Kumar among the top 200 in BRICS rankings?
(1) 26 (2) 6 (3) 20 (4) 16
36. The Parliament of which country became first
Parliament in the world to run entirely on Solar 44. Which State of India was declared in January,
Power? 2016 as the "first organic farming State of
(1) Japan (2) UK India"?
(3) India (4) Pakistan (1) Haryana (2) Kerala
(3) Sikkim (4) Punjab
37. The "Paris Agreement" was adopted in the
twenty first session of Conference of Parties 45. The maximum number of 'Smokeless Villages'
in the month of: are found in the State of:
(1) January, 2016 (2) December, 2015 (1) Telangana (2) Punjab
(3) November, 2015 (4) February, 2016 (3) Karnataka (4) Rajasthan

38. In an attempt to curb black money, the 46. Who has written the book "Indomitable
Government has made PAN mandatory for Spirit"?
all financial transactions exceeding Rupees: (1) Ms. Arundhati Roy
(1) 3 lakhs (2) 4 lakhs (2) Dr. A.P.J. Abdul Kalam
(3) 2 lakhs (4) 1 lakhs (3) Justice Krishna Iyer
(4) Sh. Narendra Modi
39. Name the Indian Gymnast who has become
the first female Indian Gymnast to qualify for 47. India signed the "Paris Agreement on Climate
the Olympics. Change" in April, 2016 at:
(1) Bisweshwari Nandi (1) New York (2) Geneva
(2) Rupinder Kaur (3) New Delhi (4) Paris
(3) Dipa Karmakar
(4) Anjubala 48. ___________ is the fastest planet to revolve
around the Sun.
40. In case of death of both the President and (1) Venus (2) Mercury
Vice-President of India, who shall act as the (3) Mars (4) Jupiter
President of India?
(1) Prime Minister 49. In case the President of India wants to resign,
(2) Chief Justice of India he shall address his resignation to the:
(3) Attorney General of India (1) Vice-President
(4) The Parliament will nominate from among (2) Chief Justice of India
its Members (3) Chief Election Commissioner
(4) Prime Minister
41. Punjabi has become the ______________
most common language in the Parliament of 50. Who has scored the fastest century in a Test
Canada. Cricket match?
(1) fifth (2) fourth (1) Vivian Richards, West Indies
(3) second (4) third (2) A B de Villiers, South Africa
(3) Misbah Ul Haq, Pakistan
(4) Brendon McCullum, New Zealand

Previous Years
Page 196 CLAT & AILET Papers
ELEMENTARY MATHEMATICS 7. The angle subtended by the Minor segment
( NUMERICAL ABILITY ) of a circle at the center is __________ the
angle subtended by the Major segment at the
Direction for Questions 1-20: Choose the most center of the circle.
appropriate option: (1) greater than (2) lesser than
(3) not related to (4) equal to
1. The value of a machine depreciates every year
at the rate of 10% on its value at the beginning 8. Two consecutive even positive integers, sum
of that year. If the present value of the machine of the squares of which is 1060 are:
is Rs. 729, its worth three years ago was: (1) 20 and 22 (2) 12 and 14
(1) Rs. 1,000 (2) Rs. 947.70 (3) 22 and 24 (4) 16 and 18
(3) Rs. 750.87 (4) Rs. 800
9. 60 kg of an alloy X is mixed with 100 kg of an
2. The mean of 72 items was found to be 63. If alloy Y. If alloy X has lead and tin in the ratio
two of the items were mis-recorded as 27 and of 3 : 2 and alloy Y has tin and copper in the
9 instead of 72 and 90 respectively, find the ratio of 1 : 4, then the amount of tin in the
correct mean. new alloy is
(1) 80 kgs. (2) 36 kgs.
(1) 65.75 (2) 64.75
(3) 44 kgs. (4) 53 kgs.
(3) 66.25 (4) 64.25
10. 12 defective pens are accidentally mixed with
3. A circular park, 42 m in diameter, has a path
132 good ones. It is not possible to just look
3.5 m wide running around it on the outside.
at a pen and tell whether or not it is defective.
Find the cost of gravelling the path at Rs. 4 One pen is taken out at random from this lot.
per m2 Determine the probability that the pen taken
(1) Rs. 2002 (2) Rs. 1652 out is a good one.
(3) Rs. 2048 (4) Rs. 1672
9 11 10 7
(1) (2) (3) (4)
4. A man buys Rs. 20 shares paying 9% 12 12 12 12
dividend. The man expects to have an interest
of 12% on his money. The market value of 11. The number of 'three digit numbers' which are
multiples of 9 are:
each share is:
(1) 99 (2) 101 (3) 98 (4) 100
(1) Rs. 12 (2) Rs. 18
(3) Rs. 21 (4) Rs. 15
12. The angles between the hands of a clock when
the time is 4 : 25 am is:
5. A train which is moving at an average speed
1 1
of 40 kmph, reaches its destination on time. (1) 13 degrees (2) 12 degrees
When its average speed reduces to 35 kmph, 2 2
then it reaches its destination 15 minutes late. 1 1
(3) 14 degrees (4) 17 degrees
The distance travelled by the train is: 2 2
(1) 30 kms (2) 80 kms
(3) 40 kms (4) 70 kms 13. If a boy is standing at the top of the tower and
another boy is at the ground at some distance
6. If the product of zeroes of the polynomial ax2 from the foot of the tower, then the angle of
– 6x – 6 is 4, find the value of 'a' elevation and depression between the boys
when both look at each other will be:
1 3 3 1 (1) Angle of depression will be greater
(1) (2) − (3) (4) −
2 2 2 2 (2) Equal
(3) Cannot be predicted for relation
(4) Angle of elevation will be greater

Previous Years
CLAT & AILET Papers Page 197
14. A man earns Rs. 20 on the first day and 20. A library has an average of 510 visitors on
spends Rs. 15 on the next day. He again earns Sundays and 240 on other days. What is the
Rs. 20 on the third day and spends Rs. 15 on average number of visitors per day in the
the fourth day. If he continues to save in this month of June beginning with a Sunday?
way, how soon will he have Rs. 60 in hand? (1) 280 (2) 250
(1) on 27th day (2) on 12th day (3) 276 (4) 285
(3) on 24th day (4) on 17th day
LEGAL APTITUDE
15. The traffic lights at three different signal points
change after every 45 seconds, 75 seconds This section consists of fifty (50) questions. Follow
and 90 seconds respectively. If all change the instructions carefully and choose the most
simultaneously at 7:20:15 hours, then they appropriate option:
will change again simultaneously at
(1) 7:27:45 hours (2) 7:27:30 hours The question numbers 1-35 in this section consist
(3) 7:27:50 hours (4) 7:28:00 hours of legal proposition(s)/ principle(s) (hereinafter
referred to as 'principle') and facts. Such principles
16. What is the sum of all the natural numbers may or may not be true in the real and legal sense,
from 1 to 100? yet you have to conclusively assume them to be
(1) 5052 (2) 5000 true for the purposes of this Section. In other words,
(3) 6000 (4) 5050 in answering the following questions, you must not
rely on any principle except the principles those are
17. A part of monthly hostel charges is fixed and given herein below for every question. Further, you
the remaining depends on the number of days must not assume any facts other than those stated
one has taken food in the mess. When a in the question. The objective of this section is to
student A takes food for 20 days, she has to test your interest towards study of law, research
pay Rs. 1000 as hostel charges whereas a aptitude and problem solving ability, even if the "most
student B, who takes food for 26 days, pays reasonable conclusion" arrived at may be absurd or
Rs. 1180 as hostel charges. Find the fixed unacceptable for any other reason.
charges and the cost of food per day.
(1) 400, 40 (2) 400, 30 To answer a question, the given principle is to be
(3) 300, 30 (4) 200, 20 applied to the given facts so as to arrive at most
reasonable conclusion. Only one of the options, i.e.,
18. A shop gives 15% discount on the purchase (A), (B), (C), or (D) is the most reasonable conclusion.
of a T.V. If paid for in cash immediately, a
further discount of 12% is given. If the marked 1. Principles: An independent contractor is one
price is Rs. 15,000, what is the price of the who is employed to do some work of his
T.V if cash purchase is made? employer. He is engaged under a contract for
(1) Rs. 10,950 (2) Rs. 11,220 services. He undertakes to produce a given
(3) Rs. 11,475 (4) Rs. 12,750 result, and in the actual execution of the work,
he is not under the direct control or following
19. A patient in a hospital is given soup daily in a directions of his employer. He may use his
cylindrical bowl of diameter 7 cm. If the bowl own discretion in execution of the work
is filled with soup to a height of 4 cm, how assigned.
much soup the hospital has to prepare daily In general, an employer is not liable for the
to serve 250 patients? torts (wrongful acts) of his independent
(1) 39.5 litres (2) 38 litres contractor. But, the employer may be held
(3) 38.5 litres (4) 40 litres liable if he directs him to do some careless
acts.

Previous Years
Page 198 CLAT & AILET Papers
Facts: Ramesh hired a taxi-cab to go to Delhi was little intoxicated. The cab met with an
Airport. As he started late from his home, he accident and lady got injured.
kept on urging the taxi-driver to drive at a high
speed and driver followed the directions; and (1) Lady is not entitled to claim compensation
ultimately due to high speed an accident took as she had knowledge of the risk.
place causing injuries to a person. (2) Lady can refuse to pay the fare as she
had suffered injuries.
(1) Ramesh would not be held liable for (3) Driver can take the plea that he was lightly
damages because the driver was an intoxicated.
independent contractor and not his (4) Lady is entitled to claim compensation as
servant. she only knew about risk and there was
(2) Ramesh would not be held liable for no assumption of risk.
damages because Ramesh did not know
the consequences of such rash driving. 4. Principle:
(3) Ramesh would be held liable for damages 1. Wagering agreements are void.
as he exercised the control by giving 2. Collateral agreements to wagering
directions to the driver. contracts are valid.
(4) Ramesh would not be liable as car was
not owned by him. Facts: XYZ Bank lends Rs. 40, 000 to Sabu
in order to enable him to award as prize to
2. Principle: Where a person lawfully does Randeep who is the winner of horse race. Later
anything for another person, or delivers Sabu refuses to pay the prize stating that
anything to him, not intending to do so or to horse racing is wagering agreement. Can XYZ
provide gratuitously, and such other person Bank recover money from Sabu?
takes the benefit of that; the latter is bound
to compensate the former for something done (1) Bank can recover money from Sabu so
or thing provided, or to restore, the thing so that payment of prize money can be made
delivered. to Randeep.
(2) No, as it is a wagering contract.
Facts: Trader 'A' delivers certain eatables at
(3) Yes, as it is only a collateral agreement
B's house by mistake. 'B' consumed the
to horse racing and therefore the bank can
eatables without asking anything. Which of
recover the money from Sabu.
the following derivations is correct?
(4) Horse racing is illegal and therefore XYZ
Bank cannot recover anything from Sabu.
(1) It is the discretion of 'B' to make payment
to 'A'
5. Principles: A servant is one who is employed
(2) 'B' is bound to pay 'A' for the eatables.
to do some work for his employer (master).
(3) 'B' can be made liable to pay for the
eatables, only if 'A' establishes an express He is engaged under a contract of service.
contract between 'A' and 'B'. He works directly under the control and
(4) 'B' is not bound to pay 'A' for the eatables. directions of his master.
In general, the master is vicariously liable for
3. Principle: Consent is a good defence for civil those torts (wrongful acts) of his servant which
action in tort. But consent must include both are done by the servant in the course of his
knowledge of risk and assumption of risk, i.e., employment.
readiness to bear harm.
Facts: 'M' appointed 'D' exclusively for the
Facts: A lady passenger was aware that the
purpose of driving his tourist vehicle. 'M' also
driver of the cab, in which she opted to travel
appointed 'C' exclusively for the purpose of

Previous Years
CLAT & AILET Papers Page 199
performing the work of a conductor for the 8. Principle: Killing is not murder if the offender,
tourist vehicle. During one trip, at the end of whilst deprived of the power of self-control by
the journey, 'C', while 'D' was not on the driver's intense and sudden provocation, causes the
seat, and apparently for the purpose of turning death of the person who gave the provocation.
the vehicle in the right direction for the next
journey, drove it through the street at high Facts: 'A', a man found his girl friend sleeping,
speed, and negligently injured 'P'. in her own bed room, with another man named
'B'. 'A' did not do anything but went to his
(1) 'M' could be made liable for the act of 'C', home, picked a gun and cartridges, returned
as 'C' was employed under a contract of to the girl friend's bed room with loaded gun
service. but found the place empty. After fifteen days
(2) 'M' could not be made liable for the act of he saw his girl friend dining in a restaurant.
'C', as his (C's) act of driving the vehicle Without waiting for even a second, 'A' fired
was not in the course of his employment. five bullets at his girl friend who died on the
(3) 'M' could be made liable for the act of 'C', spot.
as his (C's) act of driving the vehicle was
within his scope of employment.
(1) 'A' could have killed both 'B' and his girl
(4) 'M' is not liable as he was not present at
friend.
the time of accident.
(2) 'A' killed his girl friend under intense and
sudden provocation.
6. Principle: Sale of liquor is illegal. All
(3) 'A' did not kill his girl friend under intense
agreements relating to prohibited items do
and sudden provocation.
not exist in the eyes of law.
(4) 'A' could have killed 'B' instead of his girl
Facts: 'A' entered into an agreement with 'B' friend.
for the sale of liquor. 'A' failed to supply the
agreed quantity of liquor to 'B'. 9. Principle: Nothing is an offence, which is
done by accident or misfortune, and without
(1) 'A' can bring a legal action against 'B'. any criminal intention or knowledge in the
(2) 'B' can bring a legal action against 'B'. doing of a lawful act in a lawful manner by
(3) 'B' cannot bring any legal action against lawful means and with proper care and
'A'. caution.
(4) 'A' and 'B' can initiate appropriate legal
proceeding against each other. Facts: 'A' takes up a gun, not knowing whether
it is loaded or not, points it playfully at 'B' and
7. Principle: Mere silence as to facts likely to pulls the trigger. Consequently, 'B' falls dead.
affect the decision of a person to enter into a
contract is not fraud. (1) B's death is accidental, as 'A' was just
pointing the gun playfully at 'B'.
Facts: 'A' sells to 'B' (A's daughter who is (2) B's death is not accidental, as there was
minor) a horse which 'A' knows to be unsound. want of proper care and caution on the
'A' says nothing to 'B' about the unsoundness part of 'A'.
of the horse. (3) B's death is accidental, as 'A' did not have
the knowledge that the gun is loaded.
(1) 'A' has not committed fraud. (4) B's death is accidental, as 'A' had no
(2) 'A' has committed fraud. intention to kill 'B'.
(3) 'B' can take plea of fraud because she is
minor.
(4) There cannot be a contract between a
father and daughter.
Previous Years
Page 200 CLAT & AILET Papers
10. Principle: Causing of an effect partly by an 13. Principle: Law never enforces an impossible
act and partly by an omission is an offence. promise.

Facts: 'A' confined her daughter 'D' in a room. Facts: 'A' made a promise to 'B' to discover
'A' also did not provide any food to her daughter treasure by magic.
'D'. Consequently, 'D' died of starvation.
(1) Law will enforce the promise only at the
(1) 'A' committed the offence of not providing option of 'B'.
food to 'D'. (2) Law will not enforce the promise
(2) 'A' committed no offence. (3) Law will enforce the promise.
(3) 'A' committed the offence of confining 'D'. (4) Law will enforce the promise only at the
(4) 'A' committed the offence of causing death option of 'A'.
of 'D'.
14. Principle: Whoever by words or writing
11. Principle: Whoever takes away any moveable conveys to others any imputation concerning
thing from the land of any person without that any person's reputation is said to defame that
person's consent, he is said to have person.
committed theft.
Facts: During a marriage ceremony, 'A'
Facts: During his visit to the house of 'C', 'A' circulated a pamphlet saying that 'S', sister
asked 'B', the son of 'C', to accompany 'A' to of the bride, is a thief, she has stolen the
the forest. Neither 'A' nor 'B' informed 'C' in shoes of the bridegroom.
this regard. 'B' accompanied 'A' to the forest.
(1) A' has defamed 'S'.
(1) 'A' has committed theft. (2) 'A' did not defame 'S' as he never intended
(2) 'A' has committed theft as soon as he it.
entered the house of 'C'. (3) A' has defamed the bridegroom.
(3) 'A' has not committed theft. (4) 'A' defamed the bride.
(4) 'A' has not committed theft till 'B' did not
accompany him. 15. Principle: The communication of a proposal
is complete when it comes to the knowledge
12. Principle: An agreement may be entered into of the person to whom it is made.
orally or in writing, or by conduct.
Facts: 'A' sent a letter making a proposal to
Facts: 'A' went to the shop of 'B' and picked a 'B' to purchase the house of 'B'.
toothbrush and gave a cheque of Rupees
twenty to 'B' and left the shop. (1) The communication of proposal is
complete when B's wife handed over the
(1) There was an agreement between 'A' and letter to 'B'.
'B'. (2) The communication of proposal is
(2) 'A' did not enter into an agreement with complete when A sent the letter.
'B'. (3) The communication of proposal is
(3) Payment of toothbrush cannot be made complete when 'B' reads the letter.
through a cheque. (4) The communication of proposal is
(4) 'A' should have carried a currency note of complete when B's wife received it.
Rupees twenty to make the payment.

Previous Years
CLAT & AILET Papers Page 201
16. Principle: A person is said to have (2) ‘A’ can never make a contract.
committed assault when an apprehension is (3) ‘A’ can make a contract when normal.
caused in the mind of a person that he is (4) ‘A’ can make a contract at any time
about to use physical force against his body. whenever he pleases.

Facts: 'A' abuses 'B' while he was sitting in a 19. Principle: In cases where there is an
moving train, by aggressively shaking his fists infringement of legal right even without any
when 'B' was standing on the railway platform actual loss or damage, the person whose right
at a distance. is infringed has a cause of action.

(1) 'A' has caused fear of assault in the mind Facts: 'P' was wrongfully prevented by the
of 'B'. Returning Officer from exercising his vote in
(2) ‘A’ has not committed assault against 'B'. an assembly election. However, the candidate
(3) 'A' has caused apprehension of assault for whom he wanted to caste his vote won
in the mind of 'B'. the election. Still, he ('P') brought an action
(4) 'A' has committed assault against 'B'. claiming damages. Which of the following
derivations is correct?
17. Principle: Letters or words not describing
quality of things can be registered as a trade (1) 'P' would not succeed in his action, as
mark. the candidate for whom he wanted to give
his vote won the election.
Facts: Ram made an application for (2) 'P' would not succeed in his action, as he
registration of alphabet 'B' written in a fancy did not suffer any loss in that election.
style as trade mark to be applied on packets (3) 'P' would succeed in his action, as it is
and cartons of shoes manufactured by him. mandatory to cast vote.
(4) 'P' would succeed in his action, as he was
(1) The alphabet 'B' can be registered as trade wrongfully prevented from exercising his
mark. legal right of voting in that election.
(2) The alphabet 'B' can be registered as trade
mark because it describes the quality of 20. Principle: Acceptance of proposal must be
things. the exact mirror image of the proposal.
(3) The alphabet 'B' cannot be registered as
trade mark because it is an English letter. Facts: 'A' made a proposal to 'B' to sell a
(4) The alphabet 'B' cannot be registered as chair for Rs. 500. 'B' expressed his desire to
trade mark because it belongs to buy the said chair for Rs. 400.
humanity.
(1) ‘B’ has not accepted the proposal of ‘A’.
18. Principle: A person, who is usually of (2) It is not clear whether ‘A’ made a proposal
unsound mind, but occasionally normal, may to ‘B’.
make a contract when he is not of unsound (3) It is not clear whether ‘B’ has accepted
mind. the proposal of ‘A’ or not.
(4) ‘B’ has accepted the proposal of ‘A’.
Facts: 'A' generally remains in the state of
unsound mind and rarely becomes capable 21. Principle: An agreement without free consent
of understanding the things. can be enforced only at the option of the party
whose consent was not free.
(1) 'A' can make a contract only for his own
benefit.

Previous Years
Page 202 CLAT & AILET Papers
Facts: A obtains the consent of 'B' to enter show that he was sufficiently mature to
into an agreement by putting a gun on the understand the nature and consequences
head of B's girl friend. of his conduct.
(3) Himesh would be protected under the
(1) 'B' cannot enforce the agreement. principle stated above because his acts
(2) 'B' can enforce the agreement. show that he was not sufficiently mature
(3) Neither 'A' nor 'B' can enforce the to understand the nature and
agreement. consequences of his conduct.
(4) ‘A’ can enforce the agreement. (4) Himesh would not be protected under the
principle stated above because,
22. Principle: Existence of all the alleged facts irrespective of the age, stealing is an
is relevant, whether they occurred at the offence.
same time and place or at different times and
places. 24. Principle: Import means bringing some
consignment into India from a foreign country.
Facts: 'A', a citizen of England, is accused of
committing murder of 'B' in India by taking Facts: A consignment from Sri Lanka entered
part in a conspiracy hatched in England. the territorial waters of India. However, this
consignment never crossed the Indian custom
(1) The facts that 'A' citizen of England is barrier nor did it enter into the stream of
accused of commission of murder in India commerce in India.
and of conspiracy hatched in England are
relevant facts. (1) The consignment will only be imported into
(2) 'A' citizen of England cannot be tried in India when it enters into the stream of
India. commerce in India.
(3) Only the fact that 'A' is accused of (2) The consignment will only be imported into
conspiracy hatched in England is relevant. India when it crosses the Indian custom
(4) Only the fact that 'A' citizen of England is barrier.
accused of committing murder of 'B' in (3) The consignment was imported into India.
India is relevant. (4) The consignment was not imported into
India.
23. Principle: Nothing is an offence which is
done by a child under twelve years of age, 25. Principle: A condition to a contract can also
who has not attained sufficient maturity of be complied with after the happening of the
understanding to judge the nature and event to which such a condition is attached.
consequences of his conduct on that
occasion. Facts: 'A' promises to pay Rs. 5000 to 'B' on
the condition that he shall marry with the
Facts: Himesh, 11 years old boy, picks up a consent of 'C', 'D' and 'E'. 'B' marries without
gold ring worth Rs 5000/- lying on a table in the consent of 'C', 'D' and 'E', but obtains their
his friend's house and immediately sells it for consent after the marriage.
Rs 2000/-, and misappropriates the money.
(1) 'B' has fulfilled the condition.
(1) Himesh would be protected under the (2) 'B' has not fulfilled the condition.
principle stated above because he is (3) The condition is illegal.
below 12 years of age. (4) 'B's marriage is not valid.
(2) Himesh would not be protected under the
principle stated above because his acts

Previous Years
CLAT & AILET Papers Page 203
26. Principle: Intentional application of force to (3) 'A' is not guilty because when he took the
another person is actionable in law. property, he believed in good faith that it
belonged to him.
Facts: 'P' and 'D' are unknown to each other. (4) 'A' is guilty of an offence of
When 'P' is about to sit on a chair, 'D' misappropriation.
intentionally pulls it away as a result of which
'P' falls on to the floor and is injured. 29. Principles:
• A person is said to abet the doing of a thing
(1) 'D' is liable as he intentionally caused injury when he instigates any other person to do
to P. that thing.
(2) 'D' is not liable as the injury is not directly • Mere acquiescence, however, does not
caused. amount to instigation.
(3) 'D' is not liable as 'P' is not seriously injured.
(4) 'D' is not liable as such jokes are common Facts: 'A' says to 'B': I am going to kill 'C'."
in the society. And, 'B' replies: "Do as you wish and take
the consequences"; whereafter 'A' kills 'C'.
27. Principle: Whoever does not arrest the killer
and report the matter to the concerned (1) 'B' has not abetted 'A' to kill 'C'.
authorities commits an offence. (2) 'B' abetted 'A' to kill 'C'.
(3) B' has abetted 'A' by conspiracy .
Facts: 'A', a woman, sees 'B' , another woman, (4) 'B' is jointly liable with 'A' for killing 'C'.
killing a third woman 'C'. 'A' neither attempted
to arrest 'B' nor informed the concerned 30. Principle: False imprisonment is a tort
authorities. (wrong) which means the total restraint of a
person's liberty without lawful justification.
(1) 'B' has committed an offence.
(2) 'A' has committed an offence. Facts: A part of a public road had been closed
(3) 'B' has not committed an offence. for spectators of a boat race. 'P' wanted to
(4) 'A' has not committed an offence. enter but he was prevented by 'D' and other
policemen because he had not paid the
28. Principle: One who dishonestly mis- admission fee. 'P' was able to enter the
appropriates or converts to his own use or enclosure by other means but was unable to
sells any movable property belonging to go where he wanted to go. The policemen
another, is guilty of the offence of refused access to where he wanted to go but
misappropriation. allowed him to remain where he was or to go
back. 'P' remained within the enclosure and
Facts: 'A' takes property belonging to 'Z' out refused to leave. Subsequently, 'P' sued 'D'
of Z's possession, in good faith, believing when for false imprisonment.
he takes it, that the property belongs to
himself. Subsequently, 'A', on discovering his (1) 'D' could not be made liable for false
mistake, without disclosing the actual facts, imprisonment as he has not touched him.
dishonestly sells the property to a stranger. (2) 'D' could not be made liable for false
imprisonment, as he did not totally restrict
(1) 'A' is not guilty as the property can be P's movements.
recovered from the stranger. (3) It was a case of false imprisonment, but
(2) 'A' may be guilty of theft but not for 'D' could not be made liable for it.
misappropriation. (4) 'D' could be made liable for false
imprisonment, as he did restrict P's
movements.

Previous Years
Page 204 CLAT & AILET Papers
31. Principle: Law does not penalise for wrongs 34. Principle: There are certain acts which,
which are of trivial nature. though harmful, are not wrongful in law;
therefore, do not give legal right to bring action
Facts: In the course of a discussion, 'A' threw in law, to the person who suffers from such
a file of papers at the table which touched the acts.
hands of 'B'.
(1) 'A' is liable for his act, as the file touched Facts: 'Prakash' has a rice mill. His neighbour,
'B's hand. Shanti, sets up another rice mill and offers a
(2) 'A' is liable for his act, as it assaulted 'B'. tough competition to Prakash. As a
(3) 'A' is not liable for his act, as it was of consequence, Prakash's profits fall down. He
trivial nature. brings a suit against Shanti for damages.
(4) 'A' is liable for insulting 'B'.
(1) Prakash can succeed in his claim for
32. Principle: Consent is a good defence in a damages, as it is a case of damage as a
civil action for tort but the act should be the result of infringement of his legal right.
same for which consent was given. (2) Prakash can succeed in his claim as it is
a case of actual damages.
Fact: 'B' was formally invited by 'A' to his (3) Prakash may succeed in his claim for
house. 'B' after sitting for some time in drawing damages, as it is a case of loss to his
room, moved to the bed room of the house. business.
'A' sued 'B' for trespass. (4) Prakash cannot succeed in his claim for
damages, as it is a case of damage
(1) 'B' has interfered with privacy of 'A' without infringement of any legal right.
(2) 'B' has offended 'A' by moving to bed room.
(3) 'B' has committed no trespass as he 35. Principle: Defamation is the publication of a
entered the house with 'A's consent. statement which tends to lower reputation of
(4) 'B' has committed trespass as there was a person in the estimation of other members
no consent of 'A' for entry in the Bed room. of the society generally.

33. Principle: Copyright law protects only work. Facts: 'A' writes a highly offensive and
'Work' means cinematographic film but does derogatory letter about 'B', and sends it
not include performance by an actor in a directly to 'B' in a sealed cover.
cinematographic film.
(1) 'A' is liable to 'B' for defamation, as the
Facts: Alia Bhatt acted in a movie. letter is highly offensive and derogatory
and is directly sent to 'B'.
(1) The acting of Alia Bhatt can be protected (2) 'A' is liable to 'B' for defamation, as the
under copyright law as professional work. letter is highly offensive and derogatory.
(2) The acting of Alia Bhatt can be protected (3) 'A' is liable to 'B' for defamation, as it has
as film producer's work. hurt his (B's) self-esteem.
(3) The acting of Alia Bhatt can be protected (4) A' is not liable to 'B' for defamation, since
under copyright law only as an artistic there is no publication to any other person
work. in whose estimation the reputation of 'B'
(4) The acting of Alia Bhatt cannot be could be brought down.
protected under copyright law.

Previous Years
CLAT & AILET Papers Page 205
Direction for Questions 36 to 50: Choose the most 42. The Supreme Court of India has struck down
appropriate option: the Constitution (Ninety-ninth Amendment)
Act, 2014 as unconstitutional. It is related to-
36. The object of which one of the following writs
is to prevent a person to hold public office (1) Religious Rights
which he is not legally entitled to hold? (2) National Judicial Appointment Commission
(1) Quo warranto (2) Certiorari (3) Land Exchange between India and
(3) Prohibition (4) Mandamus Bangladesh
(4) Jallikattu (Bull Fighting)
37. Which among the following was described by
Dr. B. R. Ambedkar as the "heart and soul of 43. 'lis pendens' means-
the Constitution of India"? (1) Awaited information.
(1) Freedom of Religion (2) Decision awaited.
(2) Right to Constitutional Remedies (3) A pending suit.
(3) Right to move throughout the territory of (4) On the basis of evidence.
India
(4) Right to equality 44. Under the Constitution of India restriction on
freedom of religion cannot be placed on the
38. 'persona non grata' means- ground of-
(1) Non-performance of promise. (1) Social justice (2) Public order
(2) Ungrateful person. (3) Health (4) Morality
(3) An unacceptable person
(4) Non-person 45. Which one of the following is not a Directive
Principle of State Policy under Part IV of the
39. 'alibi' means a plea by an accused person Constitution of India?
that he- (1) Promotion of International peace and
(1) was facing trial. security.
(2) underwent preventive detention. (2) Promotion of adult education.
(3) remained in judicial custody. (3) Organisation of village panchayats.
(4) was present elsewhere (4) Provision for just and humane conditions
of work and maternity relief.
40. If an authority is holding information about
another in a 'fiduciary capacity', the 46. As per law the minimum age for the marriage
information under the Right to Information Act, of a boy and a girl in India is-
2005 may not be obtainable. 'Fiduciary (1) 18 years in both cases
relationship' is based on: (2) 21 years and 18 years respectively
(1) Trust (2) Contract (3) 21 years in both cases
(3) Law (4) Authority (4) 18 years and 21 years respectively

41. 'audi alteram partem' means- 47. Under the Constitution of India 'Right to
(1) Following the substantive law. Pollution Free Environment' has emerged as
(2) A transferee cannot retransfer. a fundamental right from the right to-
(3) Giving opportunity of hearing of the other (1) Equality under Article 14
side. (2) Conserve culture under Article 29.
(4) Not connected to facts. (3) Freedom of movement under Article 19
(4) Life and personal liberty under Article 21

Previous Years
Page 206 CLAT & AILET Papers
48. 'obiter dicta' means- Direction for Questions 4-6 : Read the following
(1) Direction by a judge. information carefully and choose the most
(2) An opinion given by the court not necessary appropriate option:
for the decision.
(3) Judgment of a court in the case before it. In each question, there is a statement and two
(4) Basis of judicial decision. assumptions numbered as I and II. Read the
statement and find which of the given assumptions
49. Which among the following does not belong is implicit:
to the 'right to freedom of religion'? (A) If assumption I is implicit.
(1) Freedom of conscience and free (B) If assumption II is implicit.
profession, practice and propagation of (C) If neither assumption I nor Assumption II is
religion implicit.
(2) Freedom from payment of taxes for (D) If both Assumption I and Assumption II are
promotion of any particular religion implicit.
(3) Freedom from attending religious
instruction or religious worship in certain 4. Statement: To attend a convocation
educational institutions ceremony scheduled to be held on Thursday
(4) Freedom of speech and expression at GM University, Chennai, Mr X left for
Chennai on Tuesday by train.
50. Which Indian State has prescribed minimum
educational qualification for candidates Assumptions:
contesting panchayat polls? I. Mr X may reach home on Saturday
(1) Punjab (2) Gujarat II. Mr X may reach the University on
(3) Haryana (4) Kerala Wednesday

LOGICAL REASONING (1) Both Assumption I and Assumption II are


implicit.
Direction for Questions 1 - 3 : Read the following (2) Assumption II is implicit.
information carefully and choose the most (3) Neither assumption I nor Assumption II is
appropriate option: implicit.
(4) Assumption I is implicit.
A and B are good in driving bus and car. C and A are
good in driving car and scooter. C, D and E are good 5. Statement: In the State of Zuminisia, people
in driving scooter and tractor. E and C are good in prefer to travel by X airline instead of Y airline,
driving scooter and auto- rickshaw. D and B are good as X airline has advanced German security
in driving bus and tractor. system and 99% on time operations.

1. Who is good in driving scooter, tractor and Assumptions:


auto- rickshaw but not good in driving car? I. Airline X with advanced German security
(1) C (2) A (3) D (4) E system and record on time operation is
perceived better than airline Y.
2. Who is good in driving tractor, scooter, car II. Had advanced German security system
and auto-rickshaw but not bus? and on time performance record of Y airline
(1) D (2) B (3) C (4) A been implemented, it would have been
preferred over airline X.
3. Who is good in driving scooter, tractor and
bus?
(1) D (2) C (3) A (4) B

Previous Years
CLAT & AILET Papers Page 207
(1) Assumption I is implicit. an independent cause. The statements may be
(2) Neither assumption I nor Assumption II is independent causes without having any relationship.
implicit. Read both the statements and mark your answer
(3) Both Assumption I and Assumption II are as:
implicit.
(4) Assumption II is implicit. (A) If statement I is the cause and statement II is
its effect.
6. Statement: The next meeting of the executive (B) If statement II is the cause and statement I is
board of a company will be held after six its effect.
months. (C) If both statements I and II are independent
causes.
Assumptions: (D) If both statements I and II are effects of
I. Existing executive board will be dissolved independent causes.
before six months
II. The company will remain in function after 9. Statement I: In last two years, there is a
six months considerable reduction in cancellation of
flights due to fog in North India.
(1) Assumption II is implicit.
(2) Assumption I is implicit. Statement II: In last two years, there is a
(3) Neither assumption I nor Assumption II is considerable improvement in passenger
implicit. amenities on all airports of North India.
(4) Both Assumption I and Assumption II are
implicit. (1) Statement II is the cause and statement I
is its effect.
Direction for Questions 7-8: Read the following (2) Both statements I and II are effects of
information carefully and choose the most independent causes.
appropriate option: (3) Both statements I and II are independent
causes.
In a joint family, A, B, C, D, E, and F are the (4) Statement I is the cause and statement II
members. B is the son of C. A and C are husband is its effect.
and wife. C is not the mother of B. E is the brother of
C. D is the daughter of A and F is the brother of B. 10. Statement I: The Government, by legislation
has decided to make all public information
7. Which of the following is a pair of females in available to general public.
the family?
(1) AD (2) DF (3) BD (4) AE Statement II: Before passing of legislation,
general public did not have access to public
8. Who is the mother of B? information.
(1) F (2) D (3) A (4) E
(1) Both statements I and II are effects of
Direction for Questions 9-10 : Read the following independent causes.
information carefully and choose the most (2) Both statements I and II are independent
appropriate option: causes.
(3) Statement II is the cause and statement I
In each of the questions, two statements are is its effect.
numbered as I and II. There may be a cause and (4) Statement I is the cause and statement II
effect relationship between the two statements. The is its effect.
statements may be the effect of the same cause or

Previous Years
Page 208 CLAT & AILET Papers
Direction for Questions 11-40: Choose the most 17. In a bag, there are some diamonds. In another
appropriate option: bag, there are one fourth the number more
than the number of diamonds in the first bag.
11. If it is false that 'Men always pray to God', If the difference in the number of diamonds in
then which one of the following statements is the first and second bag is 3, how many
true? diamonds are there in the first bag?
(1) Men always pray to God. (1) 10 (2) 16 (3) 12 (4) 8
(2) Some men pray to God.
(3) All men pray to God. 18. If Oceans are Deserts, then Waves are:
(4) Men seldom pray to God. (1) Powerful (2) Ripples
(3) Water (4) Sand Dunes
12. Examine the series and identify the missing
number: 19. You are given a 60-inch long ribbon, and you
46, 44, 40, 38, 34, â?¦ are instructed to cut 60 1-inch long strips out
(1) 30 (2) 28 (3) 32 (4) 26 of this ribbon. The time taken to cut one strip
is one second. So how long will it take to cut
13. If it is false that 'Animals are seldom 60 strips?
aggressive', then which one of the following (1) 1 minute and 1 second
statements conveys the same meaning? (2) 30 seconds
(1) Sometimes animals are aggressive. (3) 1 minute
(2) All animals are aggressive. (4) 59 seconds
(3) At least one animal is aggressive.
(4) All animals are always aggressive. 20. Which word in the following group DOES NOT
belong to the others?
14. Examine the following numbers and identify PROSPER, EXCITE, THRIVE, FLOURISH
the next number: (1) Thrive (2) Prosper(3) Flourish (4) Excite
45; 43; 40; 36; 31; 25; ....
(1) 17 (2) 29 (3) 18 (4) 23 21. Identify the statement which cannot be true.
(1) All bachelors are faithful to their wives.
15. In a class, student X has 8th position from (2) Black body radiation is a physical
the top and 84th from the bottom. How many phenomenon.
students are there in the class? (3) Snow is white.
(1) 88 (2) 96 (3) 98 (4) 91 (4) Every natural number has a successor.

16. Four statements are given below. Group two 22. If it is false that 'There is at least one
of them in such a way that one is logically octogenarian in the room', then which one of
incorrect and the other is verifiable as a matter the following is probable?
of fact (1) One person in the room is not an
(a) The Sun does not rise in the East. octogenarian.
(b) A straight line is the shortest distance (2) All those who are in the room are
between any two points. octogenarians.
(c) Every circle has a centre. (3) No one in the room is an octogenarian.
(d) The maximum duration of a total solar (4) Some people in the room are not
eclipse is about 7.5 minutes. octogenarians.

(1) [a, c] (2) [c, d] (3) [a, d] (4) [a, b]

Previous Years
CLAT & AILET Papers Page 209
23. If it is true that 'All humans are imperfect', 28. Identify the argument which cannot be
then which one of the following is necessarily accepted.
true? (1) All rainy days are wet days. Therefore, all
(1) Every human is imperfect. wet days are rainy days.
(2) Some humans are not imperfect. (2) All unmarried women are spinsters.
(3) All imperfect beings are humans. Therefore, all spinsters are women.
(4) No humans are imperfect. (3) All equilateral triangles are equiangular.
Therefore, all equiangular triangles are
24. In certain code MAIL is written as ZNVY then equilateral.
how will FILM be written in that code? (4) All wives are married. Therefore, all married
(1) MLIF (2) SVYZ (3) NORY (4) XLMP people are wives.

25. Who among the following is the odd one in 29. This question consists of a related pair of
the following group of persons? words, followed by four pairs of words. Choose
the pair that best represents a similar
Chief Justice of India, Attorney General of relationship as the one expressed in the given
India, Solicitor General, Advocate General pair of words:

(1) Solicitor General. SANDERLING : BIRD


(2) Advocate General.
(3) Attorney General of India. (1) Water : Fish (2) Mastiff: Dog
(4) Chief Justice of India. (3) Cat : Mare (4) Frog : Toad

26. A, B, C and D have got some money with 30. If it is true that 'Good governance implies law
them. If A gives 8 rupees to B, B will have as and order in society' then identify the
much as C has and A will have 3 rupees less statement which has to be accepted along
than what C has. Also if A takes 6 rupees with the given statement?
from C, A will have twice as much money as (1) An able government very effectively uses
D. If B and D together have 50 rupees, how laws to promote peace.
much money does A and B have respectively? (2) A strong government uses force to impose
laws.
(1) 29, 32 (2) 27, 40 (3) 32, 29 (4) 40, 27 (3) A healthy society is governed by maximum
number of laws.
27. Choose the pair of words from the options (4) Law is indispensable for the society.
that best represents a similar relationship as
the one expressed in the following pair of 31. Geeta is twice the present age of Seema. If
words. age of Seema is 20 years now, how many
years ago Geeta was three times Seema's
WAITER: RESTAURANT age?
(1) 10 (2) 8 (3) 12 (4) 9
(1) Driver : Car (2) Actor : Acting
(3) Author : Book (4) Teacher : School 32. Find the odd one out from the following group.
WINDSHIELD, SPARK PLUG, CLUTCH
PEDAL, CAR, ENGINE
(1) Car (2) Windshield
(3) Spark Plug (4) Engine

Previous Years
Page 210 CLAT & AILET Papers
33. From among the given options, identify the 37. An old woman decided to divide her gold
statement which means the same as the among her daughter and daughters-in-law.
statement 'The dual nature of light is an She first kept exactly half of the gold for her
enigma'. daughter. Then she divided the rest of her gold
(1) Two contradictory descriptions of light among her daughters-in-law. The eldest one
presuppose a third description. got 26 grams more than the youngest
(2) Light has distinct properties which makes daughter-in-law. The middle one got twice as
it unique. the youngest one. If the eldest daughter-in-
(3) Light is mysterious. law got 66 grams of gold, how much was
(4) The nature of light is an enigma. received by the daughter?
(1) 186 grams (2) 192 grams
34. There is some relationship between the figures (3) 172 grams (4) 194 grams
given in the series. Find out the missing one
from the alternatives given below. 38. Identify the statement which cannot be false.
(1) Myopia is a congenital disorder.

? (2) Democracy is the best form of government.


(3) Water evaporates at 100° C.
(4) All radii of any given circle are of equal
length.
(1) (2)
39. If it is true that 'Religious fundamentalism is
dangerous to the society', then which one of
(3) (4) the following statements can also be true?
(1) Belief in any religion is dangerous to the
society.
35. Mr. X, the President of a club arrived in a (2) Disrespect for other religions is dangerous
meeting at 10 minutes to 1230 hrs. Mr X came to the society.
earlier by 20 minutes than the other (3) Religious pluralism is dangerous to the
participating members in the meeting, who society.
arrived late by 30 minutes. At what time was (4) Religion without reason is dangerous to
the meeting scheduled? the society.
(1) 1240 hrs (2) 1220 hrs
(3) 1250 hrs (4) 1210 hrs 40. A 2100 member team consisting of Team
Leaders and Athletes is attending a National
36. Assume that both premises, 'No innocent Athletic Meet. For every 20 Athletes, there is
person should be punished' and 'Socrates is one Team Leader. How many Team Leaders
innocent' are true. Then which one of the would be there in the team?
following options is necessarily true? (1) 100 (2) 105
(1) Socrates is not punished. (3) 95 (4) 110
(2) Socrates should not be punished.
(3) It is not proper to punish Socrates.
(4) Socrates may not be punished.

Previous Years
CLAT & AILET Papers Page 211
AILET Question Paper 2008
1. The Indian Territory which fell under the states on the eve of Independence was:
(A) Only 20 Percent (B) Over 80 percent (C) 54 percent (D) 46 percent

2. The Supreme Court of India enjoys:


(A) Only original jurisdiction (B) Only appellate jurisdiction
(C) Only advisory jurisdiction (D) All the above types of jurisdictions

3. A money bill can be introduced in Parliament by:


(A) The Finance Minister alone (B) Any minister
(C) Any member of Parliament (D) Any member of the ruling party

4. The most powerful legislature in the world is the:


(A) Indian Parliament (B) U.S. Congress
(C) Swiss Legislature (D) British Parliament

5. In the U.S.A., residuary powers or reserve of powers are:


(A) Left to the federal government (B) Left to the States
(C) Not defined properly (D) Given to local government

6. Members of Rajya Sabha are not associated with


1. Public Accounts Committee
2. Estimates Committee
3. Committee on Public Undertakings
(A) 1 and 2 (B) 3 (C) 2 (D) 1 and 3

7. According to the Constitution, amendments cannot be proposed in either House to


(A) The Annual Financial Statement (B) Appropriation Bill
(C) Demand for Grants (D) All of the above

8. Match the following:


(A) Nagpur session (1920) (B) Madras session
(C) Calcutta session (1928) (D) Lahore session (1929)
(E) Karachi session (1930)
(i) Poorna Swaraj
(ii) Changes in the Constitution of Congress
(III) Resolution of Fundamental Rights and National Economics Policy
(iv) Return of Gandhiji to active politics after 6 years
(v) Independence resolution passed for the first time
A B C D E
(A) iii i ii v iv
(B) i ii iv v iii
(C) ii v iv i iii
(D) i iii ii v iv

Previous Years
Page 212 CLAT & AILET Papers
9. .........are words, which appear innocent, but have a latent defamatory meaning
(A) Libel (B) Slander
(C) Innuendo (D) None of the above

10. Which article of our Constitution provides that the State shall make effective position for securing
right to work?
(A) Article 41 (B) Article 39
(C) Article 21 (D) Article 45

11. Who of the following does not constitute an exception for ‘equality before law’ (Article 14)?
(A) The Foreign Diplomats (B) The Prime Minister
(C) The Governor (D) The President

12. The Constitution of India Describes India as:


(A) A Quasi-Federal Country (B) Union of States
(C) A federation of States and Union territories (D) None of the above

13. Which one of the following Fundamental Rights has been the subject of maximum litigation and
controversies?
(A) Right to Property (B) Right to Equality
(C) Right to Freedom of Religion (D) Right to Freedom

14. The Fundamental Duties of the Indian Citizens:


(A) Formed a part of the original Constitution
(B) Were added by the Forty-Second Amendment
(C) Were added by the Forty-Fourth Amendment
(D) Were defined by the Parliament through a law enacted during the emergency

15. What is contained in the Eleventh Schedule to the Constitution of India?


(A) Forms of oath of affirmation
(B) Provisions regarding disqualification on grounds of defection
(C) Items in respect of which Panchayats shall have powers and authority to function as institutions
of Self-Government
(D) Items on which Municipalities shall have powers and authority to function as institutions of Self-
Government

16. The strength of the Council of Ministers:


(A) Has been fixed by the Constitution
(B) Has been fixed by the Parliament under Representation of People’s Act, 1950
(C) Is determined by the Prime Minister keeping in view the requirement of the time
(D) Is determined by the President

Previous Years
CLAT & AILET Papers Page 213
17. The advisory Powers of the Supreme Court of India imply that:
(A) It renders advice to the Government of India on all constitutional matters
(B) It renders advice to the Prime Minister on legal matters.
(C) IT renders advice to the President on question of law or fact which is of public importance
(D) It has power to render advice to all the above persons

18. The provision under which the Supreme Court can grant ‘special leave’ to appeal against decisions
of lower courts and tribunals is contained in:
(A) Section 10 of Supreme Court Rules (B) Article 136 of Indian Constitution
(C) Article 226 of Indian Constitution (D) Article 139 of Indian Constitution

19. Unliquidated damages mean


(A) Damage to something solid
(B) Damage caused by a firm which has gone in liquidation
(C) Damage to a firm in the hands of receivers
(D) Damage to be assessed by a Court as these are not pre-determined

20. Assault and nuisance are


(A) Wrong under Criminal law (B) Wrong under tort
(C) Wrong under neither (D) Wrong under both

21. ‘A’ by cutting the moorings of a boat in which a man and a women were sitting, caused in them a
fear for life. He is liable for
(A) Criminal use of force (B) Assault
(C) Mischief (D) None of these

22. The display of articles in a show room indicating their prices amounts to
(A) Offer (B) Counter offer
(C) Invitation to an offer (D) Mere advertisement

23. An idol of Lord Krishna in a temple is


(A) A natural person
(B) A legal person
(C) Not a person in the eye of law become only the priest of the temple will be a person
(D) Not a person because no personality can be conferred on God

24. When a court sends some one in judicial custody, what does it mean by judicial custody
(A) Sent to jail (B) Sent to police look up
(C) Sent to home under judicial protection (D) None of these

25. In Criminal law, intention is an essential constituent of an offence. In law of ‘Torts


(A) Intention is relevant (B) Intention is irrelevant
(C) Intention is relevant only in some torts (D) None of these

Previous Years
Page 214 CLAT & AILET Papers
26. The law forbids the number of partner is partnership. According to law, the maximum number of
people who can form a partnership firm is
(A)10 (B) 20
(C) 30 (D) No limit

27. The term of office of a judge of the International Court of Justice is


(A) 10 years (B) 9 years
(C) 7 years (D) 5 years

28. The democratic device, used in constitutional states, by which important political questions,
particularly question relating tot he states of a region, may be referred to the people of the region for
their final decision, is called
(A) Plebiscite (B) Referendum
(C) Recall Vote (D) Quorum

29. Ancient treatise on law?


(A) Vedas (B) Upanishads
(C) Smritis (D) Shrutis

30. Muslim religious foundations


(A) Ulema (B) Jihad
(C) Quzat (D) Wakfs

31. The maximum litigation in Courts is caused by


(A) businessmen (B) Middle class
(C) Government (D) Criminals

32. The first-regular adjudicatory mobile court in the country has been inaugurated at Punbanna village
in:
(A) Haryana (B) Bihar
(C) Andhra Pradesh (D) Punjab

33. Dissolution of Muslim Marriage Act, 1939 provides grounds of dissolution of Muslim marriage to
(A) Spouses (B) Only males
(C) Only females (D) Muslim parents and parents-in-law

34. A married man commits adultery if he has sexual intercourse with a/an
(A) Unmarried women (B) Married woman
(C) Any woman except his wife (D) Unmarried woman without her consent

35. A husband is obliged to maintain his divorced wife


(A) For ever (B) For 20 years
(C) Till the children grow up (D) Till she gets remarried

Previous Years
CLAT & AILET Papers Page 215
36. Match the following:
(i) Default (A) Failure to do something required by law
(ii) Delict (B) A wrongful act
(iii) Derelict (C) A thing thrown away by its owner
(iv) Dictum (D) A judge’s observation

(A) I-A, II- C, III-D, IV-B (B) I-D,II-C,III-B,IV-A


(C) I-C, II-D, III-A, IV-B (D) I-A,II-B,III-C,IV-D

37. You send your servant with a typewriter to be delivered to your friend. The servant takes the typewriter
home and uses it over a period of time. You discovered this a fortnight later and report him for
(A) Cheating (B) Embezzlement
(C) Fraud (D) Forgery

38. India and Britain have signed an ‘Extradition treaty’. Extradition means
(A) Exports with double taxation
(B) Order of Indian Courts will apply to Indians living in the UK
(C) Indian and the U.K. will deport criminals on reciprocal basis to each other
(D) None of above

39. Genocide occurred in all but one case


(A) Bosnia (B) Nazi Germany
(C) Rwanda (D) Gulf War

40. High sea is


(A) Sea lying more than 5 km beyond the coast of a country
(B) Sea lying beyond a distance of 3 miles from the coast of any country
(C) Sea beyond 200 km of a cost of any country
(D) Both (A) & (B)

41. Which Inn of Court called Gandhiji to the Bar?


(A) Inner Temple (B) Middle Temple
(C) Lincoln’s Inn (D) Gray’s Inn

DIRECTIONS: Answer the questions (Q. 42-50), which follow from application of the under-mentioned legal
principle:

42. PRINCIPLE:
Nothing is an offence which is done in the exercise of right of private defence. This right also extends
to lawfully causing the death of the assailant, if the offence which occasions the exercise of the right
of private defence, be –
(i) an assault which reasonably causes the apprehension of death or grievous hurt (very serious
hurt).
(ii) an assault which causes reasonable apprehension of rape.
(iii) an assault which causes reasonable apprehension of kidnapping This right is available for
protecting one’s own body, as well as the body of any other, provided the assault is not self
invited.

Previous Years
Page 216 CLAT & AILET Papers
Decide whether the right of private defence is available in the following situations.

FACTS:
A was holding the birthday party of his daughter. Some of his friends decided to present her with a
car. To create an element of surprise, they decided to kidnap her for few minutes while the party was
in full swing and then get her back in the new car. They put the plan into action. While they were
kidnapping, A got alarmed and asked the guards to open fire. The guards killed all the five friends. A
is
(A) Not protected.
(B) Protected because he did not kill, the guards did.
(C) Not protected because A’s own body was not involved.
(D) Protected because to any ordinary person there would have appeared a reasonable apprehension
of kidnapping.

43. PRINCIPLE:
When a contract has been broken, the party who suffers by such breach is entitled to receive, from
the party who has broken the contract, compensation for any loss or damage caused to him thereby,
which naturally arose in the usual course of things from such breach, or which the parties knew,
when they made the contract to be likely to result from the breach of it. Such compensation is not
given for any remote or indirect loss or damage sustained by reason of the breach.
Decide, whether and to what extent B is entitled to damages in the following situation.

FACTS:
A contracts with B to sell him 1,000 tons of iron at Rs.100/- per tone. B tells A that he needs the iron
for export purposes, and that he would be selling the iron at Rs. 200/- per tonne. A breaks the
contract. When the question comes about damages, A says he will pay only Rs. 5,000/- as damages
because the some variety of iron was available in the market at Rs. 105/- per tonne. B however
contends that he should be given Rs. 1,00,000/- because that was the profit which he would have
made had A fulfilled the contract. B had actually bought the iron at Rs. 110/- and had exported it. B
is
(A) not entitled to damages
(B) entitled to Rs. 10,000 as damages
(C) entitled to rs. 5.000 as damages
(D) entitled to Rs. 1,00,000 as damages

44. PRINCIPLE:
Contract is an agreement freely entered into between the parties.

FACTS:
Tapan was a dealer in mustard oil. The Government of India by an order issued under the Essential
Commodities Act, fixed the price of mustard oil, and also the quantity which a person can buy from
the dealer. Tapan carried on his business under this order for a while, but he refused to pay sales tax
on his sale transactions on the ground that these were not the contracts freely entered into by him.
(A) Tapan would succeed because the price and quantity were not negotiated by him.
(B) Tapan would not succeed because free consent between the parties was there despite the
restriction on price and quantity
(C) He would succeed because the Government under the new order forced him to enter into contracts
(D) Both (A) & (C)

Previous Years
CLAT & AILET Papers Page 217
45. PRINCIPLE:
An occupier is not normally liable to a trespasser except in respect of willful act intended to cause
him harm or done with reckless disregard.
FACTS:
Tony, a richman, had kept a ferocious dog to guard his house. He strictly instructed all his servants
not to go near the dog. Further a special attender was hired to take care of the dog. Visitors were
warned by a prominent warning sign board about this dog.
One day, a 13 year old boy playing in the neighborhood, running after his ball got into the house. The
dog attacked him and killed him. Tony was sued for damages.
(A) Tony was not liable because the boy was trespasser
(B) Tony is not liable because a 13 year old boy ought to have known about the presence of the
ferocious dog
(C) Tony is liable of the negligence of his servant to keep watch on such a ferocious dog during the
day time
(D) Both (A) and (B)

46. PRINCIPLE:
A master will be liable for the wrongful acts of his servants in the course of employment.

FACTS:
Maria was an old widow who opened an account with the Indian Overseas Bank, whereby she
should deposit Rs. 5/- everyday in the bank. Stephen was neighbour who used to collect the amount
and deposit them in the bank. Stephen would get a small commission from the bank for the money
deposited. One day it was discovered that Stephen who had not deposited the money for more than
three months, had vanished with the amount. Maria filed a suit against the Bank.

(A) Bank would not be liable because Stephen was not a employee of the bank.
(B) Bank would not be liable because Stephen was paid commission by the bank for doing its work
(C) Bank would be liable because Stephen was paid commission by the bank for doing its work
(D) None of the above

47. PRINCIPLE: Vishal, a famous artist was requested by Arun, an industrialist to draw the portrait of
his deceased wife and paid Rs. 20,000/- in advance and agreed to pay when the work was completed,
sum of Rs. 2 lakhs. When the portrait was half drawn, Vishal died due to heart attack. His son also
a fine artist completed his father’s work and demanded the money from Arun. Arun refused to pay
and to accept the portrait drawn by Vishal’s son and also demanded the advance to be returned.
(A) Arun cannot demand the advance amount from the son of Vishal, because in normal cases the
son is not responsible for the father’s contractual obligation
(B) Son of Vishal cannot demand the rest of amount from Arun because Arun has not assigned him
the work
(C) Arun can demand the rest of the amount
(D) Both (A) and (B)

Previous Years
Page 218 CLAT & AILET Papers
48. PRINCIPLE: Ignorance of law is not an excuse in India with the practice that every person in India
should be acquainted with the law of the land.

FACTS: Mr. Jackson, a foreigner, came to Delhi in the winter season. He saw the people in Delhi set
fire on road side during night and get their body warm. One night he came out of his hotel and asked
two labourers to cut down a dry tree in Pandara Road and when they agreed he paid them Rs. 150/
- Indian currency for cutting down the tree. They cut and made the long into small pieces and the
foreigner along with the tree cutters set fire and got their body warm. After some time, the Police
Patrol car watched it and arrested the foreigner and two labourers on the spot. The foreigner pleaded
that the tree was dry and he did not know the cutting of tree from road side is a offence in India.
(A) The foreigner shall not be responsible for the offence because he himself has not cut the tree but
got it cut through the Indian people
(B) The foreigner shall be punished because in India cutting of tree from road side is an offence
(C) The pretence that he does not know such act is an offence is not an excuse for a foreigner also
(D) Both (B) and (C)

49. PRINCIPLE:
The occupier of a premises owes a duty of care to all his invitees and visitors.
FACTS:
Devi who was the owner of a big home with a compound wall, constructed an underground tank to
store water. This was covered by jute bags since the work was incomplete. The postman who came
inside to deliver registered letter, fell into this tank and hurt himself. There was also a box on the
outside of the compound wall, where all the mail could be deposited. The injured man filed a suit
against Devi claiming compensation.
(A) Devi is not liable, because she did not invite the postman to her house
(B) Devi is not liable, because the postman could have delivered the letter in the box on the outside
of the compound wall
(C) Devi is not liable because the postman was required to take care of himself
(D) Devi is liable because the postman came into the premises in the course of his duty.

50. PRINCIPLE: Even if the sovereign functions of the State are discharged negligently the State is not
vicariously liable in tort.
FACTS:
A’ was a trader in gold. There he was arrested by Police and was detained in the police lock up after
search. The gold with him along with sundry other things were seized. Later he was discharged. His
possessions seized by the police were returned, except the gold. He moved against the State in
tort. In the words of Supreme Court, “There can be no escape from the conclusion that the Police
Officers were negligent in dealing with the property after it was seized.” One of the Constables was
a Muslim. He fled with gold to Pakistan.
(A) ‘A’ succeeded because the servants of the State were negligent and thus caused injury
(B) ‘A’ failed because the Constable who seized the gold had fled to Pakistan and the gold was not
with the State at all
(C) ‘A’ failed because the acts of search and seizure by the Police Office were part of the sovereign
function of the State.
(D) There was some other relief given to ‘A’

Previous Years
CLAT & AILET Papers Page 219
DIRECTIONS: For questions 51 and 52, select the pair of the words nearest in meaning to the original.

51. FRAYED: FABRIC


(A) Watered : Garden (B) Dilapidated : Building
(C) Frozen : Ice (D) Crumpled : Paper

52. LEVITATE: MAGICIAN


(A) Cook: Mother (B) Argue : Lawyer
(C) Float : Astronaut (D) Sky jump: Parachutists

Fill in the blank with the appropriate option:

53. Judicial decrees may not change the heart, but they can_____ the heartless.
(A) Transform (B) Subdue (C) Diarm (D) Regulate

DIRECTIONS: In the following questions 54 to 57, choose the word which is most nearly the SAME in
meaning to the word and mark it in the Answer Sheet.

54. Charlatan
(A) Monster (B) Pre-historic man
(C) Quack (D) An ex-convict

55. Cavil
(A) Frivolous objection (B) To criticize sharply; reprove
(C) Moving away from centre (D) Working of brain; thought process

56. Cynosure
(A) Able to coexist (B) Brief
(C) Centre of attention (D) Abridgement

57. Consummate
(A) One of zenith of perfection (B) Existing from birth
(C) Constant; always present (D) A history

DIRECTIONS: In the following questions 58 to 62: choose the word which is most nearly the OPPOSITE
in meaning to the word and mark it in the Answer sheet.

58. Equanimity
(A) Resentment (B) Dubiousness (C) Duplicity (D) Excitement

59. Palliate
(A) Apologise (B) Hesitate (C) Wait impatiently (D) Cure completely

60. Obsequious
(A) Brusque (B) Quick-witted (C) Sharp-tongued (D) Luxurious

61. Obstreperous
(A) Critical (B) Unruly (C) Unpleasing (D) Weak

Previous Years
Page 220 CLAT & AILET Papers
62. Endemic
(A) Decorative (B) Frustrating (C) Terrorising (D) Universal

DIRECTIONS: Pick the correct answer choice for filing the blanks in each of the following sentences.

63. I was surprised............


(A) At his letting out the secret (B) Why he let out the secret
(C) That he let out the secret (D) By his letting out the secret

64. It is not possible to............


(A) Mix oil in water (B) Mix oil into water
(C) Mix oil with water (D) Mix oil by water

65. He............and escaped


(A) Forced the door open (B) Forced open the door
(C) Open forced the door (D) The door forced open

66. Let.....finish this work as soon as possible


(A) You and I (B) You and me
(C) I and you (D) Me and you

67. ............the room, the door hit him on the brow.


(A) While entering (B) As he entered
(B) On entering (D) On having entered

DIRECTIONS: For the questions 68 to 72, some of the sentences have errors and some have none. Find
out which part of the sentence has an error.

68. He returned back / when he sensed / that there was danger. / No error
A B C D

69. The progress of those plants / is not being very encouraging / and they are in various stage of decay. /
A B C
No error.
D

70. Not only the bandits robbed / the traveller of his purse / but also wounded him grievously. / No error.
A B C D

71. No less than twenty persons / were killed in / the air crash. / No error.
A B C D

72. The police went through the building / with fine toothcombs / but they found no evidence / No error.
A B C D

Previous Years
CLAT & AILET Papers Page 221
DIRECTIONS: For the questions (Q. 73-75) the first and the last sentence of the paragraph are given.
Other sentences are given in jumbled form. You have to arrange the sentences to form a readable passage.

73. (1) Some people seem to have a compulsive need to work continually, they may be called work-
addicts.
(M) However they never quite reach the top.
(N) There are companies which expect everyone to be at work early and stay back late.
(O) Because of their diligence they tend to get rapid promotion at first.
(P) Work addicts generally end up in such organisations.
(6) [in the original paper sentence 6 was missing]
The correct sequence should be:
(A) PNMO (B) MOPN (C) OMNP (D) NPOM

74. (1) If you feed a dog or tame a bear by hand


(P) or until they succeed in getting it
(Q) and tear and pull at it
(R) they get their teeth into the meat
(S) until they bite a piece off
(6) all out of your hand.
The correct sequence should be:
(A) PSRQ (B) SQRP (C) RPSQ (D) RQSP

75. (1) St. Francis taught that


(P) and that it was easier to be good
(Q) if men would be happy
(R) they must be good
(S) and happy if one were poor
(6) and did not trouble oneself about money
The correct sequence should be:
(A) PQRS (B) RPSQ (C) QRPS (D) PSQR

76. In France, children in pre-school programs spend a portion of each day engaged in a program of
stretching and exercise. Pre-school programs in the United States, however, seldom devote time to
a daily stretching and exercise program. In tests designed to measure cardiovascular fitness, children
in the United States were out-performed by their French counterparts. It can therefore be determined
that children attending pre-school programs in the United States can achieve cardiovascular fitness
only by engaging in a daily school program of stretching and exercise.

Which one of the following is an assumption on which the argument depends?


(A) A daily program of stretching and exercise will allow all children to achieve cardiovascular fitness.
(B) Cardiovascular fitness is integral to one’s overall health.
(C) It has been proven that children who participate in stretching and exercise programs in pre-
school have better cardiovascular fitness than adults.
(D) Stretching and exercise are necessary components of French children’s superior cardiovascular
fitness programs.

Previous Years
Page 222 CLAT & AILET Papers
77. The level of blood sugar for many patients suffering from disease Q is slightly higher than the level of
blood sugar in the general population. Nonetheless, most medical professionals believe that slightly
increasing blood sugar levels is a successful means by which to treat disease Q.

This apparently contradictory argument can best be resolved by which one of the following statements?
(A) Blood sugar levels for patients who have been cured of disease Q are virtually identical to the
level of blood sugar found in the general population.
(B) Many of the symptoms associated with severe cases of disease Q have been recognized in
laboratory animals with experimentally induced high blood pressure, but none of the animals
developed disease Q.
(C) The movement from inactive to advanced states of disease Q often occurs because the virus that
causes Q flourishes during periods when blood sugar levels are slightly low.
(D) The blood sugar level in patients with disease Q fluctuates abnormally in response to changes
in blood chemistry.

78. Medical studies indicate that the metabolic rates of professional athletes are substantially greater
than those of the average person. So, most likely, a person’s speed and strength are primarily
determined by that person’s metabolic rate.

Which one of the following, if true, most strengthens the argument?


(A) Some professional athletes are either faster or stronger than the average person.
(B) Some professional athletes do not have higher metabolic rates than some people who are not
professional athletes.
(C) The speed and strength of people who are not professional athletes are not primarily determined
by choices of diet and exercise.
(D) Drugs that suppress metabolic rates have been shown to have the side-effect of diminishing the
speed and strength of those who are not professional athletes.

79. Since Oscar received extensive training in how to repair motorcycles, he is able to repair many of
their most common mechanical problems. However, Oscar does not understand how internal
combustion engines work. When Oscar was given Lucy’s motorcycle to repair, he was able to fix
the problem, despite the fact that he did not understand what was causing it.

From the above paragraph, which of the following can be properly inferred?
(A) The problem with Lucy’s motorcycle involved its engine
(B) Not all mechanical problems can be repaired only by mechanics who understand how an internal
combustion engine works
(C) At least some good mechanics are able to fix mechanical problems without an understanding of
what is causing them
(D) Oscar’s mechanical training in how to repair motorcycles was incomplete

Previous Years
CLAT & AILET Papers Page 223
80. It cannot be true that the lack of success of third-party candidates in national elections is due to the
difficulties such candidates encounter in securing space on national ballots. Everyone who identifies
him or herself as a supporter of a third party has voted for a major-party candidate in at least one
national election when a third-party candidate was listed on the ballot.

Which one of the following most accurately describes a reasoning flaw in the argument?
(A) The argument overlooks the possibility that the lack of success of third-party candidates in
national elections may be due to the fact that their views on major issues prevent them from
gaining broad support
(B) The argument takes for granted that the media coverage devoted to third-party candidates for
national office is comparable to that devoted to major-party candidates for those same offices
(C) The argument treats as contradictory to some claim evidence that may instead provide support
for that claim
(D) The argument draws its conclusion through the use of a set of facts, not all of which can be true

81. The petrol prices shot up by 7% as a result of the hike in the price of crudes. The price of petrol
before the hike was Rs. 28 per litre. Vawal travels 2400 kilometers every month and his car gives a
mileage of 18 kilometers to a litre. Find the increase in the expenditure that Vawal has to incur due
to the increase in the price of petrol (to the nearest rupee)?
(A) Rs. 270 (B) Rs. 262 (C) Rs. 276 (D) Rs. 272

82. The cost of setting up the type of a magazine is Rs. 1000. the cost of running the printing machine
is Rs. 120 per 100 copies, the cost of paper, ink and so on is 60 paise per copy. The magazines are
sold at Rs. 2.75 each. 900 copies are printed, but only 784 copies are sold. What is the sum to be
obtained from advertisements to give a profit of 10% on the cost?
(A) Rs. 730 (B) Rs. 720 (C) Rs. 726 (D) Rs. 736

83. A sum is divided between A and B in the ratio of 1 : 2. A purchased a car from his part, which
depreciates per annum and B deposited his amount in a bank, which pays him 20% interest per
annum compounded annually. By what percentage will the total sum of money increase after two
years due to this investment pattern (approximately).
(A) 20% (B) 26.66% (C) 30% (D) 25%

84. Two cubes of bronze have their total weight equivalent to 60 kg. The first piece contains 10 kg of pure
zinc and the second piece contains 8 kg of pure zinc. What is the percentage of zinc in the first
piece of bronze if the second piece contains 15 per cent more zinc than the first?
(A) 15% (B) 25% (C) 55% (D) 24%

85. An ant moved for several seconds and covered 3 mm in the first second and 4 mm more in each
successive second than in its predecessor. If the ant had covered 1 mm in first second and 8 m
more in each successive second, then the difference between the path it would cover during the
same time and the actual path would be more than 6 mm but less than 30 mm. Find the time for
which the ant moved (in seconds)
(A) 5 s (B) 4 s (C) 6 s (D) None of these

Previous Years
Page 224 CLAT & AILET Papers
86. The number of natural numbers of two or more than two digits in which digits from left to right are in
increasing order is
(A) 127 (B) 128 (C) 502 (D) 501

87. The Bubna dam has four inlets. Through the first three inlets, the dam can be filled in 12 minutes;
through the second, the third and the fourth inlet, it can be filled in 15 minutes; and through the first
and the first inlet, in 20 minutes. How much time will it take all the four inlets to fill up the dam?
(A) 8 min (B) 10 min (C) 12 min (D) None of these

88. Find the sum of all two-digit numbers that give a remainder of 3 when they are divided by 7.
(A) 686 (B) 676 (C) 666 (D) 656

89. A train travels with a speed of 20 m/s in the first 10 minutes, goes 8.5 kms in the next 10 minutes,
11 kms in the next 10 minutes, 8.5 kms in the next 10 and 6 kms in the next 10 minutes. What is
the average speed of the train in kilometer per hour for the journey described?
(A) 42 kmph (B) 35.8 kmph (C) 55.2 kmph (D) 46 kmph

90. Two solutions of 90% and 97% purity are mixed resulting in 21 litres of mixture of 94% purity. How
much is the quantity of the first solution in the resulting mixture?
(A) 15 liters (B) 12 litres (C) 9 litres (D) 6 litres

DIRECTIONS: For questions (Q. 91-94), some information is provided in the form of statements. On the
basis of that information find the answer to the questions which follow.
Coach Balkishen is trying to put together a team of four players for a tennis tournament.
He has seven players available: males A, B and C and the females M, N, O and P. All players are of equal
ability and there must be at least two males on the team. For a team of four, all players must be able to
play with each other player.
Player B cannot play with player M
Player C cannot play with player P
Player M cannot play with player O

91. If player O is selected and player B is rejected the team will consist of which foursome?
(A) A, C, M and O (B) A, C, N and O
(C) A, C, P and O (D) A, N, P and O

92. If player M is on the team, what other players must be on the team as well?
(A) A, B and N (B) A, C and N
(C) A, C and O (D) A, C and P

93. What statement is false?


(M) Player B and C are never selected together
(N) Player C and O are never selected together
(O) Player C and N are never selected together

(A) Only M (B) Only N


(C) Only O (D) All the three

Previous Years
CLAT & AILET Papers Page 225
94. Which statement must always be true?
(M) If M plays, A plays
(N) If O plays, B plays
(O) If M plays, O plays

(A) Only M (B) Only N


(C) Only O (D) Only M and O

95. The door of my house is towards east direction. From the backside of my house I drove straight 100
metres, then turned towards right and drove for 100 metres, then turned towards left and drove for 50
metres and reached my destination. In what direction am I from the starting point?
(A) North-west (B) West
(C) North (D) East

DIRECTIONS: Each interrogative statement (Qs. 96-97) is followed by two arguments. You are to pick up
one of the following responses as the answer.

96. Should divorce laws be made more favourable to women in India?


(X) Yes, because women are persecuted by their women in India.
(Y) No, because it will disrupt family life.
(A) Argument ‘X’ is forceful (B) Argument ‘Y’ is forceful
(C) Neither ‘X’ nor ‘Y’ is forceful (D) Both ‘X’ and ‘Y’ are forceful

97. Is leisure irrelevant to modern life?


(M) Yes, because life has become too much mechanical today.
(N) No, because without leisure efficiency will fall, body and brain must be given rest.
(A) Only argument ‘M’ is forceful (B) Only argument ‘N’ is forceful
(C) Both ‘M’ and ‘N’ is forceful (D) Neither ‘M’ nor ‘N’ is forceful

98. The island of Bombay was acquired by the East India Company from
(A) Netherlands (B) France
(C) Portugal (D) Spain

99. The outstanding achievement of Todar Mal was in the field of


(A) Military Conquests (B) Revenue Administration
(C) Religious Reforms (D) Art and Architecture

100. The deity most praised in Rigveda is


(A) Indra (B) Agni
(C) Mitra (D) Varuna

101. The Peacock throne of Shahjahan was taken away by


(a) Changez Khan (B) Timur
(C) Nadir Shah (D) Ahmad Shah Abdali

Previous Years
Page 226 CLAT & AILET Papers
102. Shivaji’s Council of Ministers was called
(A) Nava Patnas (B) Ashta Pradhans
(C) Ashta Diggajas (D) Mantri Parishad
(NLUD 2008)
103. Charak was the court physician of
(A) Asoka (B) Kanishka
(C) Chandragupta Maurya (D) Samudragupta

104. Which of the following reasons was mainly responsible for the unpopularity of Muhammad-bin-
Tughlaq?
(A) Bad luck (B) Lack of resources
(C) Subordination of the theologians (D) Transfer of capital from Delhi to Devagiri

105. The empire of the ‘Satavahana kings’ is known by which of the following names at present?
(A) Andhra Pradesh (B) Punjab
(C) Gujarat (D) Uttar Pradesh

106. The High Courts at Calcutta, Madras and Bombay were established under the
(A) Indian High Courts Act, 1911 (B) Government of India Act, 1909
(C) Indian High Courts Act, 1861 (D) Indian High Courts Act, 1865

107. Which of the following rulers was illiterate?


(A) Ala-ud-din Khilji (B) Mohammad-Bin-Tuglak
(C) Akbar (D) Aurangzeb

108. If the original clay sediments are subjected to high temperature and pressure, they successfully
change into rocks, the correct order of which is
(A) Slate, Shale, Schist (B) Slate, Schist, Shale
(C) Shale, State, Schist (D) Shale, Schist, State

109. Exfoliation in granite is regarded as a process of


(A) Chemical weathering (B) Mechanical weathering
(C) Faulting (D) Folding

110. The tree species most commonly used in social forestry is


(A) Gulmohar (B) Mango (C) Pipal (D) Eucalyptus

111. The Hindustan-Tibet road connects


(A) Kalimpong with Lhasa (B) Shimla with Gangtok
(C) Leh and Lhasa (D) Gangtok with Gyangyse

112. The Aswan Dam is constructed at a point where


(A) There are several waterfalls
(B) There are several cataracts
(C) There are several pigeon towers to facilitate the collection of droppings needed to keep the land
productive
(D) The old traditional basic irrigation facility exists

Previous Years
CLAT & AILET Papers Page 227
113. The difference in time per degree longitude between any two places on globe is
(A) 4 minutes (B) 5 minutes
(C) 15 minutes (D) 30 minutes

114. The limit beyond which stars suffer internal collapse is called
(A) Chandrasekher limit (B) Eddington limit
(C) Hoyle limit (D) Fowler limit

115. The best anti-knock compound used in petrol to increase mileage is


(A) Ethyl-magnesium chloride (B) Sodium ethoxide
(C) Zinc ethyl (D) Tetraethyl lead

116. Rainbow is produced when sunlight falls on drops of rain. Which of the following physical phenomena
are responsible for this?
(I) Dispersion
(II) Refraction
(III) Internal Reflection

(A) II and III only (B) I and II only


(C) I, II and III (D) I and III only

117. Which of the following bombs, when dropped on a city, will kill only the inhabitants?
(A) Atom Bomb (B) Hydrogen Bomb
(C) Chemical Bomb (D) Neutron Bomb

118. Which of the following will you use to remove rust stains on cloth?
(A) Alcohol (B) Oxalic acid solution
(C) Kerosene oil (D) Lime

119. “Mach Number” is a term associated with the speed of


(A) Ships (B) Cars
(C) Aeroplanes (D) Light

120. If an object is placed midway between two parallel plane mirrors facing each other, then the number
of images that will appear in mirrors is
(A) Four (B) Infinite
(C) Two (D) Zero because the images will cancel each other

121. The spherical shape of rain drops is due to


(A) Atmospheric friction of air (B) Gravity of spherical earth
(C) Surface tension of rain water (D) Viscosity of rain water

122. What is the colour of pure diamond?


(A) Light green (B) Light yellow
(C) Light blue (D) Light pink

Previous Years
Page 228 CLAT & AILET Papers
123. Geiger counter is an instrument
(A) Determine the heart beat rate (B) Two detect radioactive radiation
(C) To measure intensity of visible light (D) To count the baggage of an airport

124. The film of oil and soapy water owe their brilliant colours to a combination of light reflection and
(A) Refraction (B) Polarisation
(C) Diffraction (D) Interference

125. When a ship enters a sea from a river, its portion under water will
(A) Increase (B) Decrease
(C) Show no change (D) Increase or decrease alternatively

126. For digestion of food, hydrochloric acid is secreted into stomach at a pH value of
(A) 2 (B) 4
(C) 6 (D) 18

127. Which of the following diseases is genetically linked?


(A) Epilepsy (B) Diabetes
(C) Colour blindness (D) Leucoderma

128. An eye defect in which one cannot distinguish between vertical and horizontal lines is called
(A) Myopia (B) Astigmatism
(C) Trachoma (D) Glaucoma

129. The gland which is attached to the digestive system but does not have any role to play in digesting
food is
(A) Salivary gland (B) Liver
(C) Spleen (D) Pancreas

130. A reptile with a four-chambered heart is


(A) Snake (B) Lizard
(C) Crocodile (D) Turtle

131. Ahmedabad-based Pioma Industries is best known for which brand?


(A) Maggi (B) Kissan
(C) Brooke-Bond (D) Rasna

132. Australian cricketer Brett Lee models for which watch company?
(A) Timex (B) HMT
(C) Titan (D) All of these

133. Sunita Williams, renowned astronaut of Indian origin, spent a record _____ days in space.
(A) 195 (B) 185
(C) 200 (D) 201

Previous Years
CLAT & AILET Papers Page 229
134. ‘Bullion’ refers to gold, silver or other precious metals in the form of
(A) Ingots or bars only (B) Bars as well as coins
(C) Coins only (D) Jewellery

135. What does “R” in Maruti Suzuki’s small car WagonR stand for?
(A) Responsible (B) Remarkable
(C) Reliable (D) Recreation

136. Which of the following took over the Anglo-Dutch steel Company Corus?
(A) Arcelor Mittal (B) Jindal Steel
(C) Neelanchal Ispat Nigam Ltd. (D) Tata Steel

137. What is Phishing?


(A) A Government Plan
(B) A stem cell
(C) Fradulent way of acquiring PIN and bank passwords using e-mail
(D) None of these

138. Which organisation is headed by Indian environmentalist R.K. Pachauri who has been awarded the
Nobel Peace Prize, 2007 along with Al Gore?
(A) International Panel on Climate Change (IPCC)
(B) International Pollution Control Board
(C) International Environment Panel
(D) International Panel on Global Warming

139. The Government of India recently decided to halt the Future Trading of which of the following
commodities for the time being?
(A) Jeera (B) Soya Oil
(C) Wheat (D) Gram

140. Which game will be played for the first time in Asian Games, 2010?
(A) Kabaddi (B) Twenty20 Cricket
(C) Squash (D) Kho-Kho

141. Who is the Managing Director of the Delhi Metro Railway Corporation?
(A) B.L. Joshi, Lt. Governor of Delhi (B) Sam Pitroda
(C) Chairman of the Indian Railway Board (D) Shri. E. Sreedharan

142. Which country has recently adopted its new National Anthem?
(A) Israel (B) Bangladesh
(C) Indonesia (D) Nepal

rd
143. Which among the following films has got the ‘Swarna Kamal Award’ for Best Feature Film in 53
National Film Awards?
(A) Parineeta (B) Sringarm
(C) Iqbal (D) Kaalpurush

Previous Years
Page 230 CLAT & AILET Papers
144. Which of the following is/ are included in the UNESCO’s natural heritage site in India?
(1) Keoladeo (2) Kaziranga National Park
(3) Manag Tiger Project` (4) Nanda Devi Biosphere Reserve

(A) 1 and 2 (B) 2 and 3


(C) 3 and 4 (D) All

145. Which of the following is India’s indigenous helicopter weaponised version made its first flight recently?
(A) Dhruv (B) Cheetah
(C) Chetak (D) Rajhans

DIRECTIONS: Passage in this section is followed by a group of questions (Qs. 146-150) to be answered
on the basis of what is stated or implied in the passage. For some questions, more than one of the choices
could conceivably answer the question. However, you are to choose the best answer, that is, the response
that most accurately and completely answers the question and blacken the corresponding space on your
answer sheet.

One of the most prolific authors of all time, Isaac Asimov was influential both in science fiction and in the
popularization of science during the twentieth century, but he is also justly famous for the scope of his
interests. Although the common claim that Asimov is the only author to have written a book in every
category of the Dewey decimal system is untrue, its spirit provides an accurate picture of the man: a
dedicated humanist who lauded the far-reaching power of reason. His most famous work, the Foundation
trilogy, can be read as an illustration of Asimov’s belief in reason and science, but even while he expressed
that belief, science itself was calling it into question.

Foundation describes a time in which a vast Empire spanning the galaxy is on the verge of collapse. Its
inevitable doom is a consequence not of its size, but of the shortsightedness of its leaders. In this environment,
a scientist named Hari Seldon devises an all encompassing plan to help human civilization recover from
the trauma of the Empire’s coming collapse. Using mathematics, Seldon is able to predict the future
course of history for thousands of years, and he takes steps that are geared toward guiding that future in
a beneficial direction. The trope of the benevolent and paternalistic scientist shaping existence from behind
the scenes, present in much of Asimov’s fiction, is never more explicit than in the Foundation series, which
describes with an epic sweep the course and progress of the Seldon Plan.

As naive and, perhaps, self-serving as the conceit of Foundation may seem to contemporary readers, it
retains to some degree its ability of comfort by offering an antidote to the complex and unpredictable nature
of experience. Science in Asimov’s time was, in popular conceptions, engaged in just this pursuit: discerning
immutable laws that operate beneath a surface appearance of contingency, inexplicability, and change.
But even while Asimov wrote, science itself was changing. In Physics, the study of matter at the subatomic
level showed that indeterminacy was not a transitory difficulty to be overcome, but an essential physical
principle. In Biology, the sense of evolution as a steady progress toward better adapted forms was being
disturbed by proof of a past large-scale evolution taking place in brief explosions, of frantic change. At the
time of Asimov’s death, even Mathematics was gaining popular notice for its interest in chaos and
inexplicability. Usually summarized in terms of the so-called ‘butterfly effect’, chaos theory showed that
perfect prediction could take place only on the basis of perfect information, which was by nature impossible
to obtain. Science had dispensed with the very assumptions that motivated Asimov’s idealization of it in
the Seldon Plan. Indeed, it was possible to see chaos at work in Foundation itself: as sequels multiplied

Previous Years
CLAT & AILET Papers Page 231
and began to be tied into narrative threads from Asimov’s other novels, the urge to weave one grand
narrative spawned myriad internal inconsistencies that were never resolved.

146. Which one of the following most accurately expresses the main point of the passage?
(A) Isaac Asimov’s greatest work, the Foundation trilogy, is an expression of the common trope of
the benevolent and paternalistic scientist
(B) Popularizations of science are always to some degree dependent idealizations and simplifications
of that science, as Isaac Asimov’s work demonstrates
(C) The impossibility of the conceit on which Isaac Asimov’s Foundation trilogy is based demonstrates
that Asimov’s fiction was based on imperfect understandings of science.
(D) Isaac Asimov’s idealization of science as revealed in his Foundation series was called into
question by the science of his time, which was increasingly focused on Chaos and indeterminacy

147. Which one of the following statements most accurately expresses the purpose of the final paragraph?
(A) The ultimate failure of the Foundation series as a coherent scientific narrative is discussed
(B) A claim is made about the purpose of Asimov’s writing and then is finally rejected
(C) A key theme of Asimov’s Foundation series is described and discoveries in science that seem
contrary to that theme are outlined
(D) The history of science is used to demonstrate the falsity of a widely believed claim about the
power of human reason

148. The author’s reference to a common claim made about Isaac Asimov, serves to
(A) Demonstrate that many untrue beliefs are held about him
(B) Illustrate the broad scope of his interests and writings
(C) Undermine the claim that he was prolific writer
(D) Substantiate his belief in the power of human reason

149. With respect of the Seldon Plan, the author’s attitude can most properly be described as
(A) Amused at the naive conception of history it implies
(B) Uncertain of the practical impossibility of its application
(C) Ambivalent because of the reliance on human reason it requires
(D) Convinced that it illustrates Asimov’s attitude toward science

150. Which one of the following statements best illustrates the “butterfly effect” as it is described in the
passage’s third paragraph?
(A) A system implemented to predict the weather worldwide for the next century is soon found to be
inaccurate because it was supplied with the incomplete data
(B) Efforts to predict the result of a nuclear reaction fail because of indeterminacy inherent in the
behavior of subatomic particles
(C) The fossil record indicates that certain adaptations found in many organisms appeared soon
after a past catastrophic event
(D) Scientific predictions about the future course of human history are found to be reasonably
accurate once existing social theories are reconciled.

Previous Years
Page 232 CLAT & AILET Papers
AILET Question Paper 2009

Directions: Choose the most appropriate word:

1. He is ___________ of spelling reforms.


(A) a protagonist (B) an advocate (C) an envoy (D) a champion

2. Negotiable : cheque::
(A) Frozen : Asset (B) Inventory : Merchandise
(C) Bank : Money (D) Trade : Tariff

3. Hedger : Shrubbery :: ? : Stick


(A) snuffer (B) cougher (C) Whittler (D) stickler

4. Honour : Governor :: ? : Duke


(A) Excellency (B) Majesty (C) Highness (D) Grace

Directions: The sentences in the middle of a passage have been removed. You are provided with the
beginning and the end of the passage and the other sentences in a jumbled order. You are to choose the
correct order that will make the passage complete and coherent.

5. 1. When Mrs. Bates came down, the room was strangely empty, with a tension of expectancy.
P: Meantime her anger was turned pale with fear.
Q: She took up her sewing and stitched for some time without raising her head.
R: She rushed to the stair door and opened it, listening.
S: The clock struck eight and she rose suddenly, dropping her sewing on the chair.
6. Then she went out, locking the door behind her.
(A) S Q P R (B) Q P S R (C) S R Q P (D) P R S Q

6. 1. Above all.
P: in the present age of light reading
Q: it is well if something heavier is cast now and then
R: of reading hastily and thoughtlessly
S: that is,
6. into the midst of the reading public.
(A) P Q R S (B) P S R Q (C) P R QS (D) Q S R P

Directions: In the following questions, choose the word which is most nearly the OPPOSITE in meaning
to the bold word and mark it in the Answer Sheet.

7. Ductile
(A) virtuous (B) grotesque (C) contentious (D) stubborn

8. The Prime Minister’s radio broadcast galvanised the people’s spirit


(A) destructed (B) frightened (C) distracted (D) dampened

Previous Years
CLAT & AILET Papers Page 233
Directions: In the following questions, choose the word which is most nearly the SAME in meaning to the
bold word and mark it in the Answer Sheet.

9. Obsequious
(A) poor (B) cheerful (C) servile (D) sullen

10. Dialectic
(A) argumentative (B) instructive (C) constructive (C) destructive

Directions: In the following questions, some of the sentences have errors and some have non. Find out
which part of the sentence has an error.

11. He not only denied / having borrowed money from me, / but also having ever met me. / No error.
(A) (B) (C) (D)

12. The running party stood / for implementation of the Bill / and was ready to stake their political
(A) (B) (C)
existence. / No error.
(D)

13. If I were he, / I should not / accept the post. / No error


(A) (B) (C) (D)

Directions: In the question a paragraph with many numbered blank spaces is given. In case of all spaces,
some phrases for filling up, are suggested. The candidate is to pick up the correct response.

____ 14 ____ evening we decided to ____ 15 ____camp as the weather was not encouraging. The wind
was high and - 16 - storm clouds ____ 17 ____ a wild wet night. Moreover we ____ 18 ____ at a spot which
looked ____ 19 ____ for a camp. A level ____ 20 ____ in the lee of a high hill ____ 21 ____some shelter
from the wind; fresh water was near ____ 22 ____ in a storm which flowed across the plain; a ____ 23 ____
of trees provided adequate supplies of fuel; and the dry grass which ____ 24 ____ on the hill side would
enhance the comfort of our beds. Each member of the part ____ 25 ____ a task. Some erected the tens;
others prepared a scanty meal; yet others ____ 26 ____ to the needs of the ponies, now exhausted after
a very strenuous day. As the angry sun sank, the bustle of activity was hushed into silence and each man
____ 27 ____ down to sleep.

14. (A) A the (B) After the (C) In the (D) Before

15. (A) dig (B) fixed (C) plant (D) pitch

16. (A) collecting (B) gathering (C) accumulating (D) moving

17. (A) preceded (B) proceeded (C) predicted (D) proved

18. (A) arrived (B) have arrived (C) would arrive (D) had arrived

19. (A) promise (B) promiseful (C) promising (D) prosperous

20. (A) expense (B) land (C) expanse (D) piece of land

Previous Years
Page 234 CLAT & AILET Papers
21. (A) gave (B) afforded (C) supplied (D) cast

22. (A) in hand (B) on hand (C) at hand (D) of hand

23. (A) group (B) cluster (C) collection (D) grove

24. (A) abundant (B) luxuriant (C) abounded (D) abounded in

25. (A) allotted (B) was allotted (C) had allotted (D) has allotted

26. (A) will attend (B) had attended (C) attended (D) would have attended

27. (A) settled (B) settles (C) laid (D) lay

28. At a particular speed, a bus starts vibrating violently due to the phenomenon of
(A) Pitch (B) Resonance (C) Rumbling (D) None of the above

29. When a bullet is fired upward vertically, it gains in


(A) Speed (B) Acceleration (C) Kinetic energy (D) Potential energy

30. The metal that is most abundant in the earth is


(A) Silicon (B) Iron (C) Aluminium (D) Nickel

31. A motorcycle passing by disturbs radio and TV reception. The disturbance is caused because
(A) Intense sound waves shake the delicate electronic components of the receiver
(B) Metallic parts of the vehicle deflect the radio waves
(C) The sparking in the spark-plug of the vehicle generates electromagnetic radiation
(D) Modern motorcycle contains an electronic ignition system which emits ratio waves

32. Match the following scientists with their contributions


(A) Weismann I. Theory of Mutation
(B) Darwin II. Principle of Independent Assortment
(C) Mendal III. Theory of Evolution
(D) Hugo-de-varis IV. Continuity of Germplasm

(A) I - (A) II - (B) III - (C) IV - (D)


(B) I - (B) II - (C) III - (D) IV - (A)
(C) I - (D) II - (C) III - (A) IV - (B)
(D) I - (D) II - (C) III - (B) IV - (A)

33. Who was the first Sultan of Delhi to introduce the practice of ‘Sijda’?
(A) Balban (B) Muhammad Tughlaq
(C) Alauddin Khilji (D) Firoz Tughlaq

34. Which of the following is rightly regarded as a milestone in the field of education in India?
(A) Sir Charles Wood’s Despatch (B) Stanley’s Despatch
(C) Hunter Commission Report (D) University Commission

Previous Years
CLAT & AILET Papers Page 235
35. A Public Works Department was set up in India by
(A) Lord Dalhousie (B) Lord Ripon
(C) Lord Warren Hastings (D) Lord William Bentinck

36. The Theory of Economics Drain from India to England was propounded by
(A) R. C. Dutt (B) B. G. Tilak (C) Dadabhai Naoroji (D) L. K. Jha

37. The Calcutta session of the Indian National Congress held in September 1920, passed a resolution
which led to the
(A) Non-Cooperation Movement (B) Civil Disobedience Movement
(C) Home Rule Movement (D) Quit India Movement

38. A cyclone is a system of wind in which the wind blows spirally


(A) towards the centre of low pressure
(B) towards central region of high pressure
(C) towards a region of low pressure
(D) outwards from a central region of high pressure

39. Contour bunding is used


(A) to stop the winds in sandy deserts (B) to irrigate desert areas
(C) to prevent erosion in hilly areas (D) None of the above

40. Which of the following is not a closed sea?


(A) Caspian sea (B) Aral sea
(C) Black sea (D) Red sea

41. If Greenwich Mean Time is ahead by 12 hours, the place may be (10 = 4 minutes)
(A) 1800 West (B) 1800 East
(C) 900 West (D) None of the above

42. The orbit of the earth is an ellipse and not a circle. The distance between earth and sun thus varies.
On January 3, earth is closest to the sun (Perihelion). Similarly, earth is said to be at Aphelion,
when it is farthest from the sun on
(A) March, 23 (B) July, 4
(C) December, 23 (D) April, 21

43. What is the meaning of ‘Gilt edged market’?


(A) Market in Government securities (B) Market in smuggled goods
(C) Market of auctioned goods (D) None of the above

44. “Reduction in rate of taxation leads to more than proportionate increase in tax yield”. This law is
known as
(A) Giffins Effect (B) Laffer Effect (C) Gresham’s Law (D) None of the above

45. The VDIS-Voluntary Disclosure of Income Scheme was the brainchild of


(A) P. Chidambaram (B) Ram Jethmalani
(C) Atal Behari Vajpayee (D) Sonia Gandhi

Previous Years
Page 236 CLAT & AILET Papers
46. A company is said to be ‘Sick’ when the accumulated loss at the end of any financial year leads to
erosion of ...... percent of its net wealth.
(A) 100% (B) 75% (C) 50% (D) 25%

47. Gunnar Myradal has dealt with the problem of Asian countries in her book ASIAN DRAMA ‘regarding
(A) poverty (B) modern industries
(C) military dictatorship (D) neo-colonialism

48. Wealth tax on agricultural property is levied by


(A) Central Government (B) State Government
(C) Both Central and State Governments (D) Non of the above

49. One of the following statements was not among the Simon Commission’s recommendations
(A) dyarchy to be abolished in the provinces
(B) reconstitution of the Central Legislature
(C) establishment of the Provincial Public Service Commission for all Provinces
(D) Indian Council is not needed to advice the Secretary of State for India.

50. Which of the following are the principal features of Government of India Act, 1919
1. Introduction of dyarchy in the executive government of the provinces
2. Introduction of separate communal electorate for Muslims
3. Devolution of legislative authority by the Centre to the Provinces
4. Expansion and reconstitution of Central and Provincial Legislature
Codes:
(A) 1, 2 and 3 (B) 1, 2 and 4
(C) 2, 3 and 4 (D) 1, 3 and 4

51. There were ............... Articles and ............ Schedules in the draft of the Constitution of India.
(A) 315, eight (B) 319, nine
(C) 327, ten (D) 317, nine

52. The Indian National Congress asserted in ............. that India would not accept any constitution
made by anyone other then people of India and without outside interference.
(A) 1942 (B) 1936
(C) 1927 (D) 1935

53. Almost all political parties participated and contributed their share in the formation of Indian
Constitution. Which one of the following party was not associated with the Constituent Assembly?
(A) Communist Party of India (B) Indian National Congress
(C) Hindu Mahasabha (D) Scheduled Castes Freedom

54. The expenditure from the Consolidated Fund of India for which the approval of Parliament is not
necessary according to the Constitution of India, is called
(A) Charged Expenditure (B) Extra Expenditure
(C) Special Provision Fund (D) None of these

55. Proclamation of Emergency on the ground of internal disturbance was for the first time made in
(A) 1971 (B) 1972
(C) 1974 (D) 1975

Previous Years
CLAT & AILET Papers Page 237
56. Which one of the following conditions regarding acquisition of citizenship by naturalisation has been
wrongly listed?
(A) He belongs to a country where citizens of India are permitted to acquire citizenship by
naturalisation
(B) He has been residing in India or serving the Government of India for at least 10 years preceding
the date of application
(C) He possesses workable knowledge of an Indian language
(D) He has consistently supported the Indian cause at various national and international forums

57. Which one of the following statements is correct?


(A) The Right to Private Property was incorporated in the Constitution by the Forty-Second Amendment
(B) The Right to Private Property granted by the original Constitution but it has since been removed
from the list of Fundamental Rights
(C) The Right to Private Property was never a Fundamental Right under the Indian Constitution
(D) The Right to Private Property which was granted by the original Constitution has been made
more sacrosanct by the Forty-Forth Amendment

58. Which one of the following Directive Principles is non Gandhian?


(A) Promotion of cottage industries in rural areas
(B) Prohibition of the use of intoxicants except for medicinal purposes
(C) Prevention of slaughter of cow, calves and other milch cattle
(D) Provision o free and compulsory education for all the children up to the age of fourteen years

59. Which one of the following steps cannot be taken by the President during the Financial Emergency?
(A) He can direct the Union and State Governments to observe such canons of financial propriety as
he deems desirable
(B) He can suspend the Fundamental Rights of the Indian citizens
(C) He can order reduction of salaries and allowances of all civil servants
(D) He can order the reduction of the salaries of the Supreme Court and High Court judges

60. A death sentence by a lower court


(A) Must be confirmed by High Court
(B) Must be confirmed by Supreme Court
(C) Is operational if no appeal is made to higher courts
(D) Must be confirmed by the President

61. How many members are required to support the introduction of a No-Confidence Motion in the Lok
Sabha?
(A) Two-thirds of the membership of the House
(B) 50 members
(C) 80 members
(D) 60 members

62. The decision of a High Court is


(A) Binding on other High Courts
(B) Not binding on other High Courts
(C) Occasionally binding on other High Courts
(D) Of no value for other High Courts

Previous Years
Page 238 CLAT & AILET Papers
63. Irresistible impulse is
(A) not covered under insanity
(B) covered under insanity
(C) covered under certain specified circumstances
(D) covered under circumstances leading to certain consequences

64. The Supreme Court in its judgment held that non-payment of minimum wages is a type of forced
labour:
(A) Asiad workers case (B) Minerva Mills case
(C) Lokhandwala Mills case (D) T. Krishnamachari case

65. X duly posts a letter of acceptance to Y. But the letter is lost in transit by the negligence of the Post
Office
(A) There is no contract concluded because the acceptance has not reached the proposer
(B) There is no contract concluded because the proposer had not received the letter
(C) The contract is concluded because the acceptance is complete from the date of despatch not
with standing any delay or miscarriage in its arrival from causes not within the acceptor’s control
(D) None of these

66. Ramesh asks his servant to sell his cycle to him at a price less than that of market price. This
contract can be avoided by the servant on the ground of
(A) fraud (B) mistake
(C) undue influence (D) coercion

67. In book depot, a catalogue of books enlisting the price of each book and specifying the place where
the particular book is available is
(A) an invitation to offer
(B) an offer
(C) an invitation to visit the book shop
(D) just a promise to make available the particular book at a particular place at the listed price

68. Mark the incorrect answer

The main purpose of the Law of Contract is


(A) The satisfaction of human desires in the highest practicable degree
(B) where there are conflicting human interests and desires by establishing a judicial and administrative
system that acts with reasonable degree of uniformity
(C) to do something in accordance with the norm of prescribed law
(D) all of the above

69. Frustration of contract implies


(A) commercial hardship
(B) physical impossibility due to disappearance of the subject matter of the contract or the object
has failed to materialise
(C) both (A) & (B)
(D) neither (A) nor (B)

Previous Years
CLAT & AILET Papers Page 239
70. With the approval of the Parliament the National Emergency can continue for
(A) a maximum period of three years
(B) a maximum period of one year
(C) an indefinite period
(D) a maximum period of six months

71. Libel is addressed to the eye: slander to the ear: State which of the following statement are slander
(A) slanderous words are uttered by the characters on the dramatic stage
(B) slanderous words are uttered by the characters on the cinema screen
(C) abuses recorded in the gramophone disc
(D) both (B) and (C)

72. Choose the correct statement

Doctrine of double jeopardy means


(A) a person should not be punished more than once for the same offence
(B) a person should be punished more than once for the same offence
(C) a man may be put twice in peril for the same offence
(D) a man may commit the same offence twice but will get punishment once only

73. “What cannot be done directly cannot be done indirectly”. This statement epitomises the doctrine of
(A) pith and substance (B) implied powers
(C) ancillary powers (D) colourable legislation

74. The 39th Amendment laid that election of any person to Lok Sabha holding the office of Prime
Minister cannot be challenged before a Court of Law, but only before an authority established by
Parliament. This was declared unconstitutional by the Supreme Court in
(A) Keshvananda Bharti Case (B) Menaka Gandhi Case
(C) Indira Gandhi v. Raj Narain Case (D) None of these
75. The Janata Party Government of Morarjee Desai constituted.........to find out the truth about the excesses
committed by the Indira Gandhi Government during emergency (1975-77)
(A) Shahbano Commission (B) Jagmohan Commission
(C) Shah Commission (D) Nayyar Commission

76. Public holidays are declared under


(A) Negotiable Instruments Act, 1881 (B) Contract Act, 1872
(C) Public Employees Act, 1967 (D) None of the above

77. According to law, the maximum number of people who can form a partnership firm is
(A) 10 (B) 20 (C) 30 (D) No limit

78. A Hindu wife can marry immediately after divorce. A Muslim wife
(A) can also marry immediately
(B) has to wait till period of Iddat (seclusion) is over
(C) has to wait for one year
(D) None of the above

Previous Years
Page 240 CLAT & AILET Papers
79. Sometimes an accused seeks pardon from the court and offers to give evidence against all others
involved in a crime. He is called..........
(A) witness (B) clone (C) approver (D) accomplice

80. Under the Motor Vehicles Act, 1988, the minimum compensation to be awarded in case of death
(vide 1994 amendment) is
(A) Rs. 25,000 (B) Rs. 30,000 (C) Rs. 40,000 (D) Rs. 50,000

81. What is the legal meaning of the word ‘Battery’?


(A) Cells as used in torch, tape recorder etc
(B) Battering a person to death
(C) Actual or intended striking of another person
(D) Assault resulting in, at least, 6 months hospitalisation

82. Match the following:


(A) Malfeasance I. One who falsely pretends to be sick
(B) Malingerer II. Minor offences
(C) Misdemeanor III. Improper performance of legal duty
(D) Misfeasance IV. Doing an unlawful act

(A) I-A II-B III-C IV-D


(B) I-B II-C III-D IV-A
(C) I-C II-D III-A IV-B
(D) I-D II-A III-B IV-C

83. Once appointed, judges of Supreme Court serve till they attain the age of
(A) 62 years (B) 63 years (C) 64 years (D) 65 years

84. Gangaram is a wood-cutter. He earns his livelihood by cutting forest trees in Nainital, Uttranchal.
The State of Uttranchal makes a law prohibiting the cutting of forest trees. Is Gangaram’s Constitutional
right infringed?
(A) Yes. Because he may not like to do any other job to earn his bread and butter
(B) No. Because the Government is sovereign and can make any law
(C) Yes. Because he has a fundamental right to life and livelihood under Article 21 of the Constitution
and the Government cannot snatch away his bread and butter by making such a law.
(D) No. Because the Government has power to make a law for prohibiting the cutting of forest trees
under Article 48A of the Constitution

85. Culpable homicide means causing death


(A) with the intention of causing death.
(B) with the intension of causing such bodily injury as is likely to cause death.
(C) with the knowledge that by such act death is likely to be caused.
(D) all the above

Previous Years
CLAT & AILET Papers Page 241
86. X takes away a girl out of the custody of her lawful guardians. Which of the following statements is
a complete defence if X is charged under section 361 of the Indian Penal Code for kidnapping on the
ground that the girl was below the age of eighteen years when taken away?
(A) the girl was a student in a college and could understand what was right or wrong for her.
(B) the girl was maltreated by the guardians and X promised her a better life.
(C) the girl looked more than 18 years of age and the accused had satisfied himself that she was
more than 18 years of age.
(D) none of these

Directions: Given below is a statement of principle followed by a facts . Apply the principle to the facts
given below and select the most appropriate answer.

87. PRINCIPLE:
The occupier of a premises owes a duty of care to all his invitees and visitors.

FACTS:
Laloo was running a dairy from his house. People used a part of his farm as a short cut to get to a
nearby railway station. Laloo who did not approve of this, put up a notice that “Trespassers will be
prosecuted”. However, since a number of these people were also his customers he tolerated them
.One day a person who was using this short cut was attacked by a bull belonging to the farm. The
injured person filed a suit against him.

DECISION:

(A) Laloo is liable for having kept a bull on his farm.


(B) Laloo is not liable in view of the clear notice against trespassers.
(C) Laloo is liable because in fact he allowed the people to use his premises.
(D) Laloo is not liable to the people other than his customers.

Directions: In each of the following questions, a series of letters or numbers has been arranged in some
sequence. Below each are given alternative responses. Find out the correct response.

88. 2, 6, 14, 11, 15, 23, 20, 24,___ , 29


(A) 31 (B) 29 (C) 28 (D) 32

89. 97, 86, 73,______, 45, 34


(A) 54 (B) 56 (C) 58 (D) 60

90. MOQ, SUM, YAC, ______


(A) FIL (B) DHJ (C) EGI (D) XAD

91. KWZ, MOX, OIV, QET, _____


(A) SAQ (B) SUR (C) RAP (D) SAR

Previous Years
Page 242 CLAT & AILET Papers
Directions: The following two (5) items consist of two Statements, one labelled the ‘Assertion A’ and the
other labelled the ‘Reason R’. You are to examine these two Statements carefully and decide if the
Assertion A and the Reason R are individually true and if so whether the Reason is a correct explanation of
the Assertion. Select your answers to these items using the codes given below and mark your answer-
sheet accordingly.
Code:
(A) Both A and R are true and R is the correct explanation of A .
(B) Both A and R are true but R is not a correct explanation of A.
(C) A is true but R is false.
(D) A is false but R is true.

92. Assertion (A) : A stranger to a contract has no right to enforce it against the parties
to the contract.
Reason (R) : Privity of contract between the parties is essential for enforcement
of contract.

93. Assertion (A) : A finder of a thing has title to it.


Reason (R) : The finder’s title to a thing is superior to that of the owner.

94. Assertion (A) : A minor is not competent to enter into any contract, even for
necessaries.
Reason (R) : For necessaries supplied to a minor, his estate can be made liable
to reimburse.

95. Assertion (A) : A legal right is a legally protected interest.


Reason (R) : An element of advantage is essential to constitute a right.

96. Assertion (A) : Customs to have the force of law must be immemorial.
Reason (R) : Custom represents common consciousness of people.

Direction: In each of the following questions one statements is followed by two arguments (A) and (B), one
supporting and the other against it.
(A) if only argument (A) is strong.
(B) if only argument (B) is strong.
(C) if either (A) or (B) is strong.
(D) if both (A) and (B) are strong.

97. Statement : Should India adopt a Presidential system?

Arguments
(A) Yes-Because our experience of Parliamentary democracy is disappointing
(B) No - Because it will concentrate power in the hands of a few people.

98. Statement : Should the death sentence be abolished?

Arguments
(A) Yes-The death sentence deprives the culprit of all chances of improving his behaviour.
(B) No - Capital punishment restrains criminal tendency.

Previous Years
CLAT & AILET Papers Page 243
Directions: In the following question, a statement is followed by two conclusions (A) and (B) . You have to
assume everything in the statement to be true, consider both the conclusions together, and then decide
which of the two given conclusions logically follow beyond a reasonable doubt from the information given in
the statement.
(A) if only conclusion (A) follows.
(B) if only conclusion (B) follows.
(C) if either (A) or (B) follows.
(D) If neither (A) nor (B) follows.

99. Statement: The greatest advantage of democracy over all other forms of governments is not that
the men who have gone to the top are exceptionally wise, but that since their power depends upon
popular support, they know that they cannot retain their position if they are guilty of more than a
modicum of injustice.

Conclusions:

(A) In a democracy, persons in power cannot act arbitrarily.


(B) Democratic countries cannot have excellent leaders.

100. I go 100 metres towards North from my house, then I turn left and go 200 metres, then turning left I
go 200 metres, then again turning left I go 100 metres. and then turning left again I go 100 metres.
In which direction am I now from my house?
(A) West (B) East (C) North (D) South

Directions: Some information is provided in the form of statements. On the basis of that information find
the answer to the questions which follow.
1. A cube has six sides each of a different colour
2. The red side is opposite black
3. The green side is between red and black
4. The blue side is adjacent to white
5. The brown side is adjacent to blue
6. The red side is face down.

101. The side opposite to brown is


(A) Red (B) Black (C) White (D) Green

102. The four colours adjacent to green are________


(A) Red, Black, Blue and White (B) Red, Black, Brown and Blue
(C) Red, Black, Brown and White (D) Black, Blue. White and Red

103. K is L’s wife’s husband’s brother. M is the sister of K. N is the sister of L. How is K related to M?
(A) Sister-in-law (B) Brother (C) Daughter (D) Wife

104. Atul is the son of Zamir. Alka is the daughter of Aman. Sheela is the Wife of Aman, Mohan is the son
of Sheela. How is Alka related to Mohan?
(A) Sister (B) Uncle (C) Son (D) Father

Previous Years
Page 244 CLAT & AILET Papers
Directions: Five adults A, B, C, D and E are sitting at Bharat Provision Store. In the group, there are one
Manager, one Accountant, one Supplier of provisions. The Accountant, who has a child earns least money.
A. who is married to C’s sister, earns more than the Manager . D is an unmarried lady and does not do any
work. There is one married couple in a group of which B is husband. E is the brother of C and is neither a
Supplier nor an Accountatnt. No lady is a Supplier or Manager. C is neither a Supplier nor an ‘Accountant’.

105. Who is Accountant?


(A) A (B) B (C) C (D) D

106. Who is Supplier?


(A) A (B) B (C) C (D) D

107. Who is the wife of B?


(A) A (B) B (C) C (D) E

108. Who is earning the highest?


(A) A (B) B (C) C (D) D

109. Who is the sister of C?


(A) A (B) B (C) C (D) D

110. It is possible to make a meaningful word with the third, the fourth, and the eleventh letters of word
‘CONTROVERSIAL’ write the first letter of that word.
(A) I (B) N (C) S (D) T

111. If (A) Chip din chunk means student attends class; (B) din sunk dink means Arjuna is student; (C)
jump nink sink means schools are good; (D) dink mup chimp means teacher is good. Then what is
the code for Arjuna?
(A) din (B) sunk (C) dink (D) chunk

Directions: In the following groups one does not belong to that group. Find the odd one.

112. (A) Cheese (B) Saffron (C) Pepper (D) Lard

113. (A) Hansom (B) Victoria (C) Growler (D) Baroque

Directions: The questions in this section are based on the reasoning contained in brief statements or
passages. For some questions, more than one of the choices could conceivably answer the question.
However, you are to choose the best answer, that is, the response that most accurately and completely
answers the question. You should not make assumptions that are by commonsense standards implausible,
superfluous or incompatible with the passage.

Consumer advocate: Businesses are typically motivated primarily by the desire to make as great a profit
as possible, and advertising helps business to achieve this goal. But it is clear that the motive of maximizing
profits does not impel businesses to present accurate information in their advertisements. It follows that
consumers should be skeptical of the claims made in advertisements.

Previous Years
CLAT & AILET Papers Page 245
114. Each of the following if true would strengthen the Consumer advocate’s argument EXCEPT:
(A) Businesses know that they can usually maximize their profits by using inaccurate information in
their advertisements.
(B) Businesses have often included inaccurate information in their advertisement.
(C) Many consumers have a cynical attitude toward advertising.
(D) Those who create advertisements are less concerned with the accuracy than with the creativity
of advertisements.
Science columnist: It is clear why humans have so many diseases in common with cats. Many
human diseases are genetically based, and cats are genetically closer to humans than are any
other mammals except nonhuman primates. Each of the genes identified so far in cats has an exact
counterpart in humans.

115. Which one of the following if true, most weakens the science columnist’s explanation for the claim
that humans have so many diseases in common with cats?
(A) Cats have built up resistance to many of the disease they have in common with humans.
(B) Most diseases that humans have in common with cats have no genetic basis.
(C) Cats have more diseases in common with nonhuman primates than with humans.
(D) Many of the diseases human have in common with cats are mild and are rarely diagnosed.
Psychologist: It is well known that becoming angry often induces temporary incidents of high blood
pressure. A recent study further showed, however, that people who are easily angered are significantly
more likely to have permanently high blood pressure than are people who have more tranquil
personalities. Coupled with the long established fact that those with permanently high blood pressure
are especially likely to have heart disease, the recent findings indicate that heart disease can result
from psychological factors.

116. Which one of the following would, if true, most weaken the psychologist’s argument?
(A) Those who are easily angered are less likely to recover fully from episodes of heart disease than
are other people.
(B) Medication designed to control high blood pressure can greatly affect the moods of those who
use it.
(C) People with permanently high blood pressure who have tranquil personalities virtually never
develop heart disease.
(D) The physiological factors that cause permanently high blood pressure generally make people
quick to anger.
A professor of business placed a case-study assignment for her class on her university’s computer
network. She later found out that instead of reading the assignment on the computer screen. 50 out
of the 70 students printed it out on paper. Thus, it is not the case that books delivered via computer
will make printed books obsolete.

117. Which one of the following, if true, most strengthens the argument?
(A) Several colleagues of the professor have found that, in their non-business courses, several of
their students behave similarly in relation to assignments placed on the computer network.
(B) Studies consistently show that most computer users will print reading material that is more than
a few pages in length rather than read it on the computer screen.
(C) Some people get impaired vision from long periods of reading printed matter on computer screens,
even if they use high quality computer screens.
(D) Scanning technology is very poor, causing books delivered via computer to be full of errors
unless editors carefully read the scanned versions.
Previous Years
Page 246 CLAT & AILET Papers
Directions: Each group of questions in this section is based on a set of conditions. Choose the response
that most accurately and completely answers each questions.

A panel reviews six contract bids - H, J, K, R, S, and T. No two bids have the same cost.
Exactly one of the bids is accepted. The following conditions must hold:

1. The accepted bid is either K or R and is either the second or the third lowest in cost.
2. H is lower in cost than each of J and K.
3. If J is not the fourth lowest in cost, then J is higher in cost than each of S and T.
4. Either R or S is the fifth lowest in cost.

118. Which one of the following could be an accurate list of the bids in order from lowest to highest in
cost?
(A) T, K, H, S, J, R (B) H, T, K, S, R, J (C) H, S, T, K, R, J (D) H, K, S, J, R, T

119. Which one of the following bids CANNOT be the fourth lowest in cost?
(A) H (B) J (C) K (D) R

120. Which one of the following bids CANNOT be the second lowest in cost?
(A) H (B) J (C) K (D) R

Directions for questions 121 and 122: Detectives investigating a city wide increase in burglaries questioned
exactly seven suspects - S, T, V, W, X, Y and Z - each on a different one of seven consecutive days. Each
suspect was questioned exactly once. Any suspect who confessed did so while being questioned.
The investigation conformed to the following:
1. T was questioned on day three.
2. The suspect questioned on day four did not confess.
3. S was questioned after W was questioned.
4. Both X and V were questioned after Z was questioned.
5. No suspects confessed after W was questioned.
6. Exactly two suspects confessed after T was questioned.

121. Which one of the following could be true?


(A) X was questioned on day one. (B) V was questioned on day two.
(C) Z was questioned on day four. (D) W was questioned on day five.

122. If Z was the second suspect to confess, then each of the following statements could be true EXCEPT:
(A) T confessed. (B) T did not confess.
(C) V did not confess. (D) Y did not confess.

123. Find the least number that when divided by 16, 18 and 20 leaves a remainder 4 in each case, but is
completely divisible by 7.
(A) 364 (B) 2254 (C) 2884 (D) 3234

124. The average age of three boys is 15 years. If their ages are in the ratio 3 : 5 : 7, the age of the
youngest boy is
(A) 21 years (B) 18 years (C) 15 years (D) 9 years

Previous Years
CLAT & AILET Papers Page 247
125. A’s salary is first increased by 25% and then decreased by 20%. The result is the same as B’s
salary increased by 20% and then reduced by 25%. Find the ratio of B’s salary to that of A’s.
(A) 4 : 3 (B) 11 : 10 (C) 10 : 9 (D) 12 : 11

126. A man sells 5 articles for Rs.15 and makes a profit of 20%. Find his gain or loss percent if he sells
8 such articles for Rs.8.40.
(A) 2.22% profit (B) 2.22% loss (C) 8% loss (D) 8%profit

127. What is the simple interest for 9 years on a sum of Rs.800 if the rate of interest for the first 4 years
is 8% per annum and for the last 4 years is 6% per annum?
(A) 400 (B) 392 (C) 352 (D) Cannot be determined

128. In Ramnagar Colony, the ratio of school going children to non-school going children is 5 : 4. If in the
next year, the number of non-school going children is increased by 20% making it 35,400. what is
the new ratio of school going children to non-school going children?
(A) 4 : 5 (B) 3 : 2 (C) 25 : 24 (D) None of these

129. Amar and Akbar left Bhubaneshwar simultaneously and travelled towards Cuttack. Amar’s speed
was 15 km/h and that of Akbar was 12km/h. Half an hour later, Anthony left Bhubaneshwar and
travelled in the same direction. Some time later, he overtook Akbar and 90 minutes further on he
overtook Amar. Find Anthony’s speed.
(A) 18 kmph (B) 24 kmph (C) 12 kmph (D) 16 kmph

130. Five boys and three girls are sitting in a row of eight seats. In how many ways can they be seated
so that not all girls sit side by side?
(A) 36,000 (B) 45,000 (C) 24,000 (D) None of these

131. A bag contains 5 red, 4 green and 3 black balls. If three balls are drawn out of it at random, find the
probability of drawing exactly 2 red balls.
(A) 7/22 (B) 10/33 (C) 7/12 (D) 7/11

132. Ajit can do as much work in 2 days as Baljit can do in 3 days and Baljit can do as much in 4 days
as Diljit in 5 days. A piece of work takes 20 days if all work together. How long would Baljit take to
do all the work by himself ?
(A) 82 days (B) 44 days (C) 66 days (D) 50 days

133. “Project Tiger” was launched in


(A) 1973 (B) 1974 (C) 1978 (D) 1981

134. Which of the following fighter air craft was flown by former President Dr. Abdul Kalam?
(A) F-16 (B) MI - 30 (C) Jaguar (D) Sukhoi - 30 MKI

135. The name “ Baichung Bhutia” is associated with


(A) Football (B) Hockey (C) Polo (D) Cricket

136. The present Chairperson of the National Human Right Commission of India is
(A) R.C.Lahoti (B) Rajendra Babu (C) J.S. Verma (D) A.S.Anand

Previous Years
Page 248 CLAT & AILET Papers
137. The present Secretary General of United Nations Organisation is
(A) Condoleeza Rise (B) Shashi Tharoor (C) Ban ki Moon (D) Kofi Annan

138. Who is the author of the book “My Life”?


(A) Bill Clinton (B) Tony Blair (C) Hillary Clinton (D) Dalai Lama

139. “Golden Handshake” is the term associated with


(A) Share Market (B) Investment of Gold
(C) Voluntary Retirement Benefits (D) Marriage without Dowry

140. National Disaster Management Authority is constituted under


(A) Natural Calamities Management Act, 2005.
(B) Disaster Management Act.2005
(C) Planning Commission Decision
(D) Cabinet Decision

141. Indian Airlines (New name: air India) have redesigned their logo which is a graphic wheel. This logo
has been inspired from which one of the following?
(A) Khajuraho Temple (B) Sun Temple, Konark
(C) Mamallapuram Temples (D) Hampi Temples

142. Who has been recently awarded Officer de La Legion Honor, the highest award of France?
(A) Shatrughan Sinha (B) Lata Mangeshkar
(C) Amitabh Bachan (D) Aishwarya Rai

Directions: Each set of questions in this section is based on the passage. The questions are to be
answered on the basis of what is stated or implied in the passage. For some of the questions, more than
one of the choices could conceivably answer the question. However, you are to choose the best answer;
that is, the response that most accurately and completely answers the questions.

Most people acknowledge that not all governments have a moral right to govern and that there are sometimes
morally legitimate reasons for disobeying the law, as when a particular law prescribes behaviour that is
clearly immoral. It is also commonly supposed that such cases are special exceptions and that, in general,
the fact that something is against the law counts as a moral, as well as legal, ground for not doing it; i.e.
we generally have a moral duty to obey a law simply because it is the law. But the theory known as
philosophical anarchism denies this view, arguing instead that people who live under the jurisdiction of
governments have no moral duty to those government to obey their laws. Some commentators have
rejected this position because of what they take to be its highly counter intuitive implications:(A) that no
existing government is morally better than any other (since all are , in a sense, equally illegitimate), and (2)
that, lacking any moral obligation to obey any laws people may do as they please without scruple. In fact,
however, philosophical anarchism does not entail these claims.

First, the conclusion that no government is morally better than any other does not follow from the claim that
nobody owes moral obedience to any government. Even if one denies that there is a moral obligation to
follow the laws of any government, one can still evaluate the morality of the policies and actions of various
governments. Some governments do more good than harm, and others more harm than good, to their
subjects. Some violate the moral rights of individuals more regularly, systematically, and seriously than
others. In short, it is perfectly consistent with philosophical anarchism to hold that governments vary
widely in their moral stature.
Previous Years
CLAT & AILET Papers Page 249
Second, philosophical anarchists maintain that all individuals have basic, non-legal moral duties to one
another-duties not to harm others in their lives, liberty, health, or goods. Even if governmental laws have no
moral force, individuals still have duties to refrain from those actions that constitute crimes in the majority
of legal systems (such as murder, assault, theft, and fraud). Moreover, philosophical anarchists hold that
people have a positive moral obligation to care for one another, a moral obligation that they might even
choose to discharge by supporting cooperative efforts by governments to help those in need. And where
others are abiding by established laws, even those laws derived from mere conventions, individuals are
morally bound not to violate those laws when doing so would endanger others. Thus, if others obey the law
and drive their vehicles on the right, one must not endanger them by driving on the left for , even though
driving on the left is not inherently immoral, it is morally wrong to deliberately harm the innocent.

143. Which one of the following most accurately expresses the main point of the passage?
(A) Some views that certain commentators consider to be implications of philosophical anarchism
are highly counter-intuitive.
(B) Contrary to what philosophical anarchists claim, some governments are morally superior to
others and citizens under legitimate governments have moral obligations to one another.
(C) It does not follow logically from philosophical anarchism that no government is morally better
than any other or that people have no moral duties toward one another.
(D) Even if as certain philosophical anarchists claim, governmental laws lack moral force, people
still have moral obligation to refrain from harming one another.

144. The author identifies which one of the following as a commonly held belief?
(A) In most cases we are morally obligated to obey the law simply because it is the law.
(B) All governments are in essence morally equal.
(C) We are morally bound to obey only those laws we participate in establishing.
(D) Most crimes are morally neutral, even though they are illegal.

145. The author’s stance regarding the theory of philosophical anarchism can most accurately be described
as one of
(A) ardent approval of most aspects of the theory.
(B) apparent acceptance of some of the basic positions of the theory
(C) concerned pessimism about the theory’s ability to avoid certain extreme views
(D) hesitant rejection of some of the central features of the theory

146. By attributing to commentators, the view that philosophical anarchism has implications that are
“counter-intuitive”, the author most likely means that the commentators believe that
(A) the implications conflict with some commonly held beliefs.
(B) there is little empirical evidence that the implications are actually true.
(C) common sense indicates that philosophical anarchism does not have such implications.
(D) the implications appear to be incompatible with each other.

Previous Years
Page 250 CLAT & AILET Papers
147. Which one of the following scenarios most completely conforms to the view attributed to philosophical
anarchists in third paragraph?
(A) A member of a political party that is illegal in a particular country divulges the names of other
members because he fears legal penalties.
(B) A corporate executive choose to discontinue illegally when she learns that the chemicals are
contaminating the water supply.
(C) A person who knows that a co-worker has stolen funds from their employer decided to do
nothing because the co-worker is widely admired.
(D) A person neglects to pay her taxes, even though it is likely that she will suffer severe legal
penalties as a consequence, because she wants to use the money to finance a new business.

148. It can be inferred that the author would be most likely to agree that
(A) people are subject to more moral obligations than is generally held to be the case
(B) government that are morally superior recognize that their citizens are not morally bound to obey
their laws.
(C) one may have good reason to support the efforts of one’s government even if one has no moral
duty to obey its laws.
(D) there are some sound arguments for claiming that most governments have a moral right to
require obedience to their laws.

149. The author’s discussion of people’s positive moral duty to care for one another functions primarily to
(A) demonstrate that governmental efforts to help those in need are superfluous.
(B) suggest that philosophical anarchists maintain that law that foster the common good are
extremely rare.
(C) imply that the theoretical under-pinnings of philosophical anarchism are inconsistent with certain
widely held moral truths.
(D) indicate that philosophical anarchists recognise that people are subject to substantial moral
obligations.

150. In the passage, the author seeks primarily to


(A) describe the development and theoretical under-pinnings of a particular theory.
(B) establish that a particular theory conforms to the dictates of common sense.
(C) argue that two necessary implications of a particular theory are morally acceptable.
(D) defend a particular theory against its critics by showing that their arguments are mistaken.

Previous Years
CLAT & AILET Papers Page 251
AILET Question Paper 2010

SECTION – A : ENGLISH

I. Read the passage and answer the questions following it : Artists should treat their art as art
and take the process of making it as seriously as anyone takes their chosen profession. Great skill
and insight are required in order to create truly original art. Transforming an idea or concept into a
technically thought- provoking or emotion-arousing work of art in any medium is a talent that few
people possess. And there you have the “purist's vision."

Now if an artist wants to create art and never sell it, then he or she never has to worry about how to price
it. That artist can afford to be a "purist," as you put it, produce art free of any encumbrances or concerns
about what the art world or anyone else might think, and avoid "prostituting" or "debasing" that art by
placing dollar values on it. But if you're an artist who wants to sell your art or who has to sell it in order to
survive as an artist, you must use whatever tools are available to figure out how much it's worth and how
best to sell it.

Let's say you're just starting out as an artist, you have little or no experience showing or selling your work,
and in a period of two minutes, you produce a pencil drawing on a piece of paper. You view this drawing as
highly significant in your evolution as an artist and rank its creation as the single most important creative
moment of your life. Consequently, you put a price of $20,000 on it because only for that amount of money
will you agree to part with such an important work of art. This is a "purist's vision" approach to pricing as
opposed to a "realities of the marketplace" approach.

From a business standpoint, you'll have an extremely difficult time selling your drawing, as you won't be
able to justify the $20,000 price to real art buyers in the real art world. You have no track record of selling
art in that price range, and you have few or no shows, critical reviews, or supporting data from outside
sources indicating that your art has that kind of value or collectibility in the marketplace. The overwhelming
majority of art buyers who have $20,000 to spend look for works of art by established artists with documented
track records of showing and selling art in that price range.Your drawing is still highly significant to you, but
what someone is willing to pay for it on the open market is a matter for art buyers to decide. You can price
it however you wish, but you can never force anyone to buy it. That's the way the art business works. So
if you want to sell it, you have to figure out what dollar amount someone is likely to pay for it on the open
market and then price it at that amount. But the tale of your drawing does not end here.

The art world may, one agree with you that the product of your two-minute moment precipitates a major
transformational turning point in your career, and is well worth a $20,000 asking price, but you're going to
have to prove first. Aspects of that drawing will have to be reflected in your art from the moment you created
it onwards, the art world will have to recognize your art both critically and from the marketing standpoints,
and you'll have to successfully produce, show, and sell for many years. Then one day, when your first
retrospective exhibition opens at the Four-Star Museum of Art, that drawing will hang framed and captioned
as the first inspiration for all subsequent work. The art world will then understand and respect its significance,
and a serious collector may well be willing to pay an extraordinary price to own this historically important
document of your career.

Previous Years
Page 152 CLAT & AILET Papers
Returning for a moment to the concept of a purist artist who creates art and never sells it, sooner or later
(hopefully later), that purist will pass on and leave behind a body of work. Unless that artist leaves specific
instructions in his or her will for that body of work to be destroyed, it will become subject to those market
forces that the artist strived for a lifetime to avoid. At the very least, it'll have to be appraised for tax,
donation, or inheritance purposes. In most cases, it eventually comes onto the market either through a
probable sale, an auction, or as represented by a dealer, gallery, or family member. The moral of the story
is that one way or another, someone somewhere at some point in time will use tried and true methods to
realistically price and either sell, donate, trade or otherwise transact any work of art that comes onto the
market in any way, shape, or form. I hope that that person will be you, the artist, and that you'll price your
art according to what the market will bear, sell plenty of it, and have a long and rewarding career.

Answer the following questions indicating your option for each question:

1. The purist's art is


(A) one that arouses emotions.
(B) thought-provoking.
(C) technically sound.
(D) free from any encumbrances or concerns about what the world or anyone else might think of it.

2. The "realities of the marketplace" approach entails.


a) "prostituting" one's art by putting a dollar price on it.
b) gauging the market value of one's art and then putting a price on it.
c) compromising on one's estimation of one's own art as far as its worth in financial terms is
concerned
d) subjecting one's art to the buyer's interpretation of it
(A) Only a (B) only b and d (C) b, c and d (D) only b

3. For an artist to sell his piece of art at a higher price range, he/she has to
(A) get endorsed by fellow artists and art critics.
(B) hold exhibitions or shows.
(C) gradually increase the collectibility of his/her art in the market place.
(D) all of the above.

4. The first piece of an artist that comes out in the market is seen as
(A) his masterpiece.
(B) the most collectible piece in his / her repertoire.
(C) a blueprint to discern the author's unique style and point of view.
(D) the inspiration for all his subsequent works.

5. From the point of view of the purist, the irony as far as pricing art is concerned is that:
a) a piece from his collection is eventually sold at the same price that he had estimated to be its
real worth years earlier.
b) his art is subjected to the same market forces against which he strove his entire life.
c) “realities of the marketplace” is a concept that negates the very attributes that we associate with
art- whim, fancy and imagination.

(A) only a (B) a and b (C) only b (D) a, b and c

Previous Years
CLAT & AILET Papers Page 153
For questions 6 and 7: Each question consists of two capitalized words that have a certain relationship
to each other, followed by 4 pairs of words. Choose the pair that is related to each other in the same way
as the capitalized pair

6. ENCUMBRANCE : BURDEN
(A) Mnemonic : Memory
(B) Blatant : Subtle
(C) Captious : Acrimonious
(D) Feral : Cultured

7. SUBSEQUENT : PREVIOUS
(A) Significant : Inconsequential
(B) Retrospect : Retrospective
(C) Visionary : Seer
(D) Caption : Legend

Choose the right option :

8. The synonym of the word PRECIPITATE in the context of the passage is


(A) Iaunch (B) trigger (C) provoke (D) accelerate

Complete the sentences by filling in the blanks with the correct tenses from the given choices by indicating
your option for each question:

9. I wish you ________ louder as I can't hear what you say.


(A) were speaking (B) spoke (C) would speak (D) speak

10. The audience ________ to take their seats please.


(A) is requested (B) have requested (C) may request (D) are requested

11. It ________all day yesterday before the garden wall collapsed.


(A) rained (B) has been raining (C) had rained (D) was raining

12. Each member of this group ________ guilty.


(A) is (B) are (C) is being (D) were

13. The Prime Minister ________ to visit the hospital tomorrow.


(A) will (B) is (C) must (D) have

14. Last night the radio said that the volcano, Etna, in Sicily ________
(A) will erupt (B) is erupting (C) erupts (D) has to erupt

15. They say that the Princess ________ incognito at the Sheraton.
(A) have stayed (B) is staying (C) stay (D) will have been staying

16. She ________ in Calcutta for a quarter of a century now.


(A) will live (B) lived (C) has been living (D) had been living

Previous Years
Page 154 CLAT & AILET Papers
17. When we went to their home, Sushma ________ some fresh coffee for us.
(A) made (B) has made (C) will make (D) is making

18. Evidence shows that Jackson ________inside the house at the time of the murder.
(A) was (B) is (C) will be (D) has been

II. Choose the word or phrase that best completes the sentence from the given alternatives.
Indicate your option.

19. To answer accurately is more important than


(A) quick finish (B) finish quickly (C) finishing quickly (D) you finish quickly

20. Professional people now-a-days appreciate ________ when it is necessary to cancel an appointment.
(A) Your calling them (B) That you would call them
(C) You to call them (D) You are calling them

21. Who will ________ the children when their mother is in the hospital ?
(A) look after (B) look for (C) look up (D) look about

22. The teacher gives many examples to ________the idea contained in the poem.
(A) bring about (B) bring in
(C) bring forth (D) bring out

23. That magnificent ________ temple was constructed by the Chinese.


(A) eight -centuries-old (B) eight-century' s-old (C) old-eight-centuries (D) eight-century-old

24. Wheat is not native to India and Barley ________


(A) isn't either (B) is either
(C) is neither (D) isn't neither

25. Encounters between people from different countries can results in misunderstandings ________
different concepts about space.
(A) because they (B) is because they
(C) is because their (D) because of their

III. Choose the correct answer which is closest in meaning to the word/ phrase in bold given
in the sentence. Indicate your option.

26. You may think at first that it is queer to talk of having too much paper money and that money is so
nice and useful that you cannot have too much of it.
(A) Ridiculous (B) Absurd (C) Anomalous (D) Odd

27. The government is under no obligation to offer contracts to companies which choose to flout
guidelines.
(A) Condemn (B) Ignore (C) Defy (D) Neglect

28. Indians exhibited a remarkable solidarity at the time of war.


(A) Coalition (B) Co-operation (C) Unification (D) Unity

Previous Years
CLAT & AILET Papers Page 155
29. His impeccable style caught the attention of all critics.
(A) Faultless (B) Inoffensive (C) Upright (D) Harmless

30. The inspector was a vigilant young man.


(A) Intelligent (B) Ambitious (C) Watchful (D) Smart

31. The young is quite sanguine about the result of his competitive examination.
(A) Depressed (B) Pessimistic (C) Anxious (D) Optimistic

32. It was a scurrilous attack on him.


(A) Serious (B) Unjustified (C) Insulting (D) Justified

33. Both parties were amenable to a peaceful settlement of the land dispute.
(A) Unresponsive (B) Responsive (C) Unwilling (D) Doubtful

IV. In the following questions, four alternatives are given for the idiom/phrase marked In bold
in the sentence. Choose the alternative which best expresses the meaning of the idiom/
phrase from the options given. Indicate your option.

34. The neighbour had to pay through his nose for a brand new car.
(A) Take huge loans (B) Pay a reasonable price
(C) Pay an extremely high price (D) Make a quick buck

35. If he phones again, I am going to give him a piece of my mind.


(A) To be nice to him (B) To take advantage of him
(C) To reprimand (D) To support him

SECTION – B : GENERAL KNOWLEDGE

36. The 2009 Nobel Peace Prize was awarded to


(A) Al Gore (B) Jane Adams
(C) Mohammed Yunus (D) Barak Hussein Obama

37. The present Prime Minister of Bangladesh is


(A) Sheik Hasina. (B) Khaleda Zia (C) Ziaur Rahman (D) None

38. The President of South Africa at present is


(A) Nelson Mandela (B) Jacob Zuma (C) Thabo Mbeki (D) Kgalema Motlanthe

39. The former President who committed suicide in May 2009 was
(A) Kim Jong II of North Korea (B) Maumoon Abdul Gayoom of Maldives
(C) Meghavati Sukarnoputri of Indonesia (D) Roh Moo-Hyun of South Korea

40. Who among the following has not won an Oscar?


(A) A.R. Rahman (B) Satyajit Ray (C) Bhanu Athaiya (D) Dev Patel

41. The present Chief Election Commissioner of India is


(A) T.N. Seshan (B) Navin Chawla (C) N.S. Gopalaswami (D) V.S. Sampath

Previous Years
Page 156 CLAT & AILET Papers
42. 76 CRPF jawans were killed recently by the Maoist in an ambush near
(A) Dantewada in Chattisgarh (B) Konarkhurd in Bihar
(C) Gadhchiroli in Maharashtra (D) Karimnagar in Andhra Pradesh

43. The National Dairy Research Insdtute in Haryana cloned a buffalo calf and named it as
(A) Gowri (B) Tusna (C) Garima (D) Shakthi

44. The former Chief Justice of India who did not become the Chairperson of the National Human Rights
Commission is
(A) Justice S. Rajendra Babu (B) Justice M. N. Venkatachalaiah
(C) Justice Lahoti (D) Justice Verma

45. The first sitting Head of the State to be charged with ‘War crimes and crimes against humanity’ by
the International Criminal Court is
(A) President Manuel Zelaya of Honduras (B) President Omar al-Bashir of Sudan
(C) President Siad Bane of Somalia (D) President Martin Torrijos of Panama

46. The former Chief Election Commissioner to be appointed as a Union Minister is


(A) Sukumar Sen (B) T. Swaminathan (C) R.K. Trivedi (D) M.S. Gill

47. India's Imperial Capital was officially shifted from Calcutta to Delhi in
(A) 1912 (B) 1935 (C) 1947 (D) 1872

48. The first Women's University was established in Pune during


(A) 1909 (B) 1928 (C) 1915 (D) 1968

49. How many official languages are mentioned in Schedule VIII to the Constitution of India
(A) 18 (B) 22 (C) 14 (D) 28

50. The present Committee on Centre-State Relations in India is headed by


(A) Justice Punnchi (B) Justice Ahmedi (C) Justice Anand (D) Justice Ruma Pal

51. In India, women were allowed to contest election and to public offices for the first time in
(A) 1909 (B) 1919 (C) 1935 (D) 1926

52. The recently constituted Telangana Committee by the Central Government is headed by
(A) Justice P.V. Reddy (B) Justice Sri Krishna
(C) Justice Venkatachala (D) Justice Banerjee

53. The Committee on Reforming Criminal Justice System in India was headed by
(A) Justice K.T. Thomas (B) Justice Krishna Iyer
(C) Justice V.S. Malimath (D) Justice Mohan

54. The first President of the Constituent Assembly of India established in 1946 was
(A) Rajendra Prasad (B) Sachidananda Sinha(C) Rajagopalachari (D) B.R. Ambedkar

55. The Constitution of India was adopted on


(A) 26 November 1949 (B) 26 January 1950 (C) 26 November 1947 (D) 26 January 1949

Previous Years
CLAT & AILET Papers Page 157
56. The film that won 14 Oscar nominations was
(A) Gandhi (B) Slumdog Millionaire (C) Benhur (D) All about Eve

57. The first Indian to get a medal at the World Boxing Championship was
(A) Jayadev Bisht (B) Suvanjoy Singh (C) Vijender Singh (D) Rajkumar Sangwan

58. At the Beijing Olympics, highest number of medals were won by


(A) China (B) USA (C) Russia (D) Germany

59. If the Anglo-Indian Community is not adequately represented in the election, the President can
nominate to Lok Sabha from that community
(A) Not more than 5 (B) Not more than 2 (C) Not more than 1 (D) None

60. The right to vote in India is a


(A) Constitutional right (B) Legal right (C) Fundamental right (D) None

61. The maximum fine that the Central or State Information Commission can impose on Public Information
Officer for not furnishing the information within the stipulate time is
(A) Rs. 10,000/- (B) Rs. 15,000/- (C) Rs. 25,000/- (D) Rs. 30,000/-

62. Maximum number of gold medals won by an individual at Olympics in any event is
(A) 3 (B) 5 (C) 7 (D) 8

63. The person who promoted the establishment of the International Committee of the Red Cross was
(A) Charles Dickens (B) Henry Dunant (C) Henry Ford (D) Robert Frost

64. The first African to become the Secretary General of United Nations was
(A) Kofi Annan (B) Dag Hammarskjold
(C) Boutros Boutros Ghali (D) U Thant

65. The present UN' Secretary General Ban Ki Moon is from


(A) South Korea (B) North Korea (C) China (D) Mongolia

66. Who said that Swaraj is my birth right?


(A) M.K. Gandhi (B) Balagangadhar Tilak
(C) Jawaharlal Nehru (D) Mohammed Ali Jinnah

67. Which of the following states got separated from India in 1937 ?
(A) Pakistan (B) Sikkim (C) Sri Lanka (D) Burma

68. The first woman Prime Minister was


(A) Indira Gandhi (B) Srimavo Bandaranaike
(C) Golda Meir (D) Elizabeth Domitien

69. The largest country in terms of territory is


(A) Russia (B) Canada (C) USA (D) China

Previous Years
Page 158 CLAT & AILET Papers
70. The phrase 'cloud computing' means
(A) Delivery of hosted services over the internet
(B) Study of clouds to predict rains and thunderstorms
(C) Study of conditions of clouds for 'cloud seeding'
(D) Use of different software for financial accounting

SECTION – C : LEGAL APTITUDE


71. PRINCIPLE:
An assault is an attempt to do a corporeal hurt to another, coupled with an apparent present ability
and intention to do that act. A battery is the intentional and direct application of any physical force
to the person of another.

A was sitting on a chair reading a book. His friend, B decided to play a practical joke on him.
Accordingly, he pulled the chair from under him, as a result of which A landed on the floor.
(A) B's act amounts to a battery.
(B) B's act amounts to an assault.
(C) B' s act amounts to an assault till the time A lands on the floor.
(D) B' s act amounts to neither because there was no intention.

72. PRINCIPLE:
False imprisonment is a total restraint of the liberty of a person, for how so ever short period of time,
without lawful excuse. (Common for Q. No. 72 and 73)

A was driving down a road heading to her house. As she reached close to her house, she found that
a few people led by B, protesting against an unfair law had blocked the road. There was no alternate
road to her house and hence she was stuck there for around 5 minutes.
(A) B and his group are liable for having falsely imprisoned A.
(B) B and his group are not liable for falsely imprisoning A, since they were exercising their right to
protest.
(C) B and his group are not liable for falsely imprisoning A, since they did not totally restrain the
liberty of A.
(D) B and his group are not liable for falsely imprisoning A, since 5 minutes is too short a time.

73. A was suspected of having committed the murder of B. C, a policeman who was investigating into B'
s murder, saw A in a market. He went up to him, caught hold of his hand and prevented him from
going anywhere.
(A) C is liable for having falsely imprisoned A, since to arrest a person, a policeman requires
permission from a Magistrate.
(B) C is not liable for having falsely imprisoned A, since for the offence of murder, a policeman need
not take the permission of a Magistrate to arrest.
(C) C is not liable for having falsely imprisoned A since the restraint was not total.
(D) C is not liable for having falsely imprisoned A since he did not take him to a prison.

Previous Years
CLAT & AILET Papers Page 159
74. PRINCIPLE: A libel is a publication of a false and defamatory statement tending to injure the
reputation of another person without lawful justification or excuse. A slander is a false and defamatory
statement by spoken words or gestures tending to injure the reputation of another.
A wrote a letter to B calling him a cheat. B' s clerk C opened the letter, as he normally did (a fact which
was known to A) and placed it on B's table. B alleges that A has committed libel
(A) B will succeed since A has published a defamatory statement against A
(B) B will not succeed because it was not written in a newspaper
(C) B will not succeed because everyone knew that A was a cheat
(D) B will not succeed since A did not follow up the letter with a speech

75. PRINCIPLE:
A Master is liable for the acts of his Servant as long as he can control the working of his servant.

A owned a taxi agency. She had hired B to drive one of her cars. On January 1, 2010, C called up A's
taxi agency and asked for a car to drop him from his house to his place of work. On the way,
because of the driver's negligence, the car hit a road divider and C was injured. He sued A for
damages.
(A) A is not liable because A was not driving the car
(B) A is not liable because A was not in the car
(C) A is liable because B was employed be her and was in her control
(D) A is not liable because B was driving as per C's instructions

76. PRINCIPLE: A Master is liable to third persons for every such wrong of his servant as committed in
the course of service. For acts committed beyond the scope of employment, the master is liable
only if he has expressly authorised the act. (Common for Q. No. 76, 77 and 78)

A owned a bus and he had hired B to drive it and C to be the conductor. One day, when B had
stepped out of the: bus to have a cup of coffee, C decided to turn the bus around so that it was ready
for its next trip. While doing so, C ran over D's leg, causing major injuries to him. D sued A for
damages.

(A) D will succeed since C was employed by A


(B) D will not succeed since A had not authorised C to drive the bus
(C) D will not succeed since the bus was not on an official trip
(D) D will succeed since turning the bus was in the course of employment

77. A owned a truck and he had hired B to drive it. On one of its trips, C flagged the truck down and
asked to be dropped to a nearby city. B agreed to do so for a small amount of money. The truck met
with an accident en route, in which C was badly injured. C sued A for damages.

(A) A will be liable since B did the act in the course of his employment
(B) A will be liable since he had not instructed B not to pick up passengers
(C) A will nor be liable since taking passengers in a truck had no connection with his business
(D) A will not be liable because B was an experienced driver

Previous Years
Page 160 CLAT & AILET Papers
78. A handed over Rs. 5,000 to her neighbour B, who was an employee of a bank, and asked him to
deposit the money in her account. Instead of doing so, B spent the money. A sues the bank for
damages.

(A) The bank will be liable since B was its employee


(B) The bank will not be liable since B was not authorised to collect money from A
(C) The bank will not be liable since A gave money to B in his capacity as a neighbour and not as an
employee of the bank
(D) The bank will not be liable since this is a criminal act

79. PRINCIPLE:
When a criminal act is done by one person in furtherance of the common intention of himself and
some other persons, each of such person is liable for the act in the same manner as if it were done
by each one of them.

A, B and C decided to commit burglary. They broke into a locked house. However, before they had
finished their work, a domestic servant appeared from the out-house and started shouting. A, B and
C left the house and started running away. They were pursued by a small crowd. A, on being caught
by X, one of the persons pursuing them, stabbed him and ran away. By the time B and C had
disappeared. X died on account of the stab wounds. Later, the police arrested all the three. They
were charged for attempted burglary and murder of X.

(A) Along with A (the person who stabbed X) B and C are also guilty of murder because A stabbed X
in furtherance of common intention.
(B) Along with A, B and C are also guilty of murder because A, at the time of stabbing X, was acting
on behalf of B and C and he wanted to save not only himself but B and C as well.
(C) A alone is guilty of murder because though there was common intention to commit the offence
of burglary, there was no common intention to commit the offence of murder.
(D) None of the above

80. PRINCIPLE:
A person is liable for any damage which is the direct consequence of his/ her unlawful act, as long
as the consequence could have been foreseen by a reasonable person.

During a scuffle, A knocked B unconscious and then placed B at the foot of a hill at night, when the
temperature was around one degree centigrade. B suffered from hypothermia and had to be
hospitalised for a week. B sues A.
(A) B will be liable for the act, since the consequence was a direct and foreseeable consequence of
his act
(B) B will not be liable because he had no intention of causing hypothermia
(C) B will not be liable because he did not know that A would suffer from hypothermia
(D) B will be liable since A was hospitalised

Previous Years
CLAT & AILET Papers Page 161
81. PRINCIPLE: The owner of a land is entitled to the column of air space above the surface ad infinitum.
But, the right is restricted to such height as is necessary for the ordinary use and employment of his
land, and the structure on it.

A had constructed a single-storeyed house on a corner site. He had no intention of building an


additional floor. B his neighbour, who ran an internet parlour got a hoarding made, which protruded
over A's house at a height of around 6 feet above the terrace. A sues B for trespass.
(A) A will succeed since B' s act amounts to trespass
(B) A will not succeed since he was anyway not planning to build an additional storey
(C) A will not succeed since the hoarding is not obstructing him
(D) A will not succeed since B has a right to erect a hoarding

82. PRINCIPLE:
Trespass to land is the wrongful and unwarranted entry upon the land of another.
A purchased a ticket to watch a movie in a theatre. After the show got over, A refused to leave the
theatre. The owner of the theatre sues A for trespass
(A) He will not succeed since A had purchased a ticket
(B) He will not succeed since A has a right to remain wherever he wants to after the show gets over
(C) He will succeed since A's authorisation to remain in the theatre ends with the show
(D) He will not succeed since A did not have the intention to trespass

83. PRINCIPLE:
Negligence is the breach of a duty caused by an omission to do something which a reasonable
person would do or an act which a prudent and reasonable person would not do. An action for
negligence proceeds upon the principle that the person has an obligation or duty on the part of the
defendant, which he/she breaches, leading to damage.
A, a surgeon operated on B. Subsequent to the operation, B complained of pain in his abdomen. On
examination, it was discovered that A had left a mop in B' s stomach while operating.
(A) A will be liable for negligence, since there is a breach of the duty of care
(B) A will not be liable for negligence since it was an accident
(C) A will not be liable for negligence since no real damage was done
(D) A will not be liable for negligence since it is not a reasonable act

84. PRINCIPLE:
A contract requires a proposal and an acceptance of the proposal. It is necessary to make a binding
contract, not only that the proposal be accepted, but also that the acceptance is notified to the
proposer. (Common for Q. No. 84, 85 and 86)

A sent a letter to B stating that he was willing to sell to B, 10 bags of rice at Rs. 20/- each. B made
telephone call to A to inform him, that he had accepted the offer. Just as he was stating his acceptance,
a very noisy aircraft flew over B’ s house. A was not able to hear the acceptance.
(A) There is no contract since the acceptance has not been notified to A
(B) There is no contract since the reply also has to be in writing
(C) This is a valid contract since B conveyed his acceptance to A. It does not matter if A heard it or
not
(D) None of the above

Previous Years
Page 162 CLAT & AILET Papers
85. A sent a letter to B stating that he was willing to sell to B, 10 bags of rice at Rs. 20/- each. B wrote
a letter to A accepting the offer and posted it,
(A) The contract is entered into the moment B posts the letter
(B) The contract is entered into only after A receives and reads the letter
(C) The contract is entered into only after A receives the letter, reads it and then gets back to B
(D) The contract is entered into the moment A makes the offer

86. A sent a letter to B stating that he was willing to sell to B, 10 bags of rice at Rs. 20/- each. B sent
an e-mail to the address mentioned in the letter-head, accepting the offer.
(A) The contract is entered into the moment the mail reaches A’ s server
(B) The contract is entered into only after A sees the e-mail
(C) The contract is entered into when A sent the letter
(D) The contract is entered into the moment B receives an automated delivery receipt

87. PRINCIPLE:
An agreement becomes a contract when it is entered into between two or more people with each
other's free consent. Two or more people are said to consent when they agree to the same thing in
the same sense. Consent is said to be free when it is not caused by coercion, undue influence,
fraud, misrepresentation or mistake.

A went to a doctor to get himself treated. The doctor asked him to sign a “consent form” and said
that unless he signs it, he will not examine him. A signed the form waiving his right to sue the doctor.

(A) This is a valid contract since A understood and consented to the terms
(B) This is not a valid contract since A had not other option, but to sign
(C) This is a valid contract since it is a doctor-patient relationship
(D) None of the above

88. PRINCIPLE:
Where both parties to an agreement are under a mistake as to matter of fact essential to the
agreement, the agreement is void.

A had a piece of land. He believed that the value of the land was Rs. 1,000 per square foot. B knew
that the value of the land was infact Rs. 1,500 per square foot. However, he did not inform A and
purchased the land at Rs. 1,000.
(A) The contract is valid since A should have been more careful
(B) The contract is valid since B paid for the land .
(C) The contract is void since A was mistaken about the cost of the land
(D) The contract is valid since the land has already been purchased

89. PRINCIPLE:
The object of an agreement is lawful unless it is forbidden by law; is of such a nature that, if
permitted, it would defeat the provisions of any law; is fraudulent; involves or implies injury to the
person or property of another person; the court regards it as immoral; it is opposed to public policy.
(Common for Q. No. 89, 90, 91 and 92)

A and B entered into a contract, whereby A agreed to get married to B, if her parents paid A Rs.
1,00,000 before the wedding. B' s parents failed to pay the promised amount. A sues B and her
parents.
Previous Years
CLAT & AILET Papers Page 163
(A) A will succeed because all the requirements of a contract are met
(B) A will succeed since the payment of the amount was the condition precedent for the wedding
(C) A will not succeed since the contract is void, its object being against the law
(D) A will not succeed since B was not a party to the contract

90. A was arrested by the police for committing an offence and was subsequently granted bail by the
Court. One of the conditions imposed by the Court for granting bail was that A arrange for a surety
for Rs. 50,000. A approached B to be a surety. B agreed, but insisted that they enter into a contract
whereby A would deposit Rs. 50,000 into B's account, which would be returned to A by B after the
case was over.
(A) The contract would be void since its effect is to defeat the provisions of the Cr. P.C.
(B) The contract would be void because A cannot enter into a contract when a criminal case is going
on against him
(C) The contract would be void because the Court is not a party to it
(D) The contract would be valid

91. A and B, a Hindu couple were married to each other. Owing to differences between them, they
decided to get divorced. They entered into a contract laying down the conditions that both parties
had to adhere with. One of the terms of the contract was that their children would not be entitled to
claim the ancestral property of A, the husband.
(A) The contract will be void since the children are not a party to the contract
(B) The contract will be void since its terms since it is opposed to the personal laws of the parties
(C) The contract will be valid
(D) The contract will be void since a woman cannot enter into a contract with her husband

92. The BCCI decided to hold an auction to sell IPL teams. 12 bidders registered for the auction.
Unknown to the BCCI, these 12 bidders had entered Into a contract that they would not bid more
than a certain amount.
(A) The contract will be void because BCCI is not a party to it
(B) The contract will be void since the object of the contract is to cause a loss to the BCCI
(C) The contract will be void since the maximum number of parties to a contract as per Indian law is
10
(D) The contract will be valid

93. PRINCIPLE:
Every agreement by which anyone is restrained from exercising a lawful profession, trade or business
of any kind, to that extent is void.

A company entered into a contract with Coca Cola Ltd. to bottle soft drinks produced by Coca Cola.
One of the terms of the contract was that the company would not bottle soft drinks for any other
competitor of Coca Cola during the pendency of the contract.
(A) The contract is void since it restrains the lawful right of the company to engage in business
(B) The contract is void because there is no consider ration being paid
(C) The contract s valid
(D) The contract is valid since the competitor is not a party to the contract

Previous Years
Page 164 CLAT & AILET Papers
94. PRINCIPLE:
An agreement is void to the extent that it restricts absolutely, a party from enforcing his contractual
rights by usual rights by usual proceedings in any ordinary court.

A and B entered into a transaction in Delhi for sale of goods based in Delhi. The contract stated that
in case of a dispute, only civil courts in Chennai would have jurisdiction. Neither A, nor B resided or
carried on business in Chennai.
(A) The contract is valid since it is still an Indian court that has jurisdiction
(B) The contract is void since it ousts the jurisdiction of the competent court
(C) The contract is void since the consent of the court is required before entering into such a
contract
(D) The contract is valid since Chennai courts have expertise with respect to contractual matters

95. PRINCIPLE:
An Agreement, the meaning of which is not certain, or capable of being made certain, is void.
A and B, who were brothers, entered into an agreement which stated that A would sell his entire
share of the ancestral property to B. The agreement did not mention the specific details (like survey
number) of the property.
(A) The contract will be void since it is vague
(B) The contract will be void because it is not possible to ascertain the property which is being
referred to
(C) The contract will be void since it is illegal
(D) The contract is valid since there is clarity about the property sought to be sold

96. PRINCIPLE:
Whoever intending to take dishonestly (with an intention to cause wrongful loss to another or wrongful
gain to himself) any moveable property without that person's consent moves that property in order to
such taking, is said to commit theft. (Common for Q. No. 93 and 97)

A had lent his watch to B for a month. Two days after he had done so, he walked into B' s office to
find the watch on B' s table. He decided to take the watch back. A was prosecuted for theft.
(A) A is not guilty of the offence since he owned the watch.
(B) A is not guilty of the offence since he was not doing the act dishonestly.
(C) A is guilty of the offence since he caused wrongful loss to B.
(D) A is not guilty of the offence since he had lent the watch to B only for a short period of time.

97. A handed over his watch to B for safekeeping. B sells the watch to C, which he was not authorised
to do. B is prosecuted for theft.
(A) B is guilty of theft because he gained from the act.
(B) B is not guilty of theft because he was in possession of the watch.
(C) B is not guilty of theft because A had given him the watch.
(D) B is guilty of theft because caused wrongful loss to A.

Previous Years
CLAT & AILET Papers Page 165
98. PRINCIPLE:
Whoever intentionally puts any person in fear of any injury to that person, or to any other, and
thereby dishonestly induces that person so put in fear to deliver to any person any property, commits
extortion.
A entered B' s house, caught hold of B' s daughter C and threatened to stab her if B did not give him
Rs. 10,000 immediately. B did so. A is prosecuted for extortion.
(A) A has committed theft, not extortion.
(B) A has committed extortion.
(C) A has committed both theft and extortion.
(D) A has not committed any offence.

99. PRINCIPLE:
Theft is robbery if in order to committing of the theft, or in committing the theft, or in carrying away
or attempting to carry away property obtained by theft, the offender, for that end, voluntarily causes
or attempts to cause to any person death or hurt or fear of instant death or instant hurt. (Common
for Q. No. 99 and 100)

A entered B's house to take away her TV. When he was carrying the TV out of the house, he
encountered B near the door. He left the TV behind and ran away.
(A) A has committed theft.
(B) A has committed robbery.
(C) A has committed both theft and robbery.
(D) A has neither committed theft nor robbery.

100. A entered B' s house and was taking away her wallet and leaving the house, when he encountered
B. He dropped the wallet, but shot her while escaping
(A) A has committed robbery.
(B) A has committed theft, but not robbery.
(C) A has neither committed theft, nor robbery.
(D) A has committed both- theft and robbery.

101. RINCIPLE:
Whoever with the intent to cause, or knowing that he is likely to cause wrongful loss or damage to
the public or to any person, causes the destruction of property, or any such change in any property
or in the situation thereof as destroys or diminishes its value or utility, of affects it injuriously,
commits mischief. (Common Q. No. 101 and 102)

A went to B' s house for dinner. He saw that B had a statue made of ice, which was kept in a freezer,
A removed the statue from the freezer for a few minutes, which led to the hand of the statue
changing shape.
(A) A has not committed mischief, since he did not have the intention to change the shape of the
hand of the statue.
(B) A has committed mischief because he should have known that his act was likely to damage the
statue.
(C) A has not committed mischief because the statue did not get destroyed.
(D) A did not commit mischief because the value of the statue has not diminished.

Previous Years
Page 166 CLAT & AILET Papers
102. A lent his laptop to B. When in possession of the laptop, B inserted a pen drive into the laptop.
When he did a virus scan, he realised that the pen drive was infected. Since he urgently required a
file that was on the laptop, he nevertheless opened the files on the pen drive, in the process infecting
the laptop .
(A) A has committed mischief because he has affected the laptop injuriously.
(B) A has not committed mischief because he did not intend to do so.
(C) A has not committed mischief because the laptop continued working.
(D) A has not committed mischief because the laptop was in his possession.

103. PRINCIPLE:
Whoever enters into or upon property in the possession of another with the intent to commit an
offence or to intimidate, insult or annoy any person in possession of such property or having lawfully
entered into or upon such property, unlawfully remains there with an intent to intimidate, insult or
annoy any such person, or with the intent to commit an offence commits criminal trespass.

A went to Delhi Law College to participate in a competition. After participating in the competition, A
hid inside the campus so that he could steal a few books from the Iibrary,
(A) A has committed theft.
(B) A has committed criminal trespass.
(C) A has not committed criminal trespass, because he entered with permission.
(D) A has not committed any offence.

104. PRINCIPLE: A second suit will not be heard on the same facts between the same parties.
(Common for 104 and 105)

Nakshatra files a suit against Chandra for getting possession of a house on the ground: that the
property passed on to her through the will executed by Surya before his death. The suit gets
dismissed as Nakshatra fails to produce the will. Nakshatra files another suit against Chandra to
get the same house from the latter, on the ground that she was entitled to the house as being the
nearest heir of Sljfya.
(A) Nakshatra will succeed as she is the nearest heir of Surya '
(B) Nakshatra will not succeed as the facts in both the cases are the same
(C) Nakshatra will succeed as the facts in the two cases deal with different situations
(D) Nakshtra will succeed as the grounds in the second case were not taken in the' first one owing
to the mistake of her advocate

105. Mr. Sampath went to a party alone in his wife Sunanda's car. He usually used his wife's car after
office hours and his wife never objected to it. At the party, he got drunk. Instead of taking the risk of
driving the car, he requested his friend Mr. Vivek to drive him back home in Sunanda's car. Mr. Vivek
was quite sober since he had moderately consumed alcohol. On the way, Vivek knocked down a
boy and injured his leg. Subsequently, on behalf of the boy a claim for compensation was brought
against Mrs. Sunanda since the car belonged to her and it was registered in her name. The insurance
company refuses to pay compensation because the police report says that the person driving the
car at the time of accident had consumed alcohol.
(A) Sunanda is liable to pay compensation, although she did not authorize Mr. Vivek to drive the car
which caused the accident, yet it was registered in her name
(B) Sunanda is not liable to pay compensation because it is the liability of Mr. Sampath who had
permission to drive the car and on the basis of this permission, he requested Mr. Vivek who was
driving the car at the time of the accident

Previous Years
CLAT & AILET Papers Page 167
(C) Mr. Vivek is liable to compensate the boy fully because he was negligent in driving the car
(D) Sunanda, Sampath and Vivek are all jointly liable to compensate the boy.

SECTION – D : LOGICAL REASONING

106. If it is true that 'men are seldom honest' , which of the following statement conveys the same
meaning?
(A) At least one man is not honest (B) Men are not honest
(C) At least on one occasion men are not honest (D) Sometimes men are honest

107. If it is false that 'men always obey the law', which one of the following statements is doubtful ?
(A) Sometimes men do not obey the law (B) All men obey the law
(C) Men never obey the law (D) Some men obey the law

108. lf it is false that 'all men strive for the happiness of all', which one of the following statements is true
beyond doubt ?
(A) All men seek their own happiness
(B) There are men who do not desire for the happiness of all
(C) No men strive for the happiness of all
(D) There are men who do not strive for the happiness of all

109. From among the given alternatives, identify the missing premise.
1st Premise : If a plane triangle is possible, then perfect straight line is possible
2nd Premise : If perfect straight line is possible, then Euclid is right
3rd Premise :
4th Premise : But Riemann is right.

Conclusion: Therefore a plane triangle is not possible.


(A) If Riemann is right, then Euclid is right (B) If Riemann is wrong, then Euclid is right
(C) If Euclid is right, then Riemann is wrong (D) If Riemann is right, then Euclid is not wrong

110. From among the given alternatives, identify the conclusion of the following argument.
If A is B, then C is not D.
If C is not D, then E is not F.
If E is not F, then G is not H.
Therefore
(A) If G is not H, then A is B (B) If A is B, then G is not H
(C) If G is H, then A is B (D) If A is not B, then G is H

111. From among the given alternatives, identify the conclusion of the following argument.
You cannot taste success in life unless you work hard.
You cannot be happy unless you taste success in life.
Therefore
(A) You can be happy unless you work hard.
(B) You cannot be happy unless you work hard.
(C) You cannot work hard unless you are happy.
(D) You can work hard unless you are happy.

Previous Years
Page 168 CLAT & AILET Papers
112. If the statement "All men desire their own welfare always" is true, then which of the following
statements must be false?
(A) There are men who desire their own welfare sometimes
(B) Some men desire their own welfare always
(C) There are men who always desire their and others' welfare
(D) It is not the case that all men desire their own welfare always

113. From among the alternatives, identify the statement which states the same fact as the statement
'The dual nature of light is an enigma' .
(A) The nature of light is an enigma.
(B) Light has some enigmatic properties.
(C) Two differing arguments regarding the nature of light are obviously wrong.
(D) It is surprising and puzzling that according to one study, light is made up of one type of
constituents and according to another, light is made up another type of constituents.
114. Identify the statement which can be neither proved nor disproved.
(A) I have head-ache.
(B) Yesterday, New York recorded 20°C at 12 noon.
(C) The gang war that broke out last week was due to business rivalry.
(D) History of Kannada literature states that Nripatunga is the earliest poet.

115. Identify the statement which cannot be doubted.


(A) Sense experience provides true knowledge.
(B) It is reasonable to accept science and most unreasonable to believe any religion.
(C) I am alive.
(D) Whenever there is unrest in society, the police will be alert.

116. If it is true that 'good governance ensures law and order', then identify the statement which has to
be accepted along with the given statement.
(A) A healthy society is governed by maximum number of laws.
(B) A strong government uses force to enforce law.
(C) An able government very effectively uses the law to promote peace and prosperity.
(D) Law is indispensable for the society.

117. Identify the statement which cannot be accepted.


(A) Mahabharatha is an ancient epic.
(B) Many Indian languages have their origin in Sanskrit.
(C) According to the heliocentric theory, the Earth is the centre of the solar system.
(D) Indian Constitution has been amended many times.

118. If it is true that 'religious fundamentalism is dangerous to society’, which one of the following statements
is also true?
(A) Religion without reason is dangerous to the society.
(B) Belief in religions is dangerous to the society.
(C) Religious pluralism is dangerous to the society.
(D) Disrespect for other religions is dangerous to the society.

Previous Years
CLAT & AILET Papers Page 169
119. Identify the argument which has to be accepted when the premiss is accepted as true.
(A) A and B are subsets of C.
Therefore A and B must have at least one common element.
(B) Every man desires happiness.
Therefore happiness is desirable.
(C) God is not perceivable.
Therefore God does not exist.
(D) Nothing comes out of nothing.
Therefore something must have existed always.
120. Identify the argument which cannot be accepted.
(A) All Indians are Asians.
All Asians are dark-coloured.
Therefore All dark coloured are Indians.
(B) Amar is a friend of Basava.
Therefore Basava is a friend of Amar.
(C) All positive integers less than 100 have only 2 digits.
Therefore all positive integers with 2 digits are less than 100.
(D) All spinsters are unmarried women.
Therefore all unmarried women are spinsters.

121. Till Nineties, clearly the policy of the government was guided by the philosophy of socialism. Under
its influence, the government tried to exercise control in every field. But today we hear about
disinvestment and privatization. Hence, there is a clear shift from socialistic principles.
Which one of the following assertions, if true, contradicts the passage mentioned above?
(A) That government is the best government which governs least.
(B) Only enterprise in a free society contributes to the real progress.
(C) Not withstanding the alleged merits of laissez-faire implicit in the passage, lack of effective.
control by the state results in the exploitation of many by a few.
(D) The state is guilty of the dereliction of duty.

122. Geographical division of the Earth resulting in the so-called sovereign nations is artificial because it
is man-made, just as division of society on the basis of religion, language, caste, etc., is artificial.
While the so-called intellectuals raise hue and cry about the latter, they maintain divine silence
about the former. Does it not amount to double standard ?
Which of the following statements, if true, is the essence of the passage mentioned above?
(A) Geographical division of the Earth is of divine origin.
(B) Only division of society on the basis of religion, language, etc., is man-made.
(C) The response to geographical and sociological divisions are very different.
(D) Patriotism is a barren concept.
[Option C in the original paper was “The consequences of geographical and sociological divisions are very different”. As none
of the options are matching with the question, option (C) has been changed to fit the sprit of the question.]

123. It is argued that prosperity depends upon production which means more production more
consumption. However, it is not the case. Consumption should be need based. Just as intake of
food in excess of digestion is pointless so also consumption of natural resources beyond requirement
is pointless.

Previous Years
Page 170 CLAT & AILET Papers
Which of the following statements, if true, contradicts the spirit of the passage?
(A) Prosperity without consumption is ideal.
(B) Prosperity without consumption is shallow.
(C) Prosperity is possible when natural resources are not abused.
(D) To increase consumption we cannot have any alibi.

124. It is true that agriculture is the backbone of a nation's economy. But it is fallacious to argue that only
agriculture should receive overriding importance. Agriculture also depends upon other sectors like
generation and distribution of power, irrigation, transportation, etc. If any one sector suffers, its fall-
out is more than obvious.

Which of the following assertions, if true, is the summary of the given passage?
(A) Budget allocation for all fields must be equal.
(B) If adequate budget allocation in any one fiscal year for every sector is not possible, then each
year one sector can be provided with adequate funds.
(C) Budget allocation for each sector should be need based, neither less nor more, i.e.it should be
optimum.
(D) Reversion to ancient method of farming is the only way out.

125. Education is one field which is totally non-remunerative. Still worse, it is a liability. But then the
government is not a money-making institution. The bottom-line of prosperity is primary education.
Hence the government should raise resources elsewhere only to pour it on primary education. But
primary education cannot be provided without higher education. So higher education stands on par
with primary education.

Which of the following statements, if true, contradicts the passage above?


(A) Money should produce money which means that that which does not generate money has no
place.
(B) Education should raise its own resources.
(C) Education should be made as economical as possible.
(D) If the government cannot spread education, private enterprise should step in.

126. 4 statements are given below. Group two of them in such a way that one is logically impossible and
the other is empirically (factually) possible.
1. A plane equilateral triangle can also be a plane right angled triangle. (A)
2. Velocity of light in vacuum is constant. (B)
3. Consumption of fuel at this rate results in reversal to bullock-cart age. (C)
4. A tall man is a man. (D)
(A) A,B (B) B,D (C) C,D (D) A,C

127. 4 statements are given. Group two of them in such a way that one is logically possible and the other
is empirically (factually) highly improbable.
1. No theories of science can explain the origin of the Universe satisfactorily. (A)
2. Solar energy is inexhaustible. (B)
3. Liberalisation and rise in consumerism are directly related. (C)
4. God exists. (D)
(A) A,B (B) A,C (C) B,D (D) A,D

Previous Years
CLAT & AILET Papers Page 171
128. Four statements are given below. One of them can be demonstrated with the help of reason, while
another requires sense experience to ascertain the truth, Identify them.
1. Ravana killed Rama. (A)
2. Area of circle is given by the formula πr2 (B)
3. Methane is poisonous (C)
4. The absolute is pure consciousness (D)

129. How do you evaluate the statement "Autobiography of Mr M.K. Gandhi" is written by Mr. Mahadeva
Desai?
(A) False (B) True (C) Probable (D) Absurd

130. All faculties of understanding, which a human being enjoys, are necessarily limited in terms of
performance. If so, which of the following alternatives is justified by the preceding statement ?
(A) Human knowledge is necessarily imperfect.
(B) Perfection one's own knowledge is a distinct possibility, but man does not know how to employ
his faculties.
(C) When progress in knowledge is linear, one day or the other the ultimate truth will be discovered.
(D) Human knowledge is not necessarily perfect.

131. 'The Universe has necessarily a beginning and an end' and 'the Universe has boundary' together
imply
(A) The universe is finite in space and time.
(B) The universe is infinite in space and time.
(C) The universe is infinite in space but not in time.
(D) The universe is finite in space but not in time.

132. Which of the following alternatives comes very close to the statement, 'Man is necessarily an evil by
nature'?
(A) As a matter of fact, man is an evil by nature.
(B) Man is not an evil by nature, but circumstances tend to make him so.
(C) Man cannot be an evil by nature.
(D) Actually, man is not an evil.

133. Which of the following alternatives is farthest from the statement, 'Sound waves cannot travel in
vacuum' ?
(A) Sound waves require some medium to travel.
(B) Sound waves may travel in vacuum very slowly.
(C) Sound waves can travel only in vacuum.
(D) Sound waves may not travel in vacuum.

134. Which of the following alternatives contradict the statement 'deterministic laws of physics are
characteristically Newtonian'.
(A) Indeterminism and physics cannot go together.
(B) Indeterminism is inherent in Newtonian system.
(C) Determinism in physics is a flaw.
(D) Determinism in physics is only apparent.

Previous Years
Page 172 CLAT & AILET Papers
135. Identify the alternative which best explains the relation between the arguments mentioned below:
Arguments:
1. It is highly probable that every student of law at University level would have done a course on
principles of ethics and Rama being a student of law, in all probability, would have done a
course on principles of ethics.
2. At University level, every faculty aims at super speciality for which emphasis is laid upon structuring
the curricula in such a way that every faculty is treated as a closed system so that no inter-
disciplinary study is acceptable. Principles of ethics being essentially an integral part of
philosophy, most probably is out of any course on law.
(A) Two arguments are mutually independent.
(B) Two arguments are mutually supportive.
(C) Two arguments are supplementary.
(D) They are incompatible.

136. 'Greater than' is said to be transitive (relation) since A is greater than B and B is greater than C imply
A is greater than C. Similarly 'friend of' is said to be symmetric since if A is a friend of B, then B is
a friend of A. If so, what kind of relation is 'analogous'?
(A) Transitive (B) Symmetric
(C) Identical (D) Transitive and symmetric

137. Since an object is identical with itself ‘being identical’ is called a reflexive relation and if a relation is
transitive, symmetric and reflexive, then such a relation is called 'equivalence'. If so, what kind of
relation is 'different'?
(A) Equivalence (B) Inequivalence (C) Asymmetric (D) Transitive

[Option D in the original paper was “Non-Transitive”. As both options (B) and (D) are matching with the question, option (D)
has been changed to “Transitive” to fit the question.]

138. Which of the following having the same form contradicts the statement 'guilty should be punished' ?
(A) Guilty will not be punished (B) Guilty may not be punished
(C) Guilty are not punished (D) Guilty should not be punished

139. If it is true that 'no innocent persons should be punished' and' Socrates is innocent', which of the
following is the correct conclusion?
(A) Socrates is not punished (B) Socrates may not be punished
(C) Socrates should not be punished (D) Socrates will not be punished

140. Identify the alternative which gives true picture of the relation between the arguments listed below:
1. A healthy society is characterised by overall development of every section. As in the case of
human body no organ is less prominent, so also in a society no section is less important. If any
human organ is sick, then the human being is sick. Similarly, if any section of society is sick,
then the society is sick.
2. It is fallacious to think that priority to production of agricultural products alone promotes the
prosperity of farmers, Prosperity has too many discussions. Transportation, communication,
network, sound economic policy, to mention a few are equally important to the economic wellbeing
of farmers. Hence they also deserve consideration.
(A) These two are irrelevant (B) They are interdependent
(C) They are analogous (D) They are compatible

Previous Years
CLAT & AILET Papers Page 173
SECTION – E : MATHEMATICS

141. Which one of the following is same as 30% of 40% of 560?


(A) 60% of 40% of 280
(B) 15% of 80% of 280
(C) 30% of 40% of 280
(D) 15% of 80% of 140

 
= , then the value of  4 + 2y − x  is
x 4
142. If
y 5  7 2y + x 
3 8
(A) (B) 2 (C) 1 (D)
7 7

143. The average height of 10 students in a class is 105 cms. If 20 more students with an average height
of 120 cms join them, what will be the average height be?
(A) 105 cms (B) 110 cms (C) 112 cms (D) 115 cms

144. Anil is half his father's age now. Fifteen years ago, he was one-third of his father's age. How old will
Anil be after five years?
(A) 45 years (B) 40 years (C) 35 years (D) 30 years

145. 10 men can finish the construction of a hut in 8 days. How many men are needed to finish the same
in half a day?
(A) 80 (B) 100 (C) 120 (D) 160

146. India's Republic day in 1996 fell on Friday. Which week day it will be celebrated in the year 2000?
(A) Tuesday (B) Wednesday (C) Thursday (D) Friday

147. An express train travels at an average speed of 120 km per hour, stopping for 4 minutes after every
80 kms. How long will it take the train to reach a destination 960 km away?
(A) 8 hrs 40 mts (B) 8 hrs 44 mts (C) 8 hrs 48 mts (D) 8 hrs 56 mts

148. In a family, the father took one-fourth of the cake and he had three times more than the others had.
The total number of family members is
(A) 3 (B) 7 (C) 10 (D) 12

149. A clock gains 5 minutes every hour. Then the angle traversed by the seconds hand in one minute will
be
(A) 390° (B) 380° (C) 360.5° (D) 360°

150. A right circular cone and a right cylinder have the same radius and the same volume. The ratio of the
height of the cone to that of the cylinder is
(A) 5 : 3 (B) 5 : 2 (C) 1 : 3 (D) 3 : 1

Previous Years
Page 174 CLAT & AILET Papers
AILET Question Paper 2011
SECTION - A : ENGLISH

Read the passage and answer the questions following it. The questions are to be answered on
the basis of what is stated or implied in the passage. For some of the questions, more than one of
the choices could conceivably answer the question. However, you are to choose the best answer;
that is, the response that most accurately and completely answers the question.

Roger Rosenblatt’s book ‘Black Fiction’, manages to alter the approach taken in many previous studies by
making an attempt to apply literary rather than socio-political criteria to its subject. Rosenblatt points out
that criticism of Black writing has very often served as a pretext for an expounding on Black history. The
recent work of Addison Gayle’s passes a judgment on the value of Black fiction by clear political standards,
rating each work according to the ideas of Black identity, which it propounds.

Though fiction results from political circumstances, its author reacts not in ideological ways to those
circumstances, and talking about novels and stories primarily as instruments of ideology circumvents
much of the fictional enterprise. Affinities and connections are revealed in the works of Black fiction in
Rosenblatt’s literary analysis; these affinities and connections have been overlooked and ignored by
solely political studies.

The writing of acceptable criticism of Black fiction, however, presumes giving satisfactory answers to quite
a few questions. The most important of all, is there a sufficient reason, apart from the racial identity of the
authors, for the grouping together of Black authors? Secondly, what is the distinction of Black fiction from
other modem fiction with which it is largely contemporaneous? In the work Rosenblatt demonstrates that
Black fiction is a distinct body of writing, which has an identifiable, coherent literary tradition. He highlights
recurring concerns and designs, which are independent of chronology in Black fiction written over the past
eighty years. These concerns and designs are thematic, and they come form the central fact of the
predominant white culture, where the Black characters in the novel are situated irrespective of whether
they attempt to conform to that culture or they rebel against it.

Rosenblatt’s work does leave certain aesthetic questions open. His thematic analysis allows considerable
objectivity; he even clearly states that he does not intend to judge the merit of the various works yet his
reluctance seems misplaced, especially since an attempt to appraise might have led to interesting results.
For example, certain novels have an appearance of structural diffusion. Is this a defeat, or are the authors
working out of, or attempting to forge, a different kind of aesthetic? Apart from this, the style of certain
Black novels, like Jean Toomer’s Cane, verges on expressionism or surrealism; does this technique
provide a counterpoint to the prevalent theme that portrays the fate against which Black heroes are pitted,
a theme usually conveyed by more naturalistic modes of expressions?

Irrespective of such omissions, what Rosenblatt talks about in his work makes for an astute and worthwhile
study. His book very effectively surveys a variety of novels, highlighting certain fascinating and little-known
works like James Weldon Johnson’s Autobiography of an Ex-Coloured Man. Black Fiction is tightly
constructed, and levelheaded and penetrating criticism is exemplified in its forthright and lucid style.

Previous Years
Page 176 CLAT & AILET Papers
Answer the following questions indicating your response for each question:

1. The author of the passage raises an objection to criticism of Black fiction like that by
Addison Gayle as it
(a) Highlights only the purely literary aspects of such works.
(b) Misconceive the ideological content of such fiction.
(c) Miscalculate the notions of Black identity presented in such fiction.
(d) Replaces political for literary criteria in evaluating such fiction.

2. The primary concern of the author in the above passage is


(a) Reviewing the validity of a work of criticism.
(b) Comparing various critical approaches to a subject.
(c) Talking of the limitations of a particular kind of criticism.
(d) Recapitulation of the major points in a work of criticism.

3. The author is of the opinion that Black Fiction would have been improved had Rosenblatt
(a) Undertaken a more careful evaluation of the ideological and historical aspects of Black Fiction
(b) Been more objective in his approach to novels and stories by Black authors
(c) Attempted a more detailed exploration of the recurring themes in Black fiction throughout its
history
(d) Calculated the relative literary merit of the novels he analyzed thematically

4. Rosenblatt’s discussion of Black Fiction is


(a) Pedantic and contentious (b) Critical but admiring
(c) Ironic and Deprecating (d) Argumentative but unfocused

5. According to the given passage the author would be LEAST likely to approve of which
among the following?
(a) Analyzing the influence of political events on the personal ideology of Black writers
(b) Attempting a critical study which applies sociopolitical criteria to the autobiographies of Black
authors
(c) A literary study of Black poetry that appraises the merits of poems according to the political
acceptability of their themes
(d) Studying the growth of a distinct Black literary tradition within the context of Black history

6. From the following options, which does the author not make use of while discussing Black
Fiction?
(a) Rhetorical questions (b) Specific examples
(c) Comparison and contrast (d) Definition of terms

Each of the following sentences contain four parts, marked a, b, c, and d. One of these parts is
incorrect. Indicate the letter of the incorrect part.

7. The Baby showed a noticeable / distaste for / these kind / of prepared baby food.
a b c d

8. They cannot go camping right now / because / they are taking care of / a three-weeks-old baby.
a b c d

Previous Years
CLAT & AILET Papers Page 177
9. If it / will rain / this afternoon, we will have / to cancel the picnic.
a b c d

10. Are / you familiar of / the latest / scientific developments in the field?
a b c d

Direction: In the following questions, some of the sentences have errors and some have none. Find out
which part of the sentence has an error.

11. Men are apt to / pick up holes / in the views of those whom they do not like. / No error
a b c d

12. The myth that some races /are naturally superior than others /has now been fully demolished. /
a b c
No error
d

13. The steep rise in the price of petroleum products / have affected / the economic development of the
a b c
developing nations. / No error
d

14. Kiran as well as Kamal / are leaving / for Mumbai. / No error


a b c d

15. Much water / has flown / under the bridge since then. / No error
a b c d

16. Everyone of the men present here / has given a day’s pay / as their contribution to the fund / No error
a b c d

17. Found guilty / on murder / the accused was sentenced to death. / No error
a b c d

Complete the sentences with suitable words indicating your option for each question

18. I saw _____movie last night.


(a) the (b) a (c) an (d) one

19. The RBI in consultation with Government of India has _____ working group to suggest measures for
revival of weak public sector banks.
(a) appointed (b) constituted (c) devolved (d) none of these

20. He stepped out and began to walk _____


(a) behind (b) around (c) besides (d) none of these

21. The lady received quick promotions when they saw how _____she was.
(a) lethargic (b) hostile (c) correct (d) hard working

Previous Years
Page 178 CLAT & AILET Papers
22. The shower in the bathroom needs mending as it has started _____
(a) trickling (b) soaking (c) dripping (d) overflowing

23. The person who is looking for sympathy talks _____


(a) petulantly (b) despicably (c) plaintively (d) deftly
24. The very nature of society is inimical _____ freedom.
(a) of (b) to (c) with (d) on

25. Wilbur Wright flew his airplane _____France in 1909.


(a) On (b) Upon (c) Until (d) Over

Direction (Q.Nos. 26 - 35): Fill in the blanks.


If you look very closely at a newspaper photograph you will see that the grey areas are made up of black
dots of various sizes. The lighter the shades of grey, the smaller the dots. In .....(26)..... area the dots are
larger. The process by which this .....(27)..... is achieved is called half-tone screening. First of all a negative
.....(28)..... is made of the picture. That is, the light and dark areas are .....(29)..... so that white becomes
..... (30)..... and black becomes white. This picture is projected .....(31)..... a special screen. The screen is
made of glass and has lines .....(32)..... on it both vertically and horizontally. So the screen is divided up
.....(33)..... tiny squares rather like graph paper. When the light .....(34)..... the negative is shone through
this .....(35)..... it is broken up into dots. This light lands on a metal plate that is coated with chemicals.

26. (a) smaller (b) darker (c) lighter (d) small

27. (a) effect (b) colour (c) tone (d) affect

28. (a) photograph (b) print (c) copy (d) paint

29. (a) reversed (b) removed (c) coloured (d) darkened

30. (a) dark (b) dotted (c) black (d) white

31. (a) into (b) above (c) through (d) on

32. (a) ruled (b) cut (c) reflected (d) deflected

33. (a) from (b) into (c) like (d) in

34. (a) from (b) of (c) on (d) at

35. (a) graph paper (b) framework (c) screen (d) paper

SECTION - B : GENERAL KNOWLEDGE

36. The world’s largest nuclear power station is


(a) Chernobyl (b) Kashiwazki-Kariwa (c) Fukushima-Daiichi (d) Kalpakkam

37. The First International Women’s Day was celebrated on


(a) 17.3.1911 (b) 8.3.1911 (c) 18.3.1911 (d) 9.3.1911
Previous Years
CLAT & AILET Papers Page 179
38. The nature and extent of nuclear accident in power plants is indicated as International Nuclear and
Radiological Event (INRE) and quantified in terms of the highest as
(a) Level 10 Accident (b) Level 12 Accident (c) Level 7 Accident (d) Level 6 Accident

39. The Minority Affairs Minister of Pakistan who was assassinated in March 2011
(a) Salman Taseer (b) Nawaaz Paintal (c) Shahbaz Batti (d) None

40. The present Central Chief Information Commissioner is


(a) Wajhat Habibullah (b) Satyananda Mishra (c) M. L. Sharma (d) Sushma Singh

41. The country that gets highest percentage of electricity from nuclear power stations is
(a) U. S. A. (b) Japan (c) China (d) France

42. The present Chief Election Commissioner of India is


(a) S. Y. Quraishi (b) H. S. Brahma (c) V. S. Sampath (d) Navin Chawl

43. Which of the following Chief Justices of India also became the Vice President of India?
(a) Patanjali Sastri (b) Hidayatullah (c) Sikri (d) None

44. India’s Foreign Exchange Reserve has become the _____ largest in the world.
(a) Eighth (b) Twelfth (c) Fourth (d) Eleventh

45. The first ever woman Secretary-General of SAARC, Ms. Fathimath Dhiyana Saeed is from
(a) Pakistan (b) Bangladesh (c) India (d) Maldives

46. Air-conditioning was invented by


(a) Carrier (b) Voltas (c) A. Volta (d) Sinclair

47. Motor Cycle was invented by


(a) Davidson (b) G. Daimler (c) Benz (d) MacMillan

48. Which of the following has the largest territory?


(a) U.S.A (b) Brazil (c) Canada (d) China

49. The country that recently joined the South Asian Association for Regional Co-operation was
(a) Myanmar (b) Iran (c) Iraq (d) Afghanistan

50. According to the survey conducted in India during 2001-2003, the highest percentage of deaths
were due to
(a) Cancer (b) Heart diseases (c) Asthma (d) HIV-AIDS
51. The 123 Agreement was entered into between
(a) Canada and India (b) France and India (c) USA and India (d) China and India

52. The largest state in India in terms of territory is


(a) Uttar Pradesh (b) Madhya Pradesh (c) Maharastra (d) Rajasthan

53. The first judge who was sought to be impeached under the Constitution was
(a) Justice V.Ramaswami (b) Justice Dinakaran
(c) Justice Sen (d) None
Previous Years
Page 180 CLAT & AILET Papers
54. The state having the maximum number of seats in Rajya Sabha is
(a) Maharastra (b) Uttar Pradesh (c) Tamil Nadu (d) Andhra Pradesh

55. Rajya Sabha members are elected for a period of


(a) 5 years (b) 4 years (c) 2 years (d) 6 years

56. The Supreme Court recently declared the appointment non-est of


(a) Navin Chawla (b) S. C. Mishra (c) P. J. Thomas (d) Both (b) and (c)

57. The maximum number of nominated members to Lok Sabha is


(a) One (b) Two
(c) Three (d) No one can be nominated

58. The special reservation of 4% in favour of Muslims was struck down as unconstitutional by
(a) High Court of Madras (b) High Court of Andhra Pradesh
(c) High Court of Delhi (d) High Court of Kerala

59. The Year Book of Industrial Statistics 2010 released by the UN Industrial Development Organization
ranks India among
(a) Top 12 countries (b) Top 15 countries (c) Top 4 countries (d) Top 10 countries

60. The European Union member states decided to temporarily withdraw the preferential tariff benefits
from August 2010 on the ground that there are significant shortcomings in the implementation of
three UN Human Rights Conventions in
(a) Sri Lanka (b) India (c) Pakistan (d) Nepal

61. The Eleventh South Asian Games was held in


(a) Kathmandu, Nepal (b) Hyderabad, India (c) Dhaka, Bangladesh (d) Colombo, Sri Lanka

62. Dr. Jean Mayer Award for Global Citizenship by the Tufts University, USA, was awarded in 2010 to
(a) Justice Verma (c) Narayana Murthy (b) M. C. Mehta (d) Sanjoy Hazarika

63. The constitutional advisor to the Constituent Assembly was


(a) Rajendra Prasad (b) B. N. Rau (c) B. R. Ambedkar (d) Kamat

64. Census takes place in India


(a) every 20 years (c) every 10 years (b) every 15 years (d) every 5 years

65. INTERPOL, a 147-nation Police Commission, has its Headquarters at


(a) Paris (b) Lyons (c) London (d) Frankfurt

66. Red Cross is an international society for relief of suffering in the time of war or disaster which has
headquarters at Geneva was founded in 1863 on the advocacy of
(a) Herbert Spencer (b) J.H. Dunant (c) W.H. Morgan (d) C.A. Barnord

67. Who among the following was the Chairman of Sixth Pay Commission?
(a) Justice Rathnaval Pandian (b) Justice Venkatachalliah
(c) Justice A.K. Majumdar (d) Justice B.N. Srikrishna

Previous Years
CLAT & AILET Papers Page 181
68. Who among the following was the Chairman of Sixth Pay Commission?
(a) Justice Rathnaval Pandian (b) Justice Venkatachalliah
(c) Justice A.K. Majumdar (d) Justice B.N. Srikrishna

69. Kyoto Protocol which was replaced by Cancun in December 2010 relates to
(a) Climate (b) Water (c) Air (d) Soil

70. Which of the following mountains has oldest rocks in India?


(a) Aravallis (b) Himalayas (c) Siwaliks (d) None

SECTION - C : LEGAL APTITUDE

71. PRINCIPLE:
A master shall be responsible for the wrongful acts of his servants in the course of his employment.
FACTS:
The Syndicate Bank was running a small savings scheme under which its authorised agents would
go round and collect small savings from several people on daily basis. These agents would get
commission, on the deposits so collected. Ananth was one such agent, collecting deposits from
factory workers engaged on daily wages. Though he regularly carried on his business for sometime,
slowly he started appropriating deposits for his personal use and one day he just disappeared. One
Fatima, who had been handing over her savings to him found that nearly for a month before his
disappearance, he was not depositing her savings at all. The Bank, when approached, took the
stand that Ananth was not its regular and paid employee and therefore, it was not responsible for his
misconduct. She files a suit against the Bank
(a) The Bank is not liable, since Ananth was not its salaried employee
(b) The Bank is liable, since Ananth was projected as its employee
(c) The Bank is not liable, since it did not know about misappropriation by Ananth
(d) None of the above

72. PRINCIPLE:
When the parties to an agreement agree on the same thing in the same sense, there arises legally
binding obligations between them.

FACTS: Zaverilal’s antique shop was a well-known shop of the locality. Taradevi, a socialite of the
locality, went to the shop and she was attracted by an earthern jar on display. Zaverilal explained to
her that the jar belonged to Hoysala period; and despite its earthern composition, it was very strong
and almost unbreakable. Taradevi replied to him that she was so captivated by the jar that it was
immaterial to her as to which period it belonged. She bought the jar and came home. She placed the
jar in a prominent place in her drawing room. One of her friends, an art critique, who happened to
visit her, told her that the jar was not at all an Antique, but Taradevi did not bother about it. One day,
it accidentally fell down and broke into pieces. Taradevi took up the matter with Zaverilal that his
both statements were wrong and therefore, he should pay damages to her.

(a) Zaverilal must compensate her, since his both the statements concerning jar were wrong
(b) Zaverilal need not compensate, since, Taradevi was not concerned with its antique value at all
(c) Zaverilal must compensate, because he carelessly made the statements
(d) None of the above

Previous Years
Page 182 CLAT & AILET Papers
73. PRINCIPLE:
Interfering with another’s goods in such a way as to deny the latter’s title to the goods amounts to
conversion and it is a civil wrong.
FACTS:
Ram went to the bicycle stand a park his bicycle and he found the stand fully occupied. Ram
removed a few bicycles in order to rearrange the stand and make some space for his bicycle. He
parked his bicycle properly and put back all the bicycles except the one belonging to Shyam. It was
rather negligent on the part of Ram and he was in fact in a hurry to get into his office. Somebody
came on the way and took away Shyam’s cycle. The watchman of the stand did not take care of it
assuming that the cycle was not parked inside the stand. Shyam filed a suit against Ram for
conversion
(a) Shyam would succeed because Ram’s act led to the stealing of his bicycle
(b) Shyam would not succeed, because Ram did not take away the bicycle himself
(c) Ram would not be held responsible for the negligence of the watchman
(d) None of the above

74. PRINCIPLE:
Damages are payable for breach of contract and the purpose of damages is to compensate for the
loss suffered and not to improve position more than what it would have been if the contract had been
duly performed.

FACTS:
A wanted to buy a house and he contracted with a surveyor S to inspect a particular house and value
it for him. S surveyed the house and valued it for Rs. 10 lakhs. S, however, failed to notice the
defective plumbing system in the house and had he taken note of it, the house would have been
worth only Rs. 8 lakhs. A followed S’ s advice and bought the house for Rs. 8 lakhs and thereafter
spent Rs. 4 lakhs for repairing the plumbing system. He filed a suit against S claiming Rs. 4 lakhs
as damages.

(a) A should get Rs. 4 lakhs, because that was the amount that he had to spend finally to make the
house worth living
(b) A should get Rs. 2 lakhs, since he paid additional Rs. 2 lakhs on account of S ‘s negligence
(c) A should get Rs. 4 lakhs, since that loss was attributable to S’s negligence
(d) None of the above

75. PRINCIPLE:
A right of action cannot arise out of an illegal activity.

FACTS:
A and B were thieves engaged in stealing cars and other vehicles. Once they stole a car; and
while driving off, they had to cross a city. They engaged a driver to drive them through the city,
since they did not know the route inside. The indicator lamp of the car was not working and the
thieves had not realised this and therefore, had not told about it to the driver. While driving, through
the city, the car was hit by another vehicle because of the faulty indicator. In the accident, the
driver was injured and he filed a suit against A and B.

Previous Years
CLAT & AILET Papers Page 183
(a) The driver would lose, because he was driving a stolen car
(b) The driver would win, because he was not a party to the stealing
(c) The driver would win, because he did not know anything about the stealing
(d) None of the above

76. PRINCIPLE:
Law does not take notice of trifles.

FACTS:
A proposes to his neighbour B that they both should go together for a morning walk. B agrees to the
proposal and it is decided that both of them would meet at a particular point at 6 AM from where they
would set off for the morning walk. Inspite of the agreement, B does not turn up. A waits for him at 6
AM every day for a continuous period of seven days. Thereafter he files a suit against B claiming
damages for the agony and mental torture suffered by him. Decide.

(a) B is guilty of breach of contract and is liable to pay damages


(b) There is no intention to enter into legal relations
(c) The matter is too small and the court will refuse to go into it
(d) None of the above

77. PRINCIPLE:
Limitation bars the remedy, not the right.

FACTS:
After the expiry of the period of limitation of three years, debtor Rohan makes a part payment of debt
to creditor Sohan. Sohan then files a suit against Rohan for recovery of the debt after two years from
the date of part payment. Decide.

(a) The part payment extends the period of limitation


(b) The suit is time barred as part payment is made after the expiry of period of limitation
(c) Fresh period of limitation begins from the date of part payment
(d) None of the above

78. PRINCIPLE:
Ignorance of law is no excuse.

FACTS:
A fails to file his income tax returns for ten years. The Income tax department issues to him notice
to show cause why proceedings should not be initiated against him for the recovery of the income
tax due from him with interest and penalty. Advise.

(a) He may take the plea that his advocate had advised him not to file the return as his income was
not taxable
(b) He may request the department to waive the interest and penalty
(c) A must pay the tax due as ignorance of law cannot be ground of defence
(d) None of the above

Previous Years
Page 184 CLAT & AILET Papers
79. PRINCIPLE:
Nobody shall make use of his property in such a way as to cause damage to others. Any such use
constitutes private nuisance, a wrongful act under Law of Torts.
FACTS:
Vasan was owing a house, adjacent to a cluster of houses, owned by Varadan. Varadan was leasing
out these houses whereas Vasan was living in his house. When Vasan was transferred to another
place, he leased out his house to a person suffering from AIDS. Fearing the spread of AIDS, the
tenants moved out of V aradan’ s houses. Varadan requested Vasan to evict AIDS patient and he
offered to fix a suitable tenant for V asan’ s house, if the AIDS patient is evicted. But Vasan refused
by arguing that AIDS would not spread as feared by V aradan’ s tenants. Varadan filed a suit against
Vasan.

(a) Varadan will win, because Vasan knowingly caused him fmancial damage
(b) Varadan will not win, because Vasan could lease his house to whomever he wanted
(c) Varadan will not win, because Vasan should not be held responsible for public mis-perception
(d) None of the above

80. PRINCIPLE:
One has to compensate another for the injury caused due to his wrongful act. The liability to
compensate is reduced to the extent the latter has contributed to the injury through his own negligence.
This is the underlying principle of contributory negligence.

FACTS:
Veerappa owns a farm at a distance of half a furlong from the railway track. He stored in his land the
stacks of dried up straw after the cultivation as is normal in farming. One day when the train was
passing through the track, the driver was negligently operating the locomotive by allowing it to emit
large quantities of spark. The high wind, normal in open fields, carried the sparks to the stacks
stored by Veerappa and the stacks caught fire thereby causing extensive damage. Veerappa filed a
suit against the Railways claiming damages. The Railways while acknowledging liability alleged
contributory negligence on the part of Veerappa.

(a) Veerappa was not liable since his use of land was lawful
(b) Veerappa’s farm being at a reasonable distance from the railway track, he cannot be held
responsible for the high winds
(c) Veerappa should have anticipated the possibility and hence he is liable for contributory negligence
(d) None of the above

81. PRINCIPLE:
A person is entitled to protect his property by using lawful means.

FACTS:
Ramlal is growing valuable vegetables and fruits in his farm and he has fenced the farm to prevent
the cattle from entering into it. In addition he has kept a ferocious dog to chase away intruding
urchins and cattles. Some children were playing in a nearby playground and the ball slipped into the
farm. A boy running after the ball came near the fence and shouted for the ball. But when there was
no response, he managed to creep into the farm to get the ball. The dog which was surreptitiously
waiting attacked the boy and badly mauled him. The boy’s parents filed a suit against Ramlal.

Previous Years
CLAT & AILET Papers Page 185
(a) Ramlal is not liable, since the fence and the dog are lawful means of protecting the property
(b) Ramlal is not liable for the boy trespassing and getting badly injured in that process
(c) Ramlal is liable, since an ordinary barking dog would have sufficed for the purpose
(d) None of the above

82. PRINCIPLE:
A person who commits an unlawful act towards another which can be imputed to him, must repair
the damage which the other person suffers as a consequence thereof.
FACTS:
Mr. Rajender Singh was riding his scooter on the right side of the road which is illegal as per the
Traffic Rules. Mr. Rajesh Chawla was driving his car in the opposite direction. The two vehicles
collided and resulted in loss of Rs. 50,000/- to Mr. Rajender Singh. This includes his medical
expenses and damage to the scooter. In this accident there is no fault on the part of Mr. Rajesh
Chawla.

(a) Mr. Rajendra Singh will not get any amount as damages
(b) Mr. Rajendra Singh will get full compensation
(c) Mr. Rajendra Singh will get part of compensation
(d) None of the above

83. PRINCIPLE:
When there is breach of contract, the resulting damages will have to be paid by the party breaching
the contract to the aggrieved party. However, the parties are free to agree as to damages payable in
advance in case there is breach of contract.

FACTS:
Mr. Ramesh entered into a contract with Mr. Ramakrishna for selling his green Alto Car for Rs. 3
lakhs. Mr. Ramakrishna was to pay Rs. 3 lakhs on or before 25th April 2005 and take possession of
the car. The party failing to honour the contract has to pay Rs. 40,000/- as damages to the other
party.
Mr. Ramakrishna has not performed his part of the contract. Mr. Ramesh is claiming Rs.40,000/-.
(a) Mr. Ramesh can sell the car in the second hand market and if he suffers any loss, then only he
can claim that amount from Mr. Ramakrishna
(b) Mr. Ramakrishna is liable to pay the agreed damages
(c) Mr. Ramesh has to prove that he has suffered Rs. 40,000/- as damages to get it
(d) None of the above

84. PRINCIPLE:
Every person has a right of self defence, if his life is under imminent threat.

FACTS:
Mr. Prashanth threatens Mr. Krishna that he will kill Mr. Krishna. After saying so, Mr. Prashanth
goes to his house saying that he would get his axe

(a) Mr. Krishna will have to run away.


(b) Mr. Krishna will have to go to the Police Station and file a complaint.
(c) Mr. Krishna cannot exercise the right of self defence.
(d) None of the above

Previous Years
Page 186 CLAT & AILET Papers
85. PRINCIPLE:
Whoever does any act so rashly or negligently as to endanger human life or the personal safety of
others is said to have committed an offence.
FACTS:
Mr. Mangeskar owns a Yamaha motorcycle which has very good pick up and speed. He is studying
in the IV semester of Mechanical Engineering degree course. One day it was getting late for the
college as he woke up late in the morning. He got ready and was rushing to the college so that he
would not miss the class. He was riding the motor cycle at a speed of 140 km per hour in Bangalore
city which was crowded. He was very good in riding the motorcycle. People who were using the road
got annoyed scared with the way Mr. Mangeskar was riding the motorcycle.

(a) Mr. Mangeskar has committed an act of rash and negligent driving
(b) Mr. Mangeskar is very good in driving, so there is no need for others to be panicky about his
driving
(c) Mr. Mangeskar is very studious student and he does not want to miss any class in the College
(d) None of the above

86. PRINCIPLE:
When two or more persons agree to do an illegal act, it is criminal conspiracy punishable with
imprisonment.

FACTS:
Mr. Bharath is a student of B.E. in Computer Science. He loves his computer very much. He
considers his computer as his close friend and companion. On 1.4.2006, while interacting with his
computer, he hacked into the Bank account of Mr. Javed and was successful in withdrawing money
from Mr. Javed’s bank account. He did it to please his girl friend.

(a) Mr. Bharath has committed an offence.


(b) Mr. Bharath has committed a cyber crime.
(c) Mr. Bharath has not committed the offence of criminal conspiracy.
(d) None of the above

87. PRINCIPLE:
Contractual liability is completely irrelevant to the existence of liability in tort (civil wrong).

FACTS:
X purchased a bottle of ginger-beer from a retailer. As she consumed more than 3/4 of the contents
of the bottle, she found a decomposed remains of a snail in the bottle. After seeing the remains of a
snail, she fell sick on the thought of what she consumed. She sued the manufacturer of the beer for
negligence, though there is no contractual duty on the part of the manufacturer.

(a) X cannot sue the manufacturer for negligence in the absence of a contract
(b) X cannot sue the retailer
(c) X can sue the manufacturer as he had a duty to take care to see that bottles did not contain any
other substance than the beer and hence liable to have broken that duty
(d) None of the above

Previous Years
CLAT & AILET Papers Page 187
88. PRINCIPLE:
The standard to determine whether a person has been guilty of negligence is the standard of care
which, in the given circumstances, a reasonable man could have foreseen.

FACTS:
The Agricultural University constructed 200 houses for its employees in its premises. Two huge bore
wells were sunk and motors were installed. They did not cover the pump rooms properly. A child, 6
years old, from one of the quarters was playing near the pumphouse. On hearing the noise of the
pump, she was curious to see the motor. She touched the motor that was not covered properly and
three of her fingers were cut.

(a) The parents of the child cannot sue the University on any grounds
(b) Inspite of the child’s act, her parents can successfully sue the University for damages
(c) The University can be made liable only to the extent of the cost of treatment as the child also
contributed to the incident
(d) None of the above

89. PRINCIPLE:
A person is liable for all the injurious consequences of his careless act.
FACTS:
Ram, a snake charmer, was exhibiting his talents to a group of people. One of the snakes escaped
and bit a child who had to be hospitalised for two days for treatment.

(a) Ram is liable to compensate the child’s family for his careless act
(b) Ram is not liable to anything as such things keep happening
(c) Ram is not in a position to compensate as he is poor
(d) None of the above
90. PRINCIPLE:
A contingent contract is a contract to do or not to do something, if some event, collateral to such
contract, does or does not happen.

FACTS:
A agrees to pay B a sum of Rs. 1 lakh if B marries C within a period of six months. B marries C
during the seventh month as the marriage hall was available only during that month. B claims Rs. 1
lakh from A.

(a) B can successfully claim Rs. 1 lakh from A


(b) B cannot claim Rs. 1 lakh from A as B married C after the duration set in the contract
(c) B can claim Rs. 1 lakh as well as special damages from A for having married C
(d) None of the above

91. PRINCIPLE:
Right to carry on any occupation, trade or business is a Fundamental Right under the Indian
Constitution. The State is under an obligation under the Directive Principles of State Policy to
organize agriculture and animal husbandry on scientific lines, and towards that goal, take steps to
prohibit cow slaughter.

Previous Years
Page 188 CLAT & AILET Papers
FACTS:
The State of X passed a legislation totally prohibiting cow slaughter. A, a butcher, trading in meat of
all animals including cows, challenged this legislation as violating the fundamental right to carry on
his business.

(a) The State of X cannot make a law taking away any fundamental right. Such law is null and void
(b) The State can prohibit cow slaughter to organize animal husbandry in scientific lines
(c) The right to carry on any occupation, trade or business is subject to reasonable restrictions.
Banning cow-slaughter is such a restriction to better animal husbandry production and it is
within the power of State
(d) None of the above

92. PRINCIPLE:
The Constitution guarantees freedom of religion and expects the citizen to respect national anthem as
a fundamental duty.

FACTS:
According to the religions tenets of a particular sect, singing the praise of any entity other than God
is forbidden. The child, belonging to that sect, refuses to sing national anthem in the course of
school assembly, in which every child must compulsorily participate. When the disciplinary action
was instituted against the child, the parents challenged the school’s order of singing the national
anthem.

(a) The School can take disciplinary action against the child as it is the fundamental duty to respect
national anthem
(b) The fundamental right to freedom of religion cannot be abridged by a fundamental duty imposed
by the State
(c) The girl is free to follow the religion of her choice and follow its rules. It cannot be said to be
disrespect on the part of the girl if she refuse to sing national anthem
(d) None of the above

93. PRINCIPLE:
All citizens shall have the right to freedom of speech and expression.

FACTS:
X, a famous writer and Novelist criticized another novelist, B stating that: “the novel of B is foolish, he
is a weakman, his Novel is indecent, his mind is impure, he is not of a good character, he should write
decent and good Novel.” Can X be sued for defamation?

(a) He is not liable because he has just expressed his personal views
(b) He is liable to be sued for defamation if his statement, was not true or said in mala fide intention
(c) He cannot be liable because he has fundamental right to freedom of speech and expression
(d) He cannot be sued, because both are writer and novelists and both can criticize each other

Previous Years
CLAT & AILET Papers Page 189
94. PRINCIPLE:
A citizen is expected to take reasonable duty of care while driving on the road and not to cause
injuries to any person. (Common for Q.94, 95 and 96)
FACTS: X, the owner of a car, asked his friend Y to drive the car to his office. As the car was near his
(X’ s) office, it hit a pedestrian P on account of Y’ s negligent driving and injured him seriously. P
sued X for damages. Which one of the following is correct?

(a) X is not liable as it was the negligence of Y.


(b) The liability was solely of Y as X was not accompanying him.
(c) As Y was driving under X’s care and authority, X is liable.
(d) X is not liable under the principle of inevitable accident.

95. The standard of care generally used in cases of negligence is the

(a) skill and care of a professional person


(b) care taken by an intelligent and prudent man
(c) foresight of a prudent man
(d) skill and foresight of an ordinary person of prudence and competence

96. Two persons are said to be joint tort-feasors when

(a) a person on account of his negligence gives opportunity to another for committing a tort
(b) two or more persons are simultaneously involved in committing a wrong
(c) a tort is committed by two or more persons or any one of them when they are engaged in
furtherance of a concerted purpose
(d) two persons together cornmit a tort
97. PRINCIPLE:
The law permits citizens to use force only for protection when necessary against imminent attack.
(Common for Q. 97, 98 & 99)

FACTS:
P with the intention of committing theft entered the house of Q. Q, on seeing him entering, struck
him with a lathi and P fell down unconscious. Thereafter, Q gave him another blow of lathi on his
head which caused his death. On being prosecuted for murder, Q took the plea of private defense.
Which of the following argument is valid?

(a) Since Q was acting in the exercise of right of private defense of his property, he had taken a valid
defence.
(b) Since in the defence of one’s property one cannot cause death of the intruder, Q has no defense.
(c) Q has used excessive force as once P fell unconscious; there was no need for the second blow.
Hence, Q’ s plea of right of private defence will not succeed.
(d) If P committed house breaking in the night, Q has the right to cause death in defense of his
property, and thus Q’s plea should prevail.

98. To whom, among the following, is the right of private defence is available?
1. Only to the defender being a preventive right.
2. An aggressor, while facing action on the part of the defender which is excessive.

Previous Years
Page 190 CLAT & AILET Papers
Select the correct answer using the code given below:
(a) 1 only
(c) 2 only
(b) Both 1 and 2
(d) Neither 1 nor 2

99. X’s farm house in outskirts of Delhi was attacked by a gang of armed robbers. X without informing
the police, at first warned the robbers by firing in the air. As they were fleeing from the farm, he fired
and killed one of them. At the trial -
I. X can avail the right of private defence as he was defending his life and property.
II. X cannot avail the right as he failed to inform the police.
III. X cannot avail the right as he caused more harm than was necessary to ward off the danger.
IV. X can avail of the right as at first he only fires in the air.

(a) I and IV (b) II only (c) II and III (d) IV only

100. PRINCIPLE:
Conspiracy is a combination between two or more persons formed for the purpose of doing either an
unlawful act or a lawful act by unlawful means.
FACTS:
X and Y conspire to poison Z. X in pursuance of the conspiracy procures the poison and delivers it
to Y in order that he may administer it to Z. Y in pursuance of the conspiracy administers the poison
in the presence of X and thereby causes Z’s death. What offence, if any has been committed by X
and Y, respectively? (Common for Q. 100, 101 & 102)

(a) Y has committed the offence of murder and X was an abettor.


(b) Both X and Y have committed the offence of criminal conspiracy.
(c) X has committed the offence of murder and Y was an abettor.
(d) Both X and Y have committed the offence of murder.

101. Suppose X & Y were apprehended by A before administering poison to Z . What is the offence
committed by X & Y ?
(a) X & Y are not guilty of conspiracy as they did not cause death of Z.
(b) Liability of X comes to an end and Y is only liable.
(c) As no offence is committed and none of them are responsible.
(d) X & Y are to be punished as it is immaterial whether illegal act is ultimate object of the agreement
or merely incidental to that object.

102. The gist of the offence of criminal conspiracy is


(a) agreement is necessary between two or more persons to do or cause to be done an illegal act.
(b) a legal act by illegal means followed by an overt act to be done or cause to be done according
to agreement.
(c) both (a) and (b)
(d) None of the above

Previous Years
CLAT & AILET Papers Page 191
103. PRINCIPLE:
Contributory negligence in an accident is a defence to a charge in criminal law.

FACTS:
X, the deceased was negligently crossing the busy road at Connaught place in Delhi while Y’s car
hit him resulting in death of X. What is the liability of Y?
(a) Y is liable for death of X.
(b) Y is partly liable as X contributed to his own death.
(c) Y is absolved from liability based on the principle of contributory negligence.
(d) None of the above
104. PRINCIPLE:
A contract without consideration is void. When at the desire of one party the other party does
something, the consideration is said to flow from the latter to the former.
FACTS:
A’s house was on fire and a child was trapped inside the house. Everyone was shouting for help. A
brave onlooker, hearing the shrieks of child, went inside the house and brought him out. The father
of the child promised to pay the rescuer Rs.10,000. Subsequently, he backtracked his promise. The
rescuer sued the promise for the breach.

(a) The father of the child must pay for the service rendered by the rescuer
(b) The rescuer is not entitled to the payment, since he acted on his own
(c) Commercial considerations cannot be applied to humanitarian instincts
(d) None of the above

105. PRINCIPLE:
A contract procured by coercion is bad under Indian Contract Act. Coercion is defined as committing
or threatening to commit any act forbidden by Indian Penal Code. Attempt to commit suicide is an
offence under India Penal Code.

FACTS:
A wanted his wife B to part with some property given to her by her father. B resisted fearing that her
husband would squander it away. A threatened her that if she does not sign the deed transferring the
property to him (i.e., A), he would commit suicide. B signed the deed. Subsequently, she challenged
the deed on the ground that the deed was bad under law.
(a) The deed is valid under the law.
(b) The transfer of property is complete once the deed ‘is entered between the owner of the property
and the buyer.
(c) The deed is not valid as it was signed under the threat of suicide by her husband which amounts
to coercion and vitiates the contract.
(d) Husband has a right over the property of the wife that was voluntarily transferred to the buyer.
(d) Husband has a right over the property of the wife that was voluntarily transferred to the buyer.

Previous Years
Page 192 CLAT & AILET Papers
SECTION - D: LOGICAL REASONING

Direction: Find out the missing word.

106. Carpet: Floor: : ? : Wall


(a) Wall paper (b) Wall hanging (c) Scenery (d) Oil painting

107. Kaziranga: Rhino : : ? : Lion


(a) Corbett (b) Kanha (c) Gir (d) Sunderbans

Direction: Find out that word number.

108. AOP, CQR, EST, GUV, ?


(a) IYZ (b) HWX (c) IWX (d) JWX

109. 6, 11, 21, 36, 56, ?


(a) 66 (b) 76 (c) 81 (d) 86

110. A father is now three times as old as his son. Five years back, he was four times as old as his son.
The age of the son is
(a) 12 (b) 15 (c) 18 (d) 20

111. A bus for Delhi leaves every thirty minutes from a bus stand. An enquiry clerk told a passenger
that the bus had already left ten minutes ago and the next bus will leave at 9.35 a.m. At what time
did the enquiry clerk give this information to the passenger?
(a) 9.10 a.m. (b) 8.55 a.m. (c) 9.08 a.m. (d) 9.15 a.m.

112. Soaking dried beans overnight before cooking them reduces cooking time. However, cooking without
pre-soaking yields plumper beans. Therefore, when a bean dish’s quality is more important than the
need to cook that dish quickly, beans should not be pre-soaked.

Which one of the following is an assumption required by the argument?


(a) Plumper beans enhance the quality of a dish
(b) There are no dishes whose quality improves with faster cooking
(c) A dish’s appearance is as important as its taste
(d) None of the other ingredients in the dish need to be pre-soaked

113. Everyone in Biba’s neighbourhood is permitted to swim at Barton Pool at some time during each
day that it is open. No children under the age of 6 are permitted to swim at Barton Pool between
noon and 5 P.M. From 5 P.M. until closing, BartonPool is reserved for adults only.
If all the sentences above are true, then which one of the following must be true ?
(a) Few children under the age of 6 live in Biba’s neighbourhood
(b) If Biba’s next door neighbour has a child under the age of 6, then Barton Pool is open before
noon
(c) If most children who swim in Barton Pool swim in the afternoon, then the pool is generally less
crowded after 5 P.M.
(d) On days when Barton Pool is open, at least some children swim there in the afternoon

Previous Years
CLAT & AILET Papers Page 193
Direction: Find out the missing alphabet/number.

114. D, H, L, R, ?
(a) T (b) X (c) I (d) O

115. 3, 7, 15, 31, 63, ?


(a) 92 (b) 115 (c) 127 (d) 131

116. If SIR is written as PSPIPR, then MAN can be written as


(a) PMANP (b) PMPAPN (c) MANP (d) MPANP

117. In a certain code language, 35796 is written as 44887. How is 46823 written in that code?
(a) 57914 (b) 55914 (c) 55934 (d) 55714

118. If day before yesterday was Tuesday, the day after tomorrow will be
(a) Monday (b) Wednesday (c) Friday (d) Saturday

119. Ram is facing North-West. He turns in clockwise direction by 90°, then 180° in the anti-clockwise
direction and then another 90° in the same direction. Which direction is he facing now?
(a) South-West (b) West (c) South (d) South-East

120. One morning after sunrise, Vikram and Shailesh were standing in a lawn with their backs towards
each other. Vikram’s shadow fell exactly towards left hand side. Which direction was Shailesh
facing?
(a) East (b) West (c) North (d) South

121. Zack’s Coffeehouse schedules free poetry readings almost every Wednesday. Zack’s offers half-
priced coffee all day on every day that a poetry reading is scheduled.

Which one of the following can be properly inferred from the information above?
(a) Wednesday is the most common day on which Zack’s offers half-priced coffee all day.
(b) Most free poetry readings given at Zack’s are scheduled for Wednesdays.
(c) Free poetry readings are scheduled on almost every day that Zack’s offers half-priced coffee all
day.
(d) Zack’s offers half-priced coffee all day on most, if not all, Wednesdays.

122. If it is true that neither Rama is a brother of Krishna, nor is he a lecturer, then which of the following
statements is true?
(a) Rama is not a brother of Krishna, but he is a lecturer.
(b) Rama is a brother of Krishna, but he is not a lecturer.
(c) Rama is a brother of Krishna and he is a lecturer.
(d) Rama is not a brother of Krishna and he is not a lecturer.

123. “Make hay while the sunshine” is logically similar to


(a) Sleep when you are tired.
(b) Take the bull by horn.
(c) Losing hope is along everything.
(d) None of the above

Previous Years
Page 194 CLAT & AILET Papers
124. The Constitution has given the right of free speech. People speak freely in order to enjoy this right.
Which of the following has the same logic?
(a) Smuggling is illegal. People indulge in smuggling because they want to violate laws.
(b) People have the right to live, but they die, since they do not want to exercise that right.
(c) Wealth gives power to enjoy material things. People enjoy these things by spending their wealth.
(d) All of the above

125. Literature is the mirror in which the society can see itself. The above statement implies that
(a) no ugly society can produce good literature.
(b) writers are essentially like newspaper correspondents, reporting faithfully what they are observing.
(c) literature enables the people to comprehend the dimensions which they are unaware of.
(d) None of the above

126. It is time that begging must be forbidden in all public places in India. Exploitation of charitable
instincts in general, far from solving the problem of poverty, may infact aggravate it by inducing
laziness. Which of the following, if true, would support the above reasoning?
(a) Destitution is the result of unwillingness to work.
(b) Most beggars are really poor.
(c) All poor people do not beg.
(d) None of the above

127. Child labour is only a reflection of the malaise in the society.


Which of the following, supports the above statement?
(a) Child labour is not abhorred by the public.
(b) The Government is not serious about eliminating child labour.
(c) Exploitation of the poor, weak and unorganized is not uncommon.
(d) All of the above

128. If it is true that ‘good governance ensures law and order’, then identify the statement which has to
be accepted along with the given statement.
(a) A healthy society is governed by maximum number of laws.
(b) A strong government uses force to enforce law.
(c) Law is indispensible for the society.
(d) All of the above

129. If it is true that ‘very rarely do we come across committed film directors’, which one of the following
is false beyond doubt?
(a) We can never come across committed film directors.
(b) No film director is committed.
(c) All film directors are committed, but we can never meet them.
(d) None of the above

130. If it is true that ‘All men are honest’ and ‘All politicians are men’, for which among the following is it
impossible to follow from the given premises?
(a) Some politicians are not honest.
(b) All politicians are honest.
(c) One politician is honest.
(d) All honest persons are politicians.

Previous Years
CLAT & AILET Papers Page 195
131. Consider a set of premises like this: If science and religion clash, then critical thinking and dogma
will clash. Dogma is not acceptable and hence religion. But critical thinking shakes the foundation
of morality.
Identify the conclusion which follows from the premises.
(a) If science is not accepted, then the foundations of morality are not shaken.
(b) If foundations of morality are shaken, then science is acceptable.
(c) If science is accepted, then foundations of morality are shaken.
(d) Though science is accepted, it cannot shake the foundation of morality.

132. If Ramesh likes oranges more than grapes, strawberries more than apples, and apples more than
oranges, then which one of the following is not true?
(a) He likes grapes more than strawberries.
(b) He likes strawberries more than grapes.
(c) He likes apples more than grapes.
(d) He likes strawberries more than apples.
133. “Justice should not only be done, but seen to have been done”. The statement intends to convey
the message that
(a) Court proceedings must be open to public.
(b) Public participation in judicial decision making is essential for timely justice.
(c) Court proceedings must be fair and judgments should be based on legal reasoning.
(d) All court cases must be publicized and reported to the public in their own language.

134. Yoga has become a very popular type of exercise, but it may not be for everyone. Before you sign
yourself up for a yoga class, you need to examine what is it you want from your fitness routine. If
you’re looking for a high-energy, fast-paced aerobic workout, a yoga class might not be your best
choice.

This paragraph best supports the statement that


(a) Yoga is more popular than high-impact aerobics.
(b) Before embarking on a new exercise regimen, you should think about your needs and desires.
(c) Yoga is changing the world of fitness in major ways.
(d) Most people think that yoga isn’t a rigorous form of exercise.

135. Which one of the following, if true, can be empirically tested?


(a) Violence is engineered by power-hungry politicians.
(b) Only poor are driven to violence.
(c) Faith in God alone can contain violence.
(d) Instinct to preserve one’s own self is at the root of violence.

136. Which one of the following, whether true or false, is irrelevant to the issue?
(a) Also, violence is due to the fact there is a tendency in man to enjoy when somebody suffers.
(b) Violence is commonplace in the world because men hardly learn philosophy.
(c) Violence is the overt manifestation of covert desire to be adventurous.
(d) Violence is the result of man’s tendency to assert himself.

137. Which one of the following makes the argument self-contradictory?


(a) The soul has finite existence. (b) Whatever is corruptible is soluble.
(c) What is spatial is material. (d) Motion is not a reality.

Previous Years
Page 196 CLAT & AILET Papers
138. Before Robert Norman worked on ‘Dip and Field Concept’, his predecessor thought that the
tendency of the magnetic needle to swing towards the poles was due to a point attractive. However,
Norman showed with the help of experiment that nothing like point attractive exists. Instead, he
argued that magnetic power lies is lodestone.

Which one of the following is the problem on which Norman and others worked?
(a) Existence of point attractive (b) Magnetic power in lodestone
(c) Magnetic power in needle (d) Swinging of magnetic needle

139. In his study of conduction of heat, James Maxwell observed that quantities which are significant
in such a study are ‘temperature, flow of heat and conductivity’. Further, he noticed that the
mathematical laws of the uniform motion of heat in homogenous media are identical in form with
those of attractions varying inversely as the square of the distance. He thought source of heat,
flow of heat and temperature correspond respectively to centre of attraction, accelerating effect of
attraction and potential. On this ground, Maxwell observed that conduction of heat proceeds by
an action between contiguous parts of a medium.
Which one of the following best explains Maxwell’s method?
(a) Mathematical calculation (b) Analogy
(c) Observation (d) Experiment

140. Henry Poincare argued that in space there are not only rectilinear triangles in which angles equal
to two right angles, but also curvilinear triangles in which angles are less than two right angles.
He, further, maintained that to name the first one straight is to subscribe to Euclidean geometry and
to name the latter straight is tantamount to subscribing to non-Euclidean system.
Which one of the following accurately represents what is at stake?
(a) Acceptability of Euclidean system (b) Acceptability of Non-Euclidean system
(c) Confusion in Geometry (d) Choice of the definition of straight line

SECTION - E: MATHEMATICS

141. Kirti took loan of Rs.800 at the rate of 11% per year for 7 months. After seven months she has to pay
(a) Rs.851 (b) Rs.852 (c) Rs.950 (d) Rs.951

142. 16 men can do a piece of work in 16 days. 4 days after they started the work, 8 more men joined
them. How many days will they now take to complete the remaining work?
(a) 10 days (b) 6 days (c) 8 days (d) 12 days

143. The number of the members of a club is increased by 10% every year. If the initial number of the
members is 500, then what will be the number at the beginning of the third year?
(a) 610 (b) 615 (c) 620 (d) 605

144. A student rides a bicycle at 8 km/hr and reaches the school 2.5 minutes late. The next day he
increases the speed to 10 km/hr and reaches school 5 minutes early. How far is the school from
his house?
(a) 4 kms (b) 8 kms (c) 5 kms (d) 10 kms

Previous Years
CLAT & AILET Papers Page 197
145. Rs. 324 is divided among three friends Sonu, Monu and Hari in the ratio 5: 6 : 7. What is Monu’s
share of money?
(a) 68 (b) 108 (c) 60 (d) 120

146. In an examination, every candidate took either Physics or Mathematics or both. 84% of the
candidates took Physics and candidates who took Mathematics were half of those who took
Physics. The total number of candidates being 1000, how many took both Physics and Mathematics?
(a) 200 (b) 240 (c) 250 (d) 260

147. A company declares a dividend of 12% on Rs.100 shares. A housewife buys such shares and
gets 15% on her investment. At what price she bought the shares?
(a) Rs.80 (b) Rs.85 (c) Rs.125 (d) Rs.76

148. The price of an article is cut by 10%. To restore it to the original value, the new price must be
increased by
1 1
(a) 10% (b) 9 % (c) 11% (d) 11 %
11 9

149. A shopkeeper mixes two varieties of Tea, one costing Rs.40/kg and another Rs.50/kg in the ratio
3 : 2. If he sells the mixed variety of Tea at Rs.48/kg, his gain or loss percent is
(a) 48.4% gain (b) 48.4% loss (c) 10% gain (d) 10% loss

150. The value of a car depreciates every year at the rate of 10% on its value at the beginning of the year.
If the present value of the car is Rs.52,488, its worth four years ago was
(a) Rs.68,232 (b) Rs.68,234 (c) Rs.69,862 (d) Rs.80,000

Previous Years
Page 198 CLAT & AILET Papers
AILET Question Paper 2012

SECTION – A : LEGAL APTITUDE

1. What is minimum duration of stay essential before a person can apply for citizenship of India?
(A) Five years (B) Seven years (C) Nine years (D) Twelve years

2. The Ordinances issued by the Governor are subject to the approval by


(A) State legislature
(B) President
(C) Central Government
(D) Parliament

3. Scheduled Castes and Scheduled Tribes


(A) have been specified by the Presidential Orders issued in consultation with the Governors of the
respective States
(B) have been specified in the Constitution by the Constitution makers in the form of a Schedule
(C) have to be specified and notified by the Parliament after detailed discussions about the backward
nature of these people
(D) are the Castes and Tribes notified by the State Government after detailed discussion as to their
socio-economic conditions

4. Free legal aid for an accused is a


(A) Fundamental right (B) Fundamental duty of the State
(C) Directive Principles of State Policy (D) Discretion of the State

5. The concept of Secular State is that the State will not make any discrimination on the ground of
religion, caste or community against any person professing any particular form of religious faith.
The Secularism is embodied in
(A) Preamble of Constitution
(B) Directive Principles of State Policy
(C) Fundamental Rights
(D) Judicial Interpretation of Fundamental Rights

6. The National Development Council consists of


(A) Members of Planning Commission
(B) Cabinet Ministers and State Chief Ministers
(C) Representatives of Union Territories
(D) All of the above

7. Who allocates portfolios among the Ministers?


(A) Prime Minister
(B) President
(C) President on the recommendation of the Prime Minister
(D) Chairperson of Ruling Political Party

Previous Years
CLAT & AILET Papers Page 199
8. Which Indian language is given the status of ‘Classical language’ by the Central Government?
(A) Sanskrit (B) Telugu (C) Tamil (D) Pali

9. Each member of the Security Council has one vote. Decisions on procedural matters are to be
taken by .............................. permanent members. Here veto does not apply. On all other matters,
there must be nine affirmative votes including those of five permanent members.
(A) 6 (B) 7 (C) 8 (D) 5

10. The first venture of Mahatma Gandhi in all India Politics was
(A) Non-cooperation Movement (B) Dandi March
(C) Champaran Movement (D) Rowlatt Satyagrah

11. The “Right to Information” (RTI) is a


(A) Constitutional Right (B) Statutory Right
(C) Fundamental Right (D) Contractual Right

12. Which of the following committees has recommended measures for banning and controlling ragging
in educational establishments in India?
(A) The Raghavan Committee (B) Jasraj Committee
(C) Narsimhan Committee (D) Soli Sorabjee Committee

13. The Constitution of India does not mention the post of


(A) The Deputy Speaker of the Lok Sabha
(B) The Deputy Speaker of the State Legislative Assembly
(C) The Deputy Chairman of the Rajya Sabha
(D) The Deputy Prime Minister

14. A Judge of the Supreme Court of India or High Court of any State can be removed by the President
of India only
(A) when the Principles of Natural Justice are followed and the alleged misconduct is proved in an
impartial enquiry
(B) if he is satisfied through the report made by the Chief Justice of India that the misconduct of the
judge has been proved
(C) when an address is made by both the Houses of Parliament in the same session asking for his/
her removal on the grounds of proven misbehaviour and incapacity
(D) when the President and the Prime Minister are satisfied that he or she has committed a misconduct
and the same has been proved before a competent authority

15. The Constitution of India is the result of considerable imitation and adaptation rather than originality
because
(A) Makers of Indian Constitution drew much from the American Constitution, Canadian Constitution
and British made Government of India Act, 1935
(B) Makers of Indian Constitution drew much from Swiss Constitution, German Constitution and
Government of India Act of 1919
(C) Makers of Indian Constitution drew much from Constitution of Singapore, Constitution of Sri
Lanka and Government of India Act of 1919
(D) Makers of Indian Constitution drew from Constitution of South Africa, Constitution of Netherlands
and Government of India Act of 1919

Previous Years
Page 200 CLAT & AILET Papers
16. For transfer of accused person from a foreign State to India for any offence within India or relating to
India is done
(A) by previous repatriation agreement between India and concerned foreign country
(B) by instant repatriation agreement between India and concerned foreign country
(C) at the desire of ambassador representing India in concerned foreign country
(D) at the desire of the ambassador who represents concerned foreign country in India

17. Whether a Bill is a Money Bill or not is certified by the ________ and his decision is conclusive
(A) President (B) Chairman of Rajya Sabha
(C) Speaker of Lok Sabha (D) Chairman of Public Accounts Committee

18. __________ seats are reserved in all Panchayats at all levels for women
(A) One-fourth (B ) One-third (C) One-half (D ) Two-third

19. A seat of a M.P. can be declared vacant if he absents himself from the House for a continuous period of
(A) Six months (B) Two months (C) Three months (D) One year

20. What is true with Members of Parliament’s immunity from arrest?


(A) In civil cases while the Legislature is in Session, for 40 days before and after and in criminal
cases no arrest during Parliament Session
(B) In civil cases, while Legislature is in Session and for 40 days before and after and in criminal
cases arrest can be made at any time
(C) In civil cases while the Legislature is in Session, for 20 days before and after and in criminal
cases no arrest during Parliament Session
(D) In civil cases, while Legislature is in Session and for 20 days before and after and in criminal
cases arrest can be made at any time

21. The maximum duration of the zero hour in Lok Sabha can be
(A) One hour (B) Unspecified (C) 3 0 minutes (D) Two hours

22. Dealing with the chapter of citizenship under the Constitution, the Supreme Court has held that
Sonia Gandhi, the President of Congress (I) is not an Italian citizen, but is an Indian citizen because
she is governed by
(A) Article 10, Constitution of India (B) Article 11, Constitution of India
(C) The Citizenship Act, 1955 (D) Article 5, Constitution of India

23. What is true with appointment of Attorney General for India?


(A) He can be appointed by President of India and is qualified to be appointed a Judge of Supreme
Court
(B) He can be appointed by President of India and is qualified to be appointed a Judge of High Court
(C) He can be appointed by a Special Commission and is qualified to be appointed as Law Minister
(D) He can be appointed by Chief Justice of India and is qualified to be appointed as a Judge of
Supreme Court

24. The earliest Codified Laws (Civil and Criminal) that defined and demarcated for first time in India
during 335-345 A.D. was followed in the
(A) Huna reign (B) Sakka reign (C) Mughal reign (D) Gupta reign

Previous Years
CLAT & AILET Papers Page 201
25. Which of the following statements is/are not correct about the Objectives Resolution?
I. It was moved by Jawaharlal Nehru in the Constituent Assembly
II. It called for just right for minorities
III. It formed the basis for the chapter on Fundamental Rights
IV. It called for the establishment of a socialist and secular polity
(A) I and II (B) I, II and III (C) III and IV (D) Only III

26. The right to constitutional remedies in India is available to


(A) only the citizen of India
(B) all persons in case of infringement of any fundamental right
(C) any person for enforcing any rights conferred on them
(D) an aggrieved individual alone

27. Money bills can be introduced in the State Legislature with the prior consent of the
(A) President (B) Governor (C) Speaker (D) Chief Minister

28. While a proclamation of emergency is in operation, the State Government


(A) cannot legislate
(B) can legislate only on subjects in the Concurrent List
(C) can legislate on the subjects in the State List
(D) is suspended

29. A political party is recognised by the Election Commission only if


I. it has been engaged in political activity for a continuous period of five years
II. has returned at least one member of the Lok Sabha for every 25 members of that House or any
fraction of that number elected from that State
III. has polled not less than six per cent of the total number of valid votes polled by all contesting
candidates at the general elections
IV. has contested elections in four or more states in three consecutive general elections

(A) I and II (B) I, III and IV (C) I, II and III (D) I, II, III and IV

30. Electioneering has to stop in a constituency


(A) one day before the election (B) 48 hours before election starts
(C) 36 hours before a poll commences (D) 48 hours before the closing hour of polling

Directions (Q. 31- Q.35): Given below is a statement of principle followed by a factual situation. Apply the
principle to the facts given below and select the most appropriate answer.

31. PRINCIPLE:
In case of a breach of contract, compensation can be awarded for the personal inconvenience
suffered by a party by reason of the breach, which naturally arose in the usual course of things from
such breach, or which the parties knew, when they made the contract to be likely to result from the
breach of it.

Previous Years
Page 202 CLAT & AILET Papers
FACTS:
Sunita and Sushmita bought bus tickets for a journey from Adyar to Mandaveli. The bus was to go
to St. Thomas Mount via Mandaveli. However, the driver mistakenly took a wrong direction and the
two girls were dropped at a distance of 2½ miles from Mandaveli on the highway. With no other
transportation in sight nor a place to stay, the two had to walk 2½ miles at midnight.

Later they filed a case against the bus company and claimed Rs. 5000 as damages for inconvenience
caused in having to walk and Rs. 6500 for Sushmita having fallen ill by catching cold during the
night.

DECISION:
(A) Both the amounts are liable to be paid because Sunita and Sushmita suffered loss for no fault of
theirs.
(B) The bus company is liable to pay both the amounts claimed because the loss was suffered on
account of the fault of the bus company and the inconvenience suffered and illness arose was in
the normal course of things from breach of contract.
(C) The compensation for inconvenience suffered by being forced to walk at night is liable to be paid
by the bus company. However, no compensation for Sushmita’s illness because this was not
expected on account of breach of contract.
(D) The bus company is not liable to pay any amount, because it was the driver’s fault

32. FACTS:
In order to ensure that people live in an amicable atmosphere the Government of India decided to
abolish courts and constituted Dispute Settlement Boards. Further to achieve this objective, the law
stipulated that lawyers should not be allowed to espouse the claims of parties, and instead their
claims be espoused by social workers.

PRINCIPLES:
(I) Any law made by the Parliament that infringes the fundamental rights of the citizens is invalid
and unenforceable.
(II) Freedom to carry on trade or profession of one’s own choice is a fundamental right.
(III) The Parliament is competent to impose reasonable restrictions on the exercise of this right.
(IV)If the restrictions, on fundamental rights imposed by the Parliament, totally removes or nullifies
any fundamental right then it will be construed as an unreasonable restriction.

DECISION:
(A) The law made by the Parliament is valid as it does not infringe any fundamental right.
(B) The law made by the Parliament is valid as even though there is restriction of fundamental right,
such a restriction is reasonable.
(C) The law made by the Parliament is invalid as it constitutes an infringement of fundamental rights
and the restriction imposed is not reasonable.
(D) None of the above answer is correct.

Previous Years
CLAT & AILET Papers Page 203
33. PRINCIPLE:
A right to action cannot arise out of an illegal activity.

FACTS: A and B were thieves engaged in stealing cars and other vehicles. Once they stole a car;
and while driving off, they had to cross a city. They engaged a driver to drive them through the city,
since they did not know the route inside. The indicator lamp of the car was not working and the
thieves had not realised this, and therefore, had not told about it to the driver. While driving, through
the city, the car was hit by another vehicle because of the faulty indicator. In the accident, the driver
was injured and he filed a suit against A and B.

DECISION:
(A) The driver would lose, because he was driving a stolen car.
(B) The driver would win, because he was not a party to the stealing of car.
(C) The driver would win because he did not know anything about the stealing.
(D) None of the above answer is correct.

34. PRINCIPLE: A man would be responsible for all direct consequences of his act, in so far as he could
reasonably foresee them as arising from his act.

FACTS: A ship carrying petroleum was moving on the high sea. On a short halt in a port, the master
of the ship engaged some stevedores to load some metallic planks onto the ship. While loading the
planks, a plank slipped from the hands of stevedore and the spark, emitted thereby, ignited petroleum
vapour and caused considerable damage to the goods. The owner of the goods filed a suit against
the master of the ship.

DECISION:

(A) The master of the ship is not liable, because he was not responsible for the act of stevedore.
(B) The master of the ship is liable, because he is responsible for the acts of stevedore since he
engaged them.
(C) The master is liable, because he should have foreseen the consequences of the stevedore’s act.
(D) None of the above answer is correct.

35. PRINCIPLES:
All minorities, whether based on religion or language shall have the rights to establish and administer
educational institution of their choice.

FACS:
Md. Yusuf wants to establish an educational institution to help the poor person of his community
and to educate the children of the community. Yusuf being a very rich man has no problem regarding
finance for institution. Therefore he applies to state government to grant him permission to establish
and administer the institution. State government rejects his plea on ground that said institution will
create communal tension in the proposed area. Yusuf has following remedies:
(A) He should file a civil suit in District Court.
(B) He should file a special leave petition before Supreme Court.
(C) He has no remedy under the law.
(D) He could file a writ petition either before the High Court or the Supreme Court.

Previous Years
Page 204 CLAT & AILET Papers
SECTION – B : GENERAL KNOWLEDGE

36. What is the sequence in which the Indian economy has undergone the process of convertibility?
I. Rupee floated on trade account
II. Part convertibility under a dual exchange rate system
III. Introduction of exim scrips
IV. Rupee made convertible on current account
(A) I, II, III, IV (B) II, I, III, IV (C) III, II, I, IV (D) IV, I, II, III

37. Which of the following currency exchange rate mechanisms has been adopted by India?
(A) Free Float (B) Managed Float
(C) Floating with a Band (D) Fixed-but-Adjustable

38. The national income estimates of India are prepared by


(A) Ministry of Finance (B) National Sample Survey
(C) Central Statistical Organisation (D) Indian Statistical Institute

39. The ‘Year of Great Divide’ refers to


(A) rapid growth rate in population after 1921 (B) the declining sex ratio after 1921
(C) the slow down in death rate after 1921 (D) all of the above

40. Aurorae are a result of the interaction of earth’s magnetosphere with


(A) the solar wind (B) the ozone layer (C) radio waves (D) none of the above

41. You are standing in a lift. The force on the floor of the lift due to your weight will
(A) become zero when the lift moves up with an acceleration of 9.8 m/s2
(B) become zero when the lift moves down with an acceleration of 9.8 m/s2
(C) become zero when the lift moves with an acceleration of 9.8 m/s2 in any direction
(D) remain unchanged whatever be the value of acceleration of the lift

42. What happens to the level of mercury in a barometer tube when it is taken down a coal mine? It
(A) falls (B) remain unaltered (C) rises (D) rises and then falls

43. A popping sound is heard in the ear when yawning or swallowing. This is caused by
(A) air in the cochlea (B) the opening of the Eustachian tubes
(C) air entering the perilymph (D) the moving of the jaws

44. The body’s thermostat is in the


(A) pineal (B) hypothalamus (C) thyroid (D) pituitary

45. India is divided into how many PIN code zones?


(A) Six (B) Seven (C) Eight (D) Nine
46. Which of the following statements are correct with regard to the Indus Valley civilisation?
I. The people of this civilisation were the earliest to make use of cotton.
II. They did not cremate their dead.
III. A grid system was followed in city planning.
IV. Stone was the chief building material.
(A) I and II (B) II and III (C) III and IV (D) I and III

Previous Years
CLAT & AILET Papers Page 205
47. Which one of the following may be called a ‘political’ cause of the 1857 Revolt?
(A) The subsidiary alliance system of Wellesley
(B) The Religious Disabilities Act of 1856
(C) Low pay and poor prospects of the sepoys
(D) Dalhousie’s annexation policy

48. Who among the following Indian rulers established embassies in foreign countries on modern lines?
(A) Haider Ali (B) Mir Qasim (C) Shah Alam II (D) Tipu Sultan

49. Which one of the following may be called a ‘political’ cause of the 1857 Revolt?
(A) The subsidiary alliance system of Wellesley
(B) The Religious Disabilities Act of 1856
(C) Low pay and poor prospects of the sepoys
(D) Dalhousie’s annexation policy

50. The first time that Mahatma Gandhi tried out his method of satyagraha was
(A) in South Africa against racist policies (B) against indigo planters in Champaran
(C) in the Kheda no-rent campaign (D) at Bardoli

51. The Royal Indian Navy ratings revolted in February 1946 to protest against
(A) the treatment meted out to them
(B) the punishment meted out to the INA officials
(C) the firing on some ratings of the HMIS Talwar
(D) the Government’s suppression of Congress activities

52. he Preamble enshrines certain ideals that were first spelt out in
(A) the speech by Jawaharlal Nehru on the banks of Ravi when he called for Purna Swaraj
(B) the Nehru Report
(C) a resolution adopted at Karachi session of the Indian National Congress
(D) the Objectives Resolution adopted by the Constituent Assembly

53. A very rapid growth in prices in which money loses its value to the point where even barter may be
preferable is known as
(A) inflation (B) hyper-inflation (C) deflation (D) disinflation

54. Unlike the fluid core of the earth, the core of the moon is
(A) plasma (B) volatile gas (C) viscous liquid (D) solid

55. Which of the following theories has no connection with the origin of the earth?
(A) Nebular hypothesis of Laplace (B) Tidal hypothesis of Jeans and Jeffreys
(C) Binary star theory of Lyttleton (D) Convection current theory of Holmes

56. On what does the escape velocity of a body depend?


I. Mass of celestial body
II. The distance from the centre of mass to the escaping object
(A) Only I (B) Only II (C) Both I and II (D) Neither I nor II

57. The blood enters the aorta from


(A) left ventricle (B) left auricle (C) right ventricle (D) right auricle

Previous Years
Page 206 CLAT & AILET Papers
58. Desert plants would be characterised by
(A) vivipary (B) aerenchyma (C) aerial roots (D) sunken stomata

59. Why is sprouted gram considered more nutritious?


(A) Seeds are storehouses of energy
(B) Germinating seeds produce enzymes which are a rich source of protein
(C) Seeds have plenty of amino acids and glucose
(D) Sprouted gram is not more nutritious; it is just an old wives’ tale that it is

60. The edible portion of which of the following vegetables is not a stem?
(A) Potato (B) Sweet potato (C) Ginger (D) Corn of Colocasia

61. What is the venue for the 20th Commonwealth Games in 2014?
(A) Glasgow (B) Beijing (C) London (D) Italy

62. Who among the following leaders topped 2011 Forbes List of “World’s Most Powerful Person”?
(A) Mr. Barak Obama (B) Ms. Pratibha Patil
(C) Mr. Manmohan Singh (D) Mr. Hu Jintao

63. According to the Srikrishna Committee, which is the most workable option out of the six option
suggested by the Committee to resolve the Telengana issue?
(A) Second (B) Fourth (C) Sixth (D) First

64. Who is convicted to death in the Beant Singh assassination case on March 31, 2012?
(A) Kehar Singh (B) Balwant Singh Rajoana
(C) Karnail Singh (D) Dilawar Singh Babbar

65. Which news corporation was recently alleged to have been using illicit hacking into voicemail
messages of prominent people to find stories?
(A) News of the World (NoW) (B) BBC
(C) CNN (D) AajTak

66. Ms. Aung San Suu Kyi, Myanmar’s democracy icon, has been released from ——— of home
arrest.
(A) 16 years (B) 15 years (C) 10 years (D) 20 years

67. Which country became the 193rd member of UN, recently?


(A) South Sudan (B) Montenegro (C) Serbia (D) Tuvalu

68. More than 50 Sikh shrines in India are named after?


(A) Flowers (B) Trees (C) Birds (D) Animals

69. The country that set the record as the one with the longest time without government, in April 2011?
(A) Burundi (B) Belgium (C) Turkey (D) Taiwan

70. It has been the practice to have ——————as the Managing Director of the IMF.
(A) an American (B) a European (C) an Australian (D) an Arab

Previous Years
CLAT & AILET Papers Page 207
SECTION- C : REASONING

71. For one academic year, all the students at a high school were observed. The aim was to test the
hypothesis that studying more increased a student’s chances of earning a higher grade. It turned
out that the students who spent the most time studying did not earn grades as high as did many
students who studied less. Nonetheless, the researchers concluded that the results of the observation
supported the initial hypothesis.
Which one of the following, if true, most helps to explain why the researchers drew the conclusion
described above?
(A) The students who spent the most time studying earned higher grades than did some students
who studied for less time than the average.
(B) The students tended to get slightly lower grades as the academic year progressed.
(C) In each course, the more a student studied, the better his or her grade was in that course.
(D) The students who spent the least time studying tended to be students with no more than
average involvement in extracurricular activities.

72. Educator: It has been argued that our professional organization should take decision about important
issues - such as raising dues and taking political stands - by a direct vote of all members rather than
by having members vote for officers who in turn make the decisions. This would not; however, be the
right way to decide these matters, for the vote of any given individual is much more likely to determine
organizational policy by influencing the election of an officer than by influencing the result of a direct
vote on a single issue.
Which one of the following principles would, if valid, most help to justify the educator’s reasoning?
(A) No procedure for making organizational decisions should allow one individual’s vote to weigh
more than that of another.
(B) Outcomes of organizational elections should be evaluated according to their benefit to the
organization as a whole, not according to the fairness of the methods by which they are produced.
(C) Important issues facing organizations should be decided by people who can devote their full
time to mastering the information relevant to the issues.
(D) An organization’s procedures for making organizational decisions should maximize the power of
each member of the organization to influence the decisions made.

73. Mayor: Local anti-tobacco activists are calling for expanded antismoking education programs paid
for by revenue from heavily increased taxes on cigarettes sold in the city. Although the effectiveness
of such education programs is debatable, there is strong evidence that the taxes themselves would
produce the sought-after reduction in smoking. Surveys show that cigarette sales drop substantially
in cities that impose stiff tax increases on cigarettes.
Which one of the following, if true, most undermines the reasoning in the argument above?
(A) A city-imposed tax on cigarette will substantially reduce the amount of smoking in the city if the
tax is burdensome to the average cigarette consumer.
(B) Consumers are more likely to continue buying a product if its price increases due to higher
taxes than if its price increases for some other reason.
(C) Usually, cigarette sales will increase substantially in the areas surrounding a city after that city
imposes stiff taxes on cigarettes.
(D) People who are well informed about the effects of long-term tobacco use are significantly less
likely to smoke than are people who are not informed.

Previous Years
Page 208 CLAT & AILET Papers
74. Gotera: Infants lack the motor ability required to voluntarily produce particular sounds, but produce
various babbling sounds randomly. Most children are several years old before they can voluntarily
produce most of the vowel and consonant sounds of their language. We can conclude that speech
acquisition is entirely a motor control process rather than a process that is abstract or mental.
Which one of the following is an assumption required by Gotera’s argument?
(A) Speech acquisition is a function only of one’s ability to produce the sounds of spoken language.
(B) During the entire initial babbling stage, infants cannot intentionally move their tongues while
they are babbling.
(C) The initial babbling stage is completed during infancy.
(D) The initial babbling stage is the first stage of the speech acquisition process.

75. Reducing stress lessens a person’s sensitivity to pain. This is the conclusion reached by researchers
who played extended audiotapes to patients before they underwent surgery and afterward while
they were recovering. One tape consisted of conversation; the other consisted of music. Those who
listened only to the latter tape required less anaesthesia during surgery and fewer painkillers afterward
than those who listened only to the former tape.
Which one of the following is an assumption on which the researchers’ reasoning depends?
(A) All of the patients in the study listened to the same tape before surgery as they listened to after
surgery.
(B) Anticipating surgery is no less stressful than recovering from surgery.
(C) Listening to music reduces stress.
(D) The psychological effects of music are not changed by anaesthesia or painkillers.

Directions: In the following questions, each question has a statement followed by two conclusions. Taking
the statement to be true, decide which of the given conclusions definitely follow from the given statement.
Indicate your answer as (A) if only I follows, (B) if only II follows (C) if neither I, nor II follows and (D) if both
I and II follow.

76. Statement : The Supreme Court gave a judgement that the maintenance of old age parents is the
responsibility of the married girls, if they do not have brothers.
Conclusions I : Constitution is always interpreted to help oppressed people out.
II : Before the Supreme Court gave the verdict, a married girl must have denied to pay for the
maintenance to her parents.

77. Statement : Ideas given by our ancestors that were once discarded as uneconomical and unviable,
turn out to be as functional and inevitable in present circumstances.
Conclusions I : In ancient period, ideas were considered either completely functional or totally
infeasible.
II : Ideas cannot change from time to time.

Previous Years
CLAT & AILET Papers Page 209
Direction: In each question below, a statement is given followed by two assumptions numbered I and II. An
assumption is something supposed or taken for granted. You have to consider the statement and the
following assumptions and decide which of the assumptions is implicit in the statement. Mark answer:
(A) If only assumption I is implicit; (B) If only assumption II is implicit;
(C) If neither I nor II is implicit; and (D) If both I and II are implicit.

78. Statement:
“Use ‘M’ Brand shoes. They are available in all sizes and last longer”- an advertisement in the
newspaper ‘A’.
Assumption I:
Some people do not know about ‘M’ brand shoes.
Assumption II:
People generally prefer shoes which last longer.

79. Statement:
Lack of stimulation in the first four or five years of life can have adverse consequences.
Assumption I:
A great part of the development of observed intelligence occurs in the earliest years of life.
Assumption II:
50 per cent of the measurable intelligence at age 16 is predictable by the age of four.

Direction (Q. 80 – Q. 84): Read the following and answer the questions:
Six persons – P, Q, R, S, T and W work in an Organisation. Each of them likes different colours, viz.,
Black, White, Blue, Green, Red and Yellow and their salaries are also different. The person who earns
maximum does not like Blue or White colour. The person who earns the least does not like Black or Red
colour. R likes Yellow colour and his salary is the second maximum. P likes White colour and earns more
than T but less than W. Q likes Black colour who earns less than P but more than T. S likes Blue colour
and T likes Red colour.

80. How many of them do earn more than Q?


(A) One (B) Two (C) Three (D) Data inadequate

81. Who among them earns least?


(A) Q (B) P (C) R (D) S

82. If they are arranged in the descending order according to their salaries, who will occupy the third
position?
(A) P (B) R (C) T (D) Data inadequate

83. W likes the colour


(A) White (B) Green (C) Blue (D) Either White or Green

84. S likes which colour?


(A) White (B) Green (C) Blue (D) Red

Previous Years
Page 210 CLAT & AILET Papers
85. A child crawls 20 feet towards North, turns right and crawls 30 feet, turns right again and crawls 35
feet. He turns left now and crawls 15 feet. He turns left again and crawls 15 feet. Finally he turns to
his left to crawl another 15 feet. How far is he from his starting point and in which direction?
(A) ) 45 feet North-East (B) 30 feet East
(C) 30 feet West (D) 15 feet West

Directions: Read the following directions and answer the questions 86 and 87.
A is the father of C. But C is not his son. E is the daughter of C. F is the spouse of A. B is the brother of C.
D is the son of B. G is the spouse of B. H is the father of G.

86. Who is the son-in-law of H?


(A) C (B) A (C) D (D) B

87. Who is the grand-daughter of A?


(A) H (B) D (C) B (D) E

88. A is older by 4 years to B at one stage. After 16 years of this stage, A will be thrice his present age
and B will be five times his present age. How old would A and B be two years before the initially
indicated stage?
(A) 8 and 4 (B) 10 and 6 (C) 6 and 2 (D) 12 and 8

89. If day before yesterday was Tuesday, the day after tomorrow will be
(A) Monday (B) Wednesday (C) Friday (D) Saturday

90. My brother is 562 days older to me, while my sister is 75 weeks older to him. If my sister was born
on Tuesday, on what day was I born?
(A) Sunday (B) Monday (C) Tuesday (D) Wednesday

91. Ram is facing North-West. He turns in clockwise direction by 90°, then 180° in the anti-clockwise
direction and then another 90° in the same direction. Which direction is he facing now?
(A) South-West (B) West (C) South (D) South-East

92. One morning after sunrise, Vikram and Shailesh were standing in a lawn with their backs towards
each other. Vikram’s shadow fell exactly towards left hand side.
Which direction was Shailesh facing?
(A) East (B) West (C) North (D) South

Direction: In the following number series (Q. No. 93 & 94) only one number is wrong.
Find out that number:

93. 1, 2, 6, 15, 20, 30, 42


(A) 30 (B) 15 (C) 6 (D) 1

94. 2, 5, 10, 17, 26, 37, 50, 64


(A) 50 (B) 17 (C) 26 (D) 64

95. D, H, L, R, ?
(A) T (B) X (C) I (D) O

Previous Years
CLAT & AILET Papers Page 211
96. 3, 7, 15, 31, 63, ?
(A) 92 (B) 115 (C) 127 (D) 131

97. PLANING is coded in a certain language as UFFHSCSA. How will AUTHORITY be coded in the
same language?
(A) FOYBTLNND (B) FYOTBNNLT (C) FBOYTLNTN (D) FBOYTNLTN

98. In certain code ELECTION is written as GLGCVIQN, then VOTER will be coded as:
(A) XOVET (B) VOXET (C) WPUFU (D) VQTGR

99. CHILD : FIRE


(A) Aged : Wisdom (B) Student : Examination
(C) Youth : Adolescence (D) Judge : Dishonour

100. GRAIN : SALT


(A) Shred : Wool (B) Cave : Stone (C) Chip: Glass (D) Blades : Grass

Direction (Q. 101 – 105): Each group of questions is based on a set of conditions. Choose the response
that most accurately and completely answers each question.
Mercotek carried out a study to compare the productivity of its night shift with that of its day shift. Every
week the company’s six crews – F, G, H, R, S and T – were ranked from first (most productive) to sixth
(least productive). There were no ties. For any given week, either G and T were the two night-shift crews or
else S and H were – the four other crews were the day-shift crews for that week. The following relationships
held for every week of the study:
F is more productive than G
R is more productive than S
R is more productive than T
S is more productive than H
G is more productive than T

101. Which of the following could be an accurate ranking of all the crews, in order from first to sixth, for
a given week of the study?
(A) F, G, T, R, S, H (B) F, R, G, T, H, S (C) G, R, T, S, H, F (D) R, F, G, S, H, T

102. If F is ranked third for a given week of the study, then which one of the following could also be true
of that week?
(A) G ranks second (B) H ranks fourth (C) R ranks second (D) S ranks fourth

103. Which one of the following CANNOT be the crew ranked fifth for any given week of the study?
(A) G (B) H (C) R (D) S

104. For any given week of the study, the ranking of all the crews is completely determined if which one
of the following is true?
(A) F ranks second that week (B) G ranks fifth that week
(C) H ranks third that week (D) R ranks third that week
105. If the night-shift crews rank fifth and sixth for a given week of the study, then which one of the
following could also be true of that week?
(A) G ranks fourth (B) H ranks fifth (C) R ranks third (D) S ranks fourth

Previous Years
Page 212 CLAT & AILET Papers
SECTION – D : MATHEMATICS

106. A purse contains some coins consisting of rupees, fifty paise coins and twenty five paise coins. If
coins be in the ratio of 2 : 3 : 10 and their total value is Rs.72, the number of twenty-five paise coins
will be:
(A) 100 (B) 140 (C) 120 (D) 80

107. In an examination, a student who secured 25% of the maximum marks fails by 60 marks but
another candidate who secures 45% of the maximum marks gets 10 marks more than required
passing marks. The maximum number of marks is:
(A) 450 (B) 350 (C) 525 (D) none of these

108. In a tour, I spent every day as many ten rupee (notes) as the number of days I had been away from
the home. My total expenditure was Rs.18,300. How long did I stay away from the home?
(A) 1 month (B) 2 months (C) 3 months (D) 4 months

109. Before 3 years, the average age of a five-member family was 17 years. A baby having been born and
the average of family is now 17 years. The present age of the baby is:
(A) 3 years (B) 2 years (C) 1 year (D) none of these

110. A publisher sells books to retailer at marked price which is 20% above his outlay. If on cash
payment, he allows a discount of 10% on the marked price. The publisher thus gains:
(A) 12% (B) 10% (C) 8% (D) 14%

111. The average age of A, B and C is 25 years. The ratio of their ages is 3 : 5 : 7. Find the age of A
(A) 21 years (B) 18 years (C) 15 years (D) Data Inadequate

112. A certain sum of money was deposited in a bank and it became two-fold in 10 years. What is the
rate of simple interest?
(A) 8% (B) 10% (C) 12% (D) 13%

113. If Rs.80 amounts to Rs.140 in 4 years, what will Rs. 96 amount to in 10 years at the same rate of
interest per annum?
(A) Rs.276 (B) Rs.306 (C) Rs.386 (D) Rs.300

114. A group of workers accepted to do a piece of work in 25 days. If 6 of them did not turn up for the work
and the remaining workers did the work in 40 days, then the original number of workers was
(A) 22 (B) 18 (C) 20 (D) 16

115. Two taps ‘A’ and ‘B’ can fill a water reservoir in 8 and 6 hours respectively. A third tap ‘C’ can empty
the tank completely in 24 hours. How long would it take to fill the empty tank when all the taps are
open?
(A) 4 hours (B) 5 hours (C) 6 hours (D) 3 hours

Previous Years
CLAT & AILET Papers Page 213
SECTION – E : ENGLISH

Directions: In the following questions, some of the sentences have errors and some have none. Find out
which part of the sentence has an error.

116. The number of marks carried by each question / are indicated / at the end of the Question. / No error.
A B C D

117. As much as I admire him for his sterling qualities / I cannot excuse him for / being unfair to his
A B
friends. / No error.
C D

118. Many times the news has been published / in the papers that the end of the world will be certain /
A B
if a nuclear war breaks out. / No error.
C D

119. She reluctantly said that/if nobody else was doing it/she will do it./No error
A B C D

120. Though child marriage/has been banned, /the custom still prevailed among some groups in India./
A B C
No error.
D
Directions: In the following questions, choose the word which is most nearly the SAME in meaning to the
bold word and mark it in the Answer Sheet.

121. LYNCH
(A) Hang (B) Madden (C) Killed (D) Shoot
122. His speech was nothing but a string of platitudes.
(A) grand statements (B) stereo-typed statements
(C) noble sentiments (D) humorous anecdotes

Directions: In the following questions, choose the word which is most nearly the OPPOSITE in meaning
to the bold word and mark it in the Answer Sheet.

123. PERENNIAL
(A) Frequent (B) Regular (C) Lasting (D) Rare

124. My first speech was a fiasco.


(A) success (B) disaster (C) fun (D) joy

Direction: Fill in the blanks.

125. If I ...................his address, I could write to him.


(A) knew (B) had known (C) know (D) will know

Previous Years
Page 214 CLAT & AILET Papers
126. When the morning.................., murder was discovered.
(A) occurred (B) arrived (C) came (D) happened

127. I...................a car to be absolutely necessary these days.


(A) consider (B) regard (C) think (D) agree

128. Do not intrude, they are talking..........................a confidential matter.


(A) on (B) for (C) over (D) in

129. I slept after lunch.........................armed chair.


(A) over (B) into (C) in (D) on

130. He is so ................. that he immediately believed my story of ghosts.


(A) innocent (B) credulous (C) vociferous (D) credible

131. I will help only ......................


(A) if I shall have time (B) if I would have time (C) if I had time (D) if I have time

132. He doesn’t work with hands, he works ....................... the machine.


(A) with (B) by (C) at (D) on

133. Every Shakespearean hero has an internal ...................... in his character


(A) defect (B) weakness (C) fault (D) flaw
Direction (Q. 134 – 135) : In this section, each passage consists of six sentences. The first and the sixth
sentences are given in the beginning. The middle four sentences in each passage have been removed and
jumbled up. These are labelled P, Q, R and S. You are required to find out the proper sequence of the four
sentences and mark accordingly on the Answer Sheet.

134. S1 : Two men held a struggling crazed dog down on a table.


S6 : The men watched him awe-struck.
P : Its mouth was smothered with saliva and a bite from its jaws might cause death.
Q : He put one end of the tube between his lips and lowered the other towards the foam covered
jaws.
R : Beside them stood Louis Pasteur holding a narrow glass-tube in one hand.
S : As the animal writhed, he carefully sucked some of the saliva up the tube.
The proper sequence should be
(A) P R Q S (B) P Q R S (C) S R P Q (D) R P S Q
135. S1 : There is only one monkey we can thoroughly recommend as an indoor pet
S6 : Finally, let me say that no other monkey has a better temper or more winning ways.
P : They quickly die from colds and coughs after the first winter fogs.
Q : It is the beautiful and intelligent Capuchin monkey.
R : The lively little Capuchins, however, may be left for years in an English house without the least
danger to their health.
S : The Marmosets, it is true, are more beautiful than the Capuchins and just as pleasing, but they
are too delicate for the English climate.
The proper sequence should be:
(A) P Q R S (B) Q R P S (C) Q S P R (D) R P S Q

Previous Years
CLAT & AILET Papers Page 215
Direction (Q. 136-145): Fill in the blanks
The Ganga is........(136).......they said, in her giving and her taking away. If you ask her for anything
she.......(137)......it gently. A calm and slow oblivion in...........(138).............. ever-flowing waves. Mridula
hoped it was true. She sat on the stone steps of the Kedar Ghat, ...........(139).................the first rays of
the Sun touch the ripples on the river with............(140)........of metallic gold. Ganga paschim vahini – the
east flowing river ...............(141)................... west at Varanasi like a moody woman. People bathed, said
prayers standing chest ........(142)................ in her water, muttering ..............(143)..............to her and to
the rising sun, repeating endlessly the .................(144)........... of prayer pouring water through their finger
in habits for which may be they never .............(145)........the meaning.

136. (A) kind (B) name (C) flow (D) cruel

137. (A) takes (B) refuse (C) ignore (D) gives

138. (A) their (B) her (C) any (D) only

139. (A) counting (B) persuing (C) dotting (D) watching

140. (A) dullness (B) dampness (C) glints (D) splint

141. (A) steers (B) hears (C) looks (D) turns

142. (A) deep (B) down (C) up (D) fallen

143. (A) appeals (B) pleas (C) invocations (D) considerations

144. (A) culture (B) rituals (C) works (D) deed

145. (A) mentioned (B) heard (C) conceived (D) knew

Directions (Q. 146 – 150): The questions in this section are based on the passage. The questions are to
be answered on the basis of what is stated or implied in the passage. For some of the questions, more
than one of the choices could conceivably answer the question. However, you are to choose the best
answer; that is, the response that most accurately and completely answers the questions.
In principle, a cohesive group - one whose members generally agree with one another and support one
another’s judgments – can do a much better job at decision making than it could if it were non-cohesive.
When cohesiveness is low or lacking entirely, compliance out of fear of recrimination is likely to be strongest.
To overcome this fear, participants in the group’s deliberations need to be confident that they are members
in good standing and that the others will continue to value their role in the group, whether or not they agree
about a particular issue under discussion. As members of a group feel more accepted by the others, they
acquire greater freedom to say what they really think, becoming less likely to use deceitful arguments or
to play it safe by dancing around the issues with vapid or conventional comments. Typically, then, the more
cohesive a group becomes, the less its members will deliberately censor what they say out of fear of being
punished socially for antagonizing their fellow members.
But group cohesiveness can have pitfalls as well: while the members of a highly cohesive group can feel
much freer to deviate from the majority, their desire for genuine concurrence on every important issue often
inclines them not to use this freedom. In a highly cohesive group of decision makers, the danger is not that
individuals will conceal objections they harbour regarding a proposal favoured by the majority, but that they

Previous Years
Page 216 CLAT & AILET Papers
will think the proposal is a good one without attempting to carry out a critical scrutiny that could reveal
grounds for strong objections. Members may then decide that any misgivings they feel are not worth
pursuing – that the benefit of any doubt should be given to the group consensus. In this way, they may fall
victim to a syndrome known as “groupthink”, which one psychologist concerned with collective decision
making has defined as “a deterioration of mental efficiency, reality testing, and moral judgment that results
from in-group pressures”.
Based on analyses of major fiascos of international diplomacy and military decision making, researchers
have identified groupthink behaviour as a recurring pattern that involves several factors: overestimation of
the group’s power and morality, manifested, for example, in an illusion of invulnerability, which creates
excessive optimism; closed-mindedness to warnings of problems and to alternative viewpoints; and
unwarranted pressures toward uniformity, including self-censorship with respect to doubts about the group’s
reasoning and a concomitant shared illusion of unanimity concerning group decisions. Cohesiveness of
the decision-making group is an essential antecedent condition for this syndrome but not a sufficient one,
so it is important to work toward identifying the additional factors that determine whether group cohesiveness
will deteriorate into groupthink or allow for effective decision making.

146. Why does the author thinks that the cohesive group can do a much better job at decision making
than it could if it were non-cohesive?
(A) The members of a highly cohesive group can feel much freer to deviate from the majority.
(B) Individuals will not conceal objections they harbour regarding a proposal favoured by the majority.
(C) Participants in the group’s deliberations are confident that they are members in good standing
and that the others will continue to value their role in the group, whether or not they agree about
a particular issue under discussion.
(D) All of the above.

147. Which one of the following most accurately expresses the main point of the passage?
(A) Despite its value in encouraging frank discussions, high cohesion can lead to a debilitating type
of group decision making called groupthink.
(B) Group members can guard against groupthink if they have a good understanding of the critical
role played by cohesion.
(C) Groupthink is a dysfunctional collective decision-making pattern that can occur in diplomacy
and military affairs.
(D) Low cohesion in groups is sometimes desirable when higher cohesion involves a risk of groupthink
behaviour.

148. A group of closely associated colleagues has made a disastrous diplomatic decision after a series
of meetings marked by disagreement over conflicting alternatives. It can be inferred from the passage
that the author would be most likely to say that this scenario
(A) provides evidence of chronic indecision, thus indicating a weak level of cohesion in general.
(B) indicates that the group’s cohesiveness was coupled with some other factor to produce a
groupthink fiasco.
(C) provides no evidence that groupthink played a role in the group’s decision.
(D) provides evidence that groupthink can develop even in some groups that do not demonstrate an
“illusion of unanimity”.

Previous Years
CLAT & AILET Papers Page 217
149. The passage mentions which one of the following as a component of groupthink?
(A) unjustified suspicions among group members regarding an adversary’s intentions.
(B) strong belief that the group’s decisions are right.
(C) group members working under unusually high stress, leading to illusions of invulnerability.
(D) the deliberate use of vapid, clichéd arguments.

150. It can be inferred from the passage that both the author of the passage and the researchers mentioned
in the passage would be most likely to agree with which one of the following statements about
groupthink?
(A) Groupthink occurs in all strongly cohesive groups, but its contribution to collective decision
making is not fully understood.
(B) The casual factors that transform group cohesion into groupthink are unique to each case.
(C) The continued study of cohesiveness of groups is probably fruitless for determining what factors
elicit groupthink.
(D) On balance, groupthink cannot be expected to have a beneficial effect in a group’s decision making.
*****

Previous Years
Page 218 CLAT & AILET Papers
AILET Question Paper 2013
SECTION – A : ENGLISH

Directions (Q. 1 - 6) : The questions in this section is based on the passage. The questions are to be
answered on the basis of what is stated or implied in the passage. For some of the questions, more than
one of the choices could conceivably answer the question. However, you are to choose the best answer;
that is, the response that most accurately and completely answers the question.
Although the legal systems of England and the United States are superficially similar, they differ
profoundly in their approaches to and uses of legal reasons: substantive reasons are more common
than formal reasons in the United States, whereas in England the reverse is true. This distinction
reflects a difference in the visions of law that prevails in the two countries. In England, the law has
traditionally been viewed as a system of rules; the United States favours a vision of law as an outward
expression of community’s sense of right and justice.
Substantive reasons, as applied to law, are based on moral, economic, political and other
considerations. These reasons are found both “in the law” and “outside the law” so to speak. Substantive
reasons inform the content of a large part of the law: constitutions, statutes, contracts, verdicts, and
the like. Consider, for example, a statute providing that “no vehicles shall be taken into public parks.”
Suppose that no specific rationales or purposes were explicitly written into the statute, but that it
was clear (from its legislative history) that the substantive purpose of the statute was to ensure quiet
and safety in the park. Now suppose that a veterans’ group mounts a World War II jeep (in running
order but without a battery) as a war memorial on a concrete slab in the park, and charges are
brought against its members. Most judges in the United States would find the defendants not guilty
because what they did had no adverse effect on park’s quiet and safety.
Formal reasons are different in that they frequently prevent substantive reasons from coming into
play, even when substantive reasons are explicitly incorporated into the law at hand. For example,
when a document fails to comply with stipulated requirements, the court may render the document
legally ineffective. A Will requiring written witness may be declared null and void and, therefore,
unenforceable for the formal reason that the requirement was not observed. Once the legal rule - that
a Will is invalid for lack of proper witnessing - has been clearly established, and the legality of the
rule is not in question, application of that rule precludes from consideration substantive arguments in
favour of the Will’s validity or enforcement.
Legal scholars in England and the United States have long bemused themselves with extreme
examples of formal and substantive reasoning. On the one hand, formal reasoning in England has led
to wooden interpretations of statutes and an unwillingness to develop the common law through judicial
activism. On the other hand, freewheeling substantive reasoning in the United States has resulted in
statutory interpretations so liberal that the texts of some statutes have ignored.
1. Which one of the following best describes the content of the passage as a whole?
(a) An analysis of similarities and differences between the legal systems of England and the United
States
(b) A re-evaluation of two legal systems with the use of examples
(c) A contrast between the types of reasons embodied in the United States and English legal
systems
(d) An explanation of how two distinct visions of the law shaped the development of legal reasoning.

Previous Years
CLAT & AILET Papers Page 219
2. It can be inferred from the passage that English judges would like to find the veterans’ group discussed
in the second paragraph guilty of violating the statute because
(a) not to do so would encourage others to act as the group did
(b) not to do so would be to violate the substantive reasons underlying the law
(c) the veterans failed to comply with the substantive purpose of the statute
(d) the veterans failed to comply with the stipulated requirements of the statute

3. From the discussion of Wills in the third paragraph it can be inferred that substantive arguments as
to the validity of a Will might be considered under which one of the following circumstances?
(a) The legal rule that a Will be witnessed in writing does not stipulate the format of the V
(b) The legal rule requiring that a Will be witnessed stipulates that the Will must be witnessed in
writing by two people
(c) The legal rule requiring that a Will be witnessed in writing stipulates that the witnessing must be
done in the presence of a judge
(d) A judge rules that the law can be interpreted to allow for a verbal witness to a Will in a case
involving a medical emergency

4. Which one of the following best describes the function of the last paragraph of the passage?
(a) It presents the consequences of extreme interpretations of the two types of legal reasons
discussed by the author
(b) It shows how legal scholars can incorrectly use extreme examples to support their views
(c) It corrects inaccuracies in legal scholars’ view of the nature of two types of legal systems
(d) It suggests how characterisations of the two types of legal reasons can become convoluted and
inaccurate

5. The author of the passage suggests that in English law a substantive interpretation of a legal rule
might be warranted under which one of the following circumstances?
(a) Social conditions have changed to the extent that to continue to enforce the rule would be to
decide contrary to present-day social norms
(b) The composition of the legislature has changed to the extent that to enforce the rule would be
contrary to the views of the majority in the present legislative assembly
(c) The legality of the rule is in question and its enforcement is open to judicial interpretation
(d) Individuals who have violated the legal rule argue that application of the rule would lead to unfair
judicial interpretations

6. The author of the passage makes use of all of the following in presenting the discussion of the
English and the United States legal systems except
(a) Comparison and contrast (b) Generalisation
(c) Explication of terms (d) A chronology of historical development

Directions (Q. 7 - 11): In the following questions, a group of sentences about a single topic are given. One
or more of the sentence(s) is/are grammatically incorrect. You have to identify the incorrect sentence(s).

7. I) It began with acquisitions in information technology and related services sector.


II) In pharmaceuticals, Wockhardt has bought C.P. Pharma of the United Kingdom for $ 10.85
million.
III) Tata Tea has taken over Tetley of the UK, the world’s biggest tea bag maker, for $ 430 million.
IV) With the processes, it has become the world’s second largest tea company.
(a) II and IV (b) IV only (c) II and III (d) I, II and IV

Previous Years
Page 220 CLAT & AILET Papers
8. I) There are two main reasons for that predatory mood.
II) Having established a domestic presence, the component makers are now looking for an
international presence.
III) Second, having improved their productivity, quality and reliability, Indian companies feel more
confident about spreading their wings abroad.
IV) Various other factors are being attributed to this Indian penchant for the takeover game in all
sectors.

(a) I only (b) I and II (c) II only (d) III and IV

9. I) Moving one by one step away from the expected with the graphics and photography can also
create reader’s interest.
II) Try using a conceptual image or photo to highlight your main message versus very first thing to
come to mind when thinking about your product or services.
III) Another form of contrast is in the actual design.
IV) An unusual fold in a brochure or direct mail piece can add excitement.

(a) I and II (b) II and III (c) I and IV (d) No error

10. I) The typeface that you choose for your print project is an important piece of the foremost overall
design process.
II) First, narrow down your choice by selecting the tone you want to present.
III) Typefaces can convey personality.
IV) For instance, if you are in the banking industry you might choose a classic serif font, such as
Garamond, to convey dependability.

(a) I only (b) II only (c) III only (d) IV only

11. I) Readability is crucial.


II) Be sure of the font we choose is legible and logical.
III) With all of the newest and interesting typefaces available today, it is tempting to pick one that
you think looks “cool”.
IV) This can work if you are going for an edgy look that will appeal to a young audience, but your
copy still needs to be easily understood.

(a) I and IV (b) II only (c) III only (d) II and III

Direction (Q. 12 - 15) : In the following questions, choose the word which is most nearly the same in
meaning to the bold word and mark it in the Answer Sheet.

12. Aberration
(a) Deviation (b) Embarrassment (c) Abhorrence (d) Absence

13. Potpourri
(a) Medley (b) Dose (c) Weird (d) Overabundance

14. Imposture
(a) Claim (b) Status (c) Destruction (d) Deception

Previous Years
CLAT & AILET Papers Page 221
15. Parley
(a)Discuss (b) Deliver (c) Sweeten (d) Race

Direction (Q. 16 -19) : In the following questions, choose the word which is most nearly the opposite in
meaning to the bold word and mark it in the Answer Sheet.

16. Protean
(a) Versatile (b) Amateur (c) Dull (d) Cautious

17. Predilection
(a) Antipathy (b) Ignorance (c) Dissonance (d) Disharmony

18. Impalpable
(a) Visible (b) Audible (c) Tangible (d) Fearless

19. Parochial
(a) Dogmatic (b) Dominant (c) Cosmopolitan (d) Niggardly

Direction (Q. 20 - 32) : Fill in the blanks.


20. The event passed ___________ without any untoward incident.
(a) of (b) on (c) off (d) away

21. Please give me __________ to drink.


(a) little water (b) a little water (c) any water (d) some water

22. Her true feelings manifested themselves in her sarcastic asides; only then was her ___________
revealed.
(a) sweetness (b) bitterness (c) anxiety (d) charm

23. The tapeworm is an example of _______________ organism, one that lives within or on another
creature, deriving some or all its nutrients from its host.
(a) a protozoan (b)a parasite (c) a hospitable (d) an autonomous

24. While the disease is in a latent state it is almost impossible to determine its existence by
(a)observation (b) analysis (c) examination (d) estimate

25. Language, culture and personality may be considered independently of each other in thought, but
they are _________ in fact.
(a) pervasive (b) inseparable (c) autonomous (d) immutable

26. The country is ushering ___________ a new era.


(a) into (b) in (c) of (d) over

27. He is a traitor ___________ the country.


(a) for (b) to (c) in (d) of

28. There is no culture in the world __________ absolutely pure.


(a) that which is into (b) that be (c) that is (d) that what is

Previous Years
Page 222 CLAT & AILET Papers
29. He led me ____________ the green lawn to the palatial building.
(a) upon (b) across (c) along (d) on

30. The lease of our premises has ____________ and we have to vacate it.
(a) run out (b) run off (c) run over (d) run down

31. His boss ____________ an explanation of his conduct with his colleagues.
(a) called up (b) called upon (c) called for (d) called off

32. He ____________ his departure for a week as his mother was not well.
(a) put off (b) put up (c) put out (d) put aside

Directions (Q. 33 - 35) : In this section, each passage consists of five sentences. The first sentence is
given in the beginning. The four sentences in each passage have been jumbled up. These are labeled P, Q,
R and S. You are required to find out the proper sequence of the four sentences and mark accordingly on the
Answer Sheet.

33. S1 : A suicide bomber killed 18 people near a hospital of Baghdad, keeping violence on the boil after
Iraq’s landmark election and ahead of an important Shia religious ceremony.
P : The bomber drove his vehicle towards local government offices and a hospital in the town, but
detonated it outside blast walls protecting buildings.
Q: Around 25 people were wounded in the attack, the second suicide car bombing in as many
days.
R: “Looking at the partial result, it appears the Sistani list will have over 50% and Kurdish parties
will come second,” said a Sunni politician.
S : Meanwhile, the two electoral power houses representing Iraq’s Shia and Kurdish communities
are poised to clinch the country’s two top jobs, with results from the landmark January 30 polls
expected anytime.

The proper sequence should be


(a) SRPQ (b) SQPR (c) PQSR (d) QPSR

34. S1 : It should be noted that Lenin, being the head of the government. held no official posts in the
Party Central Committee, but presided over sessions of the Central Committee plenary meetings
and also of the Political Bureau.
P : In performing his party duties, he was assisted by a secretary of the Central Committee, or the
head of the Secretariat.
Q: Officially, such a post did not exist at the time, but in practice one of the secretaries was
expected to
R: That being the case, Lenin not only heads the Council of People’s commissars, but was also
the de facto leader of the Party’s Central Committee.
S: When Lenin’s health deteriorated, the question arose of strengthening the secretariat and an
authoritative leader has to be found who could supervise Perth’s affairs in Lenin’s absence.

The proper sequence should be


(a) PQRS (b) SRPQ (c) SQPR (d) RPQS

Previous Years
CLAT & AILET Papers Page 223
35. S1: There is nothing in the world fascinating than watching a child grow and develop.
P: At first you think of it as just a matter of growing bigger.
Q: Then, as the infant begins to do things, you may think of it as “learning tricks”.
R: In some ways, the development of each child retraces the whole history of the human race,
physically and spiritually, step by step.
S: But, it’s really more complicated and full of meaning than that.

The proper sequence should be


(a) PQSR (b) RPQS (c) PQRS (d) RPSQ

SECTION-B : GENERAL NNOWLEDGE

36. The sun reaches its maximum angular distance from the equator at the
(a) zenith (b)solstice (c) equinox (d) noon time

37. The difference in the duration of day and night increases as one move from
(a) West to East (b) East and West of the prime meridian
(c) Poles to equator (d) Equator to poles

38. When can one record the lowest temperature of air ?


(a) Just before sunrise (b) At midnight
(c) At 3 a.m. (d) At sunrise

39. What is the difference between a geyser and a hot spring ?


(a) Water is ejected explosively in a geyser
(b) Water from a geyser may be cold
(c) Geysers are formed on volcanic mountains
(d) Geysers are more common in cold countries

40. Sher Shah is well-known for his administrative skill, especially his
(a) Market control steps (b) Land revenue system
(c) Mansabdari system (d) Law and order’

41. A monument resembling Taj Mahal was created by Aurangzeb in


(a) Ahmedabad (b) Aurangabad (c) Hoshangabad (d) Daulatabad

42. The maximum work in popularising female education in the nineteenth century was done by
(a) Ishwar Chandra Vidyasagar (b) J.E.D. Bethune
(c) D.K. Karve (d) Raja Ram Mohan Roy

43. The most learned ruler of the Delhi Sultanate who was well versed in various branches of learning
including Astronomy, Mathematics and Medicine was
(a) Iltutmish (b) Alauddin Khalji
(c) Muhammad Bin Tughlaq (d) Sikandar Lodhi

44. Permanent Settlement was introduced to


(a) help the ryots (b) earn a fixed and regular revenue
(c) help the landlords (d) improve the prospects of agriculture

Previous Years
Page 224 CLAT & AILET Papers
45. Who is, presently, the Director of the National Judicial Academy ?
(a) Prof. N.R. Madhava Menon (b) Dr. Balram K. Gupta
(c) Prof. M.P. Singh (d) Prof. K.N.Chandrasekharan Pillai

46. Which of the following is/are listed among the Directive Principles in Part IV of the Constitution of
India?
I) Equal pay for equal work
II) Uniform Civil Code
III) Small family norm
IV) Education through mother tongue at primary level

(a) I, II and Ill (b) l and II (c) II and III (d) I, II and IV

47. Who decides whether a Member of Parliament is subject to any disqualification?


(a) President (b) Speaker
(c) Election Commissioner (d) None of the above

48. The 9th Postal Zone of India covers


(a) Andhra Pradesh (b) Army Post Office
(c) Goa (d) Andaman and Nicobar Islands

49. Regarding No-Confidence Motion


(a) it is expressed against an individual minister or council of ministers
(b) no grounds have to be set out for the motion
(c) not less than 100 members have to support it for the Speaker to grant leave for its introduction
(d) it is always brought against the Prime Minister

50. Who is responsible for the


(a) Government (b) Voters (c) Political parties (d) Election Commission

51. The term fiscal deficit means


(a) total receipts minus expenditure
(b) total receipts minus interest payments on external debt
(c) revenue receipts minus expenditure
(d) revenue receipts minus defence expenditure

52. The functions of the Reserve Bank of India are


I) issuing all notes and coins
II) distributing all notes and coins
III) formulating monetary policy
IV) acting as agent of government in respect of India’s membership of the IMF

(a) I, II and III (b) II and III (c) II, III and IV (d) I, II, III and IV

53. What do you understand by Bear Raid ?


(a) An attempt to bring down the price of strong short selling
(b) Simultaneous buying of shares and debentures in view of getting more values in near future
(c) Higher rate of price paid for particular government share or debentures
(d) Any of the above

Previous Years
CLAT & AILET Papers Page 225
54. The foreign exchange reserves of India include
(a) Gold (b) SDRs (c) Foreign currencies (d) All of these

55. Among the remedies of inflation, we cannot include


(a) better capacity utilisation (b) lowering bank rate
(c) reducing budgetary deficit (d) an efficient public distribution system

56. Which one of the following is the most effective carrier of communications ?
(a) Cables (b) Radio waves (c) Microwaves (d) Optical fibres

57. Which one of the following compounds is used as a sedative ?


(a) Potassium Bromide (b) Calcium Chloride
(c) Ethyl Alcohol (d) Phosphorus Trichloride

58. In a fire siren, a jet of air is directed against a series of evenly spaced holes in a rotating disc. As the
disc speeds up, the tone
(a) maintains constant pitch (b) drops in pitch
(c) increases in wavelength (d) increases in frequency

59. After death, the


I) arteries are usually empty of blood
II) blood in arteries clots
III) veins are empty of blood
IV) veins are full of clots

(a) I and II (b) I only (c) I and IV (d) III only

60. A victim of a road accident is unconscious. Put in correct order the steps in First- Aid.
I) Treating for cardiac arrest
II) Treating for asphyxia
III) Treating for shock
IV) Arrest Haemorrhage
V) Cleanse and cover wounds

(a) I, II, III, IV, V (b) II, I, IV, Ill, V (c) II, III, I, IV, V (d) V, I, Ill, IV

61. Which one of the following international organisations does India disapprove of ?
(a) ASEAN (b) NATO (c) FAO (d) OPEC

62. Which space agency released surprising pictures of the remains of huge river which ran across
Mars at some point of time on January 17, 2013 ?
(a) NASA (b) European Space Agency
(c) Australian Space Agency (d) ISRO

63. Name the Indian woman weight-lifter who in January 2013 won Maiden National Title in National
Weightlifting Championship, 2013.
(a) Binitha Devi (b) Amanpreet Kaur (b) Manpreet Kaur (d) Karnam Malleswari

Previous Years
Page 226 CLAT & AILET Papers
64. Which of the following Indian company got the top rank among Indian companies in Top 500
companies list of Fortune India ?
(a) Indian Oil Corporation (b) ONGC
(c) RIL (d) SAIL

65. The Report of Transparency International India released in December, 2012 places India at _________
rank among 176 nations of the basis of Corruption Percentage Index.
(a) 93rd (b) 94th (c) 95th (d) 96th

66. Who is crowned as Pond’s Femina Ms. India, 2013?


(a) Zoya Afroz (b) Sobhita Dhulipala
(c) Navneet Kaur Dhillon (d) Anukriti Gusain

67. Which Hindi film was awarded 60th National Film Award, 2013 for Best Feature Film
(a) Paan Singh Tomar (b) Barfi
(c) Gangs of Wasseypur (d) Vicky Donor

68. Who is elected as the 266th Pope of the Roman Catholic Church at Rome In March, 2013?
(a) John Paul I (b) John Paul II (c) Benedict XVI (d) Francis

69. Which country joined WTO in August 2012 ?


(a)Russia (b) Belgium (c) Turkey (d) Taiwan

70. Which of the following are included in the definition of Sexual Harassment under the new Sexual
Harassment of Women at Workplace (Prevention Prohibition and Redressal) Bill?
(a) Physical contact and advances (b) Demand or request for sexual favours
(d) Making sexually coloured remarks (d) All of the above

SECTION - C : LEGAL APTITUDE

Directions (Q. 71 - 85) : Given below is a statement of legal principle followed by a factual situation. Apply
the principle to the facts given below and select the most appropriate answer.

71. LEGAL PRINCIPLE: If a person brings anything dangerous on his land which may prove harmful if
escapes, then that person must keep it at his peril. If a man fails to do so then he must be made
responsible to all natural consequences of its

FACTUAL SITUATION : A grows poisonous trees on his own land and lets the projection of the
branches of his trees on the B’s land. B’s cattle die because of nibbling the poisonous leaves.

DECISION :
(a) A is not liable to B because B must have taken due care to control his cattle
(b) A is not liable to B because trees are still on A’s land and there is no escape of dangerous thing
(c) A is liable to B because projection of branches with poisonous leaves amounts to escape
(d) A is not liable to B because he is not acting negligently

Previous Years
CLAT & AILET Papers Page 227
72. LEGAL PRINCIPLE : Master/Principal is vicariously liable for the tort committed by an servant/
agent, in the performance of his duties as an servant/agent.

FACTUAL SITUATION : A gave some cash and cheques to his friend B, who was an employee of
the State Bank of India, to deposit the same in that Bank in the account of A. B misappropriated the
amount. If A sues the Bank for damages, then the Bank is

DECISION :
(a) Liable to pay because it was the employer of B
(b) Liable to pay because the employee did it during business hours and while working as an
employee
(c) Not liable because he turned out to be the friend of the plaintiff’s husband Not liable because
while committing the fraud, he was not acting as the agent or employee of the Bank

73. LEGAL PRINCIPLE : Master / Principal is vicariously liable for the tort committed by an servant /
agent, in the performance of his duties as an servant / agent.

FACTUAL SITUATION : The plaintiff a bullion merchant was arrested by the police on a charge of
purchasing stolen goods. Some of the gold and silver ornaments were seized from the plaintiff and
were kept in the police station custody. The duty constable appropriated the gold ornaments and
escaped to a foreign country. The plaintiff after being acquitted brought an action against the State
for the compensation. In this case, compensation is

DECISION :
(a) Payable as there is misappropriation by the servants of the State
(b) Payable due to the fact that police constable has escaped to a foreign country
(c) Payable by the police constable himself and not by the State
(d) Not payable as the act was committed in discharge of sovereign function

74. LEGAL PRINCIPLES :


1) Joint tort-feasers means joint wrong doers. People can be joint tort-feasors in case of common
action, in fact or in law.
2) Joint tort-feasers are jointly and severally liable.

FACTUAL SITUATION : Two dogs belonging to two different owner acting in concert attacked a
flock of sheep and injured several sheeps. In an action for damages brought against the owners of
the dogs, if one of them puts a defence claiming that he was liable for one half only of the damage,
then wt one of the following statements is legally sustainable in the above case ?

DECISION :
(a) Neither of the owners is liable for damages done by his dog J
(b) Each owner was responsible for one half of the damage
(c) The owners themselves are not joint tort-feasers
(d) None of the above

Previous Years
Page 228 CLAT & AILET Papers
75. LEGAL PRINCIPLES :
1) Negligence is the omission to do something which a reasonable man, guided upon those con-
siderations which ordinarily regulate the conduct of human affairs, would do or doing something
which a prudent or reasonable man would not do.
2) Defendant’s duty of care depends on the reasonable foreseeability of injury which may be caused
to the plaintiff on breach of duty.

FACTUAL SITUATION : The defendants employees of the Municipal Corporation opened a manhole
in the street and in the evening left the manhole open and covered it by a canvass shelter, unattended
and surrounded by warning lamps. The plaintiff, an eight years old boy, took one of the lamps into
the shelter and was playing with it there, when he stumbled over it and fell into the manhole. A violent
explosion followed and the plaintiff suffered burn injuries. The defendants are

DECISION :
(a) Not liable because the injury to plaintiff is not foreseeable
(b) Liable because they should have completed the work before they left
(c) Not liable because they acted reasonably
(d) Liable because they acted unreasonably

76. LEGAL PRINCIPLES :


1) Master / Principal is vicariously liable for the tort committed by an servant / agent, in the performance
of his duties as an servant / agent.
2) Negligence is the omission to do something which a reasonable man, guided upon those
considerations which ordinarily regulate the conduct of human affairs, would do or doing something
which a prudent or reasonable man would not do.

FACTUAL SITUATION : A patient is brought to a hospital maintained by B. The patient is to be


operated upon. As a result of faulty oxygen supply, the patient dies on the operation theatre table,
then

DECISION :
(a) B would not be liable because the surgeon was negligent
(b) B would be liable because there is master and servant relationship between B and the surgeon
(c) B would not be liable because there is no_ master and servant relationship between B and the
surgeon
(d) B would not be vicariously liable because surgery is a highly skilled work on which B would have
no control

77. LEGAL PRINCIPLE: Defamation means publication of a false and derogatory statement about
another person without lawful justification.

FACTUAL SITUATION: A writes a defamatory letter to B containing defamatory remarks in reference


of B in Urdu language. A is aware that B does not know Urdu. B goes to C who knows Urdu and the
letter is read over by C to him. B sues A for defamation.

DECISION :
(a) A is liable
(b) A is not liable because he addresses the letter to B and not to C
(c) A is not liable because there is no publication of defamatory statement
(d) A is not liable because he is unaware that the letter can be read over by someone else to B

Previous Years
CLAT & AILET Papers Page 229
78. LEGAL PRINCIPLES: A private nuisance may consist of :
1) Any interference with a person’s use or enjoyment of land.
2) The act of wrongfully causing or allowing the escape of deletes things into another person’s land
e.g. water, smoke, smell, etc.
FACTUAL SITUATION: D erected a brick grinding machine adjoining the premises of P, a medical
practitioner. The dust from the machine polluted the atmosphere and caused inconvenience to P
and his patients. Here
DECISION:
(a) P cannot stop D because D is carrying on lawful business
(b) D has fundamental right to carry on any kind of business
(c) P can claim compensation because D’s activity amounts to nuisance for P
(d) D can claim compensation from P because P is trying to maliciously prosecute D

79. LEGAL PRINCIPLE: False imprisonment is the confinement of a person without just cause or
excuse. There must be a total restraint of the person and the onus of proving reasonable cause is on
the defendant.
FACTUAL SITUATION: A entered in B’s park where there was an artificial lake for the boating. A
paid Rs. 100 for entering the park and has to pay Rs. 100 at the time of exit. A waited for 30 minutes
but no boat was available. A came out, however, denied to pay Rs. 100 for exit, B did not allow A to
leave the park unless he paid Rs. 100 for exit. A sued B for false imprisonment.
DECISION:
(a) B is guilty of false imprisonment
(b) B is not guilty of false imprisonment
(c) A can lawfully refuse to pay Rs. 100 when no boat was available
(d) A can ask for even Rs. 100 given for entering the park as B’s services are deficient in the park
and can sue B for false imprisonment

80. LEGAL PRINCIPLES: In a suit for malicious prosecution, the plaintiff must prove the following
essentials:
1) That he was prosecuted by the defendant.
2) That the proceeding complained was terminated in favour of the present plaintiff.
3) That the prosecution was instituted against him without any just or reasonable cause.
4) That the prosecution was instituted with a malicious intention, that is, not with the mere inten-
tion of getting the law into effect, but with an intention, which was wrongful in fact.
5) That he suffered damage to his reputation or to the safety of person, or to security of his
property.
FACTUAL SITUATION: A recovered a large sum of money from Railway Co. for personal injuries.
Subsequently, Railway Co. came to know that injuries were not real and were created by doctor B.
Railway Co. prosecuted B for playing fraud on the company, but B was acquitted. B sued Railway
Co. for malicious prosecution. In the light of these facts which of the following statements is true ?
DECISION:
(a) Railway Co. is guilty of malicious prosecution because it acted without reasonable cause
(b) Railway Co. is not guilty of malicious prosecution because the Co. took reasonable care in
determining the facts and honestly believed them to be true
(c) Railway Co. is liable because it acted negligently
(d) None of the above
Previous Years
Page 230 CLAT & AILET Papers
81. LEGAL PRINCIPLE: Attempt is an act done with an intent to commit crime, and forming part of the
series of acts which would constitute actual commission of the crime, if not interrupted.
FACTUAL SITUATION: A intending to murder B by poison purchases poison and mixes the same
with a glass of water. He gave to the bearer to serve B. The bearer while approaching B, loses the
balance and the glass drops out of his tray.
DECISION:
(a) A has not committed any offence
(b) A has committed the offence of murder
(c) A has committed the offence of attempt to murder
(d) A has not committed an offence of attempt to murder because nothing happened to B

82. LEGAL PRINCIPLES:


1) The concept of joint liability comes under Section 34 of IPC which states that “when a criminal
act is done by several persons, in furtherance of the common intention of all, each of such
persons is liable for that act in the same manner as if it were done by him alone.”
2) A person abets an offence, who abets either the commission of an offence, or the commission
of an act which would be an offence, if committed by a person capable by law of committing an
offence with the same intention or knowledge as that of the abettor.
3) A criminal conspiracy takes place when two or more people get together and plan to commit a
crime and then take some action toward carrying out that plan. The action taken does not have
to be a crime itself to further the conspiracy.
FACTUAL SITUATION: X and Y conspire to poison Z. X in pursuance of the conspiracy procures
the poison and delivers it to Y in order that he may administer it to Z. Y in pursuance of the
conspiracy, administer the poison in the presence of X and thereby causes Z’s death. What offences
X and Y have committed ?
DECISION:
(a) Y has committed the offence of murder and X was an abettor
(b) Both X and Y has committed the offence of criminal conspiracy
(c) X has not committed any offence
(d) Both X and Y has committed the offence of m urder

83. LEGAL PRINCIPLES:


1) Whoever, intending to take dishonestly any movable property out of the possession of any
person without that person’ s consent, moves that property in order to such taking, is said to
commit theft.
2) Whoever dishonestly misappropriates or converts to his own use any movable property, shall be
punished with imprisonment of either description for a term which may extend to two years, or
with fine, or with both.
FACTUAL SITUATION: A takes umbrella belonging to Z out of Z’s possession in good faith, believing
at the time when he took it, that the property belongs to himself. His wife points out after some days
that the umbrella does not belong to them but to Z. After coming to know that, A dishonestly keeps
the umbrella.
DECISION:
(a) A is guilty of criminal misappropriation
(b) A is guilty of criminal breach of trust
(c) A is guilty of theft
(d) Both A and his wife are guilty of criminal misappropriation
Previous Years
CLAT & AILET Papers Page 231
84. LEGAL PRINCIPLE: When at the desire of the promisor, the promisee has done or abstained from
doing, or does or abstains from doing, or promises to do or abstain something, such an act or
abstinence or promise is called consideration for the promise.

FACTUAL SITUATION: A daughter promises to give maintenance to her uncle in consideration of


her mother making a gift of certain properties to her. The daughter pleads lack of consideration when
the uncle seeks to enforce the contract. She says that the uncle is a stranger to the consideration
and so he cannot enforce the contract. The daughter

DECISION:
(a) Will succeed because uncle being a stranger to the consideration cannot enforce it
(b) Will not succeed because uncle is a near relative and in such cases consideration is not
necessary
(c) Cannot succeed because consideration might move from any person
(d) None of the above

85. LEGAL PRINCIPLES:


1) Parties to contract should be capable of entering in to contract, only then they can lay the
foundation of a valid contract.
2) Every person is competent to contract who is of the age of majority.

FACTUAL SITUATION: A minor agreed with B to become a tenant of his house and to pay Rs.
1,000/- for the furniture therein. He paid Rs. 800/- in cash and gave a promissory note for the
balance. A occupied the premises and used the furniture for some months and then brought an
action for refund of consideration, in this case

DECISION:
(a) A is liable to pay Rs. 1,000
(b) A is liable to pay remaining Rs. 200
(c) A is liable to refund of Rs. 800
(d) Neither B is liable to refund Rs. 800 nor A is under obligation to pay Rs. 200

86. “Begar” means


(a) Voluntary work without payment (b) Involuntary work without payment
(c) Involuntary work with payment (d) Voluntary work with payment

87. In India, the reckoning date for the determination of the age of the juvenile is the
(a) date of offence (b) date of trial (c) date of judgment (d) date of arrest

88. Which of the following rights is not available to the citizens of India under Article 21 of the Constitution?
(a) Right to Privacy
(b) Right to Die
(c) Right to Health and Medical Assistance
(d) Right to Get Pollution free Water and Air

89. The “Eco-mark” is given by the Ministry of Environment and Forest to the consumer products which
(a) are degradable (b) are environment friendly
(c) conform to the standards of ISI (d) are electricity saver

Previous Years
Page 232 CLAT & AILET Papers
90. The EIA is abbreviated form of
(a) Environment Impact Assessment
(b) Environment Implementation Arrangement
(c) Environment Impact Apparatus
(d) Environmental Institute of Awareness

91. ____________ was the leader of “Chipko Movement”.


(a) Medha Patkar (b) Subhash Gheising
(c) Balasaheb Doeras (d) Sunderlal Bahuguna

92. When two persons descend from a common ancestor but by different wives, they are said to be
related to each other by
(a) Sapinda relationship (b) Half blood
(c) Full blood (d) None of the above

93. “Pacta sunt Servanda” means that


(a) The States are not bound to respect the agreements entered into by them
(b) The agreements entered into by the States will be respected and followed by them in good faith
(c) The States are under no obligation to follow the agreements in good faith
(d) Though the States are bound to recognise an agreement entered into by them but cannot be
compelled to follow it

94. An offence of breaking a divine idol


(a) Blasphemy (b) Salus populi (c) Crime (d) Sacrilege

95. What is Plea Bargaining ?


(a) A conference between opposing lawyers and judge to determine the time a case should take
place
(b) A procedure by which an accused pleads guilty in exchange for a lesser punishment
(c) A conference between opposing lawyers to settle the claim
(d) A conference between the victim and the accused to settle the claim

96. The act by members of a trade union, persuading others not to work is called as
(a) Non-cooperation (b) Picketing (c) Sit-in-strike (d) Strike

97. In India, cyber terrorism is an offence punishable under


(a) Information Technology Act (b) POTA
(c) TADA (d) Indian Penal Code

98. When a judge makes certain remarks in the course of his judgement, which are said “by the way”
and do not have direct bearing on the facts at hand, such remarks are called
(a) Ratio Decidendi (b) Obiter Dictum (c) Observations (d) Comments ordinaralis

99. In Extradition Treaty, extradition means


(a) Order of Indian Court will apply to Indian living elsewhere than India
(b) Export without double taxation
(c) Two countries will deport criminal on reciprocal basis to each other
(d) None of the above

Previous Years
CLAT & AILET Papers Page 233
100. Who is the Chairperson of the 20th Law Commission of India ?
(a) Justice Altmas Kabir (b) Justice P.V. Reddy
(c) Justice A. R. Lakshmanan (d) Justice D. K. Jain

101. __________ of the Constitution of India lays down that Union of India and the States are juristic
person and can sue and be sued.
(a) Article 225 (b) Article 268 (c) Article 300 (d) Article 348

102. A post dated cheque bears _________ date.


(a) past (b) no (c) future (d) present

103. According to the Doctrine of__________, every generation is obliged to preserve its natural and
cultural heritage for the enjoyment of the future generations.
(a) Sustainable development (b) Polluter pay principle
(c) Precautionary principle (d) Inter-generational equity

104. Which of the following is not recommended by the Justice Verma Committee ?
(a) Minimum sentence for a rapist should be enhanced from 7 years to 10 years
(b) The age of juvenile should not be lowered from 18 to 16 years
(c) Life imprisonment must always mean jail for “the entire natural life of the convict”
(d) Death sentence for “rarest of rare cases” of rape

105. High Courts has been constituted in all these States in January, 2013 except
(a) Assam (b) Meghalaya (c) Manipur (d) Tripura

SECTION - D : REASONING

106. Information that is published is part of the public record. But information that a reporter collects, and
sources that he contacts, must be protected in order for our free press to function free of fear.

The above argument is most severely weakened by which one of the following statements ?
(a) Public information is usually reliable
(b) Undocumented evidence may be used to convict an innocent person
(c) Members of the press act ethically in most cases
(d) The sources that a reporter contacts are usually willing to divulge their identity

107. Psychological novels are superior to novels of adventure. Immature readers prefer novels of adventure
to novels with less action and greater psychological depth. The immature reader, who prefers James
Bond’s exploits to the subtleties of Henry James, can be identified easily by his choice of inferior
reading matter.

A criticism of the logic of this argument would be likely to find fault with the author’s
(a) Presupposing the conclusions he wishes to prove
(b) Failure to define “adventure” clearly
(c) Failure to cite possible exceptions to this rule
(d) Hasty generalisation on the basis of a limited specific case

Previous Years
Page 234 CLAT & AILET Papers
108. Many very effective prescription drugs are available to patients on a “one time only” basis. Suspicious
of drug abuse, physicians will not renew a prescription for a medicine that has worked effectively for
a patient. This practice denies a patient her right to health.

Which one of the following is a basic assumption made by the author ?


(a) A new type of medicine is likely to be more expensive
(b) Physicians are not concerned with a patient’s health
(c) Most physicians prescribe inadequate amounts of medicine
(d) Patients are liable to suffer the same ailment repeatedly

109. Vijay claimed that the large dent in the fender of the company-owned vehicle he had borrowed was
caused by the careless act of another motorist, who backed into the car when it was parked in a
public garage. Yet Vijay’s own car has several dents in its fenders, all of which he acknowledges as
having caused by his own careless driving. Therefore, Vijay’s contention that the dent in the formerly
undented company-owned vehicle was caused by the careless act of another person is not true.

The reasoning in this argument is vulnerable because it


(a) Fails to recognise that Vijay could be lying about the dents in his own vehicle
(b) Fails to recognise that the motorist who backed into him simply did not see him
(c) Fails to acknowledge that many such accidents occur in parking garages
(d) Presumes, without justification, that because Vijay has caused similar dents to his own car, he
caused the dent in the company car

110. It takes a good telescope to see the moons of Neptune. I can’t see the moons of Neptune with my
telescope. Therefore, I do not have a good telescope.

Which one of the following most closely parallels the logic of this statement ?
(a) It takes two to tango. You are doing the tango. Therefore, you have a partner
(b) If you have a surfboard, you can surf. You do not have a surfboard. Therefore, you cannot surf
(c) You can write a letter to your friend with a pencil. You do not have a pencil. Therefore, you cannot
write the letter
(d) If you know the area of a circle, you can find its circumference. You cannot figure out the
circumference. Therefore, you do not know the area

Directions (Q. 111 - 114) : In each question below is given statement followed by two assumptions numbered
I and II. An assumption is something supposed or taken for granted. You have to consider the statement
and the following assumptions and decide which of the assumptions is implicit in the statement. Mark
answer:
(a) If only I assumption is implicit
(b) If only assumption II is implicit
(c) If neither I nor II is implicit; and
(d) If both I and II are implicit

111. Statement : As poor people in India prefer and use jaggery rather than sugar, the government
has decided to decontrol and scrap sugar distribution through Public Distribution
System (PDS).
Assumption I : Jaggery is freely available at reasonable price to all poor people.
Assumption II : PDS has lost its utility.

Previous Years
CLAT & AILET Papers Page 235
112. Statement : Unless country ‘X’ achieves total literacy, it cannot achieve its mission of
development.
Assumption I : It is possible to achieve total literacy in country ‘X’.
Assumption II : No development is possible without a proper mission.

113. Statement : Many species of animals on our earth are still not studied scientifically and if we
do not do this work urgently, many species will face extinction.
Assumption I : Earth may lose all types of life very shortly.
Assumption II : It is desirable and possible to study scientifically many animal species.

114. Statement : Herbs can safely be used for treating diseases of human animal species.
Assumption I : Herbs cannot be used for treating diseases of animals.
Assumption II : Herbs treatment is getting popular.

Directions (Q. 115 - 120): Each group of questions is based on a set of conditions. Choose the response
that most accurately and completely answers each question.

At the snack cafe at a party, Ali, Bina, Champak and Diya are eating cookies. There are five kinds of
cookies to choose from - chocolate chip cookies, oatmeal cookies, sugar cookies, peanut butter cookies
and raisin cookies. Each of these four people eats at least two kinds of cookies. Their choices are governed
by the following rules :

At most two of them eat oatmeal cookies


At least two of them eat sugar cookies
Ali does not eat any sugar cookies
Bina and Champak do not eat the same type of cookies Bina eats chocolate chip cookies
Champak eats sugar cookies
No one eats both raisin cookies and sugar cookies
If someone eats raisin cookies, they also eat peanut butter cookies

115. Which one of the following must be true ?


(a) Champak eats chocolate chip cookies
(b) Ali eats chocolate chip cookies
(c) Bina does not eat peanut butter cookies
(d) Diya does not eat raisin cookies

116. If Bina eats exactly three kinds of cookies, which one of the following must be true ?
(a) Champak eats exactly three kinds of cookies
(b) Diya eats only sugar cookies
(c) If Ali eats oatmeal cookies, Diya eats oatmeal cookies
(d) Champak eats oatmeal cookies

117. Which one of the following cannot be true ?


(a) No one eats raisin cookies
(b) Ali and Diya both eat oatmeal cookies
(c) All and Diya both eat chocolate chip cookies
(d) Bina and Champak eat the same number of kinds of cookies.

Previous Years
Page 236 CLAT & AILET Papers
118. Which pair of cookie types could each be eaten by at least three different people ?
(a) Chocolate chip and oatmeal (b) Oatmeal and peanut butter
(c) Chocolate chip and peanut butter (d) Oatmeal and sugar

119. Which pair of cookie types contains a cookie type eaten by exactly two different people ?
(a) Chocolate chip and oatmeal (b) Oatmeal and peanut butter
(c) Chocolate chip and peanut butter (d) Sugar and raisin

120. Which cookie type could be eaten by none of the people ?


(a) Chocolate chip (b) Oatmeal (c) Sugar (d) Raisin

Directions : Read the following directions and answer the questions 121 to 123.

Sunita, Rahul and Suraj are children of Mrs. and Mr. Ahuja. Rani, Rohan and Shyam are children of Mrs.
and Mr. Malik. Shyam and Sunita are married and Arjun and Sohan are their children. Gunjan and Romesh
are children of Mrs. and Mr. Gandhi. Gunjan is married to Suraj and has three children named Rupali, Sonu
and Ravi.

121. How is Rahul related ot Arjun ?


(a) Brother-in-law (b) Cousin (c) Uncle (d) Maternal uncle

122. Rani is Sohan’s


(a) Niece (b) Sister (c) Aunt (d) Sister-in-law

123. Shyam and Suraj are related as


(a) Brother-in-law (b) Cousin (c) Brother (d) Nephew

124. Shyam travels 5 km towards East and turns left and moves 6 km further. He then turns right and
moves 9 km. Finally, he turns once again to his right and moves 6 km. How far is he from the starting
point ?
(a) Brother-in-law (b) Cousin (c) Uncle (d) Maternal uncle

125. Reaching a place of appointment on Friday, I found that I was two days earlier than the scheduled
day. If I had reached on the following Wednesday, how many days late would I have been ?
(a) One day (b) Two days (c) Three days (d) Four days

126. X was born on March, 6, 1993. The same year Independence Day was celebrated on Friday. Find out
the birth day of X.
(a) Thursday (b) Saturday (c) Friday (d) Wednesday

127. Kamala would like to complete all her home work before 10 p.m. in order to watch an important TV
programme. She has 40 minutes assignment in each of her five subjects. What is the latest time at
which she can start and still complete work in time for the programme ?
(a) 6.40 p.m. (b) 6.30 p.m. (c) 7.10 p.m. (d) 7.20 p.m.

Previous Years
CLAT & AILET Papers Page 237
Direction : In the following number series (Q. No. 128 to 131) only one number is wrong.

128. 21, 28, 33, 35, 37, 36


(a) 33 (b) 35 (c) 21 (d) 36

129. 5, 13, 29, 61, 120, 253


(a) 61 (b) 29 (c) 120 (d) 253

130. 0, 7, 28, 63, 124, 215


(a) 7 (b) 63 (c) 28 (d) 215

131. 9, 19, 40, 83, 170, 340, 340


(a) 340 (b) 170 (c) 83 (d) 40

132. In a certain code, TEAMWORK is written as NBFUJQNV and SOME is written as PTDL. How is
PERSON written in that code ?
(a) QDOOPT (b) QDOMNR (c) SFQMNR (d) SFQOPT

133. In certain code, BASKET is written as 5$3%#1 and TRIED is written as 14*#2. How is SKIRT
written in that code ?
(a) 3%*41 (b) 3*%41 (c) 3%#41 (d) 3#4%1

134. Eagle : Swoops : : Duck : ?


(a) Swims (b) Flits (c) Waddles (d) Floats

135. APPLE : 50 : : ORANGE : ?


(a) 61 (b) 63 (c) 60 (d) 69

Directions : In questions 136 to 140, a particular word is given. The word is followed by four words as given
in the alternatives. One of these four words cannot be formed by using the letters given in the question.

136. ENTHUSIASTICALLY
(a) SATIATE (b) HELMINTH (c) SHALE (d) TANTALUS

137. CONCENTRATE
(a) CENTRE (b) CONCERN (c) REASON (d) TREAT

138. INTRANSIGENT
(a) STAIN (b) GRATE (c) TRACE (d) RESIGN

139. PERPETUATION
(a) REPUTE (b) RETAIN (c) PIPETTE (d) PENANCE

140. ESTRANGEMENT
(a) ENTANGLE (b) RNTREAT (c) GERMAN (d) TANGEN

Previous Years
Page 238 CLAT & AILET Papers
SECTION - E : MATHEMATICS

141. The difference between the number of numbers from 2 to 100 which are not divisible by any other
number except 1 and itself and the numbers which are divisible by atleast one more number along
with one and itself.
(a) 25 (b) 50 (c) 49 (d) none of these

142. The average of three prime numbers lying between 47 and 74 is 191/3. The greatest possible sum
between any two out of the 3 prime numbers is
(a) 120 (b) 138 (c) 132 (d) 136

143. Divya purchased 6 mangoes, 10 oranges and 5 apples for a certain amount. With 40% less amount
Anu could purchase 3 mangoes, 5 oranges and 4 apples. What percentage of the total amount did
Divya spend on apples ?
(a) 25% (b) 33.333%
(c) 40% (d) can’t be determined

144. Military camp is having the provisions for 300 people consuming 600 grams daily for 75 days. They
are joined by 60 more men and daily ration was reduced by 100 grams. How long will the provisions
last approximately ?
(a) 61 days (b) 67 days (c) 75 days (d) 88 days

145. A contractor Abhay Singh employed some men to do a piece of work which can be done by 16 men
in 14 days. At the end of 5 days, 7 of the men stopped working and 3 days later half of the remainder
stopped working; the rest finished the work in 5 days. What is the number of men originally employed?
(a) 25 (b) 27 (c) 29 (d) 33

146. Ram is travelling by car at the rate of 40 kmph. After 80 km he rests for 20 minutes. How long
will he take to cover a distance of 240 kms ?
(a) 6 hrs 40 min (b) 6 hrs 11 min (c) 5 hrs 11 min (d) 7 hrs 15 min

147. There is leak in the bottom of a tank. This leak can empty a full tank in 8 hours. When the tank is full,
a tap is opened into the tank which admits 6 litres per hour and the tank is now emptied in 12 hours.
What is the capacity of the tank ?
(a) 28.8 litres (b) 36 litres (c) 144 litres (d) 145 litres

148. A and B walk from X to Y, a distance of 27 km at 5 km/hr and 7 km/hr, respectively. B reaches Y and
immediately turns back meeting A at Z. What is the distance from X to Z ?
(a) 25 km (b) 22.5 km (c) 24 km (d) 20 km

149. Three consecutive positive even numbers are such that thrice the first number exceeds double the
third by 2, the third number is
(a) 10 (b) 14 (c) 16 (d) 12

150. Devendra, a computer accessories vendor gave a discount of 20% on a pen-drive and his profit
reduced from Rs. 200 to Rs. 100. What is Devendra’s cost price ?
(a) Rs. 300 (b) Rs. 200 (c) Rs. 400 (d) None of these

Previous Years
CLAT & AILET Papers Page 239
AILET Question Paper 2014

SECTION - A : ENGLISH

Directions (Q. 1 – 7) : The questions in this section are based on the passage. The questions are to
be answered on the basis of what is stated or implied in the passage. For some of the questions,
more than one of the choices could conceivably answer the question. However, you are to choose the
best answer; that is, the response that most accurately and completely answers the questions.

Under very early common law, all felonies were punishable by death. The perpetrators of the felony
were hanged whether or not a homicide had been committed during the felony. Later, however, some
felonies were declared to be non-capital offences. The common law courts, in need of a deterrent to
the use of deadly force in the course of these non-capital felonies, developed the “felony-murder” rule.
The first formal statement of the rule stated: “Any killing by one in the commission of a felony is guilty
of murder.” The killing was a murder whether intentional or unintentional, accidental or mistaken. The
usual requirement of malice was eliminated and the only criminal intent necessary was the intent to
commit the particular underlying felony. All participants in the felony were guilty of murder-actual
killer and non-killer confederates.

Proponents of the rule argued that it was justified because the felony demonstrated a lack of concern
for human life by the commission of a violent and dangerous felony and that the crime was murder
either because of a conclusive presumption of malice or simply by force of statutory definition.

Opponents of the rule describe it as a highly artificial concept and “an enigma wrapped in a riddle.”
They are quick to point out that the rule has been abandoned in England where it originated, abolished
in India, severely restricted in Canada and a number of other commonwealth countries, is unknown in
continental Europe, and abandoned in Michigan. In reality, the real strength of the opponents’ criticism
stems from the bizarre and of times unfair results achieved when the felony - murder rule is applied
mechanically. Defendants have been convicted under the rule where the killing was purely accidental,
or the killing took place after the felony during the later flight from the scene; or a third party killed
another (police officer killed a citizen or vice versa; or a victim died of a heart attack 15 -20 minutes
after the robbery was over; or the person killed was an accomplice in the felony).

Attacks on the rule have come from all directions with basically the same demand - re-evaluate and
abandon the archaic legal fiction; restrict and limit vicarious criminal liability; prosecute killers for
murder, not non-killers; increase punishment for the underlying felony as a real deterrent; and initiate
legislative modifications. With the unstable history of the felony - murder rule, including its abandonment
by many jurisdictions in this country, the felony - murder rule is dying a slow but certain death.

1. Which one of the following best states the central idea of the passage ?
(a) The felony - murder rule should be abolished.
(b) Some jurisdictions are about to abandon the felony — murder rule.
(c} The felony - murder rule can be unfair.
(d) Supreme Court of the United States.

Previous Years
Page 240 CLAT & AILET Papers
2. The felony - murder rule was developed in order to
(a) deter felonies
(b) deter murders
(c) deter deadly force in felonies
(d) extend the definition of murder to any malicious act resulting in death

3. Arguments in favour of the felony - murder rule may include all of the following EXCEPT
(a) We can infer that anyone undertaking a dangerous felony demonstrates an indifference to
human life.
(b) If the punishment for the use of deadly force whether intended or not is the same, criminals
will be less likely to use deadly force.
(c) Because a life has been taken, the crime is murder by force of statutory definition.
(d) The victim of murder may be an accomplice of the felony.

4. According to the passage, opponents of the felony - murder rule have raised all of the following
objections to the statute EXCEPT
(a) The felony - murder rule results in murder prosecutions of defendants who have not committed
murder.
(b) The felony - murder rule assigns a criminal liability vicariously.
(c) The felony - murder rule is based upon a presumption of malice even if death is wholly
accidental.
(d) The felony - murder rule deters the use of deadly force in non-capital felonies.

5. In which of the following situations would the defendant NOT be liable to the charge of murder
under the felony — murder rule ?
(a) In escaping from an unsuccessful attempt to rob a bank, the defendant crashes his car,
killing an innocent pedestrian in another city.
(b) A bank security officer, pursuing the defendant after a robbery, falls down a flight of stairs
and suffers serious permanent brain and spinal cord injuries.
(c) The driver of the escape car, who has not entered the bank, crashes the car killing the
armed gunman who committed the robbery.
(d) A bank teller, locked safely in the bank vault by the robber, has a stroke and dies.

6. According to the passage, the decline of support for the felony - murder rule is indicted by the
abandoning of the rule in all of the following locations EXCEPT
(a) Continental Europe (b) India
(c) England (d) Canada

7. The author believes that the felony - murder rule is


(a) unconstitutional
(b) bizarre and unfair
(c) a serviceable rule unfairly attacked by the “intelligentsia ”
(d) an unfair equating of intent to commit a felony and intent to commit murder

Directions (Q. 8 - 11) : In the following questions, choose the word which is most nearly the OPPOSITE in
meaning to the bold word and mark it in the Answer Sheet.

8. Ambiguity
(a) lucidity (b) basal (c) dull (d) necessity

Previous Years
CLAT & AILET Papers Page 241
9. Antidote
(a) medicine (b) poison (c) anodyne (d) amity

10. Which is NOT a synonym for ‘accumulation’?


(a) collection (b) conglomeration (c) assemblage (d) collagen

11. Which is NOT a synonym for ‘incline’?


(a) trenchant (b) slope (c) acclivity (d) gradient

Directions (Q. 12 - 15) : In the following questions, choose the word which is most nearly the SAME in
meaning to the bold word and mark it in the Answer Sheet.

12. Aphorism
(a) prune (b) wither (c) aphis (d) proverb

13. Passe
(a) rude (b) old - fashioned (c) modern (d) chic

14. Vituperation
(a) moisture (b) parallel (c) malediction (d) recover

15. Qualm
(a) concavity (b) amplitude (c) misgiving (d) repute

Directions (Q. 16 - 17) : Choose the exact meaning of the idioms/phrases.

16. She exhibited remarkable sang froid during the crisis.


(a) temper (b) irritation (c) composure (d) anger

17. The co-operation and esprit de corps between the soldiers and the officers was directly responsible
for their victory.
(a) bravery (b) loyalty (c) subordination (d) unity

Directions (Q. 18 - 19) : Answer the questions based on the following information.
In each of the question below, four different ways of writing a sentence are indicated. Choose the best
way of writing the sentence.

18. (a) The main problem with the notion of price discrimination is that it is not always a bad thing,
but that it is the monopolist who has the power to decide who is charged what price.
(b) The main problem with the notion of price discrimination is not that it is always a bad thing,
it is the monopolist who has the power to decide who is charged what price.
(c) The main problem with the notion of price discrimination is not that it is always a bad thing,
but that it is the monopolist who has the power to decide who is charged what price.
(d) The main problem with the notion of price discrimination is not it is always a bad thing, but
that it is the monopolist who has the power to decide who is charged what price.

Previous Years
Page 242 CLAT & AILET Papers
19. (a) A symbiotic relationship develops among the contractors, bureaucracy and the politicians,
and by a large number of devices, costs are artificially escalated and black money is
generated by underhand deals.
(b) A symbiotic relationship develops among contractors, bureaucracy and politicians, and
costs are artificially escalated with a large number of devices and black money is generated
through underhand deals.
(c) A symbiotic relationship develops among contractors, bureaucracy and the politicians, and
by a large number of devices costs are artificially escalated and black money is generated
on underhand deals.
(d) A symbiotic relationship develops among the contractors, bureaucracy and politicians, and
by large number of devices costs are artificially escalated and black money is generated
by underhand deals.

Directions (Q. 20 - 23) : In this section, each passage consists of five sentences. The first sentence
is given in the beginning. The four sentences in each passage have been jumbled up. These are
labeled P, Q, R and S. You are required to find out the proper sequence of the four sentences and
mark accordingly on the Answer Sheet.

20. L1: It is significant that one of the most common objections to competition is that it is blind
P: This is important because in a system of free enterprise based on private property, chances
are not equal and there is indeed a strong case for reducing that inequality of opportunity.
Q: Rather it is a choice between a system where it is the will of a few persons that decides
which is to get what and one where it depends at least partly, on the ability and the enterprise
of the people concerned.
R: Although competition and justice may have little else in common, it is as much a
commendation of competition as of justice that it is no respector of persons.
S: The choice today is not between a system in which everybody will get what he deserves
according to some universal standard and one where individual shares are determined by
chance or goodwill.
L6: The fact that opportunities open to the poor in a competitive society are much more restricted
than those open to the rich, does not make it less true that in such a society the poor are
more free than a person commanding much greater material comfort in a different type of
society.

The proper sequence should be


(a) RSQP (b) SRQP (c) PQRS (d) QPSR

21. L1: The chain saw howled as I finished cutting through the branch.
P: The branch crashed to the ground, taking my spectacles with it.
Q: I almost dropped the saw as I shielded my face from the twigs that brushed by.
R: Howard retrieved my glasses and handed them up to me.
5: 1 pulled the saw away, and my husband tugged against the other end of the rope that I had tied
just above the cut.
L6: Are you okay? He asked.

The proper sequence should be


(a) QRPS (b)SQPR (c) SRPQ (d) QSPR

Previous Years
CLAT & AILET Papers Page 243
22. L1: All human beings are aware of the existence of a power greater than that of the mortals — the
name given to such a power by individuals is an outcome of birth, education and choice.
P: Logically, therefore such a power should be remembered in good times also.
Q: Their other philanthropic contributions include the construction and maintenance of religious
places such as temples or gurdwaras.
R: Industrial organizations also contribute to the veneration of this power by participating in activities
such as religious ceremonies and festivities organized by the employees.
5: This power provides an anchor in times of adversity, difficulty and trouble..
L6: The top management/ managers should participate in all such events, irrespective of their personal
choices.

The proper sequence should be


(a) RPSQ (b) QRPS (c) SPRQ (d) SQRP

23. L1: A thorough knowledge of path or course to be followed is essential for achieving success.
P: Seniors must show the path clearly by laying down the precise expectations of the management
in terms of job description, key result areas, and personal targets.
Q: They should also ‘light the path’ by personal examples.
R: Advice tendered or help offered must be objectively evaluated for its effectiveness in achieving the
desired goals.
S: A display of arrogance and a false sense of ‘self-worth’, in order to belittle those who come to
help, prove dysfunctional.
L6: The individuality of each employee must be respected.

The proper sequence should be


(a) RSPQ (b) RPSQ (c) QPSR (d) PQRS

Directions (Q. 24 - 26) : In the following questions, the word at the top is used in four different ways.
Choose the option in which the usage of the word is INCORRECT or INAPPROPRIATE.

24. Passing
(a) She did not have passing marks in mathematics.
(b) The mad woman was cursing everybody passing her on the road.
(c) At the birthday party, all the children enjoyed a game of passing the parcel.
(d) A passing taxi was stopped to rush the accident victim to the hospital.

25. Bolt
(a) The shopkeeper showed us a bolt of fine silk.
(b) As he could not move, he made a bolt for the gate.
(c) Could you please bolt the door?
(d) The thief was arrested before he could bolt from the scene of the crime.

26. Fallout
(a) Nagasaki suffered from the fallout of nuclear radiation.
(b) People believed that the political fallout of the scandal would be insignificant.
(c) Who can predict the environmental fallout of the WTO agreements?
(d) The Headmaster could not understand the fallout of several of his good students at the public
examination

Previous Years
Page 244 CLAT & AILET Papers
Directions (Q. 27 - 35) : Fill in the blanks with the best alternative among the four options:

Around the world, forests are being destroyed at a rate of about thirteen million hectares a year and
deforestation accounts for an estimated 17% - 20% of all global emissions. In addition, forests and
other terrestrial carbon sinks play a ....(27).... role in preventing runaway climate change, soaking up a
full 2.6 Gt of atmospheric carbon every year. The destruction of forests, therefore, not only emits
carbon — a staggering 1.6 Gt a year, which severely …(28).... forests’ capacity to absorb emissions
from other sources - but also drastically....(29).... the amount of forested land available to act as a
carbon sink in the future.
However, the effects of deforestation extend beyond carbon. Rainforests ....(30).... a wide variety of
ecosystems services, from regulating rainfall to purifying groundwater and keeping fertile soil from ....(31)....;
deforestation in one area can seriously damage food production and ....(32).... to clean water in an
entire region. The value of global ecosystem services has been estimated at 33 trillion USD each
year (almost half of global GDP), but these services have been taken for granted without a mechanism
to make the market reflect their value. Rainforests are also a home and ....(33).... of income for a
huge number of people in Africa, Asia, and South America …(34).... this, economic pressures frequently
drive both local communities and national governments in the developing world to ....(35).... these
forests in ways that are unsustainable, clear-cutting vast areas for fuel, timber, mining, or agricultural
land.
27. (a) tough (b) important (c) vital (d) biggest

28. (a) affect (b) diminish (c) increases (d) impairs

29. (a) plagues (b) develops (c) reduces (d) shortens

30. (a) sell (b) offer (c) give (d) provide

31. (a) transforming (b) decoding (c) erupting (d) eroding

32. (a) handiness (b) excess (c) availability (d) access

33. (a) beginning (b) source (c) ways (d) reference

34. (a) despite (b) also (c) in spite (d) apart

35. (a) exploit (b) encompass (c) nurture (d) work

SECTION - B : GENERAL KNOWLEDGE


36. The Aircraft Carrier inducted in to the Indian Navy recently is
(a) INS Virat (b) INS Vikrant (c) INS Vikramaditya (d) None

37. Which of the following countries impeached its Chief Justice recently ?
(a) Bangladesh (b) Pakistan (c) Sri Lanka (d) Maldives

38. With which of the following country India signed an Extradition treaty that came into force during
October 2013 ?
(a) Afghanistan (b) Kenya (c) China (d) Bangladesh
Previous Years
CLAT & AILET Papers Page 245
39. The present Chief Election Commissioner of India is
(a) Dr. Nasim Zaidi (b) V. S. Sampath (c) H. S. Brahma (d) None

40. According to the recent World population data of 2013, the second most populous city in the world
is
(a) New Delhi (b) Tokyo (c) Mexico (d) Beijing

41. The latest country to launch a satellite during January 2013 from its soil and join the club of space-
faring nations is
(a) South Korea (b) North Korea (c) Ukraine (d) Iran

42. Who won the Women’s Singles at the Malaysian Gran Prix Badminton Championship in 2013 ?
(a) Saina Nehwal (b) Juan Gu (c) P. V. Sindhu (d) Ratchanok Intanon

43. Who is the oldest woman tennis player to be ranked No. 1 in the world ?
(a) Venus Williams (b) Chris Evert (c) Martina Navratilova (d) Serena Williams

44. How many High Courts are there in India as on January 2014?
(a) 18 (b) 21 (c) 28 (d) 24

45. Indira Gandhi Prize for Peace, Disarmament and Development for the year 2013 has been conferred
on:
(a) Ms. Angela Merkel (b) Mr. Nelson Mandela (c) Mr. Uhuru Kenyatta (d) Mr. Hamid Karzai

46. Who has won the Golden Shoe Award in Soccer three times ?
(a) Diego Maradona (b) Christiano Ronaldo (c) Lionel Messi (d) Pele

47. The phrase ‘Bitcoins’ refers to


(a) Cryptocurrency introduced by the United States
(b) Cryptocurrency introduced by France
(c) Cryptocurrency introduced by the World Bank
(d) None

48. Who was conferred the Col. C. K. Nayudu Life Achievement Award in Cricket during 2012 ?
(a) Sunil Gavaskar (b) Sachin Tendulkar (c) Dhoni (d) Kapildev

49. The Parliament of India voted to remove


(a) Justice V. Ramaswami (b) Justice P. Dinakaran
(c) Justice Soumitra Sen (d) None

50. The maximum penalty that can be imposed by CIC / SIC under the Right to Information Act, 2005 is
(a) Rs. 5,000 (b) Rs. 25,000 (c) Rs.250 (d) 10,000

51. On January 1, 2013, internet has completed


(a) 26 years (b) 30 years (c) 32 years (d) 28 years

52. Which of the following was declared by the United Nations General Assembly as “International Year
for Water Co-operation” ?
(a) 2011 (b) 2012 (c) 2013 (d) 2014
Previous Years
Page 246 CLAT & AILET Papers
53. How many states are there in the European Union ?
(a) 28 (b) 24 (c) 36 (d) 22

54. How many members can be nominated to the Rajya Sabha by the President ?
(a) 10 (b) 20 (c) 12 (d) 16

55. The highest award for sporting excellence in India is


(a) Arjuna Award (b) Dronacharya Award
(c) Bharat Ratna (d) Khel Ratna

56. Cloves, used as a spice, are derived from which of the following plant parts ?
(a) Seeds (b) Fruits (c) Flower buds (d) Young leaves

57. Which one of the following is included in the World List of Biosphere Reserves by UNESCO?
(a) Kinnaur Region (b) Spiti Valley (c) Nallamalai Hills (d) Sunderbans

58. Who among the following was the first Law Minister of India ?
(a) Jawaharlal Nehru (b) Maulana Abul Kalam Azad
(b) Dr. B.R. Ambedkar (d) T. Krishnamachari

59. Which one of the following countries is not a UN member country?


(a) Greece (b) Taiwan (c) Portugal (d) Australia

60. The main component of India’s import items from China is


(a) Rice (b) Iron ore
(c) Pharmaceuticals (d) Telecom equipments and Machinery

61. Which of the following divides India into Northern and Southern parts ?
(a) Equator (b) Tropic of Cancer
(c) Tropic of Capricorn (d) Arctic Circle

62. With which one of the following countries, India shares maximum length of the border ?
(a) Bangladesh (b) Pakistan (c) China (d) Nepal

63. What is known as Jasmine Revolution ?


(a) Czech uprising (b) Sudan uprising (c) Arab Spring (d) Libyan uprising

64. Which one of the following statements about NATO is not correct ?
(a) NATO has 28 independent member states
(b) The US is an ex-officio member of NATO
(c) NATO is a collective defence organisation in Europe
(d) Turkey is a member of NATO

65. V.R. Krishna lyer was _________ before he became a Judge.


(a) A minister (b) An ambassador (c) A Governor (d) A bureaucrat

66. What does airbag, used for safety of car driver, contain ?
(a) Sodium bicarbonate (b) Sodium azide (c) Sodium nitrite (d) Sodium peroxide

Previous Years
CLAT & AILET Papers Page 247
67. Which colour of heat radiation represents the highest temperature ?
(a) Blood red (b) Dark cherry (c) Salmon (b) Dark cherry
(d) White

68. Who is the founder of facebook?


(a) Jimmy Wales (b) Larry Page (c) Mark Zuckerberg (d) Brian Acton

69. Shashi Kant Sharma was appointed in 2013 as the


(a) Attorney General of India
(b) Comptroller and Auditor General of India
(c) Commissioner of Linguistic Minorities
(d) Chairman of the National Commission for SCs & STs

70. Mist is a result of which one of the following ?


(a) Condensation (b) Evaporation (c) Sublimation (d) Saturation

SECTION - C : LEGAL APTITUDE

Directions (Q. 71 - 97) : Given below is a statement of legal principle followed by a factual situation. Apply
the principle to the facts given below and select the most appropriate answer.

71. LEGAL PRINCIPLE : The occupier of a premise owes a duty of care to all his invitees and visitor.

FACTUAL SITUATION : Radhika’s brother, Akash, had come to visit at her place. After seeing her
wealth, Akash decided to commit theft that night. While he was trying to escape that night he got
electrocuted by the wires which were fixed on the boundary walls. Akash plans to sue Radhika. Will
his claim succeed?

DECISION :
(a) Yes, because in Indian tradition, guests are like Gods.
(b) No, because one has to be himself cautious about his safety.
(c) Yes, because it is the occupiers’ duty to take care of its visitors.
(d) No, because he himself is guilty of theft. He is no longer an invitee or visitor.

72. LEGAL PRINCIPLES :


1. An unlawful intrusion which interferes with one’s person or property constitutes trespass.
2. An easement is the right to use another person’s land for a stated purpose and has been in use
for quite some time. It can involve a general or specific portion of the property.

FACTUAL SITUATION : Vijay wanted to construct a shed on his window to stop the water from
leaking to his house. The shed was constructed but it protruded in Namit’s house. Vijay claims it is
his easementary right. Will Vijay’s claim succeed?

DECISION :
(a) Yes, because he has a reason to do the same.
(b) Yes, because right of easement exists.
(c) No, because there exists no right of easement.
(d) No, because this is encroachment on Namit’s property.

Previous Years
Page 248 CLAT & AILET Papers
73. LEGAL PRINCIPLE : A person is responsible for that which he could have reasonably foreseen or
prevented.

FACTUAL SITUATION : A chemist sold a hair conditioner to Jyoti. The conditioner was locally
manufactured and the contents, harmful chemicals, were listed on the bottle. The chemist, however,
represented to Jyoti that the chemicals used were harmless and beneficial for the hair. On using it,
Jyoti’s hair was badly damaged and she had to get hair treatment done for the same. Jyoti filed a
complaint against the chemist. Will the chemist be liable?

DECISION :
(a) Yes, as he should have informed Jyoti that the ingredients are not known.
(b) Liable because he was aware of the side effects of the ingredients.
(c) Not liable because it was the buyer’s duty to be aware about the product she is buying.
(d) Not liable as it is a natural tendency of shopkeepers to extol the virtues of the product they are
selling.

74. LEGAL PRINCIPLE : Defamation means publication of a statement injuring the reputation of a
person without !awful justification. Such statement must reflect on a person’s reputation and
tends to lower him in the estimation of right thinking members of the society generally or tends
to make them shun or avoid him.

FACTUAL SITUATION : Madam Tussauds Ltd. decided to keep a waxwork exhibition, and
placed an effigy of Babloo Prasad with a gun, in a room adjoining the “Chamber of Horrors”. Mr.
Babloo Prasad had been tried for murder in India and released on a verdict of “not proven guilty”
and a representation of the scene of the alleged murder was displayed in the Chamber of
Horrors. Is it amount to defamation?

DECISION :
(a) No Defamation as Babloo was an accused.
(b) Defamation as his guilt was not proved and he was released by the court.
(c) No defamation as there was not proper publication.
(d) None of the above.

75. LEGAL PRINCIPLE : Every partner is liable, jointly with all the other partners and also severally,
for all acts of the firm done while he is a partner.

FACTUAL SITUATION : A and B started a partnership firm for providing vehicle repairing services.
C approached the firm for getting his car repaired and noticed that only B was present in the
office. C informed the problem, and B started repairing the car. While B was repairing, he filled
petrol instead of oil in the engine. As a consequence, a small blast occurred and damaged the
car. Now, C sued both A and B for the damage so caused. Decide.

DECISION :
(a) Both liable since B was authorised to carry out the work of the firm.
(b) A is not liable since A has not authorised B to do something which was not for the benefit
of the firm.
(c) A is not liable since it was B’s fault and for that only B can be held liable.
(d) A is liable since it was negligence on his part that he was not resent in the office when C
came.

Previous Years
CLAT & AILET Papers Page 249
76. LEGAL PRINCIPLES :
1. No fault liability means liability of a person even without any negligent act on his part and
even if he has taken due care and caution.
2. If a person brings and keeps any dangerous thing on his land, then he is liable for any
damage caused if the thing escapes.
3. No one can be penalized for an Act of God which is unforeseeable and unpredictable.

FACTUAL SITUATION : B owned and managed a company supplying electricity to the nearby
locality. On a particular windy and stormy day, one of the wires snapped and was hanging
down. A, a cyclist who was driving in the night, saw the wire from a distance. There was a
nearby street light with low visibility. He came in contact with the wire and was electrocuted
immediately. His heirs sued A on ground of strict liability. Decide.

DECISION :
(a) A is not liable because B must have stayed indoor on a windy day.
(b) A is not liable because B’s negligence caused him injury.
(c) A is liable because supplying electricity is an inherently dangerous use of land and he
should have been careful.
(d) A is not liable because sudden storm and winds without A’s negligence was an Act of God.

77. LEGAL PRINCIPLE : The master/principal is liable for all acts done by his duly appointed
servant/agent for all acts done by him lawfully in the course of his employment.

FACTUAL SITUATION : A had an agency which used to lend carpenters to people on need
basis. A deputed B to do some repair work in C’s shed. While so doing, B lit up a cigarette and
threw it as soon as he saw someone coming there. The cigarette remaining lit caused a fire and
the shed was reduced to ashes. C sued A and B. Decide.

DECISION :
(a) A is liable as B was his servant.
(b) A is liable as he should have chosen responsible people.
(c) A is not liable as B’s act was not an authorised act.
(d) A is not liable but B is liable.

78. LEGAL PRINCIPLE : Where the parents of a minor child due to their negligence allow the
child an opportunity to commit a tort, the parents are liable.

FACTUAL SITUATION : The father supplied an airgun to his son who was about to turn 18 next
month. After some complaints of mischief, the father took the gun away and placed it in a
corner of their storeroom which was used by the family to store surplus and other unnecessary
stuff. The son took it out of the store and shot A. A sued his father. Is the father liable?

DECISION :
(a) No he took all necessary steps to prevent the son from using the gun.
(b) Yes, he was the one who gave the gun and allowed him to use it by giving an opportunity.
(c) No, the son was almost 17 years and 11 months of age; hence, he could think about his
well being and interest.
(d) Yes, the father was negligent in disposing off the gun.

Previous Years
Page 250 CLAT & AILET Papers
79. LEGAL PRINCIPLES :
1. Medical professionals are not immune from liability in tort on ground of negligence.
2. Services rendered to a patient by a doctor (except when given free of charge) by way of
consultation, diagnosis and treatment fall in the definition of “service” under the Consumer
Protection Act. In case of negligence, the doctors are liable in tort as well as under the
Consumer Protection Act.

FACTUAL SITUATION : A was the only child of his parents. Once he had high fever and his
parents called a doctor at home. This doctor used to work at a respectable hospital in Delhi. The
doctor administered certain medicines and asked the nurse to stay with him for the night and
administer to him a chloroquine injection. This injection was generally not suitable for young
children. The nurse, without prior test, followed instructions of the doctor and gave the injection.
As a result of an allergic reaction, the child died. The parents sued the nurse and the doctor.

DECISION :
(a) Doctor was rendering a “service”; hence liable to pay compensation.
(b) Doctor was not liable as he came to their home to give personal treatment and was not in
the Hospital.
(c) This is not a service; hence not liable.
(d) Only the nurse is liable.

80. LEGAL PRINCIPLES :


1. An act done by the consent of a person above 18 years is not an offence; provide the
offender did not intend to cause death or grievous hurt.
2. Mere pecuniary benefit is not a ‘thing done for a person’s benefit’.

FACTUAL SITUATION : A, poor man, is in dire need of money to pay off his money lenders. A
approaches Z, a doctor, to operate on him to remove one of his kidneys so that he can donate
it to needy people and earn money. The doctor explains to him the risks and thereafter proceeds
to remove his kidney. In the process, some complications develop and A develops an abdominal
tumour. Is Z guilty?

DECISION :
(a) Yes, donating kidney for money is illegal and amounts to trafficking of organs.
(b) Yes, removing kidney for money is not an act protected by this exception.
(c) No, Z performed the operation with A’s consent and fully explained him the risks involved.
(d) No, Z’s act was done for A’s benefit so that he can pay off the money lenders.

81. LEGAL PRINCIPLES :


1. An act done, even if without the consent of a person is not an offence, provided the offender
did not intend to cause death, and the act was done for the person’s benefit, in good faith.
2. Mere pecuniary benefit is not a ‘thing done for a person’s benefit’.

FACTUAL SITUATION : A is in a house which is on fire, with Z, a child. People below hold out
a blanket. A drops the child from the house top, knowing it to be likely that the fall may kill the
child but intending to save him from the fire. Unfortunately, the child is killed. Is A guilty?

Previous Years
CLAT & AILET Papers Page 251
DECISION :
(a) Yes, A had knowledge of his dangerous act. His act was not justified.
(b) Yes, A should have tried a less dangerous alternative.
(c) No, his act was done in good faith to save the child.
(d) No, he had the best of intentions and this was the only alternative.

82. LEGAL PRINCIPLE : Nothing is an offence which is done by any person who is, or who by
reason of mistake of fact, in good faith, believes himself to be bound by law to do it.

FACTUAL SITUATION : A, soldier, fires on a mob by the order of his superior officer, in
conformity with the commands of the law. B is killed due to such firing. Is A guilty of murder?

DECISION :
(a) Yes, he should have taken care to avoid any innocent person from being killed.
(b) No, he is bound by law to do it.
(c) Yes, as he has killed a person.
(d) The superior officer is guilty.

83. LEGAL PRINCIPLE : A person shall not be guilty of contempt of court on the ground that he
has published any matter which interferes with or obstructs or tends to obstruct the course of
justice in connection with any civil or criminal proceeding pending at the time of the publication,
if at that time he had no reasonable grounds for believing that the proceeding was pending.

FACTUAL SITUATION : X delivered a lecture at the local Rotary Club in favour of an accused
who is prosecuted for assaulting a police officer. He said that the accused is a victim of prevailing
corruption in the judiciary and he knows that the accused is going to be punished by the court
for being honest. Is X guilty of contempt of court?

DECISION :
(a) No, not guilty of contempt of court.
(b) Yes, guilty of contempt of court.
(c) Yes, X has dishonoured judiciary.
(d) No, X is an honest man.

84. LEGAL PRINCIPLE : Whoever intending to take dishonestly any movable property out of the
possession of any person without that person’s consent, moves that property in order to such
taking, is said to commit theft.

FACTUAL SITUATION : A, in good faith, believing property belonging to B to be A’s own


property, sells the property to C. Has A committed theft?

DECISION :
(a) No, because he did not have a dishonest intention.
(b) No, he did not move the property.
(c) No, he did not move the property that was in B’s possession.
(d) Yes, the constituent elements of theft are satisfied in this case.

Previous Years
Page 252 CLAT & AILET Papers
85. LEGAL PRINCIPLES :
1. Preparation to commit an offence is not an offence.
2. After one has finished preparation to commit an offence, any act done towards committing
the offence with the intention to commit it, is an attempt to commit the offence which is by
itself an offence.

FACTUAL SITUATION : Manish wanted to kill Nandini and had therefore gone to the market to
buy explosives to plant in her house. Manish kept those explosives in his godown as he
planned to plant them early next morning. But as the explosives were stolen in the night he
could not plant them in Nandini’s house. However, Nandini came to know about Manish’s plan and
therefore wants to file a complaint against him. Will she succeed?

DECISION :
(a) Yes, because he has done something more than mere preparation.
(b) No, because Nandini did not die.
(c) Yes, because there existed a mala fide intention.
(d) No, because mere preparation is no offence.

86. LEGAL PRINCIPLE : Whoever intentionally uses force to any person, without that person’s consent,
in order to committing of any offence, or intending by the use of such force to cause, or knowing it
likely to cause injury, fear, or annoyance to the person to whom the force is used, is said to use
criminal force to that other.

FACTUAL SITUATION : Akash was burning some crackers in his house when his dog got scared
and got unruly which scared his brother Mohsin. Can this be called criminal force?

DECISION :
(a) Yes, because he did it intentionally.
(b) No, because law doesn’t account for trivial things.
(c) Yes, because it led to annoyance of Mohsin.
(d) No, because there was no mala fide intention.

87. LEGAL PRINCIPLE : No person shall be convicted of any offence except for violation of a law in
force at the time of commission of the act charged as an offence, nor subjected to a penalty greater
than which might have been inflicted under the law in force at the time of commission of the offence.

FACTUAL SITUATION : A boy of 16 years was convicted of house trespass and theft. He was
sentenced to imprisonment for 6 months and fine was also imposed. After the judgement, the
Probation of Offenders Act came into force. It provided that a person below 21 years may not
ordinarily be sentenced to imprisonment. Now the boy claims the benefit of this Act. Should he get
it?

DECISION :
(a) No
(b) The rule of beneficial interpretation required that the benefit of ex post facto law can be applied
to reduce his sentence.
(c) A boy below 18 years is a minor and so should not be punished.
(d) None of the above.

Previous Years
CLAT & AILET Papers Page 253
88. LEGAL PRINCIPLE : No one can be punished for the same offence twice.

FACTUAL SITUATION : Aditya is accused of harassing his colleague Nimisha. She complains to
her superior and a departmental inquiry is initiated against Aditya. Aditya tries to intimidate Nimisha
and continues harassing her following which she complains to the police. The departmental inquiry
finds him guilty and terminates his services. Later, he is found guilty by a court and is jailed for 2
years. He claims that he has been punished twice.

DECISION :
(a) He has been punished twice for the same offence.
(b) He has not been punished twice for the same offence.
(c) He has been punished for different offences.
(d) None of the above

89. LEGAL PRINCIPLE : To be held guilty of an offence, one should have done the act that
causes the intended result.

FACTUAL SITUATION : A, with the intention to murder B stabs him repeatedly with a knife. B
is taken to the hospital and is found to be out of danger. Thereafter, due to the doctor’s
negligence, B’s wounds are infected and he requires surgical interventions. During the time of
operation to remove his infected leg, B died on account of administration of general anaesthesia.

DECISION :
(a) A is guilty of murder. -
(b) A is not guilty of murder though he may be guilty of attempt to murder.
(c) A is not guilty of murder but he may be guilty of causing hurt.
(d) The doctor is liable as he was negligent.

90. LEGAL PRINCIPLE : Every agreement, by which any party is restricted absolutely from enforcing
his rights in respect of any contract, by the usual legal proceedings in the ordinary tribunals,
is void to that extent.

FACTUAL SITUATION : Feroz and Pinto entered into an agreement for rendering certain services.
As per the prevailing law, the agreement may be enforced either at Jaipur or Udaipur. The
agreement itself, however, specifies that upon breach, the parties can only approach courts at
Jaipur. Feroz breaches the contract and Pinto, being a resident of Udaipur, would like to sue
him in Udaipur. He challenged the validity of the clause.

DECISION :
(a) Pinto will succeed as Feroz had made him suffer and the law must take his convenience
into account and allow him to sue in Udaipur.
(b) Pinto will succeed as the contrac’t does not allow him to institute any legal proceedings in
any court or tribunal in Udaipur.
(c) Pinto will fail as the contract does not restrain him from instituting legal proceedings in
Jaipur.
(d) Pinto will fail as he was of sound mind while entering into contract and having accepted it,
he cannot now deny his obligation.

Previous Years
Page 254 CLAT & AILET Papers
91. LEGAL PRINCIPLES : A contract is an agreement enforceable by law.

FACTUAL SITUATION : Amita invited Bina to her house for dinner. Bina accepted the invitation
but later did not go. On Bina’s failure to attend, Amita filed a suit against Bina for the price of
non-consumed food. Will the law enforce this agreement?

DECISION :
(a) No, it is a social agreement.
(b) Yes, Amita can recover amount for non-consumed food.
(c) No, as Bina did not accept the invitation in writing.
(d) No, because the law does not take account of trivial things.

92. LEGAL PRINCIPLES :


1. Once a person accepts another’s offer, and signifies such acceptance to the former, a contract
comes into existence between them.
2. Uncertain agreements are void agreements.
3. Rejected offers can be accepted only if renewed.

FACTUAL SITUATION : Bakshi wanted to purchase a particular land. He sent a letter to his cousin,
Dutt, offering him Rs. 4 lakhs for it. Dutt replied that he would not sell it below 5 lakhs. Bakshi
communicated his willingness to pay this amount. Dutt did not sell the land to Bakshi. Bakshi sued
him for breach of contract.

DECISION :
(a) Dutt is liable because once he communicates an offer to Bakshi and Bakshi accepts it, a
contract comes into existence.
(b) Dutt is liable as he has misled Bakshi by his actions.
(c) Dutt is not liable because he has rejected the offer by giving a counter offer which is also not
specific.
(d) Dutt is not liable as there is no legally enforceable contract.

93. LEGAL PRINCIPLE : An agreement is void if the court regards it as opposed to the public policy.

FACTUAL SITUATION : Sunita, while her husband Shankar was alive, promised to marry Neel in
the event of Shankar’s death. Subsequently, Shankar died, but Sunita refused to marry Neel. Neel
sues Sunita for damages for breach of promise.

DECISION :
(a) Sunita is liable as she is to bound to marry Neel.
(b) She is liable to compensate Neel for breach of promise.
(c) Neel can marry someone else.
(d) She is not liable as the contract is opposed to public policy and so void.

Previous Years
CLAT & AILET Papers Page 255
94. LEGAL PRINCIPLE : An agreement is void if its object is unlawful.
FACTUAL SITUATION : Sunil had a rich uncle who owned prime property in Chennai and had lot of
money in the bank. Being the only heir, Sunil was sure that he would inherit the property. One day,
the uncle called him to his room and announced that he planned to marry again. This angered Sunil
and he plans to murder his uncle so he hired Anuj, a murderer and entered into a contract with him
to kill his uncle. Sunil agreed to pay As. 10 lakhs to Anuj and even paid 5 lakhs as advance. The
following night Anuj entered the uncle’s house intending to kill him. On reaching there, he realised
that Sunil’s uncle was already dead so he left without doing anything. Next day, after post mortem
report, it transpired that Sunil’s uncle had died due to heart attack. Now, Sunil wants to recover the
advance from Anuj. Will he succeed?

DECISION :
(a) Yes
(b) No
(c) Anuj is liable to return the amount as the act was done by him.
(d) None of the above.

95. LEGAL PRINCIPLES :


1. Consideration must be of value in the eyes of law.
2. Consideration is not real if it is illusory.

FACTUAL SITUATION : Arjun received summons to appear at a trial as a witness on behalf of


Bitu, the accused. Bitu promised to pay him a sum of Rs. 1,000 for his trouble. On default by
Bitu, Arjun filed a suit to recover the said sum. Will he succeed?

DECISION :
(a) No, as the consideration is of no value in the eyes of law.
(b) Yes, as the contract is supported by consideration.
(c) No, as there is no consideration for the promise.
(d) Yes, as he appeared before court only after Bitu agreed to pay him the amount of Rs.
1,000.

96. LEGAL PRINCIPLE : A minor is not competent to contract.

FACTUAL SITUATION : Deep, a 9th standard student realizes that he being a minor, he is not
permitted by law to execute a contract, appoints Mandeep as his agent to conclude purchase
of a land to gift it to his mother on her birthday. Mandeep accordingly prepares the papers for
the transaction but at the last minute the seller who had agreed to sell it now refuses to sell it
contending that he does not wish to sell the land to a minor. Deep seeks to enforce the
contract against the seller.

DECISION :
(a) Deep can enforce the contract - since Mandeep is his agent, Deep is deemed to have
personally entered into a contract.
(b) Deep cannot enforce the contract - only Mandeep can, since seller has entered into the
contract with Mandeep.
(c) Deep cannot enforce the contract since he is a minor.
(d) Deep can neither appoint an agent nor enforce the contract since he is a minor.

Previous Years
Page 256 CLAT & AILET Papers
97. LEGAL PRINCIPLES :
1. Acceptance must be given only by the person to whom the offer is made.
2. Communication of acceptance to a person who did not make the offer does not bind the
offeror.

FACTUAL SITUATION : Pal sold his business to Sam without disclosing it to his customers.
Mani, an old customer sent an order for goods to Pal by name. Sam, the new owner, executed
the order. Mani refuses to accept the goods from Sam as he intended to deal only with Pal. In
a suit by Sam against Mani:

DECISION :
(a) Sam cannot recover as Mani never intended to deal with him.
(b) Can recover the price as he had supplied goods only against the order made by Mani.
(c) Sam cannot recover as it was only an invitation to offer by Mani on which no acceptance
can be given by Sam.
(d) Sam can recover as the price of the goods as an offer once accepted results in a contract.

98. The main objectives of the Judicial Standards Accountability Bill 2010 is
1. Is a legislation which aims to increase accountability of the higher judiciary in India
2. It seeks to devise a new “complaint procedure” under which any person may be able to file a
complaint in writing against any judge of a superior court
3. The issue of Judicial Standards must be seen in the context of Article 124(4) of the Constitution
4. The Bill seeks to provide a straight jacketed definition of misbehaviour in Clause 2(k)

Select the correct code:


(a) 1 and 2 (b) 1, 2 and 3 (c) only 4 (d) 3 and 4

99. On April 2013, the Supreme Court held that the modification of a well known cancer fighting drug is
not a patentable new invention against which pharmaceutical firm?
(a) GlaxoSmithkline (b) Novartis (c) Ranbaxy (d) Cipla

100. Criminal Law (Amendment) Act, 2013 does not provide that:
(a) A rape convict can be sentenced to a term not less than 20 years
(b) Death sentence to repeat offenders
(c) Stalking and voyeurism are bailable offences
(d) Acid attack convicts can get a 10 year jail term

101. In India, Uniform Civil Code is applicable in the State of:


(a) Goa (b) Nagaland (c) Jammu & Kashmir (d) Maharashtra

102. NOTA is introduced in the voting machine as one among the options based on
(a) Representation of Peoples Act, 1950
(b) Representation of Peoples Act, 1951
(c) Decision of the Supreme Court of India
(d) Direction of the Election Commission

103. How many duties are provided under Part - IV A of the Constitution?
(a) 10 (b) 11 (c) 12 (d) 08

Previous Years
CLAT & AILET Papers Page 257
104. The Inter State Council has been constituted based on the recommendation of
(a) Second Administrative Reforms Commission
(b) National Commission to Review the Working of the Constitution
(c) Sarkaria Commission
(d) Punchhi Commission
105. Which of the following is not a constitutional body ?
(a) Finance Commission (b) State Public Service Commissions
(c) Election Commission (d) Planning Commission

SECTION - D : REASONING

106. Rahul : One would have to be blind to the reality of moral obligation to deny that people who believe
a course of action to be morally obligatory for them have both the right and the duty to pursue that
action, and that no one else has any right to stop them from doing so.

Richa : But imagine an artist who feels morally obliged to do whatever she can to prevent works of art
from being destroyed confronting a morally committed anti-pornography demonstrator engaged in
destroying artworks he deems pornographic. According to your principle that artist has,
simultaneously, both the right and duty to stop the destruction and no right whatsoever to stop it.

Which of the following, if substituted for the scenario invoked by Richa, would preserve the force of her
argument?
(a) a medical researcher who feels a moral obligation not to claim sole credit for work that was
performed in part by someone else confronting another researcher who feels no such moral obligation.
(b) a manufacturer who feels a moral obligation to recall potentially dangerous products confronting
a consumer advocate who feels morally obliged to expose product defects.
(c) an architect who feels a moral obligation to design only energy-efficient buildings confronting, as
a potential client, a corporation that believes its primary moral obligation is to maximise shareholder
profits.
(d) a health inspector who feels morally obliged to enforce restrictions on the number of cats a
householder may keep confronting a householder who, feeling morally obliged to keep every
stray that comes along, has over twice that number of cats.

107. Between 1951 and 1963, it was illegal in the country of Geronia to manufacture, sell, or transport
any alcoholic beverages. Despite this prohibition, however, the death rate from diseases related to
excessive alcohol consumption was higher during the first five years of the period than it was during
the five years prior to 1951, Therefore, the attempt to prevent alcohol use merely made people want
and use alcohol more than they would have if it had not been forbidden.

Each of the following, if true, weakens the argument EXCEPT:


(a) Many who died of alcohol-related diseases between 1951 and 1963 consumed illegally imported
alcoholic beverages produced by the same methods as those used within Geronia.
(b) Death from an alcohol-related disease generally does not occur until five to ten years after the
onset of excessive alcohol consumption.
(c} The death rate resulting from alcohol-related diseases increased just as sharply during the ten
years before and ten years after the prohibition of alcohol as it did during the years of prohibition.
(d) Between 1951 and 1963, among the people with pre-existing alcohol-related diseases, the
percentage who obtained life-saving medical attention declined because of a social stigma
attached to excessive alcohol consumption.
Previous Years
Page 258 CLAT & AILET Papers
108. Unless they are used as strictly temporary measures, rent-control ordinances (municipal
regulations placing limits on rent increase) have several negative effects for renters. One of
these is that the controls will bring about a shortage of rental units. This disadvantage for
renters occurs over the long run, but the advantage - smaller rent increases - occurs immediately.
In many municipalities, especially in all those where tenants of rent-control units have a secure
hold on political power and can get rent-control ordinances enacted or repealed, it is invariably
the desire for short-term gain that guides those tenants in the exercise of that power.

If the statements above are true, which one of the following can be properly inferred from them?
(a) It is impossible for landlords to raise rents when rent controls are in effect.
(b) In many municipalities, rent-control ordinances are repealed as soon as shortages of rental
units arise.
(c) In many municipalities there is now, or eventually will be, a shortage of rental units.
(d) in the long term, a shortage of rental units will raise rents substantially.

109. A government’s proposed 8 percent cut in all subsidies to art groups will be difficult for those
groups to absorb. As can be seen, however, from their response to last year’s cut, it will not
put them out of existence. Last year there was also an 8 percent cut, and though private fund-
raising was very difficult for the art groups in the current recessionary economy, they did
survive.

The reasoning in the argument is flawed because the argument


(a) Relies without warrant on the probability that the economy will improve.
(b) Overlooks the possibility that the cumulative effect of the cuts will be more than the arts
group can withstand.
(c) Equates the mere survival of the arts groups with their flourishing.
(d) Does not raise the issue of whether there should be any government subsidies to arts
groups at all.

110. Health insurance insulates patients from the expense of medical care, giving doctors almost
complete discretion in deciding the course of most medical treatments. Moreover, with doctors
being paid for each procedure performed, they have an incentive to over-treat patients. It is
thus clear that medical procedures administered by doctors are frequently prescribed only
because these procedures lead to financial rewards.

The argument uses which one of the following questionable techniques?


(a) Assigning responsibility for a certain result to someone whose involvement in the events
leading to that result was purely coincidental.
(b) Inferring the performance of certain actions on no basis other than the existence of both
incentive and opportunity for performing those actions.
(c} Presenting as capricious and idiosyncratic decisions that are based on the rigorous
application of well-defined principles.
(d) Depicting choices as having been made arbitrarily by dismissing without argument reasons
that have been given for these choices.

Previous Years
CLAT & AILET Papers Page 259
Directions (Q. 111 - 112) : Each question contains six statements followed by four sets of combinations of
three. Choose the set in which the combinations are logically related.

111. (1) Some buildings are not skyscrapers.


(2) Some skyscrapers are not buildings.
(3) No structure is a skyscraper.
(4) All skyscrapers are structures.
(5) Some skyscrapers are buildings.
(6) Some structures are not buildings.

(a) (1),(3),(5) (b) (2),(4),(6) (c) (6),(4),(1) (d) (1),(3),(6)

112. (1) All bins are buckets. (2) No bucket is a basket.


(3) No bin is a basket. (4) Some baskets are buckets.
(5) Some bins are baskets. (6) No basket is a bin.

(a) (2),(4),(5) (b) (1),(3),(2) (c) (3),(4),(6) (d) (1),(2),(6)

Directions (Q. 113 -114) : From the alternatives, choose the one which correctly classifies the four sentences
as a

F: Fact : If it relates to known matter of direct observation, or an existing reality or something known to
be true.
J: Judgment : If it is an opinion or estimate or anticipation of common sense or intention.
I: Inference : If it is a logical conclusion or deduction about something based on the knowledge of facts.

113. (1) If democracy is to survive, the people must develop a sense of consumerism.
(2) Consumerism has helped improve the quality of goods in certain countries.
(3) The protected environment in our country is helping the local manufacturers.
(4) The quality of goods suffers if the manufacturers take undue advantage of this.

(a) IJFJ (b) JFJI (c) IJJF (d) 1FJJ

114. (1) Everyday social life is impossible without interpersonal relationships.


(2) The roots of many misunderstandings have been cited in poor relations among individuals.
(3) Assuming the above to be true, social life will be much better if people understand the importance
of good interpersonal relations.
(4) A study reveals that interpersonal relations and hence life in general can be improved with a little
effort on the part of individuals.

(a) FJIJ (b) JFIF (c) FIFJ (d) IFFJ

Previous Years
Page 260 CLAT & AILET Papers
Directions (Q. 115 -120) : Each group of questions is based on a set of conditions. Choose the response
that most accurately and completely answers each question.

There are five flagpoles lined up next to each other in a straight row in front of a school. Each flagpole
flies one flag (red, white, or blue) and one pennant (green, white, or blue). The following are conditions
that affect the placement of flags and pennants on the poles:
On a given flagpole, the pennant and the flag cannot be the same colour.
Two adjacent flagpolescannot fly the same colour flags.
Two adjacent flagpoles cannot fly the same colour pennants.
No more than two of any colour flag or pennant may fly at one time.

115. If the 2nd and 5th pennants are blue, the 2nd and 5th flags are red, and the 3rd flag is white, then
which one of the following must be true ?
(a) Two of the flags are white.
(b) Two of the pennants are white.
(c) The 4th pennant is green.
(d) If the 1st flag is white, then the 1st pennant is green.

116. If the 1st flag is red and the 2nd pennant is blue, then which one of the following is NOT necessarily
true ?
(a) The second flag is white.
(b) If the 5th flag is red, then the 3rd flag is blue.
(c) If the 4th pennant is green, then the 1st pennant is white.
(d) If the 1st and 5th flags are the same colour, then the 3rd flag is blue.

117. If the 1st and 3 rd flags are white and the 2nd and 4th pennants are blue, then which one of the
following is false ?
(a) The 4th flag is red.
(b) The 1st pennant is green.
(c) The 3rd pennant is not red.
(d) The 5th pennant is green.

118. If the 1st and 4th flags are blue, and the 3rd pennant is white, then which one of the following
must be true ?
(a) If the 1st pennant is green, then the 5th pennant is white.
(b) If the 5th pennant is white, then the 1st pennant is green.
(c) The 2nd flag is red.
(d) The 5th flag is red.

119. If the 2nd flag is red and the 3rd flag is white, and the 4th pennant is blue, then which one of the
following must be true ?
(a) If the 5th flag is white, then the two of the pennants are blue.
(b) If the 1st flag is white, then the 2nd flag is white.
(c) If the 1st pennant is blue, then the 5th pennant is green.
(d) if the 1st pennant is green, then the 5th flag is not blue.

Previous Years
CLAT & AILET Papers Page 261
120. If the 1st flag and the 2nd pennant are the same colour, the 2nd flag and the 3rd pennant are the same
colour, the 3rd flag and the 4th pennant are the same colour, and the 4th flag and the 5th pennant are
the same colour, then which one of the following must be true ?
(a) The 1st pennant is white. (b) The 2nd flag is not white.
th
(c) The 5 flag is red. (d) The 3rd pennant is blue.

121. Nicky, who is Ronald’s daughter, says to Irene, “Your mother Rita is the youngest sister of my
father, who is the third child of Sylvester”. How is Sylvester related to Irene ?
(a) Maternal uncle (b) Father (c) Grandfather (d) Father-in-law

122. Pointing to a lady, a person says to his friend, “She is the grandmother of the elder brother of my
father”. How is the girl in the photograph related to the man?
(a) Niece (b) Sister (c) Aunt (d) Sister-in-law

123. Pointing towards a boy, Aruna said to Pushpa, “The mother of his father is the wife of your grandfather
(mother’s father)”. How is Pushpa related to that boy ?
(a) Sister (b) Niece (c) Cousin (d) Wife

124. The front door of Kiran house is towards the south. From the backside of her house she walks 50
metres straight then turns towards the left and walks 100 metres and after that turns right and stops
after walking 100 metres. Now Kiran is facing which direction ?
(a) East (b) South (c) West (d) North

125. If Northwest becomes south and southwest becomes east and all the other directions change in the
similar manner, then what will be the direction for north?
(a) Southeast (b) Northeast (c) North (d) Southwest

126. A policeman goes 20 km east and then turning to the south he goes 30 km and then again turns to
his left and goes 10 km. How far is he from his starting point?
(a) 30 km (b) 20 km (c) 10 km (d) 40 km

Directions (Q. 127 - 130) : In the following number series, find out the missing number:

127. 31, 32, 36, 45, ?


(a) 55 (b) 56 (c) 62 (d) 61

128. 1, 3, 2, 5, 3, 7, 4, 9, 5, 11, 6, ?
(a) 10 (b) 11 (c) 13 (d) 9

129. 300, 296, 287, 271, ?, 210


(a) 246 (b) 250 (c) 244 (d) 261

130. 12, 15, 19, ?, 30, 37


(a) 25 (b) 21 (c) 23 (d) 24

Previous Years
Page 262 CLAT & AILET Papers
Directions (Q. 131-132) : Each question is followed by two statements, I and II. Answer each question
using the following instructions.
Choose (a) : If the question can be answered by using statement I alone but not by using II alone.
Choose (b) : If the question can be answered by using Statement II alone but not by using I alone.
Choose (c) : If the question can be answered by using either statement alone.
Choose (d) : If the question can be answered by using both the statements together but not by either
statement.

131. In a cricket match the ‘man of the match’ award is given to the player scoring the highest
number of runs. In case of tie, the player (out of those locked in the tie) who has taken the
higher number of catches is chosen. Even thereafter, if there is a tie, the player (out of those
locked in the tie) who has dropped fewer catches is selected. Aakash, Biplab and Chirag who
were contenders for the award dropped at least one catch each. Biplab dropped 2 catches
more than Aakash did, scored 50, and took 2 catches. Chirag got two chances to catch and
dropped both. Who was the ‘man of the match’ ?
I. Chirag made 15 runs less than both Aakash and Biplab.
ll. The catches dropped by Biplab are 1 more than the catches taken by Aakash.

132. Four friends, A,B,C and D got the top four ranks in a competitive examination, but A did not get
the first, B did not get the second, C did not get the third, and D did not get the fourth rank. Who
secured which rank ?
I. Neither A nor D were among the first 2.
II. Neither B nor C was third or fourth.

Directions (Q. 133 - 134) : These four (4) items consist of two statements, one labeled as the
‘Assertion (A)’ and the other as ‘Reason (R)’ you are to examine these two statements carefully and
select the answers to these items using the codes given below:

Codes :
(a) Both A and R are individually true and R is the correct explanation of A.
(b) Both A and R are individually true but R is not the correct explanation of A.
(c) A is true but R is false.
(d) A is false but R is true.

133. Assertion (A) :


The phenomenon of nuclear fission generates great energy.

Reason (R) :
The process in which a nucleus is broken into two parts is called nuclear fission.

134. Assertion (A) :


The price of a stock is determined on the basis of demand and supply of the stock.

Reason (R) :
The value of the Sensex increases whenever there is a heavy demand for the stocks which
form the Sensex.

Previous Years
CLAT & AILET Papers Page 263
Directions (Q. 135 — 136) : The question given below has a statement followed by two conclusions I and
II. Consider the statement and the following conclusions. Decide which of the conclusions follow from the
statement. Mark answer as:
(a) If conclusion I follows
(b) If conclusion II follows
(c) If both conclusions I and II follows
(d) If neither conclusion I follows nor 11 follows

135. Statement :
A degree in law is of the most wanted degrees by youth in India.

Conclusions :
I. A degree in law guarantees a good profession.
II. A degree in law is the first choice for youth in India.

136. Statement :
Any student who is caught red-handed using unfair means discredits his parents and teachers.

Conclusions :
I. Such students try to show that their teachers don’t teach properly in the class.
II. Stringent actions must be taken against such students.

Directions (Q. 137 -140) : In the following set of analogies, one word is missing. Find it out from the given
options.

137. Errata : ? : : Flaws : Jewels


(a) Manuscripts (b) Books (c) Literature (d) Prints

138. inoculation : ?: : Exposure : Toughening


(a) Immunity (b) Punctuality (c) Vulnerability (d) Contagious

139. Steel : Rails :: Alnico . ?


(a) Aircraft (b) Machinery (c) Silverware (d) Magnets

140. Conscience : Wrong : : Police : ?


(a) Thief (b) Law (c) Discipline (d) Crime

Previous Years
Page 264 CLAT & AILET Papers
SECTION - E : MATHEMATICS

141. Mohan credits 15% of his salary into his bank and spends 30% of the remaining amount on household
articles. If cash on hand is Rs. 2,3801-, what is his salary ?
(a) Rs. 5,000 (b) Rs. 4,500 (c) Rs. 4,000 (d) Rs. 3,500

142. The average marks of a student in ten papers are 80. If the highest and the lowest score are not
considered the average is 81. If his highest score is 92, what is the lowest score ?
(a) 55 (b) 60 (c) 62 (d) 61

143. Ten years ago, Sunil was half of Sudip’s age. If the age of both at present is in the ratio of 3:4, what
will be the total of their present age?
(a) 20 years (b) 30 years (c) 35 years (d) 45 years

144. A and B started a business with a total capital of Rs. 30,000. At the end of the year, they shared the
profit in the ratio of their investments. If their capitals were interchanged, then A would have received
175% more than what he actually received. Find out the capital of B.
(a) Rs.20,000 (b) Rs. 22,000
(c) Rs. 21,000 (d) Rs. 23,000

145. The ratio in which Aman and Bimal have contributed to the capital of a company is 3 : 4. Bimal has
invested his capital for only 3 months and has received half as much profit as Aman, at the end of
the year. Find out for how much time has Aman invested his capital in the company.
(a) 8 months (b) 14 months (c) 15 months (d) 1 year

146. Two whole numbers whose sum is 64 can be in the ratio of ?


(a) 7 : 2 (b) 7: 6 (c) 3 : 1 (d) 8 : 7

147. In a call centre, 6 employees working for 10 hours complete a certain task. They started working at
11:00 am. This continued till 5:00 pm and after that, for each hour one more employee is added till
the work gets completed. At what time will they complete the work ?
(a) 7:10 pm (b) 8:00 pm (c) 7:35 pm (d) 6:35 pm

148. 72% of students in a class took Physics and 44% took Mathematics. If each student took Physics
or Mathematics and 40 took both, the total number of students in the class would be
(a) 200 (b) 240 (c) 250 (d) 320

149. Amar is twice as fast as Rohit and Rohit is thrice as fast as Chanda is. The journey covered by
Chanda in 42 minutes will be covered by Amar in
(a) 14 min 25 sec (b) 7 min (c) 28 min 37 sec (d) 54 min 35 sec

150. P, Q and R are three consecutive odd numbers in ascending order. If the value of three times P is 3
less than two times R, find the value of R.
(a) 5 (b) 7 (c) 9 (d) 11

Previous Years
CLAT & AILET Papers Page 265
AILET Question Paper 2015

SECTION - A : ENGLISH to be private.’ Throughout the development of Anglo-


American law, the individual has never possessed
Directions (Q. 1 - 6) : The questions in this section absolute dominion over property. Land becomes
are based on the passage. The questions are to be clothed with a public interest when the owner devotes
answered on the basis of what is stated or implied his property to a use in which the public has an
in the passage. For some of the questions, more interest in support of this position the chairman of
than one of the choices could conceivabiy answer the board of the Wilde Lake Shopping Centre in
the question. However, you are to choose the best Columbia, Maryland said :
answer; that is, the response that most accurately
and completely answers the questions. The only real purpose and justification of any of these
centres is to serve the people in the area-not the
The Constitution ofthe United States protects both merchants, not the developers, not the architects.
property rights and freedom of speech. At times The success or tailure of a regional shopping centre
these rights conflict. Resolution then requires a will be measured by what it does for the people it
determination as to the type of property involved. lf seeks to serve.
the property is private and not open to the general
public, the owner may absolutely deny the exercise These doctrines should be applied when
of the right of free speech thereon. On the other accommodation must be made between a shopping
hand, if public land is at issue, the First Amendment centre owner’s private property rights and the public’s
protections of expression are applicable. However, right to free expression. lt is hoped that when the
the exercise of free speech thereon is not absolute. Court is asked to balance these conflicting rights it
Rather it is necessary to determine the will keep in mind what Justice Black said in 1945:
appropriateness of the forum. This requires that “When we balance the constitutional rights of owners
consideration be given to a number of factors of property against those of the people to enjoy (First
including: character and normal use of the property, Amendment) freedom(s) ..... we remain mindful of
the extent to which it is open to the public, and the the fact that the latter occupy a preferred position.”
number and types of persons who frequent it. lf the
forum is clearly public or clearly private, the 1. In which one of the following cases would the
resolution of the greater of rights is relatively straight owner of the property probably be most free
forward. to restrict the freedom of speech ?
(a) an amusement park attended by five
ln the area of quasi-public property, balancing these million people each year owned by a
rights has produced a dilemma. This is the situation multinational company.
when a private owner permits the general public to (b) a small grocery shopping mall owned by
use his property. When persons seek to use the a husband and wife
land for passing out handbills or picketing, how is a (c) an enclosed shopping mall owned by a
conflict between property rights and freedom of single woman
expression resolved ? (d) an eight-unit residential apartment
building owned by a large real estate
The precept that a private property owner surrenders company
his rights in proportion to the extent to which he
opens up his property to the public is not new. In
1675, Lord Chief Justice Hale wrote that when private
property is “affected with a public interest, it ceases

Previous Years
Page 266 CLAT & AILET Papers
2. A conflict between property rights and 6. All other things being equal, the courts must
freedom of speech might arise in all of the (a) favour First Amendment rights over
following situations, EXCEPT property rights
(a) protestors carrying signs outside a (b) favour property rights over First
cinema in an enclosed shopping mall Amendment rights
(b) a disgruntled employee passing out (c) treat property rights and First Amendment
leaflets in front of a hairdresser’s salon rights equally
(c) a religious order soliciting funds and (d) protect property rights of the owners
converts in the swimming pool area ot a
condominium Directions (Q. 7 - 11) : For each of the following
(d) a candidate for mayor handing out flyers words below, a context is provided. From the
in front of his opponent’s headquarters alternatives given, pick the word or phrase that is
closest in meaning in the given context.
3. According to the passage, an owner’s
freedom to deny freedom of speech on his 7. Alphanumeric : The inclusion of
property is determined by all of the following alphanumeric features in cellular phones has
EXCEPT made sending SMSs very convenient.
(a) whether or not the land is open to the (a) using only alphabets
public (b) using digital codes
(b) the nature of and the usual use of the (c) using OSCOLA
property (d) using both letters and numerals
(c) the type of persons who frequents the land
(d) the nature of character of the owner 8. Oligarchy : Mrigank argued that all political
parties, including those which profess
4. We can infer from the passage that the author democratic values become the instruments
believes that shopping malls in America
of their leaders who eventually become a self-
(a) should be in the service of the people who
interested and self-satisfied oligarchy.
frequent them
(a) rule of a mob or crowd
(b) have a right to prohibit distribution of
(b) small group of people having control of
advertising handbills
state
(c) have a right to control any distributed
(c) an iron-rule of democrats
materials
(d) a socialist rule of activists
(d) should permit any charitable solicitations
9. Perestroika : In the 1980’s, the “state
5. According to the passage, the idea that a
socialist’ tradiiion became totally discredited
property owner’s rights decline as the property
as Gorbachev’s programme of perestroika
is more used by the general public
revealed the fundamental failures of the
(a) is peculiar to recent Supreme Court
planned economies of the Communist bloc.
decisions
(a) closing off all entries
(b) is attested by a three-hundred-year-old
(b) manifesto of failure
opinion
(c) opening up
(c) conflicts with the idea that property
(d) nagging policies
affected with a public interest ceases to
be private
(d) is now universally accepted in Great
Britain and in Canada

Previous Years
CLAT & AILET Papers Page 267
10. Bequeath : In consequences, they 17. Ante-diluvium
bequeathed to their followers no clear vision (a) old time (b) up-to-date
of the economics of socialism, that is state (c) time period (d) against dualism
ownership and planning the means of
production, distribution and exchange, and 18. To play truant
others had to advocate market socialism, the (a) to make a narrow escape
state regulation of capital rather than state (b) to run away from work without permission
ownership of planning. (c) to be clever
(a) hand down or pass on (d) none of these
(b) a powerful will of the people
(c) a method of governing 19. To see red
(d) to gjve in promise (a) to find fault with
(b) to be very angry
11. Redundancy : Communication is a subtle, (c) to criticise others
complex and continuously fluctuating (d) to victimise someone
process, affected by a multitude of factors
both external and internal, and two such 20. To flog a dead horse
elements are noise and redundancy. (a) to do a thing in vain
(a) that part of message that is predictable (b) to act in a foolish way
or conventional (c) to criticize strongly
(b) that word or phrase which is surprising (d) try to revive interest in a subject that is
(c) that meaning of a word which is sparingly out of date
used
Direction (Q. 21 - 25) : In this section, each passage
(d) that message which has no clear meaning
consists of four/five sentences. The sentences in
each passage have been jumbled up. These are
Directions (Q. 12 - 15) : In the following questions, labelled P, Q, R, S and T. You are required to find
choose the word which is odd one out. out the proper sequence of the sentences and mark
accordingly on the Answer Sheet.
12. (a) bedlam (b) anarchy
(c) anatomise (d) insurrection
21. P : You couldn’t have asked for a nicer, more
respectable-looking fellow than Jack, the
13. (a) miscreant (b) defendant
day he arrived in town for grandpa’s
(c) accused (d) plaintiff
funeral.
Q : He’d just arrived that morning, got the
14. (a) assault (b) extortion
word that the old Judge was gone’ he said.
(c) sub-poena (d) battery
R : He rode into the yard on his motorbike -
15. (a) barrister (b) attorney the one he’d sold last summer, which
(c) juror (d) advocare wasn’t a patch on the big red one he’d
bought later.
Direction (Q. 16 - 20) : Choose the exact meaning S : He was glad he was in time for the
of the idioms/phrases. services. He sat with Paresh all through
the funeral; he even cried - or if it wasn’t
16. Alpha and Omega crying, it was a darned good show.
(a) related to science
(b) the beginning and the end The proper sequence should be
(c) A to Z (a) QRPS (b) QRSP
(d) none of these (c) SPQR (d) PRQS

Previous Years
Page 268 CLAT & AILET Papers
22. P : Anything to do with his parents had R : At the time, l took this to mean, simply,
always been vague to Manish. “Give your very best to every piece.”
Q : He had been too young to remember S : A deep-chested, powerful man with a
them; he only knew the story of the fatal rugged, gentle face, Enesco looked at me
weekend at the Airondack camel and the across the violin he held under his chin,
overturned canoe. and shook his bow.
R : Then, as the years swept by and the
tragedy faded into time, they began taking The proper sequence should be
him there; and the lake again became a (a) QPRS (b) QSPR
quite lake in the foothills’ a camping place (c) SPRQ (d) SPQR
where the three of them had a good time.
S : Grandpa had still owned the camp a long 25. P : On the one hand, I want very much for
while afterwards he and Grandma hadn’t someone else to clean our house, as
gone there. neither I nor my husband, Ed, has shown
any aptitude for it.
The proper sequence should be Q : No one but me, for instance, should have
(a) PQRS (b) QRSP to clean up the dental floss heaped up
(c) PQSR (d) QPSR like spaghetti near the wastebasket where
l toss it each night, never catching on that
23. P : The role of Western values in floss is not something that can be thrown
contemporary Indian society is a subject with a high degree ot accuracy.
on which I have pondered for years. R : On the other hand, l’d feel guilt in flicting
Q : Moreover, various stakeholders of our such distasteful drudgery on another
company - employees, investors, human being.
customers and vendors - come from S : Have always wanted and not wanted a
across the globe. cleaning person.
R : An organisation is representative of
society, and some of the lessons that I The proper sequence should be
have learnt from the West regarding values (a) SPRQ (b) RQPS
are, I think, applicable to us as a nation. (c) RPQS (d) PQSR
Here are some of them:
S : I come from a company that is built on Direction (Q. 26 - 35) : Fill in the blanks with the
strong values. most appropriate word:
T : In dealing with them over the years, I have
come to appreciate several aspects of the 26. When you are living with your ––––––– values
West’s value system. and principles, you can be straight forward,
honest and –––––––
The proper sequence should be (a) inherited, distinct
(a) PQSTR (b) PSQTR (b) core, upfront
(c) SQTRP (d) SQRPT (c) innate, durable
(d) cultural, perceptive
24. P : “To play great music,” he said, “you must
keep your eyes on a distant star.”
Q : Eleven years old, I was taking a violin
lesson with Georges Enesco, my
teacher, in his Paris studio.

Previous Years
CLAT & AILET Papers Page 269
27. The quality of ––––––– between individuals 33. But ––––––– are now regularly written not just
and the organisation for which they work can for tools but well-established practices,
be ––––––– to the benefit of both parties. organisations and institutions not all of which
(a) life, conceptualised seem to be ––––––– away
(b) interactions, improved (a) reports, withering
(c) service, evaluated (b) stories, trading
(d) work, better (c) books, dying
(d) obituaries, fading
28. Genetic engineering in humans should be
used to ––––––– diseases, not to ––––––– 34. In this context, the ______ of the British
genetic uniformity. Labour Movement is particularly ______
(a) treat, foster (b) eradicate, cater (a) affair, weird
(c) cure, generate (d) avoid, promote (b) activity, moving
(c) experience, significant
29. Stating that the ATM operations are ––––––– (d) atmosphere, gloomy
losses, the nation’s largest bank, the State
Bank plans to _____ the management of 35. Indian intellectuals may boast if they are so
some of its ATMs. inclined of being ______ to the most elitist
(a) generating, resource among the intellectual _______ of the world
(b) generating, close (a) subordinate, traditions
(c) incurring, outsource (b) heirs, cliques
(d) reporting, tighten (c) ancestors, societies
(d) heir, traditions
30. Most journalistic writing could do with –––––
–– of paragraphs to make the prose more SECTION - B :
______
(a) suppression, legible
GENERAL KNOWLEDGE
(b) removal, argumentative
(c) simplification, abstruse 36. Which State has become first to fix minimum
(d) deletion, succinct educational qualification for panchayat polls?
(a) Kerala (b) Gujarat
31. Angered by bureaucrat’s ––––––– comments, (c) Rajasthan (d) Karnataka
the reporter insisted for a more –––––––
response. 37. Which country was chosen to chair
(a) redundant, repetitive Partnership in Population and Development
(b) tactless, immediate (PPD) for the year 2015 ?
(c) circumlocutions, direct (a) India (b) China
(d) sarcastic, beneficial (c) SriLanka (d) Ghana
32. One of the most productive researches ––––
––– in contemporary neurosciences is 38. Recently, the Prime Minister has launched
devoted to ––––––– maps of human the “Give it Up” campaign for voluntarily giving
consciousness. up
(a) trajectories, reconnoitring (a) use of tobacco products
(b) designs, enunciating (b) use of plastic
(c) paradigms, elucidating (c) LPG subsidy
(d) declensions, obfuscating (d) black money

Previous Years
Page 270 CLAT & AILET Papers
39. Who bagged the best actress award at the 46. Which Country has launched the “Let Girls
62nd National Film Awards ? Learn” initiative aimed at educating 62 million
(a) Priyanka Chopra (b) Vidya Balan girls around the World?
(c) Kangana Ranaut (d) Rani Mukherji (a) United States (b) New Zealand
(c) India (d) Australia
40. Simona Halep, who has won the lndian Wells
47. Which among the following parties declared
Open Tennis Women's Singles Title 2015,
victory in the 2015 lsrael Legislative Election?
hails from which country ?
(a) Zionist Union (b) Likud
(a) Belgium (b) Romania
(c) Yesh Atid (d) Joint List
(c) Switzerland (d) Sweden
48. Which of the following measures the value of
41. The winner of prestigious Dadasaheb Phalke a currency against a basket of other
Award 2014 is currencies?
(a) Mohan Lal (b) Shashi Kapoor (a) Real Exchange Rate
(c) Mani Ratnam (d) Girish Karnad (b) Effective Exchange Rate
(c) Real Effect Exchange Rate
42. The Bandung Conference was an important (d) Nominal Exchange Rate
step towards the creation of
(a) Non-Alignment Movement 49. Which is the only regenerative organ in
(b) United Nations human body?
(c) European Union (a) Brain (b) Liver
(d) SAARC (c) Pancreas (d) Lungs

43. Recently, the Union Government has 50. What is the name of the innovative Council
appointed Amitabh Bachchan as the brand proposed in the Railway Budget -2015 to
ambassador for its campaign against which promote innovation ?
disease ? (a) Navachar (b) Rail Tech
(a) Swine Flu (b) Tuberculosis (c) Kayakalp (d) Sankalp
(c) Hepatitis B (d) Diabetes 51. Lysosomes, which are known as suicidal
bags, are produced by which organelle ?
44. The World's first hydrogen powered tramcar (a) Mitochondria (b) Golgi body
has rolled off in which country ? (c) Ribosome (d) Peroxisome
(a) China (b) Japan
(c) U. S. (d) Russia 52. India's rank in the recently released World
Press Freedom Index (WPFI) 2015 is
45. Which is the only Central University in lndla (a) 120 (b) 135
which has Prime Minister as its Chancellor? (c) 136 (d) 140
(a) Banaras Hindu University
(b) Indira Gandhi National Open University 53. Who among the following has been named
(c) Rajiv Gandhi University as new heir of Mysore Royal Family ?
(d) Visva Bharti University (a) Yaduveer Gopal Raj Urs
(b) Chaduranga Kantharaj as Urs
(c) Aditya Gurudev Urs
(d) Chandra Shekar Urs

Previous Years
CLAT & AILET Papers Page 271
54. Who among the following is often called the 61. Which among the following States has
"Greenest Chief Minister of India" ? w o n t h e 1 0 th N a t i o n a l Aw a r d fo r
(a) Pawan Kumar Chamling Excellence work in Mahatma Gandhi
(b) Anandiben Patel National Rural Employment Guarantee
(c) Prakash Singh Badal Act ?
(d) Nabam Tuki (a) Karnataka (b) West Bengal
(c) Haryana (d) Madhya Pradesh
55. Which part of the World is referred to as the
Fertile Crescent ? 62. Who is the newly appointed Chairman of the
(a) Latin Americas (b) South East Asia empowered Committee of State Finance
(c) Middle East (d) Scandinavia Ministers on Goods & Services Tax (GST) ?
(a) J. K. George (b) K. M. Mani
56. Which of the following banks launched (c) Amit Mitra (d) Saurabh Patel
'Pocket', India's first digital bank on mobile
phones ? 63. What is the code name of the military
(a) Axis Bank (b) ICICI Bank operation in Yemen against Shia Houthi
(c) HDFC Bank (d) SBl Group?
(a) Operation Destruction Storm
57. India based Rickey Kej has won the Grammy (b) Operation Decisive Storm
Award for his album (c) Operation Desert Storm
(a) Morning Phase (d) Operation Black Star
(b) Beyonce
(c) The Lonely Hours 64. Who was sworn in as the President of Sri
(d) The Winds of Samsar Lanka in January 2015 ?
(a) Mahinda Deshapriya
58. Recently, UNICEF and which football legend (b) Mahinda Rajapaksa
has launched protection fund named "7" to (c) Gotabhaya Rajapaksa
protect the World's most vulnerable (d) Mithripala Sirisena
youngsters ?
(a) David Beckham 65. Who is the new Chief Election
(b) Lionel Messi Commissioner?
(c) Christiano Ronaldo (a) H.S. Brahma (b) Vinod Zutshi
(d) Zinedine Zidane (c) Nasim Zaidi (d) R. Balakrishnan
59. According to the latest data, which among
66. A minor planet is named after which one of
the following States has attracted maximum
the following Indian Legends ?
Foreign Direct Investment (FDl) in the
(a) Viswanathan Anand
country? (b) Sachin Tendulkar
(a) Gujarat (b) Tamil Nadu (c) A. B. Rahman
(c) Maharashtra (d) Karnataka (d) Milkha Singh

60. Which one of the following countries will hold 67. Who won the title Miss India 2015 ?
the presidency of the BRICS New (a) Aafreen Rachel (b) Vartika singh
Development Bank for the first six years ? (c) Koyal Fana (d) Aditi Arya
(a) India (b) China
(c) South Africa (d) Russia

Previous Years
Page 272 CLAT & AILET Papers
68. _______, the ‘founding father’ and ‘architect’ at Rudra started a website named ‘ricky-
of modern Singapore passed away on 23 thakur-is-a-jerk.com’. She created this
March 2015. website so as to warn other girls about ‘Ricky
(a) Mr. Lee Kuan Yew Thakur’. The real Ricky Thakur files a suit for
(b) Mr. Lee Hsien Loong defamation. Decide.
(c) Mr. Lim Hng Kiang DECISION :
(d) Mr. Lee Li Lian (a) Kiara shall be held liable for defamation
as she published a statement which was
69. Election Commission (EC) has announced injurious to Ricky's reputation.
to launch Electoral Roll Authentication (b) Rudra shall be held liable as he had led
Mission (ERAM) and Purification Drive. Kiara into thinking that he was Ricky
What is the aim of the mission ? Thakur and moreover, it was his fault in
(a) To enlist all the eligible voters in voters the first place that made Kiara create this
list website.
(b) To create awareness about the voting right (c) Kiara cannot be held liable as she had
(c) To weed out bogus voters from the voters actually been referring to Rudra and not
list the real Ricky Thakur.
(d) (a) and (c) (d) Kiara cannot be held liable as her act was
done in good faith as she intended to
70. What is the name of the party of the warn other girls.
Australian Prime MinisterTony Abbott that
saw a confidence vote on spilling the top 72. LEGAL PRINCIPLE : Whoever stores a
leadership in February 2015 ? substance which could cause damage on
(a) Liberal Party escape shall be absolutely liable (i.e. liable
(b) Australian Greens even when he has exercised necessary care)
(c) Labour Party for any damage caused by the escape of the
(d) Palmer United Party substance.
FACTUAL SITUATION: Union Carbide India
SECTION - C : LEGAL APTITUDE Limited (UCIL) manufactured methyl
isocyanate, an extremely toxic gas. Due to
Directions (Q. 71 - 95) : Given below is a statement a storm, the gas that was being stored in
of legal principle followed by a factual situation. Apply sealed containers got released. Before much
the principle to the facts given below and select the could happen, the local municipal authorities
most appropriate answer. managed to contain the disaster. The
authorities filed a suit against UCIL for the
71. LEGAL PRINCIPLE : A statement is costs that were incurred in decontamination.
defamatory in nature if it is injurious to a However, later it was realized that the clean-
person's reputation and if the statement has up by the authorities could have been done
been published. withoul spending as much resources and the
FACTUAL SITUATION : Rudra had been damage was not that significant. UCIL argued
dating a girl named Kiara for three weeks. that it would pay only part of the amount
But he had introduced himself to heras Ricky demanded by the authorities, which could
Thakur (who is one of Rudra's friends) and he have dealt with the contamination.
continued to be Ricky for the rest of their
relationship. But ultimately the relationship
ended badly and Kiara being upset and angry

Previous Years
CLAT & AILET Papers Page 273
DECISION : (d) Satwik is not liable as by the time Prateek
(a) UCIL is liable only to the extent of borrowed money from Abbas, the
contamination caused. lt does not need partnership was no more in existence
to pay the authorities the entire amount
demanded by them. 74. LEGAL PRINCIPLE : Everybody is under a
(b) The authorities are entitled to the whole legal obligation to take reasonable care to
sum, as UCIL shall be held liable for all avoid act or omission which he can foresee
the repercussions of their act even if they would injure his neighbour, the neighbour for
had exercised due care. this purpose is any person whom he should
(c) UCIL can plead that the escape of the have in his mind as likely to be affected by
gas had been caused by a storm and not his act.
due to its own negligence. lt was an FACTUAL SITUATION : Krish, while driving
inevitable accident. a car at a high speed in a crowded road,
(d) The municipal authorities should have knocked down a cyclist. The cyclist died on
analyzed the damage first before jumping
the spot with a lot of blood spilling around,
into action. lt was due to their own
Lekha, a pregnant woman passing by,
negligence because of which they had to
suffered from a nervous shock, leading to
shell out more than required.
abortion. Lekha filed a suit against Krishnan
claiming damages.
73. LEGAL PRINCIPLE : A partner is liable for
DECISION :
the debts incurred by the other partners in
(a) Krish will be liable, because he owed a
the course of Partnership.
duty of reasonable care to everybody on
FACTUAL SITUATION : Satwik and Prateek
the road including Lekha.
enter into a partnership to, produce a film,
(b) Krish will not be liable, because he could
wherein Satwik also directs the movie. The not have foreseen Lekha suffering from
movie bombed at the box office. nervous shock as a result of his act.
Consequently, they run into financial (c) Krish will be liable to Lekha because he
difficulties and the partnership ends. Prateek failed to drive carefully.
goes to Abbas to borrow some money, which (d) None of the above.
Abbas understands is for-repaying the debts
from the partnership. Prateek takes the money 75. LEGAL PRINCIPLE : The occupier of a
and absconds to Malibu. Abbas sues Satwik premise owes a duty of care to all his invitees
for the amount. Decide and visitors.
DEClSlON : FACTUAL SITUATION : Lalit was running a
(a) Satwik is liable to return the money as it dairy from his house. People used a part of
was his partner, Prateek, who directed his farm as shortcut to get to a nearby railway
the movle. station. Lalit who did not approve of this, put
(b) Abbas has been negligent in not properly up a notice that “Trespassers will be
enquiring the purpose for which Prateek prosecuted”. However since a number of
borrowed the money. Satwik is not liable
these people were also his customers he
to pay him back according to the principle
tolerated them. One day a person who was
of contributory negligence
using this short cut was attacked by a bull
(c) Satwik is not liable as prateek absconded
belonging to the farm. The injured person filed
with the money instead of using it to
a suit against him.
payoff the debts in the partnership.

Previous Years
Page 274 CLAT & AILET Papers
DECISION : FACTUAL SITUATION : Parliament enacted
(a) Lalit is not liable in view of the clear notice a law, which according to a group of lawyers
against trespassers. is violating the fundamental rights of traders.
(b) Lalit is liable for having kept a bull on his A group of lawyers files a writ petition
farm. challenging the Constitutional validity of the
(c) Lalit is not liable to the people other than statute seeking-relief to quash the statute and
his customers. further direct Parliament to enact a new law.
(d) Lalit is liable because in fact he allowed DECISION :
the people to use his premises. (a) The court can quash the existing law if it
violates fundamental rights but cannot
76. LEGAL PBINCIPLE : A master shall be liable direct Parliament to make a new law.
for the acts of his servants done in the course (b) The court can quash existing law if it
of employment. violates fundamental rights and can direct
FACTUAL SITUATION : PUL, a public sector Parliament to make a new law.
undertaking, is operating a number of bus (c) No writ would lie against Parliament, as
services for its employees in Pune. These the court has no authority to direct
buses are quite distinct in their appearance Parliament to enact or re-enact a law.
and carry the board “for PUL employees only”. (d) The court cannot quash the law as
M, a villager from neighbouring state, was reasonable restrictions can be put on the
waiting for a regular bus in one of the bus fundamental rights.
stops in Pune. A bus belonging to PUL
78. LEGAL PRINCIPLE : When one person
happened to stop nearby and number of
signifies to another his willingness to do or
people got into the bus. M, without realizing
abstain from doing anything, with a view to
that it was PUL bus, got into the bus and
obtaining the assent of that person to such
soon-thereafter, the bus met with an accident
an act or abstinence, he is said to have made
due to driver's negligence. M, along with
a proposal.
several others, was injured in the accident.
FACTUAL SITUATION : Ram sends a
M seeks to file a suit against PUL claiming
telegram to Sohan, writing : “Will you sell
damages.
me your Rolls Royce car? Telegram the
DECISION :
lowest cash price.” Sohan also replied by
(a) M will succeed, because he got into the
telegram: “Lowest price for car is Rs. 20 lakh."
bus without realizing that it was PUL bus.
(b) M will not succeed, because it was for Ram immediately sent his consent through
him to find out whether it was a public telegram stating “l agree to buy the cat for
transport. Rs. 20 lakh asked by you.”, Sohan refused
(c) M will succeed, because the driver was to sell the car
anyhow duty-bound to drive carefully. (a) He cannot refuse to sell the car because
(d) PUL is not liable as the bus met with an the contract has already been made.
accident due to drive’s negligence. (b) He can refuse to sell the car because it
was only invitation to offer and not the
77. LEGAL PRINCIPLE : Only Parliament or real offer.
State Legislatures have the authority to enact (c) lt was not a valid offer because willingness
laws on their own. No law made by the State to enter into a contract was absent.
(d) lt was not a valid contract as offer and
can take away a person's fundamental right.
acceptance is conveyed through
telegram.

Previous Years
CLAT & AILET Papers Page 275
79. LEGAL PRINCIPLE : A person is said to be DECISION :
of sound mind for the purpose of making a (a) X cannot be prosecuted because he had
contract if, at the time when he makes it, he actually no knowledge about the new
is capable of understanding it and of forming notification issued two days ago
a rational judgement as to its effect upon his (b) X cannot be prosecuted because
interests. ignorance of fact is excusable
FACTUAL SITUATION : Mr. X who is usually (c) X can be prosecuted because ignorance
of sound state of mind, but occasionally of of law is not excusable
unsound state of mind, enters into a contract (d) X’s liability would depend on the discretion
with Mr. Y when he was of unsound state of of the court.
mind. Mr. Y having come to know about this
81. LEGAL PRINCIPLE : Any direct physical
fact afterwards, wants to file a suit against
interference with goods in somebody’s
Mr. X.
possession without lawful justification is
DECISION :
called trespass of goods.
(a) Mr. X cannot enter into contract because
FACTUAL SITUATION : Z purchased a car
he is of unsound state of mind when he
from a person who had no title to it and sent
entered into contract.
it to a garage for repair. X believing wrongly
(b) Mr. X can enter into contract but the
burden is on the other party to prove that that the car was his, removed it from the
he was of unsound state of mind at the garage. Has X committed any offence?
time of contract. DECISION :
(c) Mr. X can enter into contract but the (a) X cannot be held responsible for trespass
burden is on Mr. X to prove that he was of of goods as he was under a wrong belief.
sound state of mind at the time of (b) X can be held responsible for trespass of
contract. goods.
(d) Contract with a person of unsound mind (c) Z has no right over the car as he purchased
is void. it from a person who has no title over it.
(d) None of the above.
80. LEGAL PRINCIPLE : Ignorance of Fact is
excused but ignorance of law is not an excuse 82. LEGAL PRINCIPLES :
to criminal liability. 1. Bigamy is not permitted under Hindu law.
FACTUAL SITUATION : X was a passenger 2. A Hindu of sound mind can adopt a child
of the gender they don’t already have a
from Zurich to Manila in a Swiss Plane. When
child of.
the plane landed at the Airport of Bombay on
3. Only the child's father, mother or guardian
22 November 2014 it was found on searching
has the capacity to give the child up in
the X carried 34 kg of Gold Bars on his person
adoption, under Hindu law.
and that he had not declared it in the ‘Manifest
FACTUAL SITUATION : Ramesh comes
for Transit’. On 24 Novemeber 2014, the
from a family where from the past two
Government of India has issued a notification
generations; all male members have had two
modifying its earlier exemption, making it
wives. He also wants to continue this tradition
mandatory now that the gold must be
and thus he married Suman first and then
declared in the “Manifest” of the aircraft.
Tania. Ramesh has a son, Gunjan,with
Suman. However, with Tania, he is childless.
He thus wishes to adopt some children. Rajat
is the child of Ramesh’s brother. However,

Previous Years
Page 276 CLAT & AILET Papers
Rajat’s parents decide to go off to Dubai for 5 (c) Raja is liable as you don’t expect
years and they leave their son under the care anything better from a gangster.
of Ramesh’s sister. Ramesh and Tania ask (d) Raja is not liable as he was confused as
for Rajat in adoption from his sister who to whom he should be returning the
agrees and Rajat is adopted. Later, Ramesh property to.
and Suman want to adopt a daughter as well
and they do adopt a girl-Sara-from an 84. LEGAL PRINCIPLES : The state shall make
orphanage. However, at the time of signing special laws for the upliftment of citizens of
the adoption deed, Ramesh was dead drunk. the country, and these laws can be made for
Ramesh and his entire family member are the benefit of any specific caste, class or sex
Buddhists. Keeping in mind the given of people living in the society.
principles and facts, solve the questions. Is FACTUAL SITUATION : The state of Hindu
Rajat’s adoption a valid adoption ? Pradesh comes out with a law, which provided
DECISION : for reservation to Muslims in all government
(a) No, because Ramesh already has a son. and government aided institutions. This
(b) No, because Ramesh's sister has no law is challenged in the High Court of
authority to give Rajat up in adoplron. Hindu Pradesh, as being arbitrary and
(c) Yes, it is a valid adoption because contrary to the established laws. Can the
Ramesh has no child with Tania ano challenge be successful ?
Ramesh's sister is Rajat's guardian for 5 DECISION :
years at least. (a) Yes, since people from other religions
(d) Both (a) and (b) would also start making such demand’s
which would jeopardize the unity and
83. LEGAL PRINCIPLES : Whoever dishonestly integrity of the country.
takes away any property from the possession (b) No, since the state has the right to make
of another, with an intention of such taking special laws for the upliftment of the
away, without his permission is liable for theft. citizens of the country.
FACTUAL SITUATION : Raja, a famous (c) Yes, since the state has not been
gangster, moves into an apartment in mandated to make reservation, based on
Kankurgachi, Calcutta. There, he discovers a person's religion.
that the previous owner of the apartment had (d) No, since the Government cannot neglect
left behind a pair of beautiful ivory handled the minorities.
combs. Mesmerized by their beauty and
85. LEGAL PRINCIPLE : The master principal
confused as to whom he should be returning
is liable for all acts done by his duly
them to, he decides to retain them and starts appointed servant/agent for all acts done by
using them. The previous owner of the combs him lawfully in the course of his employment.
gets to know; this and registers an FIR for FACTUAL SITUATION : A, B, C and D
theft against Raja. ls Raja liable ? carried on a business in partnership. While
DECISION : making a deal with another company, B bribed
(a) Raja is liable for theft as he failed to return the clerk there. ls the partnership firm
the property even when he knew it was vicariously liable ?
someone else’s property. DECISION :
(b) Raja is not liable as he is not taken it (a) No, as bribing is not in course of
away from anyone else's possession and employment of the partners.
there was no dishonest intention. (b) Yes, as partners are agent of the firm.

Previous Years
CLAT & AILET Papers Page 277
(c) Yes, as B can be said to have implied 88. LEGAL PRINCIPLE : Nobody shall unlawfully
authority for the same. interfere with a person’s use or enjoyment of
(d) No, as this act was not authorised by the land, or some right over, or in connection with
others. it. The use or enjoyment, envisaged herein,
should be normal and reasonable taking into
86. LEGAL PRINCIPLE : A contract which is account surrounding situation.
impossible to perform becomes void. FACTUAL SITUATION : Jogi and Prakash
were neighbours in a residential locality.
FACTUAL SITUATION : Surender agreed to
Prakash started a typing class in a part of
deliver a specific quality of rice to Sonakshi
his house and his typing sound disturbed Jogi
identified by both of them. Before delivery,
who could not put up with any kind of
the rice was burnt by short circuit. ls Surender
continuous noise. He filed a suit against
discharged from the performance of the Prakash.
contract? DECISION :
DECISION : (a) Prakash is liable, because he should not
(a) Surender is discharged from performance have started typing class in his house.
as the subject matter of the contract is (b) Prakash is liable, because as a
destroyed. neighbour, he should have realised Jogi’s
(b) Surender is discharged from performance delicate nature.
as the subject matter has been (c) Prakash is not liable, because typing
specifically identified. sound did not disturb anyone else other
(c) Surender is not discharged from than Jogi
performance as he can procure rice from (d) None of the above.
other sources. 89. LEGAL PRINCIPLES :
(d) None of the above. 1. Whoever causes death by doing an act
with the intention of causing death or with
87. LEGAL PRINCIPLE : Whoever dishonestly the intention of causing such bodily injury
misappropriates or converts to his own use as is likely to cause death or with the
any movable property is guilty of criminal knowledge that he is likely by such act
misappropriation of property. to cause death commits the offence of
FACTUAL SITUATION : A finds a culpable homicide.
government promissory note belonging to Z, 2. Mens rea and actus reus must concur to
bearing a blank endorsement. A knowing that result in a crime which is punishable by
the note belongs to Z, pledges it with a banker the law.
as security for a loan, intending to restore it FACTUAL SITUATION : A and B went for
to Z at a future time. Has A committed shooting. A knows Z to be behind a bush. B
criminal misappropriation ? does not know it. A induces B to fire at the
DECISION : bush. B fires and kills Z. Has an offence been
(a) Yes since he deprived Z from using his committed ?
property and used it for his own use. DECISION :
(b) No, since he intended to return the (a) A had mens rea but no actus reus. B had
actus reus but no mens rea. No one is
property to Z in the future.
guilty.
(c) No, it is theft and not criminal
(b) A induced B to fire at the bush with the
misappropriation. knowledge that Z is there. A is guilty
(d) Yes since he deprived Z from using his c.culpable homicide but B is not guilty of
property. any offence.
(c) Both A and B are guilty.
(d) None of the above.

Previous Years
Page 278 CLAT & AILET Papers
90. LEGAL PRINCIPLES : (d) No, the shopkeeper is guilty as he was
1. The crime of kidnapping involves taking the one who circulated the counterfeit note
someone away from the custody of their to Roshni.
lawful guardian.
3. The crime abduction involves inducing or 92. LEGAL PRINCIPLES :
forcing somebody to go away from some 1. To constitute a punishable criminal
place against their will. offence, guilty intention must accompany
FACTUAL SITUATION : A steals B's slave. an illegal act.
ls it a crime ? 2. Criminal mischief means causing damage
DECISION : to public property intentionally or with the
I. Kidnapping ll. Abduction lll. Neither knowledge that harm may occur.
REASON : FACTUAL SITUATION : Neel being a
(A) Slavery is illegal. Shahrukh Khan fan went for the premier of
(B) A has taken him away from B's lawful Happy New Year. As usual, he carried his
custody. pen-knife, a gift from his dead mother. At the
(C) A has forced somebody to go with him security check, impatient of waiting in the
against his will. queue, Neel slunk past the guards and the
DECISION : metal detector when no one was watching.
(a) l (B) (b) ll (C) Later, he was apprehended in the hall and
(c) III (A) (d) I (A) charged for mischief and possession of a
weapon when it was expressly forbidden.
91. LEGAL PRINCIPLE : Whoever delivers to DECISION :
another person as genuine any counterfeit (a) Neel is not criminally liable since he had
currency which he knows to be counterfeit, no intention to commit mischief.
but which that other person is not aware of at (b) Liable for possession of the weapon since
the time when he received it, is guilty of it was expressly forbidden and mere
counterfeiting currency. possession was enough; although he
FACTUAL SITUATION : While returning might not be liable for mischief as he did
home one day, Roshni realizes that the local not do anything.
shopkeeper has given her a fake note ot Rs. (c) Neel is not liable since the pen knife has
1,000. Disappointed, she goes to the same an emotional value and rather the guards
shop and buys cosmetics worth Rs. 600. She should be punished for the security
then passes the same fake note to the breach.
shopkeeper. The shopkeeper while inspecting (d) Liable for both possession of weapon and
the note finds out that it is fake. ls Roshni criminal mischief since he slunk past the
guilty ? guards which shows his intention to
DECISION : commit the crime.
(a) No, as she was merely attempting to
return the note to the same shopkeeper 93. LEGAL PRINCIPLE :
who gave her the note. 1. Everyone has a right to defend their life
(b) No, she is not guilty of any offence as and property against criminal harm
neither did she manufacture the note nor provided it is not possible to approach
did she circulate it with a view to deceive public authorities and more harm than is
the public. necessary has been caused to avert the
(c) Yes, as she attempted to pass on a note danger.
which she knew was counterfeit. 2. Nothing is an offence which is done in
the exercise of the right of private defence.

Previous Years
CLAT & AILET Papers Page 279
FACTUAL SITUATION : The accused found to commit it, is an attempt to commit the
the deceased engaged in sexual intercourse offence which is by itself an offence.
withhis 15 years old daughter. The accused FACTUAL SITUATION : A wanted to kill B
assaulted the deceased on the head with a and had therefore gone to the market to buy
spade which resulted in his death. Accused explosives to plant in his house. After A has
claimed private defence and the prosecution planted the bomb, he felt guilty and went
claimed that the sexual intercourse was with back to remove the bomb but while he was
the consent of the daughter. Here, doing so, B saw him and called the police.
DECISION : Can A be held liable ?
(a) Accused is entitled to the right of private DECISION :
defence since the girl was only 15 years (a) Yes, because he has done something
old. more than mere preparation.
(b) Accused exceeded the right of private (b) No, because B did not die.
defence. (c) Yes, because there existed a mala fide
(c) Accused is not entitled to private defence intention.
as the intercourse was consensual. (d) No, because he had removed before
(d) Accused is not entitled to private defence anything could happen.
as the right of private defence is available
for defending one’s life and property only. 96. The Supreme Court of India constituted _____
Bench in December 2014.
94. LEGAL PRINCIPLES : Necessity knows no (a) Constitution (b) Arbitration
law, and any person facing danger may do (c) Social Justice (d) Fast-Track
all that is necessary to avert the same till he
can take recourse to public authorities. 97. Who is the author of the book “lndian
FACTUAL SITUATION : Akshay, a law Parliamentary Diplomacy - Speaker’s
abiding citizen decided to remove the weed Perspective” ?
of corruption from India society. One day, (a) Sumitra Mahajan
confronted with a bribing official, Akshay (b) Somnath Chaterjee
decided to teach him a lesson and punched (c) Meira Kumar
him on his face. Akshay (b) Manohar Joshi
(a) Can plead defence of necessity as he was
being bribed which is a crime. 98. Which of the following recently became 123rd
(b) Cannot plead defence of necessity as State party to the International Criminal Court?
there was no necessity to act in the (a) lsrael (b) Palestine
manner he acted. (c) Jordan (d) Bangladesh
(c) Can plead defence of necessity as aware
99. Which body has launched the “Group of
and vigilant citizenry forms the basis of a
Friends Against Terrorism” ?
good democracy.
(a) United Nations (b) European Union
(d) Can plead defence of necessity as there
(c) ASEAN (d) SAARC
was no time to take recourse to public
authorities. 100. Which one of the following Committee was
constituted to review environmental law in
95. LEGAL PRINCIPLES : the country ?
1. Preparation to commit an offence is not (a) Subramanian Committee
an offence. (b) Kasturi Ranjan Committee
2. After one has finished preparation to (c) Madhav Nair Committee
commit an offence, any act done towards (d) Ullas Karanth Committee
committing the offence with the intention
Previous Years
Page 280 CLAT & AILET Papers
101. Which South-East Asian country has recently consciousness, where we become aware that
banned surrogacy services to end its we live with other people. Finally, we develop
flourishing rent-a-womb industry? a sense of values, which is our overall
(a) Singapore (b) India estimation of our worth in the world.
(c) Thailand (d) Vietnam
Which one of the following would be the best
102. The Third National Lok Adalat which disposed completion of this passage?
of 56000 cases in a single day was held under (a) The sum total of all these developments
the chairmanship of we call the self-image or the self-concept.
(a) Justice H. L. Dattu (b) This estimation of worth is only relative
(b) Justice T. S. Thakur to our value system.
(c) Justice Jagdish Singh Khehar (c) Therefore, our social consciousness is
(d) Justice Anil R Dave dependent on our sense of values.
(d) The sum total of living with other people
103. In which of the following cases, the Supreme and developing a sense of values makes
Court, recently, held that demand for dowry us a total person.
can be made at any time and not necessarily
before marriage? 107. Violence against racial and religious minority
(a) State of uttarakhand v. Bhim Singh & Anr groups increased sharply throughout the
(b) State of Bihar v. Arnesh Kumar & Anr country last year, despite a slight decline in
(c) State ot Rajasthan v. Prem Kumar state wide figures. Compiling incidents from
(d) State of Punjab v. Kans Raj & Ors police departments and private watchdog
groups, the County Human Relations
104. National Green Tribunal ruled _____ old diesel Committee reported almost 500 hate crimes
vehicles cannot ply on Delhi roads. in the year, up from only 200 last year. It was
(a) 15 years the first increase since the committee began
(b) 10 years to report a yearly figure six years ago. The
(c) 20 years lower state wide figures are probably in error
(d) 5 years due to underreporting in other countries;
underreporting is the major problem that state
105. Supreme Court struck down which Section surveyors face each year.
of the lnformation Technology Act 2000, in All of the following, if true, would support the
Shreya Singhal v. U. O. I, as unconstitutional? conclusion or the explanation of the
(a) 66 B (b) 66 A discrepancy in the state and country figures
(c) 66 C (d) 66 EXCEPT:
(a) The number of hate crimes and those
SECTION - D : REASONING resulting in fatalities has increased in
neighbouring states.
106. At birth we have no self-image. We cannot (b) Anti-immigration sentiment was fanned
distinguish anything from the confusion of this year by an anti-immigration ballot
light and sound around us. From this referendum.
beginning of no-dimension, we gradually (c) Many law-abiding members of minority
begin to differentiate our body from our groups are fearful or distrustful of the
environment and develp a sense of identity, police.
with the realisation that we are a separate (d) All of the countries in the state have active
and independent human being. We then begin private watchdog groups that carefully
to develop a conscience, the sense of right monitor hate crimes.
and wrong. Further, we develop social

Previous Years
CLAT & AILET Papers Page 281
108. In the last three years, the numbers of arrests yield less than the catch of five years ago,
for burglary and robbery in Sandy Beach has and less than one quarter of the total of ten
declined by more than 30 percent. At the years ago. The cod has almost disappeared.
same time, the city has reduced the size of The numbers of fisherman in Newfoundland
its police force by 25 percent. and New England have declined, and their
Which one of the following helps to resolve
yearly earnings are now at an all-time low.
an apparant discrepancy in the information
Yet radar has made fishing methods more
above?
(a) Neighbourhood Watch programs have efficient than ever.
always been active in Sand Beach. Which one of the following identifies most
(b) The number of reported burglaries and clearly a faulty assumption in the reasoning
robberies in Sandy Beach has increased of this passage?
in the last three years. (a) The argument fails to consider that the
(c) Compared to other cities in the state, decline in the catch may be due to factors
Sandy Beach has one of the lowest crime other than pollution.
rates. (b) The argument assumes that the waters
(d) Many of the residents of Sandy Beach off Newfound land are representative of
have installed expensive security systems all the American oceans.
in their homes. (c) The pollution of the sea may have been
caused by natural as well as human
109. To be admitted to Bigshot University, you forces.
must have a 3.5 grade-point average (GPA) (d) The argument does not allow for the
and a score of 800 on the admission test, a possibility that the catch may increase
3.0 GPA and a score of 1,000 on the in size in the next five years.
admission test, or a 2.5 GPA and a score of
1,200 on the admission test. A sliding scale 111. A number of lawsuits have been brought
exists for the other scores and GPAs. against popular singing groups charging that
suicidal themes in their songs have led to
Which one of the following is inconsistent teenage suicides. So far, the courts have
with the above?
found that the lyrics are protected under the
(a) Jagan was admitted with a 2.7 GPA and
Constitution. But what if this should change
a score of 1,100 on the admission test.
and a court decides that suicidal themes in
(b) No student with a score less than 800 on
popular songs are dangerous? In fact, the
the admission test and a 3.4 GPA will be
songs that have been charged so far are anti-
admitted.
(c) More applicants had a GPA of 3.5 than suicide; they present sardonically the self-
had a GPA of 2.5. destructive behaviour of drinking, drugs, and
(d) Some of the students with a score of less escape by death. They describe a pitiful state
than 1,200 on the admissions test and a of mind, but they do not endorse it.
GPA of less than 2.5 were admitted. Blaming suicide on the arts is nothing new.
In the late eighteenth centrury, Goethe’s
110. Only 75 years ago, the best fishing in the popular novel The Sorrows of Young Werther
world was the Grand Banks of the North was said to be the cause of a rash of suicides
Atlantic. But now overfishig and man’s in imitation of the novel’s hero. If we begin to
pollution have decimated the area. There will hold suicide in books or music responsible
be no fishing industry in the Americas in a for suicides in real life, the operas of Verdi
few years. The waters off Newfoundland now and Puccini will have to go, and Romeo and

Previous Years
Page 282 CLAT & AILET Papers
Juliet and Julius Caesar will disappear from Direction (Q. 113 - 118) : Each group of questions
the high school reading lists. is based on a set of conditions. Choose the response
that most accurately and completely answers each
Which one of the following is an assumption question.
necessary to the author s argument? A science student has exactly four flasks-1, 2, 3
(a) A lyric presenting suicide in a favourable and 4-originally containing a red, a blue, a green
light should not have Constitutional and an orange chemical, respectively. An experiment
protection. consists of mixing exactly two of these chemicals
(b) Literature or music cannot directly together by completely emptying the contents of
influence human behaviour one of the flasks into another of the flasks. The
(c) Freedom of speech is most threatened following conditions apply;
by our personal freedom The product of an experiment cannot be used in
(d) The audience, not the performer, is further experiments.
responsible for the audience’s actions. Mixing the contents of 1 and 2 produces a red
chemical. Mixing the contents of 2 and 3 produces
112. Archaeologists have come to the support of an orange chemical.
Arctic anthropologists. A small minority of Mixing the contents of 3 with the contents of either
anthropologists assert that stone Age tribes 1 or 4 produces a blue chemical.
of the Arctic domesticated wolves and trained Mixing the contents of 4 with the contents of either
them to haul sleds. Excavations have 1 or 2 produces a green chemical.
recently found evidence to support this claim.
Archaeologists have found wolf bones near 113. lf the student performs exactly one
the site of a Stone Age village. They have experiment, which one of the following could
also found walrus bones that might have been be the colours of the chemicals in resulting
used on primitive sleds. The small minority three non-empty flasks ?
of anthropologists believe that their theories (a) blue, blue, green
have been proved. (b) blue, orange, orange
(c) blue, orange, red
Which one of the foliowing is true of the (d) green, green, red
evidence cited in the paragraph above ? 114. lf the student performs ex a c t l y t w o
(a) lt conclusively contradicts the experiments, which one of the following
anthropologists’ conclusions. could be the colours of the chemicals in
(b) lt neither supports nor refutes the the resulting two non-emptv flasks ?
anthropologist’ conclusions positivety.
(a) blue, blue (b) blue, orange
(c) lt conclusively supports only a part of the
(c) blue, red (d) green, red
anthropologists’ conclusions.
(d) lt supports the anthropologists’ 115. lf the student performs exactly one
conclusions authoritatively. experiment and none of the resulting three
non-empty flasks contains a red chemical,
which one of the following could be the colours
of the chemicals in the three flasks ?
(a) blue, blue, green
(b) blue, green, green
(c) blue, green , orange
(d) blue, orange, orange

Previous Years
CLAT & AILET Papers Page 283
116. lf the student performs exactly one Direction (Q. 122 - 124) : Read the following
experiment and exactly one of the resulting information carefully and answer the questions given
three non-empty flasks contains a blue below.
chemical, which one of the following must be The city K is 30 km to the southeast of Z while Y is
the colours of the chemicals in other two 50 km to the northwest of K. Also, H is 38 km to
flasks ? the southeast of Y. L lies in the direct route between
(a) both green Y and K and its distance from H is 14 km. G also
(b) both orange lies on this route and is exactly midway between L
(c) both red and Y.
(d) one green and one red 122. A car starting from K at 9 a.m. and running at
a constant speed towards Y reaches H at
117. If the students will perform exactly two 9.24 a.m. and then reaches G at
experiments and after the first experiment (a) 9.18 a.m. (b) 10.16 a.m.
exactly one of the resulting three non-empty (c) 10.36 a.m. (d) 10.42 a.m.
flasks contains an orange chemical, then in
the second experiment the student could mix 123. If M is 1 km to the southeast of L, then it is
together the contents of flasks exactly midway between
(a) 1 and 2 (b) 1 and 3 (a) H and L (b) Y and K
(c) 2 and 3 (d) 3 and 4 (c) H and Z (d) None of these

118. lf the student performs exactly one experiment 124. The distance from G to H is
and none of the resulting three none-empty (a) 26 km (b) 24 km
flasks contains an orange chemical, then the (c) 12 km (d) 16 km
student must have mixed the
(a) flask 1 with flask 2 Direction (Q. 125 - 128): Each question contains
(b) flask 2 with flask 4 four arguments of three sentences each. Choose
(c) flask 2 with one of the other flasks the set in which the third statement is a logical
(d) flask 4 with one of the other flasks conclusion of the first two.
125. (1) Some bikes are mopeds. All mopeds are
Direction (Q, 119 - 121): Read the following scooters. Some bikes are scooters.
information carefully and answer the questions given (2) All children are hairs. No hairs are red.
below. No children are red.
In a joint family of seven persons L, M, N, O, P, Q (3) No pencil is pen. Some pens are
and R, two are married couple. ‘R’ is a housewife markers. Some pencils are markers.
and her husband is a lawyer. ‘N’ is the wife of ‘M’, ‘L’ (4) Every man has a wife. All wives are
is an engineer and is the grand-daughter of ‘R’ and devoted. No devoted has a husband.
‘O’ is the father-in-law of ‘N’, a doctor, and father of
‘P’, a professor. ‘Q’ is L’s brother and M’s son. (a) (1), (2), and (3) (b) (1) and (2)
(c) (3) and (2) (d) (1), (2), (3) and (4)
119. How is P related to M ?
(a) Son (b) Brother 126. (1) No moon is not red. AII stars are moon.
(c) Daughter (d) Uncle All stars are red.
(2) All doors are open. No open is outdoors.
120. How is Q related to O ? All door are not outdoors.
(a) Grandfather (b) Uncle (3) No Japanese can fire. All Chinese are
(c) Grandson (d) brother books. Japanese and Chinese can fight.
(4) No A is B. No B is C. No A is C.
121. Who is M’s father ? (a) (1) only (b) (2) only
(a) O (b) R (c) N (d) P (c) (1) and (2) only (d) (4) only
Previous Years
Page 284 CLAT & AILET Papers
127. (1) All envelopes are rectangles. All 131. Every player will become a champ.
rectangles are rectangular. All envelopes I. Rajesh is a player.
are rectangular. ll. Rajesh will become champ.
(2) Some things are smart. Some smart lll. Rajesh is not a player.
things are tiny. Some things are tiny. lV. Rajesh will not become a champ.
(3) Learneds are well read. Well read know. (a) l, lV (b) lV, I
Learneds know. (c) III, IV (d) IV, III
(4) Dieting is good for health. Healthy foods
Direction (Q. 132 - 135) : The question given below
are rare. Dieting is rare.
has a statement followed by two courses of action
(a) (4) only (b) (3) only
numbered I and II. You have to assume everything
(c) Both (1) and (3) (d) All of these
in the statement to be true. Then decide which of
128. (1) Shahrukh is an actor. Some actors are the two suggested course of action logically follows
for pursuing.
pretty. Shahrukh is pretty.
Mark answer as :
(2) Some executives are soldiers. All soldiers
(a) lf only l follows
are patriotic.
(b) lf only ll follows
(3) All cricketers are patriotic. Some (c) ll both l and ll follows
executives are soldiers. Some executives (d) lt neither l follows nor ll follows
are patriotic.
(4) All actors are pretty. Shahrukh is not an 132. Statement:
actor. Shahrukh is not pretty. The number of people who die on the roads
(a) (4) only (b) (2) only every year is so alarming that the numbers
(c) (1) only (d) (2) and (3) every year are also close to the numbers
which can be attributed to the major diseases
Direction (Q. 129 - 131) : Each question has a main
statement, followed by four statements labelled. I, of the world.
II, III and IV. Choose the ordered pair of statement, Course of Action :
where the first statement implies the second, and I. There should be an active campaign for
the two statements are logically consistent with the sensitising people towards road safety
main statement. norms.
II. There should be an increased emphasis
129. Whenever Vijay reads late into the night, his on enforcing speed limits, road safety
grandfather reprimands him. rules and traffic management on roads
I. His grandfather does not reprimand Vijay. across the world.
II. Vijay reads late into the night.
III. Vijay reads early in the morning. 133. Statement:
IV. Vijay’s grandfather reprimands him in the An unacceptable number of children die
morning during the first year of their lives. The high
(a) lll,lV (b) II,IV incidence of infant death is a major cause for
(c) I, II (d) none of the above concern for the health ministry.
Course of Action :
130. Either Amir is angry, or he shows mock anger. I. All government hospitals should be
l. Amir shows mock anger. privatized to improve health care facilities.
ll. Amir is angry. ll. Government should commit higher levels
lll. Amir does not show mock anger. of their budget to health services.
lV. Amir is not angry.
(a) III, II only (b) IV, I only
(c) I, II only (d) lll, ll and lV, I

Previous Years
CLAT & AILET Papers Page 285
134. Statement: 137. Which one of them is lying about another
The cream of India’s Cricket team is likely to person’s tendencies ?
retire in the next three years leaving a vacuum (a) Akshay
which the lndian Cricket team is going to (b) Himanshu
struggle to overcome. (c) Anuj and Himanshu
Course of Action : (d) Anuj
l. The BCCI should start to induct Going around the village, you come across three
youngsters into the team and start to give people. One of them is a dentist, one is a barrister
them exposure to pressure situations. and one is a professor. You want to know who is
II. There should be a rotation policy adopted who.
for senior players in order to prolong their Peter: I am not a professor. Shina is not a professor.
careers and keep them injury free. Matt: Peter is not a barrister. Shina is a professor.
Shina: Peter is not a dentist. I am not a professor.
135. Statement:
The lack of employment and income during 138. Which of the following is true ?
economic depression leads to an increase (a) Shina is the professor
in the crime rates. (b) Peter is the dentist
Course of Action : (c) Matt is the barrister
I. The government should provide an (d) None of the above
unemployment compensation as done in
advanced countries. Further, you come across three women, one of
ll. The government should encourage greater whom is an excellent singer. You start questioning
economic activity and increase the them, when you notice that Minaxi is wearing a flower
number of industries in order to reduce in her hair.
crime rates. Madhuri: I am not the singer. The singer wears a
flower in her hair.
Direction (Q. 136 - 140): Read the questions below Minaxi: I am the singer. The singer is amongst us.
very carefully and choose the correct answer. Jaya: Madhuri is the singer. Minaxi is not the singer.
On an island ‘Neverland’ the inhabitants always
answer any question with two sentences - one of 139. Who is the singer ?
which is always true and the other always false. (a) Madhuri
Perhaps due to this peculiar habit, there’s been a (b) Minaxi
high rate of suicides on the island. As a doctor, you (c) Jaya
have to identify potentially suicidal people and (d) Cannot be ascertained
counsel them. You know that all people who are
suicidal feel that the life is futile. On questioning You want to expand your horizons and decide to go
three inhabitants, these are the answers you get: to the village of “Where is Who” which is deep inside
Anuj: Himanshu is suicidal. I am not suicidal. the island. You come the border of “Kya Kya” and
Himanshu : I do not want to die. Akshay does not see a road sign. One leads to left and the other
want to die. right. Threre are no other roads. You ask the
Akshay: Life is futile. I am suicidal. inhabitants.
Maroof : I do not speak to the strangers. I am new
136. Who among the three is suicidal ? to these parts.
(a) Akshay Nafish : Take road to the right. I am married to
(b) Himanshu Ayesha.
(c) Himanshu and Akshay Ayesha : I am not Nafish’s wife.
(d) None of these Maroof is not new to these parts.

Previous Years
Page 286 CLAT & AILET Papers
140. Which one of the following is true ? (a) Rs. 550 (b) Rs. 580
(a) The road to the right leads to “Where is (c) Rs. 540 (d) Rs. 570
Who”?
(b) The road to the left leads to “Where is 146. Three maths classes: X, Y and Z take an
Who”? algebra test. The average score of class X is
(c) Nafish is married to Ayesha 83. The average score of class Y is 76. The
(d) None of the above average score of class Z is 85. The average
score of class X and Y is 79 and average
SECTION - E : MATHEMATICS score of class Y and Z is 81. What is the
average score of classes X, Y and Z ?
141. A piece of string is 40 centimetres long. lt is (a) 81.5 (b) 80.5 (c) 83 (d) 78
cut into three pieces. The longest piece is 3
times as long as the middle-sized and the 147. You can collect rubies and emeralds as many
shortest piece is 23 centimetres shorter than as you can. Each ruby is worth Rs. 4 crore
the longest piece. Find length of the shortest and each emerald is worth of Rs. 5 crore.
piece (in cm). Each ruby weighs 0.3 kg and each emerald
(a) 27cm (b) 5 cm weighs 0.4 kg. Your bag can carry at the most
(c) 4 cm (d) 9 cm 12 kg. What you should collect to get the
maximum wealth?
142. Fresh grapes contain 90% water by weight (a) 20 rubies and 15 emeralds
while dried grapes contain 20% water by (b) 40 rubies
weight. What is the weight of dry grapes (c) 28 rubies and 9 emeralds
available from 20 kg of fresh grapes ? (d) none of these
(a) 2 kg (b) 2.4 kg
(c) 2.5 kg (d) 2.6 kg 148. A man has 9 friends: 4 boys and 5 girls. In
how many ways can he invite them, if there
143. A group of men decided to do a job in 8 days. have to be exactly 3 girls in the invitee’s list?
But since 10 men dropped out every day, the (a) 320 (b) 160 (c) 80 (d) 200
job got completed at the end of the 12th day.
How many men were there at the beginning? 149. Ten points are marked on a straight line and
(a) 165 (b) 175 11 points are marked on another straight line.
(c) 80 (d) none of these How many triangles can be constructed with
vertices from among the above points?
144. In a race of 200 m run, A beats S by 20 m (a) 495 (b) 550 (c) 1045 (d) 2475
and N by 40 m. If S and N are running a race
of 100 m with exactly same speed as before, 150. Number of students who have opted for the
then by how many metres will S beat N ? subjects A, B and C are 60, 84 and 108
(a) 11.11 m (b) 10 m respectively. The examination is to be
(c) 12 m (d) 25 m conducted for these students such that only
the students of the same subject are allowed
145. Total expenses of a boarding house are partly in one room. Also the number of students in
fixed and partly varying linearly with the each room must be same. What is the
number of boarders. The average expense per minimum number of rooms that should be
boarder is Rs. 700 when there are 25 boarders arranged to meet all these conditions?
and Rs. 600 when there are 50 boarders. (a) 28 (b) 60 (c) 12 (d) 21
What is the average expense per boarder
when there are 100 boarders ?

Previous Years
CLAT & AILET Papers Page 287
AILET Question Paper 2015
SECTION - A : ENGLISH

Directions (Qs. 1-9): Read the following passage carefully and then answer the questions that follow.

Rural manual workers comprise the single largest occupational category in lndia. ln 1991, according to the
National Commission on Rural Labour, 60 percent of the workers in rural India were manual workers and
they numbered more than 160 million. The changes in the working and living conditions of rural labourers
are thus central to changes in the welfare of the rural population and of the country as a whole. The
structure and working of rural labour markets in India is complex; as is well known, there is great diversity
across regions and across segments of the labour market. This article brings together an interesting body
of research that seeks to understand and explain the types of changes that have accrued in the structure
of rural labour markets over the last few decades.

The 1980s were characterised by an explosion of the rural labour force, slow employment growth in agriculture
and a rise in the share of non-agricultural employment. The decade was also characterized by a growing
casualisation of the work force (for a relative rise in casual employment as opposed to regular employment).

At the same time, it was a period when agricultural wages increased in real terms and when income
poverty declined. There was what may be called "the tension between the estimated decline in poverty on
the one hand, and the slow growth of agricultural employment and increased casualisation of the labour
force on the other. Some of the trends in the development of rural labour over for this period are a source of
concern. These include, as Radhakrishnan and Sharma note, the continuous widening of the gap between
labour productivity in agricultural and non-agricultural occupations, the burgeoning mass of rural casual
workers who have no social security safety net, and the increasing number of women employed at very low
wages in agriculture. Another matter for concern, one that emerges from a desegregation of data on rural
unemployment by age groups, is that the incidence of unemployment is higher for persons in the age group
of 15-29 than for any other age group in others words, unemployment is typically high among new entrants
to the workforce.

ln her review of trends in wages, employment and poverty, Sheila Bhalla shows that the real wages of
agricultural labourers stagnated from the time of independence to the mid 1970s and then began to rise in
all parts of the country. This was also the period in which the incidence of rural poverty began to decline.
The rise in wages was not limited to the more prosperous agricultural zones, and Bhalla argues that the
movement in real wages was co-related with the increase in the share of non-agricultural employment in
total employment. As wages in non-agricultural work are typically higher than wages in agriculture, the
expansion of non-farm work could also explain some of the decline in rural poverty. In the 1990s, the
improvement in real wages and the decline in poverty were reversed while agricultural employment expanded.
Economic development all over the world has been associated with a rise in the share of employment in the
secondary and tertiary sectors of the economy and a fail in the share of the agricultural sector. In India,
changes in the composition of the rural workforce in the 1980s points to a "structural retrogression"

1. Give an appropriate title to the passage


(a) The complex labour markets in lndia
(b) Matters of concern in rural agriculture
(c) The agricultural and non- agricultural sectors: Changing perspective
(d) The Changing Structure of Rural labour market
Previous Years
Page 288 CLAT & AILET Papers
2. The author does not say
(a) ln 1991, about 3/5 of the workers in rural India were manual workers
(b) The decade of 1980s was characterised by a relative rise in casual employment as opposed to
regular employment
(c) There is no gender bias among workers in agriculture sector
(d) Wages in agricultural employment have been lesser than those in non-agricultural employment
in the 1990s

3. Why is the increasing gap between labour productivity in agricultural and non-agricultural occupations
a cause of concern' according to Radhakrishnan and Sharma ?
(a) This would increase the wages of agricultural sector
(b) This would read to pressure on both the agricultural and non-agricultural sectors as whichever
sector expands at the expense of the other, there would be increased labour pressure on that
sector and lesser economic development in the other
(c) This would indirectly mean a pressure on agricultural sector in terms of higher wages
(d) This would indirectly mean a pressure on non-agricultural sector in terms of higher wages

4. How does Bhalla explain the fact that the real wages of agricultural labourers began to rise in all
parts of the country after mid-70s ?
(a) Economic development increased for both the sectors across the world
(b) Economic liberalization in India led to the development of the agricultural sector
(c) The agricultural labourers became rich due to successive good crops during the time of Green
Revolution
(d) There was a mass outflow of agricultural labourers into the non-agricultural Sectors and the
remaining labourers. Who were lesser in number, now apportioned the existing agricultural
sector wealth, getting richer individually

5. What is the most important problem in understanding the condition of rural labour markets in India?
(a) The rural labour markets are uneconomic in nature
(b) The rural labour markets are very complex and there exists great diversity across regions and
across segments of such markets
(c) The rural labourers are a reticent lot; not forthcoming with their problems, not very open to
suggestions on how to improve their lifestyle
(d) They are lazy and want doles from the government without undertaking any viable productive
activities

6. What sort of tension exists between the decline of poverty and the slow growth of agricultural
employment and the increased casualisation of the labour force ?
(a) A decline in rural poverty is only possible when there is increased agricultural employment and
lower casualisation of the labour force
(b) The tension exists in the fact that an increased casualisation of the labour force would increase
Poverty
(c) When there is a decline in poverty there should be faster growth of agricultural employment and
the decreased casualisation of the labour force'
(d) Both (b) and (c)

7. What sort of Passage is this ?


(a) Political (b) Social commentary (c) Economic (d) Philosophical

Previous Years
CLAT & AILET Papers Page 289
8. The author does not say which of the following statements in the passage?
(a) The National Commission on Rural Labour gives data on the activities of rural labourers in lndia
(b) Economic development basically means a rise in the share employment in the agricultural
sector at the expense of the secondary and the tertiary sectors
(c) In India, employment was higher among the new entrants to the workforce in the 1980s
(d) Real wages of the agricultural labourers started showing an upward trend from the 1970s

9. Why are changes in the working and living conditions of rural manual workers of utmost significance
to the country as a whole ?
(a) Rural workers migrate a lot to the cities, adding to the already burgeoning population of these
places and so any improvement in their living conditions which would stall this trend would
benefit
(b) The rural workers live in abject poverty and a change in their working and living conditions is
therefore very crucial
(c) They form the bulk of the rural workers and so any change in their living standards augurs well for
the country as a whole
(d) Both (a) and (b)

Directions (Q. 10 - Q. 12): For the word given at the top of each table, match the dictionary definitions on
the left
(a, b, c, d) with their corresponding usage on the right (e, f, g, h). Out of the four possibilities given below
the table select the one that has all the usages correctly matched.

10. Run down

Dictionary Definition Usage


a Trace e My cat was run down by a bus.
b Criticize f Have you run down those addresses
asked for last week?
Suraj is running for president of the
c Hit with a car g
club.
d Champaign for a government position h My father runs everyone down.

(a) ah, bf, ce, dg (b) ah, be, cf, dg (c) ag, bf, ce, dh (d) af, bh,ce, dg

11. Wind up

Dictionary Definition Usage


a Finish e She just think it’s a big wind-up
b To tighten the spring of (a clockwork f He wound up the toy top and set it on
mechanism) floor.
To become nervous, tense, etc; If he doesn’t get his act together, he is
c g
excited going to wind up in prison.
The kids always get wound up to when
d An act or instance of teasing h
uncle Ronnie comes over.

(a) ae, bg, cf, dh (b) ae, bf, cg, dh (c) ag, bf, ch,de (d) ah, bg, cf, de

Previous Years
Page 290 CLAT & AILET Papers
12. Run off

Dictionary Definition Usage


Would you mind running off 20 copies
a Make leave e
of this document for me?
b Reproduce f You shouldn’t swim where the dirty
water runs off into the ocean.
c Leave quickly g Why did you run off after the dinner?
The new government is trying to run
d To flow h
the criminals off.

(a) ah, be, cg, df (b) af, bh,cg, de (c) af, bh,ce, dg (d) ag, be, ch, df

Directions (Q. 13 - Q. 17) : Read the following newspaper report and put the verbs in brackets into the
most suitable form. lndicate your choice in the provided box.

A woman______ (13) ______ (take) to hospital after her car collided with a lorry near Noida yesterday. She
______ (14) ______ (allow) home later after treatment. The road ______ (15) _________ (block) for an hour
after the accident, and traffic had to ______ (16) ______ (divert). A police inspector said afterwards: The
woman was lucky. She could ______ (17) ______ (kiil).

13. (a) is taken (b) took (c) was taken (d) take

14. (a) allow (b) is allowed (c) allowed (d) was allowed

15. (a) was blocked (b) block (c) is blocked (d) be blocked

16. (a) diverted (b) be diverted (c) was diverted (d) is diverted

17. (a) was killed (b) have been killed (c) killed (d) kill

18. In which of the following clusters, all the words mean 'ignoring existence of God' ?
(a) Pantheism, Agnosticism, Secularism (b) Atheism, Agnosticism, Secularism
(c) Deism, Secularism, Agnosticism (d) Atheism, Deism, Secularism

19. Which set of words are only nouns?


(a) Pompous, ridiculous, photographic (b) Penance, science, porous
(c) Analysis, praxis, thesis (d) Poisonous, vocalize, stupidity

20. Which set of words are only adjectives?


(a) Ridiculous, native, psychologise (b) Astronomy, sympathy, privacy
(c) Nation, action, privacy (d) Chemical, mathematical, french

21. In which set each word is a noun, adjective and verb also?
(a) Delegate, defeat, temporary, tertiary (b) Chronic, incumbency, parent, proponent
(c) Topic, alacrity, android, auditory (d) Square, precipitate, collect, free

Previous Years
CLAT & AILET Papers Page 291
Direction (Qs. 22 - 26): Find the odd one out.

22. (a) wary (b) gullible (c) credulous (d) naive

23. (a) gusto (b) verve (c) burst (d) zest

24. (a) beatific (b) diatribe (c) blithe (d) ecstatic

25. (a) trappings (b) orifice (c) egress (d) vent

26. (a) spectre (b) apparition (c) hoodlum (d) phantom

Direction (Qs. 27 - 35): Fill in the blanks with the most appropriate word.

27. You_________mad if you think I'm going to lend you any more money.
(a) should be (b) are supposed to be (c) must be (d) ought to be

28. Who _________was coming to see me this morning?


(a) you said (b) did you say (c) did you say that (d) you did say

29. I notified _________ I had changed my address.


(a) with the Bank (b) the Bank that (c) in the Bank (d) to the Bank that

30. lf I_________a more reliable car, l_________to Surat rather than fly.
(a) had; would drive (b) would have; would drive
(c) had; had driven (d) would have had; would drive

31. I first met Sohan on a beach________Kochi. I later found out that he had been a carpenter and a
dustman,________other things'
(a) by; among (b) near; between (c) by; between (d) near; among

32. 'When did you last hear_________Don?' 'He phoned me just this morning, He's coming to Delhi
next week, so we agreed_________ a time and place to meet.'
(a) from; on (b) about; on (c) from; at (d) of; to

33. Perhaps because something in us instinctively distrusts such displays of natural fluency, some
readers approach John Updike's fiction with_______.
(a) suspicion (b) bewilderment (c) veneration (d) recklessness

34. Despite the mixture's_________nature, we found that by lowering its temperature in the laboratory,
we could dramatically reduce its tendency to vaporize.
(a) volatile (b) resilient (c) insipid (d) acerbic

35. l_________you can swim so well and I can't'


(a) hate (b) hate it that (c) hate that (d) hate it

Previous Years
Page 292 CLAT & AILET Papers
SECTION - B : GENERAL KNOWLEDGE

36. Which folk dance India has made it to the Guinness Book of World Records as the largest folk
dance in the world ?
(a) Bihu (Assam) (b) Garba (Gujarat)
(c) Nati (Himachal Pradesh) (d) Bhangra (Punjab)

37. Which e-commerce company has made its maiden entry in the Thomson Reuters top 100 Global
Innovators list 2015 ?
(a) Amazon (b) Snapdeal (c) ebay (d) Alibaba

38. Which of the following groups of international institutions are also known as 'Bretton Woods Sisters'?
(a) United Nations and Bank of International Settlement
(b) IMF and World Bank Group
(c) IMF and WTO
(d) IMF, IBRD and International Trade Organisation

39. Which of the following variables is not part of the 'lmpossible Trinity' conundrum ?
(a) Capital Control (b) Exchange Rate (c) Fiscal Policy (d) Monetary Policy

40. The measurement of poverty-line in lndia is based on the criteria of


(a) Nature of employment (b) Dwelling houses
(c) Level of education (d) Calorie consumption

41. Who termed Paris deal on climate change as 'Climate Justice' ?


(a) Ban-Ki-Moon (b) Barack Obama (c) Narendra Modi (d) Francois Hollande

42. What is R2 - D2 ?
(a) lt is an astromech droid that extinguishes fires' repairs spaceships and assists the lead characters
in the Star Wars film
(b) lt is a new atomic particle discovered
(c) lt is vaccine against Ebola
(d) lt is a video game

43. Tunisian mediators of the so called National Dialogue Quartet won the Nobel Peace Prize 20l5.
Which of the following is/are part of Quartet?
I. The Tunisian General Labour Union (UGTT)
II. The Tunisian Confederation of Industry, Trade and Handicrafts (UTlCA)
III. The Tunisian Human Rights League (LTDH)
IV. The Tunisian Order of Lawyers

The correct answer is


(a) l and ll (b) ll and lll (c) l, ll and lV (d) l, ll, lll and lV

44. The Employee's Provident Fund organisation has won the National Award on e-governance 2015-16
for launching
(a) Unique ldentification Number (b) Universal Account Number
(c) Online Pension scheme (d) Online complaint portal

Previous Years
CLAT & AILET Papers Page 293
45. Which is the World,s first airport to fully run on solar power ?
(a) Vancouver (US) (b) Dubai (UAE) (c) Kochi (lndia) (d) Zurich (Switzerland)

46. Government has introduced Start-up Scheme to promote new enterprises' Such enterprises has
been awarded_________years tax holiday.
(a) 1 (b) 2 (c) 3 (d) 5

47. Consider the following statements in respect of PAN (effective from January 1,2016)
I. Quoting PAN is mandatory for cash payments made to settle hotel bills or for buying foreign
travel tickets of an amount above Rs 50,000
II. The PAN requirement for non-luxury cash transactions is Rs 2 lakh
III. PAN for making post office deposit of over Rs 50,000 has been dispensed with.
lV. PAN is mandatory on purchase of immovable property of Rs 10 lakh
The correct answer is
(a) l and ll (b) l, ll and lV (c) ll, lll and lV (d) l, ll, lll and lV

48. How many firms, recently, have got licence from RBI for establishing Payment Banks ?
(a) 10 (b) 11 (c) 12 (d) 13

49. Government has allowed_________FDI in operations of white level ATMs.


(a) 100% (b) 74% (c) 51% (d) 4e%

50. Which of the following pairs of rivers have been interlinked formally for the first time in lndia ?
(a) Betwa - Ken (b) Son - Ken (c) Ganga - Jhelum (d) Godavari - Krishna

51. Which of the following feature films from India has been declared as the official entry for 88th Oscar
Awards ?
(a) Queen (b) Court (c) Mary Kom (d) Kaaka Muttai

52. Why the sidereal day is 4 minutes shorter than the solar day ?
(a) The Earth is round in shape. (b) The Earth revolves around the Sun'
(c) The Earth rotates on its axis. (d) The Earth has an atmosphere'

53. Which is the only industrialised country of the world that did not ratify the Kyoto Protocol?
(a) Russia (b) United States (c) Japan (d) France

54. The 8th BRICS Summit (2016) is proposed to be held in


(a) Brazil (b) lndia (c) China (d) South Africa

55. Which of the following statements is incorrect about foreign investment in India ?
(a) Foreign Portfolio lnvestment (FPls) can invest up to 5.1% in multi-brand retail
(b) FPl up to 49% through automatic route is permissible across sectors
(c) 49% FPI in Brownfield pharma projects is allowed
(d) Foreign retailers are not able to have direct management control of an lndian venture

56. An lndian car maker recently changed the name of its newly launched car because the name was
similar to an epidemic
(a) Tata Motors (b) Maruti Suzuki
(c) Mahindra & Mahindra (d) Honda Motors

Previous Years
Page 294 CLAT & AILET Papers
57. The first telecommunication company to launch pan lndia 4G mobile services
(a) BSNL (b) Vodafone (c) Reliance Jio (d) Bharti Airtel

58. WTO has recently approved membership of which country ?


(a) Zimbabwe (b) Sweden (c) Afghanistan (d) Sri Lanka

59. FM broadcast band lies in_________.


(a) HF (b) SHF (c) UHF (d) VHF

60. What percentage of Employees Provident Fund can be invested in equity shares?
(a) 0% (b) 5%
(c) 10% (d) Minimum 5% and maximum 15%

61. Liquid crystals are used In


(a) Pocket Calculator (b) Display Devices (c) Wrist watches (d) All of the above

62. Which country recently allowed women to vote in local elections for the first time ?
(a) lran (b) lraq (c) lndonesia (d) Saudi Arabia

63. 'Brass Plate Subsidiaries' are usually set up


(a) For the purpose of Income tax avoidance (b) In the metal sector
(c) To take advantage of subsidies (d) As cross border investment

64. Global fast-food chain_________has maximum number of restaurants across the world.
(a) KFC (b) Subway (c) McDonalds (d) Starbucks

65. Who has been honoured by the Rajiv Gandhi Khel Ratna Award 2015 ?
(a) Rohit Sharma (b) Jitu Rai (c) Sania Mirza (d) Deepika Pallikal

66. Which of the following countries has recently declared a 'state of economic emergency' ?
(a) Bangladesh (b) Germany (c) Burundi (d) France

67. Section 80 TTA of the Income Tax Act pertains to interest earned on
(a) Saving Bank Accounts (b) Fixed Deposits
(c) Recurring deposit (d) Government saving schemes

68. Who is the chairperson of the Film and Television Institute of India (FTII) ?
(a) Suneel Darshan (b) Pankaj Dheer
(c) Gajendra Chauhan (d) Prashant Pathrabe

69. MNREGA program provides 100 days employment in rural areas. The Government during September
2015 decided to provide additional_________days employment in drought affected areas'
(a) 20 (b) 25 (c) 40 (d) 50

70. United Nations has appointed Anupam Kher as the ambassador of


(a) 'Let the Girl Live' campaign (b) 'He for She' campaign
(c) 'Share Humanity' campaign (d) 'Youth Now' campaign

Previous Years
CLAT & AILET Papers Page 295
SECTION - C : LEGAL APTITUDE

Directions (Qs. 71 - 82): Given below is a statement of legal principle followed by a factual situation. Apply
the principle to the facts given below and select the most appropriate answer.

71. LEGAL PRINCIPLE : A reasonable classification having nexus with the object sought to be achieved
is not violative of Article 14 or Article 16 of the Constitution of lndia.

FACTUAL SlTUATlON : 'X' is a male teacher in a Women's college, who applied for the post of
principal of that College. His candidature was rejected on the basis of the Government's policy of
appointing only women as Principal of a women's college. 'X' challenges the policy on the ground of
discrimination. Whether the challenge is sustainable?

DECISION :
(a) Yes, because rejection of X's candidature amounts to sexual discrimination and deprivation of
opportunity.
(b) No, the rejection does not amount to discrimination since it is a reasonable classification
permissible under the Constitution.
(c) No, because the policy of appointment of only lady Principal in a women's college is a reasonable
classification having a nexus with the object sought to be achieved.
(d) Yes, because the policy is violative of the guarantee of equality before law under Article 14 of the
Constitution.

72. LEGAL PRINCIPLE : Any institution or body can be a 'State' if it is created under the Constitution
or a statute; or if it is substantially financed by the Government; or the Government holds its share
capital.

FACTUAL SITUATION : K approached the High court by filing a writ petition against the Board for
Control of Cricket in lndia (BCCI). The argument advanced was that BCCI is a 'State' within the
meaning of Article 12 of the constitution of lndia. The question is whether the argument is acceptable?

DECISION :
(a) Yes, because the Board has monopoly on cricket in India.
(b) No, because the monopoly on cricket is neither State conferred nor State protected.
(c) No, because the control of the government on BCCI, if any, is only regulatory.
(d) No, because neither the Board is created under a statute nor any part of share capital of the
Board is held by the government and no financial assistance is given by the government to the
Board.

73. LEGAL PRINCIPLE : A suit shall be instituted in the court within whose jurisdiction the cause of
action arises; or the defendant actually and voluntarily resides or carries on business, or personally
works for gain.

FACTUAL SITUATION : 'Y' carries on business in Mumbai. 'Z' carries on business in Delhi. 'Z' buys
goods of 'Y' in Mumbai through his agent and request 'Y' to deliver them at Delhi. Accordingly, 'Y'
delivered the goods at Delhi. But he did not get the price of the goods delivered in Delhi. Therefore,
he intends to move the Civil Court for recovery of amount from 'Z'. Which court may 'Y' approach ?

Previous Years
Page 296 CLAT & AILET Papers
DECISION :
(a) 'Y' may institute the suit either at Delhi where Z carries on business or at Mumbai where the
cause of action arose.
(b) 'Y' may institute the suit at Delhi where 'Z' carries on business.
(c) 'Y' may institute the suit simultaneously at Delhi where 'Z' caries on business and at Mumbai
where the cause of action arose.
(d) 'Y' may institute the suit at Mumbai where the cause of action arose.

74. LEGAL PRINCIPLE : The acceptance of an offer will be valid only if it is made in the way it was
expected to be made.

FACTUAL SITUATION : There was a telephonic discussion between 'J' and 'K' for negotiating the
sale of the shop of former to the latter. Upon reaching an agreement as to the price of the shop of 'J'
at Rs 20 lakh, 'J' told 'K' to send a letter to him within two weeks confirming that she wishes to buy
the shop for the price finalized. Two days thereafter, 'K' gave her acceptance to 'J' over telephone but
sent the letter of confirmation after lapse of one month. ls 'J' bound by acceptance of 'K' ?

DECISION :
(a) Yes, because the acceptance was conveyed within two weeks over telephone and it was followed
by a letter of acceptance as stipulated.
(b) No, because although the acceptance over telephone was conveyed in time but not in the mode
specified and the letter of acceptance was also not sent within two weeks.
(c) No, because sale of immovable property cannot be finalized online; neither any acceptance can
be given over phone. Hence, the entire negotiation is invalid.
(d) Yes, because no law can compel the purchaser to give his acceptance through the mode
prescribed by the vendor.

75. LEGAL PRINCIPLE : A power conferred by a statute cannot be withdrawn by a subordinate legislation.

FACTUAL SITUATION : The Cinematograph Act conferred powers upon the District Magistrate
(DM) to grant license subject to the control of the government. The government framed Rules under
the said Act. The effect of these Rules was that the licensing power stood transferred to the Government
itself and the District Magistrate was rendered powerless. Whether such Rules are valid ?

DECISION :
(a) The licensing power was granted by the Cinematograph Act. Any withdrawal or transfer thereof
was possible only through an Amending Act and not by any Rules made under the Parent Act.
(b) Although the legislature has conferred power upon the DM to grant license but the government
being the implementing agency might find it unfeasible. Therefore, the government rightly withdrew
it from the DM.
(c) The Rules are valid since these are framed under the Parent Act in order to better implement it.
(d) The Rules are valid since the DM under the Parent Act was not independent but subject to the
control of Government'

76. LEGAL PRINCIPLE : Clause (1) of Article 15 of the Constitution of lndia prohibits the State from
discriminating between citizens on the ground only of religion, race, caste, sex, place of birth or any
of them.

Previous Years
CLAT & AILET Papers Page 297
FACTUAL SITUATION : The admission Rules of an Engineering College located in XYZ State of
India provided that no capitation fee shall be charged from the residents of the XYZ State but the
non-residents shall be required to pay capitation fee. Whether the Rules are violative of Article 15 (1)
of the Constitution ?

DECISION :
(a) yes, because Article 15 (1) prohibits discrimination between citizens on the ground only of
religion, race, caste, Sex, place of birth or any of them.
(b) yes, because Article 15(1) prohibits discrimination on the basis of place of birth which impliedly
includes place of residence.
(c) yes, because Article 15 (1) prohibits discrimination between citizens on the ground only of
religion, race, caste, sex, place of birth and the provision suffers from causus omissus and
"place of residence" is inadvertently omitted.
(d) No, because Article 15 (1) does not prohibit discrimination based on the place of residence.

77. LEGAL PRINCIPLE : ln the employer - employee relationship, the employer is held liable for all the
wrongs committed by his employees in the course of employment.

FACTUAL SITUATION : David was employed as a Driver in ABC & Co over the past 15 years and
has been appreciated by the General Manager for his hard work and sincerity. He has been rewarded
by the company for his accident free record. David's younger brother wanted to join the same
company as a driver. He obtained a Learner's Licence, joined a Driving School and was learning
driving during the last three months. He was on the verge of completion of the training and appear for
the Driving test. He wanted to have more practice before the test and requested his brother David for
using the Company's car for two days. David also allowed him to use the office car for the practice.
While he was practising driving, a truck came from the wrong side, hit the company's car driven by
David's brother, which in turn hit a pedestrian and injured him. The pedestrian sues the company for
damages.

DECISION :
(a) The Company is not liable as it was driven by David's brother
(b) The Company is liable as David allowed his brother to drive the car
(c) David's brother is personally liable
(d) The Company can shift the responsibility on to the truck driver

78. LEGAL PRINCIPLE : No person shall be deprived of his life or personal liberty except according to
procedure established by law and Civil Courts have coercive powers to compel attendance of witness
only within its local territory.

FACTUAL SITUATION : Puchu, a resident of Faridabad was summoned by the Delhi High Court as
a witness in a civil case regarding wrongful possession of immovable property filed by Amu against
Kichu. He refused to appear before the court due to his office job. He was prosecuted by the court.
ls he liable ?

DECISION :
(a) He is not liable because he is not the resident of Delhi.
(b) He is not liable because he has fundamental right under Article 21 of personal liberty.
(c) He is liable because he is called as a witness in a civil trial and it is a procedure established by
law.
(d) He is not liable because he has no interest in the suit property.

Previous Years
Page 298 CLAT & AILET Papers
79. LEGAL PRINCIPLE : Article 19(1) (g) of the Constitution of India guarantees to all citizens the right
to practice any profession, or to carry on any trade, occupation and business but Article 19 (6)
empowers the State to impose reasonable restrictions on this right in the interest of public.
FACTUAL SITUATION : Having experienced acute shortage of labour for agricultural purpose due to
engagement of agricultural labourer in manufacture of Bidis, the state Government enacted a law to
prohibit such engagement of agricultural labour in the manufacture of Bidis. Whether the law violates
the constitutional provisions ?
DECISION :
(a) No, because the law in a reasonable restriction in the interest of public as if labourers would not
be available for agricultural purposes there can be shortage of food grains and wastage of crops.
(b) No, because Bidis are harmful for health of people so any law preventing people from engaging
in manufacture of Bidis is in the interest of public.
(c) Yes, because the law imposes an unreasonable restriction as it indirectly makes the two
sectors (manufacture of Bidis and agriculture) alternative options for the labourers where as
some people would like to work in both of these.
(d) Yes, because the object sought to be achieved by this law is to keep sufficient labour supply for
agricultural purpose, which could have been easily achieved by restraining the employment of
agricultural labour in Bidi manufacturing during the agricultural season only. Absolute restriction
amounts to withdrawal of the right. Hence, the law is unconstitutional.

80. LEGAL PRINCIPLE : When a person unlawfully intervenes in the chattel of another person by which
the latter is deprived of its use, the former commits the tort of conversion. And nobody shall enrich
himself at other's cost.
FACTUAL SITUATION : X, a patient suffering from fibroids in her uterus approached KLM Medical
Institute. X was suggested to undergo surgery to remove the fibroids from her uterus. The operation
was successfully performed and X was discharged after few days. one of the researchers of the
KLM lnstitute discovered some rare and unique cells in the fibroids of x and using these cells, the
laboratory of KLM developed some life-saving drugs and earned rupees twenty crore from a leading
lnternational Pharma Company. When X came to know about it, she claimed five crore from the
Institute.
DECISION :
(a) KLM lnstitute need not share its income with X because X far from being deprived of the use of
her fibroids was actually benefitted by its removal.
(b) KLM lnstitute need not share its income with X because the medical institute instead of destroying
the waste fibroids of X conducted research on its own and invented new life-saving drugs.
(c) KLM lnstitute must share its income with X because KLM could not have achieved its success
without the fibroids of X.
(d) KLM lnstitute must share its income with X on moral grounds.

81. LEGAL PRINCIPLE: A judgment which binds only the parties to a suit in which the judgment was
passed is called judgment in personam; whereas a judgment which binds all men irrespective of
whether they were party to suit or not is known as judgment in rem.
FACTUAL SITUATION : "Judgment of a competent court determining contractual obligations of the
parties to a contract is an example of judgment in personam; but a judgment of a competent court
declaring a party to be insolvent is an example of judgment in rem." Comment on the correctness of
this statement.

Previous Years
CLAT & AILET Papers Page 299
DECISION :
(a) The statement is incorrect because a judgment relating to contract is a judgment in rem as it
binds both the parties to the suit as well as the strangers. But a judgment relating to insolvency
applies only to the person who has been adjudged to be an insolvent; hence it is a judgment in
personam.
(b) The Statement is wrong as both the judgments are judgments in rem as both bind not only the
parties to the suit but also others.
(c) The Statement is wrong as both the judgments are judgments in personam as both the judgments
bind not only the parties to the suit but not the others.
(d) The statement is correct.

82. LEGAL PRINCIPLE : According to Article 20 (1) of the Constitution, no person shall be convicted of
any offence except for violation of the law in force at the time of the commission of the act charged
as an offence, nor be subjected to a penalty greater than that which might have been inflicted under
the law in force at the time of the commission of the offence.

FACTUAL SITUATION : 'P' was charged with an offence punishable with imprisonment for a term of
one year. The Magistrate convicted him and awarded him a punishment of one year imprisonment.
While 'P' was undergoing the sentence, the law under which 'P' was convicted came to be amended
and the punishment for the offence of which 'P' was convicted was reduced to six months. The
defense filed an application to the Magistrate for review of sentence and to commute it to six
months. Can the application be allowed ?

DECISION :
(a) No, because penal laws only have prospective application.
(b) No, because a penal statute cannot be given retrospective effect.
(c) No, since at the time of coming into force of the amended law, 'P' was already suffering the
sentence and had not completed the full term. Hence, his case should not be dealt under the
new law.
(d) Yes, because retrospective application of criminal law if it is beneficial to the accused is not
against Article 20 (1) of the Constitution.

Directions (Qs. 83 - 89) : The following questions consist of two statements, one labelled as, Assertion'
and the other as 'Reason'. Read both the statements carefully and answer using the codes given below.

(a) Both A and R are true and R is the correct explanation of A


(b) Both A and R are true but R is not the true explanation of A
(c) A is true but R is false
(d) A is false but R is true

83. Assertion (A) : The entries in the three legislative lists are not always set out with scientific
precision.
Reason (R) : The entries are not powers but are only fields of legislation.

84. Assertion (A) : No action lies for mere damage caused by some act which does not violate a
legal right.
Reason (R) : An action lies for interference with another's legal right even where it causes no
actual damage.

Previous Years
Page 300 CLAT & AILET Papers
85. Assertion (A) : The parties to the contract must be competent to contract othenrvise it will be a
void contract.
Reason (R) : All wagering agreements are void.

86. Assertion (A) : Custom to have force of law must be followed from time immemorial.
Reason (R) : Custom represents common consciousness of the people.

87. Assertion (A) : An accused person cannot be forced to give his thumb impression.
Reason (R) : An accused person cannot be compelled to be a witness against himself.

88. Assertion (A) : ln federalism, there is division of powers between the centre and the States.
Reason (R) : The legislation is not invalid merely because it incidentally encroaches upon matters
which have been assigned to another organ.

89. Assertion (A) : lnternational law is not law at all.


Reason (R) : International law has no judicial system to enforce the law by applying sanctions.

90. The constitution (One hundredth Amendment) Act, 2015 amended the________ Schedule of the
Constitution to give effect to an agreement entered into by India and_________on acquiring and
transferring of territories between the two countries.
(a) Ninth; Pakistan (b) Second; Pakistan (c) First; Bangladesh (d) First; China

91. Who is the CEO of NITI Aayog ?


(a) Sindhushree Khullar (b) Arvind Panagariya
(c) Bikesh Debroy (d) Amitabh Kant

92. Allahabad High Court has held that the___________of deceased Government employees are eligible
for appointment on compassionate ground.
(a) wife (b) daughter (c) mother (d) married daughter

93. Which party has won the general elections of Spain in spite of its worst result since 1989 ?
(a) People's Party (b) Pedro Sanchez's Socialist party
(c) Popular Union (d) Ciudadanos Party

94. Pension Fund Regulatory and Development Authority has started using________ for online registration
under the National pension system scheme.
(a) PAN Card (b) passport (c) Aadhar Card (d) Driving License

95. When a vehicle is financed by a bank, what kind of charge does the bank have over the primary
security ?
(a) Pledge (b) Hypothecation (c) Assignment (d) Lien

96. Under which Act can action be taken against wilful defaulters of banking loans ?
(a) Under Section 420 of IPC
(b) SEBI Act
(c) Banking Regulation Act
(d) Securitisation and Reconstruction of Financial Assets and Enforcement of Security Interest Act
(SARFAESI Act)

Previous Years
CLAT & AILET Papers Page 301
97. Which of the following court / tribunal ordered levy of an environment compensatory charge on
commercial vehicles not bound for the capital yet passing through Delhi ?
(a) Supreme Court of India (b) Delhi High Court
(c) National Green Tribunal (d) None of these

98. Parliament has enacted which of the following Legislation in 2015 - 16 ?


I. The Juvenile Justice (Care and Protection of Children) Act
II. The Election Laws (Amendment) Act
III. The Scheduled Caste and Scheduled Tribes (Prevention of Atrocities) Amendment Act
IV. Commercial Courts, Commercial Division and Commercial Appellate Division of High Courts Act

(a) l and lll (b) lV only (c) l and lV (d) l, ll, lll and lV

99. ln the following Sates more than 60% seats are in reserved category in educational institutions
against 50% cap as fixed by the Supreme Court of India
I. Karnataka II. Tamil Nadu III. Odisha IV. Bihar
The correct answer is :
(a) only ll (b) l and ll (c) ll and lV (d) I' ll and lll

100. According to SEBI norms, a person found guilty of indulging in unfair trade practices shall be liable
to a penalty of
I. Rs. 25 crore
II. Three times the amount of profits made out of such practices,.....

The correct answer is :


(a) only l (b) only Il (c) either I or Il (d) neither I nor Il

101. The minimum wages in_________are the highest in the northern region.
(a) Rajasthan (b) Dethi (c) Punjab (d) Haryana

102. The correct sequence in ascending order of their creation of the following international institution is:
I. WTO II. GATT III. UNCTAD IV. NAFTA

(a) ll, lll, l, lV (b) lll, ll, l, lV (c) lV, ll, l, lll (d) l, ll, lll, lV

103. Government of India has decided to establish_________in selected High Courts.


(a) Tax Division (b) SC/ST Division
(c) Commercial Division (d) Economic Offence Division

104. 'Zero Rating' is a recent term used in


(a) lnsurance (b) Credit Rating (c) Energy Efficiency (d) Net Neutrality

105. Which Article of the Constitution of India was used to impose President Rule in Uttarakhand and
placing the Assembly under suspended animation in March 2016 ?
(a) Article 102 (b) Articte 143 (c) Article 356 (d) Arricle 365

Previous Years
Page 302 CLAT & AILET Papers
SECTION - D : REASONING

Directions (Qs. 106 - 109) : A passage is given below followed by several inferences. You have to examine
each inference separately in the context of the passage and decide upon its degree of truth or falsity. Mark
your answer as:
(a) lf the inference is 'definitely true'
(b) lf the inference is 'probably true'
(c) lf the 'data provide in inadequate'
(d) lf the inference is 'probably false'
Investors today have more investment options than were available just a few years ago. Choice in any
decision-making is good in so far it provides variety, differentiation and benchmarking. lt could also, however,
at times lead to clutter and "noise" if the options are mostly similar and undifferentiated. To make sense of
this choice conundrum, it is imperative for an investor to define objective - both returns and digestible risk
and then identify the possible options. The investor also needs to select the mix and regularly monitor that
objectives and investment outcomes remain aligned. Sounds simple, but can present the most confounding
situation which multiplies with the quantum of wealth.
106. lnvestors need to critically evaluate the risk of each investment option.

107. Present day investors need to use their judgement more critically before investing.

108. Multiple investment options of similar types helps in making better investment decisions.

109. In the past, investors were generally guided by the fund managers.

Directions (Qs. 110 -115): Read the following short passages and answer the questions that follow each
passage.

110. For some women, the cost of giving birth can be an unexpectedly a large burden. The average
normal birth now costs Rs. 3,200 and a birth with complications can cost thousands of rupees
more. Of women in the primary child-bearing age range of eighteen to twenty-four, who account for
about 40 percent of all births in this country annually, more than 25 percent have no health care
insurance to pay maternity costs.
lf the above statements are true, which one of the following must also be true ?
(a) Each year, about 75 percent of all births in this country are to women who have health care
coverage of maternity costs.
(b) Each year, about 60 percent of all births in this country are to women who are younger than
eighteen or older than twenty-four.
(c) For an average birth, health care insurance pays about 75 percent of Rs. 3,200.
(d) In this country, about 75 percent of the women who do not have health care coverage of maternity
costs are younger than eighteen or older than twenty-four.

111. Products sold under a brand name used to command premium prices because, in general, they
were superior to non-brand rival products. Technical expertise in product development has become
so widespread, however, that special quality advantages are very hard to obtain these days and
even harder to maintain. As a consequence, brand-name products generally neither offer higher
quality nor sell at higher prices' Paradoxically, brand names are a bigger marketing advantage than
ever.

Previous Years
CLAT & AILET Papers Page 303
Which of the following, if true, most helps to resolve the paradox outlined above?
(a) Brand names are taken by consumers as a guarantee of getting a product as good as the best
rival product.
(b) Consumers recognised that the quality of products sold under invariant brand names can drift
over time
(c) In the acquisition of one corporation by another, the acquiring corporation is interested more in
acquiring the right to use certain brand names than in acquiring existing production facilities.
(d) In the earlier days when special quality advantages were easier to obtain than are now, it was
also easier to get new brand names established.

112. The extent to which a society is really free can be gauged by its attitude towards artistic expression.
Freedom of expression can easily be violated in even most outwardly democratic of societies.
When a government's Arts Council withholds funding from a dance performance that its members
deem "obscene", the voice of a few bureaucrats have in fact censored the work of the choreographer,
thereby committing the real obscenity of repression.

Which of the following, if true, would most seriously weaken the argument above?
(a) Members of government Arts Council are screened to ensure that their beliefs reflect those of
the majority,
(b) The term obscenity has several different definitions that should not be used interchangeably, for
rhetorical effect.
(c) Withholding financial support for a performance is not the same as actively preventing or inhibiting
it.
(d) The Council's decision could be reversed if the performance were altered to conform to public
standards of appropriateness.

113. lt is not generally realised that when a court upholds or invalidates legislation or executive action, it
neither approves nor condemns any legislative policy, nor is if concerned with the wisdom or expediency
of the administrative action. lt merely determines whether the legislation or executive action is in
conformity with or contrary to the provisions of the Constitution. lt discharges the function of guarding
the Constitution, no more, no less. Judicial activism is to be properly understood in the context of
the extent and the vigour and the readiness with which the courts exercise their power of judicial
review. When courts actively perform an interventionist role, we witness the phenomenon of judicial
activism. When the judiciary exercises self-restraint in exercising the power of judicial review and
limits its role, there is absence of judicial activism. But the pendulum of judicial review is never static
and judicial activism, or lack of it, is a variable phenomenon.

Which one of the following, if true, would defeat the case presented above ?
(a) The judiciary is plagued with the Public lnterest Litigations against some or the other forms of
administrative action.
(b) Often, the judiciary is seen to be making disparaging remarks about the executive, pulling up
officials for lapses in conduct, many a time condemning them.
(c) The judicial review exercises that the judiciary often undertakes is what comprises judicial
activism.
(d) Judicial activism is not mandatory under the Constitution. lt is for the judiciary to decide whether
it wants to intervene in a certain issue or not.

Previous Years
Page 304 CLAT & AILET Papers
114. Traditionally, decision-making by managers that is reasoned and step-by-step has been considered
preferable to intuitive decision-making. However, a recent study found that top managers used
intuition significantly more than did most middle or lower-level managers. This confirms the alternative
view that intuition is actually more effective than careful, methodical reasoning.

The conclusion above is based on which of the following assumptions ?


(a) Methodical, step-by-step reasoning is inappropriate for making many real-life management
decisions.
(b) Top managers have the ability to use either intuitive reasoning or methodical, step-by-step
reasoning in making decisions.
(c) The decisions made by middle and lower-level managers can be made as easily by using
methodical reasoning as by using intuitive reasoning.
(d) Top managers are more effective at decision-making than middle or lower-level managers.

115. Banker: By transferring income to a retirement account at our bank, people can save money by
delaying payment of taxes.
Accountant: That plan won't actually save money because the taxes will have to be paid sometime
in the future when the money is withdrawn.
Which one of the following best explains the conflict between the Banker and the Accountant ?
(a) The Banker is primarily concerned with recruiting new customers for the bank but the Accountant
is not.
(b) The Accountant misunderstands the application of the tax laws.
(c) The Banker and the Accountant disagree on the application of the term "save".
(d) Retirement accounts are nothing more than a tax shelter, which the Parliament intends to cut
out of next year's tax amendment.

Direction (Qs. 116 - 120) : Read the following information carefully and answer the questions given below.
i. Five friends A, B, C, D and E travelled to five different cities of Chennai, Kolkata, Delhi, Bengaluru
and Hyderabad by different modes of transport viz. bus, train, aeroplane, car and boat from Mumbai.
ii. The person who travelled to Delhi did not travel by boat.
iii. C went to Bengaluru by car and B went to Kolkata by aeroplane.
iv. D travelled by boat whereas E travelled by train.
v. Mumbai is not connected by bus to Delhi and Chennai.

116. Which of the following combinations of person and mode is not correct ?
(a) A - Bus (b) D - Boat (c) C - Car (d) E - Aeroplane

117. Which of the following combination is true for A ?


(a) Delhi - Bus (b) Chennai - Bus (c) Hyderabad - Bus (d) Hyderabad - Car

118. Which of the following combinations of place and mode is not correct ?
(a) Hyderabad - Train (b) Hyderabad - Bus (c) Chennai - Boat (d) Delhi - Train

119. The person travelling to Hyderabad went by which of the following modes ?
(a) Train (b) Bus (c) Boat (d) None of these

120. Who among the following travelled to Delhi ?


(a) D (b) A (c) E (d) None of these

Previous Years
CLAT & AILET Papers Page 305
Direction (Qs. 121 - 125) : Read the following information carefully and answer the questions given below.
All the streets of a city are either perpendicular or parallel to one another. The streets are all straight.
Streets N, O, P, Q and R are parallel to one another. Streets S, T, U, V, W, X and Y are horizontally parallel
to one another,
i. Street N is 1 km east of Street O.
ii. Street O is % km west of Street p.
iii. Street Q is 1 km west of Street R.
iv. Street S is 1/2 km south of Street T.
v. Street U is 1 km north of Street V.
vi. Street W is 1/2 km north of Street X.
vii. Street W is 1 km south of Street Y.
121. lf W is parallel to U and W is 1/2 km south of V and 1 km north of T, then which two streets would
be 1 & 1/2 km apart ?
(a) U and W (b) V and S (c) V and T (d) W and V

122. Which of the following possibilities would make two streets coincide ?
(a) X is 1/2 km north of U (b) P is 1 km west of E
(c) O is 1/2 km east of N (d) R is 1/2 km east of O

123. Street R is between O and P, then the distance between p and e is


(a) 1/2 km (b) 1 km (c) 1.5 km (d) 1.25 km

124. R is between O and P, then which of the following is false ?


(a) Q is 1.75 km west of N (b) P is less than 1 km km from Q
(c) R is less than 1 km km from N (d) Q is less than 1 km km from O

125. Which of the following is necessarily true ?


(a) R and O intersect (b) Q is 2 km west of O
(c) Y is 1.5 km north of X (d) O is at least 2 km west of N

Direction (Qs. 126-129) : Study the following information carefully and answer the questions given below.
A family consists of six members H, l, J, K, L and M. There are two married couples. I is a doctor and father
of L. M is grandfather of J and is a contractor. K is grandmother of L and is a housewife. There is one doctor,
one contractor, one nurse, one housewife and two students in the family.
126. What is the profession of H ?
(a) Doctor (b) Student (c) Doctor or Nurse (d) Nurse

127. Who is the husband of H ?


(a) l (b) M . (c) J (d) None of these

128. Which of the following are two married couples ?


(a) LK, JM (b) MK, lH (c) MK, JH (d) MK, IL

129. Who is the sister of L ?


(a) J (b) H (c) K (d) Information insufficient

130. lf Saturday falls four days after today, which is 6th January, on what day did the 1st of December of
previous year fall ?
(a) Sunday (b) Monday (c) Tuesday (d) Wednesday

Previous Years
Page 306 CLAT & AILET Papers
131. At what angle the hands of a clock are inclined when it is half past 8 ?
(a) 70° (b) 80° (c) 75° (d) 60°

Direction (Qs. 132 - 135): Find the missing numbers/letters in following series.
132. 3, 20, 63, 144, 275, ?
(a) 468 (b) 461 (c) 467 (d) 469

133. 113, ?,164, 215, 283, 368


(a) 132 (b) 130 (c) 129 (d) 128

134. AYBZC, DWEXF, GUHVI, ?, MQNRO


(a) LSJTL (b) JSKLT (c) JSKTL (d) LTSKY

135. 21 A, X2D, V6G, T21J, R88M, ?


(a) P440P (b) N2670S (c) N2676S (d) P445P

Directions (Qs. 136 -140): A fact situation and result is presented. Numbered statements follow the result.
Each statement has to be separately evaluated in relation to the fact-situation and result' Evaluate these
statements with the following sequences of decisions in the order of a, b, c and d. The first of these that
you cannot eliminate is the correct answer.
(a) lf the statement is inconsistent with, or contradicts, the fact situation, the result, or both together. lf
so Choose (a).
(b) lf the statement present a possible adequate explanation of the result.
(c) lf the statement is deducible from something in the fact-situation, or the result, or both together.
(d) lf the statement either supports or weakens a possible explanation of the result.
Situation : Major X, an able officer in the Kapistan Army, failed to receive a promotion for eight years.
Then he had been reassigned to a military supply depot in Khawalpindi, despite his university
training in engineering and electronics, and his remarkably high performance ratings from his
commanding officers. X had never been an active member of any political party as a youth nor
as an adult, yet neither had he given the party or his superiors any cause to doubt his absolute
loyalty to Kapistan. X's brother-in-law had been a diplomat in the Kapistan government until
his death in 1971 in a plane crash on Koviet soil while he was working in the Kapistan Embassy
in Kosco. X had always assumed that the mishap was indeed an accident, until his friend Y,
a middle-level officer in the lnterior Ministry, broadly hinted that the plane crash had been an
act of sabotage. Soon after the talk with Y, X visited his sister, the diplomat's widow, in her
Kosco apartment. During the visit she asked X several questions that struck him as strange
and inappropriate. As he was leaving her apartment, she asked X to wear her late husband's
scarf and to return to his hotel by way of a certain park. Bewildered, but not wishing to offend
his sister, X obeyed her odd instructions.
Result: Two months later, X received a promotion and was made Commander of the Kapistan missile
division in Kahore.
136. X's sister had arranged for her husband's fatal accident.

137. X's sister was an operative in the Kapistan Secret Police, and her husband had been acting as a spy
before his death.

138. X's promotion and reassignment to the Kosco Army Headquarters came as a result of Y's
recommendations.
Previous Years
CLAT & AILET Papers Page 307
139. For X, a missile base is a more highly prized assignment than a supply depot.

140. X's failure to win a promotion for eight years was the result of the clerical error in his records.

SECTION - E : MATHEMATICS

141. A die is rolled twice. What is the probability that sum of the numbers on the two faces is 5 ?
(a) 5/12 (b) 1/9 (c) 1/6 (d) 5/36

142. Twenty five workers were employed to complete a compound wall in 12 days. Five workers left after
working for 4 days. The remaining 20 workers completed the work. In how many days the total job
was completed ?
(a) 15 days (b) 16 days (c) 14 days (d) 18 days

143. In a garden, there are B rows and 10 columns of papaya trees. The distance between the two trees
is 2 metres and a distance of one metre is left from all sides of the boundary of the garden. The
length of the garden is
(a) 24 metres (b) 14 metres (c) 20 metres (d) 18 metres

144. A man can row 14 km/h in still water. ln the stream flowing with the speed of 10 km/h he takes 4
hours to move with the stream and come back. Find the distance he rowed the boat.
(a) 11.71 km (b) 13.71 km (c) 14.71 km (d) 12.71 km

145. What will be the difference in simple and compound interest on Rs. 2, 000 after three years at the
rate of 10 percent per annum ?
(a) Rs. 60 (b) Rs. 42 (c) Rs. 62 (d) Rs. 40

146. A man covers a certain distance between his house and office on a Scooter. Having an average
speed of 30 km/hr, he reaches office late by 10 minutes. However, with a speed of 40 km/hr, he
reaches his office 5 minutes earlier. The distance between his house and office is
(a) 30 km (b) 10km (c) 20 km (d) 40 km

147. A box contains Rs, 56 in the form of coins of one rupee, 50 paisa and 25 paisa' The number of 50
paisa coins is double the number of 25 paisa coins and four times the number of one rupee coins.
The number of 50 paisa coin in the box is
(a) 14 (b) 16 (c) 32 (d) 64

148. The price of 7 bananas is equal to the cost of 3 kiwis The price of 2 kiwis is equal to the cost of 1
banana and 5 chikoos. lf Rambo has just enough money to buy 30 chikoos, then how many bananas
Rambo could buy with the same amount ?
(a) 22 (b) 20 (c) 2s (d) 11

149. ln a certain class, 72% of the students prefer cold coffee and 44% prefer fruit juice' lf each of them
prefers cold coffee or fruit juice and 48 likes both, the total number of students in the class is
(a) 240 (b) 200 (c) 300 (d) 250

150. The average price of 10 pens is Rs. 12 while the average price of 8 of these pens is Rs. 11.75. Of the
remaining two pens, if the price of one pen is 60% more than the price of the other, what is the price
of each of these two pens ?
(a) Rs. 12, Rs. 14 (b) Rs. 5, Rs. 7.50 (c) Rs. 8, Rs. 12 (d) Rs. 10, Rs. 16

Previous Years
Page 308 CLAT & AILET Papers
CLAT and AILET
Solutions
CLAT Solutions 2008

1. c The answer might appear to one as (b), but since her 12. b, d Both the options show the same spelling, hence both
grandmother persuaded her to develop interest in the of them can be considered to be the right answer.
nature, and not to study zoology, the right answer ‘negligence’ means ‘the quality of being negligent’.
is (c).
13. c The right spelling is ‘grievance’, which means ‘a wrong
2. a Refer to the line, “I didn’t get my first camera considered as grounds for complaint’.
until...needed a method of recording the sea
creatures...” This clearly mentions that the author got 14. b The correct spelling is ‘hierarchical’, which means
her first camera to record what she had seen at the ‘belonging to a hierarchy’.
sea.
15. d The correct spelling is ‘guarantor’, which means ‘a
3. d Refer to the line, “I became keen very quickly...didn’t person or group that guarantees’.
have much money...more black and white photography
than color...” Thus, option (d) clearly follows from this 16. d The right answer is (d) because the sentence refers
line. to a place where there is noise.

4. b The author talks about how some photographers pick 17. c The consistency in the sentence can be maintained
up sea creatures from the shores to picture them in through option (c), as the sentence indicates the
beautiful ponds. They do so to make their pictures comparative degree of adjective.
attractive but in the process change the creature’s
natural surroundings drastically. The author finds 18. b The sentence is talking of Ben in Active Voice, to refer
herself different from other photographers in this to the same idea in continuation, the next following
respect. Thus, option (b) is the correct choice. sentence should be ‘he walks to work...’

5. a The ‘them’ in the line “In so doing you’re actually...make 19. d The sentence is in simple past tense and hence the
them prettier” refers to sea creatures which are being answer is (d).
captured by the photographers.
20. b The correct usage of interrogative simple past tense
6. d Not thinking about the animals in the first place is what is ‘why did you have to...?’
the author means by ignorance in people’s behavior.
This can be arrived at from the first few lines of the 21. d ‘turned out’ is the correct phrase to be used in the
fourth paragraph. sentence.

7. b The author has stated in paragraph four that it has 22. c ‘put off’ is the right phrase for the sentence.
become difficult for the wildlife photographer to find
and click wild animals. This is because of animals 23. c The damage was caused ‘to’ the other car.
getting scared of people rushing towards them. Thus,
option (b) is the correct choice. 24. a One objects ‘to’ something or some clause.

8. d The passage clearly states that photography can 25. b Restrained – to hold back
“educate people about what is out there and what Rescinded – to make void or annul
needs conserving”. Thus, option (d) is an obvious Compelled – to force or drive to a course of action
choice. Conferred – to consult together
‘Rescinded’ is the correct choice.
9. a The last paragraph helps us to lead to this answer, as
the author says that she is keeping herself busy 26. b Sentence (iv) introduces the topic followed by (i),
through writing. which talks about revolutionizing, followed by (iii)
talking about the changes and benefits and then (ii)
10. b The tone of the author is sensitive throughout the referring to the change expected in future.
passage as she is concerned about the condition of
the wild life. 27. c The paragraph talks of letters, with sentence (iv)
introducing the topic, (iii) continuing the idea followed
11. c The right spelling is ‘acquaintance’, which means ‘a by (ii), and (i) being the last line talking of modern
person known to one, but usually not a close friend’. postal services.

Previous Years
CLAT & AILET Papers Page 1
28. d The paragraph is about the complications faced while 43. a A balance of payment surplus is the desirable positive
travelling. (iii) introduces the topic, followed by (iv). state of account in a nation’s balance of payments in
And then (ii) and (i) elaborating the complications. which payments made by the country are less than
payments received by the country. This is also termed
29. a The paragraph begins with the quote of the author, a favorable balance of payments. It’s considered
followed by her name in (iv), and then (i) and (iii) favorable because more currency is flowing into the
describing her quote. country than is flowing out.

30. d (ii) sentence introduces the idea of the paragraph. 44. b Kosovo declared itself as Independent in the year
Sentence (iv) talks about a speech disorder, (ii) 2008.
describes it further and (i) concludes the paragraph.
45. b Former Prime Minister Morarji Desai. Morarji Desai
31. a Set down – to put in a particular place presented the maximum number of times. He held the
Set aside – for a particular purpose finance portfolio from 1959-64, 1967-70 and between
Set about – to start 1977 and 1979. He is seconded by Current President
Set forth – to give an account of of India Mr. Pranab Mukherjee who has presented it
seven times.
32. d Go over – to repeat or review
Go about – to perform 46. a Vijay Kelkar was the Chairman of 13 the Finance
Go after – to strive for Commission constituted in the year 2007.
Go at - attack
47. c The Hyde act of the United States had a extraterritorial
33. b Come forward – to offer one’s service jurisdiction so as to effect the sovereignty of India in
Come across – to find or encounter matters of nuclear relations with United States.
Come around – to recover consciousness Parliament is only recognized by the constitution of
Come upon – to meet unexpectedly India as the sovereign not any foreign or domestic
institution.
34. d Stand in – to act as a stand-in
Stand down – to withdraw 48. a Vijay Mallaya the Indian business tycoon has recently
Stand for – to represent been in spotlight because of grave financial issues
Stand out – to project surrounded his company kingfisher airlines.

35. b Come away – to leave in a certain condition 49. c Chenab River is located in State of Jammu and Kashmir.
Come through – to finish successfully
Come up – to arise 50. c Saudi Arabia ‘s king “Abdullah (Abdullah bin Abdulaziz
Come down – to lose wealth or rank al Saud) ” is the monarch of Saudi Arabia.

36. c The correct meaning is ‘at first view’. 51. b Kishore Biyani is the CEO of future group and managing
director of Pantaloon Retail.
37. a The correct meaning is ‘without setting a fixed day’.
52. c Tata Consultancy Services has retained its position at
38. c The correct meaning is ‘in good faith’. the top and in terms of revenue in the present year
has widened the revenue gap with its business rival
39. b The correct meaning is ‘present condition’. Infosys.

40. b The correct meaning is ‘as per law’. 53. c The term “Taikonaut” is used for an astronaut of
Chinese Ethnicity. Similarly the term “Cosmonaut” is
41. c Creamy Layer is excluded on the basis of the Mandal for an astronaut of Russian descent.
commission‘s recommendations. ‘Creamy layer’ is
interpreted by the court as ‘socially-advanced Persons 54. c Setve Balmer ‘s personal wealth in the present year
and hence were excluded from the other eligible has been estimated at $15.7 billion, ranking number
castes for availing reservation Benefits. 19 on the Forbes 400.

42. c A “Hedge fund” term is used to describe a very generic 55. c The term “Gross National Happiness” was coined in
form of funds, which can undertake a wider range of 1972 by Bhutan’s fourth Dragon King, Jigme Singye
investment and trading activities. The primary aim of Wangchuck is intended to comprehend not only
hedge funds is to minimize the risk of financial economic aspect of growth but psychological and
transaction. social progress in an attempt to be more holistic.

Previous Years
Page 2 CLAT & AILET Papers
56. c Singapore’s government is one of the “cleanest” in the between the Central Ministers and the State
world largely because its top ministers are paid Governments. The Prime Minister of India heads it.
exorbitant salaries (in excess of $1-million) annually.
69. b V. K. R. V. Rao (Vijayendra Kasturi Ranga Varadaraja
57. c The Impasse in the year 2008 in Doha Round of Rao) was a prominent Indian economist, politician,
Negotiations was centered around agricultural professor and educator. The prize instituted on his
subsidies provided by developed countries. name i.e. VKRV Prize is given for the development in
the field of social science research in India.
58. b An equinox occurs twice a year (around 20 March
and 22 September), when the tilt of the Earth’s axis is 70. b Arabian Peninsula is the largest peninsula in the world.
inclined neither away from nor towards the Sun, the
center of the Sun being in the same plane as the 71. b Prasanta Chandra Mahalanobis founded Indian
Earth’s equator. Statistician Institute.

59. b Pascal Lamy is the Current Director-General of the 72. a The Term ‘Social Forestry’ first used in 1976 by The
World Trade Organization. National Commission on Agriculture, Government of
India. It was then that India embarked upon a social
60. c Capital Account Convertibility (CAC) also known as forestry project with the aim of taking the pressure off
Capital Asset Liberation is a feature of a nation’s currently existing forests by planting trees on all
financial regime that centers on the ability to conduct unused and fallow land.
transactions of local financial assets into foreign
financial assets freely and at country determined 73. a The Great Barrier Reef is the World ‘s Largest Coral
exchange rates. Reserve Bank of India coined the Reef System. The reef is located in the Coral Sea, off
term. The Tarapore Committee has given its report on the coast of Queensland in northeast Australia. It is
CAC. the world’s biggest single structure made by living
organisms.
61. d Koto protocol is the United Nations Framework
Convention on Climate Change (UFCCC) aimed at 74. a A nautical mile is equal to 1825 metres.
reducing Global Warming. The Protocol was initially
adopted on 11 December 1997 in Kyoto, Japan, and 75. b Geomorphology (Geo= earth, morph=form,
entered into force on 16 February 2005. Logos=study) is the scientific study of landforms and
the processes that shape them. It aims to understand
62. b Carbon Credits which is an prominent concept of why landscapes look the way they do, to understand
environmental economics are allotted under the Kyoto landform history and dynamics.
Protocol and are tradable certificates representing the
right to emit one tonne of carbon dioxide or the mass 76. b Norway is known as the Land of Midnight Sun
of another greenhouse gas with a carbon dioxide
equivalent (tCO2e) equivalent to one tonne of carbon 77. c Padmasambava was the monk who spread Buddhism
dioxide in Tibet and Far-East regions.

63. b Dumping is an economic offence and is prohibited by 78. c TRIPS stand for Trade Related aspects of Intellectual
World Trade Organization ‘s Anti-Dumping treaties. Property Rights. In the year 2005 India brought The
Patents (Amendment) Act 2005 to make it compatible
64. b Article 32 of the Constitution of India empowers with TRIPS.
Supreme Court of India to issue five kinds of Writs for
enforcement of fundamental rights viz. Prohibition, 79. b Carbon dating method is used to determine the age of
Habeas Corpus, Certiorari, Mandamus, Quo Warranto. Fossils

65. c Bankimchandra Chatterjee composed Vande Mataram. 80. c E. Sreedharan is the Managing Director of Delhi
Metropolitan Railway Corporation.
66. a Greenwich Mean Time is an absolute time reference
and does n’t change with seasons. It was established 81. b 18 carat gold signifies 18 parts of gold and 6 parts of
in 1884 at the International Meridian Conference, when other metal
it was decided to place the Prime Meridian at
Greenwich, England. 82. a Bali Road map adopted in December 2007 provided
for fixation of greenhose gas emission limits.
67. c Article 1 of Indian Constitution defines India as an
‘Union of States’. 83. a Enriching uranium increases the proportion of uranium
atoms that can be “split” by fission to release energy
68. c National Development Council (NDC) was setup in 1951 (usually in the form of heat) that can be used to
with a view to bringing about a coordination of plans produce electricity.

Previous Years
CLAT & AILET Papers Page 3
84. b Homi Jehangir Bhabha, is also known as father of 99. c China in the year 2008 was the second biggest
Indian nuclear program. greenhouse gas emitter (after the USA) in the world.

85. b Salwa Judam (meaning Purification Hunt) refers to the 100. b J. Peters is the author of the management principle
militant movement in state of chhattisgarh India which viz. In a hierarchy, every employee tends to rise to his
is aimed at countering the naxalite violence in that level of incompetence.
region.
101. b The Uruguay Round, which was the 8th round of
86. b Raman Magsaysay award is also considered as “Asia Multilateral trade negotiations (MTN) conducted within
‘s Nobel Prize” the framework of the General Agreement on Tariffs
and Trade (GATT), spanning from 1986-1994 and
87. a Lula de Silva won the Raman Magasaysay award in embracing 123 countries as “contracting parties”. The
2007. Round transformed the GATT into the World Trade
Organization.
88. b
102. c The “wailing wall” is associated with Jews.
89. b Affirmative Action refers to the governmental policies,
which includes factors like “race, color, religion, gender, 103. c The Ex-President Abul Kalam Azad who was also
sexual orientation, or national origin”. In India Education Minister got Bharat Ratna also.
reservation system represents affirmative action.
104. b Ozone Layer protects Earth from ultraviolet radiation.
90. c Special Economic Zones in India were setup with the
aim of promotion of exports. The Special Economic 105. b Nile is the world’s largest river.
Zone Act 2005 regulates functioning of such zones in
India. 106. c Tsunami is caused by underwater volcanic activity.

91. c Sunita Williams has again ventured into another space 107. a Chipko Movement followed gandhian priciples of non-
mission and this time she is accompanied with Russian violent resistance. The modern Chipko movement
cosmonaut Yury Malenchenko and Japan’s Akihito started in the early 1970s in the Garhwal Himalayas of
Hoshide. Uttarakhand, Then in Uttar Pradesh with growing
awareness towards rapid deforestation.
92. c Kevin Rudd led the country in atoning for the past
wrongs. 108. a Magadhan Empire was the first Great Indian Empire.

93. b In My Experiments with Truth, Gandhiji expounded his 109. b W.C. Banerji was the first President of Indian National
economic ideas. Congress.

94. c Bio fuels began to be in controversy because it was 110. a Jahangir gave permission to establish East India
found out that they reduce food cultivation. Company in India.

95. c Evergreening is the strategy used by the 111. c Mohammad Iqbal conceptualized the idea of Pakistan.
pharmaceutical companies to reap the benefits of
patent beyond the normal span allotted by the law of 112. c The Khilafat movement (1919–1924) was a pan-
20 years. These patent term extension strategies entail Islamic, political protest campaign launched by Muslims
using the intricacies of patent prosecution procedures. in British India to influence the British government and
The Indian Patents Act is rather rigid as to the time to protect the Ottoman Empire during the aftermath of
lines for priority, patent term and patentable subject World War.
matters. Hence, ‘Evergreening’ in India is not a serious
concern. 113. c The Nehru Report (1928) was a memorandum
containing a proposed new dominion constitution for
96. a By signing Poona Pact in the year 1932 with Gandhiji, India.
Ambedkar gave up his demand for separate
electorates. 114. b M.N. Roy

97. c The maximum amount of foreign exchange is earned 115. d Jnapath Award is given for Literature It is presented
by India through export of jams and Jewelleries. by the trust Bharatiya Jnanpith.

98. a Sunita William made a record for spending of 195 days 116. a Dorris Lessing won Nobel Prize for Literature in the
in 2008. year 2007.

Previous Years
Page 4 CLAT & AILET Papers
117. d Plea Bargaining was dealt by the 144th report of Law 132. b International Court of Justice is situated at The Hague
Commission of India and was included through in the Netherlands. Justice Dalveer Bhandari from India
amendment in 2005 which came force in 2006. has been appointed to ICJ recently.

118. b Arbitrator is the term used to describe the person 133. c The Liberhan Commission which received repeated
appointed by two parties to settle a dispute. extensions has been inquiring into demolition of Babri
Masjid.
119. b In Maneka Gandhi case, it was held that right to travel
outside India is included under Article 21 of the 134. a The Competition Act has been enacted in the year
Constitution of India. 2002. The Competition Commission ‘s chairperson is
presently Mr. Ashok Chawla.
120. b Genetically modified seeds are controversial because
they have adverse impact on flora around. 135. b JJustice Ramaswami is the only judge against whom
a motion of impeachment was introduced into
121. a The right to legal aid is recognized under article 21 of Parliament
the Constitution of India. (Hussainara Khantoon Case).
136. b Malimath Committee suggested reforms on Criminal
122. b The members of Constituent Assembly who framed Justice System.
the Constitution were indirectly elected.
137. b Leila Seth was the first Woman Chief Justice of High
123. b B. R. Ambedkar was the Chairman of the Drafting Court in India.
Committee in the Constituent Assembly.
138. a Legal Aid scheme was first introduced by Justice P.N.
124. a Parliament is the Sovereign body of India. The Bhagwati. Legal Services Authority Act 1987 was
Legislative powers solely vest in the legislature unless introduced keeping in mind Article 39A of the
it falls under exceptions mentioned in Indian Constitution of India which provides that “State shall
Constitution. secure that the operation of the legal system promotes
justice on a basis of equal opportunity, and shall in
125. a It is a constitutional requirement that the Parliament particular, provide free legal aid, by suitable legislation
shall meet at least thrice in a year. or schemes or in any other way, to ensure that
opportunities for securing justice are not denied to
126. b The Governor of a State can be removed by The any citizen by reason of economic or other disability.
President of India.
139. c In 2008 Nepal elections were globally significant
127. b The Sovereignty under the Constitution belongs to the because a militant movement joined the mainstream.
people.
140. a Fidel Castro defied the United States of America.
128. b The Supreme Court upheld Mandal Commission Report
in Indra Sawhney v. Union of India. 141. c Let Raju earns Rs. x in each of the other eleven
months.
129. c The Indian Constitution does not recognize property Then, Raju’s earning in March = Rs. 2x
right as a fundamental right. In the year 1977, the 44th Raju’s annual earning = Rs. (11x + 2x) = Rs. 13x
amendment eliminated the right to acquire, hold and
2x 2
dispose of property as a fundamental right. However, ∴ Required fraction = =
in another part of the Constitution, Article 300 (A) was 13x 13
inserted to affirm that no person shall be deprived of
his property save by authority of law. 142. b Let the new S.P. be x.
C.P. × (100 − loss%)
130. a Justice B.N.Srikrishna was the Chairman of Sixth Pay Then, 1st S.P. =
100
Commission.
1140 × 100
131. d Right of Children to Free and Compulsory Education ⇒ = C.P.
(100 − 5%)
Act, 2009 has been notified recently. The apex court
upheld the constitutional validity of the Act and directed C.P. × (100 + Gain%)
all schools, including privately run schools, irrespective 2nd S.P. =
100
of the board they are affiliated to, to admit from this
academic year (2012-13) at least 25% students from 1140 × (105)
socially and economically backward families. These ⇒x=
95
students will be guaranteed free education from class
I till they reach the age of 14. ⇒ New S.P. = Rs. 1260.

Previous Years
CLAT & AILET Papers Page 5
143. a Mixture of 40 litres of milk and water contains 10% of Short cut:
water. Using successive percentage change method:
⇒ Milk = 36 litres and water = 4 litres 15 × 15
Now let quantity of water to be added be x litres. = −15 + 15 − = −2.25%.
100
New mixture contains 20% of water
4+x
∴ × 100 = 20  2
40 + x 149. d Quantity of milk =  60 ×  litres = 40 litres
 3
⇒ 20 + 5x = 40 + x ⇒ x = 5 litres.
Quantity of water = (60 – 40) = 20 litres
New ratio = 1 : 2
 5  Let the quantity of water to be added be x litres. Then,
144. c Speed of the train =  54 ×  m / sec = 15 m / sec
 18  40
milk : water =
Time taken by the train to pass a bridge = 20 sec (20 + x)
Let the length of the bridge be ‘x’ m.
40 1
100 + x ∴ = ⇒ 20 +x = 80
∴ = 15 20 + x 2
20
⇒ x = 60
⇒ 100 +x = 300 ⇒ x = 200 m
Quantity of water to be further added = 60 litres.
145. b According to the given condition, we have
150. c Let the length of the piece be ‘x’ m.
Mohan’s age - Sameer’s age = Sameer’s age – Arun’s
age 70
Then, cost of 1 m of piece = Rs.
∴ Mohan’s age + Arun’s age = 2 Sameer’s age x
Also, Mohan’s age +Arun’s age = 48
⇒ 2 Sameer’s age = 48  70 
∴ (x + 4)  − 2  = 70
∴ Sameer’s age = 24 years.  x 
⇒ (x + 4) (70 – 2x) = 70x
1 1 1 4
146. c Net part filled in 1 hour = + − = ⇒ 70x − 2x 2 + 280 − 8x = 70x
2 3 6 6
6 1 ⇒ 2x 2 + 8x − 280 = 0
∴ The tank will be full in hours, i.e. 1 hours.
4 2 ⇒ (x + 14)(x − 10) = 0
⇒ x = 10 m
147. c Let the three numbers be a, b and c.
b 151. b Going by the information, 10% of the students i.e. 5
Now, a = and a = 2c. students scored in the 90%-95% range. 3/5th of the
3
remaining 45 students, i.e. 27 students scored in the
a+b+c range of 75% - 90%. The remaining 18 students scored
Also, = 27 .
3 below 75%. Option A is not possible because a
maximum of 32 students who have scored 75% and
⇒ a + b + c = 81
above can opt for Physics. This is maximum of 64%.
a Option C is also not possible because at best 32
∴ a + 3a + = 81 students can opt for Physics and each of them has to
2
opt for Mathematics as per restrictions given. Even if
⇒ a = 18, b = 54, c = 9 no student who scores below 75% opts for
∴ Largest of the three numbers is 54. Mathematics, the number of students taking Physics
will still equal the number of students taking
148. b Let the original length of the rectangle be x and original Mathematics. D is also not possible. To be eligible to
breadth be y. study Physics, a student has to score above 75%
∴ Original area = xy while to study Mathematics, a student has to score
Now, new length = 1.15x above 70%. So there would be some students scoring
New breadth = 0.85y between 70% and 75% who would be eligible to study
∴ New area = 0.9775xy Mathematics but not Physics. B is the only possible
0.9775xy − xy option. If 35% of those who scored below 75% scored
Percentage change = × 100 = −2.25% , between 75% and 70%, then they can opt for
xy
Mathematics and Astrophysics.
where negative sign shows a decrease in the area of
rectangle so formed.

Previous Years
Page 6 CLAT & AILET Papers
152. d Option A violates restriction A. Option B violates statement about human dishonesty not clearly
restriction B. Option C violates restriction C. Only option supported by the passage.
D does not violate any restriction.
158. b This option establishes the link between incorruptibility
153 a Option B and D are straightaway discounted. Option B and immortality and thus completes the argument.
includes C,D and F and violates restriction C. Option D
includes B but does not include E. Therefore, it violates 159. d If what is not matter is also vulnerable to motion then
restriction B. Option C is not a certainty but a possibility the human soul will be subject to dissolution. This
because in option C while C is included. A is not included. would seriously affect the argument.
This does not fulfill restriction A at all. Option A meets
all the restrictions and is therefore a certainty. 160. b The debate talks about the merits and demerits of
capital punishment as a form of punishment. Those
154. c Option A is not acceptable because it includes C and E who oppose it talk of man’s rights and privileges. The
but not D. Therefore, it violates restriction C. Option B defenders talk of punishing the criminal. However,
is not acceptable because it includes both A and D these are reason that both the parties use to put forth
which violates restriction A. Option D is not acceptable their opinion regarding capital punishment. Hence,
because it includes C and D but not E which again option (a) and (c) themselves are not the focus of the
violates restriction C. Only option C meets all the debate. Option (d) is irrelevant. Option (b) is the correct
restrictions and is therefore an acceptable choice.
combination.
161. a The original premise was the ‘Slow Rotation of Venus’,
155. b The argument bases itself on the premise that political which was changed by the scientist on the basis of
turmoil is caused by violence and the government his later conclusions. Since this is the only fact given
policies. It concludes that a dictatorial government in the passage and the rest are all assumptions and
which rules strictly and imposes stringent regulations observations, (a) is the apt answer.
can crush political turmoil. Option A does not weaken
the argument much because it is not clear from the 162. b The passage clearly says ‘He first theoretically derived
passage that existing legislations would be effective the relation between distances and time…”. Therefore
in curbing political turmoil. C talks about educating theoretical analysis characterised Galileo’s method.
people which may weaken the argument somewhat
but still does not address whether a strict government 163. b The correct option is (b). The method used by Holmes
would be required or not. Option D also talks about the is clearly one of ‘Experiment’. This is not mere
effectiveness of dialogue and that too in a certain observation, because he experimented by dissolving
type of society i.e.’free’ society. Only option B if true the pills in water and then in milk.
would directly weaken the need for a strict government
which rules dictatorially. 164. a An analogy is drawn with the frying of pancakes in
order to determine the formation of lunar craters.
156. b Only option B, which mentions the economic policy of
‘press from top’ broadly strengthens the case for a 165. d The passage says ‘Equal weights at equal distances
dictatorial government which imposes strict rules from are in equilibrium.
above. Option A talks about only one factor (espionage
activities by enemy nations) that affects political turmoil 166. c Since equilibrium is determined by both weight and
and does not strengthen the conclusion that much. distance and the passage clearly mentions that equal
Option C which talks about ‘corrupt establishment’ also weights at unequal distances cause disequilibrium, it
does not directly strengthen the case for a touch is highly probable that inequal weights at equal
government. Option d makes a far-fetched statement distances would be in disequilibrium. We cannot say it
not related to the issue at all. is ‘true’ because this case is not explicitly mentioned in
the passage.
157. b Only if an austere life is encouraged can one expect
people to stop indulging in greed. In such a case one 167. a Since the passage explicitly states that equal weights
can expect both the state to be free of corruption and at inequal distances are in disequilibrium, then the
the rich to stop exploiting the poor. This is the least statement ‘inequal weights at inequal distances’ can
dubious of all conclusions that can emerge out of the be considered to be true.
para. Option A which talks about abolishing production
is unrealistic. C does not address the issue of corruption 168. a ‘Three dominant talls to one recessive short’ indicates
at the state level. Also it does not address the impact that the distribution of dominant and recessive
that heavy taxation may have on the rich. Furthermore characteristics is systematic.
the passage does not mention taxation clearly as the
only means of control the state has over industrial and
agricultural production.D also makes a very broad

Previous Years
CLAT & AILET Papers Page 7
169. b Einstein’s remark with reference to playing dice (a 176. b If a widow is remarried when the ‘question of
game of probability) refers to his reservations about a succession’ is opened then she cannot claim property.
probabilistic interpretation of the behaviour of quantum
objects while Bohr has expresses an opposing point 177. c Even death of a person would not be an offence if it is
of view suggesting that this could be possible. proved it was for self-defence.

170. b Since the issue with an environment friendly public 178. d The principle 2 talks about consumers, which includes
transport system is the impact on cash flow to the both Ram and Tom. Now by applying principle 4 to the
treasury and better job opportunities, then developing facts of the case it is clear that it is the manufacturer
alternatives that would reduce the adverse impact on “Coca-Cola Company” who is responsible for the
these would be the opinion of the author of this quality of the products, which contained nitric acid.
passage. The seller, being unaware of the quality of the product
will definitely not be liable for selling unmarketable
171. a The opening sentence of the paragraph raises the goods.
question about the effectiveness of democratic
government. It goes on to questions Lincoln’s view on 179. a It is clear from the facts that Amar has committed a
the same by mentioning how he fails to clarify the real wrong while acting on behalf of Somu. This directly
attitude that leads a candidate to seek election. The falls within the purview of principle 2, hence only
paragraph also quotes the view of Socrates that no Amar will be liable.
honest man willingly seeks the position of a ruler. The facts do not state as to whether or not Amar had
Therefore option A which more or less restates the disclosed before Gulab that he is acting under the
question at the opening of the para follows strictly authorisation of Somu, hence principle 3 is not attracted
from the passage. The passage does not give enough in this case.
information to verify any of the other options completely.
180. c Crucial here is Principle 2- that although you can use
172. d According to Lincoln, no contestant to a democratic the water flowing below your land, you cannot have
election takes part in it unwillingly. Rather he is driven ownership over it. This means that you cannot deprive
by his own passions and motives. Socrates has a others of their right to the water even if it is flowing
contrasting view on this and suggests that no honest under the land owned by you. The question of
man seeks democratic election willingly and does so demolishing the tank does not arise due to 2 reasons:
only when he fears that he may otherwise serve one, because of principle 3, any construction on your
under an inferior man. Thus option D which talks about land belongs to you, and second, the neighbours had
difference in the mindset of respective men who seek given him an option to share the water instead of
election is the right answer choice. being forced to demolish it.

173. a The last part of the passage suggests that it may be 181. c Upon reading the facts, it is clear that principle 2 applies
wrong to kill any living being in the interest of hygiene. in this case. Nagamma must exercise reasonable care
Option A which suggests that disease-spreading to find out whether a person is actually acting in the
bacteria should be eradicated in the interest of health course of employment.
and cleanliness therefore runs counter to the spirit of
the passage. 182. c It is clear from the facts that the arrangement between
Rama Bhai and Keshav was private in nature, based
174. b According to the passage if reservation holds true for upon the mutual understanding between two relatives.
those opting for the teaching profession then this may The Course of employment has to be decided in view
see the appointment of substandard teachers who of facts and circumstances. It is important to note that
have gained their qualifications not on the basis of Keshav was already out of the service when the
merit but on the basis of reservation. If such teachers cause of action took place. The transaction between
fail to impart quality education then every student Rama Bhai and Keshav had no involvement with that
including those benefiting from the reservation system of the Syndicate bank per se.
will fail to derive the benefit of quality education.
Therefore at least in the field of ensuring quality 183. c From the facts it is clear that the cricket fan volunteered
teachers reservation will be self-defeating. to watch the match from a place where the cricket
organisers failed to foresee any danger of injury being
175. d Only if those who are meritorious among the caused to the spectators. Hence by applying principle
downtrodden groups are made teachers can we 1 and 2 it is evident that the organisers of the cricket
ensure that they will impart quality education. Only match are not at fault and therefore not liable to
this will circumvent the situation outlined in the para. compensate the cricket fan for any injury caused to
him by the cricket ball.

Previous Years
Page 8 CLAT & AILET Papers
184. a From the facts it is clear that as soon as the appendix 188. b A simple rule of syllogism is to deduce reasoning from
is operated out of the patient it becomes the property a major premise and connect it to a minor premise.
of the hospital. Thereafter, the research work that Here, the major premise is the first sentence of the
fetched the commercially viable drug was conducted problem which talks of an obligation for all motor vehicle
solely under the initiative of the hospital and no one to have a third party insurance. The Second sentence
else. Thus, the patient prima facie has no right to claim by way of merely defining what qualifies as motor
share from the profit earned by the hospital from the vehicle becomes the minor premise in the problem.
use of the removed appendix. Now, by way of connecting the minor premise to that
of the major premise it is clear that vehicles without
185. d Here, you have to read the facts very, very carefully, mechanical devices are not motor vehicles and thus,
and then apply plain and simple logic, and nothing else! vehicles which are not motor vehicles falls out of the
The facts tell you that she was thoroughly checked purview of “All Motor vehicles” ( as given in the first
before being allowed to enter the examination hall, so sentence). Hence it is deduced that vehicles which
the search must have been conclusive, and this are not motor vehicles need not have Third Party
satisfies reason (ii). However, reason (i) is the most Insurance.
appropriate, because it tells you what can be
reasonably and correctly presumed, and what cannot. 189. c It is a well settled rule to the effect that contract will
not be enforced if it is in violation of law. In any society,
186. c Contracts are agreements which are enforceable by governmental entities enact laws and make policies.
law. It is essential for all agreements to have a valid Public policy can be generally defined as a system of
acceptance to the offer made in the agreement. laws, regulatory measures, courses of action, and
Therefore it is clear, in order to a contract come into funding priorities concerning a given topic promulgated
existence, it is necessary that an offer made in the by a governmental entity or its representatives. Thus
agreement is accepted. However, keep in mind, All a subject matter of a contract which is opposed to the
Contracts are Agreements but all Agreements are not public policy cannot be validated.
Contracts.
190. a The answer should be (a), because it has the most
187. a Applying the principle of syllogism it is clear from the direct link with the principle.
facts that a minor cannot work in a shop.
One of the essential element for a valid contract is
competency of parties. Under section 11 of the Indian
Contract Act, 1872 it is stated that any person may
become a party to the contract except for who is a
minor or of unsound mind or forbidden by law.

Previous Years
CLAT & AILET Papers Page 9
CLAT Solutions 2009

1. a. Antonym for the word ‘deleterious’, which means 10. d Statements I, II and III are all false according to the
‘harmful’, is ‘beneficial’. passage. The author talks about extinction of the
human species, risks and harmful effects of nuclear
2. b The most important objective of the author is to illustrate war and post-war survivors being exposed to lethal
the devastating effects of the use of nuclear weapons radiations. Also, he talks about severe consequences
on mankind. Option (a) and (c) are incorrect. The use in areas not affected directly. Thus, option (d) is correct.
of nuclear weapons is not highlighted as a population
control measure neither has the author talked about 11. a Option (a) has the correct spelling, ‘renaissance’.
the supremacy of the nations which possess nuclear
weapons. Option (b) may seem correct but it is not. 12. d Option (d) has the correct spelling, ‘malaise’.
The author does talk about the long lasting biological
effects of nuclear weapons but summarizing these 13. b ‘Irrelevant’ is the correct spelling.
effects is not his objective. He has done so to stress
how harmful the use of nuclear weapons is to mankind 14. b ‘Surveillance’ is the correct spelling.
which is the main objective of the author.
15. a ‘Gaiety’ is the correct spelling.
3. c The second paragraph states that more than fifty
percent of the scientists are engaged in the 16. b ‘Mala Fide’ means in bad faith.
manufacture of destructive weapons. The passage
does not comment upon their number, their competence 17. c ‘Pro rata’ means in proportion of something.
or whether they are engaged in the industry against
their will. Thus, option (c) is the correct choice.
18. d ‘Vice versa’ means the other way round.
4. d The argument is that the use of nuclear weapons has
19. a ‘Ab initio’ means from the beginning.
a deleterious effect on mankind. The author states
that this argument will become stronger if moral and
20. b Alibi is a claim or piece of evidence that one was
legal considerations are combined. Thus, option (d)
elsewhere when an act, typically a criminal one, is
follows.
alleged to have taken place. Thus, option (b) is the
5. d The author has mentioned at several places that correct choice.
utilization of valuable skill for the manufacture of
weapons of destruction,when it must be engaged in 21. c ‘To give away the game’ means to reveal a secret.
other constructive activities, is inhuman. Option (a)
and (b) are complete opposites of this. The author 22. d ‘To cool one’s heels’ means to rest for some time.
doesn’t bring in the availability of funds to justify the
use of nuclear weapons. So, option (c) does not 23. b ‘To bury the hatchet’ means to forget the enmity.
follow. Option (d) is correct since it brings out the
author’s view perfectly. 24. c ‘Gift of the gab’ means to be a glib talker or a good
conservationalist.
6. d* There is no correct answer of this question. Only
option (d) can be accepted as an answer. 25. a ‘To smell a rat’ means to suspect a trick or a foul play.

7. c The author has used the word ‘devoted’ sarcastically 26. b ‘Inferior’ and ‘superior’ are idiomatically followed by
to highlight how the valuable skill is being abused in ‘to’ in typical constructions like the one given.
making weapons of mass destruction. Thus, option
(c) is the correct choice. 27. c ‘Annoyed’ must be followed by ‘with’. One gets
annoyed ‘with’ something/someone, not ‘against’,
8. d Options (a), (b) and (c) are mentioned as the ‘towards’ or ‘upon’.
consequences of the nuclear weapon. Option (d) is
not a consequence of nuclear weapon nor has it be 28. d Certain words are followed by particular prepositions
mentioned in the passage. only. ‘Averse’ is always followed by ‘to’. Other words
of this type are ‘conform to’, ‘comply with’, ‘absolve
9. b First line of the passage states, “There is a fairly from’ etc.
……..far enough”. This leads us to option (b).

Previous Years
Page 10 CLAT & AILET Papers
29. a To send ‘for’ someone/something is to call that person/ 40. b The events mentioned are logically related in the
thing. To send ‘in’ is to cause (someone) to arrive in or sequence given in option (b). ii, ‘we went to the pond’
become involved in a particular place or situation. Both starts the passage, followed by iv, ‘stood knee-deep...’
the options are correct here but option (d) makes more This is followed by iii which talks of throwing stones
sense in the context of the statement. to create ripples. i comes after iii which describes the
ripples.
30. c ‘to what’ is the correct choice. ‘listening’ should be
followed by ‘to’. 41. c A capital market is a market for securities (debt or
equity), where business enterprises (companies) and
31. b ‘Among’ is used when talking about things that are not governments can raise long-term funds. It is defined
distinct items or individuals. ‘Between’ is used when as a market in which money is provided for periods
talking about distinct individuals or items even if they longer than a year, as the raising of short-term funds
are more than two. Since, the sentence here talks takes place on other markets (e.g., the money market).
about ‘certain tribes’ which are similar entities, among The capital market includes the stock market (equity
is the correct choice. securities) and the bond market (debt). Money markets
and capital markets are parts of financial markets.
32. d To bring out means to uncover or make visible
something that was previously hidden/not visible. 42. This question has no correct answer.
Note: The National River Linking Project (NRLP) is
33. b For constructions like the one given in this sentence, designed to ease water shortages in western and
southern India while mitigating the impacts of recurrent
where the author talks about things which he wants
floods in the eastern parts of the Ganga basin. The
to happen, anticipates will happen or imagines
NRLP, if and when implemented, will be one of the
happening we use ‘were’ instead of was.
biggest interbasin water transfer projects in the world.
34. a ‘Turn up’ means to make an appearance, arrive. The
43. b “The Audacity of Hope: Thoughts on Reclaiming the
correct choice is option (a).
American Dream” is the second book written by then-
Senator Barack Obama. In the book, Obama expounds
35. c ‘from’ here is the correct choice. It is used to indicate
on many of the subjects that became part of his 2008
separation, exclusion or removal.
campaign for the presidency
36. b The correct sequence is given in option (b). ii starts 44. c The Wholesale Price Index or WPI is “the price of a
the sentence which is followed by iii since it tells what representative basket of wholesale goods. Some
the ‘dams should receive’. The next part is ii followed countries use the changes in this index to measure
by iv which completes the sentence. Starting the inflation in their economies, in particular India – The
sentence with “to ensure...the collector said...” is Indian WPI figure was earlier released weekly on every
wrong. Thus, option (d) is incorrect. Thursday and influenced stock and fixed price
markets. The Indian WPI is now updated on a monthly
37. d ii starts the sequence with the sentence talking about basis. The Wholesale Price Index focuses on the price
the ‘person’ and his ‘shirt’. iv follows ii since it of goods traded between corporations, rather than
describes the pockets and what is contained in them. goods bought by consumers, which is measured by
This is followed by i, them referring to toffees and the Consumer Price Index. The purpose of the WPI is
chocolates. iv ends the sequence by talking about the to monitor price movements that reflect supply and
reaction of the man. demand in industry, manufacturing and construction.
This helps in analyzing both macroeconomic and
38. a i starts the topic, followed by iii which adds more to microeconomic conditions.
the description of the legislation. ii follows iii by
providing the condition that the legislation needs fulfill 45. a A progressive tax is a tax by which the tax rate
to become a law as mentioned in iv. increases as the taxable base amount increases.
“Progressive” describes a distribution effect on income
39. c i opens the paragraph by talking about what the or expenditure, referring to the way the rate
farmers do for the country. This is followed by iv progresses from low to high, where the average tax
which concludes on i’s basis that they are the most rate is less than the marginal tax rate. It can be applied
useful members of the society. iii follows iv since it to individual taxes or to a tax system as a whole; a
gives the other side of the story. ii comes last as it year, multi-year, or lifetime. Progressive taxes attempt
states that because of this exploitation it is our duty to to reduce the tax incidence of people with a lower
protect them. ability-to-pay, as they shift the incidence increasingly
to those with a higher ability-to-pay.

Previous Years
CLAT & AILET Papers Page 11
46. a Chak De! India is a 2007 Indian woman’s sports film 54. c Dilip Kumar has been Awarded Nishan-E-Pakistan, the
about field hockey in India. Directed by Shimit Amin highest civilian award conferred by government of
and produced by Yash Raj Films, the film stars Pakistan, for his effort to bridge gap between India
Shahrukh Khan as Kabir Khan, the former captain of and Pakistan.
the Indian hockey team.
55. b
47. c X-ray computed tomography, also computed
tomography (CT scan) or computed axial tomography 56. b
(CAT scan), is a medical imaging procedure that utilizes
computer-processed X-rays to produce tomographic 57. b
images or ‘slices’ of specific areas of the body. These
cross-sectional images are used for diagnostic and 58. b
therapeutic purposes in various medical disciplines
59. a
48. This question has no correct answer.
Note : The 2009 World Food Prize was awarded to Dr. 60. a ndian Railways has declared the Year 2007 as
Gebisa Ejeta of Ethiopia, whose sorghum hybrids ‘Cleanliness Year’ as part of which special campaigns
resistant to drought and the devastating Striga weed to ensure cleanliness in station complexes, passenger
have dramatically increased the production and trains, railway lines, waiting rooms etc. will be
availability of one of the world’s five principal grains,
undertaken.
enhancing the food supply of hundreds of millions of
people in sub-Saharan Africa.
61. c Ashok Kumar (13 October 1911 – 10 December 2001)
also fondly called Dadamoni in Bengali, was an Indian
49. a The ISO 9000 family of standards is related to quality
film actor. Born in Bhagalpur, Bengal Presidency he
management systems and designed to help
attained iconic status in Indian cinema. The Government
organizations ensure that they meet the needs of
of India honoured him with the Dadasaheb Phalke
customers and other stakeholders while meeting
Award and the Padma Bhushan in 1998 for his
statutory and regulatory requirements related to the
product. The standards are published by ISO, the contributions to Indian cinema.
International Organization for Standardization, and
available through National standards bodies. 62. c he Outstanding Parliamentarian Award is an award
given by the Indian Parliamentary Group to an
50. d AGMARK is a certification mark employed on outstanding sitting Member of the Indian Parliament. It
agricultural products in India, assuring that they conform was instituted in 1995 by Shivraj Patil who was then
to a set of standards approved by the Directorate of the Speaker, Lok Sabha from 1991-96. In 2007, the
Marketing and Inspection, an agency of the award was conferred upon Mr. Mani Shankar Aiyar.
Government of India. The AGMARK is legally enforced
in India by the Agricultural Produce (Grading and 63. a The world’s largest sea bridge that has taken shape in
Marking) Act of 1937 ( amended in 1986). The present 2007 is in China. The 26-mileJiaozhou Bay crossing
AGMARK standards cover quality guidelines for 205 connecting the port city of Qingdao, south-east of
different commodities spanning a variety of Pulses, Beijing, to the industrial district of Huangdao is currently
Cereals, Essential Oils, Vegetable Oils, Fruits & the longest sea bridge in the world.
Vegetables, and semi-processed products like
Vermicelli. 64. d Vikram Shankar Pandit (born 14 January 1957) is an
Indian-born American banker.He is the chief executive
51. c of Citigroup, a position he has held since December
2007.
52. a The NAL Saras (Sanskrit: Crane) is the first Indian
multi-purpose civilian aircraft in the Light Transport 65. d Maankombu Sambasivan Swaminathan (born 7 August
Aircraft category designed by the National Aerospace 1925) is an Indian geneticist and international
Laboratories (NAL). The first Saras (PT1) completed administrator, renowned for his leading role in India’s
its maiden flight at the HAL airport in Bangalore on 29 “Green Revolution,” a program under which high-yield
May 2004. varieties of wheat and rice seedlings were planted in
the fields of poor farmers.
53. a Sirimavo Bandaranaike (April 17, 1916 – October 10,
2000) was a Sri Lankan politician and the modern 66. b The International Tribunal for the Law of the Sea
world’s first female head of government. She served (ITLOS) based in Hamburg, Germany is an
as Prime Minister of Ceylon and Sri Lanka three times, intergovernmental organization created by the mandate
1960–65, 1970–77 and 1994–2000, and was a long- of the Third United Nations Conference on the Law of
time leader of the Sri Lanka Freedom Party. the Sea. It was established by the United Nations

Previous Years
Page 12 CLAT & AILET Papers
Convention on the Law of the Sea, signed at Montego 76. b The Samjhauta Express commonly called the Friendship
Bay, Jamaica, on December 10, 1982. The Convention Express, is a twice-weekly train – Tuesdays and
entered into force on November 16, 1994. Mr. P. Fridays – that runs between Delhi and Attari in India
Chandrasekhara Rao is the Indian judge in the tribunal and Lahore in Pakistan. The word samjhauta means
that has a set of 21 serving judges from a variety of “Understanding”,”accord” and “compromise” in both
states parties. Hindi and Urdu.

67. a Ozone the layer of gas that forms a protective covering 77. b Blue Revolution is the water equivalent of the green
in the Earth’s upper atmosphere is formed when revolution and primarily refers to the management of
oxygen molecules absorb ultraviolet photons and water resources that can steer humanity to achieve
undergo a chemical reaction known as photo drinking water and crop irrigation security.
dissociation or photolysis, where a single molecule of
oxygen breaks down to two oxygen atoms. The Ozone 78. d Bharat Ratna Dr. Avul Pakir Jainulabdeen Abdul Kalam
Layer thins down as a result of a chain chemical born October 15, 1931, Tamil Nadu, India, usually
reaction that separates from the Oxygen layer. referred to as Dr. A. P. J. Abdul Kalam, who was the
eleventh President of India, serving from 2002 to 2007
68. d The South Asian University is an International has been appointed as the Chancellor of IIST
University sponsored by the eight Member States of Trivandrum.
the South Asian Association for Regional Cooperation
(SAARC). The eight countries are: Afghanistan, 79. b In Uttar Pradesh ‘Kanya Vidyandhan Yojna’ is
Bangladesh, Bhutan, India, Maldives, Nepal, Pakistan operational.
and Sri Lanka. South Asian University started admitting
students in 2010, as a temporary campus at Akbar 80. c Parvez was the name of Emperor Shahjahan before
Bhawan, India. Its permanent campus will be at Maidan he became Emperor.
Garhi in South Delhi, India, next to Indira Gandhi National
Open University (IGNOU). 81. b A savanna, or savannah, is a grassland ecosystem
characterized by the trees being sufficiently small or
69. b The University of Bombay established in the year 1857. widely spaced so that the canopy does not close. The
open canopy allows sufficient light to reach the ground
70. d Vitamin K is necessary for normal blood clotting and to support an unbroken herbaceous layer consisting
synthesis of proteins found in plasma, bone, and primarily of grasses. Among the given options largest
kidneys and is found in spinach, lettuce, kale, cabbage, amount of Savannah is in Africa, therefore, Africa is
cauliflower, wheat bran, organ meats, cereals, some the most likely correct response.
fruits, meats, dairy products, eggs.
82. a State of Andra Pradesh launch a health insurance
71. a Gilt edged market deals in securities issued by central, scheme ‘Aarogya Sri’ for the below poverty line
state and autonomous govt authorities such as families.
municipalities, city corporations, state electricity boards
etc and development financial institutions such as IDBI,I 83. c India’s First Nuclear Reactor was Apsara. It was also
FCI, NABARD etc. Since government issues these the first nuclear reactor in Asia. Apsara went critical
classes of securities, they all are considered to be at Bhabha Atomic Research Centre (BARC), Trombay
quite safe and liquid for investment. Hence the term on August 4, 1956. It heralded the arrival of India’s
“gilt -edged securities” nuclear energy programme. Dr. Homi Bhabha himself
conceptualised the design of the reactor and the
72. c Sumatra is the largest island in the world (if Australia reactor was built entirely by Indian engineers in a
is not considered an island). record time of about 15 months.

73. c Sania Mirza is the youngest recipient of Padma Shri so 84. a ‘Jet lite’ airlines of today is an offspring of Jet Airways
far. Padma Shri (also Padmashree) is the fourth highest and Sahara.
civilian award in the Republic of India, after the Bharat
Ratna, the Padma Vibhushan and the Padma Bhushan. 85. a The Suez Canal, also known by the nickname “The
It is awarded by the Government of India. Highway to India”, is an artificial sea-level waterway
in Egypt, connecting the Mediterranean Sea and the
74. d Red Sea. Opened in November 1869 after 10 years of
construction work, it allows transportation by water
75. a The territorial waters of India extend up to 12 nautical between Europe and Asia without navigation around
miles. Africa.

86. d In Malaysia, the word ‘bhumiputra’ refers to Buddhists.

Previous Years
CLAT & AILET Papers Page 13
87. c HINDRAF or Hindu Rights Action Force with its slogan 93. a SP of 11 oranges = Re. 1
of People’s Power began as a coalition of 30 Hindu
1
non-governmental organisations committed to the SP of an orange = Re.
preservation of Hindu community rights and heritage 11
in a multiracial Malaysia. . HINDRAF has made a major CP of an orange (Using the concept of multiplying
impact to the political landscape of Malaysia in staging index)
the 2007 HINDRAF rally. After several warnings by SP
the Malaysian government HINDRAF was officially MF =
CP
banned on October 15, 2008.
 1
88. b The Intergovernmental Panel on Climate Change (IPCC)  
CP =   = Re.
11 10
is a scientific intergovernmental body, set up at the
0.9 99
request of member governments.It was first
established in 1988 by two United Nations SP of an orange if sold on profit (Using the concept of
organizations, the World Meteorological Organization multiplying index)
(WMO) and the United Nations Environment Programme 10 1
(UNEP). Its mission is to provide comprehensive SP = 1.1 × = Re.
99 9
scientific assessments of current scientific, technical
and socio-economic information worldwide about the Number of oranges sold for a rupee = 1/(1/9) = 9
risk of climate change caused by human activity, its
94. b Let the time taken in walking and riding the distance be
potential environmental and socio-economic
W hrs and R hrs respectively.
consequences, and possible options for adapting to
W+R=3 ... (i)
these consequences or mitigating the effects. It is
W+W=5 ... (ii)
chaired by Rajendra K. Pachauri.
Using the above two equations, we get
W = 2.5 hrs and R = 0.5 hrs
89. a Kandhamal is a district of the state of Orissa, India.
Thus, time taken to ride both ways = 2 × 0.5 = 1 hr.
The district headquarters of the district is Phulbani.

90. c World No Tobacco Day (WNTD) is observed around 1


95. a Required percentage = × 100 = 12.5%.
the world every year on May 31. It is meant to 8
encourage a 24-hour period of abstinence from all
forms of tobacco consumption across the globe. The
day is further intended to draw global attention to the  1
96. c As we know that 12.5% is equivalent to   of a
widespread prevalence of tobacco use and to negative 8
health effects, which currently lead to 5.4 million certain quantity.
deaths worldwide annually. The member states of the
 1
World Health Organization (WHO) created World No Thus, required value =   × 80 = 10
Tobacco Day in 1987. 8

91. a Total monthly income of 5 members in the family 97. b The logic here is that two alternate series are given as
= 5 × 1000 = Rs. 5,000 (2, 5, 8, 11) and (1, 4, 7, 10, ?). Now, the term replacing
Total yearly income of 5 members in the family (?) would be 10 + 3 = 13.
= 5000 × 12 = Rs. 60,000
Total yearly income of 5 members in the family after
increment = 60,000 + 12,000 = Rs. 72,000 9
98. c Option (a) = = 0.9
Increased average yearly income of 5 members 10

72000 11
= = Rs. 6,000 Option (b) = = 0.9167
12 12
Increased average monthly income of 5 members
23
6000 Option (c) = = 0.82
= = Rs. 1,200. 28
5
32
Option (d) = = 0.96
92. d It should be noted that he uses 800 gm weight for 33
1000 gm.
23
Difference in weight = 1000 – 800 = 200 gm Thus, is the smallest among the given fractions.
28
200
Profit percentage = × 100 = 25%
800

Previous Years
Page 14 CLAT & AILET Papers
99. * There is some error in the question as no 106. c Ratio between male to female = 1000 : 1075 = 40 : 43
information is given regarding the amount paid
by any one of them.  40 
Number of males in the town = 155625 ×  
 83 
100. a Total age of 29 boys in the class = 29 × 14 = 406 years = 75000
Total age of 29 boys and the class teacher Number of females in the town = 155625 – 75000
= 30 × 15 = 450 years
= 80625
Age of the class teacher = 450 – 406 = 44 years
Number of literate, in town = 75000 × 0.4 + 80625 ×
0.24 = 30000 + 19350 = 49350
101. b Let us assume that 6 workers will take ‘n’ days. Then,
8 × 96 = 6 × n 49350
Required percentage = × 100 = 31.7%
 96  155625
n = 8 ×   = 8 × 16 = 128 days
 6 
107. d Total price of 10 sheeps = 10 × 450 = Rs. 4500
102. c Let us assume Shyam’s income be Rs. 100. Total price of 5 pigs = Rs. 6000 – Rs. 4500 = Rs. 1500
Thus, Ram’s income = 100 – 20 = Rs. 80
Rs.1500
Difference in their income = Rs. 100 – Rs. 80 = Rs. 20 Average price of a pig = = Rs. 300.
5
 20 
Required percentage =   × 100 = 25% .
 80  108. a Let Shyam’s weight be x kg.
Therefore, Ram’s weight = (x + 25) kg
103. d Let the monthly salary of A, B and C be Rs. 2x, Rs. 3x According to the question,
and Rs. 5x respectively. x + x + 25 = 325
Difference between the monthly salary of C and A
⇒ x = 150
= 5x – 2x = Rs. 3x
According to the question, ∴ Shyam’s weight = 150 kg.
3x = 1200
⇒ x = 400 109. b The series is moving as (n2 – 1), where n is a natural
B’s monthly salary = 3x = 3 × 400 = Rs. 1200 number starting from 2. Thus, 34 doesn’t satisfy the
It seems that B’s monthly salary is asked given condition.
instead of his/her annual salary. None of the
options are close to B’s annual salary. So we 110. c Place value of 7 = 7 × 104 = 70000.
have kept it to B’s monthly salary.
111. b Manusmriti
Manismriti (translated “Laws of Manu”) is regarded as
2
104. a Number of foreigners = 94500 ×   = 21000 the foundational work of Hindu law and ancient Indian
9 society, compiled and written in c.200 CE in India. It is
Number of immigrants = 6400 attributed to the legendary first man and lawgiver,
Number of natives = 94500 – (21000 + 6400) Manu. It is also known as Mânava-Dharmaúâstra.
= 94500 – 27400 = 67100
112. c Conflict of laws
105. b All of the given information can be tabulated as:
The term Conflict of law describes the body of law of
each country or state that is designed to resolve
A B C
problems arising from the differences between legal
Income A B C system of different countries.
Expenditure 0.80A 0.85B 0.75C
(a) Civil law- seeks to resolve non-criminal disputes
Saving 0.20A 0.15B 0.25C such as disagreements over the meaning of
contracts, property ownership, divorce, child
Their savings are in the ratio of 8 : 9 : 20. custody, and damages for personal and property
∴ For 0.20 A = 8, A = 40 damage.
For 0.15B = 9, B = 60 (b) Local laws- laws passed by a legislative body
For 0.25C = 20, C = 80 and intended to apply only to one part of the area
Now, A + B + C = 144000 under its jurisdiction
⇒ 40x + 60x + 80x = 144000 (d) Common law- laws developed by judges through
⇒ 180x = 144000 ⇒ x = 800 decisions of courts
Thus, C’s salary = 80 × 800 = Rs. 64,000.

Previous Years
CLAT & AILET Papers Page 15
113. a Earnest money 118. b Vicarious liability
When a buyer makes an offer to buy residential real (a) Strict liability- A rule which makes a person legally
responsible for the damage and loss caused by
estate, he/she generally signs a contract and pays a
his or her acts and omissions regardless of
sum acceptable to the seller by way of earnest money culpability (including fault in criminal law terms,
(b) Advance- is the part of a contractually due sum typically the presence of mens rea).
that is paid or received in advance for goods or (c) Tortous liability- Obligation of one party to a victim
services, while the balance included in the as a result of a civil wrong or injury
invoice will only follow the delivery. (d) Absolute liability – commonly used to describe
offences in which it is not open to an accused to
(c) Interest- amount paid by a borrower of assets to
avoid criminal liability on the ground that he acted
the owner as a form of compensation for the use under a reasonable mistake of fact which, if the
of the assets. It is most commonly the price paid facts had been as the accused believed them to
for the use of borrowed money, or money earned be, would have made his act innocent.
by deposited funds.
(d) Solatium- is a form of compensation for emotional 119. a Tresspass to property is the unauthorised invasion to
the property of another. If someone enters onto
rather than physical or financial harm
another‘s land without his permission the person
entering shall be liable for tresspass. In tresspass
114. a Co-heirs possession of the property is sufficient, ownership is
(b) Coparceners- two or more people inherit a title not essentially required.
equally between them
(c) Successors- a person who succeeds another 120. c Public Accounts Committee: The Public Accounts
Committee (PAC) is a committee of selected members
in an office, position, or the like.
of Parliament, constituted by the Parliament of India,
(d) Joint owners- two parties owning property for the auditing of the expenditure of the Government
together of India. The PAC is formed every year with a strength
of not more than 22 members of which 15 are from
115. d Locus standi Lok Sabha, the lower house of the Parliament, and 7
(a) Right ‘in rem’- property rights enforceable against from Rajya Sabha, the upper house of the Parliament
the entire world
121. d Keshavananda Bharti v. State of Kerala
(b) Right ‘in Personem’ - personal right attached to a (a) Golaknath v. State of Punjab- Supreme Court held
specific person, such as contract rights, a tort that Parliament could not curtail any of the
award against a defendant, or a license. Fundamental Rights in the Constitution.
(c) Fundamental rights- basic human freedoms
which every Indian citizen has the right to enjoy 122. d MRTP Act
The Monopolies and Restrictive Trade Practices Act,
for a proper and harmonious development of
1969 was promulgated with the aim to prevent
personality concentration of economic power to the common
detriment, provide control for control of monopolies
116. a Bicameralism and probation of monopolistic, restrictive and unfair
The Legislative body of India having two houses: trade practices, and protect consumer interest. It has
House of the people (Lok Sabha) and Council of been replaced by Competition Act, 2002.
states (Rajya sabha)
123. c IImperfect right
(b) Universal adult franchise- All citizens with age Imperfect right is a right recognized by law, but could
above 18 have the right to vote. not be enforceable due to its form or some other
(c) Dyarchy- A form of government controlled by defects
two rulers (a) Universal right- One which is correspondent to
(d) Federalism- A form of government wherein union legal duty which is common to all purposes,
conditions, or situations:
of states under a central government is distinct
(b) Perfect right- one which is correspondent to legal
from the individual governments of the separate duty, and not only recognized by law but also
states. enforced by law.
(d) Fundamental rights- is defined as basic human
117. a State of Maharashtra v. Prafull B. Desai freedoms which every citizen has the right to
enjoy.

Previous Years
Page 16 CLAT & AILET Papers
124. a Geneva Convention 134. c Sedition includes all acts which create hatred or
The Geneva Conventions are rules that apply in times contemp for, or dissatisfaction towards the
of armed conflict and seek to protect people who are government established by law.
not or are no longer taking part in hostilities. These
include the sick and wounded of armed forces on the 135. b 1945
field, wounded shipwrecked members of armed forces India was a founding member of United Nations in
at sea, prisoners of war and civilians. This convention October 1945, despite it being a British colony. India,
comprises of four treaties, and three additional Canada, the Union of South Africa, New Zealand and
protocols that establish the standards of international Australia were all British colonies but were given
law for the humanitarian treatment of the victims of independent seats in the UN General Assembly.
war. The singular term Geneva Convention denotes
the agreements of 1949, negotiated in the aftermath 136. c Litigant
of the Second World War (1939–45), which updated A party to a lawsuit; “plaintiffs and defendants are
the terms of the first three treaties (1864, 1906, 1929), both litigants”.
and added a fourth treaty. (a) Accused- a person charged with a crime.
(b) Plaintiff- A person who brings a case against
125. d Negotiable Instruments Act another in a court of law.
Negotiable Instruments Act, 1881 defines “Public (d) Complainant- Synonymous to Plaintiff
Holiday” in the explanation of its Section 25.
137. a Justice M. M. Punchi
126. b Prosecution The commission was constituted on April 28, 2007 to
take a fresh look at the relative roles and responsibilities
127. c Compoundable offence of the various levels of government and their inter-
relations. The terms of reference of the Justice Punchhi
128. a Conjugal right Commission are comprehensive and are a step
forward from the Justice Sarkaria Commission set up
129. b Died without making a will in 1983.
Justice Punchi was the 28th Chief Justice of India
130. c Perjury from January 18, 1998 until his retirement on October
9, 1998.
131. a Posthumous (b) Justice Nanavati- The Justice G..T. Nanavati
commission was established by the Indian
132. b Power of Attorney Government in 2000 to investigate the 1984 anti-
(a) Affidavit-A written statement confirmed by oath sikh riots.
or affirmation, for use as evidence in court. (c) Justice Bharucha- Justice Sam Piroj Bharucha
(c) Will- A document in which a person specifies the was the 30th Chief Justice of India, serving from
method to be applied in the management and November 2001 until his retirement in 2002.
distribution of his estate after his death. Justice Bharucha is responsible for many
(d) Declaration- a paper subscribed by a grantee of significant legal decisions. He was part of the
property, acknowledging that he holds it in trust five judge constitutional panel which unanimously
for the purposes and upon the terms set forth. ruled on the 2001 dismissal of J. Jayalalitha as
Chief Minister of Tamil Nadu. It was the first and
133. a Parole only such dismissal of a chief minister in India’s
The conditional release of a person convicted of a history.
crime prior to the expiration of that person’s term of (d) Justice Kuldip Singh- Acted as the Chair person
imprisonment, subject to both the supervision of the of the Delimitation Commission set up on 12th
correctional authorities during the remainder of the July, 2002. This commission is established by
term and a resumption of the imprisonment upon Government of India under the provisions of the
violation of the conditions imposed. Delimitation Commission Act. The main task of the
(b) Amnesty- An official pardon for people who have commission is to redraw the boundaries of the
been convicted of political offences. various assembly and Lok Sabha constituencies
(c) Discharge- a release, as of a person held under based on a recent census.
legal restraint
(d) Pardon- The action of an executive official of the
government that mitigates or sets aside the
punishment for a crime.

Previous Years
CLAT & AILET Papers Page 17
138. c Double Jeopardy 146. a Trust
(a) Burden of proof- A duty placed upon a civil or Fiduciary relationship is a relationship where one
criminal defendant to prove or disprove a disputed person places complete confidence in another in
fact regard to a particular transaction or one’s general
(b) Double conviction- a convict found guilty of an affairs or business. The relationship is not necessarily
offence after transportation. formally or legally established as in a declaration of
(d) Corpus Delicti- Term which refers to the principle trust, but can be one of moral or personal responsibility,
that a crime must have been proven to have due to the superior knowledge and training of the
occurred before a person can be convicted of fiduciary as compared to the one whose affairs the
committing that crime fiduciary is handling. Ex: Lawyer- Client, Doctor-
Patient, Teacher-Student relationships
139. d Accomplice
147. b
140. a Land Revenue
148. c U.S.A
141. b Bhopal Judicial Review is a constitutional doctrine that gives
National Judicial Academy is a government funded to a court system the power to annul legislative or
training institute established on August 17, 1993 under executive acts which the judges declare to be
the Societies Registration Act, 1860. This institution unconstitutional. The Constitution of USA does not
focuses on training judicial officers working in the expressly provide that the federal judiciary has the
Supreme Court and High Courts during their power of judicial review. However, after the Marbury
vs. Madison case judicial review was exercised by
142. c Attorney General the American judiciary under Article III of the
The Attorney General for India is appointed by the constitution of America.
President of India under Article 76 of the Constitution
of India. It is the duty of the Attorney General for India 149. b Obligation
to give advice to the Government of India upon such
legal matters and to perform such other duties of legal 150. b Infanticide
character as may be referred or assigned to him by
the President. In the performance of his duties, he has 151. c Sacrilege
the right of audience in all Courts in India as well as the
right to take part in the proceedings of Parliament 152. d Absconder
without the right to vote. In discharge of his functions,
the Attorney General is assisted by a Solicitor General 153. a Warning
and four Additional Solicitors General. (b) An injunction- A court order by which an individual
is required to perform, or is restrained from
143. a The word ‘morality’ doesn’t find its mentioned in the performing, a particular act
preamble of the Constitution of India. (c) Writ- An order issued by a court requiring that
something be done or giving authority to do a
144. c A Court of Record is the court which has been specified act.
conferred the power to punish and impose fine for its (d) Certiorari- A type of writ wherein a higher court
contempt. issues an order to review the decision and
proceedings in a lower court and determine
145. d Proven misbehaviour or incapacity whether there were any irregularities.
According to article 124(4) of the Indian constitution: A
member of higher judiciary which is the chief justice 154. b Wakfs
of India and the state high courts can be removed Wakf Board is a statuary body constituted under the
from the service through the process of impeachment Wakf Act to administer, control and manage the Wakf
on the ground of proven misbehaviour. In India, there properties comprising of any moveable or immoveable
is no other process by a judge can be removed from property which is pious , religious, or charitable. Charity
the office before his or her term comes to an end. is the dominant feature of a Wakf. The creator of a
As per the judge’s inquiry act 1986, the impeachment Wakf is called Wakif. The income of the Wakf Property
of judges can be done on grounds of “proven is utilized for charity, assistance to the poor and needy,
Misbehaviour” and “incapacity”, if the judge of India is maintenance of the Mosque, Dargah, Graveyards,
to be Impeached than the recommendation have to be Khankah, educational institution, hospitals. A Wakf is
made by the chief Justice of India to the president of managed by the mutawalli or a managing committee or
India. The motion is passed by the two-third majority directly by the Wakf Board.
members present and Voting must be done separately
in the each house of the parliament. If the motion is
passed than the formal announcement is done by the
president of India.

Previous Years
Page 18 CLAT & AILET Papers
155. — High sea is the open ocean, esp. that not within 158. a Both A and R are individually true and R is the correct
any country’s jurisdiction. As per maritime law, the explanation of A.
extent of high sea commences beyond 200 Nautical
Miles from the coastline. Amongst the given options, 159. a Both A and R are individually true and R is the correct
none of them seems to be correct. Firstly because, as explanation of A.
per maritime law, the extent of territorial waters is
always measured in Nautical Miles (NM) but the given 160. a Both A and R are individually true and R is the correct
options are either in miles or in kilometres. The correct explanation of A.
answer ‘would’ have been C if, instead of 200 Kms, it
would have been mentioned as 200 Nautical Miles. 161. b Both A and R are individually true but R is not the
correct explanation of A.

162. b Both A and R are individually true but R is not the


correct explanation of A.

Territorial
163. a Both A and R are individually true and R is the correct
explanation of A.

164. b Both A and R are individually true but R is not the


correct explanation of A.

165. a Both A and R are individually true and R is the correct


explanation of A.

166. a It has been suggested that the patient’s condition will


improve after the operation. This leads us to the
assumption that the patient can be operated.

167. b The statement highlights that detergents should be


used to clean the clothes. It implies that detergent will
remove the stains of dirt and greese.

168. d It is clearly stated that there is a great demand of the


tickets. This removes assumption I. As there is a
restriction of five tickets we cannot assume that people
are not willing to buy more than five tickets.

169. b The advertisement is created to attract people. So it


can be assumed that people want their money to grow.
But statement I is beyond the scope of information
given in the question statement.

170. b The question states that films are indispensable for


the entertainment of the people. So, II is definitely
assumed. Assumption I is beyond the scope of
information given in the question statement.

171. d The statement given does not tell us whether the


candidate reads the newspaper. Nor does it tell that
recent news are broadcast on only on radio. Thus,
both the assumptions are not implicit.

172. a The statement makes a comparison between the book


in question with other books on the same topic. Thus,
156. a Both A and R are individually true and R is the correct it assumes that other books were available on the
explanation of A. topic. However, statement II is not implied in the
statement.
157. b Both A and R are individually true but R is not the
correct explanation of A.

Previous Years
CLAT & AILET Papers Page 19
173. b One cannot assume that all companies have a lawyer. 189. b Black ad white are considered opposites. So, if black
The question statement highlights that the lawyer of is to absence then white is to presence.
the company can be called for further clarification.
This implies that the lawyer is thoroughly briefed about 190. d What Governer is to the state, President it to the nation.
the topic. Similarly, what Chief Minister is to the state, Prime
Minister is to the nation.
174. a Assumption I is implicit in the statement. Assumption II
is not implicit as we cannot say that education alone is 191. b The question statement highlights the problem of the
responsible for the nation. food processing industry so, the course of action
should address this problem. II is the course of action
175. a It is mentioned that children are influenced by their that will help the problem raised in the question
teacher. This implies that students consider their statement. Course of action I is far-fetched.
teachers role models. Duration of time spent is not
related to the influence of teachers on the students. 192. a The officer is suspecting something and he is not
having any evidence. So, there is no need to call the
176. b The lady in the photograph is the mother of the speaker police. He should first check the money. This makes I
and Ram’s maternal grandmother. Thus, the speaker the desired course of action.
is the wife of Ram’s father.
193. b The question statement is highlighting the importance
177. c The man is the only son of that woman who is that of having experienced people in the team. Course of
woman’s mother. Thus, the woman is that man’s sister. action II is an extension of the idea mention in the
question statement. I is not a course of action.
178. d The grandson of Shyam’s mother is Shyam’s son and
his son’s wife is his daughter-in-law. Or Shyam is her 194. a The sale of a product has gone down considerably so
father-in-law. This is the best possible answer as he an enquiry into the rival product is a desirable course
could be her father-in-law’s brother. of action. II is not the desirable course of action.

179. a Sunita’s husband’s wife is she herself. Now, the man 195. b Since Mr. X is part of the union, his bad behaviour
is the brother of her daughter. It means that the man is should be communicated to the union. But, transferring
Sunita’s son. Mr. X will not solve the problem of bad behaviour.

180. c Man’s brother’s father is that man’s father. Now, that 196. d Argument I does not provide a strong reason for ban
woman’s grandfather is also that Man’s grandfather. on trapping of wild animals. Just because trappers
This in turn implies that the man and the woman are are earning a lot of money does not provide a reason
siblings. for banning trapping of wild animals. Similarly, argument
II is not strong because it is not a reason for not
181. c Legislature is responsible for the enactment and banning. It is merely a statement that tells that the bans
executive is responsible for the implementation. are generally not effective.

182. a ‘Uttaranchal’ is carved out of UP. Similarly, Jharkhand 197. c Both the arguments are strong. One highlights the
is carved out of Bihar. need and second highlights problem.

183. b Gold and silver are two different metals, similarly cotton 198. c Either argument II or I is strong. Argument I gives the
and silk are two different fibres. reason for offering more incentives. Argument II gives
the reason for not increasing the incentives.
184. a Botany is a study of flora and zoology is a study of
fauna. 199. d Both the arguments are not strong. For curbing crimes,
banning of hotels is not a solution since the international
185. c Cold wave is a phenomenon of winter and loo is a criminals can find another place to operate. Similarly,
wind system of summer. argument II is not true since banning luxury hotels
does not mean that affluent tourists won’t have any
186. a King belongs to royalty similarly a saint belongs to a place to stay.
religion.
200. b Only argument II is strong. This highlights the problem
187. c Sculptor creates a statue and a poet composes a of implementing the suggestion of the question
poem. statement. Argument I is not related to the question
statement.
188. a Laughter is caused by happiness and someone cries
because one is sad.

Previous Years
Page 20 CLAT & AILET Papers
CLAT Solutions 2010
1.a From the given jumbled sentences, (iv) logically follows 10. c Antigone begs her sister Ismene to not desert their
(i) as (iv) shows when the church was destroyed. (ii) brother Polynices who lay dead and unburied, and
is a good introductory sentence. Looking at the options thought of it as a challenge to her royal blood. She
given, (b) and (c) get eliminated since in these two reminds Ismene of the "holiest law of heaven" which
options, (iv) does not follow (i). Out of options (a) and hints at one's responsibility towards one's kin.
(d), one can comprehend that the sequence in option
(a) is logically correct. 11. c Antigone decides to defy the orders of Creon to fulfill
her duty and prove that she is worthy of her royal
2. c From the given jumbled sentences, one can see that blood. She believes that her sister must not "shrug off
(iv) logically follows (i) as it provides the verb to the the burden" and "defy the holiest laws of heaven".
subject mentioned in (i). This is because (i) is an
incomplete sentence. Notice that sentence (iii) reads 12. d Creon, the new king, "resolved to …..by refusing the
"and no law containing a declaration that it is for giving right of honourable burial. The penalty of death was
effect to such policy shall be called in question in any promulgated against any who should defy this order".
court on the ground that it does not give effect to such This, coupled with Ismene's fear "how could you dare-
policy." and (i) and (iv) together form a complete when Creon has expressly forbidden it?" gives one
sentence that is "no law giving effect to the policy of enough reason to believe that Antigone might have
the state towards securing all or any of the principles been executed if she had attempted to stand up against
laid in part IV shall be deemed to be void in the ground Creon's orders and tried to give her brother an
that it is inconsistent with or takes away or abridges honourable burial.
any rights conferred by article 14 or 19" Since option
(c) is the only option out of the ones given, that has (i) 13. c This is quite clear from Ismene's response to Antigone
followed by (iv), option (c) is correct. when the latter urges the former to defy Creon's orders
and help her give their brother, Polynices, an
3. a From the given jumbled sentences, one can see that honourable burial. ("Antigone, we are women, it is not
(ii) logically follows (iii) and that whatever may be the for us to fight against men.")
sequence, this pair would always come together. This
is because (iii) talks about something that the President 14. d Option (a) is incorrect since the curse of the Gods
may (or may not) do and (ii), as an extension to (iii) was upon Oedipus and not his sons. Option (b) is
reads that "if he does so", the action talked about in incorrect since there was no mention of greed being
sentence (ii) shall take place, that is, 'if the President the reason for the fight amongst the two brothers.
sends a notification summoning the Houses to meet in Option (c) too, is incorrect since there is no mention of
a joint sitting for the purpose specified in the notification, there being a conflict between Oedipus's sons over
the Houses shall meet accordingly'. Now, only option inheritance. Option (d) is correct since the passage
(a) has this above mentioned pair in the required says that "Creon resolved to make an example of the
sequence. Therefore, option (a) is correct. brother who had fought against him".

4. a ‘Pick holes in’ means to find faults. 15. c A ‘carrion’ is a bird which eats dead animals' flesh.

5. d The word 'run down' is opposite to the underlined 16. b Refer to the line "Creon resolved to make an example
phrase in meaning . [ However, if we substitute the of the brother who had fought against him, ……
phrase in the sentence by run down, sentence sounds honourable burial."
awkward. Other options do not fit in at all.]
17. a Refer to the lines, “How could you dare...it is not for us
6. a ‘Buttering up’ means to praise insincerely. to fight against men”. Ismene was clearly afraid of the
king because he had given orders of death penalty for
7. a ‘Perfect’ here is a verb which means to make something anyone who would defy his orders. She did not have
completely free from faults and defects. the courage to go against the king in order to help her
sister. Thus, option (a) is the correct choice.
8. c ‘Kindly’ here is an adverb (of manner) which means ‘in
a kind manner’. 18. a This is clear from the beginning of the passage "…
Oedipus, that tragic figure… who had been cursed by
9. a ‘Very’ is used here for emphasis which modifies ‘fast’, Gods for mistakenly killing his father and subsequently
the adjective. Thus, it is an adverb. marrying his mother and assuming the throne of
Thebes."

Previous Years
CLAT & AILET Papers Page 21
19. a Vicarious liability means the responsibility of the 36. a ‘with’ is the correct preposition. ‘done away with’ is a
superior for the acts of their subordinate, or, in a phrasal verb which means to discard something,
broader sense, the responsibility of any third party stop doing something.
that had the "right, ability or duty to control" the activities
of a violator (Source: Wikipedia). In the passage, no 37. c ‘take after’ someone, meaning to look or behave like
one is responsible for other person's act and hence, someone is the correct phrasal verb.
option (a) is correct.
38. d The first ‘take on’ means hire; the second ‘take on’
20. b ‘Hit upon’ means to find something by chance. ‘Chanced means to undertake.
upon’ is the closest in meaning to this.
39. c ‘To take something in stride’ means to cope up with
21. d ‘Sticking out’ means to continue to endure something something calmly; without disrupting one’s
unpleasant for a long time waiting for the things to normal routine.
become better. Option (d), persistent in demanding, is
40. d ‘take off’ is to achieve wide use or popularity.
the correct choice.
41. a
22. c To ‘break off’ is to stop talking, take a pause. ‘Break
down’ is to start crying. 42. c Periyar Wildlife Sanctuary is a protected area in the
districts of Idukki and Pathanamthitta in Kerala, south
23. c ‘To lead someone by the nose’ is to control someone India. It is notable as an elephant reserve and a tiger
and to make them do exactly what you want them reserve. The protected area covers an area of 925
to do. Thus, option (c), ‘to follow submissively’, is the km2 (357 sq mi). 350 km2 (140 sq mi) of the core zone
best choice. was declared as the Periyar National Park in 1982.

24. b ‘up to the eye’ means to the extreme, completely, or to 43. d


have too much of something. ‘Completely’ is the
correct choice, i.e. the mill has been mortgaged to the 44. b In 1024 CE, the Somnath Temple located in the Prabhas
last penny. Kshetra near Veraval in Saurashtra, on the western
coast of Gujarat was destroyed by Mahmud of Ghazni.
25. c ‘Loaves and fishes’ is an idiom for a miracle of Jesus
Christ wherein He provided food to the poor 45. c Cape Trafalgar (Spanish: Cabo Trafalgar) is a head-
and hungry from only five loaves of bread and two land in the Province of Cádiz in the south-west of
fishes. After feeding thousands of men, a lot Spain. It lies on the shore of the Atlantic Ocean, north-
of food was still left. Thus, ‘loaves and fishes’ here west of the Strait of Gibraltar at 36°10’58"N, 6°2’2"W.
refer to the material benefits that people seek to get The International Hydrographic Organization defines
from God. the Western limit of the strait as a line that joins Cape
Trafalgar to the North to Cape Spartel to the South.
26. a Option (a) gives the correct spellings.
46. c Lumbinî is a Buddhist pilgrimage site in the Rupandehi
27. b Option (b) gives the correct spellings. district of Nepal. It is the place where Queen Mayadevi
gave birth to Siddhartha Gautama, who as the Buddha
28. b Option (b) gives the correct spellings. Gautama founded the Buddhist tradition. The Buddha
lived between roughly 563 and 483 BCE. Lumbini is
29. a Option (a) gives the correct spellings. one of four magnets for pilgrimage that sprang up in
places pivotal to the life of the Buddha, the others
30. c Option (c) gives the correct spellings. being at Kushinagar, Bodh Gaya and Sarnath.
31. c Lexicon is a dictionary, especially Greek, Hebrew, 47. b A galvanometer is a type of ammeter: an instrument
Arabic etc. for detecting and measuring electric current.
32. d Hex is a cast or spell, a curse. 48. a The Mudrarakshasa (“The Signet of the Minister”) is a
historical play in Sanskrit by Vishakhadatta that nar-
33. b Seminary is a college that prepares students to be rates the ascent of the king Chandragupta Maurya to
priests, ministers etc. power in Northern India. It is dated variously from the
late 4th century[1] to the 8th century.
34. c Litergy (also spelt as liturgy) is a form of religious rite
prescribed for public worship. 49. c The 6th Asian Games were held from December 9,
1970 to December 20, 1970 in Bangkok, Thailand.
35. d Laity refers to lay people i.e. ordinary people, different
from the clergy. Thus, church members who are 50. b The year 1929 is known for congress resolution for
not ordained priests (clergy) constitute the laity. complete independence of India.

Previous Years
Page 22 CLAT & AILET Papers
51. a INS Airavat (L24) is the third Shardul class amphibious 71. b 20 to 25% is the limit which was announced by Jairam
warfare vessel of the Indian Navy and was built by Ramesh that India would work for voluntary reduction
Garden Reach Shipbuilders & Engineers in Kolkata at before Indian team left for Kopen Hagen Conference.
Yard 3016
72. d Traditionally, landmasses completely or mostly sur-
52. a 53. d 54. c rounded by water are classified as continents (when
they are large enough) or islands. According to this
55. d Muridke was not the base of L.T.T.E. before being classification, mainland Australia (7.6 million km2) is
caputured b Srilankan Forces. the smallest continent and Greenland (2.1 million km2)
is the largest island.
56. b
73. c Indo-Pak 2001 summit was held in Agra.
57. d Carl Bildt is the Foreign Minister of European Union.
74. d In 2009, Sultan Azlan Shah Chaimponship (Hockey)
58. b Indian team defeated Malaysian team.

59. c Raj Babbar belongs to Congress Party. 75. a Tripura is a state in Northeast India, and one of the
Seven Sister States. The third-smallest state of India,
60. b 61. b 62. c 63. b it occupies an area of 4,051 sq mi (10,490 sq.km).
Tripura is surrounded by Bangladesh on the north,
64. a Mohmmad Naseed is the incumbent President of south, and west. The Indian states of Assam and
Maldives. Mizoram lie to its east. The state capital is Agartala. A
landlocked state, Tripura’s geography is character-
65. d Arvinda Rajkhwa is a member of United Liberation ized by several north-south hill ranges with interven-
Front of Assam (ULFA). ing valleys, and plain in the western part of the state.

66. c The breakthrough in the ideas of Darwin on the evolu- 76. b Kesari is a newspaper founded in 1881 by
tion of species came after his visit to Galapagos, which LokmanyaBalGangadharTilak, a prominent leader of
is a cluster of islands. Wherein, he saw that each the Indian Independence movement. Bal
island supported its own form of finch. GangadharTilak used to run his two newspapers, the
Kesari, in Marathi and Maratha in English from Kesari
67. d Gandhi was nominated in 1937, 1938, 1939, 1947 Wada. The Wada still has the offices of Kesari, and
and, finally, a few days before he was murdered in mementos of Tilak, including his writing desk original
January 1948. The omission has been publicly regret- letters and documents, and the first India national flag
ted by later members of the Nobel Committee; when unfurled by Madame Cama.
the Dalai Lama was awarded the Peace Prize in 1989,
the chairman of the committee said that this was “in 77. c Jahangir (the mughal emperor) gave permission to
part a tribute to the memory of Mahatma Gandhi”. Hawkins (ambassador of King James) to set up a
factory at Surat 1612.
68. b
78. c The Group of Eight (G8) is a forum for the govern-
69. b The University of Mumbai (informally known as Mumbai ments of eight of the world’s largest economies. (It
University or MU) is a public state university located in excludes, however, two of the actual eight largest
Mumbai, Maharashtra, India. It was known as the Uni- economies by nominal GDP: China, 2nd, and Brazil,
versity of Bombay until 1996 when the city of Bombay 6th). The forum originated with a 1975 summit hosted
was renamed as Mumbai. University of Mumbai was by France that brought together representatives of
established in 1857 consequent upon “Wood’s Edu- six governments: France, Germany, Italy, Japan, the
cation Dispatch”, and is one amongst the first three United Kingdom, and the United States, thus leading to
Universities in India. the name Group of Six or G6. The summit became
known as the Group of Seven or G7 the following
70. c Harare (known as Salisbury before 1982) is the larg- year with the addition of Canada. In 1997, Russia was
est city and capital of Zimbabwe. It has an estimated added to group which then became known as the G8.
population of 1,606,000 (2009), with 2,800,000 in its
metropolitan area (2006). Administratively, Harare is 79. b Jalal-ud-Din Muhammad Akbar, also known as
an independent city equivalent to a province. It is Shahanshah Akbar-e-Azam or Akbar the Great (14
Zimbabwe’s largest city and its administrative, com- October 1542 – 27 October 1605), was the third
mercial, and communications centre. The city is a trade Mughal Emperor. He was of Timurid descent; the son
centre for tobacco, maize, cotton, and citrus fruits. of Emperor Humayun, and the grandson of the Mughal
Manufactures include textiles, steel, and chemicals, Emperor Zaheeruddin Muhammad Babur, the ruler who
and gold is mined in the area. founded the Mughal dynasty in India. At the end of his
reign in 1605 the Mughalempire covered most of north-

Previous Years
CLAT & AILET Papers Page 23
ern and central India. He is most appreciated for hav- In the 1970s, the Malaysian government implemented
ing a liberal outlook on all faiths and beliefs and during policies which The Economist called “racially discrimi-
his era, culture and art reached a zenith as compared natory” designed to favour bumiputras (including af-
to his predecessors. firmative action in public education) to create opportu-
nities, and to defuse inter-ethnic tensions following
80. b Shah Jahan was born as Prince Shihab-ud-din the extended violence against Chinese Malaysians in
Muhammad Khurram, in 1592 in Lahore, Pakistan as the 13 May Incident in 1969. These policies have suc-
the third and favourite son of the emperor Jahangir ceeded in creating a significant urban Malay middle
from his Rajput wife Gossaini. The name Khurram- class. They have been less effective in eradicating
Persian for ‘joyful’-was given by his grandfather Akbar. poverty among rural communities. Some analysts have
His early years saw him receive a cultured, broad noted a backlash of resentment from excluded groups,
education and he distinguished himself in the martial in particular the sizeable Chinese and Indian Malay-
arts and as a military commander while leading his sian minorities.
father’s armies in numerous campaigns against Mewar
(1615), the Deccan Sultanates (1617 and 1621), 87. b Premchand who was born as Dhanpat Rai was an
Kangra Fort (1618). Indian writer famous for his modern Hindi-Urdu litera-
ture. He is one of the most celebrated writers of the
81. d Products made from neem trees have been used in Indian subcontinent, and is regarded as one of the
India for over two millennia for their medicinal proper- foremost Hindi-Urdu writers of the early twentieth
ties: neem products are believed to be anthelmintic, century.
antifungal, antidiabetic, antibacterial and antiviral. Neem
products are also used in selectively controlling pests 88. d The Old Man and the Sea is a novel written by the
in plants. It is considered a major component in American author Ernest Hemingway in 1951 in Cuba,
Ayurvedic and Unani medicine and is particularly pre- and published in 1952. It was awarded the Pulitzer
scribed for skin disease. Prize for Fiction in 1953 and was cited by the Nobel
Committee as contributing to the awarding of the Nobel
82. a Responsibility of executive to legislature is the prin- Prize in Literature to Hemingway in 1954.
ciple on which parliamentary democracy operates.
89. a Muhammad (c. 570 – c. 8 June 632 was a leader from
83. d Björn Rune Borg ( born 6 June 1956) is a former world Mecca who unified Arabia into a single religious polity
no. 1 tennis player from Sweden. Between 1974 and under Islam. He is believed by Muslims and Bahá’ís to
1981 he won 11 Grand Slam singles titles. He won be a messenger and prophet of God, and by most
five consecutive Wimbledon singles titles (a record Muslims as the last prophet sent by God for mankind.
equalled by Roger Federer) and six French Open Muhammad is generally considered to be the founder
singles titles (a record broken by Rafael Nadal). He is of Islam, although this is a view not shared by Mus-
considered by many to be one of the greatest tennis lims. Muslims consider him to be the restorer of an
players of all time. uncorrupted original monotheistic faith of Adam, Noah,
Abraham, Moses, Jesus and other prophets.
84. a Vande Mataram (“I bow to thee, Mother”) is a poem
from the famed novel Anandamath which was written 90. a World War I (WWI) was a global war centred in Eu-
by Bankim Chandra Chattopadhyay in 1882. It was rope that began on 28 July 1914 and lasted until 11
written in Bengali and Sanskrit. In 1950 (after India’s November 1918. It was predominantly called the World
independence), the song’s first two verses were given War or the Great War from its occurrence until the
the official status of the “national song” of the Repub- start of World War II in 1939, and the First World War
lic of India, distinct from the national anthem of India, or World War I thereafter. It involved all the world’s
Jana GanaMana. great powers, which were assembled in two oppos-
ing alliances: the Allies (based on the Triple Entente of
85. c “Sick man of Europe” is a nickname that has been the United Kingdom, France and Russia) and the Cen-
used to describe a European country experiencing a tral Powers (centred around the Triple Alliance of
time of economic difficulty and/or impoverishment. The Germany, Austria-Hungary and Italy)
term was first used in the mid-19th century to de-
scribe the Ottoman Empire (Modern Turkey) but has 91. c absolute liability even without any negligence or fault.
since been applied at one time or another to nearly The principle of “No fault liability” is encapsulated under
every other mid-to-large-sized country in Europe. Section 140 of the Motor Vehicles Act 1988 which
provides for liability of the owner of the Motor Vehicles
86. a Bumiputera or Bumiputra is a Malaysian term to de- to pay compensation in certain cases, on the principle
scribe Malay race and the indigenous peoples of South- of no fault. This means that the claimant need not
east Asia, and particularly in Malaysia. The term comes prove negligence on the part of the motorist. Liability is
from the Sanskrit word bhumiputra, which can be automatic.
translated literally as “prince of land” (bhumi= earth or
land, putra=prince).

Previous Years
Page 24 CLAT & AILET Papers
92. a liability on property 102. c right to property
An encumbrance means a legal liability on property. It The Forty-Forth Amendment of 1978 deleted the right
constitutes a burden on the title which diminishes the to property from the list of fundamental rights. A new
value of the land. It may be a mortgage or a deed of provision, Article 300-A, was added to the constitution
trust or a lien of an easement. An encumbrance, thus, which provided that “no person shall be deprived of
must be a charge on the property. It must run with the his property save by authority of law”. Thus, the right
property. to property is no longer a fundamental right, though it
is still a constitutional right.
93. d conjugal right
Conjugal means belonging to marriage. Conjugal rights 103. a male and a Christian female
include fellowship of husband and wife, and the right The Quran explicitly allows Muslim men to marry chaste
of each to the company, society, co-operation, women of the People of the Book, a term which
affection and aid of the other. includes Jews and Christians.

94. b parole 104. c Under the United States Constitution, the power of
Parole is a legal sanction that lets a prisoner leave the judicial review i.e subjecting the actions of the
prison for a short duration, on the condition that s/he legislature and the executive to the scrutiny of the
behaves appropriately after release and reports back Judiciary, is not explicit and has been inferred from
to the prison on termination of the parole period. While the structure, provisions, and history of the
parole is granted to a prisoner detained for any offence Constitution. After the Marbury vs. Madison case
irrespective of the duration of imprisonment. judicial review was exercised by the American
judiciary under Article III of the constitution of America.
95. c conviction
In criminal law, conviction is the result of a criminal trial 105. b The part of the judgment which possesses authority
which ends in a judgment or sentence that the accused is called Ratio Decidendi. It is the rationale for a decision
is guilty as charged whereas an acquittal formally passed by the Court and as a general rule, binding on
certifies the accused is free from the charge of an courts of lower and later jurisdiction unlike obiter dicta
offense. which is a remark or an observation make by the
Judge which does not necessarily form part of the
96. c Mutawalli Court’s decision.
The term waqf literally means “confinement and
106. a division of powers between the central and provincial
prohibition” or causing a thing to stop or standstill. The
government
legal meaning of Waqf is the detention of specific
The Act, passed during British India, provided a dual
thing in the ownership of waqif and the devoting of its
form of government (a “dyarchy”) for the major
profit or products in charity of poors or other good
provinces. In each such province, control of some
objects. A person,who manages the Waqf property is
areas of government were given to the Provincial
called “Mutawalli”. The legal position of Mutawalli is
Council. All other areas of government remained under
that of the Manager or Superintendent of the Waqf
the control of the Viceroy.
properties.
107. a paid by an employer in respect of the fringe benefits
97. b The given answer is C but it should be B. Ipso facto provided or deemed to have been provided
means, by reason of that fact. Employees get many benefits at the cost of the
98. c taking control of property temporarily employers. Where the benefits are usually enjoyed
Blacks Law Dictionary defines Requisition as a demand collectively by the employees and cannot be attributed
in writing, or formal request or requirement. The taking to individual employees, they shall be taxed in the
or seizure of property by government. hands of the employer, and this tax is called Fringe
Benefit Tax. An employer is liable for fringe benefit tax
99. b The answer should be (b) instead of (C). Corroborative even if they do not have any income which is
evidence means those evidences which supports chargeable to the income tax. The fringe benefit tax
other evidence already taken on record by the court. was abolished in the 2009.
100. d without hearing the opponent 108. b The court may order the transfer of the ownership of
ex parte means a legal proceeding brought by one the property.
person in the absence of and without representation
or notification of other parties. 109. d Ram can never be a prosecutor because in criminal
cases prosecution is always brought on behalf of the
101. b British state.
A unitary system is governed constitutionally as one
single unit, with one constitutionally created legislature. 110. d The Code of Criminal Procedure is the main legislation
The UK is an example of a unitary system. on procedure for administration of substantive criminal
law in India.

Previous Years
CLAT & AILET Papers Page 25
111. c Forensic science supposed to be directly or indirectly concerned in or
Forensic Science is the application of a broad privy to an offence to whom a pardon is granted under
spectrum of sciences to answer questions of interest Section 306 of the Code with a view to securing his
to a legal system. This may be in relation to a crime or testimony against other persons guilty of the offence.
a civil action.
121. d all the elected members of both the Houses of
112. a a judge other than a Chief Justice Parliament and all the elected members of all the
The word puisne means junior and is used to Legislative Assemblies
distinguish High Court judges from senior judges sitting The President is elected, from a group of nominees,
at the Court of Appeal. The term is only used in relation by the elected members of the Parliament of India (Lok
to judges of a superior court, e.g., the Supreme Court Sabha and Rajya Sabha) as well as of the state
or the High Court of a jurisdiction. The equivalent term legislatures (Vidhan Sabhas) for a term of 5 years.
in the United States is Associate Justice. Whenever the office becomes vacant, the new
President is chosen by an electoral college consisting
113. a within the powers of the elected members of both houses of Parliament,
It is a latin term which implies “within the powers” as the elected members of the State Legislative
opposed to Ultra vires which means “beyond the Assemblies (Vidhan Sabha) and the elected members
powers” If an act requires legal authority and it is of the legislative assemblies of the Union Territories of
done with such authority, it is characterised in law as Delhi and Pondicherry.
intra vires If it is done without such authority, it is ultra
vires. 122. d right to work
The Indian constitution refers to the right to work under
114. b Only X committed the offence of theft. the “directive principles of state policy”. Article 39
urges the State to ensure that “the citizens, men and
115. b legal problems in the form of imaginary cases, argued women equally, have the right to an adequate means
by two opposing students before a bench pretending to livelihood”, and that “there is equal pay for equal
to be a real court. A moot court is an extracurricular work for both men and women. Further, Article 41
activity at law schools in which participants take part stresses that “the state, shall within the limits of its
in hypothetical court proceedings, which usually economic capacity and development, make effective
involves drafting briefs (or memorials) and participating provision for securing right to work...”
in oral argument.
123. b Geneva
116. b religiously neutral The International Labour Organization (ILO) is an
The Scheduled Castes (SCs), also known as the Dalit, agency of the United Nations that deals with labour
and the Scheduled Tribes (STs) are two groupings of issues pertaining to international labour standards and
historically disadvantaged people that are given decent work for all. Its headquarters are in Geneva,
express recognition in the Constitution of India are not Switzerland.
based on any religion.
124. c all irrespective of religion
117. a Justice Gyan Sudha Mishra The Act applies to the citizens per se and its applicability
The first woman to be appointed to the Supreme Court is not restricted to any religion.
was Justice Fatima Beevi in 1987. She was later
followed by Justices Sujata Manohar(1994), Ruma 125. a Not bound to pay as he did not ask the shopkeeper to
Pal (2000) and Gyan Sudha Mishra(2010). deliver the bag.
The action of leaving the flour bag at the doorstep of Y
118. c movable property was uncalled for. Besides, the message that X leaves
An article of personal property; any species of property with the bag does not constitute any offer but is a
not amounting to a freehold or fee in land. All property mere statement. Usage of flour by Y does not amount
which is not real estate is said to be chattel. to any acceptance or a counter offer. Y is not bound
to pay anything to X. This could be compared with
119. d plaintiff, defendant sample sachets that are given with a newspaper/
Plaintiff is the term used for the party who initiates a magazine copy. Mere usage of those samples by the
law suit. Defendant is the party who has been formally consumer does not bind him to pay to the provider,
charged or accused by the plaintiff. The party who irrespective of any message that is given with the
appeals a court’s decision is called the Appellant. sample.
Accused is the person who is charged with a crime.
A person who initiates legal proceedings against 126. b Kerala High Court
someone is called a prosecutor. The state of Kerala was formed by the States
Reorganisation Act, 1956. which also created the
120. c approver Kerala High Court. The Act also extended the jurisdiction
The term “approver” is usually applied to a person, of the Kerala High Court to Lakshadweep.

Previous Years
Page 26 CLAT & AILET Papers
127. d Decides international crimes 139. b Ahmed is healthy because of his mother's care.
ICJ is the primary judicial organ of the United Nations. Therefore, (b) is the correct option.
Its main functions are to settle legal disputes submitted
to it by states and to provide advisory opinions on 140. b Rate of crime is low in the city because of the
legal questions submitted to it by duly authorized efficiency of the police. Therefore, (b) is the correct
international organs, agencies, and the UN General option.
Assembly.
141. a Assumption I follows as that is the condition for starting
128. b Banks were nationalised under a law during the Prime the activities. ‘Reading the instructions’ assumes that
Ministership of Mrs. Indira Gandhi. R.C.Cooper Vs. they are understood as well. Assumption II does not
Union of India (1970) (the Bank Nationalisation Case). follow, as 'beginning the activity accordingly' may not
mean 'would be able to act accordingly'. Therefore,
129. d Property of an intellectual (a) is the correct option.
Rest options are a form of intellectual property which
is given protection under various laws. 142. a Assumption I follows as children do not generally know
what is crime. Assumption II is tenuous. Therefore, (a)
130. a The multinational corporation is the correct option.
The Competition Act, 2002 was introduced, inter alia,
to repeal MRTP Act and prevent activities that have an 143. d None of the Assumptions follow as right to reject entails
adverse effect on competition in India. an employer to reject for any reason. So 'impartiality
or eligibility' cannot be assumed to be a factor.
131. a Justice R.C. Lahoti Therefore, (d) is the correct option.
Law Commission of India is an executive body
established by an order of the Government of India. Its 144. d Governments' decision to reduce custom duty may
major function is to work for legal reform. Justice B. P. lead to the outcome mentioned in I or II. But
Jeevan Reddy was the Chairman of the Commission Government's decision may not have been based on
from 2000-01 followed by Justice M. Jagannadha Rao these assumptions. Therefore, (d) is the correct
who served as the Chairman of the Commission until option.
2007 after which Justice A. R. Lakshmanan took over.
145. d Both the statements are far fetched. Therefore, (d) is
132. a Wajahat Habibullah was the first chief Information the correct option.
Commissioner of India.
146. b Stude nt City He ight Ge nde r
133. d R.T.I. stands for Right to information A Mumbai Short Girl
B Mumbai Short Boy
134. b Trust C Delhi Short Girl
A fiduciary is someone who has undertaken to act for
D Delhi Tall Girl
and on behalf of another in a particular matter in
circumstances which give rise to a relationship of E Delhi Short Boy
trust and confidence. Such relationships could be F Delhi Tall Boy
termed as fiduciary relationships.
Hence, (b) is the correct option.
135. d 10th December
The date was chosen to honor the United Nations 147. c P, Q, R and S are siblings. Since, the gender of Q is not
General Assembly’s adoption and proclamation, on 10 clear, Q can be either brother or sister of S. Therefore,
December 1948, of the Universal Declaration of Human (c) is the correct option.
Rights (UDHR), the first global enunciation of human
rights and one of the first major achievements of the 148. c Lamb means young sheep.
new United Nations.
149. d Pyorrhea is a disease of teeth.
136. a Because of the action in 1st statement the effect in
the 2nd statement has taken place. Therefore, (a) is 150. a Blue-moon means something that happens in a long
the correct option. time. ‘Blue-black’ is a discoloration of the skin due to
coagulation of blood. Blue sheep means unreliable or
137. d Both the statements are unrelated i.e. are independent disreputable member in a family or society. Thus, all
effects of some other cause. the given color phrases are idiomatic expressions.

138. d Both incidents in statements I & II could be (could not 151. c The correct answer is (c) over here as we apply the
be) the result of same cause. Therefore, (d) is the principle directly and the above facts make it a case of
correct option. neighbour being interefered. Therefore the correct
option in (c).

Previous Years
CLAT & AILET Papers Page 27
152. d (d) is the correct option as X has a right and he may 167. b Anand > Gulab > Pramesh > Jairam > Mohan
opt to speak or not to speak. Thus, Anand is the heaviest.

153. c (c) is the correct answer and the Bank is not liable as 168. b Amit’s son’s brother is Amit’s son. Therefore, Amit is
there is no master servant relationship between the the father of Ravi.
Bank and Y. And Y collected the money not on the
advice or direction of the bank but for his own motive 169. c From CABLE, ABLE is coded as CDAY. Therefore,
and the bank does not share any responsibilty for the STABLE is coded as TPCDAY.
same
170. d From CHARTER, ARTER is coded as OEPYE. Therefore,
154. b (b) is correct as answer. PARTNER is coded as AOEPCYE.

155. d (d) is the correct answer as the accused entered the 171. a As a constituent assembly makes a constitution,
property not with an intention to commit a crime but similarly parliament formulates a statute, meaning a
came on the invitation that he recieved by the girl. In permanent rule made by a body or institution of the
this case since the intention is not to commit the crime government.
so he is not liable for any offence including the criminal
trepass. 172. d As a right brings along with it a duty, similarly power
brings along liabilities.
156. a From option (a), the code for Ram is Ten i.e., R = T, A =
E and M = N. Therefore, the code for Kumar should be 173. c The young one of an elephant is called a calf, likewise,
_ _ net. Option (a) satisfies this. Hence, the code for cub is the young one of tiger.
Ram Kumar is Ten Ronet.
174. d As a patient approaches a doctor for help, similarly a
157. d Except 214, all other are odd numbers. litigant, which is a party involved in a law suit,
approaches a lawyer.
158. d Spinach is the only leafy vegetable of the group. Rest
all roots. 175. b As a prosecutor is opposed to an accused; plaintiff,
the complainant, is opposed to the defendant which is
159. a ‘House’ is the odd one out, rest three terms are technical the person being sued.
financial terms.
176. a The head of Lok Sabha is the Speaker, currently Ms.
160. c ‘Morality’ is a system of ideas of right and wrong Meera Kumar. The chairperson of Rajya Sabha is
conduct. Law, court and judge are all legal the Vice-President of the country, currently Mr. Hameed
terms. Ansari.

161. d The first three are fundamental rights, right to make a 177. d The minimum age for becoming the president of India
contract is not. is 35 years, similarly, the minimum age for becoming a
member of Rajya Sabha is 30 years.
162. c Both are strong from two different perspectives.
Therefore, (c) is the correct option. 178. b India follows a parliamentary system while the US
follows presidential system.
163. b I is wrong as we do not know if Indian Railway is non
profitable. Argument II is strong. Therefore, (b) is the 179. b The real executive power lies with the President,
correct option. similarly judiciary is headed by the Chief Justice.

164. c Both are strong from two different perspectives. 180. c The United Nations was instituted after the World War
Therefore, (c) is the correct option. II. League of Nations was instituted after World War
I.
165. d Even if only law can solve a problem then also it is not 181. b 289 = 17 × 17 = 17.
certain that the problem will be solved (use of
necessary condition). Argument II is weak since it
doesn’t give any reason. It just states the benefit of 182. d 330 = 2 × 3 × 5 × 11.
solving the problem rather than answering the
question. Therefore, (d) is the correct option. 183. b 1122 = 2 × 3 × 11 × 17.

166. b Only II is very strong as it justify the necessity of such 184. d 21 is divisible by 3 and 7. However, all the other numbers
right. I doesn’t give any substantial reason for not are divisible by 1 and the number itself. Therefore, 21
making it a fundamental right. Therefore, (b) is the is not a prime number.
correct option.

Previous Years
Page 28 CLAT & AILET Papers
185. c All the even numbers from 1 to 24 are divisible by 2, 4
i.e. 24, 22, 20, 18, 16, 14, 12, 10, 8, 6, 4 and 2. greater than the denominator only in case of . Thus,
3
∴16 will be at the 8th place from the bottom.
4
has the highest value.
186. a Average age of 10 persons = 32 years 3
Total age of 10 persons = 32 × 10 = 320 years
Average age of 10 persons and instructor 195. d In both the cases, one gardener is using one grass
mower.
= 34 years
Total age of 10 persons and instructor
We know,
(M1 × H1 ) = (M2 × H2 )
= 34 × 11 = 374 years
W1 W2
∴ Age of the instructor = 374 – 320 = 54 years.


(4 × 4 ) = (8 × H2 )
187. b 12% of 5000
400 800
12 ⇒ H2 = 4 hours.
= × 5000 = 12 × 50 = 600.
100
196. a From the natural numbers written in ascending order,
the numbers taken out are 1, 5, 10, 16, ... These
188. c 400 = 20 × 20 = 20.
numbers follow the following pattern:

189. a 3254710 1 5 10 16 (23)


= 3000000 + 200000 + 50000 + 4000 + 700 + 10 + 0
4 5 6 7
∴ Place value of 5 is 50,000.
Thus, the required term is 22.
190. b 6 × 3(3 – 1) or
= 6 × 3(2) = 6 × 6 = 36. (2, 3, 4), (6, 7, 8, 9), (11, 12, 13, 14, 15),
(17, 18, 19, 20, 21, 22).

191. c 197. c The series is:


3 km 117 = 13 × 9
A 104 = 13 × 8
5 km
91 = 13 × 7
78 = 13 × 6
65 = 13 × 5
1 8 km 1 5 km Thus, the required term is 65.

198. d The series is:


88 = 8 × 11
5 km 96 = 8 × 12
As both the starting point and the finishing point of ‘Y’ 104 = 8 × 13
is A, therefore, ‘Y’ is 0 km away from his starting point. 112 = 8 × 14
Thus, the required term is 112.
192. c The series is:
9=3×3 P×r × t
81 = 9 × 9 199. b S.I. = , where P is principal, r is rate of interest
100
6561 = 81 × 81
and t is time.
Thus, the missing term is 81.
1 13
193. a The series is: S.I. = 520 × × = Rs. 33.80
2 100
100 100 100 100 100 Total amount to be paid
, , , , = Rs. 520 + Rs. 33.80 = Rs. 553.80.
1 2 3 4 5
i.e., 100, 50, 33.33, 25, 20.
200. a From the options given:
Thus, the missing term is 25.
20 is divisible by 2 and 5,
21 is divisible by 3 and 7,
3 4 2 1 22 is divisible by 2 and 11,
194. b = 0.6 , = 1.33 , = 0.4 and = 0.5 19 is divisible by only 1 and itself.
5 3 5 2
Thus, 19 is a prime number.
Also, it is clear from the options that the numerator is

Previous Years
CLAT & AILET Papers Page 29
CLAT Solutions 2011
1. b Option (a) is incorrect as P.C. Mahalanobis believed in 11. d The passage talks about the strangeness in the non-
rigid control by the government over all activities. Dalit representations of Dalits. He mentions two books,
Option (c) is incorrect, as the effect of restrictions on one by Manu Joseph and the other by Rohinton Mistry.
large companies is not given in the passage. Option Both these books are written by non- Dalits and both
(d) is incorrect, as Spratt’s views on the ‘devaluation talk about Dalits without having any insight into the
of the rupee’ are not given. Option (b) can be inferred Dalit world. The non-Dalit portrayal of Dalits in literature,
from the lines “…. Philip Spratt was writing a series of cinema and art has remained a norm in India but theses
essays in favor of free enterprise…”. portrays fail to depict the truth about Dalit lives, their
movements and figures like B.R. Ambedkar. Thus,
2. a Option (a) cannot be inferred from the passage. Option option (d) is the correct choice.
(b) can be inferred from the first paragraph. Refer to
lines “…..Shroff began a forum…government, of 12. a Refer to the lines, “Mistry seems to have not veered
India….”. Option (c) can be inferred as he championed too far from the road charted by predecessors like
the cause of independent entrepreneur. Option (d) is Mulk Raj Anand and Premchand. Sixty years after
indicated in the lines “Spratt was a Cambridge Premchand, Mistry’s literary imagination seems stuck
communist”. Hence, the correct answer is option (a). in the empathy-realism mode, trapping Dalits in
abjection.” Hence, option (a) is the correct choice.
3. a Option (a) is correct as the author has tried to highlight
the presence of advocates for free market by first 13. d Refer to this line in thrid paragraph, “In a society that is
mentioning Shroff and then Spratt. Option (b) is incorrect still largely unwilling to recognize Dalits as
as the mention of Indira Gandhi and related politics is equal…..representation of Dalits.” The correct answer
limited to only the last paragraph. Options (c) and (d) is option (d).
are beyond the scope of the passage.
14. c Refer to this line of third paragraph, “…we must
4. d The correct answer is option (d). Refer to the line, engage with what Dalits are writing –not because for
“Spratt was a……revolution in the subcontinent.” reasons of authenticity, or as a concession to identity
politics, but simply because of the aesthetic value of
5. c The correct answer is option (c). Refer to the first this body of writing and for insights it offers into the
paragraph. Shroff complained that independent human condition.” Thus, option (c) cannot be inferred.
entrepreneurs were treated with indifference by the
government. 15. d Refer to last line of the second paragraph – “…..and
the continued and unquestioned predominance of a
6. c The correct answer is option (c). Refer to this line in Brahminical stranglehold over cultural production have
second paragraph. “The books he read in the prison, led us to a place where non-Dalit portrayal of Dalits in
and his marriage to an Indian woman afterwards, literature, cinema and art remains the norm.” Hence,
inspired a steady move rightwards.” option (d) is correct.

7. d The correct answer is option (d). Refer to the last 16. c Option (c) has not been suggested in the passage;
lines of the last paragraph - “After the rupee was rest all three options are mentioned. Refer to the lines
devalued…. and returning to economic autarky”. “…we must engage with what Dalits are writing –not
because for reasons of authenticity, or as a
8. b Nationalization of industries in 1969 has been quoted concession to identity politics, but simply because of
by the author as a consequence of ‘Indira Gandhi led the aesthetic value of this body of writing and for
politics’. Hence, correct answer is option (b). insights it offers into the human condition.” “Mistry
seems to have not veered too far from the road charted
9. c The correct answer is ‘was’. In the given sentence, by predecessors like Mulk Raj Anand and Premchand.
verb should agree with the closest available subject. Sixty years after Premchand, Mistry’s literary
Since A. D. Shroff is singular, hence, both ‘is’ and imagination seems stuck in the empathy-realism mode,
‘was’ are correct. In light of the given passage, ‘was’ trapping Dalits in abjection. Mistry happily continues
becomes more appropriate as we are referring to the the broad stereotype of the Dalit as a passive sufferer,
past. without consciousness of caste politics.”

10. d ‘Inveigh’ means to criticize strongly or complain bitterly. 17. d Option (d) is neither mentioned nor implied in the
The correct answer is ‘remonstrate’ which means to passage and hence is the correct answer.
protest or complain about something/somebody.
18. b The author has criticized Rohinton Mistry for presenting
Dalits in the same stereotyped image that was created

Previous Years
Page 30 CLAT & AILET Papers
by Premchand and Mulk Raj Anand. Dalits have been 27. d The author has referred to the history of various
perceived as passive sufferers. Refer to the lines religions living peaceably together in India in order to
“Mistry seems to have not veered too far from the show its openness to unique cultural identities. Hence,
road charted by predecessors like Mulk Raj Anand the correct answer choice is option (d).
and Premchand. Sixty years after Premchand, Mistry’s
literary imagination seems stuck in the empathy-realism 28. d According to the author, the criticism directed at these
mode, trapping Dalits in abjection. Mistry happily foreign authors has become more important than the
continues the broad stereotype of the Dalit as a passive book. This criticism is about the foreignness of these
sufferer, without consciousness of caste politics.” authors and not about the content of their books. Refer
Hence, option (b) is the correct choice. to the lines “What is more interesting about these
appraisals…not what has been written. Hence, the
19. c ‘Sly’ means ‘devious’ in the passage. Option (c) is correct answer is option (d).
correct.
29. b Refer to the second paragraph. The debate of ‘who
20. c As the subject is plural (Dalit movements) and the gets to write about India’ has triggered another debate
sentence is in past tense, the verb should be ‘were’ which is about who shall decide that ‘who gets to
(plural verb). write about India’. Hence, the correct answer choice
is option (b).
21. d Refer to lines “The book is not necessary in these
cases, for the argument is about who can write about 30. a The correct answer is option (a). The subject
India, not what has been written.” “In recent ‘sensitivity’ is singular in number and the sentence is
weeks,…not their interpretations, but their in present tense. Usage of present perfect tense is
foreignness.” The author states that the criticism appropriate for the given blank. Therefore, ‘has’ is the
directed is mostly about the foreignness of authors apt verb.
rather than about their literary styles. Hence, the
correct answer is option (d). 31. d Refer to the penultimate and the last paragraphs. The
author has given several examples to emphasize how
22. a ‘Fusillade’ refers to a spirited outburst of criticism; a poverty and starvation do not always result in revolt
number of shots fired simultaneously or in rapid and how the link is not necessary. The correct answer
succession. ‘Barrage’ refers to a rapid or is option (d).
overwhelming outpouring of many things at once. “The
teacher’s rapid-fire barrage of homework 32. b Refer to second paragraph. “Part of the difficulty lies
assignments went by too fast for me to write them all in the possibility…the declared rationale of public
down”. Therefore, among the given options, barrage is commitment to remove poverty”. Option (c)
the most suitable substitute for fusillade. is incorrect as the passage does not definitely state
that there is no connection between poverty and
23. b Refer to paragraphs 1 and 6. Both talk about critics violence. The correct answer is option (b).
who have attacked foreign authors not for their literary
styles but for their foreignness. Hence, the correct 33. d The author stresses that policies of poverty removal
answer is option (b). should not be framed and justified on the premise that
they are needed to avoid violence and strife as this
24. c Refer to lines “The book is not necessary in these notion has may shortcomings. Option (d) expresses
cases, for the argument is about who can write about the main idea of the passage effectively.
India, not what has been written.” “In recent
weeks,…not their interpretations, but their 34. d ‘Perilous’ means involving potential loss or injury. Hence,
foreignness.” The author states that the criticism ‘dangerous’ is the correct choice.
directed is mostly about the foreignness of authors
rather than about their literary styles. Option (c) is 35. c The author states that it is not surprising that often
correct. enough intense and widespread suffering and misery
have been accompanied by unusual peace and silence.
25. d Refer to paragraph 5 – “A country with a millennial He/She then gives the example of famines of Ireland
history of Hindus, Christians, Jews…..I have not heard and Calcutta to substantiate his/her assertion. Thus,
an Indian ask whether outsiders have a right to write, option (c) is the correct choice. Option (d) is close but
think or exist on their soil.” The correct answer is is narrow as it focuses only on people who are dying
option (d). of starvation and fails to convey the broader idea of the
author that deals with suffering and misery in general.
26. d The author says that in today’s India, outside of the
elite intelligentsia, people think that a person should be 36. b ‘Destitution’ refers to the state of lack of sufficient
awarded for what he does and not for who he is. money or material possessions. ‘Indigence’ is a state
Hence, the correct answer is option (d). of extreme poverty. Therefore, option (b) is the correct
choice.

Previous Years
CLAT & AILET Papers Page 31
37. c Option (a) can be inferred from the last paragraph. 47. b Jasmine revolution or the Tunisian revolution which
Option (b) can be inferred from the first paragraph, occurred in the year 2011 in which in which President
“There has, in fact been,…some evident attractions.” Zine El Abidine Ben Ali was forced out of the
Refer to the second paragraph, “This is not to deny presidency by popular protests, Currently, Moncef
that poverty and inequality can- and do-have far Marzouki holds the presidency in Tunisia.
reaching consequences with conflict and strife, but
these connections have to be examined and 48. a The Naïve and the Sentimental Novelist’ has been
investigated with appropriate care and empirical written by Orhan Pamuk.
scrutiny, rather than being casually invoked with
unreasoned rapidity in support of a good cause.” The 49. a Rajiv Shukla is the incumbent chief of Indian Premier
author does not imply that links between poverty and League (IPL).
violence must never be emphasized but advocates
the need for prudence and empirical scrutiny before 50. a The movie ‘Uddan’ is directed by Mr. Vikramaditya
linking them. Thus, option (c) cannot be inferred. Option Motwane.
(d) can be inferred from the lines “Destitution can be
accompanied not only by economic debility, but also 51. b Rani Kumudini Devi was the first woman barrister of
by political helplessness.” India.

38. c Refer to the lines “Basing public policy…rather than 52. c James Laine authored the book Shivaji - The Hindu
moral, relevance.” Thus option (c) is correct. King in Muslim India.

39. c Refer to the second paragraph of the passage. As 53. a The Kherlanji massacre refers to the 2006 lynching-
per the reference to economic reductionism in the style murders of a Dalit family by members of the
passage, option (c) states the meaning implied by the politically dominant Kunbi “OBC “. The killings took place
term effectively. in a small village in India named Khairlanji, located in
the Bhandara district of the state of Maharashtra.
40. d As ‘mobilize’ is a verb, the preposition ‘to’ is appropriate.
‘Into’ is incorrect because it is used to show movement 54. a The Shunglu panel was constituted to probe the
from exterior to interior position/location. It is used to Commonwealth Games Scam.
show the state of movement; for example: I am going
into the class. The phrase ‘easier to mobilize’ is 55. d National Innovation Council has to prepare a road map
grammatically correct. for the decade of 2011-2020'.

41. b Arundhati Roy investigated in the year 2011 for 56. d Nira Radia is currently investigated by Serious Fraud
sedition for saying that Kashmir is not an integral part Investigation Office of Ministry of Corporate Affairs
of India. Section 124 of Indian Penal Code 1860 makes for 2-G spectrum scam.
such a kind of act punishable.
57. d Irom Sharmila has been fasting for the last 10 years to
42. a For Man Booker Prize Damon Galgut’s ‘In a Strange protest against Application of the Armed Forces
Room” was short-listed in the year 2011. (Special Powers) Act 1958 in Manipur.

43. a Aruna Shanbaug is a nurse from Haldipur, Uttar 58. c Thein Sein is the president of Burma(Mayanmar).
Kannada, Karnataka. On 24th January 2011, after she
had been in this status for 37 years, the Supreme Court 59. d Baglihar dam is on the river Chenab in the state of
of India responded to the plea for euthanasia filed by Jammu and Kashmir it has been the matter of
Aruna’s friend journalist Pinki Virani, by setting up a controversy between India and Pakistan as Pakistan
medical panel to examine her. The court turned down claimed that design parameters of Baglihar project
the mercy killing petition on. However in its landmark violated the Indus Water Treaty.
judgment, it allowed passive euthanasia in India.
60. d TINDERBOX - The Past and Future of Pakistan is the
44. c Nagoya Protocol, signed by India on 30th October, name of the book written by M.J. Akbar who is a
2010 is an international treaty to ensure that the benefits leading journalist and editorial director of popular
of natural resources and their commercial derivatives magazine ‘India Today’.
are shared with local communities.
61. b BJP’s flag yatra was stopped at the Lakhampur border
45. a Julian Assange, the founder of WikiLeaks, was of Jammu and Kashmir
arrested in United Kingdom in the year 2011.
62. d P.C. Chacko chaired the Joint Parliamentary Committee
46. b Germany, Britain, Hungary, Qatar and Spain are the (JPC) on the 2G Spectrum allocation issue.
countries who have decided to bid for 2017 World
Athletics Championships. 63. a

Previous Years
Page 32 CLAT & AILET Papers
64. d Moner Manush is a Bengali feature film based on the 81. c Late Pandit Bhimsen Joshi belonged to Kirana Gharana
life and philosophy of Fakir Lalan Shah, a noted spiritual of Classical singing.
leader, poet and folk singer of Bengal in the 19th
century. The film is directed by Goutam Ghose. 82. b Alam Ara is a 1931 film directed by Ardeshir Irani. It
was the first Indian sound film.
65. b Justice P.C. Phukan Commission of Inquiry was
constituted to enquire into Clashes between Bodos 83. b Kenny G, collaboration with Rahul Sharma in the music
and Muslims in Northern Assam’s Udalguri district on album ‘Namste India’
August 14th 2008
84. d “War on Terrorism or American Strategy for Global
66. a Fathimath Dhiyana Saeed, from Maldives is the first Dominance” is written by Manzoor Alam.
woman Secretary General of SAARC.
85. c Courts of Arbitration for sports has headquarter in
67. b Under HD Deve Gowda ‘s premiership the Women’s Swizerland (Lausanni)
Reservations Bill (to secure quotas for women in
Parliament and state legislative assemblies) was first 86. c Karun Chandok is a Formula One driver.
introduced in Parliament.
87. c The United Nations Framework Convention on Climate
68. b Kevin O’ Brien from his country Ireland scored the Change meeting of 2010 December was held in
fastest Century in the history of World Cup Cricket. Cancun.

88. a Munammar Gaddafi is from Libya.


69. d 2-G Spectrum scam was brought from the Central
Vigilance Commission (CVC) action on the illegalities
89. c The Right of Children to Full and Compulsory Education
in the spectrum allocation.
Act 2009 requires private schools to ensure that 25
percent of their students come from weaker sections
70. d Gopa Sabharwal was appointed as the first Vice
and disadvantaged groups. Supreme Court has
Chancellor ofNalanda International University in the
recently upholded constitutionality of the
year 2011.
aforementioned act when a writ petition was filed
71. a Sadhvi Pragya Singh Thakur is an Indian woman challenging its’ constitutionality in the case Society for
accused of orchestrating the Malegaon blasts of 2008 Un-aided Private Schools of Rajasthan Vs. Union of
while Swami Aseemananda is associated with Ajmer India.
blast.
90. c Srikrishna Committee submitted its report in the year
72. a PJ Thomas headed Central Vigilance Commission 2011 on Telengana issue.

73. d In Hyderabad 70th Common Wealth Law Conference 91. b Let ‘x’ years ago, Akbar’s age was exactly 5 times
was held. that of Jahangir.
Then, 50 – x = 5 (18 – x)
74. c Gustavo Santaolall who composed the music to the ⇒ 4x = 40 ⇒ x = 10
song ‘Stranger Lives’ in the movie ‘Dhobi Ghat’ is from Hence, required Akbar’s age = 50 – 10 = 40 years.
Argentina.
92. c Net effect of each lift (prior considering height of next
75. c Rashtriya Swayamsevak Sangh was not awarded a lift) = 1.5 – 0.5 = 1 feet
portion of the contested land by the judgment of the ∴ Required number of lifts = 17 + 1 = 18.
Allahabad High Court in the year 2010 pertaining to the
Ayodhya dispute. 93. c (i) Tax for first Rs.1,50,000 = 0
(ii) Tax for Rs.1,50,001 to Rs.3,00,000
76. d Vishnu Vardhan belong to the sports of tennis. 10
= (3,00,000 − 1,50,000) × = Rs.1,500
100
77. a
(iii) Tax for Rs. 3,00,001 to Rs.5,00,000
78. b Dongria Kondh is the tribe living in the Niyamgiri Hills, 20
= (5, 00,00 – 3,00,000) × = Rs.40,000
which is at the heart of the controversy surrounding 100
Vedanta Resources’ mining operations. Total tax liability of Jogen = 0 + Rs. 15,000 + Rs.40,000
= Rs.55,000.
79. b Siddartha Shankar Rai who died in year 2011 was a
politician. 94. a Let 4x and 5x be two numbers. Then,
4x + 20 6
80. b Joseph Lelyveld is the author of the book ‘Great Soul: = ⇒ x = 10
5x + 20 7
Mahatma Gandhi and his Struggle with India’, which Therefore, numbers are 40 and 50.
was criticized for its content. Hence, sum of two numbers = 40 + 50 = 90.

Previous Years
CLAT & AILET Papers Page 33
95. d Let 5x, 6x and 7x be the number of students studying 102. c Total number of cases are: {HT, HH, TH, TT}
in Arts, law and commerce respectively. 3
After increment, new ratio ∴ Required probability = .
4
= 5x × 1.2 : 6x × 1.3 : 7x × 1.4 = 30 : 39 : 49.
103. c Let AB be the pole and C be the far end of the shadow.
96. c Since 10% of making charges is Rs.100, total making
charges = Rs.1,000. B
Marked Price = 1000 + 900 = Rs.1,900
Discount offered in percentage
900 × 100
= × 100 ≈ 47%. 1 8m
19
97. c Let D be the distance between man’s house and
railway station. Then,
D D 9 + 6 15 1 C A
– = = = ⇒ D = 10.5 km. 9 .6m
6 7 60 60 4 Using Pythagoras’ Theorem,
BC2 = (18)2 + (9.6)2 ⇒ BC = 20.4 meters.
98. b Let x and y be two numbers, where x > y.
x–y=9 ... (i)
x2 – y2 = 981 104. a The given series is an A.P. whose first term is 5 and
common difference is 3.
⇒ (x – y) (x + y) = 981
∴ T10 = 5 + 9 × 3 = 32. [ 3 Tn = a + (n –1)d]
⇒ x + y = 109 ... (ii)
Solving (i) and (ii), we get
x = 59 and y = 50 105. d Total number of balls = 19 (Red) + 37 (Blue) + 27
Hence, lowest of the two numbers is 50. (Green) = 83
37
99. a Let x be the average score of Ms. Jhulan Goswami ∴ Required probability = .
83
after 17th innings. Then,
106. a
17 × x + 102
= x + 5 ⇒ x = 12
18
Hence, after 18th innings, her average score
= 12 + 5 = 17. 6r

100. a On the basis of the given information, we cannot find


out the number of teachers who drink only black coffee r
and milk coffee and all the three beverages. So we
cannot find the number of teachers who drink only
tea. Let ‘r’ be the radius of balls, so radius and height of
B la ck co ffe e M ilk co ffe e the cylinder will be r and 6r.
(13 ) (7) 4 3
∴ Volume of a tennis ball = π r = 240
3
y
⇒ π r 3 = 180
Volume of the container = π r × h = πr (6r)
2 2

z
x 9 –x = 6πr 3

= 6 × 180 = 1080 cm3.

107. b Let ‘r’ be the radius of the park. Then,


15 22 5
Tea 2π r = ×5 ⇒ 2× ×r =
60 7 4
⇒ r ≈ 0.200km or 200 meters.
101. a Perfect square numbers from 1 to 90 are 1, 4, 9, 16,
25, 36, 49, 64 and 81. 108. d Given expression can be rewritten as,
9 1
∴ Required probability = = . –3 2(2x – 6) 9 4x –12– 3 9
90 10 7 7 7 7 7
9 ×9 =   ⇒  = 
    9 9 9
∴ 4x – 15 = 9 ⇒ x = 6.

Previous Years
Page 34 CLAT & AILET Papers
109. d Let x (in Rs.) be the price of one egg. we cannot say with certainity that the effect will also
Then, price of 12 eggs = Rs.12x not be there. Therefore, option (b) is correct.
Selling price of 12 eggs = Rs12x × 0.9
= Rs.10.8x 115. b
Price to be recovered by selling 6 eggs A th eist
= 1.2x + 6x = Rs.7.2x
7.2x – 6x
∴ Percentage mark-up = ×100% = 20%. C o m m u nist
6x

110. c Let l, b and h be length, breath and height of the brick


respectively.
Original volume of the brick = lbh According to Suleiman, communists set must be a
Volume after change in dimensions subset of atheist set (as shown in the figure). But if
= 1.1l × 1.1h × 0.8b = 0.968 lbh Sheeba contradicts this then at least a part of
∴ Required percentage change in volume communists set must be outside the atheist set.
lbh – 0.968lbh Therefore option (b) is correct.
= × 100 = 3.2%.
lbh
116. d
111. a S

F
P ing os

S
B yro nic

According to the figure drawn above, both pingos


and Shalisto are subsets of Byronic but they do not
This argument is similar to ‘if P then Q’ type in which P
necessarily intersect each other. Therefore, none of
is cause and Q is the effect. If this argument is true
the options given in (a), (b) and (c) follow. Option (d)
then whenever there is smoke there will be fire (Smoke
is correct.
is cause and fire is the effect). Therefore, to make this
argument false we have to show that at times smoke
does not cause fire. Option (a) is correct. 117. c A (1) B (2) D (4) G (7) K (11) P (16 ) V(22)

112. b This argument is similar to ‘if P then Q’ type in which P +1 +2 +3 +4 +5 +6


is cause and Q is the effect. If this argument is true
then whenever there is Poverty there will be thieves 118. d Position numbers of the letters B, C, E, G, K, M, Q, S
(‘Poverty’ is cause and ‘thieves’ is the effect). are 2, 3, 5, 7 11, 13, 17 and 19 respectively i.e.
Therefore, to make this argument false we have to increasing prime numbers. Therefore, the position
show that at times poverty does not cause thieves. number of the next letter will be 23. Hence, the next
Option (b) shows that even if there is poverty in Bhutan letter will be W.
there are no thieves.
119. b Z(26 ) X (24 ) T(20 ) N (14) F(6)
113. d Sugar is bad for people with diabetes. Leela does not
eat sugar. From these statements, we cannot conclude
–2 –4 –6 –8
whether Leela has diabetes or not. She may not be
diabetic and still avoids taking sugar because she 120. c All the words in the given series are in the order of
does not like sugar. So, option (a) and (b) does not how they will appear in a dictionary. For example
follow. Option (c) cannot be concluded on the basis of ‘Apple’ will come before ‘Application’ in a dictionary.
the given statements. There is a chance that a person Hence the word that fits the blank best will be
suffering from diabetes may still eat sugar because ‘Appropriate’.
he is either unaware or less careful about his/her
health. None of the above is the best choice. 121. a The difference between every two consecutive days
in the given series is increasing by 1, 2, 3, 4, 5, 6 and
114. b This argument is similar to ‘if P then Q’ type in which P 7. Hence, the next term will be Sunday.
is cause and Q is the effect. If this argument is true
then whenever there is ‘reading fashion magazine’ 122. d In each term, larger number is removed and the
there will be ‘no reading fiction’ (reading fashion remaining numbers are rearranged in reverse order in
magazine is cause and no reading fiction is the effect). the next step. Hence, missing term will be 42783.
In this kind of argument, when the cause is not there

Previous Years
CLAT & AILET Papers Page 35
123. b None of the words contains any vowel but Mythic inconsistency with mathematics or physics. Option
does. (d) is incorrect as it does not answer the question. It
gives a conclusive statement regarding the possibility
124. b In the given series, in each word first two letters are of finite space, i.e. it is inconceivable. Option (a) is
same. Hence, the next word will be Uulium. correct as it states that Lucretius showed that believing
the space to be finite leads to a contradiction. This is
125. a The higher literacy rate in China is attributed to greater to say that his thought experiment wherein he posited
efficiency of the Communist system and the lesser the space to be finite lead to contradictory results.
literacy in India is attributed to the lack of efficiency in This is consistent with the given argument and hence
the democratic system. Among the given options, is the correct choice.
option (a) disproves the efficiency aspect, in the
respective systems, in improving the literacy rate. 131. b Utilitarians believe that the action that produces
Hence, it undermines the argument. maximum happiness is the right action. Option (b) states
that one should do one’s duty without giving any
126. c Option (a) is not correct as we do not know if he was consideration to the consequences (happiness or
convicted under unjust law. Option (b) makes Socrates’ misery). Thus, option (b) is incompatible with the belief
commitment to obeying law due to long residence a of Utilitarians.
general argument of obeying law. Moreover, absence
of explicit commitment may not be a factor in deciding 132. b Senthil draws his conclusion about the entire Ambala
whether to accept a pronouncement of offence. Option city on the basis of a limited and insufficient data. He
(d) in any way does not refute the basic argument of had only seen the route from railway station to the
Socrates. Option (c) provides reasons to believe that hotel and not the entire city.
Socrates need not necessarily obey the law that has
implicated him. 133. a A conclusion about the time that will be taken by India
to achieve internal stability has been drawn on the
127. a The argument states that the government is justified in basis of the data related to Europe. This may not be
banning all soft drinks from the Indian market because true unless the process of attaining internal stability
they have been shown by scientists to be bad for the follows a general pattern. Therefore, option (a)
teeth. The necessary assumption here is that the provides the correct assumption. Option (b) weakens
government is justified in banning anything that is bad the claim. Option (c) is irrelevant as the argument talks
for the teeth. Option (b) is irrelevant as it does not help about the time span and not about the process of
us in arriving at the conclusion in any way. The concern achieving stability.
for dental hygiene also does not necessarily justify
the government’s action since we do not know if 134. c Option (a) can be verified therefore, it’s a fact. Similarly,
banning anything of great concern is justified or not. options (b) and (d) can also be verified. In Option (c),
Thus, option (a) is the correct choice. the issue of Indian policy being misguided is a
subjective issue and cannot be verified. Hence, it’s an
128. c Older than 25 years and not being bankrupt are the opinion.
two criteria for being eligible for election to Lok Sabha.
Jatinder Singh fulfills the criterion related to age but 135. c What is more pleasant to live and what is less pleasant
does not fulfill the one related to bankruptcy. So he is is a subjective issue and the answer about it may
not eligible for election to Lok Sabha. As the conclusion vary from subject to subject. It cannot be verified.
says that he cannot be elected as the Speaker of the Thus, it is not a fact but an opinion.
Lok Sabha, this means that the author has assumed
that one who is not eligible for election to the Lok 136. c Whether Mumbai is larger or smaller than Pune can be
Sabha is also not eligible for being the Speaker of Lok verified. So, option (a) and (b) are facts. Similarly,
Sabha. Thus, option (c) is correct. whether Mumbai is more crowded than Pune can be
verified and is a fact. A place being more cultured than
129. a The passage states that the behavior of people at the the other is a subjective issue and opinions on this
end of the century is similar to the behavior of people may vary from subject to subject. Thus, option (c) is
that are at the end of their lives. Just as those who are an opinion.
at the end of their lives start reflecting on the past A e ro Ba Roc
events of their lives similarly, those who are alive in
1999 (i.e. those who are at the end of the century) will
reflect on the events of the century (twentieth). Thus,
option (a) is the logically correct choice.

130. a The question asked is regarding the form of Lucretius’s


argument, not about what is the take –away from his
thought experiment. Option (b) and (c) are out of scope 137. d A, B, C or D do not follow. Therefore, option (d) is the
as there is no mention of the thought experiment’s correct choice.

Previous Years
Page 36 CLAT & AILET Papers
138. a 147. b The argument discusses the importance of social
F
context in making learning a language easier and
A effective. Option (b) summarizes the argument
effectively.
W
148. d All of the given statements present facts that contradict
the given argument and thus weaken it.

A and B can be concluded. Whales are both fish and 149. b The argument talks about the goodness/richness of
amphibians is correct but only whales are both fish tribal culture and the lack of need to dilute the richness
and amphibians is incorrect as data is not sufficient to of these cultures. The conclusions implied in the
conclude this. Thus, option (a) is the correct option. argument are: 1. Following/developing their own
culture is good for tribes as well as for the country. 2.
139. d Tribal cultures do contribute to the diversity of the
L ab Indian culture. 3. Forceful adoption of another culture
by tribal people would reduce their cultural distinctness
Hos and diversity.
L ib
Option (b) which states “Tribal customs should not be
allowed to change in any respect” takes the argument
too far and is not implied in the passage.

No laboratory is a hostel and library is a sub-set of 150. d The argument talks about the goodness/richness of
laboratory. Therefore, no library is a hostel. Option (d) tribal culture and the lack of need to dilute the richness
is the correct conclusion. of these cultures. Option (d) which states “The tribes
should assimilate as far as possible into non-tribal
140. d The argument states the models and theories of culture as a condition of full citizenship” contradicts
economics (that are based on mathematical formulae) the spirit as well as the essence of the passage.
are too idealistic and far from realily. Option (d)
summarizes the argument effectively. 151. c The argument does not mention the conditions that
demand the tribal people to be flexible and change
141. a The argument demands the mathematical models used some of their customs. Moreover, the idea of modern
by economists to be more realistic than idealistic. It medicines is beyond the scope of the passage. Thus,
does not ask to stop the use of mathematical models. option (c) is not an assumption implicit in the argument.

142. b Option (b) would contradict the argument. If the 152. b The passage itself states that in India, the states
conditions in the real word are very much similar to governed by the Communist party respect human
the theoretical conditions used in the mathematical rights. Option (b) contradicts this by stating that
models, then the argument that these models fail to communist states never respect human rights. Thus,
incorporate real conditions gets contradicted. option (b) is not a conclusion implicit in the passage.

143. b The paragraph talks about the spread of religion 153. a The argument states that the concept of human rights
(Buddhism) vis-à-vis the geography of that place/ enshrined in the constitution of India is the reason
country. Option (b) summarizes the paragraph behind the Communist party’s respect for human rights
effectively. (in India). Option (a) weakens the argument by stating
that the communist governments are not motivated by
144. b The statement “The Hindkush mountains made no the principles of the constitution (of India). Instead
difference to the spread of Islam” nullifies the argument they are motivated by their own Communist principles.
that geography (particularly mountains) played a role
in the spread of religion. 154. a The argument discuses that democracy ceases to be
a liberating force if the people are poor and
145. b The conclusion of the argument is mentioned in its last unemployed. Option (a) states that ‘democracy has
lines “That is, friendship connoted intimacy in England ceased to be a liberating force’. Thus, option (a) fails
while in Rural India, brotherhood conveyed intimacy.” to convey the meaning implied in the argument and is
Therefore, the attitude of people of England towards incorrect.
brotherhood and friendship is different from that of
rural India. 155. a The argument is that in the absence of economic
democracy, political democracy loses its luster and
146. c The argument concludes that “friendship connotes significance. Option (a) states that political democracy
intimacy in England”. Option (c) which states that is inseparable from economic democracy i.e. they
‘people in England do not think that friendship connotes always co-exist. Hence, it weakens the argument.
intimacy’ contradicts the argument.

Previous Years
CLAT & AILET Papers Page 37
156. c Refer to Rule D. Fundamental Rights can only be 165. b The point to consider is whether the action taken by
enforced against the State and the administrative the Governor is ‘absolutely necessary’ the given case.
actions of the state. Gajodhar Pharmaceuticals being It cannot be found to be such and though it promotes
a private limited company, no fundamental rights can the well being of the women it is not ‘absolutely
be enforced against it. necessary’.

157. d Applying Rule D and B to the facts, we can derive 166. a The discrimination based on age would fall under ‘any
that, one Fundamental Rights can be enforced by Syed other status’ under Rule A. It does not get protected
since this is a law made by the State, two, the law in under Rule D. Therefore this is a case of direct
question curtails Syed’s freedom to decide whom to discrimination.
associate with. Therefore option d is the most relevant
choice. 167. d There is no mention of percentage of graduate
population in Bihar in the question itself. No answer
158. a Neither of the rules are applicable to the given facts. can be possibly arrived at. Hence the given option D
Nor does the law in question forces any individual to should be taken as ‘None of the above’ instead of All
join any association. Therefore a is an appropriate of the above.
answer.
168. a Note the absence of phrase “absolutely necessary”
159. d Applying Rule D to the question. Refer explanation to in Rule E as against Rule D above. The order can now
156. be justified as is promotes the well-being of women,
who were being held in prison, and had a punishment
160. c The right to freedom of association does not extend to period of less than one year.
realizing the objectives of forming such an association.
Strikes are only an objective in the given case and 169. b The principle clearly states that in case of a guardian
therefore would not amount to any violation of the the age of majority becomes 21. And being a minor if
fundamental rights as mentioned in the rules. she enters into a contract with Ajay, the same is not
enforceable as per the principle. Chaaru can justifiably
161. b Applying the first part of the rule, Elizabeth is entitled challenge the sale transaction.
to keep the ear ring unless the true owner claims it. In
the absence of any claim, airport cannot presume 170. a The only exception wherein money can be recovered
ownership of the ear ring just because it was found from a minor is if the other party was deceived. The
on its property (applying third part of the principle). principle makes no reference to honest belief. Hence,
Therefore, selling of the ear ring by the airport is wrong a is the correct answer.
and Elizabeth should be compensated for the same.
Now, out of option (b) and (c) , both of which talk 171. a The sale would be valid and enforceable only if Bandita
about compensating Elizabeth, the reasoning of the is a major.
former is more appropriate as it relates with the
principle given in the question. 172. c is the only appropriate option as knowing the Bandita
is 18 years and the property is being looked after by
162. b Elizabeths travelling in a particular class is not relatable. her mother Ajay had induced rather than convincing
Bandita to sell the land.
The first part of the principle talks about “finding of an
unattended object” and this without any qualifications
173. a It will surely defeat the purpose of law if Ajay is
or limitations.
allowed to recover, as the protection is rendered to
minor persons considering they may not be able to
163. d The right to confiscation can be exercised on any
make a rational decision. Therefore any transaction
object that is unattended and is found by the Airlines
entered into with a minor is not enforceable. If Ajay is
Staff. However, when Elizabeth finds the ear ring, as
allowed to recover then the purpose of law to offer
per the principle, she has the right to keep it till the true
protection to minors would be of no value and effect.
owner claims it back. Therefore, applying the same
explanation as in 161, in the absence of any claim by
174. c Refer Rule C. Dhanaraj calls Chulbul and threatens
the true owner, the air line is liable to compensate
him.
Elizabeth for selling the ear ring further.
175. c The case here is one of coercion and not of undue
164. a Direct application of Rule A qualifies the act of the
influence. Undue influence is exercised in cases
State as discrimination. Rule D, the exception to Rule A
where one person has a position of authority over the
is not applicable as the discrimination in question in not other and manipulates such to unduly influence the
justified on any of the grounds mentioned in the Rule. other person. This is a case of coercion.
Now, out of Rule B and C, the former is applicable and
therefore this amounts to a direct discrimination being 176. a Chulbul was coerced by Dhanraj into entering into the
made on the grounds of sex alone. employment contract and hence he is justified in

Previous Years
Page 38 CLAT & AILET Papers
refusing the enforcement. Out of option (a) and (b) 189. c Lucky would still be guilty of theft as long as he did
the reasoning provided in (a) is more logical and not have Indiras consent on it. Indira had only asked
relatable to the principle. him to take away the pile of newspapers. Removing
anything apart from that from the possession of Indira
177. b Baalu had not coerced Chulbul in the given case and if would amount to theft.
this is proved, he would be entitled to get the job.
190. c Kamala is liable for theft as the rule talks about having
178. b Since the question demands applicability of Rule A, the intention to take property from the possession of
option b would hold good. A measure, the outcome of another without his consent. The person in question
which is certain and would amount to violation of Rule need not necessarily be an owner. All other
requirements of Rule A is established and hence it
A, should not be put to enforced.
makes Kamala liable for theft as per the principle.
179. d The qualifications in the lottery mentioned in each other 191. d No act done by Kamala amounts to reducing the value
option is very arbitrary and just not ensure equal chance of the property in question. Hence there was no criminal
as is required under Rule B. Therefore, option is the damage. Options a and b are thus ruled out. The facts
only appropriate choice. do not make any reference to damage of property
after it is in Kamala’s possession. Therefore, (d)
180. c Rule B talks about equal chance. Hence, a lottery for becomes the only appropriate answer.
all without any further qualification can be upheld under
Rule A. 192. d Only fixtures are deemed to be sold with the land. But
as per Rule B, the carpet was never attached to the
181. c Here the measure is sending helicopters and if that is land, and therefore was not a fixture. Khaleeda had
not provided for everyone equally, Rule A would be the right to remove the carpet after sale.
defeated.
193. b Door was a fixture and was attached to the house
182. d Application of Principle A leads us to infer that Ashish and therefore cannot be removed as per Rule A. Out
Mathew is not an employee of the company. Now out of options (b) and (d), the former is more appropriate
of option (b) and (d), the latter is more appropriate due as it relates to the principle.
to better reasoning.
194. b For the fixture to be deemed to be sold with the land,
it is relevant to know if it amounts to fixture or not.
183. d Rule A talks about “mode and manner of carrying out Applying Rule B, the test would be to see if the
the work”. Clearly by regulating the work hours Ashish moveable thing was merely placed on the building and
becomes an employee of the company. were fixed to it or not.

184. a The concept of “during the course of employment” is 195. d Applying the new rule, carpet also become a fixture
not applicable to the present facts in the absence of and thus could not have been removed. Hence the
any employer employee relationship. Therefore the case would be in favour of Gurpreet in both the
company is not liable for any compensation and a is situations.
the most appropriate answer.
196. c Rule A provides no classification and gives the owner
185. c There was no direct relationship between Ashish and an infinite right over the space above and beneath his
the company and therefore the concept of “during the property.
course of employment” is not applicable to the present
facts even though the act in question by Ashish while 197. d Hoisting a flag of 75 ft is not essential to the enjoyment
of rights of a person to his property. Please note that
he got injured was incidental to his duties.
out of option (a) and (d), the latter is chosen because
of the reasoning that relates to the principle.
186. c Out of all the situations mentioned in the options, option
c best explains “during the course of employment” as 198. a Since the right provided to an owner under Rule A is
he was required to travel in the bus as per his contract. without any limitation, Shazia shall succeed under Rule
A only.
187. c Lucky had only taken the pile of newspapers with the
consent of Indira, but not the painting. Moreover he did 199. d Ramesh had no reasonable use of property at such a
not return the painting to Indira after discovering it and height and the satellite passing over his property at
this establishes the theft. such a great height does not interfere with his right to
use or enjoyment of property.
188. a The rule of criminal damage talks about the intention of
the person in question. Here Lucky had only pasted 200. d In her favour because, Rule C talks about the fact that
the painting in order to prevent it from tearing any reasonable enjoyment of the land shall not be affected.
further. Therefore in the absence of any intention of The permanent shadow casted by the hoarding affects
reducing the value of the property, Lucky has not the reasonable use and enjoyment of Shazia’s
committed any criminal damage. property.

Previous Years
CLAT & AILET Papers Page 39
CLAT Solutions 2012
1. D According to the passage, the new order of 5. C Refer to the lines, “If egalitarianism is to endure...This
egalitarianism can be established only through radically calls for substitution of material values by purely
changing the mind and attitude of people. Thus, it directly spiritual ones”. Option A is completely incorrect; it
follows from the passage that egalitarianism won’t should not be thrust upon the people. Option B is
survive if people’s outlook towards it is not radically incorrect as it is incomplete. It should also be based on
changed. So, option D is correct. Option A is what unity and peace, equality and universal brotherhood
egalitarianism demands for its survival. It requires and so on. Option D is also incorrect. It is an outcome
voluntary renunciation of material goods. Hence, option of a successful egalitarian society, not a way to make
A is incorrect. Option B is incorrect since it speaks true egalitarianism last.
only in the context of India and is not the central reason
6. A* Consider the second paragraph. It talks of a possibility
for the death of egalitarianism.
of eventually breaking away from barriers that curtail
the possessive instinct in an individual by force.
2. D* Option A is incomplete since it does not tell who these
Moreover, the line “this enforced egalitarianism
others are. Option B is merely stating the same thing
contains, in its bosom, the seed of its own
as the question and doesn’t answer it. Option C is destruction” also hints at a social order based on
close but love and respect from descendants are not oppression and coercion. This makes option A the
talked about in the passage. Option D is given as the best choice out of all other options.However, it can’t
correct answer by CLAT but it is not. The reason that be said that people will overturn such a social order.
is mentioned in the passage for man valuing his The paragraph just talks of a possibility and that too in
possessions more than his life is that he wants the context of possessiveness. Option B and C are
perpetuation and survival of his descendants. Option incorrect since there is no mention of basic needs and
D doesn’t take into account preservation of his name conciliation and rapprochement in regard to a social
through the ‘descendants’. Hence, all options seem order. Option D is incorrect as it talks of people
incorrect. overturning a social order not congenital to the spiritual
values of people. This is nowhere mentioned in the
3. C Option C is the correct option. Refer to the lines, “the passage.Hence, no option fits in perfectly here.
establishment of a new social order...this unfinished
part of his experiment...”.The ‘social order’ mentioned 7. B It is directly mentioned in the third paragraph of the
here refers to egalitarianism. Hence, establishment of passage. Refer to the line, “The root cause of class
an egalitarian society was the unfinished part of conflict is possessives or the acquisitive instinct”.
Gandhi’s experiment. Option A is incorrect as nothing Hence, option B is the correct choice.
regarding educating people to avoid class conflict is
mentioned in the passage. Option B is incorrect since 8. D Option A is true as the passage says, “A new order
he did achieve political freedom for the country and so cannot be established without radically changing the
mind and attitude of men...”Option B is true. Refer to
it is not the unfinished part. Option D is a means to
the last couple of lines of the passage, “...the
establish an egalitarian society. Hence, option C is the
acquisitive instinct inherent in man can be
correct choice.
transmuted by the adoption of the ideal of
trusteeship...”Option C also follows directly from the
4. B* The answer given by CLAT for this question is option passage, “this enforced egalitarianism contains, in
B. Option B certainly is correct i.e. not true in the its bosom, the seed of its own destruction”. Option D
context of the passage but it is not the only correct is false as the passage says ideal of new order is to
choice. Man values his possessions more than his life secure minimum material satisfaction and not maximum.
and not the other way round. However, even option A Hence, option D is the correct answer.
is correct. It says true egalitarianism can be achieved
by giving up one’s possessions under “compulsion”. 9. B* Option A cannot be deduced from the passage. The
This is not true sincethe passage says there should passage does not talk about how political freedom
be ‘voluntary’ renunciation of one’s possessions. can be achieved and so we can’t conclude that a
Hence, option A is not true in the context of the passage social order based on truth and non-violence alone
and is also a correct option. Options C and D are true; can help achieve it. Option B is given as the correct
refer to the lines, “Mahatma Gandhi has shown us answer but in fact it is incorrect. Refer to the line, “In
how the acquisitive instinct inherent in man...” and “In establishing a new social order on this pattern, there
the political struggle, the fight was against a foreign was a strong possibility of a conflict”. But the option
power...”. says the opposite. It says such a possibility will ‘hardly
exist’. Thus, option B cannot be deduced from the

Previous Years
Page 40 CLAT & AILET Papers
passage and is incorrect. Option C is the 18. C Commensurate refers to a correct or suitable amount
correctanswer. Property here refers to possessions. when compared with something. Hence, rewards are
It is difficult to root out attachment to property because commensurate ‘with’ the work. Thus, option C is
of an inherent acquisitive instinct in man. Option D is correct.
incorrect as in an egalitarian society, the ‘haves’ yield
place to the ‘have-nots’ and do not enjoy material 19. A The correct preposition used with index is ‘of’. Index
satisfaction at the expense of others. is a list ‘of’ items. Hence, option A is correct.

10. B ‘Adoption of the ideal of trusteeship’ refers to sharing 20. A ‘Smacks of’ is a phrasal verb meaning something having
of wealth by the haves with the have-nots. This is an unpleasant quality. If something ‘smacks of’
given in option B. Option A is incorrect as the passage something, it means it has that unpleasant quality. ‘Your
says equating material satisfaction with progress conduct smacks of recklessness’ means your conduct
neither spells peace nor progress. Option C is not the is reckless.
correct choice as the ideal of trusteeship doesn’t mean
voluntary remuneration of possessive instinct. 21. D Grope means trying to feel something with hands when
Also,remuneration is incorrect with respect to the you can’t see it with your eyes. ‘Gropes for’ is a phrasal
passage; the word should have been renunciation. verb that means to try to look for the right answer, the
Option D is incorrect. Material values should be right word for something. A good judge will never
substituted by spiritual values. ‘grope for’ the conclusion means he won’t consciously
look for a correct conclusion rather he would naturally
11. D* Lethargy means a state of sluggishness or arrive at one.
drowsiness. Though the answer is given as option D,
laxity, the correct answer should be listlessness. 22. D Genius is a very rare, exceptionally brilliant skill. A
Listlessness means lack of spirit, energy which is a person is called a genius when he is exceptionally
synonym of lethargy. Laxity means deficient in brilliant at something. The correct preposition used
firmness; loose. Serenity means calmness or peace. here should be ‘at’ which refers to general area of
Impassivity is being unsusceptible to physical feeling. proficiency. Genius is followed by ‘at’. Good can be
Hence, the correct answer is listlessness. followed by both ‘at’ and ‘in’. ‘in’ is more specific; for
example, we use ‘in’ for school subjects. “I am good
12. C Emaciated means losing flesh so as to become very in math” and “I am good at basketball”.
thin. Thus, the correct choice is option C. Languid
means sluggish in character, or lacking force and 23. C Averse refers to a strong dislike for something. The
quickness in movement. correct preposition for averse is ‘to’. For example, “I
am averse to spiders” means that I dislike spiders.
13. A Latent means something that is hidden. Concealed is Hence, option C is correct.
the correct synonym. Hence, option A is correct.
24. D Delve means to carefully research something or
14. C Sporadic is something that occurs irregularly or examine in detail. But delve is an intransitive verb and
randomly. Occasional is the correct synonym for the can’t have a direct object. For example, “The
same. Epidemic is something that spreads very fast. philosopher delves into his book for insights”.Hence,
Whirling refers to moving in a circular motion with option D is correct.
force. Stagnant means something that doesn’t advance
or develop. 25. C Bearing means to have an effect on something. Hence
option C, ‘on’, is the correct preposition for bearing.
15. D Compendium is a list or collection of a number of items.
For example, ‘He published a compendium of folk 26. A ‘Burning one’s fingers’ means getting into trouble as a
tales’. Collection is the correct synonym. Summary is result of being a part of something. Hence, option A is
a brief account of something. Index is an alphabetical the correct choice.
list of items with reference to where they occur,
typically found at the end of a book. Reference means 27. D ‘Feathering your own nest’ means to make a profit for
the act of mentioning or hinting at something. Hence, oneself especially by taking advantage of one’s
option D is correct. position. Thus, option D is the correct choice.

16. D Pertinent means having a clear and definite relevance 28. C ‘In the blues’ refers to being sad or depressed. Hence,
to the matter at hand. The correct preposition used depressed is the right choice for the same.
with pertinent is ‘to’. Hence, option D is correct.
29. D ‘Like a fish out of water’ means to be uncomfortable or
17. B Reverberate means ‘to echo’. The voice would reflect restless. Option D is the correct choice.
‘from’ the walls to give rise to an echo or reverberation.
Hence, ‘from’ is the correct preposition.

Previous Years
CLAT & AILET Papers Page 41
30. B Riding a high horse means behaving in a way as if he/ teacher of Alexander the Great and his writings cover
she is better than everyone else. Thus, riding a high many subjects, including physics, metaphysics, poetry,
horse means to appear proud and arrogant. Thus, theater, music, logic, rhetoric, linguistics, politics,
option B is correct. government, ethics, biology, and zoology. Together
with Plato and Socrates (Plato’s teacher), Aristotle is
31. D i talks about Indians, which is described in iv, hence iv one of the most important founding figures in Western
follows i. v completes iv because it tells what Buddha philosophy. Aristotle’s writings were the first to create
and Gandhi taught. ii follows v since it comes up with a comprehensive system of Western philosophy,
the other perspective of the sentence. iii follows ii and encompassing morality, aesthetics, logic, science,
has to come before vi because the ‘nor’ in vi has to politics, and metaphysics.
follow the ‘not’ in iii. Hence, option D is the correct
order.
42. C December 2nd is World Computer Literacy Day. The
aim of World computer literacy day at is to educate
32. B itells us that there has to be an observation that follows
and spread awareness among the masses about the
it, since it has the words ‘on the basis of experiments’.
ii fits in here perfectly because we have health experts benefits of using computers.
putting forth their results. v follows ii, as it is the
beginning of the condition for the conclusion that 43. A Jaean-Jacques Rousseau’s ( 1712 – 1778) teachings
humans can live up to 100 years. iii adds to v with and philosophy influenced the French Revolution as
more conditions. iv gives the conclusion and falls in well as the overall development of modern political,
place before vi. Option B has the correct order. sociological and educational thought. He is a Genevan
philosopher, writer, and composer of 18thcentury
33. C ii and iv form a mandatory pair as the ‘it’ in iv refers to Romanticism of French expression. This Solitary
the bomb in ii. This eliminates option A and D. Similarly, Walker exemplified the late 18th-century movement
v has to come before vi because the ‘they’ in vi refers known as the Age of Sensibility, featuring an increasing
to the ‘scientists’ in v. Hence, option C is the correct focus on subjectivity and introspection that has
choice. characterized the modern age. His Discourse on the
Origin of Inequality and his On the Social Contract are
34. A iii and v form a mandatory pair as the mechanism of cornerstones in modern political and social thought.
demand and supply in iii is explained in v. Such a
combination is only there in option A, hence it is the 44. D The 2ndAfrica-India Summit was held at Addis Ababa,
correct choice. with India and 15 African Countries participating. The
leaders discussed significant aspects of the India-
35. D iii follows the opening sentence and tells more about Africa partnership with the objective of enhancing
India. the ‘it’ in iii refers to India of i. ii and iv fall together and widening its ambit for mutual benefit.
as both are examples of famous personalities who
Africa and India agree to continue their cooperation
have had an impact on India and the world. v has to
in the areas enumerated below:
come before the last sentence as it talks about shaking
1. Economic Cooperation
the foundation of British rule in India which was done
2. Political Cooperation
by Mahatma Gandhi. Hence, option D is correct.
3. Cooperation in Science, Technology, Research and
36. B Mala fide means in bad faith. Hence, option B is the Development
correct choice. Bona fide means in good faith. 4. Cooperation in Social Development and Capacity
Building
37. A Tabula Rasa refers to absence of any preconceived 5. Cooperation in Health, Culture and Sports
notions, a clean slate. Option A is the correct choice. 6. Cooperation in Tourism
7. Cooperation in Infrastructure, Energy and
38. B Carte Blanche means having complete freedom to act Environment
as one wishes. Option B, complete discretion, is the 8. Cooperation in the area of Media and
correct answer. Communications

39. C De jure means ‘legally’ or by right. Option C, by law, is 45. B Akshardham (in Gandhinagar) is one of the largest
the correct answer. temples in the Indian state of Gujarat. The temple
complex combines art, architecture, education,
40. B Raison d’etre is a French phrase which means reason exhibitions and research at one place. The temple
for being. Option B, reason for existence, is correct. came to international attention when two heavily armed
terrorists attacked it in September 2002.It was
41. B Aristotle (384 BC – 322 BC) the famous Greek inaugurated on November 2, 1992, during the
philosopher and polymath is the person to whom the centenary celebrations of Yogiji Maharaj.
given quote is attributed. He is a student of Plato and

Previous Years
Page 42 CLAT & AILET Papers
46. C DhyanChand Award is India’s highest award for lifetime 52. B The liver is the largest gland and the largest internal
achievement in sports and games, given by the organ in the human body. A gland is defined as any
Government of India.The award is named after the group of cells that manufactures and releases
legendary Indian hockey player DhyanChand. The substances for use somewhere else in the body. The
award was initiated in 2002 and carries a cash prize liver not only makes and secretes substances, but it
of INR 5 lakh, a statuette,ceremonial dress and a scroll has over 5,000 functions that have been identified,
of honour. including immune protection, waste removal,
In August 2011, the following 3 sports personalities metabolism and energy storage.
were awarded the DhyanChand Award : Shabbir Ali
(Football), SushilKohli (Swimming), and Rajkumar 53. A David A. Vise, formerly a Pulitzer Prize-winning
(Wrestling). reporter for The Washington Post, authored the book
titled ‘ The Google Story ’ which chronicles the birth
47. B The Pushkar Fair, or Pushkar ka Mela, is the annual and rise of Google as a search engine giant.
five-day camel and livestock fair, held in the town of
Pushkar in the state of Rajasthan, India. It is one of the 54. B The Strait of Gibraltar is a narrow strait that connects
world’s largest camel fairs, and apart from buying and the Atlantic Ocean to the Mediterranean Sea and
selling of livestock it has become an important tourist separates Gibraltar and Spain in Europe from Morocco
attraction and its highlights have become competitions in Africa. The name comes from the Rock of Gibraltar,
such as the “matkaphod”, “longest moustache”, and which in turn originates from the Arabic Jebel Tariq
“bridal competition” . (meaning “Tariq’s mountain”) .Europe and Africa are
separated by 7.7 nautical miles (14.3 km; 8.9 mi) of
ocean at the strait’s narrowest point. The Strait’s depth
48. A
ranges between 300 and 900 metres (160 and 490
fathoms; 980 and 3,000 ft)
49. C The Swedish Academy has awarded the 2011 Nobel
prize for literature to one of its own: the Swedish poet
55. C Taiwan, officially the Republic of China, is a state in
Tomas Tranströmer.
East Asia. Originally based in mainland China, the
Tranströmer becomes the eighth European to win the
Republic of China now governs the island of Taiwan
world’s premier literary award in the past 10 years, (formerly known as Formosa), which makes up over
following the German novelist HertaMüller in 2009, the 99% of its current territory, as well as Penghu, Kinmen,
French writer JMG le Clézio in 2008 and the British Matsu, and other minor islands. Neighboring states
novelist Doris Lessing in 2007. include the People’s Republic of China to the west,
Japan to the east and northeast, and the Philippines to
50. B 2011 Winners of UNESCO King Sejong Literacy Prize the south. Taipei is the capital city and economic and
was National Literacy Service - Burundi . The UNESCO cultural centre of the country, while New Taipei is the
King Sejong Literacy Prize was created in 1989 through most populous city.
the generosity of the Government of the Republic of
Korea and rewards the activities of governments or 56. D Ashok Amritraj is an Indian American film producer,
governmental agencies and non-governmental former tour professional tennis player, and chairman
organizations (NGOs) displaying merit and achieving of Hyde Park Entertainment. Amritraj has produced
particularly effective results in contributing to the fight over 100 films, including Hollywood films such as
for literacy. It gives special consideration to the Antitrust, Walking Tall, and Bringing Down the House,
creation, development and dissemination of mother- which starred Steve Martin and Queen Latifah and
tongue languages in developing countries.The Prize Tamil film Jeans. Among his upcoming movies are
consist of a sum of US$20,000, a silver medal and a projects based on Lee Falk’s comic strip characters
certificate. Mandrake the Magician and The Phantom.

51. D Human Rights Day is celebrated annually across the 57. C The 2012 Summer Olympic Games, officially the Games
world on 10 December. of the XXX Olympiad, also known informally as London
The date was chosen to honor the United Nations 2012, began in London, United Kingdom on 27 July
General Assembly’s adoption and proclamation, on 10 and will continue until 12 August 2012.
December 1948, of the Universal Declaration of Human
Rights (UDHR), the first global enunciation of human 58. B West Bengal won the Santosh Trophy Football
rights and one of the first major achievements of the Championship in 2011. Santosh Trophy is an annual
new United Nations. The formal establishment of Human Indian football tournament which is contested by states
Rights Day occurred at the 317th Plenary Meeting of and government institutions. The first winners were
the General Assembly on 4 December 1950, when the Bengal, who also lead the all-time winners list with 31
General Assembly declared resolution 423(V), inviting titles till date.
all member states and any other interested
organizations to celebrate the day as they saw fit. 59. D Excess of money supply as compared to supply of
goods results in Inflation.

Previous Years
CLAT & AILET Papers Page 43
60. C Ostrich is the largest living flightless bird. There are 69. B Osama bin Laden was killed in Abbottabad, Pakistan
about forty species of flightless birds in existence on May 2, 2011, shortly after 1:00am local time by a
today, the best known being the ostrich, emu, United States special forces military unit. The operation,
cassowary, rhea, kiwi, and penguin. code-named Operation Neptune Spear, was ordered
by United States President BarackObama and carried
61. A The Atlantic Ocean shaped like the letter S is the out in a U.S. Central Intelligence Agency (CIA) operation
second-largest of the world’s oceanic divisions. With by a team of United States Navy SEALs from the United
a total area of about 106,400,000 square kilometres States.
(41,100,000 sq mi), it covers approximately 20% of 70. C The XIth Five Year Plan envisaged the highest growth
the Earth’s surface and about 26% of its water in the sector of Services.
surface area. The first part of its name refers to Atlas
of Greek mythology, making the Atlantic the “Sea of 71. A Light-year is a unit of length equal to just under 10
Atlas”. trillion kilometres (or about 6 trillion miles). As defined
by the International Astronomical Union (IAU), a light-
62. C The White Sea – Baltic Sea Canal opened on 2 August year is the distance that light travels in a vacuum in
1933, is the longest ship canal in the world. It connects one Julian year.
the White Sea with Lake Onega, which is further
connected to the Baltic Sea. 1 light-year = 9460730472580800 metres (exactly)
˜ 5878625 million miles
63. C Charles-Édouard Jeanneret, better known as Le ˜ 63241.1 astronomical units
Corbusier ( October 6, 1887 – August 27, 1965), was ˜ 0.306601 parsecs
an architect, designer, urbanist and writer, famous for
being one of the pioneers of what is now called 72. C The 2012 BRICS summit was the fourth annual BRICS
modern architecture. He was born in Switzerland and summit, an international relations conference attended
became a French citizen in 1930. His career spanned by the heads of state or heads of government of the
five decades, with his buildings constructed throughout five member states Brazil, Russia, India, China and
Europe, India and America South Africa. The summit was held at at TajMahal Hotel
in New Delhi, India on 29 March 2012 and began at
64. A India was a founding member in October 1945, despite 10:00 Indian Standard Time. This is the first time that
being a British colony. India, Canada, the Union of India has hosted a BRICS summit. The theme of the
South Africa, New Zealand and Australia were all summit was “BRICS Partnership for Global Stability,
British colonies but were given independent seats in Security and Prosperity.
the UN General Assembly.
73. C The Arihant class submarines are nuclear-powered
65. B India exports the largest quantity of iron ore to Japan. ballistic missile submarines under development by the
In May 2012, it has been decided that India will export Indian Navy. The lead vessel of the class, INS Arihant,
two million tonne of iron ore to Japan annually for the is expected to complete its harbour acceptance trials
next three years and initiate the process for the in February 2012. Four vessels of the class are under
Japanese government’s equity participation in Delhi development and expected to be in commission by
Mumbai Industrial Corridor Development Corporation 2015. The Arihant class vessels are India’s first
to the extent of 26% indigenously designed and built nuclear submarine.
They were developed under the US$2.9 billion
66. D National Highway 44, commonly referred to as NH Advanced Technology Vessel (ATV) project to design
44(old NH 7), is the longest Highway in India that runs and build nuclear-powered submarines.
through the states of Uttar Pradesh, Madhya Pradesh,
Maharashtra, Andhra Pradesh, Karnataka, and Tamil 74. C
Nadu.
75. C The Mahatma Gandhi National Rural Employment
67. B Indira Gandhi Canal is the longest canal in India.It starts Guarantee Act (MGNREGA) is an Indian job guarantee
from the Harike Barrage at Sultanpur, a few kilometers scheme, enacted by legislation on August 25, 2005.
below the confluence of the Sutlej and Beas rivers in The scheme provides a legal guarantee for one
Punjab state. hundred days of employment in every financial year
to adult members of any rural household willing to do
68. C Leukemia (from the Greek leukos “white”, and haima public work-related unskilled manual work at the
“blood”) is a type of cancer of the blood or bone statutory minimum wage of 120 (US$2.17) per day in
marrow characterized by an abnormal increase of 2009 prices. The Central government outlay for scheme
immature white blood cells called “blasts”. Leukemia is is 40,000 crore (US$7.24 billion) in FY 2010–11.
a broad term covering a spectrum of diseases. In turn,
it is part of the even broader group of diseases affecting 76. A The Indian Grand Prix (sometimes referred to as the
the blood, bone marrow, and lymphoid system, which Grand Prix of India) is a race in the calendar of the FIA
are all known as hematological neoplasms. Formula One World Championship. It is currently held

Previous Years
Page 44 CLAT & AILET Papers
at the Buddh International Circuit in Greater Noida, is the 11th consecutive appointment of a European to
Uttar Pradesh, India. The first event took place on 30 head the IMF. In 2011, Lagarde was ranked the 9th
October 2011 as the 17th race of the 2011 Formula most powerful woman in the world by Forbes
One season, The new race track was officially magazine.
homologated on 1 September by Charlie Whiting, and
the inaugural race was won by Germany’s Sebastian 82. A The International Criminal Court (commonly referred to
Vettel. as the ICC) is a permanent tribunal to prosecute
individuals for genocide, crimes against humanity, war
77. C Legendary Bengali actor Soumitra Chatterjee was crimes, and the crime of aggression (although it cannot,
conferred with the prestigious Dadasaheb Phalke until at least 2017, exercise jurisdiction over the crime
Award by Vice-President HamidAnsari at the 59th of aggression).
National Awards for 2011 at Vigyan Bhawanon 3rd It came into being on 1 July 2002—the date its founding
May 2012 for his outstanding contribution to the growth treaty, the Rome Statute of the International Criminal
of Indian cinema. Court, entered into force and it can only prosecute
Dadasaheb Phalke Award is India’s highest award in crimes committed on or after that date. The Court’s
cinema given annually by the Government of India for official seat is in The Hague, Netherlands, but its
lifetime contribution to Indian cinema.The Award is proceedings may take place anywhere.
given to a prominent personality from the Indian film
industry, noted and respected for significant 83. B Ajeet Bajaj (born 1965) is the first Indian to ski to the
contributions to Indian cinema. A committee consisting North Pole and the South Pole within a year and has
eminent personalities from the Indian film industry is undertaken travel in 17 countries spanning six
appointed to evaluate the award. Introduced in 1969, continents. In July 2008 he kayaked along the coast of
the birth centenary year of DadasahebPhalke, Greenland as part of an Indo-American team. The
considered as the father of Indian cinema, award is expedition’s aim was to create awareness about the
given to recognise the contribution of film personalities effect of global warming on glaciers.
towards the development of Indian Cinema and for
distinguished contribution to the medium, its growth 84. A Kanchan Chaudhary Bhattacharya,an IPS officer of
and promotion. the 1973 batchis the first woman Director General of
Police of any state. Popular Hindi teleserial ‘Udaan’
78. A The 23rd Arab League Summit was the third one held was based on her life. She retired as the DGP of
in Baghdad and the first one since 1990, before the Uttarakhand state in the year 2007.
start of the Gulf War. The decision to grant the host
rights to Iraq was made at the previous summit in 85. A The 2011 ICC Cricket World Cup was the tenth Cricket
Syria. Among the subjects discussed were the Iraqi World Cup. It was played in India, Sri Lanka, and (for
debts to its neighbors and the uprising in Syria. The the first time) Bangladesh. Pakistan was also
summit marked the first time since the Invasion of scheduled to be a co-host, but after the 2009 attack
Kuwait that an acting Emir (Sabah Al-Ahmad Al-Jaber on the Sri Lanka national cricket team in Lahore, the
Al-Sabah) paid a visit to Iraq. International Cricket Council (ICC) cancelled that, and
the headquarters of the organising committee,
79. B The divisioanl bench of Justice G.S. Singhvi and Justice originally in Lahore, was transferred to Mumbai.
Gyan Sudha Mishra delivered the 2-G judgment. Pakistan was to have held 14 matches, including one
semi-final. Eight of the games (including the semi-final)
80. A The Speaker of Lok Sabha presides over the joint were awarded to India, four to Sri Lanka, and two to
parliament session. Bangladesh.

81. C Christine Lagarde is a French lawyer and Union for a 86. B The United Nations Children’s Fund (UNICEF) has
Popular Movement politician who has been the chosen Bollywood actress and former Miss World
Managing Director (MD) of the International Monetary Priyanka Chopra as one of their brand ambassadors.
Fund (IMF) since 5 July 2011. Previously, she held In a statement, Karin Hulshof, the UNICEF representative
various ministerial posts in the French government: to India said that the signing ceremony will take place
she was Minister of Economic Affairs, Finances and on August 10 in New Delhi, after which the actress
Industry and before that Minister of Agriculture and will be seen supporting UNICEF in the promotion of
Fishing and Minister of Trade in the government of child and adolescent rights.
Dominique de Villepin. Lagarde was the first woman
ever to become Minister of Economic Affairs of a G8 87. D The 2012 Republican primaries are the selection
economy, and is the first woman to ever head the IMF. processes by which the Republican Party selects
delegates to attend the 2012 Republican National
On 28 June 2011, she was named as the next MD of Convention from August 27–30. The series of
the IMF for a five-year term, starting on 5 July 2011, primaries, caucuses, and state conventions culminates
replacing Dominique Strauss-Kahn. Her appointment in the national convention where the delegates vote

Previous Years
CLAT & AILET Papers Page 45
on and select a candidate. A simple majority (1,144) of 94. C Let the number be x.
the total delegate votes (2,286) is required to become According to the question, we get
the party’s nominee. On May 29, according to projected 0.06% of x = 84
counts, Mitt Romney crossed the threshold of 1,144
delegates to become the party’s presumptive nominee. ⇒ x = 140000
∴ 30% of x = 30% of 140000
88. B SalwaJudum (meaning “Peace March” or “Purification
30
Hunt” in Gondi language) refers to a militia in = × 140000 = 42000.
Chhattisgarh, India, which is aimed at countering the 100
Maoist(Naxalite) violence in the region. The militia
consisting of local tribal youth receives support and 95. B Let Q’s share be Rs.x.
training from the Chhattisgarh state government. On Then, P’s share = Rs.2x and R’s share = Rs.4x
July 5, 2011, the Supreme Court of India declared the According to the question, we get
militia as illegal and unconstitutional. The court directed 4x – x = 3675 ⇒ 3x = 3675 ⇒ x = 1225
the Chhattisgarh government to recover all the firearms Hence, the total sum = 2x + x + 4x = 7x
given along with the ammunition and accessories. It = 7 × 1225 = Rs.8,575.
also ordered the government to investigate all instances
of alleged criminal activities of SalwaJudum. The use 96. A Ratio of the areas of two squares = 25 : 36
of SalwaJudum by the government for anti naxal ∴ Ratio of the lengths of their sides
operations was criticized for its violations of human = 25 : 36 = 5 : 6
rights, use of child soldiers and poorly trained
uneducated youth for counter-insurgency roles. ⇒ Ratio of their perimeters = 5 : 6.

89. B As per the Indian Union Budget of 2012-13, the income- 97. D Let the numerator be x.
tax exemption limit for persons below 65 years of age Therefore, denominator = x + 11
is Rs.2,00,000. x
and the original fraction = ⋅
x + 11
90. C The 2011 United Nations Climate Change Conference
x+8 x+8
was held in Durban, South Africa, from 28 November New fraction = =
to 11 December 2011 to establish a new treaty to limit x + 11 + 8 x + 19
carbon emissions. The conference agreed to a legally According to the question,
binding deal comprising all countries, which will be
x+8 3
prepared by 2015, and to take effect in 2020.There = ⇒ x = 25
was also progress regarding the creation of a Green x + 19 4
Climate Fund (GCF) for which a management 25
framework was adopted. The fund is to distribute Hence, the original fraction is ⋅
36
US$100 billion per year to help poor countries adapt to
climate impacts.
While the president of the conference, MaiteNkoana- 98. C Given, x = 2 + 3, y = 2 − 3
Mashabane, declared it a success, scientists and 1 1 1 1
environmental groups warned that the deal was not + = +
( ) ( )
2 2 2 2
sufficient to avoid global warming beyond 2 °C as x y 2+ 3 2− 3
more urgent action is needed.
(2 − 3 ) + (2 + 3 )
2 2
91. A Let P paid Rs.x for the table. 4+3+4+3
= = = 14.
(2 + 3 ) (2 − 3 )
2 2 2
 2
( 3 ) 
Then, 1.1 × 1.12x = 246.40 2
(2 ) −
⇒ x = 200. 
99. B Let the radius of the sphere be r cm.
92. D By putting x = 17, we get the least value of x
i.e. 17 × 19 = 323, a non-prime number. 4
Then, volume of the sphere = πr 3
3
93. C Time taken by the train to cross the bridge
2
and surface area of the sphere = 4πr
Length of the train + length of the bridge
= According to the question, we get
Speed of the train
4
300 + 200 πr 3
= = 20 seconds. 3 = 27
25 2
4 πr

⇒ r = 81.

Previous Years
Page 46 CLAT & AILET Papers
100. B Let the number be x. 105. B Let the rate of growth per annum be r%.
According to the question, we get According to the question, we get
1 1 2
× × x = 12  r 
3 4 48400 = 40000  1 + 
 100 
⇒ x = 144 .
2 2
 22   r 
⇒   = 1 +  ⇒ r = 10%.
101. C 10 = 4 × 2 + 2  20   100 
23 = 10 × 2 + 3
50 = 23 × 2 + 4
1
(105) 10 4 = 50 × 2 + 5 106. D Given, x + =3.
x
216 = 105 × 2 + 6 On squaring both the sides, we get
439 = 216 × 2 + 7
1
Hence, 104 is the wrong number in the series. x2 + +2=9
x2
102. A Let the price of a trouser and a shirt be Rs.x and Rs.y
1
respectively. ⇒ x2 + =7.
According to the question, we get x2
2x + 4y = 1600
⇒ x + 2y = 800 ... (i) 7
And, x + 6y = 1600 ... (ii)
107. D 4x3 + 6x 2 + 2x +
2
On solving (i) and (ii), we get
4x − 3 16x 4 + 12x 3 − 10x 2 + 8x + 20
y = 200
Hence, for 12 shirts, one has to pay Rs.(200 × 12) 16x 4 − 12x3
= Rs.2400. ( −) ( + )
24x3 − 10x 2 + 8x + 20
2 −4
3 2 24x3 − 18x 2
103. A x=   
2 3 ( −) ( + )
2 4
3 3 8x 2 + 8x + 20
⇒x=   
2 2 8x 2 − 6x
6 ( −) ( + )
3
⇒x=  14x + 20
2
21
14x −
3
12 2
⇒ x2 =   ( −) ( + )
2
61
12
−2 2 2
⇒x =  .
3 7
Therefore, the quotient is 4x3 + 6x 2 + 2x + and the
2
104. A Let the marked price of the article be Rs.x.
Therefore, discount = Rs.0.05x 61
remainder is ⋅
S.P. = x – 0.05x = Rs.0.95x 2
(100 + 25) × 380
⇒ 0.95x = 108. A Let the first number be x.
100
Therefore, the second number is 2490 – x.
125 × 380 According to the question, we get
⇒x=
95 8.5% of x = 6.5% of (2490 − x)
⇒ x = 500 ⇒ 8.5x = 6.5(2490 − x)
Hence, marked price of the article is Rs.500.
⇒ 8.5x = 6.5 × 2490 − 6.5 x
⇒ 15x = 16185
⇒ x = 1079
Therefore, the two numbers are 1079 and 1411.

Previous Years
CLAT & AILET Papers Page 47
109. B Originally, the food would have lasted for 200 days. colleges primarily to study. Hence, any distraction in
Number of men who died after 5 days = 30 the same would kill the purpose of such an institution.
∴ Number of men left = 90 Argument II is weak because all great leaders being
Let the food that would have been consumed by 120 from students’ unions leaders is not reason enough
men in 195 days was consumed by 90 men in x days. for keeping the unions. Option A is the correct choice.
∴120 × 195 = 90 × x
116. B Here, only argument II is strong. It is fair to say that
⇒ x = 260 days.
people are mature and responsible by this age and
then they should get married. Inability of changing a
110. C Let the total income be Rs.x. social practice does not justify its existence. Hence,
Amount spent on house rent = Rs.0.2x
argument I is weak. Option B is correct.
Remaining amount = (x – 0.2x) = Rs.0.8 x
Amount spent on household expenditure = 0.7 × 0.8x 117. C X and Y are siblings and siblings are known to quarrel
= Rs.0.56x
often. So, X and Y quarrel often can be logically
His savings = Rs.1,800. deduced from the other two statements. Thus,
∴ x = 0.2x + 0.56x + 1800 statements I, iii and iv are logically related.
⇒ x – 0.76x = 1800
⇒ x = 7500 118. B Mangoes is a subset of fruits. ‘Fruits’ is a subset of
Hence, his total income is Rs.7500. sweet. Thus, ‘mangoes’ is a subset of sweet. This
makes option B correct.
111. A The statement questions if there is a need for making
pay scale and conditions of employees applicable to 119. A Frogs is a subset of amphibians. ‘Amphibians’ is a
private sector employees. Argument I talks of inertia subset of cold blooded. Thus, ‘frogs’ is a subset of
and inefficiency that may creep in to adversely affect cold blooded. So, statements in option A are logically
the spirit of competition among the employees. It’s a related.
serious concern for any organization. Paying more
and still having inefficient employees would be a bad 120. C Short-heighted men is a subset of intelligent.Sudhir is
situation. Hence, it is a strong argument. Argument II is a subset of short-height. So, Sudhir falls in the set of
not strong because there may be other methods that intelligent. Hence, option C is correct.
can enhance institutional loyalty. Moreover, paying
more does not relate to greater dedication and loyalty. 121. D Valley and depth are not antonyms while all other
Option A is the correct choice here. pairs of words are antonyms of each other. Thus,
option D is the odd one out.
112. A Only argument I is strong in this case. Transparency
and accountability are important parameters for the 122. C All options except C have the second part as an
success of any administration. However, lack of extension of the first, like hand being joined to the
flexibility and initiative is not a very big problem as body, foot to the ankle and finger to the wrist. Eye and
compared to the benefits that it may have. ear are separate and so option C is different from the
others and thus, correct.
113. C Both arguments here are equally strong. Argument I
talks about the possibility of favoritism which is a major 123. B The second animal in each option is a prey of the first.
issue. It may beat the entire purpose of assessing Goat doesn’t eat hen. Option B is the correct choice.
students; questions on its fairness can be raised.
Argument II is strong because it is very important for 124. D Conclusion I: All girls are students.
the teaching faculty to have some control over the This is same as statement I given in the question. So,
assessment of students. In the absence of internal conclusion I follows.
assessment, students may not value their teachers Conclusion II: Some students are girls.
because they would know that their marks would It is given that all girls are students, so there will be
depend only on external assessment. Thus, option C some students who are girls. Thus, conclusion II
is the correct answer. follows.
Conclusion III: Some students are doctors.
114. A Only argument I is strong as it talks of punctuality and It is given that all doctors are students, so there will be
discipline, the two most important virtues of student some students who are doctors. Thus, conclusion III
life. Argument II is incorrect as it says military training also follows.
should be given only to physically fit students. On the Conclusion IV: All doctors are girls.
contrary, students who are physically unfit would Girls and doctors both are subsets of students. But
require it the most. Thus, option A is correct. we don’t know how doctors and girls are related.
They can be disjoined, intersecting or overlapping sets.
115. A Argument I is strong since it talks about the distraction Thus, conclusion IV doesn’t follow.
of students from academic pursuits. Students come to Hence, option D is correct.

Previous Years
Page 48 CLAT & AILET Papers
125.B* The answer given by CLAT is option B. However, followed by rain (effect). Option B is incorrect because
none of the options given have the correct combination. it says that sometimes there is rain (effect) and no
This is explained as following. Conclusions II, III and IV clouds (cause). It does not make the statement false
follow. Conclusion II is same as statement II. since it is not mentioned that there is only one cause
Researchers are a subset of sociologists. Statement II for rain i.e. clouds. It can rain because of some other
says some researchers are professors. Combining reason too. Option C is incorrect as it doesn’t affect it
these two, we can definitely say that there is atleast in any way.
one professor who’s a sociologist. Hence conclusion
III also follows. Conclusion IV follows from statement Solutions for questions 130 to 132: According to the
I, since all researchers are sociologists, so there is at information given, the following can be concluded.
least one sociologist who’s a researcher. Thus, the
right answer should be II, III and IV which is given in
Friends Good Bad
none of the options.
W Hindi, Science English,
126. B Conclusion I: No democracy is a monarchy. Mathematics
Both the statements do not talk about the relation X Hindi, English
between democracy and monarchy. Democracy is an Science,
intersecting set with dictatorship while dictatorship Mathematics
and monarchy are disjoined sets. So monarchy and
Y English, Social Studies
democracy can be anything, from disjoined and
Science Mathematics
intersecting to overlapping. Hence, conclusion I does
not follow. Z English, Social Studies
Conclusion II: No dictatorship is a democracy. Mathematics,
It does not follow because we can conclude from Science
statement one that at least one dictatorship is a
democracy. 130. A W is not good in Mathematics but good in Hindi. Hence,
Conclusion III: Some democracies are monarchy. Option A is correct.
It does not follow for the same reason as given for
conclusion I. 131. C From the table above, it is seen that Y and Z both are
Conclusion IV: Some dictatorships are democracies. good in English and Science.
This conclusion follows because we can deduce from
statement one that there is at least one dictatorship 132. B Only option B is true that all four friends are good in
that is democracy. science.
Hence the correct answer is option B.
133. A Athletes and vegetarians are disjoined sets. Players
127. C* CLAT has given the answer as C, but it does not are a subset of athletes, so it can be concluded that
follow from the question. It is incorrect as the given no player is a vegetarian. Hence, option A is correct.
statements just say that cheese is bad for people with
high cholesterol. It doesn’t tell us whether people with 134.B* This statement is similar to ‘if A then B’ type where A is
high cholesterol eat it or not. It is possible that high the cause and B is the effect. It says all persons who
cholesterol patients, despite knowing that it is bad for have done any creative work (cause) can be
them, still eat it because they like it or are simple responsible critics (effect). It doesn’t mean that creative
unaware of its ill effects and so eat it. Option A and B work (cause) is the only way to be a responsible
are incorrect as ‘Sumit doesn’t eat cheese’ may mean critic (effect). This is so because the effect can have
that probably he doesn’t like it or doesn’t eat it for no or some other cause too. Hence, no conclusion
some other reason, not necessarily that he has high about Z can be made given the two statements.
cholesterol or it is bad for him respectively. Hence,
Option D is the correct answer. 135. A Option A just rephrases the given statement; therefore
it logically follows from it. The given statement is of the
128. B The statement says that all democrats are secularists. form ‘A (one who has squared a circle) is not B
Option B which says ‘My father is a democrat but he (mathematician)’. It can be logically concluded from
is not a secularist’ makes the statement false because here that ‘No A (no one who has squared a circle) is
there is one democrat who is not a secularist. Thus, B (mathematician)’. Hence, option A is correct.
we can’t say all democrats are secularists.
136. B Only reason I is correct here. The Supreme Court of
129. A The statement is ‘if A then B’ type. It says that India is encouraging Public Interest Litigation so that
whenever there is a cloud (cause), there is rain people from different and disadvantaged sections of
(effect). Option A makes the statement false. It says the society can benefit by filling PILs which are in their
that there are times when clouds (cause) are not interest. PILs have nothing to do with quickening the
pace of justice.

Previous Years
CLAT & AILET Papers Page 49
137. C The paragraphs points to the fact that though Yoga is follows the given argument exactly and hence is the
becoming a very popular form of exercise, it would correct answer.
not suit all fitness enthusiasts. Refer to the line, Option D: This option has both the premise (John is his
“Therefore, evaluate your fitness requirements before father’s favorite son) and conclusion (John is his
joining yoga classes”. Hence, option C is correct. father’s favorite son) as the same. Therefore, this
option does not follow the same line of reasoning.
138. D Refer to the lines, “Statistics...can also be manipulated
to perpetuate untruth...” and “Data has power to 146. D Both the arguments given in the question are weak.
mislead people”. Option D directly follows from these Judicial activism leading to executive dictatorship is
lines and thus, is the correct choice. an exaggeration of the situation. Similarly, Judiciary
should stay in constitutional limits is not reason enough.
139. C Refer to the line, “At first, development of new Why it should stay within constitutional limits or what
technology was slow...there has been a tremendous effects it will have are not mentioned, which makes it
growth in technology sector”. These lines suggest that a weak argument. Hence correct option is D.
today technologies are developing rapidly. Hence,
option C is correct. 147. A Argument I is strong since it is important to have
impartiality in the judicial system which can be done
140. C Reduction of import duty (II) on toys has resulted in the by making it independent of executive and legislature.
decrease in their prices (I). Thus II is the main cause Argument II is a weak argument since it is an excuse
and I is the main effect. Option C is the correct answer. and not a solid reason. Legislature and Executive may
develop inertia even when they have judiciary under
141. C Reserve Bank of India changes the interest rates to its purview. Thus, option A is correct.
cater to the rise and fall of inflation rate in the country.
Thus, decrease in inflation rate (I) is a result of increase 148. C Both the given arguments are strong. The introduction
in interest rates (II) by the government. Hence II is the of e-governance at every level of public administration
main cause and I is the main effect. Option C is the may help weed out corruption. This is a very strong
correct choice. reason in support of e-governance. On the other hand,
it may lead to unemployment which is a very serious
142. D The introduction of Bar Examination by the Bar Council problem. Hence, option C is correct.
of India is an effect which can be caused by increasing
number of students opting for legal education (cause) 149. B Only argument II is strong since there is all possibility
along with some other reasons. Thus, II is the main that rich and powerful nations may dominate the world
effect but I is not the main cause. government, which beats the whole purpose of forming
such a government. Argument I is weak because it
143. C The phenomenon of global warming is a serious talks of inter-state conflict. There is more to a world
problem (cause) and the rise in the sea level is a major government than eliminating inter-state conflicts. There
effect of the same. So, II is the main cause and I is the are other international organizations like the UN which
main effect. aim to eliminate such conflicts. Hence, elimination of
inter-state conflict is not a strong argument in favor of
144. A Due to the unregulated nature of the financial world government.
institutions (cause), the economy is getting affected.
As a result, the world is passing through an economic 150. A The first statement means that according to some
crisis (effect). Thus, I is the main cause and II is the philosophers, a concept which can’t be verified can
main effect. still be considered valid because of the inner reasoning
attached to it and which gives it a noble character.
145. C The argument is of the form A (Himalayan sparrows) The given statement is true in light of the above
is a subset of B (disappearing), A is C (Indian bird), statement. It says that every person has ‘certain
and thereforeB is C. inherent and inalienable rights’ (i.e. concepts) which
Option A: It is completely out of sync with the given can’t be verified i.e. ascertained whether they are
argument. It has four elements viz. industrialists, most justified or not, right or wrong. Because of the nature
taxes, Z and wealthy man. The given argument has of such rights and the fact they are inalienable to an
only three elements and so both are not on the same individual, these rights are considered valid and given
line of reasoning. the status of protection by the Rule of Law.
Option B: Here pineapple is a subset of fruit. Mango is
also a subset of fruit. But they necessarily can’t be 151. C As per the Constitutional structure, Court is the
the same sets. Hence, the reasoning is flawed and custodian of fundamental rights. The principle clearly
not on the same lines as in the argument. states that only Parliament or State Legislature can
Option C: This option can be broken down as A (Snow enact laws. Courts can only interfere if there is violation
tigers) is a subset of B (endangered species), B is a of fundamental right. It is an interpreter of the
subset of C (protected) and so A is a subset of C. It Constitution of India and not a law making body.

Previous Years
Page 50 CLAT & AILET Papers
152. B When Shyam replied that lowest price for car is 20 166. B All illegal contract are void, but not all void contracts
lakhs that was an invitation to offer and Ramanuj had are illegal.
to make another offer to which Shyam could have
given an acceptance or refusal. 167. B Supreme Court Bar Association declared 26 November,
Law day in the year 1979.
153. B Section 11 of Indian Contract Acts provides the
definition of “who are competent to contract”. In the 168. A The eternal nature of Fundamentals Rights ensure
case of Mohri Bibi v. Dharmadas Ghose it was held states law’s conformity with its law making power.
that any agreement with a minor is void ab initio.
169. A The state shall frame its policy in lines with Directive
154. C Burden of proof lies on the person who first asserts Principles of State Policy (DPSP) keeping inview the
the fact, and not on the one who denies that fact to be scheme of Fundamentals Rights envisaged under part
true. The responsibility of the defendant to prove a III of the Constitution of India.
fact to be true would start only when the authenticity
of the fact is proved by the plaintiff. 170. C The government shall (through financial aids) promote
religious and minority institutions. As mandated under
155. D Art 14 is a general provision which ensures equality Constitution of India.
but simultaneously Art. 15 have been interpreted by
the Courts as an exception to Article 14. Art. 15 talks 171. A The Supreme Court of India is a final interpreter of the
of positive discrimination wherein unequals are to be Constitution of India.
treated differently.
172. D Since the member of Lok Shabha are elected by the
156. A Sharman acted on necessity in order to avoid a bigger people of India the appropriation of public money (viz.
harm and acted in good faith without any criminal money bill) is entrusted upon them.
intention, hence, he has committed no offence.
173. A Damages are assessed on the basis of actual loss
157. A In the present case the police can book Mr. Tiwari for suffered by the party and is meant to reinstate the
drunken driving but not for willful negligent driving. party in a position, had the loss would not have occured
to him.
158. A Y moved the property (severed it from the ground)
with the dishonest intention of taking it. Therefore, Y 174. A Price is set by market forces of demand and supply.
can be prosecuted for theft.
175. A Although Y was insane and cannot be responsible
159. B ‘Injuria Sine Damno’ means violation of legal rights for his actions, X was defending himself.
without causing any damage i.e. there is injury though
there is no damage. Even in the absence of any 176. D Section 34 of IPC defines common intention.
damage, if there is violation of legal right, the plaintiff
can claim damage. 177. A Regardless of a contract, a non-gratuitous act must
be compensated.
160. * The options are incorrect. The correct answer should
be “X has committed culpable homicide NOT amounting 178. A The state can impose ‘reasonable restrictions’ upon
to murder”. Besides, there is nothing in the facts to the freedom of speech.
show that X has committed the act in unsound state of
mind. 179. A Imperative theory advocates “Law is a command of
the sovereign”.
161. C X can be prosecuted because he was ignorant about
the new notification that was passed and the law 180. C Unlawful assembly has been defined in Section 149
says that ignorance of law is non-excusable. of IPC.

162. A The servant did not know about the offer when he 181. B The preamble state as following: “WE, THE PEOPLE
traced the missing child. Hence, his action would fail. OF INDIA, having solemnly resolved to constitute India
into a SOVEREIGN SOCIALIST SECULAR DEMOCRATIC
163. B Here the word luck has not been defined in the REPUBLIC and to secure to all its citizens: JUSTICE,
agreement. Hence it is void of uncertainty. social, economic and political; LIBERTY of thought,
expression, belief, faith and worship; EQUALITY of
164. C X cannot be held liable because he did not say anything status and of opportunity; and to promote among them
about the mental state of the horse. all FRATERNITY assuring the dignity of the individual
165. A X can be held responsible since he had no lawful and the unity and integrity of the Nation”.
justification for removing the car from the garage.

Previous Years
CLAT & AILET Papers Page 51
182. D Art. 21A. The State shall provide free and compulsory 192. C The promise had been made jointly by P, Q and R.
education to all children of the age of six to fourteen Since Q has been declared insolvent he is out of the
years in such manner as the State may, by law, picture and P can claim Rs. 1500 from R.
determine. (Eighty -sixth Amendment Act, 2002)
193. B Law does not take account of trifle and trivial acts. If
183. C Affirmative actions are basically the measures taken Y’s children consumed 1 kg. of sweets, no offence is
by the State which are aimed to ensure equality of made out on part of Mr.Y hence he can not be held
status and opportunity in the society. These are also liable in law.
called, positive actions of the state.
194. B Article 19 states Protection of certain rights regarding
184. A A principle of government that defines the relationship freedom of speech, etc.—(1) All citizens shall have
between the central government at the national level the right - (g) to practice any profession, or to carry
and its constituent units at the regional, state, or local on any occupation, trade or business; Article 21 talks
levels. about Right to protection of life and personal liberty

185. C Rafique Bhikan v. Union of India 195. B Y is guilty of attempt to commit theft because Y with a
dishonest intention acted in order to steal the jewels.
186. D An agreement with an unlawful object is against public
policy and hence, unlawful. 196. B Raju will be held guilty of Criminal breach of trust as he
misappropriated by running away with the money
187. B One of the members in his private capacity informed which was entrusted upon him by the lady.
Ramu about the decision of the body. Hence, Ramu
cannot claim damages 197. D

188. C The fact that Mr.X got his pocket picked because of 198. A The emergent situation in this case is that Mr. Samay is
overcrowding is a remote consequence and hence to severely hurt and hence should be operated first.
that extent railway authorities can not be sued.
199. A Republic means head of the State is elected by the
189. C X had not committed any offence and had no intention people. Secular means that the State does not
to kill Z and in good faith he was trying to save Z. recognize any religion as the religion of the State.
Democracy is a form of government where will of the
190. A No person can obtain a permanent injunction people prevails, i.e. they are free to elect their
restraining the theatre from running the film just on the representative.
ground that his personal religious feelings and
sentiments are being hurt. 200. B The answer given in CLAT key sheet is D but it should
be B. A Bill regarding admission or establishment of a
191. D Concurrent List = Australia new State, formation of new States and alteration of
Rule of Law = Britian areas, boundaries or names of the existing ones
Directive Principles of State Policy = Ireland. (Article 2,3 and 4) requires to be passed by simple
Procedure established by Law = Japan majority.

Previous Years
Page 52 CLAT & AILET Papers
CLAT Solutions 2013
1. B The whole passage deals with governance and how maintain their governance and rule. Phrases like
the ruling class controls education to maintain it. Option “handmaid of the ruling class”, “while the
(A) is unlikely to be the answer because it does not landlords…fought duels for the slightest offence” and
talk about rights while the phrase ‘The Divine right “restricting the function of the state to a mere keeping
Theory’ contains the word. Option (C) and (D) are of laws” somewhere suggests that the author is
unlikely answers because they do not talk about voicing his opinion about the matter. Moreover,
governance. Therefore, option (B) turns out to be the ‘unconcerned’ shows ‘lack of care because the topic
best of all answer choices and is hence the correct is unimportant’ and therefore, it has a slight negative
answer. connotation. So, option (A) appears to be the correct
answer and not option (B); the answer is contentious
2. B Refer to the following lines in the passage – “During nevertheless.
the Renaissance, education passed…into the hands
of the prince… Under the control of the monarch, 9. B ‘Gospel’ means a set of ideas that somebody believes
education…” From this, we can infer that the prince or in and tries to persuade others to accept. Therefore,
the monarch ruled during the Renaissance. Hence, option (B) is the correct answer. Option (D) is incorrect
option (B) is correct. because it is vague and incomplete.

3. D Refer to the following lines – “These gospels and 10. C ‘Vogue’ means a fashion for something. Therefore,
teachings were no other than a philosophy for the option (C) is correct.
maintenance of the existing society. It taught the poor
man to be meek and to earn his bread with the sweat 11. A The correct spelling is “paraphernalia”. Hence, the
of his brow, while the priests and the landlords lived in correct answer is option (A).
luxury and fought duels for the slightest offence.”
From this, we can infer that the poor man was taught 12. B The correct spelling is “entrepreneur”. Hence, the
to serve the rich, who lived in luxury. Therefore, option correct answer is option (B).
(B) is correct. All other options are incorrect because
the passage gives no evidence to support them. 13. C/D The correct spelling is “onomatopoeia”. Hence, the
correct answer can be both option (C) and option (D)
4. B Refer to the following lines from the passage – “During as they contain the same spellings.
the Christian era, the ecclesiastics controlled the
institution of education and diffused among the people 14. C The correct spelling is “haemorrhage”. Hence, the
the gospel of the Bible and religious teachings.” correct answer is option (C).
Ecclesiastics mean a priest or clergymen. Therefore,
option (B) is the correct answer. 15. B The correct spelling is “dyslexia”. Hence, the correct
answer is option (B).
5. C ‘Infallibility’ means never wrong; never making mistakes.
Therefore, option (C) is correct. 16. C If the clause, other than the one containing ‘if’ or
‘unless’, contains ‘will +verb form 1’, the conditional
6. A The idiom ‘sweat of one’s brow’ means by one’s hard clause should be in simple present tense. For example,
work. Therefore, option (A) is correct. if you do your work in time, I will take you to the zoo. All
other options are incorrect due to the same reason.
7. A The phrase ‘Laissez Faire’ means the policy of allowing
private businesses to develop without government 17. C Some clauses containing ‘if’ are like hypothesis,
control. Therefore, option (A) is correct. suppositions or desires and in such cases, we use
past tense forms to talk about present and future. In
8. A* Options (C) and (D) are clearly incorrect because such cases, if the verb is a form of ‘be’ in the conditional
nothing in the passage tells us that the author is clause, the form of ‘be’ to be used is ‘were’ irrespective
aggressive or frustrated. Out of options (A) and (B), of the subject. For example, if she were not a writer,
(A) is better of the two because the author appears she would be an actress. Therefore, the correct
concerned about the idea that education had always sentence will be ‘If I were rich, I would help all the poor
been a puppet that the ruling class could exploit to people’. Hence, the correct answer is option (C).

Previous Years
CLAT & AILET Papers Page 53
18. B The blank will take a verb that is in simple past tense Kauai. Hence, option (C) is the correct answer.
since the statement contains a specific time reference.
This renders option (A) incorrect since ‘have heard’ 26. C d is the opening sentence since it introduces the topic
suggests that the news was heard at an unknown of the paragraph: starting and managing small business.
time. Option (C) and (D) are incorrect because ‘hear’ a follows d since it discusses the backgrounds of
belongs to the category of verbs that generally are those ‘people who start up a business’ mentioned in d.
not used in their ‘ing’ verb forms like see, love, ac is a mandatory pair. ‘Both the backgrounds’ in c
remember, etc. Had option (C) been ‘was listening to’, refers to the two backgrounds discussed in a, i.e.,
it would have been correct. Hence, option (B) is the ‘business family background’ and ‘steady professional
correct answer. family background’. b will follow c because c says
that people from both the backgrounds will face
19. C Grammatically, all the options are correct. However, difficulty and b adds that the difficulty faced by each
logically, only option (C) fits in best because one is will be different. Hence, the correct sequence is dacb,
likely to be confident while speaking about his making option (C) the correct answer.
prospects. ‘Confidentially’ (secretively and with trust),
‘consciously’ (knowingly) and ‘conscientiously’ 27. B a starts the paragraph by introducing the topic of
(carefully) do not fit in the context better than discussion: venture capital. d follows a because where
confidently. Hence, option (C) is the correct answer. a talks about the success brought about by venture
capital, d carries the idea forward by talking about its
20. D If a statement is negative, then the question tag (short impact in the Indian context. bc is a mandatory pair
questions that we put at the end of sentences) is because ‘Despite this’ in c refers to the information
positive and vice-versa. Also, the question tag takes given in b. c says that an average Indian entrepreneur
the same verb as used in the preceding part of the hardly appreciates venture capital, despite ‘the latter
statement. For example: He did his homework, didn’t being used by several companies’, the information
he? Similarly, in the given sentence, the verb is ‘is not’. mentioned in b. Thus, the correct sequence is adbc,
Therefore, the correct question tag would be ‘is he’. rendering option (B) the correct answer.
Hence, option (D) is the correct answer.
28. B bac is a mandatory sequence. b talks about the
21. C ‘We’ is the subject in the phrase ‘we should unparalleled progress made in the field of medicine
never…means’. Therefore, the personal possessive and a tells us about the manner in which the progress
pronoun will be in accordance with ‘we’. Hence, option was made - rapid and breath taking. c follows a by
(C) is the correct answer. citing a few examples of the progress. This sequence
is present only in option (B). Hence, option (B) is the
22. D Option (D) is the correct answer because the correct correct answer.
usage is ‘to request someone’, not ‘request to/onto/ of
someone’ (to do something) . Therefore, option (C) is 29. D ba is a mandatory pair because b says that earthquakes
incorrect. usually do not kill people directly and a tells us how
earthquakes kill indirectly. Both c and d cite concerns
23. C Sense of humour cannot be ‘quick’ or ‘beautiful’. during earthquakes, they should therefore come
Therefore, options (B) and (D) are incorrect. ‘Better’, together and follow a. Therefore, option (D) is the
being a comparative adjective is incorrect in this context correct answer.
because the statement shows no comparison. Hence,
the correct answer is option (C). 30. D cd is a mandatory pair. ‘This’ in d refers to the
‘Winchester disk system’ in c. a should precede b
24. C The statement quoted is a universal fact and should since a introduces the Winchester or hard disk drives
therefore be written in simple present tense. Hence, and b gives more information about them. Hence, option
option (C) is correct. (D) is the correct answer.

25. C The statement before the conjunction ‘before’ should 31. A ‘El Dorado’ refers to a place of great wealth which the
be in past perfect tense because the task of doing the 16 th century explorers believed to exist in South
work had already finished before the guests arrived. America. Hence, we can say that it is an imaginary
Past Perfect is used to express that an action occurred place, making option (A) the correct answer.
before another action in the past. For example, I had
never seen such a beautiful beach before I went to

Previous Years
Page 54 CLAT & AILET Papers
32. A The phrase could not be located anywhere. Therefore, 42. *Correct answer is not present among the given
collaborating the meanings of ‘quantum’ and options.
‘ramification’ individually can lead to the correct answer Required number = HCF((926 – 2), (2313 – 3)) = 462.
. ‘Quantum’ means the smallest amount of something.
‘Ramification’ is the result of an action, decision, etc. 43. *Correct answer is not present among the given
Therefore, it can be conclude that ‘quantum ramifactus’ options.
means the amount of damage suffered. There is Let the amount on which discount is being offered be
another phrase ‘quantum damnificatus’, which means Rs. 100.
an issue directed by a court of equity to be tried in a SP after the first discount of 15% = Rs. (100 – 15)
court of law, to ascertain by a trial before a jury, the = Rs. 85
amount of damages suffered by the non-performance
 5 
of some collateral undertaking which a penalty has SP after second discount of 5 % = Rs.  85 − × 85 
been given to secure. When such damages have thus  100 
been ascertained the court grants relief upon their = Rs. 80.75
payment. In case the intended phrase is ‘quantum
100 – 80.75
damnificatus’ and not ‘quantum ramifactus’, the answer Hence, net discount percentage = × 100
still remains option (A). 100
= 19.25.
33. D ‘Corpus delicti’ refers to the material evidence upon
which a crime has been committed or evidence that 44. Correct answer is not present among the given options.
can be seen, for example a dead body. Hence, option Let the CP be Rs. x.
(D) is the correct answer. Loss = Rs. (x – 2345)
x − 2345
34. C ‘Vis-a-vis’ is a French word that literally translates to Loss percent = × 100 = 19
face-to-face. Hence, option (C) is the correct answer. x
⇒ x = 2895.
35. A ‘Carte blanche’ is a French word, which means to
give complete authority. Hence, option (A) is the correct
1
answer. 45. A Required area = × 1× 1.5 = 0.75m2 .
2
36. C ‘To blaze a trail’ means to be the first to do or to discover
something that others follow. Hence, option (C) is the
π
correct answer. 46. C The angle in a semicircle is .
2
37. A ‘A snake in the grass’ refers to a person who pretends
to be your friend but who cannot be trusted. Hence, 47. D Total amount of food available = (250 × 33 × 125)
option (A) is the correct answer. Option (B) is incorrect = 1031250 gm.
because an unreliable person is someone who cannot Total number of students in the school after new
be trusted. He may or may not pretend to be a friend. students joined = 250 + 80 = 330.
38. A ‘Have too many irons in the fire’ means to do too many  1031250 
things at once. Hence, option (A) is the correct Hence, the required number of days =  
 330 × 125 
answer.
= 25.
39. B ‘A fair weather friend’ is a friend whose friendship
cannot be relied on in times of difficulty. Hence, option 48. * As we do not know that whether each of the students
(B) is the correct answer. reads at least one of the two given languages, the
answer can not be determined.
40. D ‘A panacea’ is a solution or remedy for all difficulties or
diseases. Hence, option (D) is the correct answer. 49. C For the given equations to have infinite number of

solutions, k = 3 = k − 3
p 12 k k
41. B A rational number is of the form , where p and q are Hence, k = 6.
q

2 50. C The daily mean expenditure on food


integers and q ≠ 0. Hence is an irrational number.
3
=
(125 × 4 + 175 × 5 + 225 × 12 + 275 × 2 + 325 × 2)
4 + 5 + 12 + 2 + 2
= Rs. 211.

Previous Years
CLAT & AILET Papers Page 55
51. D Let the number of green marbles in the jar be x. 57. C Part of the tank filled by pipe A and pipe B together in 3
x  1 1 33
The propability of a drawn marble being green = hours =  + ×3 = .
24  10 12  60
x 2 27
⇒ = Remaining part of the tank to be filled = .
24 3 60
Time taken by B to fill the remaining part of the tank
∴ x = 16.
Hence, the number of blue balls in the jar = 24 – 16 = 8.  27 
 60  27
52. D Population of the city after two years = = hours i.e. 5 hours and 24 minutes.
 1 5
 12 
 2 
2  
= 250000 ×  1 +  = 260100.
 100 
Hence, the required growth = 260100 – 250000 58. B Total area of the road
= 10100. = (100 × 5 + 80 × 4) − 5 × 4 = 800m2
53. A Point (x, y) in OXY plane is equidistant from (-1,1) and ∴ Required cost = 800 × 10 = Rs. 8000.
(4,3).
59. D Let the cost price of 1 article be Re. 1.
∴ ( x + 1) + ( y − 1) = ( x − 4 ) + ( y − 3 )
2 2 2 2
Cost price of 10 articles = Rs. 10.
Selling price of 10 articles = Cost price of 11 articles
⇒ 10x + 4y = 23. = Rs. 11.
(11 − 10 ) × 100 = 10%.
54. A Sum of first 15 multiples of 8 Profit =
10
= 8 × 1 + 8 × 2 + 8 × 3 + ... + 8 × 14 + 8 × 15
= 8 × (1 + 2 + 3 + ... + 14 + 15) = 8 × 120 = 960. 60. A Let the angles of the quadrilateral be 3x, 4x, 5x and
8x.
55. B The volume will be the same in both the cases. Sum of all angles of a quadrilateral = 360º.
∴ π × (1)2 × 30 = π × (r)2 × 300 ∴ 3x + 4x + 5x + 8x = 360o

⇒r =
1 ⇒ x = 18o.
cm
10 Hence, the smallest angle = 3x = 54º.

2 61. D Belgium
Hence, the required diameter = cm.
10
62. B PSLV C 11
56. B D
63. A Israel
5
64. B Sri Lanka

A E 65. A Mexico

66. D Danish 10th Century King


10
67. A Meghalaya, Manipur

68. B Chenab
B 4 C
69. A Mohan Das Karam Chand Gandhi
Let AB and DC be the two poles standing on the ground
and AD be the pole touching the top of both the poles. 70. B Brazil

∴ DE = (5 ) 2
− ( 4 ) = 3m.
2
71. D 5.1 %
∴ Required area = Area of the trapezium ABCED 72. C Odisha
1
= × (10 + 13) × 4 = 46m2. 73. B 1954
2

Previous Years
Page 56 CLAT & AILET Papers
74. B Ms Wenxia Yu 103. C Old Stone Age

75. C Kamal Hasan 104. B 1911

76. B Shooting 105. D J B Kripalani

77. C Wrestling 106. B 1986

78. D 6 107. A 2010

79. C Argentina 108. C Maues

80. B Neil Armstrong 109. A mgh

81. B Dilsukhnagar 110. A Tehran

82. A 14-1-2013 For questions 111 to 113: From the given information, the
following table can be drawn,
83. B 8th March
A(White) F(Black)(Shortest) D(Golden)
84. D XI

85. B Haryana
C (Bronze) B(Metallic) E(Silver)(Tallest)
86. D Venezuela
111. D 112. D 113. B
87. C Mexico
114. B Since R wants either Foreign or Human Resources or
88. C Palaniappan no portfolio, options A and D can be ruled out. U does
not want any portfolio if S gets one of the five. But in
89. D New York City option C, both U and S have got portfolios. Thus, option
C can also be ruled out. Hence, the correct option is
90. D Shah Alam II option B, as it doesn’t violate any condition.

91. B Andhra Pradesh 115. D Since Q says that if S gets Industry and Commerce or
Rural Development then she
92. B Distance must get the other one, option D is not a valid
assignment.
93. D The vertical line passing through the C.G. of the tower
falls within the base 116. B The relationship between the key pair is of cause-
effect. ‘Action’ is a cause and ‘reaction’, its effect.
94. A John Milton Similarly, ‘assail’, which means ‘to attack violently’, is a
cause and ‘defend’ is its effect because one defends
95. B Padmanabha Swamy Temple of Thiruvananthapuram oneself only when one is attacked. Therefore, option
(B) is correct. Option (A), like the key pair, is also a
96. A Henry Dunant pair of antonyms but it is not the answer because
option (B) also exhibits the cause-effect relationship
97. C 8th September that (A) does not. Other options are clearly incorrect
because they do not exhibit that relationship.
98. B Reeva Steenkamp
117. A ‘Misery’ which means great suffering of the mind or
99. C Italy body is a degree higher in intensity in comparison with
‘sorrow’. Similarly, ‘Obsession’ is also a degree more
100. A Reservation in matters of promotions for Scheduled intense than ‘love’. Therefore, option (A) is correct.
Castes and Scheduled Tribes Option (C) and (D) are incorrect because there is no
difference in terms of degree between the paired
101. B 24 words in each pair. Option (D) is incorrect because
one feels hatred for one’s enemy.
102. D King George VI

Previous Years
CLAT & AILET Papers Page 57
118. B Just as audience are people that watch dramas, 131. D The solution to the problem at hand could have been
spectators are those that watch games. Therefore, suggestions that would have aided Indian students to
option (B) is correct. Critics critique pieces of art, improve in science and mathematics. However, I and II
vagabonds (a person who has no home or job and are impediments that are capable of further hindering
who travels from place to place) have nothing to do the desired improvement. Therefore, option (D) is
with brawl (a noisy and violent fight involving a group correct.
of people, usually in a public place) and actors act in a
movie. 132. B I does not follows because it cannot be understood
why the government can’t forbid child labour by taking
119. A The idiom ‘the nuts and bolts of something’ means the the requisite measures. Also, not making a law that
basic practical details of a subject or an activity and must be made, only because you think it cannot be
‘nitty-gritty’ also refers to the basic or most important enforced is unjustified. Now, let’s look at II. If proper
details of an issue or a situation. All other options are education that enlightens the people towards the
unrelated and therefore, option (A) is the best of all importance of education is imparted, there is a
the answer choices. possibility that people start sending their children to
school and the word ‘may’ in II talks about a possibility.
120. C Just as an author writes a book, a composer composes Therefore, II is a valid course of action. Hence, option
or writes symphonies, which is a long complicated (B) is correct.
piece of music for a large orchestra, in three or four
main parts called movements. Therefore, option (C) is 133. D I may or may not follow because the statement does
correct. Rains can cause flood, switch can be used not tell the details of Kyoto protocol. II also does not
to turn on lights and a song contains music. follow because it is unrelated to the statement. Hence,
option (D) is correct.

121. B Moni is the daughter of my wife’s brother. Hence she 134. A The problem at hand is high rate of school-dropouts. I
is my wife’s niece. follows because if parents are aware of the
importance of education, they will try to educate their
122. A Annu is my cousin. Hence Pari should call Annu as her children instead of seeking their children’s help to
Maternal aunt. increase their income. II does not follow because it is
unrelated to the statement. Hence, option (A) is correct.
123. B Abahi is Rajiv’s cousin. Hence Rajan will be Rajiv’s
nephew. 135. C I follows because if the problem of smoking tests the
will power of the smoker, it can be inferred that it is
124. B Daughter of Deepak’s father’s wife will be his sister. very difficult for a smoker to quit smoking even if he
The brother of her sister will be his brother. Hence, wills. II also follows because the statement refers to
the person playing with football is Deepak’s brother. smoking as ‘one of those human weaknesses’. So,
option (C) is correct because both the statements
125.* If the grandfather is the woman’s mother’s father, then follow.
Rajesh will be the father of the woman. If the
grandfather is the woman’s father’s father, then Rajesh 136. A In the given series, each term is the sum of previous
will be woman’s uncle. Hence, either of the options A two terms. Hence the next term will be 34.
or B is possible.
137. B The letters are increasing by 4 in alphabetical order
For questions 126 to 130: The following table shows the and the corresponding numbers are increasing in AP
days and corresponding fruit prescribed for that day. with common difference of 5. Hence the next term will
Sunday Monday Tuesday Wednesday Thursday Friday
be Q-30.
Banana Fast Grape Pomegranate Papaya Apple
138. D Each term in the sequence, starting from the second
term, is 2 more than twice the previous term. Hence,
126. A the missing term will be 302.

127. D 139. C The given sequence is a combination of two


sequences 2, 4, 6, 8 and 1, 3, 5, 7.
128. B
n
129. D 140. B Each term is of the form of n . Hence the next term
5
will be 5 i.e. 3125.
130. A

Previous Years
Page 58 CLAT & AILET Papers
For questions 146 to 150: The following diagram represents
141. A G G – G ra vy the possible sitting arrangements.
V V – Veg eta bles D D
L F B B F
L – L un ch

As we can see in the Venn diagram given above,


option (A) follows and therefore, is the correct answer. H G or G H

142. D G D – G o od D irecto rs A
GD I E E A
D – D irecto rs
K D I – Inte llig en t C C
K – K ara n Jo ha r 146. C

147.* We can see that both options A and B are possible.


As we can see in the Venn diagram given above,
option (D) follows and therefore, is the correct answer.
148. D
In case, they are taking ‘good directors’ and ‘directors’
as a single category, the following Venn diagram can
149. C
be made. In that case option (B) would be the correct
answer. However, the directions clearly state that the
150. B
statements have to be true even if they seem to be at
variance with commonly known facts. Thus, one
151. D As per the principle, a promise is legally binding only
cannot make such assumptions as mentioned above.
when acceptance is unconditional. Shyam's
I acceptance was accompanied by another condition
which amounts to counter offer. Hence, Shyam cannot
D I – Inte llig en t
compel Ram to sell his house.
GD D – D irecto r
K 152. B Although Smith's offer is negligible when it is compared
G D – G oo d D ire ctor
with Anson's offer, however It does have a substantial
K – K a ra n Jo ha r value in the eyes of law and hence is a valid
consideration.
143. B 153. C A margin of 30% has been provided to students for
G
B – B lue miscellaneous contingencies (as was the case here).
B P2 P1 G – G ree n As per the university rules no student is allowed to
appear in exams under any condition without attending
P – P in k minimum 70% of classes. Hence, Anand's challenge
will fail.
As we can see in the Venn diagram given above,
option (B) follows and therefore, is the correct answer. 154. * All the options state that Komal cannot succeed. But,
if we read option "D" as "Komal can succeed…." then
it would be closest to the principle because Komal
144. D T was the owner of the watch. Since Sonal did not had
T – Tall the ownership of the watch and mere possession,
B P2 B – B oys she could not have transferred ownership to anyone
P3
including Monal.
P – P un ja bi
155. D As per the principle, reasonable restrictions can be
imposed on fundamental rights by law in the interest
As we can see in the Venn diagram given above,
of general public. And the restrictions imposed by
option (D) follows and therefore, is the correct answer.
authorities fall in that category.

145. A C R – R ina 156. * The facts seem to be insufficient because it is not


clear whether Suresh was the third party to the
R C – C o lleg e
G contract or not. He was merely being assured by
G – G irls Dinesh that his profit from the contract would be used
As we can see in the Venn diagram given above, to repay his debt. However, if we assume Suresh to
option (A) follows and therefore, is the correct answer. be the third party, then option "D" would be the most
appropriate.

Previous Years
CLAT & AILET Papers Page 59
157. D As per the principle, acceptance is complete when the assumption that he had quit smoking and drinking,
the letter of acceptance is properly posted and in the we can mark option "A" as the answer.
present scenario, this was done before Sani sold his
house to Gani. Hence option D would be the correct 168. C It has been made clear in the principle that acting in a
answer. cinematographic film will not be considered as 'work',
it cannot be protected under copyright law.
158. C S never went to court with clean hands.Had he fulfilled
his part of the agreement and then asked for relief, he 169. A Since government is party to this agreement, they are
would have had succeeded but strike cannot be the first owner of the copyright in the work, as there
granted as a valid excuse. was no agreement to the contrary by the professor.

159. * The facts are insufficient to state the object of the 170. B Only a certain kind of restricted rights have been
agreement entered between P and T. If we consider granted to B, that of distribution that to in Mumbai only.
the object to be the benefit of children through a child Hence the agreement is in the nature of license.
care centre, then we can narrow down on option "C"
as the answer. 171.* Although this question involves application of Legal
Knowledge contrary to the instructions given, still "C"
160. C It does not matter whether Hari had taken proper would be the most appropriate answer
precautions or not in the present scenario. Damage
was due to an act (Carrying on a chemical and fertilizer 172. * There is insufficiency in the principle and facts
industry) of Hari and damages need to be paid. provided. The principle only state what can be
registered as a trademark but is silent on the legal
161.* There is no nexus between the Principle and facts technicality of registration. Hence, there is no
because the principle is silent upon the legal implications conclusive answer which can be arrived at.
after making a gift. However, if we have to go ahead
with the options given, option "C" would be considered 173. D Since taking away from the land or possession is an
the best out of the lot. important part of theft and has not yet been executed,
option D would be correct.
162. C The agreement is based on an act which is impossible
in nature, and as per the principle any such impossible 174. C The principle given is not self-explanatory and requires
act cannot be enforced by the court of law. Hence, legal knowledge to be applied. But going by the
"C" is the correct answer. understanding of the principleOption "C" would be the
correct answer as leaving the abdominal pack inside
163. A Since the law specifically provides for the agreement cannot be justified in any way by a surgeon.
relating to transfer of the copyright in novel between
author of a novel and the producer need to be in 175. A Consent has no part in kidnapping as per the principle.
writing, author will succeed. The boy took away the minor girl out of the keeping of
her parents. He would be liable for kidnapping.
164. * The principle and facts are insufficient to conclusively
reach an answer. The principle only states who is a 176. * Facts provided are insufficient to reach a conclusive
person of unsound mind but is silent upon the answer. However, taking assumption that the men
repercussions of the agreement entered into by a were voluntarily having carnal intercourse against the
person of unsound mind. So, Legal knowledge had to order of nature, we can mark "C" as the correct option.
be applied in this case. Option "A" and "D" are equally
possible answers. 177. D The omission of A of not giving food to his father and
his act of beating his father together resulted in his
165. C Since A had used criminal intimidation to obtain B's father's death.
consent, B will succeed and A will not be able to
enforce the agreement. 178. A A is not seven years of age on December 30, 2011
and hence, as per the principle, not qualified to commit
166. D The father has misused his position as a father to an offence. Therefore, A is the correct answer.
obtain a bond of Rs. 30,000 from his son Shyam. And
as per the principle, such an agreement is not 179. C Since B was a minor, he could not have entered in a
enforceable. pact to suffer harm, Hence A has committed an offence.

167. * The principle and facts are insufficient to reach at a 180. D Although A is not committing an offence because of
conclusive answer. The facts are silent on the aspect his madness, B has the right to defend himself as per
whether Y had quit smoking and drinking. Going with the principle, hence option D is correct.

Previous Years
Page 60 CLAT & AILET Papers
181. A B specifically asked A to state the mental condition of 191. A When A gave Rs 10 lacs to B, C was already dead
the horse. A stayed silent but his silence amounted to making the condition impossible to be carried out.
speech indicating that the mental state of the horse is
sound. Thus, it would amount to fraud. 192. B Condition precedent means that the condition should
be carried out before the happening of the event. In
182. B The professor has not committed any offence as he
this case, B married without the consent of C,D and E.
wrote a comment in a National Daily which generated
Obtaining their consent after the marriage is not
an academic debate and no sort of disaffection
towards the government. Hence, the professor has fulfilling the condition precedent.
not committed any offence.
193. B To claim the benefit of the agreement, the condition
183. A The principle states that where more than one person subsequent i.e. the condition that follows the
have made a complaint for the grant of compulsory agreement, needs to be fulfilled which in the present
license, it shall be granted to that complainant 'only' case has not been. B did not go to England and hence,
who would best serve the interests of general public. his interest in the farm does not continue.
The principle shows that the license will be granted to
only one complainant. 194. A All the alleged facts are relevant in this case. There is
no fact which does not hold importance with respect
184. B The principle mentions that any police officer not below
to the fact scenario.
the rank of sub-inspector shall be competent to 'seize'
without warrant in case any copyright infringement is
found. In the facts given, the Superintendent of Police 195. A Since A is asserting the facts in the given situation,
(SP) made an 'arrest' which as per the principle is not hence, A must prove that B, C and D have committed
within his powers. the crime.

185. * The principle given in this question is incomplete as 196. * The facts are insufficient in the given question. Based
the definition of 'stealing' is not provided. But if we go on various assumptions, options A, B and C, all three
by the general understanding of the offence of can be possible correct choices.
stealing, then "C" would be the most appropriate
answer. It would be an attempt to steal because A 197. A Although the shopkeeper has not committed any
opened the box in which the jewel s used to be kept wrongful act but indirectly his act was the cause of
but he did not steal as there was no jewel in the box.
unlawful interference with other's use and enjoyment
By opening of the box, it is clear that he made an
of land. Hence, the shopkeeper will be liable for
attempt to steal the jewel. Hence, he has committed
the offence of attempt to stealing. nuisance.

186. B A has not defamed Z because there was no intention 198. B The agreement between Sultana and Marjina is valid
to harm his reputation and moreover, it was done as a as it only provides that in case of remarriage, forfeiture
marriage ritual. of right over husband's property would happen which
cannot be considered as a restrain on marriage.
187. C The death of the employee was caused due to the
negligence of the other employee making the employer 199. * The facts are insufficient in the given question. We do
devoid of the liability as stated in the principle. not know the final course of action taken by Mr.
Sharman at the end. Assuming that he ran down the
188. A A will be entitled to damages as his right to vote has
boat with only two passengers aboard, "A" would be
been violated. It does not matter if the candidate he
the most appropriate answer. The act of killing two
wanted to vote for has won the election anyways.
Hence, (A) is the correct option. passengers on boat C would result from the necessity
of saving more lives which is exactly what the principle
189. C When A wrote to B on 11th May that his services will provides.
not be needed, he had refused to perform his part of
the contract giving B the liberty to end the contract 200. * From the given facts and principle, it is not clear whether
and go ahead with other options. the court can direct the Parliament to make a new law.
Hence, "B" and "C", both remain debatable options.
190. B M company will not be liable for damages because A The court cannot enact laws according to principle,
had voluntarily came to see the race and was aware however the court can very well quash a law if it
of the fact that there are chances of accidents violates fundamental rights.
happening in such races.

Previous Years
CLAT & AILET Papers Page 61
CLAT Solutions 2014

1. D Since the sentence states that the action of 'proposing 'Rohan and I' are subjects who performed the action
a vote for thanks' was done at some point of time (at of 'going to Paris' not objects on whom the action was
the end of the meeting) in the past, we will use the performed, so 'I' (used for subjects) and not 'me' (used
simple past tense. Hence, option (D) is the correct for objects) is appropriate here. Hence, option (D) is
answer. Option (A) is incorrect because the subject the correct answer and all other options are incorrect.
'vote' is singular and so it should not be followed by a
plural verb 'were'. Option (B) is incorrect because the 6. B 'Shall' is the helping verb (generally used with first
correct phrase is 'vote of thanks', which refers to an person pronouns like 'I' and 'We') in question tags to
informal vote to thank some chairman or other officer politely seek permission. For example - I was thinking
of an organization for the work he has done on the of borrowing your dress, shall I/may I? Hence, option
organization's behalf. Option (C) is incorrect because (B) is the correct answer. Option (A) would have
the action (proposing a vote for thanks) did not happen been correct if the sentence were to be - 'We can't go
at some point in the past before some action or time in for a walk, can we?' Option (C) would have been
the past (as it happens in the case of simple past correct if the sentence were to be - 'We can go for a
tense). walk, can't we?' Option (D) would have been correct
if the sentence were to be 'We should go for a walk,
2. A According to the sentence, recession will force shouldn't we?'
companies to lay off/dismiss their workers or the
companies will be forced/compelled to lay off their 7. D In case of conditional sentences of the type 'If X had
workers. Hence, option (A) is the correct answer. existed (or Had X existed), Y would have happened',
Option (B) is incorrect because the phrase 'will have we use:
the force to' is grammatically incorrect in the given 'had + verb form 3' (had existed) in the conditional
context. Option (C) is incorrect because the sentence phrase and 'would have + verb form 3' (could have
does not mean that the companies will dismiss its happened) in the other phrase.
employees willingly but that recession will force the
For example - had I / If I had owned a house in
companies into doing it. Option (D) is incorrect because
the correct phrase would have been "companies will Delhi, I would not have stayed at my Uncle's house
be forced into laying off workers". last week. Hence, option (D) is the correct answer.
Option (A) is incorrect because 'been', which is
3. B Option (B) is the correct answer because 'to have generally followed by 'verb form 1 + ing', is inept in this
good command over a language' means to have good context. Here, the action of Anil having 'missed the
amount of knowledge to use that language. Option train' is not a continuous action. Option (B) is incorrect
(A) is incorrect because 'expertise' is not followed by because 'had not' does not connote 'past possibility'.
'over' but 'in'. Options (C) and (D) are incorrect because Option (C) is incorrect because the action would have
it is incorrect to say that one has 'control/authority' happened in the past while 'will' suggests future.
over language to mean that one has good knowledge
of some language. 8. B Option (B) is the correct answer because an infection
can spread 'across/throughout' the state. Option (A)
4. D Option (D) is the correct answer because the is incorrect because 'over' which means 'at a position
profitability of the industrial plant is likely to have above or higher than' is logically inept in this context.
increased (after the manager took over the Option (C) is incorrect because an infection cannot
administration) as the Chairman spoke in 'favour' of spread 'far the state'. Option (D) is incorrect because
the manager. Option (B) is logically incorrect due to the sentence does not mean that the state is the place
the same reason. Option (A) is incorrect because an from where the infection is spreading to villages. Also,
industrial plant can start to make profit but we can't 'something spreads from somewhere to/till some other
say that its 'profitability has arisen'. Option (C) is place'.
inappropriate because 'added' cannot be used in this
context to mean 'increased'. 9. A Option (A) is the correct answer because 'cut off',
which means separated is logically correct in this
5. D In positive and neutral contexts, the correct placement context. 'Cut down' means to make something fall down
of pronouns in a statement is in the following order : by cutting it at the base; 'cut away' means to remove
2nd person pronoun → 3rd person pronoun → something from something by cutting and 'cut out'
1st person pronoun. means to remove something from something larger by
For example - You, he (any name) and I will be cutting, usually with scissors.
responsible for the decorations. Also, in this context,

Previous Years
Page 62 CLAT & AILET Papers
10. B* Options (B) and (C) are equally good because immediately disclaims that these rules are not absolute
dissidents by virtue of holding different opinions can but vary. Statement (b) should be followed by
both 'cause' and 'pose' (be a great problem statement (c) because (c) further states how these
themselves) a great problem in every political party. conventions vary. Hence, option (D) is the correct
So, although according to the answer key released answer.
by CLAT, option (B) is the correct answer but there is
no reason to negate option (C) and choose (B) over it. 15. D The correct sequence is cbad. Statements (c) and (b)
Option (A) is incorrect because one can get rid of form a mandatory pair because 'this' in (b) refers to
one's problems by giving them to somebody else but 'study of nightmares being neglected' discussed in
we cannot use 'give' in this context to express 'cause'. (c). Statement (b) should be followed by statement (a)
Option (D) is incorrect because it is meaningless to because (b) talks about the fact that the study of
say that dissidents 'hold' great problems in parties. nightmares is not neglected like before and (a) gives
its example. Statements (a) and (d) form a mandatory
11. C The correct sequence is acbd. Statements (a) and (c) pair because the 'results' in (d) refer to those of
form a mandatory pair. Statement (a) discusses the 'studies' referred to in (a). Hence, option (D) is the
system of rate of exchange i.e., there is a certain correct answer.
value of one currency with respect to that of another
and statement (c) gives its example. Statements (b) 16. C 'Ex officio' means by virtue of one's position or status.
and (d) form a mandatory pair because statement (b)
states that the rate of exchange changes from time to 17. D 'Ultra Vires' means beyond one's legal power or
time and statement (d) gives an example of it. Since authority.
only option (C) exhibits such a pairing, it is the correct
answer. 18. B 'Quid pro quo' means a favour or advantage granted
in return for something.
12. D The correct answer is option (D) as the correct
sequence is bcad. Statements (b) and (c) form a 19. A 'Inter vivos' means between living people.
mandatory pair because the 'it' in (c) refers to the
'newly coined word' referred to in statement (b). 20. C 'Corpus juris' means a body of law, especially the
Statement (c) should be followed by statement (a) laws of a nation or state.
because the former talks about noting down the newly
coined word on a citation slip and the latter talks about 21. D The correct spelling of the word is 'Consensus'.
arranging these citation slips when editing a dictionary.
Statement (d) should come in the end because it 22. B The correct spelling of the word is 'Proceed'.
concludes the idea (of passing of a word from the
realm of hearing to that of writing) talked about in (b), 23. A The correct spelling of the word is 'Accommodate'.
(c) and (a).
24. D The correct spelling of the word is 'Foreword'.
13. A The correct sequence is cdab. Statement (c) should
be followed by statement (d) because the former talks 25. B The correct spelling of the word is 'Argument'.
about the existence of corruption at all levels and the
latter carries forward the idea by stating that such 26. A 'To make clean breast of' means 'to tell the truth about
corrupt people have also been brought to the book. something so that you no longer feel guilty'. Hence,
Statement (d) should be followed by statement (a) option (A) is the correct answer.
because (d) talks about offenders not being given
deterrent punishment and (a) gives its reason. 27. C 'A man of straw' means 'a person of little substance'.
Statement (a) should be followed by statement (b) Hence, option (C) is the correct answer.
because (a) talks about people not checking corruption
and (b) gives a consequence of it. Hence, option (A) 28. D 'A wild-goose chase' means 'a search for something
is the correct answer. that is impossible for you to find or that does not exist
and which makes you waste a lot of time'. Hence,
14. D The correct sequence is dabc. Statement (d) is an apt option (D) is the correct answer.
beginning of the paragraph as it is a general statement
that introduces the main idea - value of conventions in 29. A 'Put on the market' means 'to offer for sale'. Hence,
everyone's life. Statement (d) should be followed by option (A) is the correct answer.
statement (a) because (d) talks about conventions
being necessary part of everyone's life and (a) carries 30. B 'To meet someone halfway' means 'to reach an
the idea forward by telling what these conventions agreement with somebody by giving them part of what
are. Statement (a) should be followed by statement they want'. Hence, option (B) is the correct answer.
(b) because (a) describes conventions as accepted
rules (which may be considered absolute) and (b)

Previous Years
CLAT & AILET Papers Page 63
31. C The passage talks about the role higher education that not all universities should be modeled on these
plays in the development of the India, the quiet crisis in premier institutes. Hence, option (C) is the correct
higher education in India and the challenges that answer.
impede the process of improvement and the spread of
higher education in the country. Hence, option (C) is 39. A 'Cutting Edge' means highly advanced; innovative or
the correct answer. pioneering. So, 'conventional' is its apt antonym. Hence,
option (A) is the correct answer.
32. B The passage analyzes and criticizes the existence of
insufficient number of Universities to provide higher 40. B 'Discernible' means visible or noticeable. Hence, option
education in India and also states that the quality of (B) is the correct answer.
education provided in most of them is not up to the
mark. Hence, option (B) is the correct answer. Option 41. B Each term of the sequence is as follows:
(A) is incorrect because the theme/content of the +10
19  +8
→ 29 → +6
37 → +4
43 → 47
passage is academic but the manner/style in which
the passage is written cannot be called academic. 42. D The required probability
1 1 1 1
33. D Refer to the following sentence from the fourth = × × = .
5000 20000 100 10000000000
paragraph, "It is only an inclusive society that can
provide the foundations for a knowledge society."
Hence, option (D) is the correct answer. 120
43. B 5 + 12 × 10 ÷ = A × 10
240
34. A Option (A) is incorrect because the first paragraph of ⇒ 5 + 120 × 2 = A × 10 ⇒ A = 24.5.
the passage describes IITs and IIMs as "islands of
excellence". Therefore, option (A) is the correct
44. A L.C.M. of 0.12, 9.60 and 0.60
answer. Option (B) is true as evidenced from the
following sentence of the second paragraph, "The 3 48 3
proportion of our population…that enters the world of = LCM of , and
25 5 5
higher education is around 7 per cent, which is only
one half the average of Asia." Option (C) is true as the LCM (3,48 and3) 48
= = = 9.6.
passage talks about the importance of primary and HCFof (25,5and5) 5
higher education being essential for the growth of a
country.
45. B The average age of the whole class
35. C The passage describes the sorry state of affairs 30 × 10 + 40 × 8
= = 8.86 years.
concerning higher education in India and the need for 30 + 40
improvement in this field. Hence, option (C) is the correct
answer. 46. C Let the total income be Rs.x.
36. D Option (D) is false because it is not mentioned x x
∴ x −  +  = 630 ⇒ x = 1512
anywhere in the passage. Hence, it is the correct 3 4
answer. All other options are true because the passage
states the need of existence of excellent institutes, 1512
House rent = = 378.
massive expansion of opportunities and substantial 4
improvement in the quality of higher education.

37. D Options (A), (B) and (C) are all correct because the 2 × 30 × 40
47. A The average speed (in km/h) = ≈ 34.3
passage states that "primary education is absolutely 30 + 40
essential because it is the base", "universities are the
life-blood of higher education" and "transformation of 48. B The required ratio = 8 × 7 : 5 × 9 = 56 :45.
economy and society in the 21st century would
depend, in significant part, on the quality of education 49. D Father’s present age = 2 × 20 + 3 = 43 years.
…particularly in the sphere of higher education". The father’s age when the daughter was born
Hence, option (D) is the correct answer. = 43 – 20 = 23 years.

38. C Refer to the following sentence from the first B


50. B A × 0.80 = B × 0.20 ⇒ = 4 ⇒ 5x% = 4 ⇒ x = 80
paragraph, "Islands of excellence in professional A
education, such as Indian Institutes of Technology (IITs)
and Indian Institutes of Management (IIMs), are valuable 51. B Le the amount of water to be added be x L.
complements but cannot be substitutes for universities
0.1× 40 + x 20
which provide educational opportunities for people at ∴ = ⇒ x = 5.
large." From this, we can infer that the passage states 40 + x 100

Previous Years
Page 64 CLAT & AILET Papers
52. C The part of work done by A and B together in one day 64. B The Reserve Bank of India was established on April
1 1 7 1, 1935 in accordance with the provisions of the
= + = Reserve Bank of India Act, 1934. Though originally
20 15 60
privately owned, since nationalisation in 1949, the
60 4
The required number of day = =8 . Reserve Bank is fully owned by the Government of
7 7 India.
53. A Let the distance be x km.
65. D One of the most common and popular mudras, which
x x are found, depicted in Buddha statues is the
∴ − = 3 ⇒ x = 36.
(5 − 1) (5 + 1) Bhumisparshamudra, translated as the earth touching
gesture. Buddha statues with this mudra are
54. A The ratio of investment of A to B = 5000 × 12 : 6000 × 7 commonly known as the "earth-witness" Buddha and
= 10 : 7. these iconographic representations are one of the
10 most popular Buddhas you can find anywhere in the
Share of A = × 34000 = 20000 world.
17
And share of B = 14000.
66. D The right to vote is the fundamental right that has
55. A Let the total number of goats be x and that of hens be y. been the source of the most significant Supreme
Court litigation. Rights and Duties are inseparable. So
∴x + y = 80 and 4x + 2y = 200
as we have Fundamental Rights that we can enjoy
Solving the above equation, we get x = 20. there are also some Fundamental Duties that we must
follow & do. It is very true that the original constitution
56. B The side of the field = 324 = 18m didn't contain fundamental duties. The 42nd
Perimeter = 4 × 18 = 72m. Constitutional Amendment, 1976, has added it to the
constitution.
57. D The volume of metal = outer volume – inner volume
= 30 × 20 × 10 – (30 – 2) × (20 –2) × (10 – 2) 67. C The Ashokan inscriptions were I two scripts.
= 1968 cm3. Kharoshti in the north-western version and Brahmani
in the rest. This script is known to write from right to
58. C The required difference left. Brahmi, written from left to right is ascribed to
2 2
Brahma who, in iconic terms, is shown to hold a
 r   8  sheaf of palmleaves in his right hand.
= P  = 2000 ×  100  = 12.8 ≈ 13.
 100   
68. B In 1615 Sir Thomas Roe was instructed by King James
59. D Let the CP of the goods be Rs. 100. I to visit the Mughal Emperor Jahangir, who ruled
SP = 100 × 1.2 × 0.9 = 108 over 70% of the Indian subcontinent. The meeting
Percentage profit = 8. was a success and a commercial treaty was made
which gave the British East India Company exclusive
60. A Let the number of Rs.10 and Rs.20 be x and y rights to trade to and from India and England. Emperor
respectively. Jahangir sent a letter to the King through Sir Thomas
Roe.
∴x + y = 150 and 10x + 20y = 2000
Solving the above equation, we get x = 100. 69. A Sir Charles Wilkins was an English typographer and
Orientalist, notable as the first translator of Bhagavad
61. C Justice Kuldip Singh is known as the 'Green Judge' Gita into English in 1785 in London, England.
as he has created fear among the polluting industries
and anti-environment lobbies, including, bureaucracy 70. D a-1, b-4, c-2
has petitioned the apex court for justice. Chandrakanta is a popular Hindi novel by Devaki
Nandan Khatri. It is considered to be the first work of
62. D National Law Day in India is celebrated on 26 prose in the modern Hindi language. Durgeshnandini
November. It marks the anniversary of the drafting is a Bengali historical romance novel written by Indian
of the constitution by the constituent assembly in writer Bankim Chandra Chattopadhyay in 1865.
1949. Premchand wrote over three hundred short stories
and fourteen novels, many essays and letters, plays
63. B The DRDO was formed in 1958 following the merger and translations. "seva Sadan", the novel was
of the Technical Development Establishment (TDE) of originally written in Urdu under the title Bazaar-e-
the Indian Army, the Directorate of Technical Husn, but was first published in Hindi from Calcutta,
Development & Production (DTDP) and the Defense under the title Seva Sadan ("The House of Service"),
Science Organisation (DSO). in 1919.

Previous Years
CLAT & AILET Papers Page 65
71. D a-2, b-3, c-4, d-1. 77. D In most countries, interest rates on savings bank
Nagendra Singh was an Indian lawyer and accounts are set by commercial banks based on
administrator who served as President of the market conditions. In India, RBI fixes the interest rate
International Court of Justice from 1985 to 1988. on savings accounts in all the nationalized commercial
Sarosh Homi Kapadia was the thirty-eighth Chief banks.
Justice of India. N. R. Madhava Menon is a legal
educator from India. He was instrumental in setting 78. D All of these. Water, Ether, Ethanol, Toluene,
up the National Law School of India University (NLSIU) Chloroform and Ethanol are all used to dissolve other
in Bangalore, and was its founder-director. He was substances.
the founder and vice-chancellor of West Bengal
National University of Juridical Sciences (NUJS) in 79. C It has been argued extensively whether viruses are
Calcutta, run on the NLSIU model. He was the founder living organisms. Most virologists consider them non-
Director of the National Judicial Academy, Bhopal, living, as they do not meet all the criteria of the
and an institute for training judges. Madhava Menon generally accepted definition of life. They are similar
has worked for nearly five decades to improve Indian to obligate intracellular parasites as they lack the
legal education. He is Scholar-in-Residence of means for self-reproduction outside a host cell, but
Faculty of Law; A.M.U. Veeravalli Sundaram Sampath unlike parasites, viruses are generally not considered
is the current chief election commissioner (CEC) of to be true living organisms. A primary reason is that
the Election Commission of India. viruses do not possess a cell membrane or metabolize
on their own - characteristics of all living organisms.
72. C Justice HR Khanna was never a Chief Justice of Since they can be crystallized for years together,
India. they remain inactive and dormant without the host,
this is a trait similar to that of non-living.. Therefore,
73. B The Attorney General for India is the Indian viruses can be considered both living and non-living.
government's chief legal advisor, and its primary
lawyer in the Supreme Court of India. He is appointed 80. B Veteran poet and film lyricist Gulzar has been chosen
by the President of India and holds office during the for the coveted Dadasaheb Phalke Award for 2013
pleasure of the President. M C Stelvad was the first announced by the ministry of information and
Attorney-General of India (1950-1963). broadcasting. The multi-faceted Gulzar, whose real
name is Sampooran Singh Kalra, is a lyricist, director,
74. B Under the leadership of Gandhiji, the Civil screenwriter, producer and poet. He is the 45th
Disobedience Movement was launched in AD 1930. It winner of the Dadasaheb Phalke Award.
began with the Dandi March. On 12 March 1930,
Gandiji with some of his followers left the Sabarmati 81. C India's largest paramilitary force CRPF has appointed
Ashram at Ahmedabad and made their way towards five-time world boxing champion and Olympic bronze
Dandi, a village on the west coast of India. After medallist Mary Kom as its brand ambassador.
travelling for twenty-five days and covering a
distance of three hundred and eighty-five kms, the 82. D The award can be considered to be conferred only if
group reached Dandi on 6 April 1930. Here, Gandhiji the recipient's name is published in The Gazette of
protested against the Salt Law (salt was a monopoly India-a public journal and an authorised legal
of the government and no one was allowed to make document of the Government of India, and a
salt) by making salt himself and throwing up a recipients' register maintained under the direction of
challenge to the British government. The Dandi March the President. Till date (2014) 43 individuals have
signified the start of the Civil Disobedience Movement. been bestowed the award including 11 posthumous
declarations.
75. C a-2, b-4, c-3, d-1
a. Dandi March-Illegal manufacture of salt 83. A Malala Yousafzai is a Pakistani school pupil and
b. Chauri-Chaura - Withdrawal of movement education activist from the town of Mingora in the
c. Simon Commission -Country-wise agitation Swat District of Pakistan's northwestern Khyber
d. Morley Minto Reforms - Communal Electorate. Pakhtunkhwa province. She is known for her activism
for rights to education and for women, especially in
76. C The planet with the most moons in the Solar System the Swat Valley, where the Taliban had at times
is Jupiter, with a total of 63 confirmed moons (as of banned girls from attending school.
2009). Eight of Jupiter's moons are regular satellites,
with 4 large, spherical moons, and 4 smaller moons 84. A Arundhati Bhattacharya has taken over, as the new
that orbit closer to Jupiter. Jupiter has an additional chairperson of the State Bank of India (SBI). She is
55 tiny irregular satellites. the first woman to be appointed to the top job.

Previous Years
Page 66 CLAT & AILET Papers
85. C The Fourteenth Finance Commission has been set up 95. B In a public interest litigation it was argued that "a
under the Chairmanship of Dr. Y.V.Reddy [Former Governor, as the Head of the State, holds a high
Governor Reserve Bank of India]. constitutional office which carries with it important
constitutional functions and duties; that the fact that
86. D Justice A.K. Mathur is the Chairperson of the Seventh the Governor is appointed by the President and that
Pay Commission. he holds office during the pleasure of the President
does not make the Governor an employee or a servant
87. C The United Nations Industrial Development or agent of the Union Government; and that his
Organization (UNIDO), French/Spanish/Portuguese independent constitutional office is not subordinate
acronym ONUDI, is a specialized agency in the United or subservient to the Union Government and he is
Nations system, headquartered in Vienna, Austria. not accountable to them for the manner in which he
carries out his functions and duties as Governor".
88. D First Plan (1951 - 56), was based on Harrod-Domar Thus was submitted "a Governor should ordinarily
Model. Community Development Program launched in be permitted to continue in office for the full term of
1952.It was focused on agriculture, price stability, five years; and though he holds office during the
power and transport. It was a successful plan pleasure of the President, he could be removed
primarily because of good harvests in the last two before the expiry of the term of five years, only in
years of the plan. rare and exceptional circumstances, by observing
the following constitutional norms and requirements."
89. D Levying the higher duty on exports does not aid in
improving India's balance of Payment. 96. C G. Sankara Kurup, better known as Mahakavi G (The
Great Poet G), was the first winner of the Jnanpith
90. C The chronological sequence of the persons who Award, India's highest literary award.
visited India at one time or another is:
Megasthanese -> Fa-Hien -> Hiuen-Tsang -> I-Tsing 97. C a-2, b-3, c-1, d-4
(Yijing) a. Vishakhadatta-Drama
b. Varahamihira-Astronomy
91. D Bengal Gazette (founded in 1779) was the first c. Sushruta-Surgery
newspaper of India published in Calcutta, the editor d. Brahmagupta-Mathematics
of which was James Augustus Hickey who was a
pioneer in bringing the start of journalistic activities in 98. D The Arjuna Awards were instituted in 1961 by the
the country. government of India to recognize outstanding
achievement in National sports. The award carries a
92. A The color of a star is linked to its temperature and its cash prize of Rs. 500,000, a bronze statuette of
temperature is linked to what processes are taking Arjuna and a scroll.
place in and on the surface of the star. Stars evolve
during their lives, going through different phases of 99. A The economic growth rate projected by the IMF for
elements burning. Say a star has finished burning its India in the fiscal year 2014-15 is 5.4 percent.
hydrogen and goes to the helium burning phase, this
will change the temperature (hence the color) of the 100. B Matteo Renzi, 39, has been sworn in as Italy's
star. youngest prime minister ever.

93. C To qualify as a planet in orbit around our Sun, a chunk 101. B The Ashok Chakra is an Indian military decoration
of rock must have been made round by its own awarded for valor, courageous action or self-
gravity; have cleared its neighbourhood of other sacrifice away from the battlefield. It is the peace
debris; and not be a satellite of another planetary time equivalent of the Param Vir Chakra, and is
body. Mercury, Venus, Earth, Mars, Jupiter, Saturn, awarded for the "most conspicuous bravery or some
Uranus and Neptune all fulfil these criteria. But Pluto daring or pre-eminent valour or self-sacrifice" other
is just one of many bits of icy debris in orbit at the than in the face of the enemy. The decoration may be
edge of our Solar System, known as trans-neptunian awarded either to military or civilian personnel and
objects. Pluto's membership of the trans-neptunians may be awarded posthumously.
disqualifies it from being a fully-fledged planet
because it has not 'cleared its orbit'. 102. D Hydrogen, being the lightest existing gas (14 times
less dense than air), seems to be the most appropriate
94. C Dr. Raghuram Rajan assumed charge as the 23rd gas for lifting.
Governor of the Reserve Bank of India.
103. C Gregor Mendel is currently known as the father of
modern genetics. This lesson goes through a brief
history of his life, workings as a scientist, and his
findings.

Previous Years
CLAT & AILET Papers Page 67
104. B International Women's Year (IWY) was the name 114. C Since the man has no sister or brother, so the man’s
given to 1975 by the United Nations. Since that year father’s son is the man himself .
March 8 has been celebrated as International Hence, the person in the photograph is his son.
Women's Day, and the United Nations Decade for
Women, from 1976-1985, was also established. 115. C The son of only bothers of father’s wife in the son of
maternal uncle.
105. A Automated teller machine is an electronic Hence, they are cousin.
telecommunications device that enables the
customers of a financial institution to perform financial 116. D The wife of Bhumika’s husband is Bhumika herself
transactions without the need for a human cashier, and her son’s sister is her daughter.
clerk or bank teller.
117. D The family relation is represented below.
106. C The general elections for 2014 is taking place in 9
phases and encounters to be the longest election Josep h
poll in the country's history from 7th April to 12th May,
Son D a ug hter
2014 to constitute 16th Lok Sabha in India. The voting
is carried out in all 543 parliamentary constituencies
of India for electing members of parliament in Lok V icto r S ister R o sy
Sabha. D a ug hter D a ug hter
107. C Justice Rajendra Mal Lodha, the senior most judge of P inky L ucy
the Supreme Court, was on Sunday sworn in as the
41st Chief Justice of India by President Pranab Hence, Josheph is the grandfather of Lucy.
Mukherjee.
118. A The daughter of Pramod’s grandmother is his aunt
108. D The ninth Ministerial Conference of the WTO, held and her only bother is father of Pramod.
during 3 December - 6 December 2013, was
concluded at Bali, Indonesia. For 119 to 123. The information given is summarized as
follow:
109. D The Nobel Prize in Literature 2013 was awarded to
Alice Munro "master of the contemporary short story".
M oh an Van ita
110. A ISRO's Polar Satellite Launch Vehicle, PSLV-C24,
(E n gine er, R e d) (Tea che r, O ra ng e )
successfully launched IRNSS-1B, the second satellite
in the Indian Regional Navigation Satellite System
(IRNSS), on April 04, 2014 from Satish Dhawan Space C h ara n N a m ita
Centre SHAR, Sriharikota.
(C A ,__ _) (_ __ , W hite)
For 111 to 113
The information given is summarized as follow:
Raman S a rita
Day Persons Subjects (_ __ , _ __ )
(P rincip al, B lack)
Monday Satish English
Tuesday Rajesh Geology 119. D

Wednesday Rehman Zoology 120. D

Thursday Rakesh Physics 121. B


Friday Vineet Botany
122. B

111. B 123. D

112. D

113. D

Previous Years
Page 68 CLAT & AILET Papers
124. A The movement of the first bar B1 and second bar B2 is 130. C Both the statements are required to find the age of
shown below Diana.
Fina l
131. A The pattern of the series is as follows:
+3 +5 +7 +9
25 25 B1 35 0 3 8 15 24 .

B1 1 50 km B2 B2 132. B The pattern of the series is as follows:


In itial 15 Fina l In itial +8 +1 2 +1 6 +2 0
8 16 28 44 6 4.

25 133. C The pattern of the series is as follows:


The required distance = 150 – (25 + 25 + 35)
= 65 km. 0 6 24 60 1 20 2 10 3 36

125. D Sitting arrangement is shown below


6 18 36 60 90 1 26
M
L N N 12 18 24 30 36

K G W E
6 6 6 6

S 134. D The pattern of the series is as follows:


J H
+3 +3 +3
I P S V Y
Hence, J is in South-West.
+3 +3 +3
126. A The sitting positions the persons and the direction O R U X
they are facing is shown below.
+3 +3 +3
N Q T W Z
V in ay
135. A The pattern of the series is as follows:
W E +2 +3 +4 +5 +6
Vaibh av R o sh an A C F Z O U
1 3 6 10 15 21
S
S u m it 136. D Just as an arrow is shot with a bow, a bullet is shot
from a pistol.

127. D The standing position is as follow. 137. A Just as an eye winks, the heart beats/throbs. Option
(C) is incorrect because the beating of the heart is
P a va n Tavan C h avan V ip in N a ku l not called pumping of the heart; the heart pumps
blood in the arteries.
Hence, Tavan is fourth from the right.
138. C Just as an ocean comprises water, a glacier
128. B The shadow of the pole fell exactly to right of Suraj.
comprises ice.
Hence the sun is in the left of Suraj, so he must be
facing south.
139. A Just as 'prima facie' means 'on the first view', 'in pari
delicto' means is a legal term used to indicate that
For 129 and 130.
two persons or entities are equally at fault.
Let the age of the Jackson be x years and that
Edward be y years.
140. D Just as 'delusion' and 'hallucination' are synonyms,
The age of Diana = 4x years
which refer to the experience involving the apparent
The age of Stephen = 2y years.
perception of something not present, 'chagrin' is a
3 Jackson is older than Edward synonym of annoyance. 'Cogent' means conclusive.
∴x > y

129. B

Previous Years
CLAT & AILET Papers Page 69
141. C 145. D

A ccep te d o ffers In dian H o ckey tea m


A g ree m en ts X1

C o ntracts G oo d H ockey p la ye r X2

As we can see in the Venn diagram, option (C) As we can see in the Venn diagram, neither of the
follows and is therefore, the correct answer. conclusions follow and therefore, option (D) is the
correct answer.
142. A
146. C Just as the Indian flag contains tricolor, the USA flag
contains stars and stripes.
G oo d da nce rs 147. A Just as 'statute' means law, 'proviso' means clause.

148. D Just as the Buddhist temple is called a 'pagoda',


B e au tiful A ctre sse s
'synagogue' is the place where the Jews meet for
w o m en
religious worship or instruction.

149. C
As we can see in the Venn diagram, only option (A) G re at de m a nd
follows and is therefore, the correct answer.
L aw g ra du ate s
143. B
R a je sh a nd K rish na

B o oks M ag azin e N o ve ls 1

N o ve ls 2
As can be seen in the Venn diagram given above,
As we can see in the Venn diagram, only conclusion option (C) is the correct answer.
(II) follows and therefore, option (B) is the correct
answer. 150. B

144. A
G re at player

S u cce ssfu l sp orts


H ig h er a dm inistra to rs C a ptain
H ig h er Stud e nts e du ca tion 3
e du ca tion 1

H ig h er E xcursio ns
e du ca tion 2
As can be seen in the Venn diagram given above,
option (B) is the correct answer.

As we can see in the Venn diagram, only conclusion 151. A,C A person has to signify his willingness to do or to
(I) follows and therefore, option (A) is the correct abstain from doing anything to another person
answer. according to the principle. In the present scenario,
there is no other person present and therefore there
is no proposal.

Previous Years
Page 70 CLAT & AILET Papers
152. D As per the principle, in the case of arrangements 166. A D would be liable as he published defamatory material
regulating social relations, it follows as a matter of against P and had no evidence to prove whatever he
course that the parties do not intend legal published was true.
consequences to follow. The present case is a
general agreement regulating a social relation (father 167. D The principal clearly states that gift of future property
son relation.) would be void. Since the sale of adjacent land has
not been completed, it would be considered a future
153. B Communication of acceptance to 'z', a complete property with respect to 'X' gifting it away.
stranger does not give rise to any legally binding
obligation. 168. B 'A' should have been cautious and had tested out
whether the cloth he is purchasing is fit for his
154. B Two years cannot be determined as a reasonable purpose i.e. for making uniforms or not. Since he has
period of time taking into account that the equipment not done that, therefore he cannot succeed in getting
worth was Rs. 1000, which would vary due to a remedy against the Seller.
various factors such as inflation etc. in two years of
time. 169. B Since 'X' himself had obtained the bike illegally, he
cannot sell the bike as an owner and therefore 'Y'
155. C The agreement becomes void from the very beginning has no title.
as 'A' is a minor.
170. A It was failure on the part of manufacturer to exercise
156. A The consideration is real and it does not matter is it is the standard of care required by law, resulting in
adequate or not. damage to D who was the consumer and was getting
directly affected.
157. C The agreement is void as the object is unlawful and
hence, cannot result into a contract. 171. C The act of going for tea to a shop and picking up a
fight is in no way connected to D's employment.
158. D The agreement is void as the object is unlawful and
hence, cannot result into a contract. 172. D It has been clearly provided in the principle that right
to life does not include right to die and hence it cannot
159. D Since consent of 'Y' was obtained through coercion, be granted.
the contracts becomes voidable at the option of 'Y'.
173. A 'A' was directly interfering with 'B's premises with
160. C This is an Agreement in restraint of marriage and the help of tangible object (stone).
hence is void (cannot be enforced in a court of law).
174. B Had A directly planted a tree on B's land, it would
161. A X gave money to Y in his house not only because he have had amounted to trespass. However A planted
was a cashier, but because he was his neighbor a tree on his own land and consequently the
and X trusted him. Y as an employee was not told by branches started projecting on B's land which would
the bank to collect money from X at his house. amount to nuisance.
Therefore the Bank is not vicariously liable for the
act of Y as it was committed outside the course of 175. B The Action of R(taking out S's bike and not putting it
employment. back again) led to S's bike getting stolen away and
hence amounts to conversion.
162. B The very Act of purchasing ticket match signifies 'X'
consenting to the injury as it is common knowledge 176. B Apprehending someone who has committed murder
that in a cricket match, the ball, hit by batsmen does is not unlawful by a police officer even without a
falls into spectator area occasionally. warrant.

163. B X does not have any remedy and will have to pay all 177. C The act of killing Darshan was done togather in
his dues as he cannot take ignorance of law as a furtherance of a common intention and hence all of
defence. them would be liable.

164. A It can be established that damage was being caused 178. B A was communicating his diagnosis to his patient in
to ES as they had to reduce their school fee but at good faith which cannot amount to any offence.
the same time it can also be established that there
was no violation of legal rights of ES. 179. B Sawant knowingly and willfully furnished false
information to the magistrate of the murder happening
165. B There was no damage caused to A as the candidate on his property and hence is liable.
he wanted to vote for won. But his legal right to vote
was violated; therefore he has to be compensated.

Previous Years
CLAT & AILET Papers Page 71
180. A K knows that his act of travelling while having cholera 191. B All of them are guilty since they conspired to commit
is likely to spread the infection of a disease the crime even though no crime was committed or
dangerous to life and he still does it. Therefore, K is even attempted.
guilty. 192. C Since X took the Watch away from Y without his
consent, X is causing wrongful loss to Y.
181. C X's action of rash and negligent driving caused a
pedestrian's death and hence he is guilty. 193. B No person in his ordinary senses would have
complaint of taking a plain sheet of paper from his
182. B A knew Z was behind the bush. He intentionally and drawer.
knowingly made B to fire at the bush and hence A is
guilty of culpable homicide. 194. A Since X was insane his actions would not amount to
an offence, still Y would get a chance to defend
183. B Z voluntarily gave the plate to A and it was not taken himself according to principle.
out of Z's possession without his consent. Therefore
A has not committed theft. 195. C Since Z (property) was transferred to A after B's
death, such transfer shall, at the option of the
184. C A used Z's seal in order to sell the estate to B. A has transferee(C), operate on any interest which the
committed forgery. transferor(A) may acquire in such property at any
time during which the contract of transfer subsists.
185. A A stopped Z's ride by use of force in order to rob him.
A is guilty of using criminal force on Z. 196. B Since the deity is not a living person, X cannot transfer
his property as this would be invalid according to the
186. B since B (16) is not of legal age (18), she is not eligible principle.
to marry under the special marriage Act.
197. C There is no money involved and transfer of one
187. B Rohit went to US with a reason and with the consent property is being done in lieu of other. According to
of his wife. This cannot amount to desertion. the principle this transaction is exchange.

188. C As per the principle a person once given in adoption 198. A Even after not taking part in the selection process,
cannot be given in adoption again. Hence first Selection of A would be invalid.
adoption i.e. adoption of Tarun by Manoj is valid.
199. C Closing down of the factory by the employer till the
189. A Sufficient work has been expended Michael in making strike ends would amount to lock-out.
the street directory and hence it can be copyrighted.
200. A Suspension is a dispute between workmen and
190. A It does not matter whether X was physically present employer which is connected with employment and
in India or not. The very fact the offence of cheating non-employment of the workmen. Hence it can be
was committed in Bombay (India) makes him liable brought under the purview of trade disputes.
for cheating.

Previous Years
Page 72 CLAT & AILET Papers
CLAT Solutions 2015
1. B The correct sequence is ABBBA. Farther refers to a 13. A The title is the most appropriate because it covers all
physical distance whereas further refers to something the aspects of the passage. It includes the in-law’s
over a great expanse of time and space. Historic offer as well as the professional aspect.
refers to something which is famous or important in
history while history refers to something concerning 14. D The title covers the two types of inertia and their effect
history. In the context of the sentence it will be historic. on a human being. No aspect in the passage is left
Mistrust refers to suspicion and then, a film can only excluded in the title.
be based on a true story. Discreet means to be careful
while discrete refers to something separate or distinct. 15. C Specious means superficially plausible, but actually
wrong. Apart from option (C), the other options are
2. A (Incorrect) The CLAT answer key is option (A). synonyms of specious. Credible is an antonym.
However, we believe that the correct answer should
be BBBBA (option not given). Critics cannot censor 16. D Options other than (D) are synonymous words. All the
but censure. “Censure” means disapproval of words refer to preventing or stalling something. Bolster
something. Furthermore, in the 3rd option, range is means to support or strengthen. It can also mean a
incorrect. The word should be rage. long, thick pillow that is placed under other pillows for
support.
3. D Interment refers to burial of a corpse while internment
refers to imprisonment. Caustic refers to something 17. A Obsolete means something which is out of vogue or
acerbic. Flare refers to a gradual widening of shape, archaic. Hence, options (B), (C) and (D) are synonyms.
especially towards the hem of a garment. Pealing Option (A) is the odd one out.
refers to the sound of bells. Bait means to taunt or
annoy someone. 18. D Parsimonious refers to someone who is unwilling to
spend money. Options (A), (B) and (C) are synonyms.
4. B Buckle refers to a flat, typically rectangular frame Option (D) is altruistic which means showing a
with a hinged pin, used for joining the ends of a belt or disinterested and selfless concern for the well-being
strap. Break into a buckle does not have any meaning. of others or being unselfish.

5. B Run over means to overflow or exceed. It can also 19. A Facetious means treating serious issues with
mean a pedestrian being hit by a car. In the context of deliberately inappropriate humour; flippant.Jovian
the sentence run over is wrong. Hence, statement D relates to Jove or Jupiter. Hence, option (A) is
is incorrect. inappropriate.

6. D In statements C and D, there should be the indefinite 20. D After reading the passage, option (D) seems to be the
article ‘a’ before friend and profitable respectively. most likely answer because an unreasonable man
will try to do things which are inappropriate in a
7. B The statement made by the author is obviously very particular moment.
blunt. Blunt in the context of the sentence means a
straightforward or direct statement. 21. A After reading the passage, we know that an
unreasonable man will do things which are
8. A A principal amount accrues interest. Hence, interest inappropriate in the context of a particular time. Hence,
is the answer. option (A) seems to be the most likely answer.

9. D Apart from fines, the other options are incorrect in the 22. A Option (A) is grammatically correct. The sentence
context of the sentence. wants to say that the management cannot scold as
and when they wish. Umbrage in option (B) means
10. C Properties are often attached when taxes are not annoyance.Typing Error: In the question ‘but’ should
paid. Hence, attached is the right answer. not have been underlined.

11. B Seized is the most appropriate word in the context of 23. A Option (A) is grammatically correct. The other options
the sentence. Automobiles can be seized if taxes are are grammatically incorrect. Typing Error: In the
not paid. question ‘no telling’ should not have been underlined.

12. D Offender is the right word because a tax payer who 24. A The sentence wants to say that Romanians are close
has not paid taxes regularly has committed an offence to Moscow in terms of ideological and military aspects.
in the eyes of the law. Hence, option (A) is the answer.

Previous Years
CLAT & AILET Papers Page 73
25. C (Incorrect) 34. A Refer to the third paragraph where the author mentions
The CLAT answer key is option (B). However, we that India isolationism in the global economic arena
believe that option (C) is the correct answer. Dulcet was to be blamed for India’s poor performance.
refers to a sweet and soothing sound while raucous
means a loud noise. Hence, dulcet and raucous are 35. B (Incorrect)
antonyms. Similarly, palliative refers to relieving or The CLAT answer key is option (A). We believe that
soothing of pain whereas, exacerbate refers to the correct answer choice should be option (B). Refer
worsening of something. Palliative and Exacerbate to the first sentence of the third paragraph. Secondly,
can be considered as antonyms. Option (D) also refers there is a typing error. Projectionist should have been
to antonyms and hence, can be the correct answer protectionist.
but it has to be kept in mind that Theory is noun while
Practical is adjective. Option (B) refers to synonyms. 36. B The author cites the example of the car Cielo to highlight
the popularity of the car in the Indian market.
26. A Malapropism refers to the mistaken use of a word in Furthermore, the comment also shows that India has
place of a similar-sounding one, often with an amusing been a poor performer in the international market. Refer
effect. Hence, Malapropism is closely associated with to the second last sentence of the second paragraph.
words. Similarly, Anachronism refers to a thing
belonging or appropriate to a period other than that in 37. A Refer to the fourth last sentence in the second last
which it exists, especially a thing that is conspicuously paragraph. “To see beyond their noses” means
old-fashioned. Hence, Anachronism is closely shortsightedness or myopic character.
associated with time. The relation is the same as
Malapropism and words. 38. A Grandiose (adj.) refers to something magnanimous
or huge. Imposing is another synonym of Grandiose.
27. C This is an example of homophone. Both the words
have the same pronunciation but have different 39. B Spry (adj.) refers to someone who is active, agile or
meaning and spellings. lively. Nimble is another synonym of spry.

28. A The author’s attitude is certainly critical. S/he furnishes 40. D Fudge (n. and vb.) refers to tampering with something.
examples and elaborate explains his point to examine It can also mean evading something. Falsify is a good
and analyse India’s economic policies. synonym of fudge.

29. C Refer to the sixth sentence of the third paragraph. 41. B Attukal Pongala is an extremely popular, essentially
The author clearly mentions that the government women’s festival celebrated in ancient Bhagavathy
believed that businessmen were crooks and adopted temple (Mudippura) at Attukal in Thiruvananthapuram
unholy practices. district of Kerala.

30. C Refer to the sixth sentence of the second last 42. B Anil Kumble, the former India captain and Betty Wilson,
paragraph. The explains why the government changed one of the greatest women’s cricketers of all-time,
its economic policies. Furthermore, there is a were inducted into the ICC Cricket Hall of Fame during
typographical error in option (C). The word change the ICC Cricket World Cup 2015.
should have been exchange.
43. A The Indian Councils Act 1909 effectively allowed the
31. B The author ends the passage with a note of pessimism. election of Indians to the various legislative councils in
The ending of the passage reflects the frustration of India for the first time. Previously some Indians had
the author. been appointed to legislative councils. The majorities
of the councils remained British government
32. B Refer to the second sentence in the second paragraph. appointments. Moreover the electorate was limited to
The author mentions that India had a better specific classes of Indian nationals.
infrastructure compared to the other South Asian
countries. India did not suffer significantly during the 44. A On 24 March 2015 IRCTC in collaboration with Union
Second World War. Bank of India (UBI) launched IRCTC RuPay Debit Card
exclusively for booking train tickets on railway portal.
33. D Both statements A and B are correct. Refer to the third RuPay card is India’s first payment gateway system
sentence in the second paragraph where the author launched by National Payments Corporation of India
mentions that India suffered hardly during World War (NPCI).
II. Furthermore, refer to the following sentence where
the author mentions that India had a population who 45. C Exercise ‘GARUDA SHAKTI-III’ is the third one in the
could speak English and had business skills or ongoing series of joint exercises between armies of
acumen. India and Indonesia.

Previous Years
Page 74 CLAT & AILET Papers
46. A Option (A) is the correct answer. CEO of Axis Bank, the committee members include S.
Raman, whole-time member, SEBI; Shubhalakshmi
47. A India’s Beyond Visual Range (BVR) air-to-air missile Panse, Chairperson and Managing Director, Allahabad
ASTRA was successfully test fired by the Indian Air Bank; Pratip Kar, former Executive Director, SEBI, and
Force off the coast of Odisha near the Integrated Test Joydeep Sengupta, Director, McKinsey & Company.
Range, Balasore. The missile has been indigenously
designed and developed by the Defence Research 58. A Paramparagat Krishi Vikas Yojana (Traditional Farming
and Development Organisation (DRDO). In test flight Improvement Programme) has been launched by
an ASTRA missile carrying telemetry equipment in place Government of India to support and promote organic
of the warhead was fired from a Sukhoi-30 aircraft farming and thereby improving soil health.
against a Lakshya (Pilotless Target Aircraft) target.
The target was successfully engaged and it was 59. A Sebastian Vettel won 2015 Malaysian Grand prix.
captured by Telemetry and Electro-optical tracking
stations. 60. D Kuchipudi was never a solo affair and required a
number of performers. Kuchipudi was performed in
48. C The Indian Space Research Organization has the open air by performers who were given a vigorous
developed a Flood Hazard Atlas by mapping flood prone training in abhinaya, music, dancing and singing.
and vulnerable areas in Assam. Kathak is traditionally a solo dance form. Mohiniattam,
essentially a solo dance, is nowadays being performed
49. C Thailand’s military-dominated parliament passed a law in groups as well. The repertoire of Mohiniattam closely
criminalising and banning foreigners from seeking follows that of Bharatanatyam. There are a number of
surrogacy services in the South-East Asian country. forms in Manipuri. These are the Ras Lila, the Pung
The law prohibits non-Thai couples and same-sex Cholom, Nupa Cholom, Thoibi and a host of others.The
couples accessing surrogacy services, ending a Ras Lila is the most important dance form in the Manipuri
booming “rent-a-womb” industry that made Thailand a style. The theme revolves around the love of Krishna
top destination for fertility tourism. and the milkmaids (gopinis).

50. A The Bombay Stock Exchange, popularly known as 61. D Rajtarangini is a metrical historical chronicle of north-
BSE, is the oldest one in Asia. BSE was established in western Indian subcontinent, particularly the kings of
1875 as The Native Share and Stock Brokers Kashmir, written in Sanskrit by Kashmiri Brahman
Association. Kalhana in 12th century.

51. A Amaravathi is the new capital of Andhra Pradesh. 62. B Indian golfer Anirban Lahiri’s purple patch continued
as he clinched the Hero Indian Open.
52. B Ufa is the capital city of the Republic of Bashkortostan,
Russia, and the industrial, economic, scientific and 63. C A SpaceX rocket blasted off from Cape Canaveral Air
cultural center of the republic. Force Station to put the world’s first all-electric
communications satellites into orbit.
53. B The ease of doing business index is an index created
by the World Bank Group. Higher rankings (a low 64. C Maharaja Suraj Mal or Sujan Singh was ruler of
numerical value) indicate better, usually simpler, Bharatpur in Rajasthan, India. Historians has described
regulations for businesses and stronger protections him as “the Plato of the Jat people” and by a modern
of property rights. writer as the “Jat Odysseus”.

54. B Prime Minister of India has launched the Give it Up 65. A Bhalchandra Vanaji Nemade is a Marathi writer from
campaign for voluntarily giving up LPG subsidy. Maharashtra, India. Famous for his books “Hindu” &
Through this campaign he appealed to well-to-do “Kosala”. Also he is known for his novel “Hindu
sections of Indian society to voluntarily give up LPG jagnyachi samrudhha adgal”. He is winner for 2014
subsidy, so that the benefit of the same could be Jnanpith Award.
shared more widely with poor sections of the society.
66. B Snickometer is associated with cricket.
55. B The South Eastern Railway (SER) is one of the
seventeen railway zones in India. It is headquartered 67. C A solstice happens when the sun’s zenith is at its
at Garden Reach, Kolkata, West Bengal, India. furthest point from the equator. In June solstice it
reaches its northernmost point and the Earth’s North
56. C Alibaba is world’s largest e-commerce company. Pole tilts directly towards the sun, at about 23.5
degrees. It’s also known as the northern solstice
57. A The Reserve Bank of India has constituted an expert because it occurs when the sun is directly over the
committee to review the governance of bank boards Tropic of Cancer in the Northern Hemisphere.
in India. Chaired by P.J. Nayak, former Chairman and

Previous Years
CLAT & AILET Papers Page 75
68. C Option (C) is the correct choice. 81. B Goa will organise the 36th National Games 2016.

69. B Switzerland is the First country to officially submit 82. C NS Varsha is a new naval base being developed under
INDC. Project Varsha for the Indian Navy. This base will be
the home of the navy’s new fleet of nuclear
70. B Lysosomes are called suicide sacks. They are submarines and ships. It was planned to be located
produced by the Golgi body. They consist of a single within a radius of approximately 200 kilometres from
membrane surrounding powerful digestive enzymes. Visakhapatnam, the headquarters of the navy’s
It acts as the “garbage disposal” of the cell by breaking Eastern Naval Command.
down cell components that are no longer needed as
well as molecules or even bacteria that are ingested 83. C K Srikanth won Swiss Grand Prix Gold.
by the cell.
84. A Finance minister Arun Jaitley, in his Union Budget 2014,
announced a new 24-hour TV channel Kisan for
71. B Repo is the single policy rate to unambiguously signal
farmers that will be dedicated to the agriculture sector,
the stance of monetary policy as recommended by
with an outlay of Rs.100 crore for providing real-time
RBI. information to farmers.

72. B Kolkata, West Bengal’s capital city, has become India’s 85. B The Muthoot Group, as a part of its CSR initiative
first fully WiFi-enabled metro city. Reliance JIO is Muthoot Haathi Mera Saathi, in partnership with World
reportedly providing the services. Wide Fund for Nature- India (WWF-India) has launched
“Friends for Life” an elephant conservation project
73. B The Narendra Modi government has created a cyber- that is dedicated towards effective management of
security chief’s position under the Prime Minister’s human-elephant conflict (HEC) and protecting the
Office and has appointed Gulshan Rai. habitats of Asian elephants across six states in India.

74. A Union foreign ministry has launched “Madad” an online 86. A The National Industrial Corridor (NIC) is proposed to be
grievances monitoring portal for Indians living abroad. established in Pune (Maharashtra).
The portal will help the Indians living abroad to approach
the government easily for any help. Madad is a 87. A Only option (A) A is correct.
combined effort of MEA and MOIA to enhance
collaboration and contact with Indians abroad. 88. D Madhav Gadgill was chosen for Tyler prize 2015.

75. C Income Tax is a direct tax while the rest are indirect 89. B As part of military modernisation drive, The government
taxes. today allocated Rs 2.29 lakh crore for the Defence
Ministry in the Budget, marking an increase of around
12.5 per cent from the last fiscal as it raised the FDI
76. D Space Pioneer award for the year 2015 was presented
limit in the defence sector up to 49 per cent from 26
to Indian Space Research Organisation (ISRO) in the
per cent now.
Science and Engineering category during the 34th
Annual International Space Development Conference
90. A CA-125 is a protein that is found more in ovarian cancer
held at Toronto in Canada. cells than in other cells. This blood test is often used to
monitor women who have been diagnosed with ovarian
77. D Prime Minister Narendra Modi unveiled CSRS radar in cancer. The test is useful if CA-125 level was high
Seychelles. when the cancer was first diagnosed.

78. C The evolution of the Law of the Sea, which gives 91. C Total number of units manufactured by company C
countries new spaces of sovereignty and areas of overall the years together
jurisdiction without specifying their delimitation, is the = (2.6 + 2.2 + 2.1 + 2.8 + 2.6) × 100
source of the dispute between Greece and Turkey in = 12.3 × 100 = 1230
the Aegean Sea.
92. D Required increase percentage
79. D In order to honour Tata Group founder Jamsetji
(1.7 − 1.4)
Nusserwanji Tata by releasing commemorative coins = × 100
1.4
to mark his 175th birth anniversary. The coins, a first
for an industrialist, are being minted on the instructions 0.3
of the Prime Minister’s Office with an eye to enthuse = × 100 = 21.42%
1.4
Indian businesses for Modi’s ‘Make in India’ campaign.
= 21% (Approx.)
80. B Sachin Bansal & Binny Bansal are founders of Flipkart.

Previous Years
Page 76 CLAT & AILET Papers
93. D Required percentage 100. B Out of the 5 girls, 3 girls can be invited in 5C3 ways.
Nothing is mentioned about the number of boys that he
2.2
= × 100 = 73.33% has to invite. He can invite one, two, three, four or
3 even no boys. Each boy can be invited or not. He can
invite them in 24 ways. Thus, the total number of ways
94. A Total number of units manufactured by company A is 5C3 × (2)4 = 10 × 16 = 160.
and B together in the year 2009
= (1 + 2.4) × 100 101. D Let the no. of students in front row be x.
= 3.4 × 100 = 340 So, the no. of students in next rows be x – 3,
Total number of units sold by company x – 6, x – 9,…. so on
A and B together in the year 2009 If n i.e. no. of rows be 3, then
= (0.4 + 1.3) × 100 x + (x – 3) + (x – 6) = 630
= 1.7 × 100 = 170
⇒ 3x = 639
Hence, required ratio = 340 : 170 = 2 : 1
⇒ x = 213
95. B Required average number of units So possible.
Similarly, for n = 4,
(2.2 + 1.9 + 1.5 + 1.2 + 1.1) × 100
= x + (x – 3) + (x – 6) + (x – 9) = 630
5 ⇒ 4x – 18 = 630
7.9 × 100 790 648
= = = 158 ⇒x= = 162
5 5 4
∴ x = 4 to possible.
x0.4 32 If n = 5,
96. A = 2.6 ⇒ x3 = 512 ⇒ x = 8 (4x – 18) + (x – 12) = 630
16 x
⇒ 5x – 30 = 630
⇒ x = 120
97. C [(0.111)3 + (0.222)3 – (0.333)3 + (0.333)2 + (0.222)]3 Again n = 5 is possible.
= [(0.111)3 + (0.222)3 + (–0.333)3 –3(0.111)(0.222)(0.333)]3 If n = 6,
=0 (5x – 30) + (x – 15) = 630
[If a + b + c = 0, then a3 + b3 + c3 – 3abc = 0] ⇒ 6x – 45 = 630
⇒ 6x = 675
⇒ x ≠ Integer
98. B Let the number be x.
Hence, n ≠ 6.
 5 9
x + 2× 2 + 3 102. D Number of possible outcomes = 36
∴   = 25
6 Favorable outcomes = (1, 4), (4,1), (2, 3) and (3, 2)
5 1
The required probability =
9
 5 9
⇒  x +  × + 3 = 30
 2 2
v1 t2 16
7 103. B As we know = = = 4 : 3.
⇒x= v2 t1 9
2

104. B The total time gain in 24 × 7 + 2 hrs = 6 minutes 48


99. A x = 163 + 173 + 183 + 193 is an even number seconds (i.e. 6.8 minutes)
Therefore, 2 divides x. To show the correct time it will have to gain 2 minutes

a3 + b3 = (a + b)(a2 – ab + b2 ) 24 × 2 + 2
2 minutes gain can obtain in × 2 = 50hrs
6.8
⇒ a + b always divides a3 + b3.
Hence it will sow the correct time at 2 pm on
Therefore, 163 + 193 is divisible by 35. Wednesday.
183 + 173 is divisible by 35.
105. C The required percentage = 0.75 × 0.2 + 0.8 × 0.25
Thus, x is divisible by 70.
= 35.
Hence, option (A) is the correct choice.

Previous Years
CLAT & AILET Papers Page 77
106. C The sum of the number divisible by either 8 or 12 113. A The official answer key says (C) but the correct
= (the sum of number divisible by 8) + = (the sum of answer should be A.
number divisible by 12) – = (the sum of number The focus of the principle is on the competence to
divisible by both) contract and not on the burden of proof. The principle
( 200 + 208 + …….. 600) + (204 + 216 + ……+ 600) clearly states that the person should be of sound mind
– (216 + 240 + …….+ 60) at the time of entering into the contract, and says
nothing about burden of proof. Using that, the answer
51 34 17
= (200 + 600) + (204 + 600) − (216 + 600) should be A since Mr. X was not of a sound state of
2 2 2 mind at the time of entering into the contract.
= 27132
114. B First telegram had asked 2 questions, one regarding
401
Sum of all the number = [200 + 600] = 160400 the willingness to sell and the other regarding the
2
lowest price. In reply, only lowest price was quoted
Hence, the sum of the number which are divisible by and this quoting of price was not an offer. Third
8 nor by 12 = 160400 – 27132 = 133268. telegram 'I agree to buy' was an offer and not an
acceptance of the offer. Since this offer has not been
107. D In each team, T j there are two players, one it accepted, there is no binding contract.
shares with T j–1 and other with T j–1 . Other Quoting the price is always an invitation to offer, and
(k – 2)players team T j shares with no other not an offer which means that the Parties are at the
team. So, total players which play for only one team stage of negotiation only. The principle doesn't elaborate
= (k – 2)n. on the distinction between offer and invitation to offer,
One player is common in T1 and T2, one in T2 and T3 so it presumes legal knowledge.
and so on.
Number of such players = number of pairs = n 115. A Clearly, getting back from the airport was part of the
So, total players = (k – 2)n + n = n(k – 1) task the Manager had assigned to Sanjay. Anything
that happened during the course of that time was the
108. C 1111111112 = 12345678987654321. Master's responsibility to compensate for. So
Brookebond is liable.
109. C Let the number of stones be ‘n’.
As the person covers 4.8 km, he covers 2.4 km on 116. A The principle defines nuisance as unlawful
one side and 2.4 km on other side. interference with someone's enjoyment of their land,
So total distance covered by him = 20 + 40 + 60 +……. and the long queues due to the defendant's actions
n definitely interfered with the other shopkeepers' right
∴ 2400 = 2 × 20 + (n − 1) 20  = 10n (n + 1)
2
to enjoy their property.

(Here n is the number of stones) 117. C A's actions of throwing his son inside the well and
After solving, we get n = 15 then jumping in himself show that he was incapable of
∴Total number of stones = 15 + 15 + 1 = 31 understanding the consequences of what he was
doing.
( )
502
110. A 72008 = 74 = (2401)502 = (2400 + 1)502
118. C This question presumes legal knowledge in a reasoning
= k × 100 + 1 question. The maxims translate to 'ignorance of law is
∴ The last 2 digits = 01. not an excuse' and 'ignorance of fact can be.'
The fact situation clearly tells us that the law was in
111. A The principle clearly stresses on the intention to take force before George carried that gold. He was at fault
property out, and since Ramu cut the tree with the for not declaring it in the Manifest for transit.
intention of taking it away, he can be prosecuted for
theft. 119. B This question pertains to remoteness of damage, and
the question here is whether or not Krishnan could
112. B This question requires legal knowledge since the reasonably have foreseen that a pregnant woman
meaning of injury and damage has not been clearly would experience nervous shock on seeing the blood
explained. However, this fact situation is also one of on the road and have an abortion. He could reasonably
the first examples of injuria sine damnum that's given have foreseen the cyclist's death, but not the remote
while teaching. It is clear that even though there was consequence of someone else facing problems with
no actual harm from Monu's inability to cast his vote, that death.
his right to vote nonetheless got violated (causing him
legal injury) and he can sue.

Previous Years
Page 78 CLAT & AILET Papers
120. B Preparation means to arrange necessary measures property. A is factually true, but R isn't because 'person'
for the commission of the intended criminal acts. can refer to lifeless things like corporations, religious
Intention alone or intention followed by preparation is and charitable institutions.
not enough to constitute the crime. Preparation has
not been made punishable except in few cases 128. B Social control refers to the act of establishing rules
because a person might change his mind last minute that must be followed by the members of a society,
and not commit the act even after having made all and law is how that control is achieved. It signifies the
necessary preparations. role of law in maintaining peace and harmony in the
However, the answer may also be (D) since the society. So A is true.
principle doesn't specify what special offenses are, As per Utilitarian philosophy, yes, the ultimate aim of
and whether the attempt to kill someone qualifies as a law should be to attain the greatest happiness for the
special offense, we don't really know if preparation greatest number of people. So R is also true, but R
would be an offense in this case or not. isn't the correct explanation for A because laws don't
exist for Utilitarian purposes alone.
121. B This agreement is not capable of being made certain
as it is difficult to determine what luck, good or bad, 129. B The official answer key says (A) but the correct option
the horse had brought to the buyer. Hence, the should be (B).
agreement which is uncertain or incapable of being A and R are factually true, but the purpose of having
made certain, is void. freedom of speech and expression is grounded in the
individual's right to speak their mind in a democracy as
122. C As per the principle, A would have a duty to speak every person is equal in the eyes of law and should
only in situations where the circumstances are such have the freedom of speech and expression. It has
that his silence could amount to speech. Since no also been provided for in our fundamental rights
discussion regarding the soundness of the horse was guaranteed by the Constitution of India in consonance
initiated at the auction either by A or by B, A had no with the guiding principle. It doesn't have to do with
obligation to speak up. mankind's quest for truth.

123. A Removing someone's car, however innocently, 130. A Both A and R are factually true, and R is the reasoning
amounts to direct physical interference. Since the behind making attempt punishable. If not for external
principle says nothing about the intention of the person circumstances, the person would've ended up
doing it, X can be held responsible. committing the crime, so it doesn't make him very
different from the person who succeeds.
124. D The principle deals with the reasonableness of the
interference, which none of the options deals with. 131. Correct option not provided
Pavan may be liable for nuisance only if it is proved The official answer key provides (D) as the correct
that any reasonable person will be disturbed by the answer, but according to us the correct answer should
typing sound. But options A, B and C do not provide be - Both A and R are false.
proper explanation. Hence, the answer would be (D). A is false because several High Courts serve more
than one state or union territory. For instance, the High
125. B The principle states that prosecution and punishment Court of Punjab and Haryana looks after Punjab,
are both needed. Confiscation doesn't amount to Haryana and Chandigarh. While there are 29 States,
prosecution or punishment. Maqbool will be liable there are only 24 High Courts in India.
because confiscation of gold by the custom authorities R is false because Article 214 of the Constitution of
does not amount to prosecution. Judicial proceedings India states that "There shall be a High Court for each
alone can be said to constitute prosecution and can State" and not "in each State".
lead to punishment.
132. D A is false because the Council of Ministers is
126. D Custom derives its force from the fact that it has been accountable only to the Lok Sabha. Theoretically, there
the standard practice for a while, and recognized as is nothing to bar a Rajya Sabha member from becoming
such by the community. Custom can be recognized by a Union Minister, it is not the usual practice. Interestingly,
the Sovereign or the Court only later in time but the Indira Gandhi wasn't a member of either house when
question of any prior recognition does not arise in the she became the Minister for Information and
case of custom. So A is false. Broadcasting in 1964, and got elected to the Rajya
Custom can be codified into law by an act of the Sabha only later.
Parliament making it a source of law, but it isn't law by
itself. So R is true. 133. D A is false because 33% reservation across the
legislatures requires Constitutional Amendment,
127. C There are two types of person namely, Natural person whereas R is true because allocation of seats by the
and Juristic person. Juristic persons are created by political party is an internal matter for the party.
law. Idol is a Juristic person, capable of holding the

Previous Years
CLAT & AILET Papers Page 79
134. A A is factually true, as it's been quoted verbatim from 142. B The Supreme Court of India scrapped 214 coal blocks
the Preamble. As per the Preamble, India is a out of 218 coal blocks. The four coal blocks spared in
democratic republic and R is the intent of the Preamble. the judgment were those of Reliance Power, NTPC
R is a sound explanation for having a democratically and SAIL.
elected Head of State, and explains the rationale for
calling the country a Democratic Republic. 143. C The Constitution of India doesn't make it obligatory for
anyone to sing the National Anthem as it would violate
135. A Both are factually true, and 26th January is celebrated the right to freedom of speech and expression and
as the Republic Day because the Constitution came also, the right to freedom of conscience and practice
into force that day. and propagation of religion.

136. A The principle establishes the threshold for an act being 144. A Under the new Certificate of Practice and Renewal
considered attempt, and says that it's only when if not Rules, 2014, the Bar Council of India has stated that
for an external event, the person would definitely have an advocate cannot start his/her practice in the
gone ahead with the deed, would their actions be Supreme Court unless they have at least five year
considered attempt. practice experience in the lower and High Court of
Here, Rani could've changed her mind before jumping India.
into the well at any point between the starting point
and her actually jumping, so her action of starting to 145. A The Union Cabinet approved 33 per cent reservation
run doesn't qualify as attempt. for women in police forces of all Union Territories,
including the Delhi Police, through direct recruitment in
137. A The reasoning given in the option is self-explanatory. non-gazetted posts. The move aims at increasing
SINY cannot be held liable for something he never representation of women in police and to make the
intended to do. force gender sensitive. This reservation for women
will be applicable "horizontally and in each category
138. A Here, JAM's actions match the threshold for attempt. (SC/ST/OBC and others) through direct recruitment in
Had it not been for JANE managing to escape by non-gazetted posts - from constables to sub-
herself, JAM had done enough to ensure that she inspectors." The reservation will be available to
ended up dead. women against all such vacancies existing on the
date of issue of order and vacancies arising thereafter.
139. A Also called the National Judicial Appointments
Commission Bill, it was passed by the Lok Sabha on 146. C The Governor within his state is the supreme
14th August 2014. executive head.

140. C Brought in by former Chief Minister Narendra Modi and 147. C The NALSA was established to provide free and
known as The Gujarat Control of Terrorism and competent legal services to the weaker sections of
Organized Crime Bill, 2015, it empowers police to tap the society, so that their economic disabilities don't
telephonic conversations and submit them as evidence prevent them from getting justice. Hon'ble Mr. Justice
in court. It also makes confessions before police H. L. Dattu, the Chief Justice of India is the Patron-in-
officers admissible in court, and has therefore been Chief and Hon'ble Mr. Justice Tirath Singh Thakur,
the cause of much controversy. The new Bill is a re- Judge, Supreme Court of India is the Executive
worked version of the Gujarat Control of Organised Chairman of the Authority.
Crime Bill (GUJCOC), 2003, which was earlier rejected
twice by the President due to some of its contentious 148. C The purpose of extradition is to ensure that criminals
provisions. are prosecuted under the more appropriate jurisdiction.
Such treaties ease the rendition of criminals from one
141. A 20th Law Commission headed by Justice (retd.) A P country to another.
Shah in its 210th Report on Humanization and
Decriminalization of Attempt to Suicide had 149. C It is a mock court at which students argue imaginary
recommended that Section 309 of IPC needed to be cases for practice. There are international as well as
effaced from the statute book because the provision domestic moots.
is inhuman, irrespective of whether it is constitutional
or unconstitutional. 18 States and four Union Territories 150. B Parole is usually given after proven good behavior
have backed the government's decision. S. 309 of the within the prison, before the expiry of the sentence.
IPC deals with attempt to commit suicide. It was In India, the grant of Parole is largely governed by the
understood as a cruel and irrational provision, since it rules made under the Prison Act, 1894 and Prisoner
only caused additional trauma to someone who would Act, 1900. Each of the States has its own parole rules.
already be dealing with the consequences of not being There are two types of parole- custody and regular.
able to commit suicide. It also has implications for the The custody parole is granted in emergency
right to die and the euthanasia debate. circumstances like death in the family, serious illness

Previous Years
Page 80 CLAT & AILET Papers
or marriage in the family. It is limited to a time span of three-dimensional justice, as provided for in the
six hours during which the prisoner is escorted to the Preamble. This Bench would function from 12th
place of visit and return there from. The grant of parole December 2014 and in order to ensure that these
is subject to verification of the circumstances from matters are monitored on regular basis, will continue
the concerned police station and is granted by the to sit on every working Friday at 2.00 p.m.
Superintendent of Jail. Regular Parole is allowed for a
maximum period of one month, except in special 155. D The purpose of the 14th finance commission was to
circumstances, to convicts who have served at least give recommendations on specified aspects of Centre-
one year in prison. It is granted on certain grounds State fiscal relations during 2015-20.
such as: Serious Illness of a family member, accident
or death of a family member, marriage of a member of 156. B As per article 159, the Chief Justice of the concerned
the family, delivery of child by wife of the convict, High Court administers oath to the Governor.
maintain family or social ties, serious damage to life or
property of the family of convict by natural calamities, 157. D Article 123 of the Indian Constitution empowers the
pursue filing of a Special Leave Petition etc. Certain President to promulgate ordinances during the recess
categories of convicts are not eligible for being of the Parliament. The ordinances are temporary laws
released on parole like prisoners involved in offences having the effect of an act. It is considered one of the
against the State, or threats to national security, non- important legislative powers of the President, however
citizens of India etc. People convicted of murder and it is subject to limitations. An ordinance is valid for six
rape of children or multiple murders etc. are also months from the date of the start of the next session.
exempted except at the discretion of the granting
authority. 158. B A quasi-federal state combines the features of a
federal government and those of a unitary government.
151. C This question doesn't have a principle, so one would India is not a true federation. It combines the features
have to rely on legal knowledge alone. The principle of of a federal government and the features of a unitary
damnum sine injuria would apply as there's damage government which can also be called the non-federal
but no legal injury. Over-crowding is a common enough features. Because of this, India is regarded as a semi-
occurrence on trains, and there cannot be a remedy federal state. Prof. K. C. Wheare describes it as "a
against it. Authorities are not legally bound to ensure quasi-federal state". The Supreme Court of India also
the convenience of every passenger. describes it as "a federal structure with a strong bias
towards the Centre".
152. C All three are correct
(i) Fraud is committed with a dishonest intention 159. A As per Article 53, Presidential executive powers can
whereas representation is when you believe be exercised either directly or through officers
something to be actually true. subordinate to him.
(ii) Misrepresentation renders the contract voidable
i.e. it gives an option to the innocent party to avoid 160. C The vote on accounts deals only with the expenditure
the contract but under torts, one can claim in a government's budget. The government gives an
damages if the contract is rescinded rightfully or estimate of the funds it needs to meet the expenditure
if one has suffered damages because of non- it incurs during the first 3-4 months of a financial year.
fulfillment of the contract. The Indian Constitution says that all revenues received
(iii) One cannot complain of mis-representation if had by the Union government and the loans raised by it are
the means of discovering the truth with ordinary to be put into the Consolidated Fund of India. This does
diligence and even in cases where fraud has not include anything that is put into a Contingency
been committed by maintaining silence. Fund. Since Parliament is not able to vote the entire
budget before the commencement of the new financial
153. A year, it is necessary to keep enough money at the
disposal of government to allow it to run the
154. D The Supreme Court of India has set up a Social Justice administration of the country. When the government
Bench, which comprises of two judges devoted to needs to withdraw any money from the Consolidated
the delivery of speedy justice in a range of social Fund of India to cover its expenditure (especially during
issues related to the downtrodden and socially the time when elections are underway and a caretaker
marginalised groups. It's a brainchild of Chief Justice government is in place), it has to seek approval from
of India H.L. Dattu and the Bench is headed by Justice the Parliament. A special provision is, therefore, made
Madan B. Lokur and also has Justice U.U. Lalit. The for a vote-on-account by which the government
purpose behind constituting the new Bench was to obtains the vote of Parliament for a sum sufficient to
streamline cases highlighting social issues before one incur expenditure on various items for a part of the
court and thus facilitate the Supreme Court's monitoring year. This sanction of Parliament for withdrawal of
and review of the government's action in such cases. money from the Consolidated Fund of India to meet the
The social justice bench has been set up to ensure government's expenses is generally known as a vote-
on-account.

Previous Years
CLAT & AILET Papers Page 81
For questions 161 to 168: 170. C I did it before C since 27 is the wrong choice for I.
X had items: 2 chessboard cost of each is Rs.500 & 1 record Similarly, A, D and E made keys before C. Hence,
player of cost Rs. 2000 4 people made their keys before C.
so X had in beginning items of total cost of Rs. = 3000
after transaction he had only 1 cycle of cost Rs. = 1000 & 1 171. D Both G and H can’t be sources to any of F, B or I and
chessboard of his own loss in transaction = 1500
hence, (d) is the correct option.
Y had items: 3 cricket bat each of Rs.700
So Y had in beginning items of total cost of Rs = 2100
after transaction he had only 1 record player of cost 172. C F introduced wrong answer to question 14, because
Rs = 2000 & 1 bat of his own nobody else has done question 14 wrong.
profit in transaction = 600
Z had items: 1 cycle cost of Rs.1000 & 1 walkman of cost 173. D A, D and G have one distinct wrong answer and no
Rs.700 blank answers. So they must have the same source.
So Z had in beginning items of total cost of Rs. = 1700 E and H also have a common wrong answer and no
after transaction he had only 1 bat of cost Rs. = 700 & 1 blanks.
chessboard of cost = 500 and 1 camera = 1500 Hence, both the groups had identical sources.
total = 2700
W had items: 2 camera cost and profit = 1000 of one is 174. D The first as well as the last digit is even. Hence the
Rs. 1500 & other one of cost Rs.3500
correct code is $KEFM$.
so X had in beginning items of total cost of Rs. = 5000
after transaction he had only 1batof cost Rs. = 700 & 1 camera
of his own of cast Rs. = 3500 & a walkman of Rs. 700 175. B The last digit is 0. Hence, the correct code is E%*BA#
loss in transaction = 100.
176. C The correct code is EFBRK@
161. B Rs.1700
177. D The first as well as the last digit is odd. Hence the
162. A W has the costiest items. correct code is XFAK%X

163. B X does not have cricket bat after exchange. 178. A The first as well as the last digit is odd. Hence the
correct code is XAFK*X
164. D Z gained the highest.
179. B Statements B and D are certainly facts. Statement C
165. C The maximum less is Rs.1500.
begins with “assuming” which means the sentence is
166. B (Incorrect) an inference.
The total amount is Rs.11,800, so the correct answer
should be option (B). While the official answer key is 180. (Incorrect)
option (D). The CLAT answer key is option (A). However,
statement B is a judgment and not a fact as suggested
167. A by the answer. Hence, option (A) is incorrect. The
correct answer choice is not given.
168. (Incorrect)
Walkman faced the highest exchange value, which is 181. B Statements A and C are certainly judgments because
not in the option. they reflect the opinion of the author. Statement B is
clearly a fact while statement D is an inference
For questions 169 to 173: The last row of table should be
because it is based on certain premises.
I 27 17, 46, 90 and according to this the solution as follow.
182. (Incorrect)
If a person copies from one source, he must have the same
blank answers as the source and exactly one wrong answer The CLAT answer key is option (A). However, we
more than the source (as the copier has introduced one believe that option (D) is the correct answer because
wrong answer on his own). If the person copies from two option (D) represents the correct sequence of
sources, the distinct wrong answers from the two sources inference. There is a printing error in statement (b).
would be left blank by the copier and the same wrong answers Wealth should have been health.
in the two sources would be copied as it is.
183. C The correct sequence is AEB. Since MBAs are in great
169. B A and D have only one source since they have only demand (A) and we know that Samrat and Akshita
one wrong answer each. are MBAs (E), we can infer that Samrat and Akshita
C may have copied from only one source i.e. I and will be in great demand as well.
hence, only B has two sources.

Previous Years
Page 82 CLAT & AILET Papers
184. (Incorrect) 194. A From the Venn diagram, it is evident that no fruit is
The CLAT answer key is option (b). However, we black and some fruits are vegetables. However, the
believe the answer choice is wrong. Statement B second conclusion cannot be inferred. Hence, option
states: Infotech employees are knowledge workers (A) is correct.
which means all Infotech employees are knowledge
workers. However, statement A and C combined imply 195. C The movement is as follows
that Infotech employs knowledge workers. It does not
necessarily mean that All Infotech employees are N
knowledge workers.
W E
185. C (Incorrect) Sta rting P o in t
Applying condition 2 the relation represented below S

Q M

B I
Which represent I is nephew of Q. The official answer
196. C The arrangement of the houses is as follows:
key is option (D) which is wrong.
Sandeep Mrintuyay Nayak Mishra Aalekh Iliyas
186. (Incorrect) [It may be in reverse order also but answer will be
Option (C) and (D) are same. Any can be correct same in the both the cases].
answer.
197. (Incorrect)
187. A (Incorrect) The CLAT answer key is option (B). However, option
The CLAT answer key is option (C). Statement II in this (B) is incorrect because we can make at least 3 words
question is unnecessary. The answer can be inferred from Laparoscopy / Lap, Par and Coy.
from Statement I itself. Hence the correct choice is
option (A). 198. B Number of odd days is 22 i. e. 1. Hence the required
day is Tuesday
188. B When a person becomes a slave to a certain habit,
s/he finds it difficult to try anything else. Hence, his/ 199. D The order of letter after interchanging is as follows:
her life becomes rigid. The first statement doesn’t
answer the question. Envying others and becoming LGS O T OGPRIAH
out of habit are two different things.
200. D The family diagram is represented below:
189. D “Bad teaching” or “dull home environment” does not
include the ability to learn. Hence, statement I is M
insufficient. Furthermore, statement II is off the mark
because possessing intelligence and performing well Son
in school are two completely different things.
N
190. C Letters at the odd places have been moved one place
back, and letters at the even places have been moved B S R
one place ahead regarding the position of codes given
M is great grandmother/father of B.
in the table

191. A Wrong Question, The correct question should be


FASHION is z64t7Uw, then POSITION is. Assuming
this, the correct answer is option (A).

192. B Letters have been moved one place right regarding


the position of codes given in the table

193. A It is clear from the Venn diagram that all players are
not tall. Hence, only conclusion I follows.

Previous Years
CLAT & AILET Papers Page 83
CLAT Solutions 2016
English 12. 2 The phrase introduced by 'along with' modify the earlier
word (Mani in this case), but it does not compound the
1. 4 Option (1) is incorrect because 'the' is redundant. subjects (as the word and would do). So, the blank
Option (2) is incorrect because 'most' indicates the will take a word that goes with Mani (singular). Hence,
greatest in number, quantity etc. So, 'most' will not go option (2), 'goes' is the correct answer.
with 'reading'. Option (3) is also incorrect. 'Best' means
excellent. It cannot be used to make comparison 13. 1 'To come across' means to meet or find by chance.
between two things. Option (4) is the correct answer. Hence, option (1) is the correct answer. 'Come around'
'More than' is used to compare two things. means to agree to do something eventually, after a
long wait. 'Come to' means to become conscious.
2. 2 The grammatically correct phrase is 'not to get', 'Come at' means to make a threatening move toward
rendering option (2) the answer. someone or something.

3. 2 'Whoever' means whatever person. 'Whomever' is 14. 1 'At' is used to indicate the place where someone or
the objective case of 'whoever'. So, 'whoever' fits in something is. 'On' is used to indicate the part or object
the blank, rendering option (2) the answer. by which someone or something is supported. 'To' is
used to indicate the direction of something. 'In' is used
4. 2 'Knock' takes the preposition 'at' after it. Hence, option to indicate location or position within something. Only
(2) is the correct answer. 'at' fits in the meaning of the sentence, rendering option
(1) the answer.
5. 2 When someone dies as a result of a disease, we say
that they 'die of' the disease. Hence, option (2) is the 15. 2 'In' is used as a function word to indicate means,
correct answer. medium, or instrumentality. Hence, option (2) is the
correct answer. 'From' is used to indicate the starting
6. 4 'Pedagogy' is the art, science, or profession of teaching. point of a physical movement or action. 'With' is used
Hence, option (4) is the correct answer. 'Philately' is to say that people or things are together in one place.
the study or collection of postage stamps. 'Paediatrics' 'On' is used as a function word to indicate position in
is the branch of medicine concerned with children or in contact with an outer surface.
and their diseases.
16. 2 The correct spelling of the word is 'sacrilegious'.
7. 1 'In hand' means receiving or requiring immediate Hence, option (2) is the correct answer.
attention. So, the first blank will take 'in'. The second
blank will take 'the' since we are talking about a 17. 1 The correct spelling of the word is 'barrister'. Hence,
particular picnic. option (1) is the correct answer.

8. 4 'Break away' means to leave or to escape from 18. 1 The correct spelling of the word is 'deceive'. Hence,
someone who is holding you. 'Break off' means to option (1) is the correct answer.
separate a part from a larger piece, or to become
separate. 'Break from' means a time away from work 19. 4. The correct spelling of the word is 'collaborate'. Hence,
or your regular activity, or a holiday. 'Break out' means option (4) is the correct answer.
to start suddenly. Only option (4) fits in the meaning of
the sentence and hence, is the correct answer. 20. 1 The correct spelling of the word is 'integrity'. Hence,
option (1) is the correct answer.
9. 2 The sentence suggests that the professor is still
teaching. So, the first blank will take 'has been'. The 21. 4 'Interfere with something' means to prevent something
second blank will take 'since' as 'since' is used to from happening or developing in the correct way.
refer back to a previous point in time while 'for' is used 'Interfere in' means to deliberately become involved in
to refer to the length of a period of time. Hence, option a situation and try to influence the way that it develops,
(2) is the correct answer. although you have no right to do this. Hence, option
(4) is the correct answer.
10. 4 'Any' means even the smallest amount or number of.
Hence, option (4) is the correct answer. Option (3) 22. 3 'Get out' means to come out. Hence, option (3) is the
would have been correct had it been 'a lot of'. correct answer. 'Get off' means to start, as on a trip.

11. 4 'Being' is the present participle of 'be'. Hence, option 23. 2 Out of the four options, only 'correcting' goes with
(4) is the correct answer. The blank can also take 'to 'pronunciation'. Hence, option (2) is the correct answer.
be'.

Previous Years
Page 84 CLAT & AILET Papers
24. 2 'Ship' is personified as female. So, option (2) is the 40. 1 Refer to the line "The ingenuity of the scientist brought
correct answer. him enough wealth…". Hence, option (1) is the correct
answer.
25. 4 'Faux pas' means social blunder. Hence, option (4) is
the correct answer.
General Knowledge
26. 2 'Burry the hatchet' means to end a feud with an enemy.
Hence, option (2) is the correct answer. 1. 1 The Narendra Modi government has announced the
names of the first 20 cities which will be developed
27. 1 'Ab initio' is a Latin phrase which means from the into smart cities in 2016. These 20 cities were chosen
beginning. from 98 cities shortlisted for the 'Smart Cities Mission'.

28. 4 The literal translation of 'amicus curiae' is friend of the 2. 3 The Boy Scouts of America (BSA) is one of the largest
court. Hence, option (4) is the correct answer. youth organizations in the United States, with more
than 2.4 million youth members and nearly one million
29. 3 'When ignorance is bless it is folly to be wise' is an adult volunteers. It's motto is 'Be Prepared'.
idiom which means if knowing something makes you
unhappy, it would be betternot to knowit. Hence, option 3. 4 The Election Commission of India announced the
(3) is the correct answer. schedule for Assembly elections in four States and
one Union Territory, which will be conducted in multiple
30. 2 'Still waters run deep' is a proverb which means a phases from April 4 to May 16. Elections in Tamil Nadu,
quiet or placid manner may conceal a passionate Kerala and Puducherry will be held in a single phase
nature. Hence, option (2) is the correct answer. on May 16, said Chief Election Commissioner Nasim
Zaidi.
31. 1 The passage is descriptive in nature. It talks about
people who invented certain things with a good 4. 3 Beyond the Lines - An Autobiography" is authored by
intention but their inventions were used for negative Mr. Kuldip Nayar. As a young law graduate in Sialkot
or destructive purposes. Hence, option (1) is the (now in Pakistan), Kuldip Nayar witnessed at first
correct answer. hand the collapse of trust between Hindus and Muslims
who were living together for generations.
32. 1 Refer to the line "In 1939, fearing that the Nazis would
win…launch an American programme on nuclear 5. 2 The Thai Baht is the currency of Thailand.
research." Hence, option (1) is the correct answer.
6. 2 Telangana has become the first State to introduce
33. 2 Refer to the paragraph "Paradoxically, Nobel's life was compulsory gender education at the graduate level;
a busy one…worked for the promotion of peace." without repeating gender stereotypes in its bilingual
Hence, option (2) is the correct answer. textbook titled, 'Towards a World of Equals.'

34. 1 We can infer from the second paragraph that 7. 2 Japan has again demonstrated its prowess in high-
Immanuel's interest in dynamites influenced Alfred's speed rail travel with its state-of-the-art maglev train
inclination for working with explosives. Hence, option setting a world record of just over 600km/h (373mph),
(1) is the correct answer. just days after it broke its previous 12-year-old record.
The seven-car maglev - short for "magnetic levitation"
35. 1 'Endorsement', as used in paragraph 5, means - reached a top speed of 603km/h during what officials
expressing one's approval or support. described as a "comfortable" zip along a test track
near Mount Fuji.
36. 2 'Accomplished', as used in paragraph 4, means
completed successfully. 8. 2 Parliament passed two bills to repeal outdated 1053
old laws which had become redundant legislations.
37. 3 The paradox indicates that the writer has an analytical The Appropriation Acts (Repeal) Bill 2015 that seeks
mind. Hence, option (3) is the correct answer. to repeal 758 old appropriation acts which have lost
relevance and The Repealing and Amending (Third)
38. 4 Refer to the line "Paradoxically, Nobel's life was a Bill, 2015 to repeal 295 enactments and to amend certain
busy one yet he was lonely Paradoxically, Nobel's life other enactments were passed by the Rajya Sabha
was a busy one yet he was lonely". Hence, option (4) on Wednesday by voice vote.
is the correct answer.
9. 3 With a view to facilitating visually impaired train
39. 1 Refer to the fifth paragraph. It clearly says that the passengers, tactile maps of the railway station and
Manhattan project was initiated to carry out nuclear train schedules in Braille were unveiled at the Mysuru
research. Railway station. Speaking to reporters after Mysuru
MP Pratap Simha inaugurated the facility; Divisional

Previous Years
CLAT & AILET Papers Page 85
Railway Manager Rajkumar Lal said Mysuru railway 23. 3 Long jumper Mayookha Johny fetched India a gold
station has become the country's first visually medal while sprinter Dutee Chand settled for a bronze
challenged friendly station in India. after shattering national record in 60m dash in the
Asian Indoor Athletics Championships.
10. 1 Samrupa, the world's first cloned buffalo calf, was to
be India's answer to Dolly the sheep. But unlike Dolly, 24. 1 Sania Mirza was conferred the Rajiv Gandhi Khel Ratna
the first mammal cloned 13 years ago, who lived for award during August, 2015.
seven years, Samrupa succumbed to a lung infection,
five days after it was born. 25. 4 The author of "Crime & Punishment" is Fyodor
Dostoyevsky
11. 4 Delhi girl Priyadarshini Chatterjee has bagged the crown
for FBB Femina Miss India World; she will now go on 26. 3 The first finance minister of independent India was R.
to represent India at the Miss World 2016 pageant. K. Shanmukham Chetty, who also presented its first
Budget. The incumbent Arun Jaitley, of the Bharatiya
12. 1 The Prime Minister is Ex-Officio Chairperson for NITI Janata Party, has held office since 26 May 2014.
Aayog.
27. 4 The Constitution (One Hundredth Amendment) Act,
13. 1 Sir B.N. Rau was the Constitutional Advisor to the 2015 was enacted to give effect to the transfer of
Constituent Assembly in framing the Indian Constitution. certain territories by India to Bangladesh and transfer
of certain territories from Bangladesh to India.
14. 2 India has the largest diaspora in the world, followed
by Mexico and Russia. In 2015, 16 million people from 28. 3 15 March is World Consumer Rights Day (WCRD), an
India were living outside of their country, a growth annual occasion for celebration and solidarity within
from 6.7 million in 1990, the survey stated. the international consumer movement. It marks the date
in 1962 President John F Kennedy first outlined the
15. 3 Maithripala Sirisena is the current and 6th President definition of Consumer Rights.
of the Democratic Socialist Republic of Sri Lanka.
29. 4 The last-minute overtures made by Greece to its
16. 1 The official language of the Union shall be Hindi in international creditors for financial aid could not save
Devanagari script. The form of numerals to be used the country from becoming the first developed
for the official purposes of the Union shall be the economy to default on a loan with the International
international form of Indian numerals. Monetary Fund (IMF).

17. 4 The President has the power to summon and prorogue 30. 1 The direction to hold floor test to prove majority in the
either House of Parliament or to dissolve Lok Sabha. Legislative Assembly of Uttarakhand , to be held on
10th May, 2016 has been given on 6th May, 2016 , by
18. 2 The Duke and Duchess of Cambridge have sat together the Supreme Court of India.
on the bench at the Taj Mahal in India where Diana,
Princess of Wales, was pictured 24 years ago. Prince 31. 3 Katerina Lehou lit the torch from the sun's rays
William and Kate Middleton posed at the mausoleum in reflected in a parabolic mirror during the Olympic flame
Agra on the final day of their tour of India and Bhutan. lighting ceremony for the Rio 2016 Olympic Games at
the site of ancient Olympia in Greece on 22nd April,
19. 4 Council of Minister of Nepal consists of the Prime 2016.
Minister, six Deputy Prime Ministers, seventeen
Ministers and four State Minister. The incumbent Prime 32. 2 Prime Minister, Mr. Narendra Modi, in March, 2016,
Minister is Khadga Prasad Oli. launched an ambitious programme "Setu Bharatam".
The programme is aimed at making all National
20. 1 In 2015, Prime Minister Mr. Narendra Modi launched a Highways railway level crossing free by 2019.
new campaign, "Start up India, Stand up India". The
campaign is aimed at promoting bank financing for 33. 4 The highest peace time gallantry award Ashok Chakra
start ups and offer incentives to boost was awarded posthumously during 2016 to Mohan
entrepreneurship and job creation. Nath Goswami.

21. 1 Karnataka is the most preferred state by foreign 34. 3 Former Chief Justice of India HL Dattu took charge as
students in India, according to a report prepared by the Chairperson of the National Human Rights
the Confederation of Indian Industry (CII). Commission (NHRC). He is the seventh Chairperson
of the Commission. Justice KG Balakrishnan completed
22. 1 The Indian Parliament comprises of the President and his tenure on May 11, last year following which member
the two Houses-Lok Sabha (House of the People) and of the Commission Justice Cyriac Joseph held charge
Rajya Sabha (Council of States). as the acting Chairperson

Previous Years
Page 86 CLAT & AILET Papers
35. 4 On 18 October 2010, Justice Lokeshwar Singh Panta executive director, (Corporate Communications and
became its first Chairman. Currently it is chaired by Branding), IndianOil, said that of the 120 "smokeless
Justice Swatanter Kumar since 20 Dec 2012. The villages" in the country, the maximum were in
Tribunal's dedicated jurisdiction in environmental Karnataka. The country's first "smokeless village" was
matters shall provide speedy environmental justice Vychakurahalli of Gauribidanur taluk of Chickballapur
and help reduce the burden of litigation in the higher district.
courts.
46. 2 Indomitable Spirit is an awesome book written by
36. 4 Pakistan's parliament became the first in the world to Dr.A.P.J Abdul Kalam. It is the experience from his
completely run on solar power, a venture supported own life's journey from the shores of Rameswaram to
by close ally China with a whopping $55 million funding. the hallowed portals of Rashtrapati.

37. 2 The "Paris Agreement" was adopted in the twenty 47. 1 India signed the Paris Agreement on climate change,
first session of Conference of Parties in the month of adopted by more than 190 countries in December
December, 2015. 2015,in New York in April 2016.

38. 3 In an attempt to curb black money, the Government 48. 2 Mercury is the fastest planet, which speeds around
has made PAN mandatory for all financial transactions the sun at 47.87 km/s. In miles per hour this equates to
exceeding Rupees 2 lakhs. a whopping 107,082 miles per hour.

39. 3 Dipa Karmakar created history by becoming the first 49. 1 In case the President of India wants to resign, he shall
Indian woman gymnast to qualify for Olympics as she address his resignation to the Vice-President.
booked a berth for the Rio Games after a strong
performance at the final qualifying and test event. 50. 4 Brendon McCullum has blasted the fastest century in
Test history, a 54-ball effort that broke the record jointly
40. 2 In case of death of both the President and Vice- held by Viv Richards and Misbah-ul-Haq.
President of India Chief Justice of India shall act as the
President of India. Elementary Mathematics
41. 4 Punjabi is spoken as a minority language in the world 1. 1 Let the value of machine three years ago was Rs. x.
and is the third most spoken language in Canada! 3
Recently, Punjabi has become the third most spoken  10 
∴ x 1 −  = 729 ⇒ x = 1000.
language in Parliament of Canada after English and  100 
French.
2. 2 Error in sum = (77 – 27) + (90 – 9) = 126
42. 1 Mars is the fourth planet from the Sun and is the second 126
smallest planet in the solar system. Named after the ∴ Correct mean = 63 + = 64.75.
72
Roman god of war, Mars is also often described as
the "Red Planet" due to its reddish appearance. Mars 3. 1 Inner radius of that park = 21 m
is a terrestrial planet with a thin atmosphere composed Outer radius of the park = 21 + 3.5 = 24.5 m
primarily of carbon dioxide.

43. 4 Indian educational institutes occupy 16 places among


( 2 2
)
∴ Area of path = π (24.5) − (21) = 500.5 m2

top 200 universities in Times Higher Education BRICS ∴ Cost of gravelling = 500.5 × 4 = Rs. 2002.
and Emerging Economies rankings for 2016. At 16th
place, Indian Institute of Science, Bangalore is the 4. 4 Let the market value of share be x.
only Indian institute to feature in the top 20. ∴ 12% of x = 9% of 20
12 9
44. 3 Sikkim is declared as First Organic State of India. Shri ⇒ x= × 20 ⇒ x = 15.
100 100
Narendra Modi, Hon'ble Prime Minister, declared Sikkim
as first organic state of India. He appreciated Sikkim
for its transformation into an eco-friendly farming state. d d 15
5. d − = ⇒ d = 70 km.
Hon'ble Prime Minister inaugurated a state flower 35 40 60
show.
6. 2 Product of zeros = 4
45. 3 Around 60 villages in Karnataka have successfully
−6 −3
moved away from traditional stoves that use firewood ⇒ =4⇒a= .
and cow dung cakes to liquefied petroleum gas (LPG) a 2
stoves, thereby becoming "smokeless villages".
Announcing this on Wednesday, Indrajit Bose, 7. 2 Lesser than.

Previous Years
CLAT & AILET Papers Page 87
8. 3 x2 +(x + 2)2 = 1060 (x > 0) Legal Aptitude
⇒ x = 22
So the two numbers are 22 and 24 1. 3 Ramesh would be held liable as he directed the taxi
[Note: use option] driver to drive at high speed and basing on his
directions taxi driver drove at high speed, which
2 resulted in the accident. Basing on the third Principle,
9. 3 Amount of tin in alloy X = × 60 = 24 kg Ramesh would be held liable.
5
1 2. 2 Since "B" has consumed the eatables, which was
Amount of tin in alloy Y = × 100 = 20 kg delivered by "A", basing on the principle he is bound
5
to pay for the same.
∴ Amount of tin in new alloy = 24 + 20 = 44 kg.
3. 4 Basing on the Principle lady is entitled to claim
10. 2 Total number of pen = 132 + 12 = 144 compensation as she only knew that the driver of the
cab was little intoxicated and there was no assumption
132 11 of risk. Basing on the Principle, consent can be good
The required probability = =
144 12 defence only when there is both knowledge of risk
and assumption of risk.
999 99
11. 4 The required number = − = 100 4. 3 The Money which was lend to Sabu, was a collateral
9 9 agreement between XYZ bank and Sabu and had
nothing to do with the horse racing. The money was
12. 4 The required angle lend, in order to enable Sabu to award as prize to
Randeep, who was winner of Horse Race.
 11   11  1
θ =  30h − m  =  30 × 4 − × 25  = 17 degree.
 2   2  2 5. 2 'M' could not be made liable for the act of 'C', as 'C'
was appointed for the purpose of performing the work
of Conductor and not as driver of the bus. Act of 'C'
13. 2 Angle of elevation = angle of depression.
falls outside the course of employment and hence 'M'
cannot be made liable for the act of 'C'.
14. 4 In two days saving = Rs. 5
Saving in 16 days = Rs. 40 6. 3 The Principle clearly states that sale of liquor is illegal
Income on 17th day = Rs. 20 and any agreement related to such prohibited item
In hand on 17th day = Rs. 64. doesn't exist in the eyes of law, and hence the
agreement between 'A' and 'B', with regard to sale of
15. 1 LCM of 45, 75 and 90 = 450 liquor is not a valid agreement in the eyes of law and
i.e. 7 minutes 30 seconds hence can't be enforced.
∴ The required time = 7:27:45.
7. 1 Basing on the Principle, mere silence does not amount
to fraud and hence 'A' has not committed Fraud. Since
100 × 101 the Principle is not talking anything about agreement
16. 4 The required sum = = 5050.
2 with a minor, or agreement between a father and a
daughter, Option 3 and 4 would not be the right
17. 2 Let the fixed charge be Rs. x and the cost of food per answer.
day be Rs. y. 8. 3 This fact is not related to intense and sudden
∴ x + 20y = 1000 ...(i) provocation. Killing is not murder, when there is intense
x + 26y = 1180 ...(ii) and sudden provocation. Here 'A' killed his girlfriend
Solving the above equations, we get after 15 days, so this can't be the case of intense and
x = 400 and y = 30. sudden provocation.
9. 2 B's death is not accidental, as there was want of
 15   12  proper care and caution on the part of 'A'. It was the
18. 2 Price of TV = 15000  1 −  1 − 
 100   100  duty of 'A' to check the gun, before pointing it on 'B'.
= Rs. 11,220. There was negligence on the part of 'A'. The last line
of Principle clearly says that there must be proper
19. 1 The required amount of soup care and caution. If any of the ingredients of the Principle
fails, it can't be said that the death was by accident.
2
7 3
= π ×   × 4 × 250 = 38500 cm = 38.5 litres. 10. 4 The Principle says that causing of an effect partly by
2 an act and partly by an omission is an offence. Here
'A' has not only confined his daughter but also did not
provide any food to her and because of his act 'D',
5 × 510 + 240 × 25
20. 4 The required average = = 285. died of starvation and hence 'A' committed the offence
30 of causing death of 'D'.

Previous Years
Page 88 CLAT & AILET Papers
11. 3 To constitute an offence of theft, one must takes away 22. 1 Basing on the Principle, option 1 is the correct answer.
any movable thing from the land of any person without The fact that "A" citizen of England is accused of
that person's consent. Here 'A' just asked 'B' to commission of murder in India and of conspiracy
accompany him to the forest and has not moved any hatched in England are relevant facts.
movable things out of 'c' possession without 'c'
consent. 'B' is not a thing. 23. 2 Principle states that nothing is an offence which is
done by a child under 12 years of age, who has not
12. 1 Principle states that an agreement can be entered attended sufficient maturity of understanding to judge
orally, or in writing or by conduct. Here although here the nature and consequences of his conduct on that
is no oral or written agreement but conduct of 'A' and occasion. Here Himesh would not be protected under
'B' clearly implies that there was an agreement between the principle, because his act shows that he was
them. sufficiently mature to understand the nature and
consequences of his conduct.
13. 2 Principle clearly states that Law never enforces an
impossible promises and hence promise between 'A' 24. 3 Principle clearly states that import means bringing some
and 'B' can't be enforced. consignment into India from a foreign country. Here,
the consignment from Sri Lanka has entered the
14. 1 Basing on the Principle, which is talking about territorial waters of India, where India has its jurisdiction
defamation, 'A' has defamed 'S', by circulating pamphlet and it can be said the the consignment has been
saying that 's' is a thief. imported to India.

15. 3 The Principle states that "Communication of proposal 25. 1 The condition as to getting of consent for marriage
is complete when it comes to the knowledge of the from "C", "D", "E" has been complied by "B", after he
person to whom it is made. The Communication of got married. The Principle clearly states that, the
proposal made by 'A' will be complete, when 'B' reads condition to a contract can also be complied with after
the letter, sent by 'A" to "B'. the happening of event to which such condition is
attached. Hence, "B" has fulfilled the condition.
16. 2 Here 'A' cannot be said to have committed assault as
there was no apprehension as to use of physical 26. 1 "D" intentionally pulled the chair, when "P" was about
force. 'A' was sitting in a moving train, while 'B' was to sit and as a result of which 'P' falls on the floor and
standing on the railway platform at a distance and it is injured and hence basing on the Principle, 'D' is liable
was not possible for 'A' to hit 'B'. as he intentionally caused injury to 'P'.

17. 1 According to the Principle, letters or words not 27. 1 'A' has failed to arrest 'B', who killed 'C' and neither
describing the quality of things can be registered as informed the concerned authorities about the same
Trademark. The alphabet "B", used by Ram in way and hence, basing on the Principle 'A' has committed
describe the quality of Shoes manufactured by him. the offence.
Hence, the alphabet 'B' can be registered as trademark.
28. 4 The Principle is talking about the offence of
18. 3 A person who is usually of unsound mind, but "Misappropriation". Here, 'A' even after knowing that
occasionally of sound mind, may make a contract when property of "Z', does not belongs to "A", dishonestly
he is of sound mind. Basing on the Principle, it can be sells the property to stranger and hence, 'A' is guilty
said that 'A' can make the contract, when is of sound of offence of misappropriation.
mind 29. 1 Here 'B' has not abetted 'A' to kill "C'. Mere
acquiescence, does not amount to instigation. Here
19. 4 The Principle is based on the legal maxim, "Injuria sine "B" has not instigated "A" to kill "c".
damno". Here , Although there is no actual damage to
'P', as the candidate for whom he wanted to vote has 30. 2 Here "D" was totally justified in not allowing "P" to go
won the election, his legal right to cast vote has been where he wanted to go as "P" has not paid the required
violated and hence "P" would succeed in his action. admission fee, and allowed him to stay where he was
or to go back. Hence, "d" could not be made liable for
20. 1 Here "B' has not accepted the Proposal of "A" and has false imprisonment, as he did not totally restrict P's
made a counter offer to purchase the chair for Rs. movement.
400. A proposal is said to be accepted, when it is
accepted in the same form as made by the proposer 31. 3 Principle states that Law does not penalize for wrongs
and there is no change into it. which are of trivial nature. Here, the act of "A" was of
very trivial nature and hence "A" can't be made liable
21. 2 Principle clearly states that, "an agreement without for his act.
free consent can be enforced only at the option of the
parties whose consent was not free. Here, the 32. 4 Here "B" has committed the trespass as there was no
consent of 'B' was not free as it was obtained by use consent of "A" for entry in the bedroom. He was
of force, so only "B" can enforce the agreement. formally invited by "A" and hence he cannot take the
defence of consent.

Previous Years
CLAT & AILET Papers Page 89
33. 4 According to the Principle, Copyright law protects only For questions 7 and 8: The family tree is given below:
work and "work' means cinematographic film but does M ale
not include performance by an actor in a E C A
Fe m ale
cinematographic film and hence the acting of Alia Bhatt M arried cou ple
cannot be protected under copyright law.

34. 4 Principle is based on legal maxim, "Damnum Sine F B D


Injuria", which means injury without any legal damage.
Here, Prakash cannot succeed in his claim for 7. 1 8. 3
damages, as it is a case of damage without infringement
of any legal right. 9. 2 Both the statements are effects of independent
causes. There have been cancellations because of
35. 4 According to the principle defamation is the publications the fog while there has been considerable
of statement which tends to lower the reputation of a improvement in passenger amenities because of some
person in the estimation of other members of the unstated reason.
society generally.
Here, A's action can not be made liable under 10. 3 The government passed a legislation to make public
defamation, as there is no publication of the letter to information available to general public because the
any other person in whose estimation the reputation public did not have access to public information earlier.
of B's could be brought down. It was directly send to So, statement II is the cause and statement I is the
"b" and hence "A" cannot be made liable. effect. Hence, option (3) is the correct answer.

36. 1 37. 2 38. 3 39. 4 40. 1 11. 4 If men do not always pray to god, then the correct
statement will be that they seldom pray to god. Hence,
41. 3 42. 2 43. 3 44. 1 45. 2
option (4) is the correct answer.
46. 2 47. 4 48. 2 49. 4 50. 3
12. 3 Pattern of the sequence is as follows:
Analytical Reasoning 48 – 2 = 44, 44 – 4 = 40, 40 – 2 = 38, 38 – 4 = 34
34 – 2 = 32
For questions 1 to 3:
13. 1 The statement that convey the same meaning as that
Person Bus Car Scooter Tractor Auto given in the question is that 'Sometimes animals are
aggressive'. Hence, option (1) is the correct answer.
A √ √ √
B √ √ √ 14. 3 Patter of the sequence is as follows:
45 – 2 = 43, 43 – 3 = 40, 40 – 4 = 36, 36 – 5 = 31,
C √ √ √ √
31 – 6 = 25, 25 – 7 = 18
D √ √ √
15. 4 The required number of students = (84 + 8) – 1 = 91.
E √ √ √

1. 4 2. 3 3. 1 16. 3 It is a fact that sun rises in the east. So, a is logically


incorrect. We can measure the maximum duration of
4. 2 Since the convocation is on Thursday, we can assume every solar eclipse. So, it is verifiable. Hence, option
that he may reach the university on Wednesday. We (3) is the correct answer. (b) is not verifiable. Although
cannot assume when he will come back since he it is a fact that straight line is the shortest distance
might stay in Chennai for some time. Hence, option (2) between any two points but we cannot verify that for
is the correct answer. any two given points on planet Earth. Similarly, it is a
fact that every circle has a center, but that cannot be
5. 1 Assumption I is implicit because the statement clearly verified for any given circle.
says that people prefer to travel by X airline.
Assumption II is not implicit because even if advanced 17. 3 Let the number of diamond in the first bag = x.
German security system and on time performance
∴ The number of diamond in the second bad
record is implemented in airline Y, there still may be
x 5x
other factors that will make airline X more preferable = x+ =
than airline Y. Hence, option (1) is the correct answer. 4 4
According to given condition
6. 2 Assumption I is not implicit because there is no need to 5x
dissolve a board to hold a meeting. Next meeting can − x = 3 ⇒ x = 12.
be held even if the previous board is not dissolved. 4
Assumption II is implicit. Since there will be a board
18. 4 Waves are formed in oceans due to wind. Similarly,
meeting after 6 months, we can assume that the
sand dunes are formed in deserts due to wind. Hence,
company will remain in function after 6 months.
option (4) is the correct answer.

Previous Years
Page 90 CLAT & AILET Papers
19. 4 The number of cut require to cut the ribbon in 60 pieces 28. 4 All married people are not wives. So, if all wives are
is 59. married, we cannot assume that all married people
Hence, the required time is 59 seconds. are wives. Hence, option (4) is the correct answer.

20. 4 'Prosper', 'thrive' and 'flourish' are synonymous to each 29. 2 'Sanderling' is a type of bird. Similarly, 'mastiff' is a
other. 'Excite' is the odd one out, rendering option (4) type of a dog. Hence, option (2) is the correct answer.
the correct answer.
30. 1 The statement says that good governance conveys
21. 1 Option (1) cannot be true since bachelors do not have that there is law and order in the society. If the statement
wives. Hence, option (1) is the correct answer. is true, then we have to agree that an effective
government effectively uses laws to promote peace.
22. 3 If it is false that there is at least one octogenarian in Hence, option (1) is the correct answer.
the room than it means that there is no octogenarian in
the room. Hence, option (3) is the correct answer. 31. 4 Let the present age of Seema = 20 years
The present age of Geeta = 40 years
23. 1 If all humans are imperfect, then every human has to 10 years ago Geeta will be thrice that of Seema.
be imperfect. Hence, option (1) is the correct answer.
32. 1 All the other options are parts of a car. Hence, option
24. 2 Coding pattern is as follows: (1) is the correct answer.
+13
M → Z 33. 4 Option (4) simply rephrases the given statement. Hence,
+13 option (4) is the correct answer.
A → N
+13 34. It is clear from the pattern that the required figure must
I → V
contain one black dot.
+13
L → Y
35. 4 Mr. X came at 12:20 other members came at 12:40.
Similarly,
Since other members were 30 minutes late. So, meeting
+13 was scheduled at 12:10.
F → S
+13
I → V 36. 2 Since Socrates is an innocent person and no innocent
person is punished, Socrates should not be punished.
+13
L → Y Hence, option (2) is the correct answer.
+13
M → Z 37. 1 Amont with eldest daughter-in-law = 66 g
Amount with youngest daughter-in-law = 40 g
25. 4 Chief Justice of India does not act on behalf of Amount with middle daughter-in-law = 80 g
government, while the others act on behalf of the Amount with all the daughter-in-law = 186 g
government. Chief Justice of India is a part of judiciary. ∴ Amount with daughter = 186 g
[As half is given to the daughter and rest half is given
26. 4 Let the amount with B is Rs. x. to daughters-in-law.]
So, according to the condition amount with the person
is given below: 38. 4 Congenital disorder is a condition existing at or before
A → x + 13 birth. Myopia does not exist at or on birth. So, option
B →x (1) is incorrect. Option (2) is incorrect because it is an
C→ x + 8 opinion that it is the best form of government. Option
(3) is incorrect because water boils and not
x + 19 evaporates at 100 degree Celsius. Option (4) is correct.
D →
2 All radii of any given circle are equal.

x + 19 39. 2 Fundamentalism has a religious connotation that


Now x + = 50 ⇒ x = 27
2 indicates unwavering attachment to a set of irreducible
∴ Amount with A and B are Rs. 40 and Rs. 27 beliefs. So, if 'religious fundamentalism' is dangerous
respectively. to society, then disrespecting other religions will also
be dangerous to society. Hence, option (2) is the
27. 4 Just like a waiter works at a restaurant, a teacher correct answer.
works at a school. Hence, option (4) is the correct
answer. 40. 1 Let the member of team leaders be x.
∴ The member of Athletes = 20x.
Now x + 20x = 2100 ⇒ x = 100.

Previous Years
CLAT & AILET Papers Page 91
AILET Solutions 2008
1. C Over 54 percent of the Indian Territory fell under the 12. B The Constitution of India Describes India as a union of
states on the eve of Independence. states. (Article 1 of the Constitution of India).

2. D The Supreme Court has three types of jurisdiction -: 13. B Right to Equality is a fundamental right, which has been
1. Original Jurisdiction: (Article 131): This refers the subject of maximum litigation and controversies.
to the power to entertain those disputes which
can be initiated at Apex court at first Instance. 14. B The Fundamental Duties of the Indian Citizens were
2. Appellate Jurisdiction: (Article 132-133) This added by the Forty-Second Amendment in they year
refers to power to entertain those civil and 1976 which obliged them morally to perform mentioned
constitutional disputes which any High Court has duties therein.
given ‘certificate of appeal’ to the Apex Court.
15. C Eleventh Schedule to the Constitution of India contain
3. Advisory Jurisdiction:(Article 143)
Items in respect of which Panchayats shall have
This refers to the power to render an opinion by
powers and authority to function as institutions of Self-
Supreme Court of India on any question of public
Government.
importance on being asked by The President of India.
16. A The strength of the Council of Ministers has been fixed
3. B Any minister can introduce money bill in the parliament
by the constitution. Article 164 of the Constitution of
but no Money Bill can be introduced except on the
India. The Constitution (91st Amendment) Act, 2003,
recommendation of the President. It can be introduced
which limits the size of all ministries in India, comes into
in Lok Sabha only, not in Rajya Sabha. Article 110 of
force on July 7, 2004. This Act stipulates that the
Indian Constitution defines money bill. It should be
strength of a council of ministers should not exceed 15
remembered that only speaker of the Lok Sabha
percent of the total number of members in the Lok
certifies whether a bill is money bill or not.
Sabha (in case of the central government) or the
4. A The British Parliament is the most powerful legislature relevant state assembly. An exception has been made
in the world. only for smaller states such as Sikkim, Mizoram and
Goa where the strength of the assembly is 40 or less.
5. A In the U.S.A. residuary powers or reserve of powers There, the state government can have a maximum of
are vested with states as they have one list i.e. Federal 12 ministers.
List so all the other subjects fall under the ambit of
state legislations. 17. C The advisory Powers of the Supreme Court of India
(mentioned under Article 143 of the Constitution.) imply
6. C Estimates Committee does not have any members that it renders advice to the President on question of
from Rajya Sabha. It is a convention which has been law or fact, which is of public importance.
model on the lines with Westminster Model.
18. B Under article 136 of Indian Constitution, the Supreme
7. D As per our constitution, amendments regarding the Court can grant ‘special leave’ to appeal against
Annual Financial Statement, Appropriation Bill and decisions of lower courts and tribunals.
Demand for Grants cannot be proposed in either House.
19. D Unliquidated damages means damage to be assessed
8. C Nagpur session (1920) related to changes in the by a Court as these are not pre-determined
Constitution of Congress. Madras session (1929) is
related to Independence resolution passed for the 20. D Assault and nuisance are wrong under both tort and
first time. Calcutta session (1928) is related to Return criminal law.
of Gandhiji to active politics after 6 years. Lahore
21. A Criminal Force has been defined under Section 350 of
session (1929) is related to Poorna Swaraj. Karachi
IPC as whoever intentionally uses force to any person,
session (1930) is related to Resolution of Fundamental
without that person’s consent, in order to the
Rights and National Economics Policy.
committing of any offence, or intending by the use of
9. C Innuendos are words, which appear innocent, but such force to cause, or knowing it to be likely that by
have a latent defamatory meaning. the use of such force he will cause injury, fear or
annoyance to the person to who m the force is used,
10. A Article 41of our Constitution provides that the State is said to use criminal force to that other.
shall make effective position for securing right to work.
22. C The dispaly of the goods by the seller is an ‘invitation to
11. A The Foreign Diplomats does not constitute an offer’.
exception for ‘equality before law’ mentioned under
article 14 of the Constitution of India. 23. B An ideal of Shri Krishna is a legal person and treated
having juristic personality.

Previous Years
Page 92 CLAT & AILET Papers
24. A When a court sends some one in judicial custody, it if, instead of 200 Kms, it would have been mentioned
means he is sent to jail. as 200 Nautical Miles.

25. C There are certain Torts where element of ‘intention’ is


relevant while committing the act. For eg: assault,

Territorial
battery, false imprisonment, trespass etc.

26. B The minimum number of partners must be two, while


the maximum number can be 10 in case of banking
business and 20 in all other types of business.

27. B The term of office of a judge of the International Court


of Justice is 9 years.

28. A The democratic device, used in constitutional states,


by which important political questions, particularly
question relating tot he states of a region, may be
referred to the people of the region for their final
decision, is called ‘Plebiscite’.

29. C Shrutis are ancient treatise on law.

30. D Wakfs are the Muslim religious foundations.

31. C Government cause maximum litigation in Courts.

32. D The first-regular adjudicatory mobile court in the country


has been inaugurated at Punbanna village in Punjab.

33. A Dissolution of Muslim Marriage Act, 1939 provides


grounds of dissolution of Muslim marriage to Spouses.

34. C A married man commits adultery if he has sexual


intercourse with any woman except his wife.

35. D A husband is obliged to maintain his divorced wife till


she gets remarried.

36. D (i) Default = Failure to do something required by law


(ii) Delict = A wrongful act
(iii) Derelict =A thing thrown away by its owner
(iv) Dictum = A judge’s observation.

37. B There are certain Torts where element of ‘intention’ is


relevant while committing the act. For eg: assault, 41. A The Inner Temple called Gandhiji to Bar.
battery, false imprisonment, trespass etc.
42. D A clearly in the present case was under reasonable
38. C India and Britain have signed an ‘Extradition treaty’. apprehension of kidnapping. He exercised private
Extradition means Indian and the U.K. will deport defense for his daughter on seeing an act which to an
criminals on reciprocal basis to each other. ordinary person would considered (as facts of the
case are silent about A’s knowledge of surprise)
39. D Genocide did n’t occurred in Gulf War. kidnapping in progress.

40. C High sea is the open ocean, esp. that not within any 43. C In normal course of business, the same iron was
country’s jurisdiction. As per maritime law, the extent available in the market at Rs. 105 per ton. B , however,
of high sea commences beyond 200 Nautical Miles. bought it for Rs. 110 per ton and suffered a loss of Rs.
Amongst the given options, none of them seems to be 10,000, being the amount more than what he would
correct. Firstly because, as per maritime law, the extent have paid under the contract. As per the principle, the
of territorial waters is always measured in Nautical loses which naturally arose in the usual course of
Miles (NM) but the given options are either in miles or in things would be the loss which was incurred by B over
kilometres. The correct answer ‘would’ have been C and above the market rate at which that iron was

Previous Years
CLAT & AILET Papers Page 93
available in the market at the time when the contract 54. C The word ‘charlatan’ means a ‘cheat or a quack’
was breached. Thus, the difference between the loss
incurred and the market rate is the rightful amount which 55. A ‘To cavil is to raise frivolous objections.
can be claimed by B, i.e., 10,000 – 5,000 = 5,000.
56. C ‘Cynosure’ is the centre or focus of attention.
44. B It is also essential to remember in this regard that the
duty to abide by legal obligations does not affect free 57. A ‘consummate’ as an adjective is used to mean very
consent in any way whatsoever. skilful or perfect.

45. D There is nothing to show that Tony had intended to 58. D ‘Equanimity’ means ‘composure’ and the answer is
cause any harm, or had acted with reckless disregard. hence ‘excitement’
Option (b) is not exactly fitting into the principle, but
you have to take the best from among what is available. 59. A ‘palliate’ as a verb is to reduce or to mitigate.

46. A Undoubtedly, the Bank is liable because the Bank had 60. C ‘Obsequious’ is an adjective menaing ‘servile’ or ‘given
authorized its agent Stephen to collect money. Even to flattery’. The answer would be sharp-tongued.
though on commission basis, Stephen was acting on
behalf of and for the Bank when he collected the 61. D ‘Obstreperous’ means ‘unruly,difficult to control/
amount from Maria. The rule of ‘course of employment’ discipline’ . The word which is most nearly the opposite
is clearly made out in this case. is ‘ weak’.

47. C Even though Vishal’s son finished the portrait, Arun is 62. D
under no obligation to accept it. Vishal was given the
contract because of his own artistic skills which cannot 63. A The best answer is A. as ‘To express surprise at...’ is
be transferred or inherited by his son. Without performing an idiomatic expression.
his part of the contract Vishal died. When his son
completed the portrait himself, he ratified the contract 64. C The best answer is C. We require the preposition
and also all the obligations thereunder. As a result, ‘with’ here as the two components -water and oil -
Vishal’s son must repay the advance paid by Arun. are complementary to each other and one is not given
importance over the other.
48. D The principle in law is ‘ignorantia legis non excusat’
i.e. ignorance of law is no excuse and the same applies 65. A The correct answer is A. only this option uses the
to the given facts. correct sequence of words, especially the placement
of the modifier ‘open’.
49. D Devi is liable because postman was acting in course
of his since the letter was a registered post under a 66. B This is an imperative senrence, where the subject is
duty to handover the letter to Devi. understood. The pronoun ‘you and me’ are both objects
of the verb ‘let’ and should be used in the objective
50. C The principle absolves all the liability of the State even case.
if the act was done in a negligent manner. Therefore,
applying the principle over facts, even if the Police 67. A The best answer is A. The conjunction ‘while ‘ best
Officers were negligent in their act they shall not be brings out the meaning that the person was hit when
held liable. he entered the room.

51. B ‘Frayed’ is to ‘ worn down’ - this is what happens to 68. A This is a classic case of redundancy – ‘back’ is not
‘fabric’ due to use over a period of time. The best required when ‘returned’ has already been used.
answer is therefore ‘dilapidated: building’. ‘crumpled: 69. C We reruire the plural ‘stages’ after ‘various’
paper’ may seem close— nevertheless, when
crumpled paper is not exactly worn down. 70. A The modifier ‘not only’ has been placed incorrectly; it
should have been ‘The bandits not only robbed..’
52. D ‘levitate ‘ is to float/fly up in the air , and is a trick
performed by magicians. In the same way sky jump is 71. A It should have been ‘No less than twenty people(not
a stunt performed by parachutists. persons)’

53. D The sentence tells us what judicial decrees cannot 72. B Part b should have been something like ‘with a fine-
achieve- they cannot change somebody’s heart. The tooth(or fine-toothed) comb’.
second part is about what these decrees can achieve.
In such a context ‘regulate ‘ is the best option as it 73. D The paragraph begins by an introduction about work-
goes along with the idea of preserving law and order. addicts. (N) should follow it as we need to talk first
‘subdue’ is too extreme. about the perspective and ideology of the companies.
Such organization in (P) refer to the organization

Previous Years
Page 94 CLAT & AILET Papers
mentioned in N. Therefore, NP is a mandatory pair. (O) which one does not require the understanding of
follows P as O mentions what happens when work- internal combustion engines. The answer is clearly B.
addicts start working in such organizations. (M) C is also a very close contender but this requires us to
mentions the contrast. Thus, OM is a mandatory pair. call Oscar good mechanic and it is a far-fetched thing
Option (D) is the correct choice. to do so.

74. D The first sentence to follow 1 should be R, the ‘they’ 80. C The claim is that third party fail to succeed in the
in R refers to the animals described in 1 it also tells us elections because they encounter difficulties in
the first thing that the animal is likely to do i,e to get securing space on national ballots. 'Securing space
their teeth into the meat. Q and S would follow this, an national ballots' means proper recognition as a party
telling us what they would do after getting their teeth national level. The evidence provided in the argument
into the meat. Next would be P as it would directly ends up supporting the claim. Though (C) is the correct
precede 6. option, it need to be slightly reframed. The option should
be rephrased as "The evidence provided in the
75. C This is a relatively easier question. Sentence 1 ,which argument instead of refuting / contradicting the claim,
describes what St. Francis taught can be followed ends up supporting it."
only by sentence Q as it begins with the apprpriate
conditional ‘if’ . The options contain only one such 81. B Price of petrol before the hike = Rs. 28 per liter
choice. Furthermore , the sequence RPS makes Price of petrol after the hike = Rs. (1.07 x 28) per liter
complete sense. = Rs. 29.96 per liter
For traveling 2400 kms, total quantity of petrol
76. D The para in the question says how children in France
2400
do stretching and exercise on a daily basis, while consumed = = 133.33 liters
their American counterparts do not do so. French 18
students also outperform their American counterparts Increase in expenditure = Rs.(29.96 – 28) x 133.33 =
in tests to measure cardiovascular fitness. The author Rs. 261.32 = Rs. 262
therefore concludes that the American students can
achieve cardiovascular fitness only if they take up 82. C Total Cost Price = Rs. (1000 + 120 x 9 + 0.60 x 900) =
stretching and exercises. What the author has Rs. 2620
assumed here is that stretching and exercise is the Total Selling Price = Rs. (784 x 2.75) = Rs. 2156
only(or necessary) cause of cardiovascular fitness For having 10% profit, the selling price should be Rs.
and no other factor plays a role in developing (1.1 x 2620) = Rs. 2882
cardiovascular fitness. The answer is clearly D. Sum to be obtained from the advertisements = Rs.
(2882 – 2156) = Rs. 726
77. C The apparent contradiction in the argument is that
patients suffering from disease Q report a slightly 83. A Let the total amount be Rs. 300
high level of blood sugar. But this abnormal condition Amount with A and B is Rs. 100 and Rs. 200
of high blood sugar is considered a means to treat the respectively.
very same disease of which it is a symptom. This Value of the car bought by A after 2 years
contradiction/paradox is best explained in C where it 2
is explained that high blood sugar can stop the   1 
= 100  1 −    = Rs. 73.469
flourishing of the virus and as a consequence one   7 
may even benefit from high level of blood sugar. Value of the money deposited by B after 2 years
= 200 (1 + 0.2)2 = Rs. 288
78. D The argument is something like this— the metabolic Total worth of money with A and B after 2 years = Rs.
rates of professional athletes are found to be 73.469 + Rs. 288 = Rs. 361.469
substantially higher than that of ordinary people. So Percentage increase (approximate)
the speaker infers that a person’s speed and strength = ((361.469 – 300)/300) x 100 = 20%
are primarily determined by his metabolic rate. This
argument is best augmented in D by providing 84. B Consider first and second piece of bronze be x kg
evidence that drugs when they supresss one’s and (60 – x) kg respectively.
metabolic rate also causes one’s speed and strength According to the given condition:
to diminish.
8 10 15 8x − 600 + 10x 3
− = ⇒ =
79. B In the question Oscar was able to repair the 60 − x x 100 x(60 − x) 20
motorcycle despite not knowing what was causing ⇒ 3x2 + 180 x – 12000 = 0 ⇒ x = 40
the problem. It is also known that he has received therefore % age of zinc in in the first piece of bronze
extensive training, can repair most mechanical
problems and also does not understand howinternal 10
= × 100 = 25%
combustion engines work. We can clearly infer from 40
here that there are some mechanical problems, to fix

Previous Years
CLAT & AILET Papers Page 95
85.C Let the ant move for ‘t’ seconds Therefore quantity of the first solution in the 21 liters
Required Difference = (0.5t(2 + (t – 1)8)) – (0.5t(6 + (t 21
– 1)4)) = 4t2 – 3t – (2t2 + t) = 2t2 – 4t of resulting mixture = 3 × = 9 liters
7
Also, 6 < 2t2 – 4t < 30
So, from the options the value of ‘t’ should be 4s. For questions 91 to 94: Going by the information,
combinations BM , CP and MO are not possible .So the possible
86. C Total number of two digit numbers combinations are ABCN , ABCO , ABNO , ABNP , ABOP , BCNO,
8×9 ACMN , ACNO, APMN, APNO
=8+7+6+…+ = 36
2 91. B If player O is selected and player B is rejected then
Total number of three digit numbers Option A and C are straightaway discounted as these
= 28 + 21 + 15 + 10 + 6 + 3 + 1 = 84 consist of combinations MO and CP respectively. Option
Total number of four digit numbers D is not acceptable as it consists of only a single male.
= 56 + 35 + 20 + 10 + 4 + 1 = 126 Only option B does not violate any restriction.
Total number of five digit numbers
= 70 + 35 + 15 + 5 + 1 = 126 92. B explanation same as in Q 91
Total number of six digit numbers
= 56 + 21 + 6 + 1 = 84 93. D All the three statements are false
Total number of seven digit numbers
= 28 + 7 + 1 = 36 94. A From the possible combinations it is clear whenever M
Total number of eight digit numbers plays, A plays but same is not true for other cases.
=8+1=9
Total number of nine digit numbers 95. A
=1
Hence, total required numbers = 502.

87. B Consider the 4 inlet pipes be A, B, C and D


1 00 m
According to given condition :
1 1 1 1 O
+ + = (1) 1 00 m
A B C 12
1 1 1 1
+ + = (2) 96. C X is not even relevant to the question asked as it is
B C D 15 about ‘women being persecuted by their women’. Y
1 1 1 attempts to answer the questiuon but the reasoning
+ = (3)
A D 20 provided- that it will disrupt family life is a very vague
adding the above equations : one. So neither of the answers are reasonable.
 1 1 1 1 1 1 1
2 + + +  = + + 97. B Argument M clearly fails to address the issue raised
 A B C D  12 15 20 in the question. N answer the question perfectly.
1 1 1 1 1
therefore , + + + = 98. C The island of Bombay was acquired by the East India
A B C D 10
ie. The four inlets can fill the dam in 10 min. Company from the Portugal.

99. B Todarmal was the Diwan-I- Ashraf or the revenue or


88. C The two digit numbers that gives remainder 3 when
Finance Minister in Akbar’s Darbar of the Mughal empire
divided by 7 are
(one of the ‘Navratnas’ of Akbar’s court). Akbar
17, 24, 31, 38 ............ 94 , which is an A.P. with
introduced fresh reforms in the land-revenue system
a = 17, d = 7 and l = 94
with his assistance.
12
therefore Sn = {17 + 94} = 666
2 100. A Indra is the chief gods described in Rigveda under
Hindu Mythology.
89. C Distance travelled with speed 20 m/s in the first 10
min. = 12 km 101. C Nadir Shah took the Peacock throne of Shahjahan.
Total distance travelled = (12 + 8.5 + 11 + 8.5 + 6) = 46
102. B The council of eight ministers of Chhtrapati Shivaji
km
were known as “Ashta Pradhans”.
Total Time = (10 + 10 + 10 + 10 + 10) = 50 min
Therefore Average speed = 46 x 60 /50 = 55.2 km/h 103. B Charak was the court physician of Kanishka. Charak
wrote the Charaksamhita in the second century A.D
90. C By aligation : The ratio in which two solutions are
104. D* 105. A 106. C 107. C
97 − 94 3
mixed = = ie . 3 : 4
94 − 90 4 108. C* 109. B 110. D

Previous Years
Page 96 CLAT & AILET Papers
111. B The Hindustan-Tibet road connects Shimla with 130. C Crocodile is a reptile with a four-chambered heart.
Gangtok. The Hindustan-Tibet road, which passes
through Himachal Pradesh, is a 300-mile (480km) 131. D Rasna is the Ahmedabad-based Pioma Industries.
highway which runs through Shimla, once the summer Recently, the rasna girl Taruni Sachdeva died in a plan
capital of India, and crosses the Indo-Tibetan border crash in Nepal.
near Shipki Pass.
132. A Brett Lee acted as a model for Timex.
112. D The Aswan Damis an embankment dam situated
across the Nile River in Aswan, Egypt. 133. A 134. B 135. D 136. D

113. A Chandrasekher limit is the limit beyond which stars 137. C Phishing refers to the act of fraudulent way of
suffer internal collapse. acquiring PIN and bank passwords using e-mail.
114. A Tetraethyl lead is the best anti-knock compound used
138. A
in petrol to increase mileage.
139. C*
115. D Tetraethyl lead is the compound which is used in petrol
to increase mileage.
140. B T-20 has been included in Asian Games since the
116. C Rainbow is produced because of the combine year 2010.
phenomena of dispersion, refraction and internal 141. D Shri. E. Sridharan is the incumbent Managing Director
reflexation. of Delhi Metro Railway Corporation.

117. C Chemical bomb will kill only the inhabitant in use in the 142. D Nepal in the year 2008 adopted a new National Anthem.
city.
143. D
118. B Oxalic acid solution is used to remove rust stains on
cloth. 143. D*

119. C “Mach Number” is a term associated with the speed of 144. D Recently, Western Ghats from India has been included
Aeroplanes. into UNESCO’s World natural heritage site.

120. B If an object is placed midway between two parallel 145. A


plane mirrors facing each other, then the number of
images that will appear in mirrors will be Infinite. 146. D The answer is clearly D . the author begins by stating
what Asimov believes in and mentions in the same
121. C It is due to Surface tension of rainwater that gives breath that science itself was questioning whatever
raindrop a spherical shape. he supported in his time. This is roughly what the Ist
paragraph does. The remaining part of the passage is
122.* A pure diamond is colorless. an elucidation of this point.

123. B Geiger counter is an instrument to detect radioactive 147. C In the last paragraph, the author reveals Asimov’s
radiation. beliefs as indicated through the FOUNDATION serries
. He also mentions several fields ( like Physics , biology
124. D The film of oil and soapy water owe their brilliant and Maths.) where the same beliefs are increasingly
colours to a combination of light reflection and being proven wrong.
Refraction.
148. B The answer can be easily gleaned from the first five
125. B When a ship enters a sea from a river, its portion lines of the passage.
under water will decrease.
149. D The author clearly indicates that the Seldon Plan ,though
126. A For digestion of food, hydrochloric acid is secreted naive it may sound, clearly reflects Isaac Asimov’s
into stomach at a pH value of 2. attitude towards science. The last sentence of the 2nd
paragraph strongly points to this.
127. C Color blindness is a disease, which is genetically linked.
150. A ‘The butterfly effect’ is explained in the passage as a
128. B Astigmatism is the disease in which one cannot ‘chaos theory’ which ‘showed that perfect prediction
distinguish between vertical and horizontal lines. could take place only on the basis of perfect
information, which was by nature impossible to obtain’.
129. C Spleen is the gland, which is attached to the digestive This idea is best captured in the example provided in
system but does not have any role to play in digesting Option A.
food.

Previous Years
CLAT & AILET Papers Page 97
AILET Solutions 2009
1. B ‘An advocate’ meaning a person who publicly supports 15. D ‘Pitch’ camp means to set up a campsite.
or recommends a particular cause or policy is the best
word to complete the sentence. 16. B The correct adjective for clouds here is ‘gathering’.

2. A Just as ‘negotiable’ is a quality of cheques, ‘frozen’ is 17. A ‘Preceded’ meaning to come before something is the
a quality of ‘asset’. logical word to fill in the blank.

3. C A ‘hedger’ is a gardener who takes care of and trims 18. A The passage is in simple past tense, thus, ‘arrived’ is
the shrubbery. Similarly, a whittler the correct choice.

4. D ‘Honour’ is used to address a ‘Governor’. Likewise, 19. C ‘Promising’ meaning ‘likely to develop in a desirable
‘grace’ is used to address a ‘duke’. manner’ is the correct word to fill in the blank.

5. B Q follows 1 because it tells what Mrs. Bates started 20. C Expanse is the correct choice. It means a wide and
doing after coming down. This is followed by P; open extent, as of surface, land or sky.
meantime refers to ‘some time’ for which she was
sewing and stitching. SR is a mandatory pair which 21. A ‘Gave’ is the correct word. Shelter is always given,
has a sequence of events (‘rose suddenly’ must be not supplied, afforded or cast.
followed by ‘rushed to the stair door’). Thus, option
(B) is the correct choice. 22. C ‘At hand’ meaning near in time or place or within reach
is the correct phrase.
6. B P must follow 1. This is followed by S and R which
form a mandatory pair describing the ‘present age of 23. D ‘grove’ of trees,’ meaning a group of trees, is the correct
light reading’. Q has to precede 6 since 6 mentions usage.
where something heavier is cast (into the midst of the
reading public). 24. C ‘Abounded’ is the past tense of ‘abound’ which means
to exist in large numbers.
7. D ‘Ductile’ means easily influenced. So ‘stubborn’ is an
opposite of ‘ductile’. 25. B Since each member was given a task, ‘was allotted’ is
the correct choice. ‘Allotted’, ‘had allotted’ and ‘has
8. D ‘Galvanize’ means to arouse or stimulate awareness allotted’ are incorrect because they give the sense
or action. ‘Dampen’ meaning to diminish the activity or that the members gave the tasks to someone else
vigor ofsomething is the best antonym for it. whereas the passage states that they were given the
tasks.
9. C Obsequious is being obedient or compliant. ‘Servile’ is
the correct word which means the same. 26. C ‘Attended’ must be used to maintain consistency of
tense in the sentence.
10. A ‘Dialectic’ is the art or practice of arriving at the truth
by the exchange of logical arguments. ’Argumentative’ 27. D ‘Lay’ is used both in past and present tense but its
is the correct word. meaning is different in both the cases. In present tense
’lay’ needs an object to it, as in “I lay the book on the
11. A The modifier ‘not only’ must be placed after ‘denied’. table” which means to put something down .Its past
The sentence talks of ‘he’ denying two things- not tense is ‘laid’, as in “I laid the book on the table”. ‘Lay’
only having borrowed money but also having ever is also used as past tense of ‘lie’ meaning to recline.
met me. “He lay down early for sleep yesterday”. Here, ‘lay’,
the past tense of ‘lie’, is the correct choice since the
12. C ‘Their’ is an incorrect pronoun for ‘party’. The correct passage is in simple past tense.
pronoun should be ‘its’.
28. B Due to the phenomena of resonance at a particular
13. B The sentence talks of something that is/cannot be speed, if attained by a bus its starts vibrating violent.
true. ‘Would’ in place of ‘should’ must be used to talk of
ideas like hypothesis. 29. D A built fired in the sky gains potential energy.

14. C ‘In the’ is the correct usage, since that is time when 30. A
the decision was taken.
31. D

Previous Years
Page 98 CLAT & AILET Papers
32. D The principle of independent assortment is given by 50. D Introduction of separate communal electorate for
Mendal. Muslims was not one of the features of the principal
features of Government of India Act, 1919.
33. A Balban was the first Sultan of Delhi to introduce the
practice of ‘Sijda. 51. A There are currently 448. Articles and 12 Schedules in
the Constitution of India.
34. C The Hunter Commission published its detailed report in
1884 with its main focus was to explain the failure of 52. B The Indian National Congress asserted in the year
Charles Wood’s Education Dispatch of 1854 and to 1936 that India would not accept any constitution made
recommend reform. The principal objective of Wood’s by anyone other then people of India and without
Dispatch was to spread government and mission outside interference.
education to the broader population in India.
53. C Hindu Mahasabha did not participated and contributed
35. A Sir Charles Wood’s Dispatch for the first time in India it’s share in the formation of Indian Constitution.
setup a Public Works Department.
54. A The expenditure from the Consolidated Fund of India
36.C Dadabhai Naoroji propounded the theory of economic
for which the approval of Parliament is not necessary,
drain. He also authored a book by name of Poverty
according to the Constitution of India, is called charged
and un-Britrish rule in India.
Expenditure. (Article 112 of the Constitution of India.)
37.A Non-Cooperation Movement was started pursuant to
the resolution of Calcutta assembly in September 1920. 55. D Proclamation of Emergency on the ground of internal
disturbance was for the first time made in year 1975.
38. A Sir Charles Wood’s Dispatch for the first time in India In 1975, Fakhruddin Ali Ahmed (then The President of
setup a Public Works Department. India) was advised by Prime Minister Indra Gandhi to
declare such a sate of emergency under Article 352,
39. C Contour bunding is used to prevent soil erosion in hilly which empowers such state to declare on ground of
areas. internal disturbance.

40. D 56. D Supporting Indian cause at various national and


international level is not a criteria for acquiring
41. B citizenship either in Citizenship Act 1955 of under Part
II of the Constitution of India.
42. B Aphelion happens on December 23rd.
57. B The Constitution originally provided for the right to
43. A Gilt Edged Market is the market of government securities. property under Articles 19 and 31. Article 19
These securities are No risk securities. guaranteed to all citizens the right to acquire, hold and
dispose of property. The Forty-Forth Amendment of
44. B Laffer Effect states “Reduction in rate of taxation leads 1978 deleted the right to property from the list of
to more than proportionate increase in tax yield”. The fundamental rights. A new provision, Article 300-A,
aforementioned is the long-term affect described by was added to the constitution which rendered it merely
the theory and short-term affect “arithmetic affect” a constitutional right and not a fundamental right.
leads to shortage of government revenue.
58. D The Right of Children to Free and Compulsory Education
45. A VDIS-Voluntary Disclosure of Income Scheme was Act or Right to Education Act (RTE), which was
brainchild of P.Chidambaram. passed by the Indian parliament on 4 August 2009,
describes the modalities of the provision of free and
46. A A company is said to be ‘Sick’ when the accumulated compulsory education for children between 6 and 14
loss at the end of any financial year leads to erosion in India under Article 21A of the Indian Constitution.
of 50 % percent of its net wealth during the immediately India became one of 135 countries to make education
preceding four financial years. a fundamental right of every child when the act came
into force on 1 April 2010.
47. A Gunnar Myradal has dealt with the problem of poverty
in Asian countries in her book ASIAN DRAMA . 59. B In case of a financial emergency under Article 360,
the President can reduce the salaries of all government
48. B Wealth tax on agricultural property is levied by state officials, including judges of the Supreme Court and
government because it is the subject of state list under High Courts. All money bills passed by the State
the seventh schedule of constitution of India. legislatures are submitted to the President for his
approval. He can direct the state to observe certain
49. D Simon Commission’s recommendations did not principles (economy measures) relating to financial
mentioned that Indian Council is not needed to advice matters. He can not suspend the fundamental rights
the Secretary of State for India. of the citizens.

Previous Years
CLAT & AILET Papers Page 99
60. A Criminal Procedure Code mandates that if a death 71. C Gramophone disc once played shall address to ear,
sentence is passed by a Court of Session (Sessions hence it will constitute slander.
Judge), it must essentially be confirmed by the
concerned High Court. 72. A A partial protection against double jeopardy is a
Fundamental Right guaranteed under Article 20 (2) of
61. B Motions of no confidence against the government can the Constitution of India. This states that “No person
only be introduced and passed in the Lok Sabha. If shall be prosecuted and punished for the same
passed by a majority vote, the Prime Minister and the offence more than once”. This provision enshrines
Council of Ministers resigns collectively. The Rajya the concept of autrefois convict, that no one convicted
Sabha has no power over such a motion and hence of an offence can be tried or punished a second time.
no real power over the executive. Thus, both the
executive and the legislature in India have checks and 73. d The doctrine of colourability is the idea that when the
balances over each other. legislature wants to do something that it cannot do
within the constraints of the constitution, it colours the
62. B The decision of a High Court is not binding on other law with a substitute purpose which will still allow it to
High Courts. A judgment passed by the High Court of accomplish its original goal.
one state has only persuasive value for other states.
74. C Smt. Indira Nehru Gandhi vs Shri Raj Narain And Anr.
63. B
Decided on 7 November, 1975.
64. A Peoples Union for Democratic Rights v. Union of India:
[AIR 1982 SC 1473] Also known as the Asiad Workers 75. C Shah Commission was a commission of inquiry
case. The Supreme Court held that non payment of appointed by Government of India in 1977 to inquire
minimum wages is a type of forced labour. into all the excesses committed in the Indian Emergency
(1975 - 77). It was headed by Justice J.C. Shah, a
65. C Section 4 of the Indian Contract Act 1872 states that former chief Justice of India.
the communication of an acceptance is complete, as
against the proposer, when it is put in a course of 76. A Section 25 of Negotiable Instrument Act 1881, the
transmission to him so as to be out of the power of the expression” public holiday” includes Sundays and any
acceptor and; as against the acceptor, when it comes other day declared by the Central Government, by
to the knowledge of the proposer. notification in the Official Gazette, to be a public holiday.

66. C A contract is said to be induced by “undue influence” 77. B The minimum number of partners must be two, while
where the relations subsisting between the parties the maximum number can be 10 in case of banking
are such that one of the parties is in a position to business and 20 in all other types of business.
dominate the will of the other and uses that position to
obtain an unfair advantage over the other. Ramesh 78. B In Islam, iddah or iddat is the period of waiting which a
being the master in this case there exists all possibility woman must observe after the death of her spouse
that undue influence may be exercised on his part. or after a divorce, during which she may not marry
another man.
67. A When someone displays an article indicating its price,
law of contract considers it to be an invitation to offer 79. C The term approver is neither defined nor used in the
and not offer. In simple terms, the person invites others Criminal Procedure Code, but is usually applied to a
to offer which he may or may not accept. person, supposed to be directly or indirectly concerned
in an offence and to whom a pardon is granted by the
68. A When someone displays an article indicating its price, court with a view to securing his testimony/statement
law of contract considers it to be an invitation to offer against other persons guilty of the offence.
and not offer. In simple terms, the person invites others
to offer which he may or may not accept. 80. D Under Section 140 of Motor Vehicle Act, the minimum
compensation to be awarded in death cases id
69. B A contract to do an act which, after the contract is Rs.50,000/
made, becomes impossible, or, by reason of some
event which the promisor could not prevent, unlawful, 81. C Battery is the tort of intentionally and voluntarily bringing
becomes void when the act becomes impossible or about an unconsented harmful or offensive contact
unlawful. with a person or to something closely associated with
them. Unlike assault, battery involves an actual contact.
70. C Once approved by the Parliament, the emergency
remains in force for a period of six months from the 82. B Malfeasance means committing an unlawful act;
date of proclamation. In case it is to be extended beyond Malingerer means one who falsely pretends to be
six months, another prior resolution has to be passed sick; Misdemeanor constitutes minor offences and
by the Parliament. In this way, such emergency Misfeasance means improper performance of duty.
continues indefinitely.

Previous Years
Page 100 CLAT & AILET Papers
83. D Correct answer is D and not A The Supreme Court of Z – 2 = X, X – 2 = V, V – 2 = T, T – 2 = R
India comprises the Chief Justice and 30 other Judges There is some error in the question. As no option is
appointed by the President of India. Supreme Court matching. But, we know that answer is between b
Judges retire upon attaining the age of 65 years. and d. Then the best possible answer is option (D).

84. C Cutting wood being the sole source of livelihood for 92. ‘Privity of contract’ means a contract can’t confer rights
Gangaram, the government’s action is not justified. of obligation to any person except the parties to it.
His wood cutting can be regulated in a reasonable
manner but it can not be absolutely restrained. 93. C A finder of a thing does n’t becomes its owner.

85. D Section 299 of IPC defines Culpable Homicide as who 94. A minor is an individual under the age of 18 years.
ever causes death by doing an act with the intention
of causing death, or with the intention of causing such 95. C A right’s definition does n’t include of element of
bodily injury as is likely to cause death, or with the advantage.
knowledge that he is likely by such act to cause death,
commits the offence of culpable homicide. 96. B Although the custom represents common conscience
but this feature of custom has no bearing of it being
86. D As per Section 361 of IPC, whoever takes or entices ‘force of law’.
any minor under 1[sixteen] years of age if a male, or
under [eighteen] years of age if a female, or any 97. B Only argument (b) is strong since it talks of a
person of unsound mind, out of the keeping of the consequence which, if comes true, would greatly
lawful guardian of such minor or person of unsound affect the nation. Argument (a) is not strong enough a
mind, without the consent of such guardian, is said to reason to do away with the parliamentary democracy
kidnap such minor or person from lawful guardianship since disappointing past experience can be attributed
hence none of the given options would be taken as a to poor implementation of the same. It does not call for
defence of kidnapping. changing the system entirely. Thus, option (B) is the
correct choice.
87. D Laloo can not be held liable as he had taken the due
care by putting up a caution board. Law does not 98. C If we examine both the statements, they are true
prohibit tolerance towards customers or visitors, it independently depending on the individual situations.
only mandates duty of care which was duly complied Hence, option (C) is correct.
by Laloo.
99. A The passage talks of top men depending upon popular
88. D 2, 6, 14, 11, 15, 23, 20, 24, ?, 29 support to exercise their power. This, in a way, means
The logic here is that: that their actions are closely watched by the public
(32) and so they cannot behave as they like. Thus,
2 6 1 4 11 15 23 20 24 ? 29 conclusion (a) definitely follows. Conclusion (b) is a
far-fetched statement and thus, does not follow.
+ 4 + 8 –3 +4 + 8 –3 +4 +8 – 3
100. A I am in West direction with respect to my house.
89. C There is no certain logic for this question. But assuming
that the question is true, then among the best possible 200 m
answer it should be 58.
100 m
97 86 73 58 45 34 200 m
O
– 11 –1 3 –1 5 – 13 – 11 100 m

100 m
90. C* MOQ, SUM, YAC, ?
There is some error in the question at it should be ‘W’ For questions 101 and 102:
in place of ‘M’ in second term. All of the given information can be represented as
The logic here is that:
B lue
M + 6 = S, S + 6 = Y, Y + 6 = E(5)
O + 6 = U, U + 6 = A, A + 6 = G(7) B lack B ro w n
Q + 6 = W, W + 6 = C, C + 6 = I(9)
Hence, best possible answer is option (C). W h ite
G re en

91. D* KWZ, MOX, OIV, QET, ?


The logic here is that- Red
K + 2 = M, M + 2 = O, O + 2 = Q, Q + 2 = S
W – 8 = O, O – 6 = I, I – 4 = E, E – 2 = C 101. C The side opposite to ‘Brown’ is ‘White’.

Previous Years
CLAT & AILET Papers Page 101
102. C Four colors adjacent to ‘Green’ are ‘White’, ‘Black’ 113. C Hansom, Victoria and baroque are all horse-drawn
‘Brown’ and ‘Red’. carriages that were used in olden times in Europe.
Growler is a person or thing that growls. Thus, it does
103. B All of the given information can be represented as- not belong to the group.

S ister B ro th er 114. C The argument states that businesses maximize their


M K L
profits through advertisements which generally carry
inaccurate information. Thus, the consumers should
S ister be wary of such advertisements. Only option (C) does
not strengthen this argument since it states that
N consumers have a cynical attitude towards
advertisements. This statement does not affect the
∴ K is brother of M. argument at all and hence is the correct choice. Option
(A), (B) and (D) all add to the consumer advocate’s
104. A All of the given information can be represented as: argument by substantiating his reasoning. Option (A)
and (B) both talk about businesses knowing that
inaccurate information will help them make money and
Sh using that knowledge too. Option (D) talks of
Z Am
advertisement makers also contributing to inaccurate
Son information in the advertisements.

115. B The science columnist concludes that humans and


At A lka M oh cats have many similar diseases because of the
similarity in their genetic makeup. He states that since
many diseases in humans are genetic and humans
∴ Alka is the sister of Mohan. and cats have similar genes, thus they also have
common diseases. This is weakened by option (B)
For questions 105 to 109: If we arrange all of the given because it states that the diseases that humans and
information, then there are a total of 7 conditions. cats have in common are not genetic in nature. Thus,
From condition 2, we can say that A is not Manager. Using it negates the science columnist’s reasoning of humans
condition 1, 2, 4 and 5 we can say that B is the husband of C and cats having similar diseases because of their
and she is the sister of E. Now, using condition 6, we can say genetic similarity.
that E is manager. Thus, we can conclude that A earns more
than E, Manager. Using condition 1, we can conclude that B is 116. B The author of the passage links anger to permanent
accountant and A is supplier. The final arrangement would look High B.P to heart diseases. He assumes a causal
like this: relationship between anger and High B.P. Option B
makes this assumption incorrect as those with
Manager Accountant Supplier permanent high blood pressure will have to take
A × × √ Male medication for the same and this medication will make
them quick to anger (mood swings). Hence, in light of
B × √ × Male
option B, being quick to anger is an effect of permanent
C × × × Female high B.P and not the other way round. This means that
Female both psychological factors and heart diseases are a
D × × × result of permanent high blood pressure.
(unmarried)
E √ × × Male 117. A The professor arrived at the conclusion that computers
have not made books obsolete because of a majority
105. B 106. A 107. C 108. A 109. D of her students printing the material instead of reading
it on the computer screen. This argument is
110. D Third letter = N, fourth letter = T and eleventh letter = I. strengthened by option (A) because it echoes the
Hence, the meaningful word is TIN. same sentiments as of the professor. It has been found
by several colleagues of the same professor that their
111. B From statements (a) and (b), ‘student’ is coded as students also behave similarly. The rest of the options
‘din’. give different reasons for the conclusion which limits
From statements (b) and (d), ‘is’ is coded as ‘dink’. the scope of the professor’s assertion. Thus, they
Hence, ‘Arjuna’ is coded as ‘sunk’. cannot strengthen the argument.

112. A Saffron, pepper and lard are all spices. Cheese is the
odd one out.

Previous Years
Page 102 CLAT & AILET Papers
118. B Option (A) is not true as it violates condition 2.
15
Option (C) is not true as it violates condition 1. 126. C CP of 5 articles = = Rs.12.5
Option (D) is not true as it violates condition 3. 1.2
Thus, option (B) is correct as it satisfies all the
12.5
condition. ∴ CP of 1 article = = Rs.2.5
5
119. A Based on the given conditions the following ∴ CP of 8 such articles = 8 × 2.5 = Rs.20
combinations are possible: ∴ SP of 8 such articles = Rs.18.40
1. J R S K T H ∴ Loss = Rs.20 – Rs.18.40 = Rs.1.60
2. J S R K T H
1.60
3. J R S T K H Hence, loss percentage = × 100 = 8%.
4. J S K R T H 20
5. J R T K S H
6. J R T K H S 127. D The answer to the question is “cannot be determined”.
7. J S K T R H As no information is given regarding one of the years.
Thus, H cannot be fourth lowest in cost.
128. C Let the number of school going children and non-school
120. B Based on the above given combinations, we can say going children be 5x and 4x respectively.
that J cannot be the second lowest in cost. New number of non-school going children = 4x × 1.2
5x 25
For questions 121 and 122: Using the conditions we can Hence, new ratio = = or 25 : 24.
infer that exactly two confess after T which leaves us with 5,
4x × 1.2 24
6 and 7. Also using the condition 5 and 6 we can say that W
can not be placed before T. Now following combinations are 129. A Let the speed of Anthony be S km/hr and t be the time
possible. taken to over took Akbar.
6
⇒ =t ...(i)
1 Z Z Y Y S − 12
2 X V Z Z
7.5 3
3 T T T T and =t+ ...(ii)
S − 15 2
4 V X X V ×C
On solving equation (i) and (ii), we get
5 Y Y V X C S = 10 or 18 (but Anthony’s speed must be greater
6 W W W W C than that of Amar)
Hence, S = 18 km/hr.
7 S S S S ×C

121. B 130. A Required number of ways = All of them are arranged


– when 3 girls are arranged together
122. D = 8! – 6! × 3! = 36000.

123. C LCM of (16, 18, 20) = 720 5


c 2 ×7 c1 7 .
Required number = 720 + 4 = 724 131. A Required probability = =
12 22
Series of such number would be n × 720 + 4, c3
where n is a natural number. 132. C Ratio of time taken by Ajit, Baljit and Daljit = 8 : 12 : 15
Among the given options, 2884 satisfies the condition = 8x : 12x : 15x
2884 = 4 × 720 + 4 and is divisible by 7 too. According to the given condition,
124. D Total age of three boys = 3 × 15 = 45 years
1 1 1 1
Age of youngest boy ⇒ + + =
8x 12x 15x 20
3 3
= × 45 = × 45 = 9 years.
3+5+7 15 15 + 10 + 8 1 33 1
⇒ = ⇒ =
120x 20 120x 20
125. C Let A’s salary be Rs.A and B’s salary be Rs.B
11
Final salary of A = A × 1.25 × 0.8 ⇒x =
Final salary of B = B × 1.20 × 0.75 2
3 Result is same. Hence, time taken by Baljit alone to complete the work
⇒ A × 1.25 × 0.80 = B × 1.20 × 0.75 11
= 12 × = 66 days.
B 1.25 × 0.80 10 2
⇒ = = or 10 : 9
A 1.20 × 0.75 9

Previous Years
CLAT & AILET Papers Page 103
133. A 146. A Options (B), (C) and (D) are incorrect as they are
beyond the scope of information given in the passage.
134. D ‘Counter-intuitive’ is something that does not seem to
be true when assessed using insitition or common
135. A sense. Only option (A) communicates the intention of
the author correctly.
136. B
147. B The third paragraph highlights that people have a moral
137. C obligation to care for one another, which they might
even discharge in their effort to support those in need.
138. A This is best expressed in the scenario given in option
(B) where the executive quits her job (discharged her
139. C moral obligation to help others) learning of the harmful
effects of the chemicals. Thus, option B is the correct
140. B choice.

141. B 148. C Refer to the line, “Moreover, philosophical


anarchists...to help those in need.” It is clear from this
142. B statement that people will support the laws and ef-
forts of the government which are directed at helping
143. D The passage mainly talks about the viewpoint of those in need because of their inherent moral obliga-
philosophical anarchism which states that tion to care for others. Thus, the author must agree to
governments do not have a moral right to govern, but the statement given in option (C).
still people have non-legal moral duties to not harm
other individuals. This point is best captured in option 149. D Options (A), (B) and (C) are beyond the scope of the
D. Option A is incorrect as it is not the main point of the information of the question. Fourth paragraph first two
passage. The author has elaborated on how lines clearly state that individuals should respect
philosophical anarchism is not counter-intuitive. Option individual freedom. This makes option (D) the correct
B and C are individually incorrect as they are only one choice.
of the claims of the commentators that the author has
clarified. 150. D The author in the last two paragraphs is trying to
clarify the position of philosophical anarchism. Author
144. A Refer to the line, “It is also commonly is also trying to answer the critics on the issue by
supposed...simply because it is the law.” Thus, the clarifying the doubts raised by the critics. Hence, (D)
author has stated that following the laws is considered is the correct answer.
a moral obligation because of it being a law. Option (A)
follows.

145. B In the last two paragraphs, the author is trying to


clarify the ideas of philosophical anarchism. This implies
that the author is in consonance with the ideas of the
theory. This is highlighted in option (B). Option (A) is
incorrect because of the word ‘ardent’ which makes
the author biased towards the theory, which is clearly
not the case. Options (C)and (D) are incorrect as the
author is neither negative about the theory nor is he
rejecting it.

Previous Years
Page 104 CLAT & AILET Papers
AILET Solutions 2010

1. D In the second paragraph, the author states, "That the 6. A Encumbrance is defined as a 'thing that impedes or is
artist can afford to be "purist" … about what the art burdensome'. Hence, Encumbrance and Burden are
world or anyone else might think…" hence justifying synonyms. Out of the options given, the only that
the correctness of option (D). contains a pair that represents a similar relationship is
option (A). Mnemonic is defined as 'of or relating to
2. D The author has mentioned in the passage that in order memory or mnemonics' and hence, Mnemonic
to sell one’s work in the market, one has to price it for represents a similar relationship with Memory like the
an amount that someone would be likely to pay for it. In one represented by the pair Encumbrance and Burden.
other words, one cannot price it arbitrarily but must do
so according to its market worth. Thus, (b) definitely 7. A Subsequent and previous are antonyms of one
follows. (a) is the ‘purist’s vision’ where the artist another. The only pair in the given options that
sees pricing his art as debasing it. (c) is incorrect as represent a similar relationship is Significant and
one need not compromise one one’s estimation of one’s Inconsequential. Hence option (A) is correct.
own art always. Renowned artists often manage to
get the price that they demand for their works. (b) 8. B The word 'precipitate' is used in the passage in the
does not follow as pricing does not involve subjecting line "… the product of your two-minute moment
the art to buyer’s interpretation. Thus, option (D), only precipitates a major transformational turning point in
(b), is correct. your career…" Precipitate here is undoubtedly used
in the context as something that 'triggers' a
3. D For an artist to sell his piece of art at a higher price transformational change in the author/his art.
range, "the art world will have to recognize your art
both critically and from the marketing standpoints, and 9. B Whenever, we use 'I wish', past subjunctive must
you'll have to successfully produce, show, and see follow(here in this case 'spoke' not 'speak').
for many years" (paragraph 5).
10. D ‘Audience’ is a collective noun which is considered
4. D The author, in paragraph 5, states, "… drawing will singular or plural depending on the context in which it
hang framed and captioned as the first inspiration for is used. When it refers to the group of people
all subsequent work." Option (B), though close, is comprising the audience as one entity, as a whole,
incorrect since the first drawing according to the then it is considered as singular. For example- “The
author, will be a "historically important document of audience is enjoying the performance.” However,
your career" and hence would be a collectible piece in when we talk of people in the audience as separate
the artist's repertoire, maybe for a sentimental value, individuals, it is considered as a plural noun. In the
but nowhere does the author mention that this piece given sentence, audience refers to separate
would also be the artist's most important piece. Similarly, individuals who are requested to take their respective
options (A) and (D) are neither mentioned nor implied seats. Thus, here ‘audience’ is a plural noun that must
in the passage. take a plural verb ‘are’.

5. C In the last paragraph, the author states "Unless that


11. C Since the event of 'raining' has happened before that
artist leaves specific instructions… it (the art) will
of 'wall collapsed', past perfect must be used.
become subject to those market forces that the artist
strived for a lifetime to avoid." So statement (B) is
12. A The subject in the given sentence is 'Each member'
correct. Statement (A) on the other hand, is incorrect
which is singular, so singular verb must be used.
since the author claims that (only) when an artist
becomes very successful and holds a retrospective
13. B Since 'The Prime Minister' is singular, 'have' can't be
exhibition and has his originally over-priced painting
used, and of the other given options, only 'is' fits in the
"a serious collector may well be willing to pay an
sentence.
extraordinary price to own this historically important
document of your (the artist's) career". Thus (A) is
14. B Among the given option, only option (B) best fills the
incorrect as the price that the collector will be willing
blank. (A) is incorrect as we can’t predict beforehand
to pay may well be higher than what the artist thought.
that the volcano is going to erupt. Thus, (A) makes the
Option (C) is out of scope of the passage. Moreover,
sentence logically incorrect. Option (C) doesn’t fit in
‘purists’ do not wish to sell their work so they would
grammatically. (D) is incorrect since we can’t make
not take the ‘realities of the market place’ approach.
such a definitive statement (has to erupt) about a
Hence, the answer is statement (B) alone.
volcano. (B) logically and grammatically completes the
sentence and thus, is the right choice.

Previous Years
CLAT & AILET Papers Page 105
15. B Options (A) and (C) are wrong because they don't 33. B Amenable means ready or willing to answer,
agree with the singular subject. 'The Princess' 'will responsive.
have been staying' is wrong tense. Only option (B) is
correct. 34. C To pay through nose means to pay too much for
something.
16. C An action that started in the past is still going on, so
present perfect continuous must be used. 35. C To give someone a piece of mind means to speak
angrily to someone because they have done something
17. A Sentence started with the past tense. Simple past wrong.
must follow.
36. D Incumbent President of the United States, Mr. Barrak
18. A Sentence talks about a past event. Simple past must Obama was awarded Nobel Peace Prize in 2009. He
follow. is the also holds the first rank in the list of world s most
powerful Leaders by Forbes.
19. B Correct form of adverb 'quickly' should be used and
the sentence should have same parallel structure, 37. A Sheik Hasina is the current president of the Bangladesh.
hence option (B) 'finish quickly' is the right option.
38. B The Incumbent President of South Africa is Jacob
20. A 'They' appreciate your action of calling, not 'you', hence Zuma.
option (A) is correct. Moreover, an object should follow
the verb ‘appreciate’. Only the phrase in (A) can be 39. D Roh Moo-Hyun who was Ex- President of South Korea
used as a noun phrase, hence as an object. committed suicide.

21. A 'look after' is the correct idiomatic expression. It means 40. D Dev Patel the protagonist in the world famous Indian
to take care of. movie Slumdog Millionaire has not won the Oscar.

22. D 'bring out' is the correct idiomatic expression. It means 41. B Mr. V. S. Sampath is the incumbent Chief election
to reveal/expose. Commissioner of India.

23. D 'eight century old' is used as an adjective to the noun 42. A In the year 2010, 76 CRPF jawans were killed by the
'temple', hence it is not used as plural. Thus, ‘eight Maoist in an ambush near Dantewada in Chattisgarh.
centuries old’ will not be used.
43. C Garima is the name of the cloned buffalo calf created
24. A Two simple sentences are joined by a conjunction; so by The National Dairy Research Insdtute in Haryana.
they must be in parallel form. 'is not' should be followed
by 'is not either'. 44. D Ex- Chief Justice of India Hon’ble Justice Lahoti did not
became the chairperson of National Human Right
25. D Two different parts of the sentence (before and after Commission.
the blank) must be joined by a conjunction (in this case
because). So (C) and (D) are eliminated. 'because 45. B President Omar al-Bashir of Sudan was the first head
they different concepts' doesn't make sense; it should of state to be charged with ‘War crimes and crimes
be 'because they differ about..', hence (A) is wrong. against humanity’ by the International Criminal Court.
Only option (D) is correct.
46. D M.S. Gill was appointed in 2010 as a Union Minister.
26. B Queer means deviating from normal or expected.
Absurd is the closest synonym. 47. A In 1912, the imperial capital was officially shifted from
Calcutta to Delhi to mark the entry of the Governor-
27. C Flout means to disregard, show disdain. Defy is the general of India into the new Capital.
closest synonym.
48. C In the year 1915 Shreemati Nathibai Damodar
28. D Solidarity means unity. Thackersey (SNDT) the first Women’s university in
India was established.
29. A Impeccable means flawless, faultless.
49. B In Schedule VIII to the Constitution of India 22 official
30. C Vigilant means keenly watchful to detect danger. languages have been listed. Recently there is a motion
in the parliament for inclusion of one more language
31. D Sanguine means cheerfully optimistic. “bhojpuri” into that list.

32. C Scurrilous means grossly or obscenely abusive,


insulting.

Previous Years
Page 106 CLAT & AILET Papers
50. A Justice Punnchi headed the committee on Centre-State 68. B Srimavo Bandaranaike was the first woman Prime
Relations in India. Minister.

51. B In 1919, Women were allowed first time in India to 69. A Russia is the largest country in terms of territory. India
contest election and to hold public offices. is the 7th Largest in the List.

52. B Justice Sri Krishna headed Telangana Committee. 70. A Cloud Computing is a Internet based computing model.
Cloud Computing has No sole Inventor.
53. C Justice V.S. Malimath headed committee on reforming
Criminal Justice System. 71. D The correct option is (d) as in the abovementioned
facts B’ never had the intention to hurt his friend and
54. B Sachidananda Sinha was the first President of the he was only playing a joke and he never used the
Constituent Assembly of India which was established required force which would make him liable for the
in the year 1946. offence of Battery or Assault.

55. A The Constitution of India was adopted on 26 November 72. C A’s movement was not completely stopped or
1949. restrained as required by the principle. He had the
option to return.
56. B Slumdog Millionaire got 14 Oscar nominations.
73. B There is a lawful excuse with the policeman who
57. C Vijender Singh was the first Indian to get a medal at was trying to catch suspected criminal.
the World Boxing Championship.
74. A The moment when the clerk read the letter it is
58. A China in 2010 Beijing Olympics won highest number of considered to be published and the libel is complete.
medals.
75. C B being a servant, was acting under the control and
59. B If the Anglo-Indian Community is not adequately direction of A and B and was doing the work assigned
represented in the election, the President can nominate by A’ only. Hence A shall be held liable.
not more than 2 members to Lok Sabha from that
community. 76. B B being a servant, was acting under the control and
direction of A and B and was doing the work assigned
60. A The right to vote in India is a constitutional right. Article by A’ only. Hence A shall be held liable.
326 of the Constitution of India provides the mandates
adult suffrage, which means that every person who 77. C The act of B’ by stopping the truck and giving the lift is
is a citizen of India and who is not less than 18 years an unauthorized act which A’ never expressly
of age can vote. authorized. Hence A will not be liable.

61. C Rs. 25,000 is the maximum amount of fine that can be 78. B Bank will not be held liable because ‘B’ even though
imposed by the Central or State Information Commission was an employee of the Bank, he acted beyond the
on Public Information Officer for not furnishing the scope of his employment and did an act which was
information. not authorized by the Bank.

62. D Till now maximum number of gold medals won by an 79. C The common intention formed by A,B and C was to
individual at Olympics in any event stands at 8 medals. commit burglary and not murder. It was in course of
running away that A was caught and he stabbed X
63. B Heneri Dunant promoted the establishment of the which was never conspired or preconceived amongst
International Committee of the Red Cross. A, B and C.

64. C Boutros Boutros Ghali was the first African to become 80. A A’ will be liable as the direct consequence or effect of
the Secretary General of United Nations not Kofi putting someone on the foothills at that time when the
Annan. temperature is one degree would be hypothermia. So
it could be said that it was reasonably foreseeable
65. A The present UN’ Secretary General Ban Ki Moon is and the person is therefore liable.
from South Korea.
81. C Since the height of 6 feet is not sufficient air space in
66. B Balagangadhar Tilak gave the slogan “Swaraj is my consideration of the problem. Therefore A will not
birth right”. succeed because the hoarding being at a height of
6ft.was not causing obstruction in the ordinary use of
67. D Burma got separated from India in the year 1937. A’s house.

Previous Years
CLAT & AILET Papers Page 107
82. C By virtue of the movie ticket, A was authorized to 94. A Contract is valid as it does not absolutely restrict the
enter the theatre till the time movie show gets over. party to approach to any court; rather it just restricts
Once this time lapses, the presence of A in the theatre the party to go to the other court. So restriction was
will be considered unauthorized and shall amount to not absolute and this is a valid contract.
trespass.
95. B The certainty to the contract is not present. Since the
83. A Since surgeon has breached the duty and the moment property an important part of the contract has not
he does that negligence is complete. been described briefly. Also the certainty to the
property is missing in the contract so it is not a valid
84. A In the present fact the contract is not complete as the contract. Entire ancestral property is a very general
acceptance to the contract never reached to the term, for a valid transfer, law requires that it must be
offeror. Acceptance was not communicated to the specifically mentioned.
offeror.
96. B The intention at the time when A took back the watch
85. B B only posted the letter but the contract would be was not dishonest. He neither caused wrongful loss
complete when A receives the letter and reads it. The to B nor wrongful gain to himself. So he is not liable for
theft.
agreement shall transform into a contract once the
letter of acceptance is received by A and not before
97. B Since B has the entrusted possession of the property
that.
therefore this is a case of breach of trust rather than
theft. Theft is done when something is taken out of
86. B Communication of acceptance is an essential element
possession without that person’s consent, but breach
of contract. A contract shall be entered into only after
of trust is done when someone gives the property in
A reads the email otherwise it will remain incomplete.
trust and other misuse or misappropriates the property
against trust created.
87. A No force or compulsion was used and the consent
given for examination was free consent. So it is a 98. B By direct application of principle it is clear that threat
valid contract. and fear were used for getting the money so there is
extortion.
88. C Both the parties were under mistake of the actual
price of the property and therefore the agreement 99. A A has committed theft only and not robbery as he
was never complete and it was a void agreement. moved the property without using force.

89. C Object or purpose of the contract is against the public 100. A (A) is the correct answer because A shot at B while
policy and morals. As getting married in return of the escaping after taking away the wallet i.e. it was an
money is not correct morally and also it is a crime in attempt to cause to any person death or hurt. Hence,
the law. So the contract is not valid contract. A has committed robbery.

90. A The purpose of the agreement in the facts was to 101. B In present set of facts A is liable as he knew that his
defeat provision of Criminal procedure code which act is likely to cause wrongful loss and damage to the
wants a third person to take the surety of the accused property and therefore it is mischief.
on individual and independent basis. So it has defeated
the provision of the requisite law. 102. A A is the correct option as he possessed the knowledge
that his action is likely to cause destruction of the
91. B Correct answer should be (b) and not (c). In present property and therefore he is guilty of mischief.
facts by entering into the contract couple disentitled
there children from the ancestral property and right 103. B Answer is very clear as intention of A is to remain in
over ancestral property cannot be taken away or the campus unlawfully in order to commit the offence
limited. So contract entered to do the same is not a of theft.
valid contract.
104. B The ultimate object in both the cases is to obtain
92. B The bidders by entering into the contract wanted to possession through the order of the court. In both the
cause loss to the BCCI which is against the principle cases, facts and cause of action are same and suit
of bidding and against the public policy so the contract cannot be initiated again on the same fact.
entered is void.
105. A In absence of any relevant Principle, (a) is the most
93. A Contract is void as contract is restricting company in appropriate answer as it was Sunanda’s car.
there lawful trade.

Previous Years
Page 108 CLAT & AILET Papers
106. C The given statement can be understood as ‘men are 118. D The meaning of fundamentalism is 'a movement or
rarely honest’. The emphasis here is not on the number attitude stressing strict and literal adherence to a set
of men but on number of occasions when they are of basic principles'. Option (D) is closet to this definition
honest or dishonest. This rules out option (A) and (B). of fundamentalism.
Seldom implies rarely so we can conclusively say that
men are dishonest on at least one occasion. This leads
us to option (C). Also, option (D) cannot be definitely 119. B
true since rarely can mean never too. In that case, D e sire
option (D) would become false. Hence, only option (C) H a pp ine ss
conveys the same meaning.

107. B If 'men always obey the law' is correct then 'all men
obey law' (as in B) is also correct. But as the statement
is false the statement 'all men obey law' (as in B) has Only in option (B) the argument logically follows the
to be wrong and hence, doubtful. The remaining three given premiss as happiness is a subset of desire.
can be deduced in some limited scope; hence, cannot
be doubtful completely. 120. * None of the options follow. Options (A) and (D) both
seem correct as both the arguments are incorrect.
108. D Only the statement in (D) is an exactly opposite to the
given one. So, if the given one has to be false, (D) 121. D Here speaker equates exercising control with socialist
must be correct. influence. Disinvestments and privatization show less
of control. Speaker reasons out that it is a shift from
109. C ‘But’ in the fourth premise requires something in socialistic principles. However, if we show that the
contrast with the statement. Only statement (C) brings same phenomenon (less of control) comes due to
out such a contrast. laxity of the government, as in option (D), we can
contradict the speaker's argument.
110. B 'If A is B' is the main cause of the subsequent
outcomes. So if 'If A is B' takes place, all the outcomes 122. C Speaker observes two different responses from the
will automatically take place. (A) can be misleading. intellectuals for two different events of similar nature.
But (A) says because the effect is there the given And he terms it as double standard. Option (C) shows
cause is also there. The effect can take place even the two different responses. It appears to him that
for some other causes (note the difference between “The response to geographical and sociological
necessary and sufficient cause). divisions are very different”.

111. B It is similar to 'only A leads to B and only B leads to C 123. B Author argues that consumption should be need based
therefore, C occurs only if A occurs'. (B) is the correct and hence, prosperity should not depend on
answer. consumption. Option (B) says without consumption
prosperity is shallow i.e. consumption is important to
112. D Statement in option (D) is directly opposite to the given prosperity. Hence, option (B) contradicts the spirit of
statement. the passage.

113. D Enigmatic means puzzling. Option (D) brings out that 124. C Speaker moots that even though the agriculture is the
puzzling fact about dual nature of light. Only (D) has backbone of a nation's economy, other sectors are no
the elements of ‘dual nature’ and ‘enigma’. less important. Therefore, budget allocation should be
need based.
114. A 'I have head-ache' is speaker's perception. No one
else can prove or disprove it. 125. A Speaker suggests to spend money on a field which is
non-remunerative. (A) contradicts the same as it
115. C If a speaker says 'I'm alive' one cannot doubt his/her suggests that money should be spent on a field which
statement as no dead person would be able to say so. generates money i.e, on a remunerative field.

116. C Law-and-order (adj) means strict laws and their 126. A (B) is factually correct and (A) is logically impossible
enforcement. Only option (C) talks of the same. (it is impossible for an equilateral triangle to become
right angle triangle)
117. C Heliocentric system is a cosmological model in which
the Sun is assumed to lie at or near a central point 127. A (A) is logically possible as we may not be able to
(e.g., of the solar system or of the universe) while the explain the riddle of origin of the Universe even in the
Earth and other bodies revolve around it. Therefore, future. (B) is factually improbable.
(C) is obviously wrong.

Previous Years
CLAT & AILET Papers Page 109
128.* (A,B) whole society and each element of the economy to
Area of circle B can be arrived at with the help of the whole economy in the same vein. So they are
reason. Whereas killing of someone can only be analogous.
experienced by using the senses of vision. No option
was given in the actual paper. 141. A 30% of 40% of 560
30 40
129. D Autobiography cannot be written by anyone other = × × 560
than the person. So it is absurd. 100 100

12
130. D When faculties of understanding become limited in = × 560 = 67.2
their performance what suffers is perfection. However, 100
it may not necessarily lead to imperfection. So (A) is
extreme. So is (B). (C) is outside the scope. Only (D) Also, 60% of 40% of 280
brings out the possibility of limited perfection in 60 40
performance. = × × 280
100 100
131. A First statement points at time and the second on space. 24
Thus, universe is finite in space and time. = × 280 = 67.2.
100
132. A Options (B), (C) & (D) negate the given statement. (A)
states the given statement in a different way. 2y − x
142. C On dividing both numerator and denominator in
2y + x
133. C (A) justifies the statement. (B), though goes against
the statement, is somewhat uncertain. (C) says by ‘y’, we get:
something completely opposite to the statement. (D),
 x  4
like (B), is also somewhat uncertain. Thus, optin (C) is 2 −  2− 
the correct answer. 4  y  4  5
+ = +
7  x 7  4
134. B If indeterminism is inherent in Newtonian System, then 2 +  2 + 
 y  5
deterministic law in Physics cannot be characteristically
Newtonian. Thus, option (B) contradicts the given
statement. 6
4 5 4 6 4 3
= + = + = + = 1.
135. D 1 talks about a course in ethics in a Law University; 2 7 14 7 14 7 7
on the other hand says the same course is most 5
probably out of any course of law. Hence, the
statements are incompatible with each other. 143. D Average height of 10 students = 105 cm
Total height of 10 students
136. C* Though none of the options are completely correct, = 105 × 10 = 1050 cm
option (C) is the closest as ‘analogous’ is something Average height of 20 new students = 120 cm
that is comparable and alike in some respect to some Total height of 20 new students
other thing. Thus, identical comes the nearest to it. = 120 × 20 = 2400 cm
Total height of the class
137. B A ‘different’ relation may be symmetric as well as = 1050 + 2400 = 3450 cm
transitive. But it cannot be reflexive. So, it is Average height of the class
inequivalence. Therefore, (B) is the correct option.
3450
= = 115 cm.
138. D The words ‘should be’ in the given statement have 30
some element of tentativeness in the suggestion. Only
option (D) has such tentativeness in suggestion. (A) 144. C Let present age of the father be 2x years.
and (C) are assertive in nature. (B) is tentative but not ∴ Anil’s age = x years
a suggestion. ∴ 3 × (x – 15) = 2x – 15
⇒ 3x – 45 = 2x – 15
139. C Since Socrates is an element of the set ‘innocent ⇒ x = 30 years
persons’, whatever is true of the set must be true of
Socrates. Thus, option (C) is correct. Anil’s age after five years
= 30 + 5 = 35 years.
140. C The speaker brings out the importance of each organ
to the human body, each section of the society to the

Previous Years
Page 110 CLAT & AILET Papers
145. D M1 × D1 = M2 × D2 148. C Let the number of members be n.
And the quantity of cake = 4x kg
1
⇒ 10 × 8 = M2 ×
2 Share of the father = x kg
⇒ M2 = 160. Share of each of the other member
3x
146. B 26 Jan. 1996 is Friday = kg
Number of odd days from: (n − 1)
27 Jan. 1996 to 26 Jan. 1997 = 2
3x
27 Jan. 1997 to 26 Jan. 1998 = 1 ∴x=3× ⇒ n = 10
27 Jan. 1998 to 26 Jan. 1998 = 1 (n − 1)
27 Jan. 1999 to 26 Jan. 2000 = 1 Total number of members = 10.

Total number of odd days 149. A Gain in 60 min = 5 min


=2+1+1+1=5
5 5
Five days after Friday is Wednesday. Gain in 1 min = min = × 60 sec. = 5 sec
60 60
147. B Time taken to cover 80 km ∴ Angle traversed by second hand
80 2 5
= hr. = × 60 min = 40 min = 360° + × 360° = 390°.
120 3 60
Stoppage time after 80 km = 4 min
∴ Effective time to cover 80 km 150. D Let the height of the cylinder be h1
= 40 + 4 = 44 min And the height of the cone be h2

As, 960 = 80 × 12
1
∴Time taken to cover 960 km ∴ πr 2 h2 = πr 2 h1
= (44 × 11 + 40) min = 484 + 40 min 3
= 524 min = 8 hr. 44 min.
h2 3
(Since, there will be no stoppage after covering the ⇒ = i.e. 3 : 1.
last 80 km) h1 1

Previous Years
CLAT & AILET Papers Page 111
AILET Solutions 2011
1. d Refer to the lines “The recent work of Addison Gayle’s 9. b ‘Will rain’ is incorrect. The correct phrase should be ‘If
passes a judgment on the value of Black fiction by it rains this afternoon’.
clear political standards, rating each work according
to the ideas of Black identity, which it propounds.” It is 10. b The preposition ‘of’ is inappropriate with the word
clear from these lines that rating by political standards ‘familiar’. ‘familiar with’ is the grammatically correct
(as done by Addison Gayle) is objectionable to the phrase.
author.
11. b ‘Pick up holes’ is grammatically incorrect. ‘Pick holes’ is
2. a The author reviews the validity of a work of criticism. the correct phrase. It means to find mistakes in
The Roger Rosenblatt’s book ‘Black Fiction’ is the book something someone has done or said, to show that it
with respect to which criticism is being validated by is not good or not correct. For example: The lawyer
the author. He raises objection to the basis of criticism, did her best to pick holes in the witness’s statement.
states what is required before writing an acceptable
criticism and discusses the strengths and 12. b With the words superior, inferior, anterior, posterior
weaknesses of the book ‘Black Fiction’. Option (b) is ‘than’ is not used. ‘Superior to’ is the correct phrase.
incorrect because no critical approach is discussed
in the passage. The author does not provide any data 13. b The subject of the sentence is ‘the steep rise’ which is
regarding the limitations of any kind of criticism. singular in number. So, ‘have’ is incorrect. ‘has affected’
Recapitulation, which means a summary, of the major is the correct phrase.
points in a work of criticism is not presented in the
passage. Thus, option (a) is the correct choice. 14. b With the words ‘with’, ‘together with’, ‘along with’, ‘as
well as’, the subject should follow the first subject.
3. d Refer to the lines “Rosenblatt’s work does leave certain Since the first subject is singular in number (Kiran), so
aesthetic questions open…especially since an the verb should be ‘is’.
attempt to appraise might have led to interesting
results.” Thus, option (d) is the correct answer. 15. b ‘Much water has passed/flowed under the bridge’ is
the correct phrase. It means that something is in the
4. b Refer to the last paragraph of the passage. past and no longer important.
“Irrespective of such omissions… Black Fiction is a
tightly constructed, and levelheaded and penetrating 16. c With the words ‘each’ and ‘every’, the verb as well as
criticism is exemplified in its forthright and lucid style.” the pronoun should be singular in number. So, usage
Thus, option (b) is the most suitable option. of the pronoun ‘their’ is incorrect.

5. c Refer to the lines “Roger Rosenbatt’s book ‘Black 17. b ‘on murder’ is incorrect because of the preposition
Fiction, manages … expounding on Black History” ‘on’. ‘Found guilty of murder’ is the correct phrase.
and “The recent work of Addison Gayle’s passes a
judgment on the value of Black fiction by clear political 18. b Since the word ‘movie’ is a countable noun, it should
standards, rating each work according to the ideas of be preceded by an article ‘a’.
Black identity, which it propounds.” It is clear from
these lines that rating by political standards (as done 19. a ‘Appointed’ in the correct word. The sentence has an
by Addison Gayle) is objectionable to the author. So error- an article ‘a’ is required before the countable
he will be least likely to approve option (c). noun ‘working group’.

6. d The passage does not provide definition of any of the 20. b ‘walk around’ is the correct phrase. It means to walk
terms mentioned in the passage (the words with no particular goal. It also means to behave in a
expressionism, surrealism demand a definition but certain manner or have certain properties; for example:
definitions are not given). Thus, option (d) is He walks around with his nose in the air.
the correct option.
21. d Hard-working is both logically as well as grammatically
7. c As the word ‘these’ is used to denote plural number, correct in the given context. Lethargic and hostile are
so ‘these kind’ is incorrect. ‘these kinds’ is the correct not positive traits and are logically incorrect in the
phrase. context of promotions.

8. d ‘Three-weeks-old; is incorrect. The correct phrase 22. c The word ‘dripping’ is logically correct here.
should have been ‘three-week old baby’. Three-week
is a chunk of tone that is expressed as a single unit 23. c ‘Plaintive’ means sounding sad, especially in a weak
and is thus singular. complaining way; mournful. For example: a plaintive
cry/voice. Thus, the adverb ‘plaintively’ is correct here.

Previous Years
Page 112 CLAT & AILET Papers
24. b The preposition ‘to’ is appropriate with the word 41. d France gets highest percentage of electricity from
‘inimical’. nuclear power stations i.e. (78.8%).

25. d ‘Over’ is the correct preposition in the given context. 42. a V. S. Sampath is the incumbent chief Chief Election
Commissioner of India.
26. b The previous sentence talks about the size of dots
when the color of the dots is on the lighter side of 43. b Justice M. Hidayatullah also became acting vice-
grey. The next sentence that talks about larger dots president while serving the post of Chief Justice of
should logically refer to darker areas. Hence, India.
option (b), ‘darker’, is the correct choice.
44. c India’s Foreign Exchange Reserve has become the
27. c The reference to half-tone and the technical nature of fourth largest in the world in year 2011.
the paragraph suggests that ‘tone’ would be the most
suitable word for the given blank. 45. d Mr. Ahmed Saleem from Maldives is the current
Secretary General of SAARC. Ms. Fathimath Dhiyana
28. b Since the initial step cannot logically be a copy, option
Saeed from same country was the first ever woman
(c) is negated. ‘Print’ is the most suitable word as per
president of SAARC.
the context.

29. a The reversal of light areas and dark areas so that 46. a Air-conditioning was invented Carrier.
black becomes white is the only logically correct idea.
Hence, ‘reversed’ is the correct choice. 47. b Motor Cycle was invented by G. Daimler.

30. c As black becomes white, so white should become 48. c Canada has the largest territory among U.S.A, Brazil
‘black’. Hence, option (c) is the correct choice. and China.

31. d The picture is projected on the screen. ‘On’ is the 49. d Afghanistan joined SAARC in year 2011.
correct preposition.
50. d According to the survey conducted in India during
32. a The verb ‘ruled’ which means arranged is the correct 2001-2003, the highest percentage of deaths were
choice. due to HIV/ AIDS.

33. b The preposition ‘into’ is the correct choice. 51. c 123 Agreement is between Indian and USA dealing
with Nuclear power Plants including only Civil Nuclear
34. a ‘Light from the negative’ is the correct phrase. power plants excluding defence one.

35. c The paragraph mentions the screen and the former 52. d Uttar Pradesh largest state in India in terms of territory.
sentence also talks about the picture being projected
on the screen. This makes ‘screen’ the correct word 53. a Justice V.Ramaswami was the only Supreme Court
for the given blank. who was sought to be impeached under the
Constitution.
36. a Kashiwazaki-kariwa Japan is the world‘s largest
nuclear power Station. Kallpakam is a small town in 54. b Uttar Pradesh largest state in India in terms of territory
Tamil Naidu and is famous for its nuclear power plants and also has maximum number of seats in Rajya Sabha.
affiliated research installations.
55. d The member of Rajya Sabha are elected for the period
37. d The first International Day for women was celebrated of 6 year. Rajya Sabha is a permanent House and is
on 19th March 1911. After that it is celebrated every not subject to dissolution. However, one-third Members
year at March 8. New Zealand is the first country in of Rajya Sabha retire after every second year
the world to give women the right to vote.
56. c A claim of non est factum(Latin for “it is not [my] deed”)
38. c Chernobyl nuclear accident occurred on 26 April 1986 means that the signature on the contract was signed
at the Chernobyl Nuclear power Plant in the Ukraine by mistake, without knowledge of its meaning, but
(then part of the Soviet Union) It is the only one was not done so negligently. If an appointment is illegal,
classified as a level 7 event on the International Nuclear it is non est or non existent in the eye of the law,
Event Scale which is the highest. which renders the appointment to be a nullity. The
Supreme Court in 2011 declared the appointment non-
39. c Shahbaz Batti who was then Minority Affairs Minister est of P. J. Thomas. The High Power Committee which
of Pakistan was assassinated in March 2011. recommended the name of P.J. Thomas did not did not
consider charge sheet relating to a corruption case
40. b Satyanand Mishra is the incumbent Chief Information pending against him.
Commissioner of India.

Previous Years
CLAT & AILET Papers Page 113
57. b Two members of the Anglo-Indian Community are 70. a The literal meaning of the term “Arravali” is “Line of
nominated by the President in the lock Sabha. Peaks” . It a range of mountains in western India and
eastern Pakistan running approximately 800 km from
58. b High Court of Andhra Pradesh struck down the special northwest to southwest across Indian states of
reservation of 4% in favour of Muslims in 2011. Rajasthan, Haryana, and Gujarat and Pakistani
provinces of Punjab and Sindh.
59. d The Year Book of Industrial Statistics 2010 released
by the UN Industrial Development Organization 71. b Undoubtedly, the Bank is liable because the Bank had
ranked India among top 4 countries. authorized its agent Ananth to collect money. Though
on commission basis, Ananth was acting on behalf of
60. a The European Union member states decided to the Bank when he collected the amount from Fatima.
temporarily withdraw the preferential tariff benefits The rule of ‘course of employment’ is clearly made out
from August 2010 on the ground that there are in this case.
significant shortcomings in the implementation of three
UN Human Rights Conventions in Shri Lanka. Recently 72. d Zaverilal should compensate as he said that the article
also, United Nations Warned Shi Lanka for the Human was strong and almost unbreakable. The fact that the
Rights violations reported by many international NGOs. article broke will hold Zaverilal liable. None of the
options given explore this possibility. Hence it’s none
61. c The 2013 South Asian Games, officially the XII South of the above.
Asian Games, is an upcoming major multi-sport event,
scheduled to take place in Delhi, India in February, 73. a Shyam would succeed because denial of title
2013. (depriving Shyam of his bicycle) was actually caused
by Ram. Ram displaced the cycle from its place leaving
62. d Dr. Jean Mayer was a World-renowned nutritionist. open the possibility of it being stolen.
An Award under his name viz. ‘Dr. Jean Mayer Global
Citizenship Award’ was established in the year 1993. 74. a The principle states that the purpose of damages is to
In the present year this award is conferred to compensate the injured person and not to improve his
Susannah Sirkin. She is a deputy director of the position, the amount has spent 4 lakhs thus he should
organisation Physicians for Human Rights (PHS). get back the amount he has spent.

63. b B. N. Rau was the constitutional advisor to the 75. b Here the illegal activity is the act of stealing. The driver
Constituent Assembly. was oblivious of this fact and his ‘right to action’ for
getting injured ensues as a result of his legal act of
64. b Census takes place in India every 10 years. The Latest driving. Driver was acting in complete bonafide belief.
Census was took in the year 2011.
76. c The principle states that the court should not take
65. b In Lyons Frans INTERPOL, a 147-nation Police notice of trifles, the fact speaks about a matter which
Commission has its Headquarters. is a trifle and of a trivial nature for the Court to entertain
the same.
66. b J.H. Dunant founded Red Cross in 1863 which has
its’ Headquarter in Geneva. 77. c Apply the principle over facts. The Limitation Act can
only bar the filing of suit but it can not bar the right of
67. d The European Union came into existence through the Sohan to recover his debt. After the limitation period
‘Maastricht Treaty’ in the year 1992 and it (treaty) also ends Rohan makes a part payment; this brings forth
let to the creation of the single European currency, the the commencement of a fresh period of limitation. As
Euro. Recently the economics comprising the European the cause of action starts from the date of part
Union commonly known as PIGGS (Portugal, Italy, payment, recovery of debt would be allowed.
Greece and Spain) were in International News because
of relentless deepening of economic crisis 78. c Since A failed to pay the income tax, he can not take
the plea that he was not aware about his legal duty to
68. d Justice B.N. Srikrishna was the Chairman of Sixth Pay pay the tax. Law presumes that every citizen should
Commission. be aware about the existing law in force.

69. a Cancun Summit occurred in Mexico in 2010 with an 79. c Vasan can not be held liable for nuisance merely on
aim to to continue efforts towards an international the ground that firstly: he has leased out his house to
deal on cutting carbon emissions. COP16 was the a person who is suffering from AIDS and secondly
official name of the Cancún summit, which is the 16th that the tenants of Varadan’s house are apprehensive
Conference of the Parties (COP) under the United that AIDS will spread in the locality. The correct option
Nations Framework Convention on Climate Change further clarifies that Vasan should no be held
(UNFCCC). responsible for ‘public misperception’.

Previous Years
Page 114 CLAT & AILET Papers
80. b Veerappa’s farm was at a distance of half a furlong 91. b State of X has not absolutely prohibited the right to
which is a reasonable distance hence cannot held carry on trade or business rather it has taken an
liable. The principle of law is Reasonability outweighs obligatory step under DPSP by merely prohibiting the
the liability. cow slaughter.

81. a According to the principle Ramlal is not liable as Ramlal 92. c Right to Religion is a Fundamental Right, the mere fact
is entitled to protect his property by using lawful means. of not singing the national anthem in the morning does
Ramlal has protected his farm in a very reasonable not amount to disrespect .Moreover it is stated in the
and legal manner as neither the fencing nor keeping a principle that the Constitution expects a citizen to
dog is illegal in order to protect one’s farm. respect national anthem as a fundamental duty. No
mandatory compliance of such is mentioned.
82. a Mr. Rajender will not get any compensation as he
violated the traffic rules by riding the scooter on the 93. b True that freedom of speech & expression has been
right side of the road. He was himself at fault hence is conferred upon citizen but this right is not absolute in
not entitled to any compensation. nature. Right to freedom of expression does not
authorize another person to defame any person.
83. b Ramakrishna is liable to pay Rs.40,000/- as agreed in
the contract. Second half of the principle is applicable 94. a According to the principle citizens is expected to take
in this case as it was already agreed between the reasonable duty of care, the fact that Y injured P
parties at the time of contract that whoever will commit proves that Y did not observe reasonable care on his
breach, will pay the amount. behalf. X cannot be made liable for the same.

84. c Krishna cannot exercise the right to self defense as 95. d Standard of care is based on the foresight of an
there is no imminent threat to his life. In this case, it is ordinary prudent person. Law takes into account what
just an intimidation and not an imminent threat. Imminent a reasonable and prudent man foresee and not what
threat arises where there is no intermittent act except the parties to the suit thinks on their own.
the act which may result in harm to the person.
96. c Tort-feasors are the persons who commit the ‘tort’
85. a The fact that Mangeshkar was driving at 140 km/hr in jointly or together.
Bangalore which is crowded and secondly his act
scared the people using the road makes the act rash 97. c The first blow is for self defense, but when the thief is
and negligent. unconscious after the first blow, there is no eminent
threat to Q’s life or property, thus the second blow is
86. c The principle clearly states that when two or more unreasonable. The right of self defence seized to exist
person, in the facts only Bharat is the person who once the thief fell unconscious.
has committed the act, thus its not criminal conspiracy.
In legal context, a computer does not fall in the category 98. a Private defence is only available only against the
of a person. defender. There is no self-defence against self
defence. Right of private defence is preventive in
87. c The manufacturers of a commodity are under a duty nature and can never be aggressively used. It has to
of care for the products it sells to its consumers. The be exercised with utmost care and caution.
fact that the bottle of beer contained a snail makes the
manufacturer liable. Whether there was a contractual 99. c Here right of ‘private defence’ cannot be claimed as
obligation or not is irrelevant here. the robbers were fleeing and there was no imminent
danger to the property. However as none of the
88. b The university is under an obligation to take reasonable options cover this scenario singularly, option c is the
care to prevent injury which a reasonable person best choice.
would foresee. The injury caused to the child in the
given case is foreseeable and not remote. Thus the 100. b As both X and Y conspired to poison Z, thus both are
university would be held liable for negligence. liable. As per the given facts, answer (d) is also correct
but as the principle is about determining and identifying
89. a The snake charmer is liable as because of his the criminal conspiracy, option B is the appropriate
carelessness the child was injured. The snake answer.
charmer is under a duty to see that the snakes do not
escape. 101. d Though murder has not taken place, but both X and Y
did conspire, making both of them liable for criminal
90. b The collateral factor in this contract was that B should conspiracy. As there is no option which covers this
have married C within a period of 6 months. Since B directly, option d is the best choice.
married in 7th month, A is under no obligation to pay
Rs.1 Lakh.

Previous Years
CLAT & AILET Papers Page 115
102. c Criminal Conspiracy has been defined under Section 114. b The alphabetical position number of each successive
120-A of IPC which states that When two or more term increases by 4, 4, 6, 6 and so on.
per-sons agree to do, or cause to be done,— an illegal The required term is X(24).
act, or an act which is not illegal by illegal means, such
115. c The series is moving as:
an agree-ment is designated a criminal conspiracy.
3 = 22 – 1
7 = 23 – 1
103. b As the principle of Contributory Negligence is given,
15 = 24 – 1
and the facts clearly states that due to the negligent
31 = 25 – 1
act of X, X died. Y would be only partly liable to the
63 = 26 – 1
extent of his negligent act.
(127) = 27 – 1
104. b The rescuer is not entitled for the payment as the 116. b P is added before each letter of SIR.
rescuer had no idea of the promise when he acted to Therefore, MAN can be written as PMPAPN.
save the child.
117. b The coding follows the pattern:
105. c The agreement is not valid as it was signed under 3+1=4
threat of suicide which amounts to coercion as per 5–1=4
the principle. 7+1=8
9–1=8
106. a Carpet covers the floor just as wall paper covers the 6+1=7
wall. Therefore, code for 46823 in the same code language
will be 55914.
107. c Kaziranga National Park is famous for rhinocerous.
Similarly, Gir is famous for lions. 118. d Day before yesterday was Tuesday.
Yesterday was Wednesday.
108. c The alphabetical position number of the three letters in Today is Thursday and tomorrow will be Friday.
each successive term increases by 2. So day after tomorrow will be Saturday.
Hence, required term is IWX.
119. d
(N o rth -W est) (N o rth -E ast)
109. c The series is moving as:
6 11 21 36 56 (81)

+5 + 10 + 15 +2 0 + 25

110. b Let the present age (in years) of son and father be x (S o uth-w e st) (S o uth-E ast)
and 3x respectively. Then,
4(x – 5) = 3x – 5 or x = 15 Therefore, Ram is facing South-East direction now.
∴ The age of son is 15 years.
120. d Sun rises in the East in morning.
111. d As the next bus is at 9:35 a.m., the last bus must (Fa cin g N o rth )
have left at 9:05 a.m. Therefore, the enquiry clerk V ikram ’s E a st
gave this information at 9:15 a.m. S h ad ow V ikram
to w ards S h aile sh SUN
112. a The argument says that beans are plumper when they w e st
are prepared without pre- soaking. So when the quality (Fa cin g S ou th )
is more important then pre-soaking should be avoided.
The assumption here is that plumper beans are of Therefore, Shailesh was facing South direction.
better quality. Thus, option (a) is correct.
121. d Option (d) is the most plausible inference. The argument
says ‘almost every Wednesday’ so the best inference
113. b The argument says that children under six are not
would be ‘most if not all Wednesdays’. Options (a) is
allowed from 12 to 5 P.M. and also after 5 P.M. to
incorrect because the question does not say that half
closing time. This means that they can go to the pool
priced coffee is offered on only every Wednesday.
only before noon. Since everyone in the neighborhood
Apart from it being done on almost every Wednesday,
is permitted to swim at the pool, if the neighborhood
it may also be done on every Friday. In that case, the
has a child under six, then the pool must be open for
most common day will be Friday. Thus, option (a) cannot
him/her before noon. Hence, option (b) is the correct
be inferred. Option (b) has the same error. Option (c)
choice.
reverses the reasoning. There may be some days
when half priced coffee is offered and free poetry

Previous Years
Page 116 CLAT & AILET Papers
readings are not scheduled. Half priced coffee may 129. d All the options present possibilities which may
be given even without the poetry schedule because be true. No option can be conclusively considered as
the question does not say that half priced coffee is a false inference.
offered only on those days when poetry is scheduled.
Thus, (c) reverses the reasoning and becomes 130. a Politician is a subset of Men and Men is a subset of
incorrect. Honest. Thus, the conclusions that can be drawn are:
1. All politicians are honest 2. At least one politician is
122. d ‘Rama is neither a brother of Krishna nor is he a lecturer’ honest.
means that Rama is not a brother of Krishna and he is All honest persons are politicians may or may not be
not a lecturer. Thus, option (d) is correct. true. Thus, option (d) is not a conclusion but a
possibility. Therefore, options (b), (c) and (d) are
123. d ‘Make hay while the sun shines’ means to make the negated. But some politicians are not honest is neither
most of one’s opportunities when one has the chance. a conclusion nor a possibility. It does not follow from
None of the options is logically similar to it. the given premises.

124. c ‘The constitution has given the right of free speech. 131. c In the given argument, science and critical thinking are
People speak freely in order to enjoy this right.’ The considered to be related. Similarly, religion is related to
argument with a similar logic is ‘Wealth gives power to dogma. As dogma is not acceptable, therefore religion
enjoy material things. People enjoy these things by is not acceptable. The argument also says that critical
spending their wealth.’ Here, wealth is playing the thinking shakes the foundation of morality. Therefore,
same role as constitution was playing. It empowers if science is accepted (which means critical thinking
people. Constitution gives right of free speech and is accepted as they both are related), then the
wealth gives power to enjoy. One exercises his right foundations of morality are shaken. Thus, option (c) is
to free speech by speaking freely; one exercises the correct.
power to enjoy material things by spending money
and buying them. Thus, these two arguments are 132. a Ramesh’s liking for strawberries, apples, oranges and
analogous and have a similar pattern of reasoning. grapes can be expressed in the following order:
Strawberry > Apple > Orange > Grape
125. d “Literature is the mirror in which the society can see Thus, the statement that he likes grapes more than
itself.” This statement implies that literature is a strawberries is false.
reflection of the society. The issues, ideologies,
patterns, problems etc. of the society are reflected in 133. a The statement that “Justice should not only be done
the literature of that time. Literature enables people to but also seen to have been done” means that people
get acquainted with the mass issues and ideologies. should know that justice is done; simply doing justice
Option (c) does not correctly bring forward the without making people aware of it is not enough. Option
meaning of the statement. It is a half-baked idea. Thus, (a) is correct. The rest of the options are beyond the
option (d) would be a better answer. scope of the given statement.
134. b The argument states that one needs to choose one’s
126. a The argument says that exploitation of charitable exercise regime (yoga or aerobics) on the basis of
instincts of alms givers may aggravate poverty by one’s requirements and expectations. Option (b) is
inducing laziness in beggars. Statement (a) which best supported by the argument. Option (d) is close
states that destitution(poverty) is a result of but can be negated easily because the argument says
unwillingness to work (laziness) supports this that yoga is not the best choice for those looking for a
reasoning. This statement means that beggars became fast paced workout. It may not be the best but it may
poor and had to beg because they were lazy and still be rigorous. Moreover, we cannot say that this is
unwilling to work. Now, if they are given alms, it will the opinion of most of the people. Thus, option (d) is
fulfill their needs and they may continue to be lazy. negated.

127. a The statement that child labor is only a reflection of the 135. b Empirical means based on experiments rather than
malaise in the society means that the society is not in theories. Faith and instinct are difficult to measure.
a good shape as depicted by the prevalence of child So, options (c) and (d) are negated. Whether only
labor. This supports the statement that people do not poor people are driven to violence or not can be easily
abhor (hate) child labour (implied by its prevalence in verified and tested. Thus, option (b) can be empirically
the society). tested. Engineer something (often disapproving)
means to arrange for something to happen or take
128. c If the quality of governance is decided on the place, especially when this is done secretly in order
basis of law and order, it means that law must be to give yourself an advantage. This may be difficult to
indispensable (important) for society (that is why law measure. Thus, option (b) is correct.
and order has been used as a criterion). Other options
are beyond the scope of the argument. 136. * The given question is incomplete.

Previous Years
CLAT & AILET Papers Page 117
137. * The given question is incomplete. 145. b Share of Monu
6
138. d The argument is based on the reason for the tendency = × 324 = Rs. 108.
of a magnetic needle to swing towards the poles. 18
Robert Norman and his predecessor offer different 146. d The given information can be shown with help of Venn
reasonings for this tendency. So, option (d) effectively diagram.
expresses the problem they are working on.
(8 4% ) P h ysics M athe m a tics (4 2% )
139. b Analogy is the process of comparing one thing with
another that has some similar features. In the given
question, James Maxwell noticed that mathematical x y z
laws of uniform motion of heat are analogous to the
laws of attraction and are inversely proportional to
square of the distance. He thought that the source of
x + y = 0.84 × 1000 = 840 .…(i)
heat is analogous to the center of attraction, flow of
y + z = 0.42 × 1000 = 420 .…(ii)
heat is analogous to the accelerating effect of attraction
x + y + z = 1000 .…(iii)
and temperature is analogous to potential. Thus, option
Adding equations (i) and (ii), then subtracting equation
(b) is correct.
(iii) from it, we get y = 260.
140. d The issue here is the criterion to define a straight line.
147. a Present value of share = Rs.100
If 180 degree angle is used as the criterion to decide
Dividend on Rs. 100 shares = Rs.12
straight, then Euclidean geometry is followed. But, if
Let the value of share at the time of purchase be
less than 180 degree is also considered straight then
Rs. x. Then,
we are referring to Non-Euclidean geometry. So, the
choice of definition of a straight line is the issue at 15
× x = 12 or x = 80
stake. 100
The shares were bought at price of Rs.80.
R
141. a S.I. = P × T × 148. d Let the price of the article be Rs.100.
100
New price = 0.9 × 100 = Rs.90
Amount to be paid after 7 months
Hence, required percentage increase
7 11
= 800 + 800 × × ≈ Rs. 851 . 10 1
12 100 = × 100 = 11 % .
90 9
142. c Number of men days required = 16 × 16 = 256
Number of men days completed in 4 days = 16 × 4 = 64
149. c Using alligations,
Number of men on work after 4 days = 16 + 8 = 24
From this instant onwards work will be completed in
R s. 4 0 R s. 5 0
256 − 64
= 8 days.
24

143. d Initial number of members = 500 R s. x


Number of members at beginning of 3rd year
2
 10 
= 500 ×  1 +  = 605.
 100  3 : 2

144. c Let the school begins at ‘t’ minutes after the boy leaves 50 − x 3
his home. = or x = 44
x − 40 2
According to question,
4
Hence, required profit percentage = × 100 = 9.09%
 t + 2.5  t – 5 44
8 ×  = 10 ×  60 
 60   
150. d Let the value of car 4 years ago be Rs.x
⇒ 2t = 70 or t = 35 According to question,
Hence, required distance
4
 10 
 t + 2.5  t – 5 x 1 −  = 52,488 or x = 80,000
= 8 ×  = 10 ×  60  = 5 km.  100 
 60   
Hence, required answer = Rs. 80,000.

Previous Years
Page 118 CLAT & AILET Papers
AILET Solutions 2012
1. A The question is ambiguous since there are numerous of Foreign Affairs, Minister of Communications and
criteria stipulated in Citizenship Act 1955 and under Transportation, Minister of Resources and
Part II of the Constitution of India to acquire citizenship. Development, Minister of Social Welfare, Minister of
Besides the criteria of minimum duration of stay to Public Works and such other portfolios as may be
acquire citizenship there are other criteria which are necessary or desirable for giving to a member of the
simultaneously required to be fulfilled. Amongst the Cabinet the primary responsibility for any Department
given options, (A) is closest answer to the question. or function of government. The President has the
primary responsibility for any Department or function
2. A An Ordinance promulgated shall have the same force of government in respect of which no allocation of a
and effect as an Act of Legislature of the State portfolio is for the time being in force.
assented to by the Governor, but every such
Ordinance shall be laid before the legislative Assembly 8. C The question is ambiguous. In 2004, the Government
of the State, or where there is a Legislative Council in of India declared that languages that met certain
the State, before both the House, and shall cease to requirements could be accorded the status of a
operate at the expiration of six weeks from the “Classical Language in India”. Languages thus far
reassembly of the Legislature. declared to be Classical are Tamil (in 2004), Sanskrit
(in 2005), Kannada and Telugu (in 2008).
3. A Scheduled Castes and Scheduled Tribes are specified
by the Presidential Orders issued in consultation with 9. D Each Security Council member has one vote. Decisions
the Governors of the respective States and in case of on procedural matters are made by an affirmative vote
Union Territories, their respective Lieutenant of at least nine of the 15 members. Decisions on
Governors. substantive matters require nine votes, including the
concurring votes of all five permanent members. This
4. C The State shall secure that the operation of the legal is the rule of “great power unanimity”, often referred
system promotes justice, on a basis of equal to as the “veto” power.
opportunity, and shall, in particular, provide free legal
aid, by suitable legislation or schemes or in any other 10. D In Champaran, a district in state of Bihar, tens of
way, to ensure that opportunities for securing justice thousands of landless serfs, indentured laborers and
are not denied to any citizen by reason of economic or poor farmers were forced to grow indigo and other
other disabilities. cash crops instead of the food crops which was
necessary for their survival. Now in the throes of a
5. A The words ‘Socialist’ and ‘Secular’ were added to the devastating famine, the British levied an oppressive
preamble of the Constitution in the year 1976. The tax which they insisted on increasing in rate. Without
term ‘Secular’ means that the State shall not have any food and without money, the situation was growing
religion of its own. progressively unlivable and the peasants in
Champaran revolted against indigo plant cultivation in
6. D The National Development Council (NDC) or the 1914 (at Pipra) and 1916 (Turkaulia) and Raj Kumar
Rashtriya Vikas Parishad is the apex body for decision Shukla took Mahatma Gandhi to Champaran and the
making and deliberations on development matters in Champaran Satyagraha began. Gandhi arrived in
India, presided over by the Prime Minister. It was set Champaran with a team of eminent lawyers.
up on August 6, 1952 to strengthen and mobilize the
effort and resources of the nation in support of the 11. B RTI is a statutory right. As the name suggests, a
Plan, to promote common economic policies in all vital statutory right is one which is conferred by a statute
spheres, and to ensure the balanced and rapid i.e. a legislative enactment. Consumer Protection Act
development of all parts of the country. The Council is another example of statutory right as it confers
comprises the Prime Minister, the Union Cabinet certain rights to consumers.
Ministers, Chief Ministers of all States or their
substitutes, representatives of the union territories 12. A In 1997, the state of Tamil Nadu first passed laws
and the members of the Commissions. It is an extra- related to ragging Subsequently, a major boost to anti-
constitutional and non-statutory body. Its status is ragging efforts was given by a landmark judgement of
advisory to planning commission but not binding. the Supreme Court of India in May 2001,in response to
a Public Interest Litigation filed by the Vishwa Jagriti
7. C The President, as soon as practicable after taking Mission. The Ministry of Human Resources
office, by writing signed by him, allocate among the Development (MHRD), following a directive by the
members of the Cabinet (including himself if he so Supreme court, appointed a seven member panel
desires) the portfolios of Ministers of Finance, Minister headed by ex-CBI director Dr. R. K. Raghavan to
recommend anti-ragging measures. The Supreme

Previous Years
CLAT & AILET Papers Page 119
Court of India based on the recommendations dated may declare his seat vacant: Provided that in computing
May 16, 2007 makes it obligatory for academic the said period of sixty days no account shall be taken
institutions to file official First Information Reports with of any period during which the House is prorogued or
the police in any instance of a complaint of ragging. is adjourned for more than four consecutive days.
This would ensure that all cases would be formally
investigated under criminal justice system, and not by 20. B There are certain privileges provided to the Members
the academic institutions own ad-hoc bodies. of Parliament which have been incorporated under
Article 105 and 194 of the Constitution of India.
13. D Constitution of India does not mention the post of Deputy
Prime Minister. This post has been recognised in India 21. B The time immediately following the Question Hour and
for practical convenience otherwise there is no laying of papers and before any listed business is
constitutional mandate that along with Prime Minister taken up in the House has come to be popularly known
there shall be a Deputy Prime Minister. as ‘zero hour’. As it starts around 12 noon, this period
is euphemistically termed as ‘zero hour’. For raising
14. C Article 217(1){b} and 124(4) of the Constitution of matters during the so-called zero hour in Lok Sabha,
India provides in detail, the legislative procedure members give notice before 10 a.m. everyday to the
through which a Judge of a High Court or Supreme Speaker stating clearly the subject which they consider
Court (as the case may be) shall be removed. The to be important and wish to raise in the House. The
Constitution of India reflects only 2 grounds of removal maximum duration of the zero hour is unspecified.
i.e. proven misbehaviour and incapacity.
22. C The case is named as : Hari Shankar Jain Vs. Sonia
15. A The founding fathers of our Constitution had before Gandhi, decided on 12th September 2001 in favour of
them the accumulated experience from the working of Mrs. Sonia Gandhi.
all the known constitutions of the world and were
aware of the difficulties faced in the working of those 23. A As per Article-76 Clause (1) of the Constitution of
constitutions. Hence, besides incorporating some India the President shall appoint a person who is
provisions from the other constitutions, a number of qualified to be appointed a Judge of the Supreme Court
provisions were included to avoid some of the to be Attorney General for India.
difficulties experienced in the working of these
constitutions. This is an important reason for making 24. D The earliest codified laws (civil and criminal) that
our Constitution the lengthiest and the most defined and demarcated for the first time in India during
comprehensive of all the written constitutions of the 335-345 AD was followed during Gupta reign.
world. It is pertinent here to mention that in the process
of drafting the Constitution, the most profound influence 25. C Jawahar Lal Nehru introduced “Objective Resolution”
was exercised by The Government of India Act -1935. in the Constituent Assembly in the on December-13,
1946. This resolution as accepted by the Constituent
16. A India has signed repatriation treaty or extradition treaty Assembly forms the basis of Indian political system. It
with various other nations which makes it possible to also guided the constitution drafting process.
secure the presence of accused in India wherein he
can be tried as per the Indian criminal procedure code. 26. C Article 32 confers the power on Supreme Court of
India and Article 226 confers the power upon High
17. C If there arises any question over the validity of the Courts in India to ensure the protection of rights to any
Money Bill, the decision of the Speaker of Lok Sabha is person.
final. The Speaker duly certifies the Bill as Money Bill
because this Bill passes through special procedures 27. B Money Bill can be introduced in the State Legislature
(Article 109 of the Constitution of India.) only with the prior recommendation of the Governor.

18. C The correct answer should be (c) instead of ‘B’. The 28. C In case of National Emergency, the State Government
proposal for enhancing reservation for women in can legislate on the subjects mentioned in the state
Panchayats from the present 33 per cent to 50 per list.
cent has been approved with the provision being
applicable to all seats filled through direct election, 29. C Following is the criteria followed by Election
office of chairpersons and of offices reserved for Commission of India to recognize a political partyin a
SC/ST. The provision of reserving half the seats for State, (A) that such party – has been engaged in
women in Panchayats applies to the total number of political activity for a continuous period of five years;
seats filled by direct election. and has, at the last general election in that State to the
House of the People, or, to the Legislative Assembly of
19. B Clause (4) of Article 101 of the Constitution of India the State, returned - either
mandates that if for a period of sixty days a member ( i ) at least one member to the House of the People
of either House of Parliament is without permission of for every twenty-five members of that House or
the House absent from all meetings thereof, the House any fraction of that number from that State;

Previous Years
Page 120 CLAT & AILET Papers
or (ii) at least one member to the Legislative Assembly 37. B Manage floating or dirty floating refers to the regime of
of that State for every thirty members of that financial practice where the central bank of a country
Assembly or any fraction of that number; as on required basis only attempt to influence their
(B) that the total number of valid votes polled by all countries exchange rate and in normal circumstances
the contesting candidates set up by such party at allow the market forces (i.e. demand and supply) to
the last general election in the State to the House determine the exchange rate of their domestic
of the People, or as the case may be, to the currency. India follows dirty floating system.
Legislative Assembly of the State, is not less than
six per cent of the total number of valid votes 38. C Central Statistical Organization was setup in year 1951.
polled by all the contesting candidates at such It is responsible for coordination of statistical activities
general election in the State. in India including but not limited to evolving and
maintaining statistical Standards.
30. D As per the Model Code of Conduct For Guidance of
Political Parties & Candidates, issued by Election 39. A The year of 1921 is referred, as year of great divide is
Commission of India, electioneering has to be stopped as it marked the massive growth rate of population
in a constituency 48 hrs. before the closing hour of revealed by the census.
polling.
40. A Aurora is the physical phenomenon associated with
31. C Illness caused to Sushmita was not a natural thing or the display of natural light he sky particularly in the
which naturally arose in the usual course of things, high latitude (Arctic and Antarctic) regions, caused by
hence the liability of the Bus company will only be to the collision of energetic charged particles with atoms
the extent of inconvenience caused by walking only. in the high altitude atmosphere.

32. C Lawyers have a right to livelihood which was 41. B The cardinal principles of physics dictate that during
completely restricted by the government and principle free fall because of force of gravity, which happens
(iv) clearly states that any restriction on fundamental at the rate 9.8 m/ s2, any physical body falling will not
right imposed by Parliament which totally removes or experience weight.
nullifies any fundamental right will be construed as
unreasonable. 42. C The density of air rises if we go below sea level that
‘s why the level of mercury rises in a barometer tube
33. B The driver of the car was acting in the bona fide when it is taken down a coal mine.
belief, he was not knowing that the car is stolen and
therefore he can still exercise his right to sue. 43. B Eustachian tubes forms the part of middle ear. It was
discovered by the sixteenth-century anatomist
34. C The master of the ship was knowing that he was Bartolomeo Eustachi.
carrying highly inflammable substance and therefore
he should have been more careful. Even if the planks 44. B Hypothalamus is the part of the brain that connects
slipped from the hands of stevedores the accident nervous system to endocrine system. Hypothalamus
which occurred was reasonably foreseeable hence is responsible for certain activities concerning
master should be held liable. metabolism and automatic nervous system.

35. D If the right to establish and administer educational 45. D A Pin Code consists of 6 digits. First Digit represents
institution of one’s choice has been conferred upon a the Zone. First and Second Digits combined indicate
person, it can not be denied by State Government the Postal Circle or States (except for North-Eastern
merely on the basis of unjustified prediction on part of India as its a Postal Circle). Third Digit indicates the
state government that such institution will create Sorting Unit/District of the respective postal circle. Last
communal tension in the proposed area. The state three digits indicate the delivery Post Office.
government can not restrict a fundamental right merely
on a presumption of communal tension. 46. D Indus Valley Civilization also known as Harrapan
Civilization was a Bronze age civilization located in
36. C Capital Account Convertibility (CAC) or a floating south Asia.
exchange rate means the freedom to convert local
financial assets into foreign financial assets and vice 47. B Chand Bibi was a Indian Muslim Warrior Muslim
versa at market determined rates of exchange. This defended Ahmednagar against the Mughal forces of
means that capital account convertibility allows anyone Emperor Akbar.
to freely move from local currency into foreign
currency and back. In India there exists full Current 48. D Tipu Sultan (also known as Tiger of Mysore) was son
Account Convertibility i.e. Indian Currency rupee is of Hyder Ali.
convertible to any other currency. The capital account
convertibility is sought to be achieved by RBI in the 49. D The 1857 revolt began in the town of Meerut. Sir Sayed
future if suitable macro-economic conditions such as Ahmed Khan wrote critiques in 1859 by title of “The
fiscal consolidation is achieved. Causes of the Indian Mutiny” in 1859.

Previous Years
CLAT & AILET Papers Page 121
50. A Mahatma Gandhi ‘s book is entitled “My Experiments the list and Incumbent prime minister of India Mr. Man
with Truth”. Mohan Singh is on 17th Rank.

51. A The Royal Indian Navy mutiny (also called the Bombay 63. C The sixth option presented by Srikrishna Committee
Mutiny) refers to the strike by the total strike followed was the most workable option i.e.”Keeping the State
by the mutiny by the Indian sailors of the Royal Indian united by simultaneously providing certain definite
Navy. Constitutional/Statutory measures for socio-economic
development and political empowerment of Telangana
52. D The extant Preamble in our constitution embodying our region through creation of a statutorily empowered
cherished ideals (Liberty, equality and Justice etc.) Telangana Regional Council”.
were first introduced through ‘the Objectives resolution’
which was moved in the Constituent Assembly on 64. B Balwant Singh Rajoana recently has been convicted
December 13, 1946 by Pt. Jawaharlal Nehru. in the Beant Singh assassination case on March 31,
2012. Beant Singh was the Ex- Chief Minister of state
53. B Inflation is measured through Wholesale Price Index of Punjab.
(WPI) in India. There are two types of Inflation Demand
Pull and Cost Pull Inflation. Hyperinflation occurs due 65. A News of the World (NOW) has been recently alleged
to continuing supply of money, which results into the using illicit hacking into voicemail messages of
real value of domestic currency not being reflected by prominent people to find stories.
the prices of the commodities.

54. D The Core of the Moon consists of volatile Gas. Recently 66. B Ms. Aung San Suu Kyi who has recently accepted the
China declared that it will launch an unmanned lunar prestigious Nobel prize conferred to her in the year
rover Chang’e-3 on moon in the year 2013. 1993. She was released after 15 years of house
arrest.
55. D Thermal Convection Current Theory propounded by
Arthur Holmes contributes to the understanding of 67. A
evolution of various landforms on earth due to thermal
convection currents it does not deal with origin of 68. B More than 50 Sikh Shrines in India are named after
earth. The theory describes the origin of for mountain trees.
building, continental drift, volcanism and formation of
ocean trenches etc. 69. B Belgium set the record as the one with the longest
time without government, in April 2011. Herman Van
56. C Rompuy was re-elected head of the states recently.

57. A The “Aorta” is the largest artery in the body, originating 70. B The Current President of IMF is Christine Lagarde from
from the left ventricle of the heart and extending down France. She is also the first woman president if IMF.
to the abdomen,
71. C The hypothesis states that the more a student studies,
58. D Sunken Stomata are found in predominantly in desert the higher are his grades. The observation is that the
plants where moisture may be unavailable or in short students who spent most time studying did not get
supply for at least part of the year The aforementioned grades as high as did many students who studied
adaptation helps survival of such plants in the regions less. The hypothesis compares the performance of a
having low supply of moisture or none by reducing student who studies say 4 hours a day with the same
water loss. student if he/she were to study 6 hours a day.
However, the observation compares the performance
59. B Enzymes are proteins that are involved in digestion of one student with another. Option (c) helps in
and every chemical reaction in our body. According to explaining why the observation does not undermine
food enzyme researcher, Dr. Edward Howell, “Without the hypothesis. It says when a student studies more,
enzymes, no activity at all would take place. Neither his performance improves relative to his own past
vitamins, minerals, or hormones can do any work — performance. Option A is incorrect since it is a
without enzymes.” comparison across students, whereas the conclusion
is about a particular student under different conditions.
60. B Sweet potato is the root of the plant. Option B is irrelevant since it does not provide any
reason for researchers sticking to their conclusion.
61. A Glasgow city, which is in Scotland, UK. the venue of Option D is irrelevant too as we are not concerned as
common wealth games in 2014. to what the students do when they are not studying.

62. A The President of USA. Barrak Obama retained as a 72. D The argument of the educator is that it is known that
top leader in Forbes list of “World’s Most Powerful the vote of an individual is much more likely to determine
Persons”. From India Sonia Gandhi is on 11 the rank in organizational policy by influencing the election of an

Previous Years
Page 122 CLAT & AILET Papers
officer by influencing the result of a direct vote. He surgery. The argument, however, says that there were
concludes to say that it is better to take decisions two types of tape that were used in the study. Option
through the election of an officer than direct vote. B is incorrect as we are not comparing which of the
Option A is incorrect; the issue is not about one two, surgery or recovery, is more stressful. Similarly,
person’s vote weighing more than any other. Option B option D is irrelevant as whatever be the psychological
can be ruled out on the same lines. The argument is effects of music, the conclusion would still hold true.
how to take decisions, not how to evaluate the
outcomes of these decisions. Option C supports the 76. B Conclusion I does not follow from the statement. It
conclusion but does not provide a link between the cannot be generalized to say that the constitution is
premise and the conclusion. Option D is correct. If the interpreted always to help oppressed people.
main aim of the procedures of the organization is to Moreover, the statement is about Supreme Court and
maximize the influence of each individual on the not about interpretation of the constitution. Conclusion
decisions made and we know that an individual has II definitely follows. There has to be some observation
greater influence in election of an officer then the on the part of The Supreme Court to give the judgement.
decisions must be made through election of an officer.
77. C Neither of the two conclusions follows. The statement
says that the ideas that were given by our ancestors
73. B* The argument states that higher taxes would produce
were then discarded for their unviability but presently
reduction in smoking. Option A may strengthen the
they turn out to be functional and inevitable. Conclusion
argument. It says that under some conditions a tax
I does not follow since the statement does not talk
would have the desired effect. Option B is the best
about the classification of ideas. Conclusion II does
wrong answer. It makes it sound like people will
not follow because the statement does not say that
continue to buy tobacco regardless. However, just
there are no new ideas in the present.
because they’re more likely to continue buying tobacco
does not mean that there was not a reduction in the 78. D Both the assumptions are implicit. Had there been no
amount of tobacco consumed. Option C weakens the one who would not know about ‘M’ brand shoes, there
argument by showing that the tax would not reduce would be no point in giving such information.
the amount of tobacco consumed; just shift the Assumption II is implicit. The statement says one should
purchase to the neighboring town. Option D is buy ‘M’ brand shoes because they are available in all
irrelevant. Just because people are more likely to quit sizes and last longer. It is clear that people would
when they’re well educated about the hazards of prefer shoes that last longer, and thus this is reason
smoking does not mean that they wouldn’t quit because enough to convince them to buy these shoes. Option
of a tax imposed on tobacco. Option C is the correct D is the correct choice.
choice, though NLU-D has given B as the correct
answer. 79. C Neither of the assumptions is valid. The statement
does not talk about the development of observed
74. A The conclusion or the main point of the argument is intelligence or measurable intelligence and such stuff.
that speech acquisition is entirely a motor control These are irrelevant to the statement and the statement
process rather than a process that is abstract or lacks information to consider these assumptions valid.
mental. In other words — the point at which we learn
to talk is entirely based on when we learn how to For questions 80 to 84:
physically control our mouths, and nothing else. If option The given information can be tabulated as follows:
A wasn’t true, it would mean that there are other
elements that impact speech acquisition. Option A is Rank in 1 2 3 4 5 6
an answer that must be true in order for the argument earnings
to be true, and it is therefore correct. Option B doesn’t
Person W R P Q T S
have to be true — we don’t need to know anything
about the entire babbling stage (this is not discussed Colour Green Yellow White Black Red Blue
directly), and “intentionally move their tongues,” though
related to ability to create sounds of languages, is not
the same thing, and therefore this is not directly 80. C W, R and P earn more than Q.
relevant. Option C is not relevant to the argument and
can be eliminated. Option D doesn’t have to be true for 81. D S has the lowest earnings.
the author to make his point.
82. A In such a case, P will earn the third highest among
75. C The argument is that those who listened to music them.
instead of conversation required fewer painkilling
things and thus it can be concluded that reducing 83. B Green
stress lessens the pain sensitivity. Option A is incorrect
as it contradicts the argument. It says all the patients 84. C Blue
in the study listen to same tapes before and after a

Previous Years
CLAT & AILET Papers Page 123
85. B 3 0 fee t 93. B The series is of the form n2 + n (where n is a natural
number).
1
2 0 fee t 2 0 fee t 12 + 1 = 2
22 + 2 = 6
1 5 fee t
32 + 3 = 12 (15)
Sta rting p oint E n d po in t 42 + 4 = 20
52 + 5 = 30
1 5 fee t 1 5 fee t
62 + 6 = 42
So the incorrect number is 15.
1 5 fee t
Clearly, the child is 30 feet to the East of the starting 94. D The pattern followed in the series is as follows:
point.
2 5 10 17 26 37 50 6 4 (65)
86. D B is the brother of C
⇒ B is a male.
G is the spouse of B and H is the father of G. +3 +5 +7 +9 +11 +1 3 + 15
⇒ B is the son-in-law of H. So the incorrect number is 64.

87. D A is the father of C and E is the daughter of C. 95. B D(4) + 4 → H(8)


H(8) + 4 → L(12)
⇒ E is the granddaughter of A.
L(12) + 6 → R(18)
So the next term will be R(18) + 6 → X(24).
88. C Let the age of A be ‘a’ years at one stage.
Then the age of B will be (a – 4) years at this stage.
96. C 3 7 15 31 63 ? (127)
After 16 years, a + 16 = 3a ⇒ 2a = 16 ⇒ a = 8
∴ At initial stage, age of A = 8 years; B = 4 years
Two years before this stage, ages of A and B were 6
years and 2 years respectively. +4 +8 + 16 +3 2 +6 4
So the next term will be 63 + (2 × 32) = 63 + 64 = 127.
89. D If day before yesterday was Tuesday, then today is
Thursday. 97. A Consider PLANING as PLANNIG.
⇒ The day after tomorrow will be Saturday. A(1) + 5 = F(6)
U(21) – 6 = O(15)
90. A My sister was born (562 + 75 × 7) = 1087 days before T(20) + 5 = Y(25)
me and it was a Tuesday. H(8) – 6 = B(2)
Number of odd days (i.e. remainder left when 1087 is O(15) + 5 = T(20)
divided by 7) is 2. R(18) – 6 = L(12)
I(9) + 5 = N(14)
⇒ I was born on a Sunday.
T(20) – 6 = N(14)
Y(25) + 5 = D(4)
91. D Initially Ram is facing North-West direction. After
Hence, the code of PLANNING is FOYBTLNND.
turning 900 CW, he will be facing North-East direction.
By turning 1800 ACW, he will be facing South-West
98. A The letters in the odd number of places are increased
direction. Finally, he turns 90° ACW which will make
by 2 positions while those at the even number of
him face in the South-East direction.
places remain same.
N Hence, the code for VOTER is XOVET.

99. B Like a child is afraid of fire, a student is afraid of


9 0° examination. Hence, option B is correct.
1 80 °
W E 100. C The relation between ‘a grain of salt’ and ‘a chip of
glass’ is that these are the smallest possible quantities
9 0° by which the salt and glass can be described. Though
‘a blade of grass’ and ‘a shred of wool’ also make
S o uth– E ast sense but these can be further broken down unlike ‘a
S grain of salt’ and ‘a chip of glass’. Option B is irrelevant;
92. D If a man stands facing the North, at the time of sunrise it isn’t analogous to the grain: salt.
his shadow will be towards his left. So Vikram was
facing the North. Hence, Shailesh was facing the South.

Previous Years
Page 124 CLAT & AILET Papers
For questions 101 to 105: 108. B Let the total number of days of my tour away from
The given information about productivity is: home be ‘n’.
F > G, R > S, R > T , S > H, G > T. My expenditure, on day 1 = 10 × 1,
Also, the night-shift crews are either G and T or S and H. on day 2 = 10 × 2,
on day 3 = 10 × 3 and so on.
101. D Option (a) is incorrect as it reads T > R, which is not Therefore, on day n = 10 × n
true.
∴ 10 × 1 + 10 × 2 + 10 × 3 + …. + 10 × n = 18300
Option (b) is incorrect as it reads H > S, which is not
⇒ 10[1 + 2 + 3 + …. + n] = 18300
true.
Option (c) is incorrect as it reads G > F, which is not ⇒ 1 + 2 + 3 + …. + n = 1830
true. n (n + 1)
Only option (d) follows all the rules. ⇒ = 1830
2
102. B Clearly, we know that three crews, S, T and H are ⇒ n(n + 1) = 3660 = 60 × 61
less productive than R.
⇒ n = 60
So R is either ranked first or second.
Also, G is less productive than the third ranked crew F. Hence, my tour lasted for a total of 2 months.
So G must be in the bottom 3 ranks.
If R is at the second place, the bottom three positions 109. B Three years before, sum of the ages of all five
will have 4 contenders, which is not true. members in the family = 17 × 5 = 85 years
∴ R is placed first. At present, their sum of ages = 85 + 3 × 5 = 100 years.
Also, G and T are surely in the bottom three (since F > But, since a baby was born, the present sum of the
G > T). age of six-member family = 17 × 6 = 102 years.
Of the remaining, S > H. ∴ Present age of the baby = 102 – 100 = 2 years.
Thus, S will be ranked second and H will be in bottom
three. 110. C Let the marked price be Rs.x.
1 5
103. C Clearly, we know that three crews, S, T and H are Then, cost price is x = Rs. x
1.2 6
less productive than R.
So R cannot be ranked fifth in any week. And the selling price is 90 x = Rs. 9 x
100 10
104. C If H is ranked third for any week, then the ranking of ∴ Profit percentage
the crew is as follows:
9 5
Rank 1 2 3 4 5 6 x− x
S.P. − C.P 10 6 × 100 = 8%
= × 100 = .
Crew C.P 5
R S H F G T x
6

105. C The fifth and sixth ranked crews can be either G and Alternate solution:
T or S and H. 20 + 10
So H can never be ranked fifth. 20 – 10 – = 8% .
100
If S and H are ranked fifth and sixth, two arrangements
(in desending order) are possible:
111. C Let the ages of A, B and C be 3x, 5x and 7x years
F G R T S H or R F G T S H
respectively.
We find that G cannot be fourth and R can be ranked
third. ∴ 3x + 5x + 7x = 25 × 3
⇒ 15x = 75 ⇒ x = 5
106. C Let the number of 1 rupee, fifty paise and twenty five Hence, age of A is (3 × 5) = 15 years.
paise coins be 2x, 3x and 10x respectively.
1 1 112. B Let the principal be Rs.P and the rate of interest per
∴ 2x × 1 + 3x × + 10x × = 72 annum be r%.
2 4
3x 5x As the amount became two-folded in 10 years, it implies
⇒ 2x + + = 72 ⇒ 6x = 72 ⇒ x = 12 that the simple interest for 10 years is equal to the
2 2
principal, Rs.P.
Hence, the number of twenty five paise coins = 10 ×
12 = 120. Hence, P = P × r ×10 ⇒ r = 10% .
100
107. B Let the maximum number of marks be x.
45 25 20 113. A Let the simple rate of interest per annum be r%.
∴ x− x = 70 ⇒ x = 70
100 100 100 80 × r × 4 100 × 60 75
∴ (140 − 80 ) = ⇒ r= =
⇒ x = 70 × 5 = 350 100 80 × 4 4
Hence, the maximum number of marks is 350.

Previous Years
CLAT & AILET Papers Page 125
120. C ‘Prevailed’ is incorrect as the sentence is in present
75
96 × × 10 tense. It talks about the custom being ‘still prevalent’.
Again, for 10 years, S.I. = 4 = Rs.180 Hence, the correct usage should be ‘the custom
100 prevails’. Option C is the correct choice.
Hence, amount after 10 years = S.I. + Rs.96 = Rs.276.
121. C Lynch means to kill by hanging. Hence, option C is the
114. D Let the original number of workers who could complete correct answer.
the work in 25 days be x.
So one worker would have completed the work in 25x 122. B Platitude refers to a statement that has been used too
days. often to be interesting or thoughtful. ‘Stereo-typed
Also, the number of workers who completed the work statements’ fits the bill perfectly making option B the
in 40 days = x – 6 correct choice.
∴ One worker completed the work in 40(x – 6) days.
As the number of days required by one worker to 123. D Perennial means perpetual or continuous. Rare is
complete the work is constant, therefore, opposite in meaning to it and hence the correct choice.
25x = 40(x – 6).
On solving the above equation, we get x = 16. 124. A Fiasco means a disaster or failure. Success is the
Hence, original number of workers was 16. correct antonym.

125. A Some clauses with if are like hypotheses so we


1 use past tense forms to talk about the present and
115. A Part of the tank filled in one hour by ‘A’ = 8
future. The correct statement should therefore use
1 ‘knew’.
Part of the tank filled in one hour by ‘B’ =
6
126. B The sentence requires past perfect tense which is
1 there only in option B. Hence, “arrived” is the correct
Part of the tank emptied in one hour by ‘C’ = .
24 choice.
∴ Part of the tank filled in one hour by the three pipes
127. A In this question the person making the statement is
1 1 1 1 giving an opinion about the car. We need a word that
together = + − =
8 6 24 4 means ‘to think of something in a particular way’. Hence
∴ The entire tank will be filled by the three pipes ‘consider’ is the right choice.
together in 4 hours.
Alternate solution 128. C ‘Over’ is used when referring to a cause of interest or
Capacity of the tank = LCM(8, 6, 24) = 24 units discussion.
Number of units filled by tap A in one hour = 3
Number of units filled by tap B in one hour = 4 129. C The correct statement should be “in an armed chair”.
Number of units emptied by tap C in one hour = 1 ‘into’ is used to show movement or transition, as in
Number of units filled by all three taps in one hour “into the water”.
=3+4–1=6
130. B The statement requires a word that indicates naivety
24 and artlessness. Hence, ‘innocent’ is the right fit.
Hence, the tank will be filled in = 4 hours.
6
131. D For situations in the present or future where we are
116. B Part B is incorrect as it has ‘subject verb agreement’ describing what hasn’t happened yet but we can
error. The subject here is ‘number’ and not ‘marks’, easily imagine it happening, we use present tense in
and since it is singular, the correct verb should be ‘is’ the conditional clause and will + verb in the main clause.
not ‘are’.
132. C The correct preposition is ‘at’. ‘At’ is used as a
117. A Since part B forms the beginning of another clause preposition of place where you are to do something
after part A, there should be a comma at the end of typical (study, watch a film, work).
part A.
133. D Defect is the general word for any kind of shortcoming
118. D The sentence is correct in its given form. or imperfection, whether literal or figurative: a defect
in eyesight, in a plan. Weakness can refer to a quality
119. C The modal verb “would” is used in a conditional sense regarded as a disadvantage. Fault is an error or a
(or in a hypothetical/theoretical situation). “Will” is used mistake. Flaw refers to a feature that mars the
when something is going to be done, not just perfection of something. Flaw is the right word here;
hypothetically/conditionally. The correct sentence an otherwise perfect hero may have an internal flaw.
should be, “She would do it”.

Previous Years
Page 126 CLAT & AILET Papers
134. B* RQ forms a mandatory pair as the ‘he’ in Q refers to 147. A The passage talks about group cohesiveness and its
‘Louis Pasteur’ in R. Also, R introduces the glass-tube impact on decision making. The author begins by
which has been later mentioned in Q and S. S follows Q mentioning that cohesiveness in a group encourages
as it explains what ‘he’ does with the tube. P cannot people to be frank and open. In the second paragraph,
come anywhere but after the opening sentence. The he talks about the ‘groupthink’ syndrome that stems
‘its’ in P refers to the ‘dog’ in S1. The RQ pair is in only from the need to conform to group majority. Option A,
one option i.e. option A. Hence, option A should be thus, is the right choice. Option B is incorrect because
correct, though the answer given by NLU-D is option B. the passage does not go into how group members
need to guard against groupthink. The passage is not
135. C Q definitely follows S1 as it tells the name of the monkey about diplomacy and military affairs per se; rather
talked about in the opening sentence. S follows Q as it they are just mentioned as examples. This negates
compares the Capuchins with Marmosets thatcould also option C as well. Option D is incorrect as the author
be candidates for the most recommended pet but for does not conclude that low cohesion groups are better
their delicacy. P explains what effect the English climate or worse than high cohesion groups.
has on Marmosets and follows S. R comes after S as it
explains how the Capuchins are better for the English 148. B The scenario presented here is of a cohesive group
climate. Thus, option C has the correct sequence. where members initially put forth their conflicting ideas
but due to lack of agreement, compromised on a
136. A ‘Kind’ is the correct adjective since the sentence talks disastrous diplomatic decision for the sake of arriving
about Ganga’s qualities of giving and taking away. at a consensus. This is similar to the groupthink
syndrome mentioned in the passage where a cohesive
137. D The Ganga ‘gives’ anything that we ‘ask’ for, hence group gives in to the need to achieve group consensus.
option D fills the blank correctly. ‘Refuse’, ‘take’ and Option B is on the same lines and is the correct choice.
‘ignore’ do not fit in the context here. Option A is incorrect as it is mentioned in the question
that the group members are closely associated. Option
138. B The Ganga is referred to as ‘her’ in the sentences C can be negated outright, since the scenario indeed
above. Same pronoun follows here. ‘Only’ and ‘any’ is an evidence of groupthink. Option D is incorrect as
do not make sense in the context of the passage. the group did demonstrate an “illusion of unanimity” by
Hence, option B is correct. coming to a ‘diplomatic’ decision that they were not
convinced about.
139. D One cannot ‘count’, ‘dot’ or ‘pursue’ the rays of the
sun. The correct verb here is ‘watching’. 149. B Groupthink, as given in the passage, is a syndrome
wherein members stop attempting to carry out a critical
140. C Gold shines or ‘glints’. Hence, ‘glints’ is the correct scrutiny that could reveal grounds for objections.
adjective for metallic gold. Rather, they give the benefit of doubt to the group
consensus. Option B is correct. Groupthink’s main
141. D The sentence talks about the river changing its component is the strong belief of the members that the
direction. Hence, ‘turn’ is the correct choice here. group decision is right. Option A is incorrect; the
passage doesn’t talk about any suspicions between
142. A The correct word is ‘deep’. People stand ‘chest deep’ group members. Option C is incorrect as it is not stress
in water which means that they are submerged in that leads to illusions of invulnerability; rather they
water chest down. both are factors contributing to groupthink. Option D is
characteristic of non-cohesive group, not groupthink.
143. C Invocations meaning ‘spoken words that are believed
to have magical powers’ is the right option. 150. A Option B is incorrect as the author states the
researchers’ findings that factors contributing to
144. B Pouring water through their fingers is a ritual that people groupthink form a recurring pattern, meaning they are
do while bathing in Ganga. Hence, option B is correct. not unique to each case. Option C is incorrect, study
of cohesiveness does contribute to determining the
145. D ‘Knew’ is the right word. Heard, mentioned and factors that elicit groupthink. The author does not make
conceived are incorrect in the context of the passage. a conclusive statement about groupthink having or not
having any beneficial effect on decision making. Hence,
146. D All of the reasons mentioned in options A, B and C option D cannot be inferred. Option A is correct; the
contribute to a cohesive group doing a better job at author mentions that highly cohesive groups
decision making than a non-cohesive group. It is experience this syndrome. Refer to the last line, “...it is
mentioned in the passage that the members feel free important to work towards...whether group
to voice their opinion being part of a cohesive group. cohesiveness will deteriorate...or allow for effective
Refer to the lines, “In a highly cohesive group...not decision making”. It suggests that its contribution is
that individuals will conceal objections...” and still not fully understood.
“...participants in the group’s deliberations...” Hence,
option D is correct.

Previous Years
CLAT & AILET Papers Page 127
AILET Solutions 2013
1. c The given passage talks about the legal systems of describing these terms in separate paragraphs. The
the United Stated and England and how different they author also uses the process of generalization in the
are in their use of legal reasons. Therefore, option (c) passage, especially in the second paragraph, when
is the correct answer. The passage doesn’t talk of he gives the example of a statute and concludes the
any similarities and differences between the legal fact that most of the judges in the United States would
systems of England and the United States, except that find the defendants not guilty, based on a few facts
in their use of legal reasons. Hence, option (a) is and what is known about legal system in the United
incorrect. Option (b) is incorrect since the author uses States. Option (d) – a chronology of the historical
examples not to re-evaluate the legal systems per se developments, is nowhere used in the passage and
but to explain the use of substantive and formal hence, it is the correct answer.
reasons in their respective legal systems. Option (d)
is incorrect as the passage doesn’t talk of the 7. b ‘With the processes’ is grammatically incorrect. The
development of legal reasoning in general but only as correct sentence should be “In the process, it has
limited to England and the United States. become…tea company.” Remaining sentences are
grammatically correct. Hence, option (b) is correct.
2. d In the first paragraph of the passage, the author tells
us that formal reasons are more common in England 8. a Since the directions clearly spell that the sentences
than substantive reasons. Then the author goes on to are about a single topic, it can be inferred that the
describe what he means by formal reasons and as an “predatory mood” talked about in sentence I refers to
example, says in the third paragraph that according to the acquisitions talked about in the previous question.
the laws that are followed in England, if a document Thus, the correct pronoun for “predatory mood” here
fails to conform to the rules that have been specified, is ‘this’. All other sentences are grammatically correct.
then the court has a right to make the document legally Hence, option (a) is correct.
ineffective. So, it can be clearly inferred that the
English judges would find the Veteran’s group guilty 9. a In sentence I, the correct phrase should be “step by
because they have clearly violated the “stipulated step”. In sentence II articles ‘a’ and ‘the’ are missing.
requirements”, which stated that no vehicles should Also, instead of ‘to’, ‘that’ is the correct usage. ‘That’ is
be taken into public parks. Hence, option (d) is the used for referring to somebody/something that has
correct answer. already been mentioned or is already known about.
The correct sentence should be “Try using a
3. d All of the first three options are circumstances under conceptual image or a photo to highlight your main
which the substantive arguments will not be message versus the very first thing that comes to
considered as none of the options give valid reasons mind when thinking about your product or service.”
for the Will to be considered enforceable. Only option Hence, option (a) is correct.
(d) is a circumstance when the Will may be considered
valid without a written witness, since it is a case of a 10. a Usage of the word ‘foremost’ with ‘overall is incorrect.
Judge allowing for a verbal witness during a medical Foremost means: the most important or famous; in a
emergency. position at the front. In the given context, foremost is
laying emphasis which is already explained by the
4. a The last paragraph of the passage describes what phrase, ‘is an important piece of the..’ Correct sentence
has happened as a result of following the extreme is: “The typeface that you choose for your print project
forms of legal reasoning (substantive reasons and is an important piece of the overall design process”.
formal reasons) in England and the United States. Hence, option (a) is correct.
Therefore, option (a) is the correct answer.
11. d Sentence II is incorrect as instead of ‘of’, ‘that’ is the
5. c Option (c) is the correct answer. Refer to the line, correct usage. The correct sentence should be “Be
“Once the legal rule…and the legality of the rule is not sure that the font we choose is legible and logical”.
in question…” This clearly suggests that in case the ‘That’ is used for referring to somebody/something
legality of a rule is in question, substantive reasons that has already been mentioned or is already known
might be warranted. about. ‘That’ refers to ‘readability’ mentioned in sentence
I. In sentence III, the usage of “newest” is incorrect.
6. d The authors makes use of option (a) – comparison The sentence is not parallel as both the adjectives are
and contrast, when he talks about the different legal not in the same form (newest is superlative, interesting
reasons that are used in England and the United States is positive). The correct sentence should be “With all
– formal reasons and substantive reasons of the new and interesting…” Hence, option (d) is
respectively. He then goes on to use option (c) by correct.

Previous Years
Page 128 CLAT & AILET Papers
12. a ‘Aberration’ means ‘the fact or an instance of deviating’. 20. c Option (a), pass of, is incorrect usage. Option (b),
Thus, option (a) is correct. Option (c), ‘abhorrence’, ‘pass on’, means ‘to place into the hands or custody
means ‘a feeling of strong hatred, especially for moral of’. Option (d), ‘passed away’ means ‘to die’, hence
reasons’. incorrect. Option (c), ‘passed off’ means ‘to continue
to completion, to occur’. Hence, option (c) is correct.
13. a ‘Potpourri’ means ‘a miscellaneous collection or a
mixture of various things that were not originally 21. b Option (a), usage of ‘little’ implies ‘negligible, none’.
intended to form a group’. Option (a), ‘medley’, means Hence, it is incorrect. Options (c) and (d) are incorrect
‘a mixture of people or things of different kinds’. Hence, as ‘any’ and ‘some’ are used when the speaker cannot
option (a) is the correct answer. specify or does not need/want to specify a number or
an exact amount. Usage of ‘a little’ implies ‘a positive
14. d ‘Imposture’ means ‘an act of tricking people deliberately idea along with an uncountable noun’. Option (b), ‘a
by pretending to be somebody else’. Option (d), little’, implies ‘a small quantity’ (of water in the given
‘deception’, means ‘the act of deliberately making context). Hence, option (b) is correct.
somebody believe something that is not true’. Hence,
option (d) is the correct answer. 22. b The word ‘sarcastic’ in the first part of the sentence
implies that the subject is using sarcasm. Sarcasm
15. a ‘Parley’ means ‘to speak with another, to confer, that is means ‘a way of using words that are the opposite of
a discussion between enemies or people who what you mean in order to be unpleasant to somebody
disagree’. Option (a), ‘discuss’ means ‘to talk about’. or to make fun of them’. Option (b), bitterness means
Hence, option (a) is the correct answer. ‘feeling unhappy or angry because you feel that you
have been treated unfair’. Option (b) clearly implies
16. c ‘Protean’ means ‘able to change quickly and easily’. unpleasant feelings of the subject in the given context.
Option (a), ‘versatile’, means ‘changing or fluctuating Hence, option (b) is correct.
readily’, so option (a) is not the correct answer. Option
(c), ‘dull’, means ‘not interesting or exciting’. It forms 23. b The definition given in the sentence is that of a parasite,
the closest antonyms among the given options. Hence, hence option (b) is the correct answer.
option (c) is the correct answer.
24. a Option (b), ‘analysis’ means ‘the separation of a
17. a ‘Predilection’ means ‘an established preference for substance into its constituent elements, usually by
something’. Option (b) is clearly not the correct answer. chemical means, for the study and identification of
Option (c), ‘dissonance’ means ‘lack of agreement’ each component’. Hence, option (b) incorrect in the
and thus is not the correct answer. Option (d) is also given context. Option (c), ‘examination’ means ‘the act
not the correct answer. Option (a), ‘antipathy’, means of examining something closely (as for mistakes) as
‘hostility’ which is most nearly the opposite of well as examination also means ‘a set of questions or
predilection. Hence, option (a) is the correct answer. exercises evaluating skill or knowledge’; hence,
incorrect in the given context. Option (d), ‘estimate’
18. c ‘Impalpable’ means something that cannot be felt means ‘a judgement that you make without having the
physically. Option (c), ‘tangible’, means something that exact details or figures about the size’. Hence, it is
can be clearly seen to exist. Thus, option (c) is most incorrect in the given context. Option (a), ‘observation’
nearly the opposite of impalpable. Hence, option (c) is means ‘an act or instance of watching or noting
the correct answer. something for a scientific or other special purpose’.
Hence, option (a) is correct.
19. c ‘Parochial’ means ‘only concerned with small issues
that happen in your local area and not interested in 25. b Option (a), ‘pervasive’ means ‘spreading gradually to
more important things; of or relating to a church parish’. affect all parts of a place or thing’. Hence, it is not
Option (a), ‘dogmatic’, means ‘characterized by or given appropriate in the given context. Option (c),
to the expression of opinions very strongly or positively ‘autonomous’ means ‘able to govern itself or control its
as if they were facts’. Thus, option (a) is not the own affairs, existing or capable of existing
correct answer. Option (d), niggardly, means ‘unwilling independently’, which is wrong in the given context.
to be generous with money, time, etc.’ Thus, option (d) Option (d), ‘immutable’ means ‘that cannot be changed;
is also not the correct answer. Option (c), that will never change’, hence it is incorrect. Option
‘cosmopolitan’ means ‘containing people of different (b), ‘inseparable’ clearly explains the meaning that
types or from different countries, and influenced by although language, culture and personality are
their culture’. Thus, option (c) is most nearly the independent of each other, yet they cannot be
opposite of parochial. Hence, option (c) is the correct separated in functional form. Hence, option (b) is
answer. correct.

Previous Years
CLAT & AILET Papers Page 129
26. b Option (a), ‘into’ is used to denote ‘to the inside or 35. a Cleary P follows the opening statement, as ‘it’ in P
interior of, for example: went into the house’. Hence, it refers to “watching a child grow” mentioned in the
is incorrect. Options (c) and (d) are also inappropriate opening statement. Q builds the context further, by
usages. Option (c), ‘in’ is used to indicate inclusion explaining infant’s growth, referring to P, So, Q follows
within or occurrence during a period or limit of time. P. R provides support to the argument made in S. So, R
‘Era’ denotes a period of time. Hence, option (b) is follows S. Hence, option (a) is correct.
correct.
36. b 37. d 38. a
27. b Option b, ‘to’, is correct as preposition ‘to’ is used to
show that two things are attached or connected. 39. a 40. b 41. b
Hence, option (b) is the correct answer in the given
usage. 42. a 43. c 44. b

28. c Option (c) is correct as ‘that’ is used for referring to 45. b 46. b 47. b
somebody/something that has already been mentioned
or is already known about. Hence, here in the given 48. b 49. b 50. d
context, ‘that is’ the correct answer.
51. c 52. c 53. a
29. b Option (b) is correct as ‘across’ is used to represent
from one side to the other side. 54. d 55. b 56. c

30. a Option (a) , ‘run out’ is an intransitive verb, which is 57. a 58. d 59. c
used to denote ‘to come to an end, expire’. Hence,
option (a) is correct. 60. b 61. b 62. b

63. c 64. a 65. b


31. c Option (c), ‘called for’ is used to denote ‘to need
something’. In the given context, the boss needs an 66. c 67. a 68. d
explanation. Hence, option (c) is correct.
69. a 70. d
32. a ‘To put something up’ means ‘to raise something or put
it in a higher position’, hence option (b) is incorrect in 71. c A is liable because growing of poisonous trees is a
the given context. ‘To put somebody out’ means ‘to be dangerous thing and its projection into B’s land
upset or offended’; hence option (c) is incorrect in the amounts to escape. And hence, B should be made
given context. ‘To put aside’ means ‘to ignore or forget liable for the consequences i.e. death of B’s cattle,
something, usually a feeling or difference of opinion’. due to the escape of the poisonous leaves.
Thus, option (d) is incorrect. Option (a), ‘put off’ means
‘to hold back to a later time’. In the given context, the 72. d The Bank does not have any liability because the cash
subject delayed his departure- “He put off his departure and the cheques handed over to B by A was made in
for a week as his mother was not well.” Hence, option the capacity of a friend. B was not working as an
(a) is correct. employee of the Bank while taking them. Therefore,
the Bank cannot be made liable under the Master-
33. c P follows the opening sentence as it continues with servant/Principle-Agent relationship.
the subject- the bomber, explaining his actions. Q
further supports the facts presented in P. So, PQ forms 73. a The State here is liable to pay the compensation
a mandatory pair. “Looking at the partial result” in R because the constable was discharging his duties
refers to polls result mentioned in S. Hence, R follows under the sovereign functions of the Government and
S and SR forms another mandatory pair. Hence, option hence comes under the ambit of Master-Servant
(c) is correct. relationship making the Govt liable to pay for the
misappropriated goods.
34. d Cleary R follows the opening statement, building further
context on Lenin’s leadership on different parties. P 74. b The dogs of both the owners acted in concert and
further talks about Lenin’s performance; ‘his’ in P refers injured the sheep. As there is a common action, the
to Lenin. So, RP forms a mandatory pair. S follows Q owners of both the dogs are jointly and severally
as events in S are based on the facts mentioned in Q. liable and if one of them claims liability for one half of
Hence, option (d) is correct. the damage then the other is also liable to pay for the
other half.

Previous Years
Page 130 CLAT & AILET Papers
75. d The Defendant employees of Municipal Corporation 83. a A is guilty of Criminal Misappropriation. When he took
did not take requisite precaution and left the manhole the umbrella out of the possession of Z, he took it in
unattended and covered it merely with a canvass good faith believing it to be his own. Therefore, the act
shelter. There was a reasonable foresight that will not amount to theft because theft needs taking
someone might stumble over and fall into the manhole. property dishonestly out of the possession. But later,
Hence, the Defendants acted unreasonably in not after knowing that it did not belong to him, converted it
covering the manhole and are thus liable. for his own use, hence, misappropriating the same.

76. b The hospital in the present case is maintained by B. 84. c Though the correct answer marked in the question is
The fault was on the part of the Hospital in providing ‘(c)’, but the question is ambiguous.
with the faulty oxygen. The surgeon was acting only
as an agent of the Hospital in conducting the operation. 85. d In the present matter, A is a minor and hence does not
Therefore, B would be liable. have the capacity to contract. Therefore, the contract
entered by him with B is not a valid contract. And
77. a A is liable because he wrote a defamatory letter which since it is not a valid contract A is not liable to pay
was read by C as B did not know Urdu. Reading by a anything to B. Therefore, (d) is the right answer.
third party amounts to publication of those defamatory
remarks and therefore A is guilty of Defamation. 86. b 87. a 88. b 89. b 90. a 91. d

78. c P can claim compensation from D. It is so because the 92. b 93. b 94. d 95. b 96. b 97. a
dust from the brick grinding machine was causing
pollution in the atmosphere causing inconvenience for 98. b 99. c 100. d 101. c 102. c 103. d
P and his patients. It will amount to wrongful escape
of deleterious thing into another’s land interfering with 104. d 105. a
his use. Therefore, it is nuisance and D is liable to
compensate. 106. b The given statement provides an argument on why
undocumented evidence should be protected. Of all
79. b There is no false imprisonment in this case as there the given options, only option (b) gives a reason
was a just cause with B to stop A from leaving the against the said argument, telling us how
park. B was justified in asking Rs.100 to be paid at the undocumented evidence can be dangerous and should
time of exit. Hence, B is not guilty of false imprisonment. thus, not remain undocumented. Therefore, the correct
answer is option (b).
80. b There is no malicious prosecution in the present matter.
Malice or dishonest intention is an essential 107. a The author does not prove his point, but instead fixates
requirement for malicious prosecution. Here, the on the assumption that readers who prefer
Railway Co. did not have any wrong intention but adventurous novels to psychological novels are
based their act on the honest belief that the injuries inferior and while readers who have an opposite
were created by Doctor B. Hence, the Railway Co. is preference order are superior. He does not provide
not guilty of malicious prosecution. an explanation, and thus, option (a) is correct.

81. c A has committed the offence of attempt to murder. A 108. d If the patient suffers an ailment repeatedly for which
had the intention to commit crime and formed a series he is not being given the drug more than one time, it
of acts by purchasing poison and giving the same to could lead to problems, and thus denies the patient
the bearer to be served to B in a drink. The act was her right to health. This is the basic assumption made
interrupted as the glass dropped and hence, the actual by the author. Hence, the correct answer is option
commission of offence got limited to attempt. Hence, A (d).
is guilty of the offence of attempt to murder.
109. d Even though Vijay has acknowledged causing dents
82. d Though the correct answer marked in the question is on his own car due to his own recklessness, that
‘(d)’, but the question is ambiguous. does not necessarily mean he handled the office car
with the same laxity. This makes the argument
vulnerable and therefore, option (d) is the correct
answer.

Previous Years
CLAT & AILET Papers Page 131
110. d The given statement has a negation in the second For questions 115 to 120: After satisfy all the given condition,
sentence, which is absent in option (a). Thus, it cannot we will get the following table.
be the answer. Option (b), though has the negation,
Cookies
follows the logic which is opposite to that given in the Persons
statement. Thus, option (b) is incorrect as well. Option Chocolate Oatm eal Sugar Peanut Raisin
(c) gives a condition where a pencil is not necessary Ali ×
and the required function can be fulfilled otherwise, Bina √ ×
unlike that in the given statement. Thus, option (c) is Cham pak × √ ×
incorrect. Option (d) follows the logic presented by
the given statement, and hence is the correct answer.
Diya √ ×

111. d* Assumption I is not implicit as, from the given statement, 115. d Diya does not eat raisin cookies.
it cannot be inferred that "all poor people" have access
to jaggery at "reasonable prices". Assumption II also 116. d Only possible combination for Bina and Champak when
cannot be inferred. Just because sugar distribution Bina eats exactly 3 cookies is
through PDS has been discontinued, one cannot say Cookies
that the entire system has lost its utility. It is very likely Persons
Chocolate Oatmeal Sugar Peanut Raisin
that PDS is helping the poor by making various other
necessities available to them at reasonable prices. Bina √ × × √ √
The correct answer, hence, should be option (c), Champak × √ √ × ×
though the answer given by NLU-D is option (d). Option (a) : As Bina eats exactly three kinds of
cookies, therefore Champak cannot eats exactly three
112. a The statement talks of the mission of development kinds of cookies. (There are only five kinds of cookies
which will only be possible when country ‘X’ achieves and Bina and Champak do not eat the same type of
total literacy. Since a mission is a realistic possibility, cookies)
therefore the condition of total literacy also has to be Option (b) : It is not trure because each of these four
a realistic possibility. Therefore assumption I is implicit. people eats at least two kinds of cookies.
We do not know if development is possible without a Option (c) : If Ali eats oatmeal cookies, Diya cannot
mission or not, so assumption II is not implicit. Option eat oatmeal cookies.
(a) is the correct answer.
117. b If Ali and Diya both eat oatmeal cookies, then one of
113. b Though it is given in the statement that earth may lose the Bina and Champak cannot eat at least two types
many of its species, if it would lose all of its species of cookies.
cannot be commented on. Thus, assumption I is not
implicit. Since there is talk of saving many species by 118. c At most two people eat oatmeal cookies, therefore
studying them scientifically, therefore it is desirable none of the options (a), (b) and (d) are true.
and possible to study many animal species. Hence, ∴ Only chocolate chip and peanut butter cookies could
assumption II is implicit. Option (b) is the correct answer. be eaten by atleast three different people.

114. b* That herbs can be used to treat diseases in humans 119. d As no one eats both raisin cookies and sugar cookies,
does not, in any way, mean that diseases of animals therefore these two cookies is eaten by exactly two
cannot be treated through the use of herbs. Hence, different people.
assumption I is not implicit. Assumption II is also not
implicit since the given statement only talks about the 120. d One of the possible combination is
use of herbs for treating diseases in humans. It does
not say anything about the popularity of herbal Cookies
Persons
treatments. Therefore, the correct answer is option Chocolate Oatm eal Sugar Peanut Raisin
(c). However, the answer given by NLU-D is option Ali √ √ × ×
(b).
Bina √ √ × ×
Cham pak × √ √ ×
Diya √ √ ×
∴ Raisin cookies could be eaten by none of the people.

Previous Years
Page 132 CLAT & AILET Papers
For solutions: 121 to 123:

M r. A h uja M rs. A h u ja M r. G a n dh i M rs. G a nd hi


M r. M a lik M rs. M alik

S u nita R a hu l S u raj G un ja n R o m e sh
R a ni R o ha n S h ya m

A rjun S o ha n R u pali S on u R a vi

M arried cou p le C h ild M ale Fe m ale

121. d 122. c 123. a

Disclaimer: We have considered the gender of the person 129. c 5 13 29 61 125 2 53


according to their names (Indian context), otherwise the
question cannot be solved.
23 24 25 26 27
124. c Hence, the wrong number in the series is 120.
9 km
130. c 13 – 1 = 0
N o rth 23 – 1 = 7
6 km 6 km 33 – 1 = 26
W est S o uth 43 – 1 = 63
5 km
S E 53 – 1 = 124
Sta rting A E n d po in t E a st 63 – 1 = 215
p oint Hence, the wrong number in the series is 28.
Required distance = SA + AE = 5 + 9 = 14 km.
131. a 9 × 2 + 1 = 19
125. c I reached on Friday 2 days earlier than scheduled day, 19 × 2 + 2 = 40
so scheduled day was Sunday. If I had reached on 40 × 2 + 3 = 83
the following Wednesday, then I would have been late 83 × 2 + 4 = 170
by 3 days. 170 × 2 + 5 = 345
Hence, the wrong number in the series is 340.
126. a Independence Day was celebrated after [ 25 (March)
+ 30 (April) + 31 (May) + 30 (June) + 31 (July) + 15 132. c T E A M W O R K
(August)]
+1 +1 +1 +1 –1 –1 –1 –1
= 162 days = 23 × 7 + 1 = 23 weeks + 1 days
∴ Number of odd days = 1 U F B N V N Q J
Hence, X was born on Thursday.

127. a Kamala takes 40 minutes for each assignment and N B F U J Q N V


she has to complete 5 assignments, so she will take
40 × 5 = 200 minutes to complete all 5 assignments. Hence, PERSON will be coded as SFQMNR in that
Hence, she can start latest by 6.40 p.m. code language.

128. b 2 1 28 33 36 37 36

+7 +5 +3 +1 –1

–2 –2 –2 –2
Hence, the wrong number in the series is 35.

Previous Years
CLAT & AILET Papers Page 133
133. a In this code language every letter is coded with a 143. b Let the price of 1 Mango, 1 Orange and 1 Apple be M,
distinct symbol or integer as follow. O and A respectively. If Rs. x be the total amount with
B–5 Divya, then
A–$ 6M + 10O + 5A = x … (i)
S–3 3M + 5O + 4A = 0.6x … (ii)
K–% Multiply (ii) by 2 and then subtract (i) from it, we get
E–#
0.2x
T–1 A=
R–4 3
I–* Hence, percentage of total amount spend an apples
D–2
5A
Hence, SKIRT will be coded as 3%*41 in that code = × 100 = 33.33%.
language. x

134. c Movement of Eagle is like swooping, similarly the 144. c Let the provisions when 60 more men join last for x
movement of Duck is Waddle. days, then
300 × 600 × 75 = 360 × (600 – 100) × x
135. c Sum of position numbers of all the alphabets of the ⇒ x = 75.
words
APPLE : 1+16 +16 + 12 + 5 = 50 145. a Let the number of men originally employed be x, then
Similarly, ORANGE :15 + 18 + 1 + 14 + 7 + 5 = 60.
⇒ 16 × 14 = x × 5 + ( x − 7 ) × 3 +
(x − 7) × 5
136. b ‘ENTHUSIASTICALLY’ does not have an ‘M’ in it while 2
‘HELMINTH’ does. Thus, option (b) is the correct
⇒ x = 25.
answer.
80
137. c ‘CONCENTRATE’ does not have an ‘S’ in it while 146. a Ram after travelling 80 km (in = 2 hours), rests for
40
‘REASON’ does. Therefore, option (c) is the correct
20 minutes, therefore in the journey of 240 kms, he
answer.
will rest twice after travelling 80 kms and 160 kms.
∴ Total duration of journey = (2 × 3) hrs + (20 × 2) min
138. c ‘INTRANSIGENT’ does not have a ‘C’ in it while ‘TRACE’
= 6 hrs 40 min.
does. Therefore, option (c) is the correct answer.
147. c Let x litres per hour be the rate at which water comes
139. d ‘PERPETUATION’ does not have two ‘N’s’ in it while
out from the leak, then
‘PENANCE’ does. Therefore, option (d) is the correct
x × 8 = (x – 6) × 12
answer.
⇒ x =18
140. a ‘ESTRANGEMENT’ does not have an ‘L’ in it while Hence, capacity of the tank = 18 × 8 = 144 litres.
‘ENTANGLE’ does. Therefore, option (a) is the correct
answer. 148. b Let the x km be the distance from X to Z, then
distance travelled by B before they meet
141. c The numbers which are not divisible by any other = 27 + (27 – x).
number except 1 are prime numbers and there are 25
prime numbers from 2 to100. 27 + (27 − x ) x
Therefore, the numbers from 2 – 100 which are not ∴ =
prime (i.e. composite) = 99 – 25 = 74 7 5
Hence, the difference = 74 – 25 = 49. ⇒ x = 22.5.

142. b Three prime numbers lying between 47 and 74, whose 149. b Let the third number be x, then the first number is
191 (x – 4) and the second number is (x – 2). Now,
average is are (53, 67, 71) and (59, 61, 71). 3 × (x – 4) = 2 + 2x
3
⇒ x = 14.
Hence, the required sum = 67 + 71 = 138.
150. a Let Rs. c be Devendra’s cost price, then
(c + 200) × 0.2 = 200 – 100
⇒ c = 300.

Previous Years
Page 134 CLAT & AILET Papers
AILET Solutions 2014
1. c The passage begins by stating what felony-murder 6. d The passage states that "the rule has been abandoned
rule is, what its proponents have to say in its favour in England where it originated, abolished in India,
but what is the real strength of the argument of its severely restricted in Canada and a number of other
opponents is -the rule oft times is bizarre and unfair commonwealth countries, is unknown in continental
and so, owing to this, it is dying a slow but certain Europe, and abandoned in Michigan". So, we can say
death. So, overall the passage states that the rule at that the felony - murder rule has not been abandoned
times can be a faulty/unfair. Hence, option (c) is the in Canada but severely restricted there. Hence, option
correct answer. Option (b) is incorrect because (d) is the correct answer. Option (a) is incorrect only
although as per the passage it is true, it is not the in case the phrase 'is unknown in continental Europe'
passage's central theme. Although the answer key is understood to mean that the rule was abandoned
released with NLUD 2014 actual question paper says so long back in time in continental Europe that it is
that the answer is option (a), option (a) is rather unknown there. Otherwise, this question has two
incorrect because the passage nowhere puts down correct answers.
his own opinion regarding the rule.
7. b Refer to the following sentence from the passage, "In
2. c Look at the fourth sentence of the second paragraph. reality the real strength of the opponent's criticism
It says that courts developed the "felony-murder" rule stems from the bizarre and of times unfair results
in order to deter the use of deadly forces. Hence, achieved when the felony - murder rule is applied
option (c) is the correct answer. mechanically." From this, we can say that the author
perceives that at times the rule is bizarre and unfair
3. d Option (a) an be inferred by the first sentence of the and hence, option (b) is the correct answer. Option
third paragraph. It says that felony demonstrates a (d) is incorrect because the author does not state that
lack of concern for human life. It further says that equating the intent to commit a felony and the intent to
crime is murder either because of a conclusive commit murder is unfair but that the rule overall is
presumption of malice or by force of statutory definition. unfair at times.
Option (b) can be inferred from the first paragraph of
the passage, which states that the first formal 8. a 'Ambiguity' means something that does not have a
statement of the rule said that any felony that involved single clear meaning. 'Lucidity' means clearness of
killing was a murder and this rule was developed to thought. Hence, option (a) is the correct answer.
deter people from using deadly force. This implies that 'Basal' means fundamental.
option (c) can also be inferred. Option (d) is the answer
as it is one of the objections raised by the opponents 9. b 'Antidote' is something that corrects or improves the
of the rule. bad effects of something. 'Poison' is something (such
as an idea, emotion, or situation) that is very harmful
4. d Options (a) and (c) can be inferred from the third or unpleasant. Hence, option (b) is the correct answer.
paragraph.It says that unfair results were achieved 'Anodyne' is something that is not likely to offend or
when rules were applied mechanically. It further says upset anyone. 'Amity' refers to a feeling of friendship.
that defendants have been convicted even when the
killing was accidental. For example: if a person dies of 10. d 'Accumulation' means collection. 'Conglomeration' is a
heart attack 15-20 minutes after the robbery, the group or mixture of different things. 'Collagen' is a
defendant is prosecuted. The last paragraph says substance that occurs naturally in the bodies of people
that vicarious criminal liability should be restricted and and animals and is often put into creams and other
limited. From this statement, option (b) can be inferred. products that are sold to make a person's skin smoother
Only option (d) cannot be from the opponents of felony and less wrinkled. Hence, option (d) is the correct
- murder rule because it speaks in favour of felony - answer.
murder rule. Hence, option (d) is the answer.
11. a 'Incline' means to lean or slope. 'Acclivity' is an
5. b According to the passage, a defendant is charged of ascending slope. 'Gradient' is a place where ground
murder if death happens whether or not he intents to slopes up or down. 'Trenchant' refers to something
kill. In options (a), (c) and (d), the action of the that is very strong, clear and effective. Hence, option
defendant results in the death of innocent people. (a) is the correct answer.
Therefore, the defendant is liable to charge of murder.
Death does not occur in option (b) and hence, it is the 12. d 'Aphorism' is a short phrase that expresses a true or
answer. wise idea. 'Proverb' is a brief popular saying that
expresses a belief that is generally thought to be true.
Hence, option (d) is the correct answer. 'Prune' is to

Previous Years
CLAT & AILET Papers Page 135
reduce (something) by removing parts that are not saw away in S after cutting the branch in L1. Qshould
necessary or wanted. 'Wither' means to become weak follow S as the twigs brush the narrator's face in Q
or dry. 'Aphis' is an animal that constitutes the genus. after her husband pulls the branch down in S.SQ
should be followed by P because the branch falls in P
13. b 'Passe' refers to something that is no longer fashionable after it is pulled in S and Q. P, R and L6 form a mandatory
or popular. Hence, option (b) is the correct answer. sequence because the narrator's spectacles fall down
from the tree in P , which are picked by her husband
14. c 'Vituperation' refers to harsh or angry criticism. Harwardin R, followed by Harward asking the narrator,
'Malediction' means curse. Hence, option (c) is the his wife about her well-being.
correct answer.
22. c The correct sequence is SPRQ. L1 talks about a power
15. c 'Qualm' is a feeling of doubt or uncertainty about that is greater than mortals and S follows it by saying
whether you are doing the right thing. 'Misgiving' is a that this power helps in times of need. RQL6 is a
feeling that something might not be right or might not mandatory sequence because 'their' in Q refers to
work as planned. Hence, option (c) is the correct 'Industrial organizations' in R. L6 follows R because
answer. 'Concavity' is the quality or state of being the 'events' in L6 refer to 'construction and maintenance
concave. 'Amplitude' is a measurement that indicates of religious places' discussed in R. The sequence
the movement or vibration of something. RQL6 should follow P since P talks about good times,
which are discussed in R, Q and L6.
16. c The phrase 'sang froid' refers to the ability to stay
calm in difficult or dangerous situations. Hence, option 23. d The correct sequence is PQRS. P follows L1 because
(c) is the correct answer. L1 states that a thorough knowledge of path is
essential for success and P goes on to say that seniors
17. d The phrase 'esprit de corps' refers to a common spirit must show this path. Q and P are mandatory pairs
of comradeship, enthusiasm and devotion to a cause because 'they' in Q refer to 'seniors' in P. RS is a
among the members of a group. Hence, option mandatory pair because R talks about the advise senior
(d),unity, is the correct answer. should give and S deals with what he should not.

18. c Option (a) is logically incorrect because the problem 24. a Option (a) is incorrect. The correct sentence is 'She
with price discrimination cannot be that 'price did not have pass marks in mathematics.' 'Passing' in
discrimination is not always a bad thing'. Option (b) is options (b) and (d) means going by or past. 'Passing
incorrect because the change in the direction of the the parcel', as used in option (c), is a game in which
argument should be marked by a conjunction 'but' and something, such as a bowl, is passed to the other
not a comma. The grammatical error in option (d) is in players.
the phrase 'is not it is always'. Had it been 'is not that
it is always', it would have been correct. Only option 25. b 'Bolt', as used in option (a), refers to a long piece of
(c) is grammatically correct and hence, is the answer. cloth wound in a roll around a piece of cardboard. In
option (c), bolt refers to a long, narrow piece of metal
19. b There is a parallelism error in option (a). Since all the that you slide across the inside of a door or window in
them, 'contractors ', 'politicians'and 'bureaucracy' are order to lock it. In option (d) it means to dart off. The
the three subjects in the statement, either all of them usage of the word is logically inappropriate in option
or none of them should be preceded by the article (b). The person cannot make a bolt for the gate as the
'the'. Options (c) and (d) are also incorrect due to the first part of the sentence states that he could not
same reason. Also, there is a modifier error in options move. Hence, option (b) is the correct answer.
(a, (c) and (d). The correct phrase should be "costs
are artificially escalated by/with a large number of 26. d In option (a, fallout refers to the radioactive particles
devices". Only option (b) is grammatically correct and that are produced by a nuclear explosion and that fall
hence, is the answer. through the atmosphere. In options (b) and (c) it means
bad effect. Option (d) is grammatically incorrect as
20. a The correct sequence is RSQP. SQP is a mandatory the use of the word 'fallout' does not make sense.
sequence. S says that the choice is not between a
system (in which everybody gets what he deserves) 27. c Forests and other terrestrial carbon sinks cannot play
and individual share. Q follows it because it states 'tough' or 'biggest' role in preventing climate change.
what the choice is. P follows Q as P tells us why the They can only play a vital or an important role.
choice given in Q is important. Hence, option (a) is the Therefore, options (a) and (d) are incorrect. Option
correct answer. (b) is incorrect because 'important' will take the article
'an' and not 'a' before it. Hence, option (c) is the correct
21. b The correct sequence is SQPR. The passage begins answer.
by saying that the writer finished cutting through the
branch. S will follow L1 because the narrator pulls the

Previous Years
Page 136 CLAT & AILET Papers
28. d Option (c) is incorrect because destruction of forests 37. c Sri Lanka impeached its Chief Justice recently.
cannot have a positive effect onforests capacity to
absorb emissions. It will only adversely affect the 38. d The extradition treaty between Bangladesh and India
forests capacity. Moreover, 'severely' means very came into effect with the handing over of the
badly or unpleasantly. So, in this context, we cannot instruments of ratification of the much-awaited pact.
use 'increase' with it. Options (a) and (b) are incorrect The two countries will now be able to exchange
because 'affect' and 'diminish' are plural verbs but the convicts or under trials as and when required.
blank requires a singular verb. Only option (d) is
grammatically and logically correct and hence, is the 39. b Veeravalli Sundaram Sampath is the current Chief
answer. Election Commissioner (CEC) of the Election
Commission of India.
29. c Option (c) is the correct answer because the
destruction of forests will reduce/decrease the amount 40. a As per World Population Data 2013, Delhi and its
of forested land that can act as carbon sink. Other extended suburbs is the second most populous urban
options are incorrect because the destruction of sprawl in the world after Tokyo.
forests will not 'plague', 'shorten' or 'develop' forest
land in this context. 41. a South Korea is the latest country to launch a satellite
during January 2013 from its soil and join the club of
30. d* The correct answer is option (d), 'provide'. However, space-firing nations..
according to the answer key released by NLUD, the
answer for this question is option (b). When someone 42. c She won Malaysian open title 2013, beating her
or something offers something, you can either take it opponent from Singapore Juan Gu. This is Sindhu's
or turn it down. But if someone or something provides maiden Grand Prix Gold title.
you something, he is making something available to
you. You don't have an option of turning it down. The 43. d According to the Women's Tennis Association (WTA),
services provided by eco-system cannot be turned Serena Williams is the oldest female player to earn the
down. Therefore, the blank will take option (d) as the top ranking.
correct answer.
44. d There are 24 High Courts at the state and union territory
31. d Option (d) is the correct answer because the level of India which, together with the Supreme Court
rainforests will prevent the fertile soil from 'eroding'. of India at the national level, comprise the country's
Other options are incorrect because fertile soil cannot judicial system. Each High Court has jurisdiction over
be 'transformed', 'decoded' or erupted'. a state, a union territory or a group of states and union
territories.
32. d Options (a) and (b) are incorrect because they render
the sentence meaningless. 'Availability' will not take 45. a Ms. Angela Merkel, the Chancellor of Germany. The
the preposition 'to' but 'of' after it and therefore, it is Indira Gandhi Peace Prize or the Indira Gandhi Prize
incorrect. 'Access' means a way of getting near, at, for Peace, Disarmament and Development is the
or to something or someone. Hence, option (d) is the prestigious award accorded annually by India to
correct answer. individuals or organizations in recognition of creative
efforts toward promoting international peace,
33. b 'Source' means cause of something. Hence, option development and a new international economic order;
(b) is the correct answer. Options (a), (c) and (d) will ensuring that scientific discoveries are used for the
render the sentence incorrect. larger good of humanity, and enlarging the scope of
freedom.
34. a 'Apart' and 'also' will not take the word 'this' immediately
after them. Therefore, options (b) and (d) are incorrect. *46. () Lionel Messi and Cristiano Ronaldo are currently the
Option (c) is incorrect since 'in spite' always takes the only players to have won the European Golden Shoe
preposition 'of' after it. Only 'despite' fits in the meaning three times, with Messi being the only one to have
of the sentence and therefore, is the correct answer. thrice been the stand-alone winner.

35. a 'Nurture' is incorrect since the sentence says that the 47. d Bitcoin is a digital currency (also called crypto-
methods used by them are unsustainable. Only option currency) that is not backed by any country's central
(a) fits in the meaning of the sentence, which means bank or government. Bitcoins can be traded for goods
that economic pressures force communities and or services with vendors who accept Bitcoins as
governments to exploit forests in unsustainable ways. payment. In short, a digital currency in which
transactions can be performed without the need for a
36. c Aircraft carrier INS Vikramaditya inducted into Indian central bank.
Navy recently.

Previous Years
CLAT & AILET Papers Page 137
48. a Sunil Gavaskar was conferred the Col. C.K. Nayudu 60. d The main component of India's import items from China
Life Achievement Award in Cricket during 2012. is Telecom equipments and Machinery.

49. a V. Ramaswami was the Justice of the Supreme Court 61. b Tropic of Cancer divides India into Northern and
of India and the first judge against whom impeachment Southern parts.
proceedings were initiated in independent India. The
only other judge to face impeachment proceedings is 62. a India shares maximum length of the border with
Justice "Soumitra Sen" of Calcutta High Court, Bangladesh.
proceedings against whom were initiated in Rajya
Sabha on 17 August 2011. 63. c Arab Spring is known as Jasmine revolution.

50. b The maximum penalty that can be imposed by CIC/SIC 64. b The U.S. is not an ex-officio member of NATO but was
under the Right to Information Act, 2005 is Rs. 25,000. a part of founding members alongwith 12 other member
countries. The founding countries are Belgium, Canada,
51. b On January 1, 2013, internet has completed 30 years. Denmark, France, Iceland, Italy, Luxembourg, the
Netherlands, Norway, Portugal, the United Kingdom,
52. c 2013 has been declared as the "International Year for and the United States. In 1952, Greece and Turkey
Water Conservation". became members of the Alliance.

53. a The European Union (EU) is a politico-economic union 65. a Before becoming the Judge of Supreme Court of India
of 28 member states that are primarily located in Europe. V.R. Krishna was the minister of law, power, prisons,
The EU operates through a system of supranational irrigation and social welfare in the Government of
independent institutions and intergovernmental Kerala, he was instrumental in bringing about many
negotiated decisions by the member states. significant changes in these sectors. In 1973, he was
sworn in as a Judge of the Supreme Court of India; as
54. c The Rajya Sabha or Council of States is the upper of 2011, he is the last Supreme Court judge to have
house of the Parliament of India. Membership is limited previously served as a politician. He was conferred
to 250 members, 12 of whom are nominated by the with Padma Vibhushan in the year 1999.
President of India for their contributions to art, literature,
science, and social services. 66. b Inside the airbag is a gas generator containing a mixture
of NaN3, KNO3, and SiO2. Therefore the correct answer
55. d Rajiv Gandhi Khel Ratna - India's highest honour given is sodium azide(NaN3).
for achievement in sports. Arjuna Award - recognizes
outstanding achievement in National sports. 67. d White color represents high intensity and hence high
Dronacharya award- an award presented by the heat of radiation.
government of India for excellence in sports coaching.
Bharat Ratna is the highest civilian award of the 68. c Mark Elliot Zuckerberg (born May 14, 1984) is an
Republic of India and not the sports award. American computer programmer, Internet entrepreneur,
and philanthropist. He is best known as one of five
56. c Cloves is nothing but the dried flower buds of the co-founders of the social networking website
plant named Syzygium aromaticum. Facebook.

57. d The Sundarbans mangrove forest, one of the largest 69. b Shashi Kant Sharma is the Comptroller and Auditor
such forests in the world (140,000 ha), lies on the General of India. He succeeded Vinod Rai as the
delta of the Ganges, Brahmaputra and Meghna rivers supreme auditor of the Constitutional body on 22 May
on the Bay of Bengal. It is adjacent to the border of 2013.
India's Sundarbans World Heritage site inscribed in
1987. The site is intersected by a complex network of 70. a Mist is formed by condensation. Although mist and fog
tidal waterways, mudflats and small islands of salt- look like smoke, they are actually tiny drops of water
tolerant mangrove forests, and presents an excellent floating in the air. In fact, they are clouds that have
example of ongoing ecological processes. The area is formed at ground level.
known for its wide range of fauna, including 260 bird
species, the Bengal tiger and other threatened species 71. d Though the occupier of a premise is under an obligation
such as the estuarine crocodile and the Indian python. to take care of his invitees and visitors but he does not
owe the same duty towards thieves. Therefore
58. c Dr. B.R. Ambedkar was the first Law Minister of Radhika is not liable
Independent India.
72. c Vijay does not have easement rights over the property
59. b Taiwan is not a UN member country. of Namit. Therefore his shed protruding on Namit's
property is an act of trespass.

Previous Years
Page 138 CLAT & AILET Papers
73. b The chemist is liable because he could have foreseen 86. d Akash never had the intention to use criminal force
that the chemicals would harm Jyoti and still he lied against his brother. He further never knew that his act
about them of bursting a cracker was likely to get his brother
scared. There is no offence by Akash
74. b There is a clear case of defamation as there is a clear
intention of depicting Babloo Prasad as a murderer 87. a the principle herein talks about giving the same amount
even after he has been proven not guilty by the courts of punishment and penalty to a person as is applicable
of law in India. at the time of commission of the offence. Therefore
the boy should not be allowed the benefit of the
75. a Both are liable as the principle clearly mentions that Probation of offenders act. (AILET official answer
partners are jointly and severally liable for the wrongs key gives option 'B' as answer)
committed.
88. b Departmental inquiry was not a proceeding for the
76. b Though the wire snapped because of storms and offence of harassment but an in house method by the
winds but the wind and storm cannot be taken as an company to punish bad employees. Therefore Aditya
act of god which was unforeseeable and has been punished only once.
unpredictable. It is clear that B is liable.
89. b A is not guilty of murder because the death of B was
77. a As per the facts of the case, B was under the course not result of his stabbing B but the negligence of
of employment when he went to work in C's shed and Hospital doctors. He can at best be charged with the
threw a lit cigarette leading to the fire. Therefore A offence of attempt to murder.
being his employer is liable for his acts.
90. c The contract has only restrained the parties from
78. d It was the duty of the father to dispose of the air gun starting proceedings in Udaipur. Pinto is free to begin
in a such a manner that his son cannot use it again. proceedings in Jaipur. Thus this is not a total restriction
The facts state that the gun was merely taken away against approaching a tribunal for proceedings and
and kept at a place where it was easily accessible to therefore the clause is not void.
the minor. The father is therefore liable.
91. a Inviting someone for Dinner is just a social obligation
79. d The act of the doctor in providing consultation and and not a legally enforceable contract.
diagnosis clearly falls in the definition of service. He is
therefore liable to pay compensation. 92. c Dutt is not liable because he did not accept the offer
given by Bakshi but gave a counter offer which was
80. b The facts clearly illustrate that selling kidney, even not specific. Therefore there is no contract in the
though done on a person's own consent, is not a present case.
"thing done for a person's own benefit" as per the
principles given. Therefore Z is guilty 93. d the fact that the initial agreement between Sunita and
Neel was against public policy (law does not allow a
81. c A acted in good faith with to save the life of the child. woman or man to marry during the subsistence of a
He anyways did not have any other option. Therefore current marriage) makes the agreement void.
he is not guilty
94. b The object of the agreement in this case was to kill
82. b As given in the principle the soldier is bound by duty to Sunil's Uncle. Thus the object being unlawful, the
obey his senior as per law. Therefore he is not guilty. contract between Sunil and Anuj is void.

83. b X is guilty of contempt of court because his comment 95. c Every person is supposed to appear before a court of
was made in connection with a criminal proceeding law when summoned. Any consideration to do an act
which he knew was pending in a court of law. More which a person is bound to do by law is no
over, he has made categorical remarks on the judiciary consideration.
and questioned the veracity of the judicial process.
96. d The principle clearly states that a minor is not competent
84. a A did not have the dishonest intention to move B' to contract. Therefore Deep can neither enter into an
property. He under good faith believed that the agreement of agency with Mandeep nor a general
property of his own. He is therefore not guilty of theft. contract with any buyer.

85. d The act of keeping explosives in the garage can only 97. a It is clear from the facts that Mani wanted to deal only
amount to preparation for the crime. Manish has not with Pal as he had sent in his order to Pal only.
done any act after that to achieve his objective of Therefore Sam cannot accept the offer when it was
killing Nandini. There is no offence by Manish. not made to him in the first place. Therefore Sam cannot
recover in the suit.

Previous Years
CLAT & AILET Papers Page 139
98. d Judicial Standards Accountability Bill, 2010 aims to bring surface. Option (d) is incorrect because it is difficult
in standards for judicial accountability. It does not give to say that the rents would increase 'substantially'.
a definition of misbehavior.
109. b Option (a) talks of improvement of economy. This does
99. b It was held in the case of Novartis v. Union of India that not affect the argument as it does not assume a
the modification of well known cancer fighting drug is change in the state of economy. Option (c) defines
not patentable. 'survival' which again does not affect the argument.
Option (d) questions the existence of government
100. c Under the Criminal Law (Amendment) Act, 2013 stalking subsidies while the argument only talks about a cut
and voyeurism are non bailable offences. and not elimination of subsidies. Moreover, this option
will strengthen the argument. The correct answer is
101. a Uniform Civil Code is applicable in the state of Goa. option (b) which states that just because the art
groups have absorbed a decrease in subsidies, it is
102. c NOTA or none of the above was introduced in the not necessary that they will be able to absorb another
voting machine as result of a Decision of the Supreme cut in their subsidies.
Court
110. b According to the passage, doctors overtreat patients
103. b Part IV-A of the constitution contains 11 duties because they have an incentive for each procedure
performed. Option (a) has been ruled out because the
104. c The inter state council has been constituted on the involvement of doctors is not coincidental. Option (c)
recommendation of the Sarkaria Commission. is incorrect because the passage does not talk about
any'well-defined principles' while option (d) is incorrect
105. d Planning Commission is not a constitutional body. It because no reasons have been given for the choices
was created by an executive order of the Government made. The correct answer is option (b) which talks of
of India. the opportunity as well as the incentive offered by it.

106. d The argument given by Richa shows a conflict of the 111. b According to the venn diagram given below, option (b)
moral obligations of two individuals. Option (d) is the is the correct answer.
correct answer because this is the only option that
clearly shows two different moral obligations which
contradict each other.

107. a The argument made by the author is that prohibition of


alcohol increased the demand of alcohol. The increase S ky-
in demand of alcohol has been concluded from the B u ilding S cra pe r
Stru ctu re
rise in deaths due to alcohol-related diseases during
prohibition. Option (b) weakens this argument because
it says that the rise in the diseases is due to the alcohol
consumed before the prohibition. Option (c) also
weakens the argument because it states that the rise
in the death rate due to alcohol-related diseases has
been constant in the period when it was prohibited 112. d According to the venn diagram given below, option (d)
and when it was not. Option (d) has also been ruled is the correct answer.
out because it attributes the rise in death rate to the
decrease in availability of medical attention due to the B u cke t
social stigma attached to it. Option (a) is the correct
answer as this does not affect/weaken the argument. B in B a ske t

108. c According to the passage, a long-tern negative effect


of enacting rent-control ordinances is that it will bring
about a shortage of rental units. Hence, option (c) is
the correct answer. Option (a) is incorrect because 113. b Statement (1) is a judgment as it is an opinion. So, the
the passage states that there would be smaller rent other options are ruled out and option (b) is the correct
increases when rent-control ordinances are in effect. answer. Statement (2) is a fact as it can be checked
Option (b) is incorrect because the passage talks using data while statement (3) is a judgment due to the
about tenants of many municipalities having the power subjective nature of 'protected environment'. Statement
to enact or repeal ordinances but it does not state if (4) is an inference based on the previous two
these people would repeal ordinances passed 'as statements.
soon as' the negative effects of rent-control begin to

Previous Years
Page 140 CLAT & AILET Papers
114. b Statement (1) is an opinion of the author and hence, is 118. b Applying the condition given in the question, the colour
a judgment. Statement (2) is expressed like an of flag and pennant is given below.
observation and hence, is a fact. Statement (3) is an
inference based on the previous statements while No. Flag Pennant
statement (4) is again a fact that has been revealed
by a study. 1 Blue
2 White
115. d Applying the condition given in the question, the colour
of flag and pennant is given below. 3 Red White
4 Blue
No. Flag Pennant
5
1 White Green
If the fifth pennant is white then first cannot be white
2 Red Blue
or blue. Hence, it must be green.
3 White Green
4 Blue white 119. a Applying the condition given in the question, the colour
of flag and pennant is given below.
5 Red Blue
No. Flag Pennant

116. c Goindthrought each option we find that option (c) is 1 Blue White
not necessarily true as the colour of the first pennant 2 Red Blue
can be green also.
3 White Green
No. Flag Pennant 4 Red Blue
1 Red Green/white 5 White Green
2 Blue
3 White 120. c As there are two colours which are common in flag
4 Green and pennants (i.e. Blue and White). So, among the
first four flag two will be blue and other two will be
5 white. Hence the colour of the fifth flag must be red.

117. d Applying the condition given in the question, the colour 121. c the relation is shown below.
of flag and pennant is given below.
S ylveste r
No. Flag Pennant
1 white Green R o na ld R ita
2 Red Blue
3 white Green M ale
4 Red Blue N icky Ire ne Fe m ale
5 Blue White

122. * Grandmother of elder brother of father is great


grandmother. Which is not present in any option.

123. c As the paternal grandfather of the boy is the maternal


grandfather of Pushpa. Hence, they are cousin.

Previous Years
CLAT & AILET Papers Page 141
124. d The movement of Kiran is shown below 132. c Using statement I alone, rank of the four persons is as
follow:
N
Rank Person

1 00 W E 1 B
50
2 C
B a ck o f h ou se S 3 D
Hence, Kiran is now facing north. 4 A

125. d Each direction is rotaed by 135 degree in AWC. Hence Using statement II alone, rank of the four persons is as
north become southwest. follow:

N Rank Person
126. * 20 10
1 B
W E
30 30 2 C
3 D
S
10 4 A

The required distance = 302 + 302 Hence, either of the statement is sufficient.
= 30 2
133. b Both the statements are true but the reason behind
Which is not present in any option.
the generation of energy is the binding of the nucleon.
Hence, option (b) is the correct answer.
127. d The series is following the given pattern:
31 + 12 = 32
134. b Both the statements are true but (R) does not give the
32 + 22 = 36
correct explanation of (A). So, the correct answer is
36 + 32 = 45
option (b).
45 + 42 = 61
135. d Conclusion I does not follow because we cannot
128. c The given series is combination of two series
determine if a good profession is the reason behind
I. 1, 2, 3, 4, 5, 6
the law degree being one of the most wanted degrees.
II. 3, 5, 7, 9, 11, 13
Conclusion II also does not follow because the
Hence, the m × t term is 13.
statement mentions 'one of the most'. Moreover, even
The series is following the given pattern.
if the law degree was the most wanted degree,
conclusion II would not follow as each and every
129. a 300 – 22 = 296
youth might not want that the majority of youth wants.
296 – 32 = 287
287 – 42 = 271
136. b The statement clearly shows that use of unfair means
271 – 52 = 246
by students is undesirable. So, stringent actions must
246 – 62 = 210
be taken against them to discourage this activity. This
makes conclusion II valid. Conclusion I does not follow
130. d 12 + 3 = 15
because we cannot judge the reason behind students
15 + 4 = 19
using unfair means. So, the correct answer is option
19 + 5 = 24
(b).
24 + 6 = 30
30 + 7 = 37.
137. b Errata is a list of errors in a printed work that are
discovered after printing and are shown with its
131. d Using both the statement together
correction on a separate sheet. So, as flaws are related
to jewels, errata will be related to books. Manuscript
Score Catch taken Catch drop
will not have errata and it is not a printed work. Literature
Akash 50 x+1 x is a subject and prints will also have flows and not
Biplab 50 2 x+2 errata. Hence, option (b) is the correct answer.

Chirag 35 __ 138. a Inoculate is to vaccinate i.e. to make a person immune


against a disease. Similarly, exposure toughens a
as x ≥ 1 hence, Akash was man of the match.
person or material. So, the correct answer is option
(a).

Previous Years
Page 142 CLAT & AILET Papers
139. d Just as steel is used to make rails, alnico is used to 146.c If Ratio of two number is x : y, then sum of the number
make magnets. Alnico is an acronym of a family of iron must be divisible by x + y.
alloys. So, option (d) is the correct answer. Among the given option only option (c) is satisfying.

140. d A conscience stops you from doing anything wrong. 147.b Let the total work to be done is 60 units.
Similarly, police stops you from committing a crime. Unit of work done 11 am to 5 pm = 6 × 6 = 36.
Thus, option (d) is the correct answer. Remaining work = 24 units.
From 5 pm onwards the unit of work done in every
141. c Let the salary of Mohan be Rs.x. consecutive hours will be 7, 8, 9 and so on.
∴x × (1 − 0.15) × (1 − 0.3) = 2380 ⇒ x = 4000. Hence, in three hours after 5pm (at 8 pm) the remaining
work will get completed i.e. 7 + 8 + 9 = 24 units.
142. b Let the lowest score be x.
148.c The percentage of students who took both the subjects
∴ 80 × 10 = 81 ×8 + 92 + x ⇒ x= 60
= (72 + 44) – 100 = 16.

143.c Let the present age of Sunil and Sudip is 3x and 4x 60


the total number of students in the class = 375.
years respectively. 0.16
3x − 10 1 149.b Amar = 2 × Rohit and Rohit = 3 × Chanda
∴ = ⇒x=5 Amar = 6 × Chanda
4x − 10 2
42
Time taken by Amar = = 7min.
144.b When their capitals were interchanged, then A would 6
have received175% more than what he actually 150.c Let P = x
received it means in actual B received 175% more ∴ Q = x +2 and R = x + 4
than A. According to question,
Ratio of their investment of A to B = 1 : 2.75 = 4 : 11.
3 x = 2(x + 4) – 3 ⇒ x = 5
11 ∴ R = 5 + 4 =9.
∴ Capital of B = × 30000 = 22000.
4 + 11

145.a Let Aman invested his capital for x months.


4×3 1
∴ = ⇒ x = 8.
3× x 2

Previous Years
CLAT & AILET Papers Page 143
AILET Solutions 2015
1. d In the first paragraph, the author says that if a property 10. a ‘Bequeth’ refers to the action of leaving property to a
is private and not open to the general public, the owner person or other beneficiary by a will. Hence, the word
can restrict the freedom of speech. Options (a), (b) is very similar to ‘hand down’ or ‘pass on.’
and (c) talk about public places which are open to the
general public. 11. a In the context of the sentence, option (a) is the correct
answer. ‘Redundancy’ refers to something that is
2. c ‘Quasi-public’ property refers to seemingly public superfluous and unnecessary. So, any part of
property. Options (a), (b) and (d) refer to quasi-public message that is predictable or conventional can be
places. Option (c) is not a quasi-public property. Hence, considered unnecessary or redundant.
conflict should not arise in the situation mentioned in
option (c). Refer to the second paragraph. 12. c ‘Bedlam’, ‘Anarchy’ and ‘Insurrection’ refer to a period
of chaos or a violent uprising against a government or
3. d In the fourth sentence of the first paragraph, the author an authority. Anatomise means to ‘examine or analyse
mentions that if the property is private and not open to something in detail’. The word can also mean ‘to dissect
the general public, the owner may absolutely deny the a body’.
exercise of the right of free speech thereon. Hence, it
can be assumed that the nature of the character of 13. d ‘Miscreant’ refers to a person who has done an
the owner determines whether the owner will deny unlawful activity. ‘Defendant’ refers to an individual or
the freedom of speech or not. an institution accused in a court of law. ‘Accused’
relates to a person who has been charged with
something. Hence, all the three words refer to someone
4. a In the second last paragraph, the author mentions that
who is on the receiving side of law. Plaintiff is a person
the main purpose of the shopping malls is to serve the
who brings a case against another in a court of law.
people in the area and not the merchants, developers
Hence, plaintiff is a person who brings a charge and
or the architects. Therefore, it can be inferred that the
is not on the receiving side of the law. Therefore,
author believes shopping malls in America should be
option (d) is the correct answer.
in the service of the people who frequent them.
****However, option (a) can also be the answer.
5. b In the third paragraph, the author mentions that in 1675,
‘Accused’, ‘Defendant’ and ‘Plaintiff’ refer to persons
Lord Chief Justice Hale wrote about private property
who have a close association with a court of law.
ceasing to be private when it is affected with a public However, ‘miscreant’ is a person who commits a crime.
interest. He is not associated with a court of law unless caught.

6. a Refer to the last paragraph of the passage. The author 14. c A subpoena is a writ ordering a person to attend a
quotes Justice Black who said in 1945 that the courts court. On the other hand, extortion is the practice of
must favour First Amendment rights over property rights obtaining something, especially money, through force
if the two come into conflict. or threats. Assault is to make a physical attack on
someone. Battery refers to a fortified emplacement
7. d An ‘Alphanumeric’ code refers to both letters and for heavy guns. Hence, extortion, assault and battery
numerals. relate to threat and attack while subpoena is a writ ?
a legal term.
8. b An ‘oligarchy’ refers to a small group of people having
control of a country or organisation. 15. c Juror is a member of a jury while the other options
refer to the position of a lawyer.
9. c ‘Perestroika’ (used in erstwhile USSR) refers to the
policy or practice of restructuring or reforming the 16. b ‘Alpha and Omega’ refer to the beginning and the end
economic and political system. First proposed by (used by Christians as a title for Jesus).
Leonid Brezhnev in 1979 and actively promoted by
Mikhail Gorbachev, perestroika originally referred to 17. a ‘Ante-diluvium’ refers to the period before the biblical
increased automation and labour efficiency, but later, flood, e.g., ‘antediluvian man’. It can also refer to a
came to entail greater awareness of economic markets very old time period.
and the ending of central planning. Hence, ‘opening
up’ can be considered to be a definition of ‘Perestroika.’ 18. b ‘To play truant’ refers to the act of remaining absent
from work without an approval. It can also mean to
run away from work without permission.

Previous Years
Page 144 CLAT & AILET Papers
19. b ‘To see red’ is to become very angry. 29. c Losses can only be ‘incurred.’ Furthermore, it is a
common practice to outsource the management of a
20. d ‘To flog a dead horse’ is to revive interest in a topic department in order to counter the losses. Hence,
which is out of vogue or date. It can also refer to a option (c) is the answer.
futile endeavor.
30. d ‘Succinct’ refers to something briefly and clearly stated.
21. d Statement P introduces the topic. It says that Jack ‘Deletion’ of paragraphs can make the prose more
arrived in town for grandpa’s funeral. Statement R ‘succinct.’ Hence, option (d) is the answer.
says, “he rode into the yard on his motorbike …” The
‘he’ in statement R refers to Jack in statement P. Hence, 31. c ‘Circumlocutious’ means explaining something in a
R follows P. Statement Q follows next since it refers to roundabout and unnecessary wordy manner. ‘Direct’
the demise of the old judge who is the grandfather of can be considered to the opposite of ‘circumlocutious.’
Jack. Statement S comes last since the statement Both the words when used together in the sentence
refers to his presence during the funeral service. makes sense.

22. c Statement P introduces the topic. Statement Q follows 32. c ‘Paradigm’ means ‘pattern’. Research pattern in
statement P. The ‘he’ in statement P actually refers to contemporary neurosciences is devoted to explaining
Manish. Statement S is followed by statement R the maps of human consciousness. ‘Elucidate’ means
because statement R speaks of the camp which was ‘to explain.’ ‘Reconnoitring’ refers to military
visited by Manish and his grandparents. observation. ‘Enunciate’ means ‘to pronounce clearly.
‘Obfuscate’ means to make something unintelligible.
23. b Statement P introduces the topic. Statement S talks None of the other options other than ‘elucidate’ is making
about the narrator’s background which logically any sense in the context of the sentence.
follows statement P. Statement Q is a continuation of
statement S. The transition word ‘Moreover’ hints 33. d You don’t write reports or stories or books for tools,
towards that. Statement T talks about the Western but ‘obituaries’ ? yes, as tools do get obsolete. Also
world’s value system and statement R further talks ‘practices’ do not wither or trade or die away, but they
about the values of the Western world. Hence, do fade away with time.
statement R comes last.
34. c Option (a) with ‘wird’ as an option can be removed
24. b Statement Q introduces the topic. Statement S and similarly, option (d) with ‘gloomy.’ They are both
describes the violin teacher Georges Enesco. using words that are not first-priority as they are
Statement P is an advice by Georges Enesco and somewhat informal. Out of the other choices, ‘activity’
Statement R is the reaction of the narrator to this advice. in option (b) is not qualified as ‘moving’ (emotional).
Option (c) fits in the best and is the best answer.
25. a Statement S introduces the topic and reflects the
requirement of a person who can clean. Statement P 35. d Option (a) can be easily eliminated since ‘being
follows statement S. Statement P portrays the subordinate’ and ‘boasting’ of it do not go together.
requirement of the narrator while Statement R reflects Option (c) is incorrect because ‘intellectuals’
the narrator’s guilt when she asks someone to clean (individuals) being ancestors to societies (collectively)
the house. Hence, statements P and R are a pair. is incorrect. Also present Indian intellectuals cannot
Statement Q comes last because it sums up the possibly be ancestors either. Option (b) is incorrect
paragraph. because ‘intellectual cliques’ is odd especially since
‘cliques’ is used in a somewhat negative sense. Option
26. d Let’s fill in the second blank space first. ‘Durable’ in (d) is correct.
option (c) does not make any sense because a person
cannot be durable. A person cannot be ‘upfront’. Hence, 36. c Rajasthan became the first State in the country to fix a
option (b) is out. The qualities mentioned by the narrator minimum educational qualification for contesting
are straight forward and honest. Taking cue from these elections to the Panchayati Raj Institutions. The State
qualities, the only possible option ‘perceptive.’ Assembly passed Rajasthan Panchayati Raj
(amendment) Bill, 2015, which makes Class VIII pass
27. b ‘Quality of interactions’ can ‘improve’ work. The other mandatory for the post of sarpanch except in tribal
options cannot be true. reserved areas, where the minimum qualification is
Class V and Class X for Zila Parishad or Panchayat
28. a Genetic engineering in humans is certainly a mode of Samiti elections.
treatment. ‘Foster’ means to ‘encourage the
development of something.’ The other options are
inappropriate in the contaxt of the sentence.

Previous Years
CLAT & AILET Papers Page 145
37. b China was selected to head the Partnership in 49. b The liver is the only human internal organ capable of
Population and Development (PPD), an inter- natural regeneration of lost tissue; as little as 25% of
governmental body promoting South-to-South a liver can regenerate into a whole liver.
collaboration in the fields of reproductive health,
population and development. 50. c Kaya kalp the innovative Council proposed in the
Railway Budget 2015 to promote innovation.
38. c Prime Minister of India has launched the Give it Up
campaign for voluntarily giving up LPG subsidy. 51. b Lysosomes are called suicide sacks. They are
Through this campaign he appealed to well-to-do produced by the Golgi body. They consist of a single
sections of Indian society to voluntarily give up LPG membrane surrounding powerful digestive enzymes.
subsidy, so that the benefit of the same could be
It acts as the “garbage disposal” of the cell by breaking
shared more widely with poor sections of the society.
down cell components that are no longer needed as
well as molecules or even bacteria that are ingested
39. c Bollywood actress Kangana Ranaut has bagged the
by the cell.
National award for Best Actress for her performance
in Vikas Bahl’s ‘Queen’, which was announced on
March 24, 2015. 52. c India has improved its ranking from 140 to 136 in the
World Press Freedom Index (WPFI) 2015, released by
40. b Simona Halep is from Romania. Reporters Without Borders (RWB).

41. b Shashi Kapoor got the prestigious Dada Saheb Phalke 53. a Yaduveer Gopal Raj Urs has been announced as new
award for contribution to Indian Cinema for the year heir of Mysore Royal Family. Yaduveer Gopal Raj Urs
2014. is the son of Leela Tripurasundari Devi, daughter of
Srikantadutta Wadiyar’s eldest sister Gayatri Devi.
42. a The Bandung Conference was a meeting of Asian
and African states, which took place in 1955 in 54. a Pawan Kumar Chamling is often called the “Greenest
Bandung, Indonesia. The conference was organised Chief Minister of India”.
by Indonesia, Burma, Pakistan, Ceylon (Sri Lanka),
and India. The conference’s stated aims were to 55. c Fertile Crescent is an arc-shaped important event
promote Afro-Asian economic and cultural cooperation which covers large parts of Middle East. The country
and to oppose colonialism or neocolonialism by any in which it falls is Israel, Lebanon, Jordan, Syria and
nation. The conference was an important step toward Iraq.
the Non-Aligned Movement.
56. b ICICI Bank, India’s largest private sector bank, launched
43. c The government has roped in Amitabh Bachchan as ‘Pockets’, India’s first digital bank for the youth.
brand ambassador for its campaign against hepatitis
B, a critical public health problem facing India.
57. d Bengaluru-based Ricky Kej’s collaborated album Winds
of Samsara won the Best New Age Album trophy at
44. a The world’s first hydrogen-powered tramcar has rolled
the 57th Annual Grammy Awards.
off the assembly line in China’s eastern Shandong
Province.
58. a Football legend David Beckham and UNICEF has
45. d “The President in his capacity as the Visitor of Visa launched new protection fund ‘7’ to protect the world’s
Bharati (Acharya) has appointed Prime Minister most vulnerable youngsters. The fund, named after
Narendra Modi as the Chancellor of Viswa Bharati for Beckham’s lucky number on his England and
a term of three years. Manchester United shirts. The fund will focus on
children at risk of violence, abuse or disease.
46. a President Barack Obama and first lady Michelle Obama
put their weight behind Let Girls Learn, an initiative to 59. c According to the latest data Maharashtra has attracted
help girls around the world attend secondary school maximum Foreign Direct Investment (FDl) in the country.
and complete their education.
60. a The BRICS bank, aimed at funding infrastructure
47. b The incumbent Prime Minister, Benjamin Netanyahu’s projects in developing nations, will be based in
Likud party has won the Israel election with Likud Shanghai, China, and India will preside over its
party picking up the highest number of votes. operations for the first six years, followed by five-
year terms for Brazil and then Russia, leaders of the
48. c Real Effect Exchange Rate (REER) is the weighted five-country group announced at the 6th BRICS -
average of a country’s currency relative to an index Brazil, Russia, India, China and South Africa - summit.
or basket of other major currencies adjusted for the
effects of inflation.

Previous Years
Page 146 CLAT & AILET Papers
61. d Madhya Pradesh won the 10th National Award for particular person is sufficient to cause the offence.
excellent work in Mahatma Gandhi National Rural Further, the principle is limited and does not deal with
Employment Guarantee Act, 2005 (MGNREGA) in any exceptions. Thus, we cannot presume any
February 2015. exceptions or defenses unless specifically mentioned
in the question.
62. b Kerala Finance Minister K M Mani is appointed as the
Chairman of the Empowered Committee of State 72. b UCIL would be liable for all the damages as mandated
Finance Ministers on Goods and Services Tax (GST). by the principle. Here, escape of dangerous substance
has occurred and authorities bore the expenses for
63. b Decisive Storm, is the international war against Houthi decontaminating the substance to avert any further
militias in Yemen, is currently under way. The loss of lives and property. It would not matter what
international alliance led by Saudi Arabia is currently precautions they took, UCIL would be responsible for
fighting against the Houthi militants in Yemen. any damage caused as a result of the gas.

64. d President Maithripala Sirisena was elected as the new 73. d Satwik will not be liable because Prateek took the
President of Sri Lanka in the presidential election held money from Abbas only after termination of the
on January 8, 2015. partnership which is clearly mentioned in the facts.The
partnership ended right after the movie bombed, and
65. c Election Commissioner Nasim Zaidi became the new since at the time of borrowing the money from Abbas,
Chief Election Commissioner (CEC) of India in April, the partnership was no longer in existence, Satwik
2015. cannot be sued for the amount.

66. a Minor planet was named after India’s chess legend 74. b As per the official answer key, option A is the correct
and first grandmaster Vishwanathan Anand by the answer. According to us, the correct answer should
Minor Planet Center. His name was given to the minor be option B. This question also featured in the CLAT
planet which was previously identified as planet 4538. 2015 paper.
Now the planet will be called as 4538 Vishyanand and Krish will not be liable because he cannot foresee that
is located somewhere between the Mars and Jupiter. a pregnant lady will suffer a nervous shock by
witnessing a road accident that had resulted in the
67. d Aditi Arya was crowned fbb Femina Miss India World death of a person. Krish was under an obligation to
2015. take care only of those people he could reasonably
have foreseen as being affected by his act. He could
68. a First Prime Minister (PM) of Singapore Lee Kuan Yew reasonably have foreseen the harm he could cause
passed away on 23 March 2015. He was 91. Mr. Lee to the cyclist, but not to Lekha since the damage to her
is described as the ‘founding father’ and ‘architect’ of was too remote.
modern Singapore. He is also considered as pioneer
for transforming the island country from a small port 75. d Since Lalit tolerated the passers-by, they can be
city into a wealthy global hub after its independence considered visitors and therefore Lalit has a duty to
from United Kingdom and separation from Malaysia. watch out for them. Lalit will be liable despite the fact
that he had put up a board “Trespassers will be
69. c Election Commission (EC) on has announced to launch prosecuted” because he did not strictly prohibit the
Electoral Roll Authentication Mission (ERAM) and entry of outsiders on his premises and continued to
Purification Drive in order to weed out bogus voters allow people to use the same.
from the voters list.
76. c PUL will be liable for damages towards all the persons
70. a Anthony John “Tony” Abbott is the 28th and current travelling in the bus irrespective of the fact that they
Prime Minister of Australia. He has held this position are employee or not. The principle says master would
since 2013, and been Leader of the Liberal Party since be liable for the act of servant done in the course of
2009. Abbott is the Member of Parliament representing employment. Hence, the driver has caused the accident
the Sydney-based Division of Warringah. due to his negligent driving and he will be considered
in course of employment because he was driving the
71. a On a straightforward application of the principle, we bus to pick up the employees of the company. Further,
find that regardless of Kiara’s intent, the real Ricky did since driving the bus carefully was within the purview
get defamed because the website was injurious to of the driver’s duties, the master will be liable for
his reputation and put in the public domain. Kiara will whatever happens during the course of the discharge
be liable because intention is irrelevant in defamation. of that duty. PUL is, therefore, liable.
Mere publication of injurious statement about a

Previous Years
CLAT & AILET Papers Page 147
77. a The court is perfectly within its rights to strike down a religion. According to the principle, the reservations
law if it violates fundamental rights as per article 13 of are invalid.
the Constitution; but the task of making new laws is
that of the legislature alone. 85. a The principle states that the partners would be liable
for something that one of them does lawfully in the
78. b This question also featured in the CLAT 2015 paper. course of employment. Bribery isn’t lawful, and thus
Here, Sohan can refuse to sell the car because he they stand absolved of liability.
quoted the lowest price of the car asked by Ram. 86. b The contract was for a specific quality of rice as
Quoting the price is always an invitation to offer, and identified by both the contracting parties and that had
not an offer which means that the Parties are at the gotten destroyed, Surender stands discharged.
stage of negotiation only. It was now down to Ram to
send an offer saying that he’d like to buy the car for 20 87. a What makes his intention dishonest is that he knew
lakhs, and it would then have been down to Sohan to that the promissory note belonged to Z and decided to
accept or reject it. use it for his own purposes despite the knowledge.

79. a As per the official answer key, option C is the correct 88. d This question also featured in the CLAT 2015 paper.
answer. According to us, the correct answer should Options A, B and C do not match the criteria as
be option A. This question also featured in the CLAT mandated by the principle. The principle specifies
2015 paper. normal and reasonable use, which isn’t evaluated by
The focus of the principle is on the competence to any of the options, hence D.
contract and not on the burden of proof. The principle
clearly states that the person should be of sound mind 89. b A had the knowledge that firing into the bush could
at the time of entering into the contract, and says cause severe harm or death to Z, so he is guilty of
nothing about burden of proof. Using that, the answer culpable homicide. B had neither knowledge nor
should be A since Mr. X was not of a sound state of intention, so he wouldn’t be liable for any offence.
mind at the time of entering into the contract.
90. c It would be neither kidnapping nor abduction, since
80. c This question also featured in the CLAT 2015 paper. slavery per se is illegal. We don’t know anything as to
Since ignorance of law cannot be an excuse, it is the willingness of the slave, and he was definitely not
presumed that X knew of the law regarding declaration under his owner’s lawful guardianship.
of gold. There appears to be a typographical mistake
in the facts. X was arrested on 22ndNovember and 91. c Regardless of the fact that she was cheated first,
changes in the law were notified on 24th November as Roshni’s act of passing on a counterfeit note with the
stated in the facts while option (A) says “…he had knowledge that it was counterfeit is punishable.
actually no knowledge about the new notification
issued two days ago”. Assuming 20th November as 92. b Neel may be liable for possession of illegal arms but
the date of notification of law, the answer will be C. not for mischief because he has not destroyed any
public property.
81. b This question also featured in the CLAT 2015 paper.
Removing a car from the garage amounts to direct 93. a Here, Accused can take the right of private defense
physical interference with goods, regardless of the as consent of the 15 year old girl for sexual intercourse
innocence of X’s intention. Hence it would amount to with deceased is immaterial.
trespass as per the principle.
94. b Akshay was not facing any danger of an imminent
82. d Since Ramesh already has a son, he cannot adopt nature such as to justify usage of all necessary means
another. Ramesh’s sister, not being the legal guardian to avert it before taking recourse to public authorities.
of Rajat, could not have consented to give Rajat up for
adoption. 95. a Preparation would’ve stopped at buying the explosives,
but actually planting them in B’s house amounts to an
83. b The ivory combs were not dishonestly taken from the attempt since barring any external interference, B
possession of their previous owner; they had been would’ve died because of the explosives. So he can
lying around the house. Raja will not be liable for theft be held guilty for attempt.
because he did not take any property from the
possession of the owner or without his consent as 96. c This question also featured in the CLAT 2015 paper.
he found the article upon entering the apartment. The Supreme Court of India has set up a Social Justice
Bench, which comprises of two judges devoted to
84. c The principle specifies that reservations can be made the delivery of speedy justice in a range of social
on grounds of caste, class or sex but doesn’t mention issues related to the downtrodden and socially

Previous Years
Page 148 CLAT & AILET Papers
marginalised groups. It’s a brainchild of Chief Justice and Bulgaria), Asia (India, Indonesia, Jordan, Pakistan,
of India H.L. Dattu and the Bench is headed by Justice Saudi Arabia and United Arab Emirates), Latin America
Madan B. Lokur and also has Justice U.U. Lalit. The (Argentina, Brazil and Colombia) and Oceania (New
purpose behind constituting the new Bench was to Zealand and Australia)
streamline cases highlighting social issues before one
court and thus facilitate the Supreme Court’s monitoring 100. a T S R Subramanian Committee was constituted to
and review of the government’s action in such review environmental laws in India. The committee
cases.The social justice bench has been set up to has submitted its recommendations to the government
ensure three-dimensional justice, as provided for in recently. The report has been criticized for failing to
the Preamble. This Bench would function from 12th make the environmental regulation more participatory
December 2014 and in order to ensure that these and democratic.
matters are monitored on regular basis, will continue
to sit on every working Friday at 2.00 p.m. 101. c Thailand has passed a law that bans foreigners from
seeking surrogacy services to end the “rent-a-womb”
97. c Former Lok Sabha speaker Meira Kumar is the author industry that made the South-east Asian country a top
of “Indian Parliamentary Diplomacy – Speaker’s destination for fertility tourism. Several surrogacy
Perspective” and this book was released at a function scandals were reported in Thailand last year, including
in RashtrapatiBhavan on March 19th, 2015. The allegations that an Australian couple had abandoned
President of India, Pranab Mukherjee received the 1st their Down’s Syndrome baby with his Thai birth mother
copy of the book from Sumitra Mahajan, Speaker, taking only his healthy twin sister back to Australia
LokSabha who formally released it. The book features with them. Another case involved a Japanese man
an account of the parliamentary diplomacy and its role who fathered at least 16 babies using Thai surrogates
in furthering foreign policy and strengthening in what local Thai media called the “baby factory”. The
international relations. It is also a collection of select law makes commercial surrogacy a crime and aims to
speeches and reports by Kumar during her stint as stop Thai women’s wombs from becoming the world’s
Lok Sabha speaker and as leader of Indian womb. This law bans foreign couples from coming to
parliamentary delegations at several international fora. Thailand to seek commercial surrogacy services and
stipulates that surrogate mothers must be Thai and
98. b Palestine became the 123rd member of the International over 25. The important part is if the couple seeking
Criminal Court.123 countries are States Parties to the surrogacy services is Thai or the couple is mixed-
Rome Statute of the International Criminal Court. Out of race, they can find a Thai woman to be their surrogate
them 34 are African States, 19 are Asia-Pacific States, provided she is over 25 and the violation of the law
18 are from Eastern Europe, 27 are from Latin American carries a “severe prison sentence”.
and Caribbean States, and 25 are from Western
European and other States. 102. b The third National Lok Adalat was organised on 14
February 2015 throughout India involving the Supreme
99. a Group of Friends Against Terrorism is a United Nations Court, High Courts, Districts Courts and taluka-level
initiative launched to combat terrorism. The group was courts, except in Assam and Uttar Pradesh. It was
launched at the initiative of Morocco, and it was organised under the patronage of Chief Justice of
presided over by Morocco’s Permanent India (CJI) H L Dattu and the chairmanship of Supreme
Representative to the UN Ambassador Omar Hitale. Court Judge Justice T S Thakur. Nearly 56,000 cases
Created at the initiative of Morocco, the group consists were disposed of by the third National Lok Adalat in a
of thirty countries. Group of Friends Against Terrorism single day. It also saw disbursal of Rs 265 crore as
will aim, among others; to create synergies between claims towards final settlement in bank recovery and
all initiatives implemented within the UN, promoting cheques bounce cases. Third Lok Adalat was
international cooperation in combating terrorism, share organised by the National Legal Service Authority
and exchange good practices in the fight against this (NALSA) to dispose cases related to bank recovery,
scourge that threatens international peace and cheque bounce cases, particularly under Section 138
security. India and Pakistan, as well as the five of the Negotiable Instruments Act, 1881. Several banks,
permanent members of the Security Council are among financial institutions and others had participated in it.
the 30 nations who have joined hands. In addition to First National Lok Adalat was held in November 2013
China, France, Russia, UK and the US, the group has and had settled a record 71.50 lakh cases pending in
representation from the five continents and is various courts. The second National Lok Adalat was
composed of African countries (Morocco, Ethiopia, held in December 2014 and had disposed of over 1.25
Egypt, Chad, Kenya, Senegal and South crore cases.
Africa),Western Europe (Spain, Germany, Norway
and Turkey), from Eastern Europe (Czech Republic

Previous Years
CLAT & AILET Papers Page 149
103. a In State of Uttarkhand v. Bhim Singh &Anr., it was held and (d) do not mention any timeframe. Thus, it is
that the demand of dowry can be made at any time, impossible to infer whether the cited measures have
even post marriage been instrumental in curbing the crime figures.

104. b The NGT has been established for speedy disposal of 109. d Option (d) cannot be calculated. We do not know what
cases relating to environmental protection and score and GPA are being referred to. Less than 1200
conservation of forests and other natural resources. and less than 2.5 can mean any number below the
The National Green Tribunal (NGT) banned all diesel cited figures. Hence, the information given is
vehicles over 10 years old and all petrol vehicles over insufficient and therefore, inconsistent with the
15 years old from plying on Delhi roads. provided data. The data do not provide any benchmark
or cut-off mark. Furthermore, the data given in option
105. b The Shreya Singhal case has been hailed as a (d) is contradicting the trend mentioned in the
welcome development in the Indian free speech paragraph. A student with a score of less than 1200
jurisprudence. It was the outcome of a petition following should have a GPA of more than 2.5 to be admitted.
the misuse of Section 66 A by political parties for The other options offer specific data which can be
suppressing valid expression of dissent on social calculated and are in line with the given trend.
media. It also has implications for intermediary liability.
The Supreme Court declared Section 66A of Information 110. b Option (b) is a wrong assumption. It is incorrect to
Technology Act as unconstitutional and struck it down. infer that Newfoundland reflects the entire scenario
This section had been widely misused by police in in and around America. The passage does not discuss
various states to arrest innocent persons for posting any factor other than over fishing and pollution. Hence,
critical comments about social and political issues and option (a) cannot be the answer. Option (c) is
political leaders on social networking sites. The court categorically stated in the second sentence while
said such a law hit at the root of liberty and freedom of option (d) is a possibility even if we take into account
expression, two cardinal pillars of democracy. the declining number of fishes.

106. a Option (a) sums up the entire essence of the passage. 111. d Option (d) if assumed, is necessary for the author’s
The passage deals with the different stages of argument. The author believes that literature or art
emotional and psychological development. Option (a) cannot influence human behavior. Had it been the case,
offers the perfect conclusion to the discussion. “These Romeo and Juliet and other works would have been
developments” in option (a) indicate that certain banned. Option (a) if assumed negates the argument.
developments in the passage are being referred to. Option (a) is exactly opposite of what the author says.
The “developments” are mentioned in the passage. Option (b) cannot be inferred because the author does
Option (b) introduces additional information. However, not talk about human behavior being influenced by
it does not conclude the passage. Option (c) cannot literature or music per se. Instead, he states that
be inferred. Similarly, option (d) does not offer the literature or music cannot be held responsible if a
perfect ending to the discussion. person commits suicide. Option (c) appears to be too
generic, wide in scope and out of context.
107. d Option (d) does not support the above premise. If
there were active watchdog groups, hate crimes would 112. b Option (b) is the answer because a few anthropologists
certainly have been lesser and there wouldn’t have believe that their findings have been proved. It means
been any discrepancy in the figures. Options (a), (b) other anthropologists have different views about the
and (c) offer an explanation. If the crime rate in findings. Thus, it is not very clear at the moment
neighboring states has increased, there would be a whether the findings positively support or refute the
discrepancy between the state and county figures. anthropologists’ conclusions. Option (a) cannot be true
Therefore, option (a) can be rejected. Similarly, if there because the evidence is not contradicting the
was an anti-immigration sentiment going on, anthropologists’ conclusions. Rather, the evidence has
discrepancy between the figures of state and county encouraged a small section of anthropologists to
would be evident. Hence, option (b) can be ruled out. believe that their theories have been proved. Option
If people do not trust law enforcement agencies, (c) cannot be the answer because some
reports of crimes would be lesser and this can lead to anthropologists believe that their conclusions have
a disparity in crime figures between states and been entirely proved by the archaeologists’ findings.
counties. Option (d) cannot be true because the last sentence
mentions that only a small minority of anthropologists
108. b If it is assumed that burglaries and robberies in Sandy believe that their theories have been substantiated.
Beach have increased in the last three years, the Hence, there is an iota of doubt in the findings.
information can explain the apparent discrepancy in
the data provided in the statement. Options (a), (c)

Previous Years
Page 150 CLAT & AILET Papers
For questions 113 to 118: The initial colours of the chemicals 120. c Q is grandson of O.
is as follows
121. a O is the father of M.
Flask Colour
1 Red For questions 122 to 124: The position of cities is shown
below:
2 Blue
3 Green 20 30
4 Orange 18 12
G Z L H
The final results of various experiments are shown below: Y K
12
4 14
Colors in
Final
Experiment No Experiments rest of two
colour
flask 12
Green, 122. b The speed of the car = = 0.5 km / min.
1 Mixing 1 and 2 Red 24
Orange The time taken to reach G from K
Red,
2 Mixing 2 and 3 Orange 38
Orange = = 76 min
0.5
Blue,
3 Mixing 3 and 1 Blue, Hence, the required time is 10:16 a.m.
Orange
Blue,
4 Mixing 3 and 4 Blue 123. b If m is 1 km to the south-east of L, then it is exactly
Red
midway between Y and K.
Blue,
5 Mixing 4 and 1 Green
Green 124. a The distance from G to H is 26 km.
Red,
6 Mixing 4 and 2 Green 125. b Refer to the venn diagrams given below:
Green
(1)
113. d From the above table we can say that option (d) is
correct.
B ike s M op ed s S coo te rs
114. c If a student preforms two experiments then it can be
(i) Experiment number 1 and 4, (ii) Experiment number
2 and 5 and (iii) Experiment number 3 and 6. Among
given options (c) is correct. (2)

H a irs
115. b According to condition experiment number 3 or 4 are
possible. C h ild re n
Red
116. a 117. d 118. d

For questions 119 to 121: Family relation is shown below:


126. c Refer to the venn diagrams given below:
(1)
M arried cou ple
O R
H o use w ife Red
L aw yer
M oo n
M arried cou ple Sta rs
P M N
P ro fe ssor D o cto r
(2)
L Q
E n gine er O pe n
D o ors O utd oo rs
119. b* P is either brother or sister of M.

Previous Years
CLAT & AILET Papers Page 151
127. c Refer to the venn diagrams given below: 134. c Both the courses of action are valid. BCCI can certainly
(1) play a pivotal role in the development of new talent in
cricket. Furthermore, a rotation policy is a very good
R e cta ng ular idea to help the senior players stay in shape, remain
free from injuries and play for a longer time. This in
R e cta ng le s
turn, will avert a possible vacuum in the Indian cricket
E n ve lo pe s team.

135. c Both statements I and II are valid courses of action. If


(3) the government provides a compensation package
for the unemployed people, it will help them deal with
K n ow lack of income. This in turn, will lead to a decline in the
crime rate. Similarly, statement II is valid because a
W ell rea d greater economic activitythat combines with growth
L ea rne ds of industry will help in generating employment and
income which can again lead to a reduction in the
crime graph.

For questions 136 and 137: Start with Akshay’s statement:


128. b Refer to the venn diagram given below:
If his second statement is true then first must be true, which is
(2)
not possible. Hence, his second statement is false i.e he is not
suicidal.

E xecu tives S o ld ie rs P a trio tic If Akshay is not suicidal, then second statement of Himanshu
is true and first is false. So Himanshu want to die i.e. Himanshu
is suicidal. Now, the first statement of Anuj is true and second
statement is false, so Anuj is also suicidal.
129. d The given statement is like an ‘if..then’ clause. Options
136. d The correct answer is option (d) because Anuj and
(a) and (b) are negated because the statement does
Himanshu are suicidal. The official answer key is (a)
not talk about the grandfather reprimanding Vijay ‘in
which is wrong.
the morning’. If the effect does not happen, we can be
sure that the cause did not exist. So, option (c) can
137. No one is lying about other person tendencies. Which
also be ruled out. So, the correct answer is option (d).
is not given in the any option.
130. d The given statement means that Amir is in only one of
138. a According to Peter’s statement either Peter or Shina
the two situations at a time: angry and show mock
is professor. If Peter is professor, then both statement
anger. If Amir is angry, he is not showing mock anger
of Shina will be true, which is not possible. Therefore,
and if Amir is not angry, he is showing mock anger. So,
Shina is the professor.
the correct answer is option (d).
139. c Start with Minaxi’s statement: If the first statement is
131. d This statement is like an ‘if..then’ clause. If the cause
true, then the second becomes also true, which is not
happens, the effect will happen but if the effect
possible (because one is true and one is false). The
happens, we cannot be sure of the cause. Similarly,
first statement of Minaxi is false it means she is not
we cannot be sure that if the cause does not happen,
the singer. Now come to Jaya statement, her second
the effect will also not happen. So, options (a), (b)
statement is true so the first must be false, hence
and (c) are ruled out. . If the effect does not happen,
Madhuri is also not the singer. Therefore, Jaya is the
we can be sure that the cause did not exist. So, the
singer
correct answer is option (d).
140. a Start with Maroof statement: The first statement must
132. c Both statements I and II address the issue of road
be false, (because he is talking to strange and saying
safety and can certainly bring fatalities related to road
that he does not speak to the stranger), so his second
accidents. Hence, option (c) is the correct answer.
statement is true it means Maroof is new to these
part.. Now come to Ayesha’s statements: The second
133. b Statement I is incorrect because we cannot infer that
statement of Ayesha is false, so first must be true, it
government hospitals lack infrastructure. Hence,
means she is not the wife Nafish. Now come to Nafish
statement I can be rejected. However, statement II is a
statement, his second statement is false so the first
possible course of action and government intervention
must be true. Hence, the road to the left leads to
in the form of allotting more funds for the betterment of
“Where is Who”.
health services can be a way to address the issue.

Previous Years
Page 152 CLAT & AILET Papers
141. c Let the largest piece = 3x
4 5 5 4 5
Middle = x Hence, b = a, c = b = × a = a
Shortest = 3x – 23 3 4 4 3 3
∴ 3x + x + (3x – 23) = 40
⇒x=9 83a + 76b + 85c
∴ the shortest piece = 3(9) – 23 = 4 Average of X, Y and Z =
a+b+c

142. c 20 kg fresh grapes will contain (0.9 × 20) = 18 kg 4 5


water and 2 kg mass. If the dry grape has to contain 83a + 76 × a + 85 × a
= 3 3
2 kg mass, it should constitute 80% of that. Hence, if 4 5
80% of dry grapes corresponds to 2 kg, its total weight a + a + a
3 3
 2 
will be   = 2.5 kg. 978
 0.8  = = 81.5
12
143. a* If x men were there on day one, there would be
(x – 110) men on the 12th day. 147. b To maximise the value of the wealth, we must carry
Hence, on an average, there were (x – 55) men. more of the one whose value per kilogram is more.

3  4 
The job takes times the normal time. Value per kilogram of ruby =   = Rs. 13.33 crore,
2  0.3 
and value per rupee of each emerald
2
Hence, the average number of people = .  5 
3 =   = Rs. 12.5 crore.
 0.4 
2
⇒ x – 55 = x It is obvious that we should carry entire 12 kg of ruby.
3
Hence, x = 165  12 
This would amount to   = 40 rubies.
The official answer key is (d), which is wrong.  0.3 

144. a In the same time as A runs 200 m in the race, 148. b Out of the 5 girls, 3 girls can be invited in 5C3 ways.
S runs 180 m and N runs 160 m. Nothing is mentioned about the number of boys that he
In other words, in the same time as S runs 180 m, has to invite. He can invite one, two, three, four or
N runs 160 m. even no boys. Each boy can be invited or not. He can
So in the same time as S runs 100 m, N will run invite them in 24 ways. Thus, the total number of ways
is 5C3 × (2)4 = 10 × 16 = 160.
 160 
 100 × 180  = 88.89 m.
  149. c The answer is 10C2 × 11 + 11C2 x 10
Hence, in a 100 m race, S will beat N by (100 – 88.89) = 45 × 11 + 55 × 10 = 1045.
= 11.11 m.
150. d HCF of 60, 84 and 108 is 12. Hence, 12 students
145. a Let x be the fixed cost and y the variable cost. Then, should be seated in each room. So for subject A we
17500 = x + 25y …(i)
30000 = x + 50y …(ii)  60 
would require   = 5 rooms, for subject B we would
Solving the equation (i) and (ii), we get  12 
x = 5000, y = 500
Now if the average expense of 100 boarders be ‘A’.  84 
require   = 7 rooms and for subject C we would
Then, 100 × A = 5000 + 500 × 100  12 
⇒ A = Rs. 550.  108 
require   = 9 rooms. Hence, minimum number of
 12 
146. a Let the number of students in classes X, Y and Z be a,
b and c respectively. Then, rooms required to satisfy our condition = 5 + 7 + 9 = 21
total of X = 83a; total of Y = 76b; total of Z = 85c rooms.

83a + 76b
Also = 79 ⇒ 4a = 3b;
a+b

76b + 85c
and = 81 ⇒ 4c = 5b
b+c

Previous Years
CLAT & AILET Papers Page 153
AILET Solutions 2016
1. d Refer to the last sentence of the first paragraph. It 9. c Refer to the first three sentences of the passage. It
says that the passage seeks to understand and explain can be inferred that the changes in working and living
the types of changes that have accrued in the conditions of rural manual workers are of utmost
structure of rural labour markets over the last few importance because they form a bulk of rural workers.
decades. Hence, option (d) is the correct answer. Hence, option (c) is the correct answer.

2. c Option (a) can be inferred from the second sentence 10. d 'Run down' as used in (e) means to collide with and
of the first paragraph. It says that in 1991, 60% (3/5) knock down someone or something. So, (e) matches
workers in rural India were manual workers. Option with (c), 'hit with a car'. 'Run down' as used in (f)
(b) can be inferred from the second sentence of the means to hunt for and locate someone or something.
second paragraph-the decade was characterized by So, (f) matches with (a), 'trace'. 'Run for' as used in
a growing casualization of work force. Option (d) can (g) means to try to be elected to a particular office. So,
also be inferred from the passage. Refer to the fourth (g) matches with (d), 'campaign for a government
sentence of the last paragraph - "wages in non position'. 'Run down' as used in (h) means to criticize
agricultural work are typically higher than wages in or deride someone or something. So, (h) matches with
agriculture". Option (c) states the opposite of what (b), 'criticize'. Hence, option (d) is the correct answer.
the author says. Refer to the penultimate sentence of
the third paragraph. It clearly says that there is gender 11. c 'Wind-up' as used in (e) means something done or
bias among workers from agricultural sector. Hence, said in usually a joking way to make someone annoyed
option (c) is the correct answer. or upset. So, (e) matches with (d), 'an act or instance
of teasing'. 'Wind up' as used in (f) means to coil
3* None of the given options is correct. The passage
something. So, (f) matches with (b), 'to tighten the
does not say why the increasing gap is a concern.
spring of'. 'Wind-up' as used in (g) means to end up
4. d Refer to the last paragraph. It says that the increase in somewhere. So, (g) matches with (a), 'finish'. 'Wind-
real wages was co-related with the increase in share up' as used in (h) means to get excited. So, (h) matches
of non agricultural employment. From this we can say with (c), 'excited'. Hence, option (c) is the correct
that there was a mass outflow of agricultural laborers answer.
to non-agricultural sectors and the wealth generated
by the agricultural sector was now distributed among 12. a 'Run off' as used in (e) means to duplicate something,
the existing laborers of agricultural sector. Hence, using a mechanical duplicating machine. So, (e)
option (d) is the correct answer. matches with (b), 'reproduce'. 'Run off' as used in (f)
means to drain away from a flat area. So, (f) matches
5. b Options (a), (c) and (d) are not stated anywhere in with (d), 'to flow'. 'Run off' as used in (g) means to
the passage. Option (b) is directly stated in the first flee. So, (g) matches with (c), 'leave quickly'. 'Run off'
paragraph. Refer to the line "The structure and…of as used in (h) means to force someone to leave
the labour market." suddenly. So, (h) matches with (a), 'make leave'.
Hence, option (a) is the correct answer.
6. * None of the given options is correct. Options(a) and
(c) are incorrect because according to the passage, 13. c An incident is reported in a newspaper after it has
when agricultural employment increases, casualization happened. So, the blank will take a word /phrase that
of labour force also increases. Option (b) is incorrect suggests that the incident has happened in the past.
because the passage says that at the time So, the blank will take 'was taken'. Hence, option (c) is
casualization of labour force was increasing, poverty the correct answer.
was declining.
14. d The blank will take 'was allowed'. The sentence means
7. c The passage deals with structure of rural labour
that the woman was allowed to go home after the
market. So, it is an economic passage.
treatment. Hence, option (d) is the correct answer.
8. b Option (a) can be inferred from the second sentence
of the first paragraph. Option (c) can be inferred from 15. a The blank will take 'was blocked'. Hence, option (a) is
the last sentence of the third paragraph. It says that the correct answer.
unemployment was high among new entrants to the
workforce. Option (d) is stated in the first sentence of 16. b 'had to' suggests that the blank will take 'be diverted'.
the last paragraph. It says that real wages stagnated Hence, option (b) is the correct answer.
till 1970s and then then began to rise. Option (b) is not
stated by the author. Hence, option (b) is the correct 17. b 'could' here will be followed by 'have been'. Hence,
answer. option (b) is the correct answer.

Previous Years
Page 154 CLAT & AILET Papers
18. b 'Pantheism' refers to the belief that God is present in 'Blithe' means happy and without worry. 'Ecstatic'
all natural things. So, option (a) is negated. 'Deism' means very happy or excited. So, options (a), (c) and
refers to belief in God, especially a God that created (d) are synonyms. Hence, option (b) is the correct
the universe but does not take part in it. So, options (c) answer.
and (d) are also negated. 'Atheism' refers to the belief
that God does not exist. 'Agnosticism' refers to the 25. a 'Trappings' is an outward decoration or dress. 'Orifice'
belief that it is not possible to know whether God is an opening (as a vent, mouth, or hole) through which
exists or not. 'Secularism' is the belief that religion something may pass. 'Egress' refers to the act of
should not be involved in the organization of society, leaving a place. 'Vent' is an opening that allows air,
education, etc. Hence, option (b) is the correct answer. gas or liquid to pass out of or into a room, building,
container, etc. Options (b), (c) and (d) are related with
19. c 'Pompous', 'ridiculous' and 'photographic' are all leaving and are thus, synonymous. Hence, option (a)
adjectives. So, option (a) is incorrect. While 'penance' is the correct answer.
and 'science' are nouns, 'porous' is an adjective. So,
option (b) is also incorrect. Option (d) is incorrect 26. c 'Spectre' means a ghost. 'Apparition' is a ghost or an
because only 'stupidity' is a noun. 'Poisonous' is an image of a person who is dead. 'Hoodlum' is a violent
adjective and 'vocalise' is a verb. 'Analysis', 'praxis' criminal, especially one who is part of a gang. 'Phantom'
and 'thesis' are all nouns. Hence, the correct answer is a ghost. All save option (c) are related to ghost.
is option (c). Hence, option (c) is the correct answer.

20. d While 'ridiculous' and 'native' are adjectives, 27. c 'Should' is used to predict that something will probably
'psychologise' is a verb. This makes option (a) happen or is expected to be the case, based on logic
incorrect. 'Astronomy' and 'privacy' are nouns while or a typical situation.'Supposed' means claimed to be
'sympathy' is a verb. So, option (b) is incorrect. 'Nation', true or real.'Must' is used to express the view that
'action' and 'privacy' are nouns, rendering option (c) something is highly likely because it's a logical
incorrect. 'Chemical', 'mathematical' and 'French' are conclusion based on something else that the speaker
adjectives. Hence, option (d) is the correct answer. knows, or it's the normal thing to expect. 'Ought' is
used to predict that something is fairly likely or expected,
21. * 'Delegate' and 'defeat' are nouns and verbs, but not based on normal circumstances or logic. The most
adjectives. 'Temporary' is only an adjective. 'Tertiary' appropriate phrase to fill in the blank is 'must be',
is only an adjective. So, option (a) is incorrect. 'Chronic' rendering option (c) the correct answer.
is only an adjective. 'Incumbency', 'parent' and
'proponent' are nouns only. So, option (b) is incorrect. 28. b Option (a) is incorrect because it does not have a
In option (c), while 'topic', 'alacrity' and 'android' are verb. Option (c) is incorrect because 'that' is redundant
nouns only, 'auditory' is an adjective only. So, option here. Option (d) is incorrect because 'you did say' is
(c) too is incorrect. 'Square', 'precipitate' and 'collect' assertive, so there is no point of asking a question.
are nouns, adjectives and verbs. 'Free' is a verb and Only option (b) is grammatically correct and hence is
an 'adjective'; it is not a noun. The noun form of the correct.
word is 'freeness'.
29. b The word 'notified' will not be followed by an
22. a 'Wary' is used to describe a person who does not preposition. So, the blank will take 'the bank that',
have or show complete trust in someone or something rendering option (b) the correct answer.
that could be dangerous or cause trouble. 'Gullible' is
someone who can be easily fooled. 'Credulous' is 30. a This is a case of past subjunctive. In such sentences,
again someone who can be easily fooled. 'Naive' is the 'if' clause is in past simple tense while main clause
used to describe a person who is innocent or simple. takes 'would'. So, the first blank will take 'had' while
So, options (b), (c) and (d) are synonymous to each the second blank will take 'would drive'. Hence, option
other, rendering option (a) the correct answer. (a) is the correct answer.

23. c 'Gusto' means great enjoyment, energy, and 31. d 'By' means next to something. 'A beach next to Kochi'
enthusiasm. 'Verve' means great energy and would be incorrect. So, the first blank will take 'near'.
enthusiasm. 'Burst' means to break open or into pieces In the phrase 'among other things', 'among' means as
in a sudden and violent way. 'Zest' means lively well as. 'between other things' is incorrect. Hence,
excitement. Clearly, options (a), (b) and (d) are option (d) is the correct answer.
synonymous to each other. Hence, option (c) is the
correct answer. 32. a The correct phrase is 'hear from someone', which
means to get a letter, telephone call, or other message
24. b 'Beatific' means showing complete happiness. 'Diatribe' from someone. 'Agree on something' means to agree
refers to an angry and usually long speech or piece of to the choice of someone or something.Hence, option
writing that strongly criticizes someone or something. (a) is the correct answer.

Previous Years
CLAT & AILET Papers Page 155
33. a* 'Distrust' suggests that the blank will take 'suspicion'. 42. a R2 - D2 is an astromechdroid that extinguishes
Since people do not trust such displays of natural fires',repairs spaceships and assists the lead
fluency, they approach John Updike's fiction with characters in the Star Wars film.
caution. Hence, option (a) is the correct answer.
43. d The National Dialogue Quartet was formed in the
34. a* The sentence says that we can reduce the mixture's summer of 2013.The quartet was awarded the 2015
tendency to vapourise. 'Despite' suggests that the Nobel Peace Prize on 9th October 2015.The National
blank will take a word that suggests that in normal
Dialogue Quartet comprises the following organizations
circumstances the mixture does not change. 'Volatile'
in Tunisian civil society:
means likely to change in a very sudden or extreme
way. 'Resilient' means able to return to an original - The Tunisian General Labour Union (UGTT, Union
shapeafter being pulled, stretched, pressed, bent, etc. Générale Tunisienne du Travail)
'Resilient' is not used for liquids. 'Insipid' means not - The Tunisian Confederation of Industry, Trade and
interesting or challenging. 'Acerbic' means expressing Handicrafts (UTICA, Union Tunisienne de
harsh or sharp criticism in a clever way.Only option l'Industrie, du Commerce et de l'Artisanat)
(a) fits in the meaning of the sentence and hence, is - The Tunisian Human Rights League (LTDH, La
the answer. Ligue Tunisienne pour la Défense des Droits de
l'Homme)
35. b The blank will take 'hate it that'. Here 'it' refers to the - The Tunisian Order of Lawyers (Ordre National
other person's ability to swim. Hence, option (b) is the des Avocats de Tunisie)
correct answer.
44. b 'The Employees' Provident Fund Organisation has won
36. c The Nati folk dance of Kullu district of Himachal Pradesh the National Award on e-Governance 2015-16 for
in second week of January 2016 was listed in the
launching the Universal Account Number (UAN). The
Guinness Book of World Records as the largest folk
UAN facility enables workers to transfer their provident
dance in the world.
fund deposits while switching jobs anywhere in India
37. a E-commerce Company Amazon.com Inc has made its easily.
maiden entry in the Thomson Reuters Top 100 global
innovators list 2015 by dropping International Business 45. c Cochin International airport, the country's first airport
Machines Corp (IBM) from the list. The Thomson built under PPP model has scripted another chapter in
Reuters Top 100 global innovators program identifies aviation history by becoming the first airport in the
innovators annually through an in-depth analysis based world that completely operates on solar power.
on a series of patent-related metrics that analyze what
it means to be truly innovative. 46. c Prime Minister Narendra Modi unveiled a slew of
incentives to boost start-up businesses, offering them
38. b The International Monetary Fund (IMF) was set up a tax holiday and inspector raj-free regime for three
along with the World Bank after the Second World years, capital gains tax exemption and Rs 10,000 crore
War to assist in the reconstruction of war-ravaged corpus to fund them.
countries. Leaders felt that financial stability was best
achieved when countries worked in an environment
47. d Very recently, the Government in its move to curb the
of interdependence. The two organisations were
circulation of black money and widening its tax base
agreed to be set up at a conference in Bretton Woods
in the US. Hence, they are known as the Bretton Woods decided that quoting of PAN is essential for any form
twins. of transaction exceeding Rs.2 lakh, regardless of the
mode of payment. Apart from trigger value of Rs 2
39. c The Impossible trinity (also known as the Trilemma) is lakh, there are several other transactions where
a trilemma in international economics which states quoting of PAN is mandatory. Given below are the
that it is impossible to have all three of the following at details:
the same time: 1. During purchase/ sale of immovable property
- A stable foreign exchange rate exceeding Rs 10 lakh.
- Free capital movement (absence of capital 2. For payment of hotel or restaurant bills exceeding
controls) Rs 50,000 at any one point through any mode of
- An independent monetary policy payment.
3. While making a time deposit with any financial
40. d The measurement of poverty-|ine in lndia is based on institution like Co-op banks, Post Office, Nidhi and
the criteria of Calorie consumption.
NBFC.
41. c Prime Minister Narendra Modi described the landmark
48. b Paving the way for revolutionising cashless payments
climate change deal reached in Paris last night as the
victory of "climate justice" and said there were no services in the country, the Reserve Bank of India
winners or losers in the outcome. granted payment bank licences to 11 firms.

Previous Years
Page 156 CLAT & AILET Papers
49. a The government has approved 100% foreign direct 59. d FM broadcasting is radio broadcasting using frequency
investment (FDI) under the automatic route for non- modulation (FM) technology. eting radio broadcasting
bank entities that operate cash machines subject to technology, so it is used for most music broadcasts.
certain conditions in a bid to boost financial inclusion FM radio stations use the VHF frequencies. The term
in the country. "FM band" describes the frequency band in a given
country which is dedicated to FM broadcasting.
50. d In an historic step in the mission to inter-link rivers, the
Godavari was formally connected with the Krishna in 60. d Union Labour Secretary Shankar Aggarwal said the
Andhra Pradesh. EPFO investment into the stock market can go up to 15
per cent of the incremental deposit every year. As the
51. b Marathi film 'Court' was selected as India's official EPFO gets around Rs 1,00,000-1,20,000 crore as
entry to the Best Foreign Language category of the incremental deposits every year, ETF investment can
2016 Academy Awards. 'Court' was selected from a go up to Rs 15,000-20,000 crore. "It (investment)
shortlist of 30 films by a Film Federation of India jury depends upon the market. The Finance Ministry norms
headed by film-maker Amol Palekar. allow me to invest up to 15 per cent (of incremental
deposits) but CBT (Central Board of Trustees) allows
52. c A solar day is the time taken for the Sun to move from me to invest 5 per cent in ETFs to start with," Jalan
its highest point in the sky back to its highest point the said.
following day. This is exactly 24 hours. A sidereal day
is the time it takes for the Earth to actually rotate once 61. d Liquid crystal technology has had a major effect many
on its axis. A sidereal day is four minutes shorter than areas of science and engineering, as well as device
a solar day. The sidereal day is shorter than the solar technology. Applications for this special kind of material
day because as well as rotating on its axis, the Earth are still being discovered and continue to provide
also orbits the Sun. effective solutions to many different problems.

53. b United States is the signatory and only industria'sed 62. d Women are registering to vote for the first time in
country of the world that did not ratify the Kyoto Saudi Arabia in a long overdue move towards equality
Protocol. for the nation. Voting registration for the December
elections starts today and on August 30, they will
54. b The 2016 BRICS summit will be the eighth annual BRICS even be allowed to register as candidates.
summit, an international relations conference attended
by the heads of state or heads of government of the 63. a 'Brass Plate Subsidiaries' are usually set up for the
five member states Brazil, Russia, India, China and purpose of Income tax avoidance.
South Africa. The summit will be held in Panaji, Goa in
India, from 15th to 16th October 2016. 64. c McDonald's is the world's most popular and largest
fast food restaurant. It has more than 34,000
55. a In a major turnaround, the BJP-led government, so far restaurants worldwide serving nearly 69 million people
opposed to foreign investments in multi-brand retail, in 118 countries daily. Eighty percent of McDonald's
has decided to allow FPI up to 49 per cent or up to the restaurants are franchised.
sectoral cap (whichever is lower) through the
automatic route."This means that foreign investors in 65. c Tennis ace Sania Mirza was conferred the Rajiv Gandhi
multi-brand retail can bring in investments in the form Khel Ratna Award even as the Karnataka High Court
of FPI up to 49 per cent without government approval," sought clarifications from the Union Sports Ministry
a Department of Industrial Policy & Promotion official following a petition from Paralympian H.N. Girisha
clarified. challenging the Ministry's decision.

56. a The Indian carmaker Tata Motors has decided to change 66. d French President Francois Hollande pledged to
the name of its heavily promoted new car, the Zica, to redefine France's business model and declared what
avoid association with alarm about the spread of the he called "a state of economic and social emergency,"
Zika virus. unveiling a 2-billion-euro ($3.19 billion Cdn) plan to
revive hiring and catch up with a fast-moving world
57. d Bharti Airtel, about one-third owned by Singapore economy.
Telecom, steals a march over Mukesh Ambani's
Reliance Jio by becoming the first to offer fourth 67. a Section 80TTA is introduced with effect from April 01,
generation mobile services across India. 2013 and will apply from AY 2013-14 and onwards.
Section is introduced to provide deduction to an
58. c The World Trade Organisation (WTO) formally individual or a Hindu undivided family in respect of
approved Afghanistan's membership at its 10th interest received on deposits (not being time deposits)
ministerial conference in the Kenyan capital Nairobi. in a savings account held with banks, cooperative
banks and post office. The deduction is restricted to
Rs 10,000 or actual interest whichever is lower.

Previous Years
CLAT & AILET Papers Page 157
68. c Amid dramatic scenes and protests by students, TV 75. a The Principle states that the power conferred by a
actor and Bharatiya Janata Party (BJP) member statute cannot be withdrawn by a subordinate
Gajendra Chauhan took charge as chairman of the legislation. The Licensing power was granted by the
Film and Television Institute of India (FTII), seven months Cinematographic Act and hence these power cannot
after his appointment, even as police detained nearly be taken away by any rule framed under the same
40 protesters. act. Any change in the licensing power can be made
only by amending the cinematographic Act and not by
69. d The Union Cabinet chaired by the Prime Minister Shri any rules made under the parent act.
Narendra Modi, has given its ex-post facto approval
to provide an additional 50 days of unskilled manual 76. d According to Clause (1) of Article 15, discrimination
cannot be done on the basis of religion, race, caste,
work in the financial year over and above the 100
sex , place of birth or any of them. It is not talking about
days assured to job card holders, in such rural areas
resident as one of the criteria and hence the rules are
where drought or natural calamities have been notified.
not violative of Article 15(1) of the Constitution.
70. b Veteran actor Anupam Kher has been appointed as 77. b The company is liable as David allowed his brother to
the UN ambassador of the 'HeForShe' campaign for drive the car. David allowed him to use the office car
gender equality. The 'HeForShe' campaign has been for practice. The employer is liable for the act of
initiated by UN Women. Its goal is to engage men as employee for all the wrongs committed by his
agents of change for the achievement of gender employees in the course of employment.
equality and women's rights, by encouraging them to
take action against inequalities faced by women and 78. a The principle states that civil courts have coercive
girls. powers to compel attendance of witness only within
its local territory and Faridabad, doesn't comes within
71. c The Principle clearly states that a reasonable the local territory of Delhi High Court and hence Puchu
classification having nexus with the object sought to is not liable.
be achieved is not violative of Article 14 and Article 16
of the Constitution of India and the policy of the 79. d basing on the principle the most suitable answer is d.
government to appoint only lady principal in a women's The objective of the government was to keep sufficient
college is a reasonable classification having a nexus labour supply for agricultural purpose and this could
with the object sought to be achieved. Hence the have been easily achieved by restraining the
challenge of Mr. X is not sustainable. employment of agricultural labour in Bidi manufacturing
during agricultural season only. Government can
72. d To constitute State, the institution must be created under impose reasonable restrictions. Absolute restrictions
the Constitution or a statute, or is financed by the amount to violation of right and hence the law is
unconstitutional.
state or government holds the share capital in the
institution. BCCI is neither create under statue nor any
80. b Here the KLM institute has not unlawfully intervened
part of share of the Board is held by the government
in the chattel of 'X'. The Surgery was to remove the
and no financial assistant is given by the government
fibroids from the uterus and it was of no use to Mrs. X.
to the Board and hence it doesn't fulfill the criteria of KLM institute need not share its income with X because
being a State under Article 12 of the Constitution. the medical institute instead of destroying the waste
fibroids of X conducted research on its own and
73. a The Principle clearly states that a suit shall be instituted invented the new life saving drugs.
where the defendant actually and voluntarily resides
or carry on business, or personally works for gain or 81. d The statement is correct. The judgment related to a
in whose jurisdiction the cause of action arises and contractual obligations, binds only the parties between
hence 'Y' may institute the suit either at Delhi, where Z whom the contract was entered and it is judgment in
carries on business or at Mumbai where the cause of personam as it does not bind the person who are not
action arose. party to the contract. But if a party is declared insolvent,
it binds everyone not to enter into any contract with
74. b Principle says that the acceptance of an offer will be him and hence is right in rem.
valid only if it is made in the way it was expected to be
made. Here, according to the factual situation 'K' was 82. d Here the principle says that person cannot be subject
supposed to send a letter within two weeks confirming to penalty greater than that which might have been
that she wishes to buy the shop for the price finalized. inflicted under the law in force at the time of the
'K' although conveyed the acceptance over phone, commission of the offence and it does not talk about
but failed to make in the way she was supposed to reduction in the sentence and retrospective
and hence 'J' is not bound by the acceptance of 'K'. application of criminal law if it is beneficial to the
accused is not against Article 20(1) of the Constitution.

Previous Years
Page 158 CLAT & AILET Papers
83. b Both A and R are true but R is not correct explanation give any data to support the claim made in option (d).
of A. Hence, option (b) is the correct answer.

84. b Both A and R are true but R is not correct explanation 111. a The passage says that these days brand-name
of A products neither offer higher quality nor sell at higher
prices.Still, brand names provide a bigger marketing
85. b Both A and R are true but R is not correct explanation advantage. This is because brand names guarantee
of A to provide products that are as good as their rival's
products. Hence, option (a) is the correct answer.
86. b Both A and R are true but R is not correct explanation
of A 112. c The passage suggests that withholding funds from a
dance performance that is deemed obscene and
87. d A is false but R is true. censoring works of choreographers are the real
obscenity. Option (c) states the opposite of this, thereby
88. b Both A and R are true but R is not correct explanation weakening the argument. Hence, option (c) is the
of A correct answer.

89. d A is false but R is true. 113. b The passage says that courts only guard the
constitution; they check if the legislation or executive
90. c 91. d 92. d 93. a 94. a actions are in conformity with the provisions of the
constitution. They neither approve nor condemn any
95. b 96. d 97. a 98. d 99. d legislative policy. Option (b) says that judiciary makes
critical remarks about executives and condemns them.
100. c 101. b 102. a 103. c 104. d This clearly defeat the case presented. Hence, option
(b) is the correct answer.
105.c
114. d The passage says that top managers use intuition
106. a Refer to the line "To make sense of this choice…then more than middle level or lower level managers and
identify possible options." Hence, the inference is this this confirms that intuition is more effective than
definitely true, rendering option (a) the correct answer. careful, methodical reasoning. This conclusion can
only be drawn if we assume that top managers are
107. a The inference is definitely true. Look at the first more effective at decision making than middle level or
sentence. It says that investors 'today have more lower level managers. This is given in option (d),
investment options' and then later in the paragraph rendering it the correct answer.
says that investors need to evaluate risk. This means
that investors today need to use their judgment more 115. c The passage presents a conflict over tax savings. A
critically. Hence, option (a) is the correct answer. banker says that by transferring money to retirement
account, people can save money as it will delay
108. d The inference is probably false. Refer to the second payment of tax. On the other hand an accountant
and the third sentences of the passage. They say that says that this is not actually saving money as one will
choice is good, but they lead to clutter and noise if the have to pay tax when one withdraws money from
products are similar. Hence, option (d) is the correct account. Clearly, the disagreement is over the
answer. application of the term 'save'. Hence, option (c) is the
correct answer.
109. c Nothing has been said about the investors in the past.
Hence, option (c) is the correct answer. Solutions for questions 116 to 120: The information is
summarised below:
110. b Option (a) is incorrect because the passage says that
'more than 25%' women do not have health care
Person City Mode
insurance. This means that less than 75% women
have health care insurance. Option (b) is correct. Refer P Hyderabad Bus
to the second sentence. It says that about 40% women
Q Calcutta Aeroplane
in our country bear children between the age of 18
and 24. This means that about 60% women in our R B anglore Car
country bear children either before attaining 18 years
S Chennai Boat
of age or after attaining 24 years of age. Option (c) is
incorrect because the passage is silent on the share T Delhi Train
paid by insurance companies to cover maternity cost.
Option (d) is incorrect because the passage does not 116. d 117. c 118. a 119. b 120. c

Previous Years
CLAT & AILET Papers Page 159
Solutions for questions 121 to 125: 124. b If R is between O and P, then Q being 1 km to the west
The vertical North South streets are N, O, P, Q, R. From the of R, would be more than 1 km to the west of P. Option
basic information we have two relative positions are available (b) is false.
– one between O, P and N and the other between Q and R.
125. d Y is 1.5 km north of X is correct in all cases as the
O P N Q R figure between X, W and Y shows. Option (d) is
correct.

Solutions for questions 126 to 129: The family relation is


presented below.
1 /2 km 1 /2 km 1 km H o usew ife C o ntra ctor
K M
D o cto r N u rse
I H

L J
Stud en t Stud en t
Figu re fo r O , P a nd N Figu re fo r Q a nd R
126. d H is nurse.
The horizontal eastwest streets are: S, T, U, V, W, X, Y.
Of these seven streets the relative positioning is given in 3
127. a I is the husband of H.
distinct part as shown here:

T 128. b M, K and I, H are married couple.


U
1 /2 km 1 km 129. d As gender of JJ is not known, so the data is
S insufficient.
V
130. b Number of odd days from 1st December to 10 January
Y next year is 40 i.e. 5 days.
Hence, 1st December will eb 5 days earlier that
1 km
Saturday i.e. Monday.
W
1 /2 km
X 11
131. c The required angle = | 30h − M|
2
121. a The solution is visible fromthe figure below. Option (a)
is correct. 11
= | 30 × 8 − × 30 | = 75°.
2
U
1 /2 km 132. a Pattern of the sequence is as follows:
Y
1 /2 km 133. b Pattern of the sequence is as follows:
V 113 + 17 = 130
1 /2 km 130 + 2 × 7 = 164
W 164 + 3 × 13 = 215
1 /2 km 2.5 + 4 × 17 = 283
X 283 + 5 × 17 = 368.
1 /2 km
T 134. c Patter of the sequene is as follows:
+3 +3 +3 +3
1 /2 km A 
→ D 
→ G 
→ J 
→M
S
−2 −2 −2 −2
Y 
→ W 
→ U 
→ S 
→Q
122. d If R is 1/2 km east of O, then R and P would coincide.
+3 +3 +3 +3
Option (d) makes two streets coincide. B 
→ E 
→ H 
→ K 
→N
−2 −2 −2 −2
123. d If R is placed between O and P, Q to N would be a Z 
→ X 
→ V 
→ T 
→R
minimum of 1.5 and a maximum of 2 kms. Option (d) +3 +3 +3 +3
correct. C 
→ F 
→ I 
→ L 
→O

Previous Years
Page 160 CLAT & AILET Papers
135. d Pattern of the sequence is as follows: 142. c Let the amount of work be 300 units.
−2 −2 −2 −2 −2 Work done by 25 worker in 4 days = 100 units
Z  → X  → V 
→ T 
→ R 
→P Remaining work = 200 units
Number 200
(1 + 1) × 2 = 2 The required number of days = 4 + = 14.
20
(2 + 1) × 2 = 6
(6 + 1) × 3 = 21 143. c The distance between 1st and 10th tree is a row = 2
(21 + 1) × 4 = 88 × 9 = 18 m
(88 + 1) × 5 = 445 ∴ Length = (18 + 1 + 1) = 20 m.
and
+3 +3 +3 +3 +3 144. b Let the required distance be ‘D’.
A 
→ D 
→ G 
→ J 
→ M 
→P
D D
+ =4
136. d The passage says that X's sister asked him to wear 14 − 10 14 + 10
her husband's scarf and return to hishotel through an
⇒ D = 13.71 km
alternate route. A possible explanation for her doing
this could be that she wanted the people, who had a
hand in the murder of her husband, to think that her 3
 10 
husband is still alive. Now if the criminals see him 145. C CI = 2000  1 +  − 2000 = 662
 100 
alive, they would try to kill the brother too, which the
sister wouldn't want. This weakens the statement 2000 × 3 × 10
that she arranged for her husband's death. Hence, SI = = 600
100
option (d) is the correct answer.
The required difference = 62.
137. b The statement is a possible explanation of the result. It
is possible that X's sister was in the secret police and D D 1
146. a − = ⇒ D = 30 km.
she wanted the people who killed her husband to 30 40 4
think that the spy is still alive so that she catches them
red handed when they try to kill him. So by making him 147. d Let the number of Rs. 1 coin be n.
wear the scarf and giving him a promotion, she Number of 50 paisa coin be = 4n
probably wanted to make the murders believe that the Number of 25 paisa coin be = 2n
spy is still alive. Hence, option (b) is the correct answer. Now
4n 2n
138. a The passage says that Y informs X that his brother- n+ + = 56 ⇒ n = 16
in-law's plane crash was an act of sabotage. No other 2 4
reference of Y has been made in the passage. So, Y ∴ Number of 50 paisa coin = 4 × 16 = 64.
recommending X for a promotion is inconsistent with
the fact situation and result. Hence, option (a) is the 148. a 7B = 3K and 2K = B + 5C
correct answer. 2×7
⇒ B = B + 5C
3
139. c Fact situation says that despite X's training in
engineering and electronics, he was reassigned to 14
military supply and the result says that he was B − B = 5C
3
promoted and made commander of missile division.
Since missile division is related to the field in which he 11
B = 5C
received his training, we can say that to X, a missile 3
base is a more highly prized assignment than a supply ⇒ 22B = 30C.
depot. Hence, option (c) is the correct answer.
149. c Percentage of students who likes both
140. b X had high performance ratings from his commanding = (72 + 44) – 100 = 16
officers. So, his failure to win a promotion could be a If the number of students be x, then
result of a clerical error. Hence, option (b) is the correct 16% of x = 48 ⇒ x = 300.
answer.
150. d Price of the xema two pens = 12 × 10 – 8 × 11.75 = 26
141. b Possible outcomes = (1, 4), (4, 5), (2, 3), (3, 2)
Now x + 1.6x = 26 ⇒ x = 10
4 1 ∴ Price of the pens are 10 and 16
The required probability = = .
36 9 Note: Among the given ?? only option (d) have value
which satisfy the condition.

Previous Years
CLAT & AILET Papers Page 161

You might also like